Sunteți pe pagina 1din 918

Principii de energie din surse regenerabile

CONȚINUTUL
Obiective de studiu
1
§1.1 Introducere
3
§1.2 Energie și durabilă
dezvoltarea
4
§1.2.1 Principii și probleme majore
4
§1.2.2 securitatea Energetică
7
§1.2.3 Un simplu model numeric pentru
durabilitatea
7
§1.2.4 resurse Globale
8
§1.3 Fundamentele
9
§1.3.1 surse de Energie
9
§1.3.2 Mediu de energie
11
§1.3.3 Primare de alimentare la final
12
§1.3.4 planificarea Energetică
12
§1.4 principii Științifice din surse regenerabile
energie
14
§1.4.1 caracteristici Dinamice
15
§1.4.2 Calitatea ofertei
16
§1.4.3 Dispersate față de centralizat
energie
17
§1.4.4 Complexe (interdisciplinare)
sisteme de
17
§1.4.5 Situație de dependență
18
§1.5 implicațiilor Tehnice
18
§1.5.1 Prospectarea
mediu
18
§1.5.2 utilizare finală cerințe și
eficiența
19
§1.5.3 corelarea cererii cu oferta:
de energie sisteme de control și
mecanisme
19
§1.5.4 Eficiență, capacitate de factori
și potențial de resurse
regenerabile de energie dispozitive
22
§1.6 Standardele și reglementările
27
§1.7 implicații Sociale
27
§1.7.1 Dispersate de viață
28
§1.7.2 Poluare și de mediu
impactul
29
§1.7.3 viitor
30
Rezumat capitol
30
Întrebări rapide
31
Probleme
31
Note
33
Bibliografie
33
CAPITOLUL

1
OBIECTIVE DE STUDIU

Definirea surselor regenerabile de energie (RE).

Apreciez științifice, tehnice, sociale și
implicațiile diferența între
regenerabile și neregenerabile resursele de energie.

Ia în considerare durabilitatea și aprovizionarea cu energie.

Știu parametrii cheie care afectează individuale
RE provizii.

Apreciez variabilitatea diferite RE
provizii.
TWIDELL PAGINARE.indb 1
01/12/2014 11:35

www.shahrsazionline.com
2
Principii de energie din surse regenerabile

Ia în considerare metode și controale pentru a optimiza
utilizarea energiei din surse regenerabile.

Se referă aprovizionării cu energie a mediului
impact.
LISTA DE FIGURI
1.1
Contrastul dintre regenerabile (verde) și finite (maro) livrările de energie.
9
1.2
Natural de curenții de energie de pe Pământ, arătând sistem de energie regenerabila.
11
1.3
Energie diagrame de flux pentru Austria in 2010.
13
1.4
Potrivire sursă de energie regenerabilă la final.
20
LISTA DE TABELE
1.1
Comparație din surse regenerabile și convenționale de energie sisteme.
10
1.2
Intensitatea și frecvența proprietăți din surse regenerabile.
15
1.3
Factorii care influențează capacitatea de factori.
26
TWIDELL PAGINARE.indb 2
01/12/2014 11:35
§1.1 Introducere
3
§1.1 INTRODUCERE
Această carte analizează întreaga gamă de furnizare de energie regenerabilă
disponibile pentru economiile moderne din întreaga lume. Este unanim recunoscut faptul că
acestea sunt necesare pentru durabilitate, securitate, și standard de viață.
În sisteme de energie regenerabilă acoperite includ energie solară
radiația (soare), eoliană, biomasă (plante), râuri (hidroenergie), ocean,
valuri, maree, căldură geotermală, precum și alte astfel continuă de resurse. Toate
aceste sisteme sunt incluse în următoarele definiție generală:
Energia regenerabilă este energia obținută de la natural repetitive și
persistente fluxurilor de energie care au loc în mediul local.
Un exemplu evident este solar (soare) energie care persistă " și "repetă'
zi după zi, dar evident nu este constant ci variabil. În mod similar, plantele
au un raport anual de vegetație, care stochează energie de la soare, în
structura lor, care este eliberat în ardere și metabolismul. Cu o
resursa de energie regenerabila, energia este deja trece prin
mediul ca un curent sau flux, indiferent de acolo fiind un dispozitiv pentru a
intercepta și de a exploata această putere. Fraza 'local' se referă
la locația de un astfel de dispozitiv să intercepteze fluxul. Naturale de energie
fluxurile care sunt frecvent valorificate în scopuri energetice sunt indicate în
§1.3. Astfel de energie poate fi, de asemenea, menționată ca energia verde sau sustenabilă
de energie.
În contrast,
Non-regenerabile de energie este energia obținută din statice magazine
de energie care rămâne în subteran, cu excepția cazului eliberat de om
interacțiune.
Exemple sunt combustibili nucleari și fosili combustibili de cărbune, petrol și naturale
de gaze. Cu aceste surse, energia este inițial un izolat de energie
potențiale, și de acțiune externă este necesară pentru a iniția furnizarea de energie pentru
scopuri practice. Pentru a evita utilizarea greoi cuvântul 'non-regenerabile,
cum ar livrările de energie sunt numite finite, consumabile sau maro energie.
Este de asemenea posibil să se includă energie din societatea deșeuri în
definiția energiei din surse regenerabile, deoarece, în practică, ele sunt de neoprit, dar sunt
ei "natural"? Astfel punctele de discuție privind resursele sunt
implicite în detaliu de capitole mai târziu.
Pentru energia din surse regenerabile la scară de aplicare în practică variază de la
zeci la mai multe milioane de wați, și totalitatea este o resursă globală.
Cu toate acestea, pentru fiecare cerere, cinci întrebări ar trebui să fie întrebat:
1
Cât de mult de energie este disponibil în mediul imediat; ce
sunt resursele?
2
Ce tehnologii pot valorifica aceste resurse?
3
Cum poate fi energia utilizată eficient; ceea ce este utilizarea?
TWIDELL PAGINARE.indb 3
01/12/2014 11:35
4
Principii de energie din surse regenerabile
4
Care este impactul de mediu al tehnologiei, inclusiv
implicațiile pentru schimbările climatice?
5
Ce este cost-eficiența aprovizionării cu energie în comparație cu
alte materiale?
Primele trei sunt întrebări tehnice luate în considerare în centrale
capitole ale acestei cărți, în funcție de tipul de tehnologia energiei regenerabile. Cea de-a patra
întrebare se referă la problemele generale de planificare, responsabilitate socială, durabilă
de dezvoltare și impact global; acestea sunt luate în considerare la încheierea
secțiunea de fiecare tehnologie capitol și în Capitolul 17. A cincea și ultima
întrebare este o dominantă întrebare pentru consumatori, dar este mult influențată
de către guvern și alte politici, considerate ca factori instituționali în
Capitolul 17. Evaluarea "eficiența costurilor" depinde în mod semnificativ
la următorii factori:
o
Apreciind distinctiv principii științifice de energie din surse regenerabile
(§1.4).
b
la eficiența de fiecare etapă a aprovizionării cu energie în ceea ce privește
minimizarea pierderilor și maximizarea beneficiilor economice și sociale (articolul 16.2).
c
Având în vedere externalitățile și costurile sociale (Cutie 17.2).
d
Având în vedere atât costurile cât și beneficiile pe durata de viață a unui proiect
(care poate fi
> ~30 de ani).
În această carte vom analiza (a) și (b) în detaliu, deoarece acestea se aplică în mod universal.
Cel de-al doilea două, (c) și (d) au aspecte care depind de anumite
economii, și astfel ne-am explica doar principiile implicate.
§1.2 ENERGIE ȘI DEZVOLTARE DURABILĂ
§1.2.1 Principii și probleme majore
Dezvoltarea durabilă poate fi definită în sens larg ca viață, producătoare
și consumatoare într-o manieră care satisface nevoile prezentului fără
a compromite capacitatea generațiilor viitoare de a-și satisface propriile nevoi.
Ea a devenit una dintre principalele principii directoare pentru politica în secolul al 21
- lea. Principiul este afirmat în întreaga lume de către politicieni, industriași,
ecologi, economisti, și teologi ca ei caută
internaționale, naționale, locale și de cooperare. Cu toate acestea, ajungând specifice fost de acord
politici și acțiuni se dovedește a fi mult mai greu!
În context internațional, cuvântul "dezvoltare" se referă la
îmbunătățirea calității vieții, inclusiv îmbunătățirea standardelor de viață în mai puțin
dezvoltate țări. Scopul durabilă de dezvoltare este de a realiza
acest obiectiv, în timp ce protejarea proceselor ecologice pe care viața
depinde. La nivel local, progresivă companii caută un rezultat pozitiv triple bottom
line (de exemplu, o contribuție pozitivă la cele economice, sociale și
de mediu, bunăstarea comunității în care își desfășoară activitatea).
TWIDELL PAGINARE.indb 4
01/12/2014 11:35
§1.2 Energie și dezvoltare durabilă
5
Conceptul de dezvoltare durabilă a ajuns mai întâi global
importanță în seminale raport al ONU Comisiei Mondiale pentru Mediu
și Dezvoltare (1987); de atunci, această temă a intrat încet
și haotic în majoritatea economiilor naționale. Este nevoie să recunoască
amploarea și denivelările de dezvoltare economică și a populației
de creștere, care pune presiuni fără precedent pe planeta noastră de terenuri,
ape, și alte resurse naturale. Unele dintre aceste presiuni sunt severe
suficient pentru a amenința însăși supraviețuirea unor populații regionale și
pe termen lung pentru a duce la perturbatoare schimbare la nivel global. Modul în care oamenii
trăiesc, mai ales în ceea ce privește producția și consumul, va trebui să se adapteze
din cauza ecologice și presiuni economice. Cu toate acestea, economice
și sociale durere de astfel de modificări pot fi facilitat de previziune, de planificare, și
politice și a comunității va.
Resursele de energie exemplifica aceste probleme. Aprovizionare fiabilă cu energie este
esențială în toate economiile pentru iluminat, încălzire, comunicații,
computere, echipamente industriale, de transport, etc. Achizițiile de energie cont pentru
5 la 10% din produsul național brut în economiile dezvoltate. Cu toate acestea, în
unele țări în curs de dezvoltare, de importurile de combustibili fosili (de exemplu, cărbune, petrol, și gaze)
poate
costa peste jumătate din valoarea totală a exporturilor; astfel de economii sunt
nesustenabile, și o provocare economică pentru dezvoltarea durabilă. Lumea
consumul de energie a crescut de mai mult de zece ori în timpul secolului 20,
predominant din combustibili fosili și cu adaos de energie electrică din
energie nucleară. În secolul 21, crește și mai mult în lumea de energie
consum poate fi de așteptat, în mare parte datorită creșterii industrializare și
anterior cererii în țările mai puțin dezvoltate, agravată de brut
ineficiențe în toate țările. Indiferent de sursa de energie, există o
nevoie stringentă de eficient de transformare, distribuție și utilizare a energiei.
Combustibilii fosili nu sunt nou format la orice rată semnificativă, și
, astfel, stocurile actuale sunt în cele din urmă finit. Locația și cantitatea de astfel de
stocuri depinde de cele mai recente sondaje. În mod clar dominant de combustibili fosili de
masă este de cărbune. Rezerva de viață a unei resurse poate fi definită ca fiind
cunoscut accesibile suma împărțită la rata de prezenta de utilizare. Prin această
definiție, durata de viață a resurselor de petrol și gaze este, de obicei, doar câteva decenii,
întrucât durata de viață a cărbunelui este de câteva secole. Economie prezice că
ca durata de viata a unui combustibil de rezervă scurtează, astfel încât prețul de combustibil crește;
ulterior, prin urmare, cererea scade și, anterior, mai scumpe
și surse alternative intra pe piață. Acest proces tinde să facă
sursa originală dura mai mult decât o imediat calcul indică.
În practică, mulți alți factori sunt implicați, în special guvernul
de politică și relații internaționale. Cu toate acestea, de bază geologice faptul
rămâne: rezervele de combustibili fosili sunt limitate și, astfel, modelele actuale de
consum de energie și de creștere nu sunt sustenabile pe termen lung.
În plus, emisiile de la utilizarea combustibililor fosili (și într-adevăr nucleare
de putere) din ce în ce determina un alt fundamentale de limitare în
utilizarea lor a continuat. Aceste emisii aduce substanțe derivate din
TWIDELL PAGINARE.indb 5
01/12/2014 11:35
6
Principii de energie din surse regenerabile
subteran materiale (de exemplu, dioxid de carbon) în atmosferă
și oceane, care nu au fost prezente înainte. În special, emisiile de
dioxid de carbon (CO
2
) din arderea combustibililor fosili în mod semnificativ
a crescut concentrația de CO
2

în atmosfera globală. Autoritate
opinie științifică este de acord că, dacă această situație continuă, cu efect de seră
efectul va fi îmbunătățită și astfel duce la importante schimbări climatice în termen de
un secol sau mai devreme, care ar putea avea un important impact negativ asupra alimentar
de producție, de aprovizionare cu apă și societate (de exemplu, prin creșterea inundații și
furtuni (IPCC 2007, 2013/2014)); a se vedea, de asemenea, §2.9. Din păcate, beton acțiunea
este lent, nu în ultimul rând din cauza reticența guvernelor din
țările industrializate să perturbe stil de viață de alegătorii lor. Cu toate acestea, potențialul de
schimbările climatice, și legate de probleme de sustenabilitate, este acum stabilit ca
unul dintre principalii factori de politică energetică.
În contrast cu combustibili nucleari și fosili, regenerabile de energie (RE) alimentare în
funcționare nu se adaugă la elementele din atmosferă și hidrosferă.
În special, nu există nici o intrare suplimentară de gaze cu efect de seră (Ges).
Deși există în mod normal o astfel de emisiile rezultate din producția de toate
tipurile de echipamente de energie, acestea sunt întotdeauna mult mai puțin per unitate
de energie generată decât emise de-a lungul vieții de combustibili fosili de plante
(a se vedea date în Anexa D). Prin urmare, atât energia nucleară și sursele regenerabile de energie
reduce semnificativ emisiile de GES dacă înlocuirea combustibililor fosili. Mai mult decât atât,
deoarece RE consumabile sunt obtinute de derulare a fluxurilor de energie din
mediul natural, toate sursele de energie regenerabilă ar trebui să fie
durabilă. Cu toate acestea, mare grijă este necesar să se ia în considerare situații reale, așa cum
sa menționat în următorul citat:
Pentru o sursă de energie regenerabilă să fie durabilă, trebuie să fie
inepuizabilă și nu deteriora livrare a mediului
bunuri și servicii, inclusiv a sistemului climatic. De exemplu,
pentru a fi durabilă, producția de biocombustibili nu ar trebui să crească net
CO
2

emisii, ar trebui să nu afecteze în mod negativ securitatea alimentară, și nici nu


necesită utilizarea excesivă de apă și substanțe chimice, și nici nu amenință
biodiversitatea. Pentru a fi durabilă, energia trebuie să fie, de asemenea, din punct de vedere economic
la prețuri accesibile pe termen lung; trebuie să răspundă nevoilor societății și
să fie compatibile cu normele sociale, acum și în viitor. Într-adevăr,
ca utilizarea de RE tehnologii accelerează, un echilibru va trebui să fie
lovit între mai multe dimensiuni ale dezvoltării durabile.
Este important să se evalueze întregul ciclu de viață al fiecărei surse de energie
pentru a se asigura că toate dimensiunile sustenabilității sunt îndeplinite.
(IPCC 2011, §1.1.5)
În analiza rău și de a beneficia, integral costurile externe ale obținerea de
materiale și combustibili, și de plata pentru daune de emisii, ar trebui să fie
internalizate în costuri, cum s-a discutat în Capitolul 17. A face astfel încât să ia în considerare:
(i) caracterul finit al fosili și de combustibil nuclear, de materiale; (ii) efectul nociv al
emisiilor; și (iii) durabilitatea ecologică. Astfel de analize fundamentale
TWIDELL PAGINARE.indb 6
01/12/2014 11:35
§1.2 Energie și dezvoltare durabilă
7
de obicei concluziona că combinarea energiei regenerabile cu eficientă
utilizare a energiei este mai cost-eficient decât utilizarea tradițională a fosilă
și nucleară combustibili, care sunt nesustenabile pe termen lung. Pe scurt,
furnizare de energie regenerabilă sunt mult mai compatibile cu sustenabile
de dezvoltare decât sunt combustibili nucleari și fosili în ceea ce privește atât resurse
limitări și impactul asupra mediului (a se vedea Tabelul 1.1).
În consecință, aproape toate planurile energetice naționale include patru factori vitali
pentru îmbunătățirea sau menținerea beneficia de energie:
1
a crescut valorificarea surselor regenerabile de consumabile;
2
creșterea eficienței de aprovizionare și de utilizare finală;
3
reducerea poluării;
4
luarea în considerare a ocupării forței de muncă, de securitate, și stilul de viață.
§1.2.2 securitatea Energetică
Națiuni, și, într-adevăr, persoane fizice, trebuie asigura aprovizionarea cu energie; ei au nevoie
să știe că, suficiente și adecvate de energie va ajunge la ei în
viitor. Fiind în controlul independent și-a asigurat provizii, prin urmare, este
important – sursele regenerabile de energie oferă acest lucru atât timp cât tehnologii funcție
și sunt accesibile.
§1.2.3 Un simplu model numeric pentru durabilitate
Luați în considerare următorul model simplu care descrie nevoie pentru comerciale
și non-comerciale, resurse energetice:
R
= E N (1.1)
Aici R este total consumul de energie anual pentru o populație de N oameni.
E este utilizarea pe cap de locuitor de energie în medie de peste un an, strâns legate
de furnizarea de produse alimentare și nealimentare. La scară mondială, cu
dominante de aprovizionare de energie din surse comerciale, mai ales fosili
combustibili; cu toate acestea, semnificativ utilizarea de non-comerciale de energie pot să apară (de exemplu,
combustibil-lemn, pasive de încălzire solară), care este de multe ori absent din cele mai
oficiale și statisticile companiei. În ceea ce privește totalul comerciale consumul de energie, E
de pe o lume pe cap de locuitor, nivelul este de aproximativ 2,1 kW, dar regională valorile medii
variază pe scară largă, cu America de Nord 9.3 kW, Europa 4.6 kW, și mai multe
regiuni din Africa Centrală 0,2 kW. Includerea non-comerciale de energie
crește toate aceste cifre, mai ales în țările cu valori reduse ale E.
Nivelul de trai se referă într-un complex și un prost definite drum spre E.
Astfel, pe cap de locuitor produsul intern brut S (o măsură brut de standard
de viață) pot fi legate la E de:
S
= f E
(1.2)
Aici f este un complex și neliniare coeficient care este ea însăși o funcție de
mai mulți factori. Acesta poate fi considerat o eficiență de transformare a energiei
TWIDELL PAGINARE.indb 7
01/12/2014 11:35
8
Principii de energie din surse regenerabile
în bogăție și, prin tradițională economie, este de așteptat să fie la fel de mare ca
posibil. Cu toate acestea, S nu crește uniform ca E crește. Într-adevăr,
S - ar putea chiar să scadă de mare E (de exemplu, din cauza poluării sau tehnice
ineficiență). Evident, inutilă risipă de energie conduce la valori mai mici
de f decât ar fi altfel posibil. Substituind E în (1.1), la
nivel național cerința pentru energia devine:
R
= (S N) / f (1.3)

astfel încât

DR/R = DS / S + DN / N - Df / f (1.4)

Acum, ia în considerare înlocuirea globală valori pentru parametrii din (1.4).


În 50 de ani, populația lumii a N a crescut de la 2,5 miliarde de dolari în 1950 la
peste 7,2 miliarde de euro în 2013. Acum este în creștere la aproximativ 2 până la 3% pe
an, astfel încât să se dubla la fiecare 20-30 de ani. Tragic mare de mortalitate infantilă
și speranța de viață scăzută au tendința de a ascunde intrinsecă presiuni a populației
de creștere în multe țări. Convenționale economiștii caută exponențială
de creștere de la S la 2 la 5% pe an. Astfel, în (1.4), la o valoare constantă a eficienței
parametrul f, creșterea totală a lumii de alimentare cu energie este efectiv
suma de populație și de creștere economică (de exemplu, de la 4 la 8% pe an). Fără
consumabile noi, o astfel de creștere nu poate fi menținută. Totuși, în același timp,
ca mai este nevoie de energie, combustibili nucleari și fosili sunt epuizate,
și debilitante poluării și a schimbărilor climatice crește.
O modalitate evidentă de a depăși aceste constrângeri este de a crește
furnizare de energie regenerabilă. Cel mai important, din (1.3) și (1.4), este vitală pentru
creșterea eficienței parametru f (adică să aibă o valoare pozitivă de
Df).
Prin urmare, dacă nu există o rată de creștere în utilizarea eficientă și generarea
de energie, atunci S (nivelul de trai) crește în timp ce R (utilizarea resurselor)
scade.
§1.2.4 resurse Globale
Cu cele mai eficiente energetic moderne echipamente, clădiri, și
de transport, justificate obiectiv pentru utilizarea energiei într-o societate modernă este E
=2
kW per persoană (adică aproximativ global actual de utilizare medie, dar
cu o mult mai mare standard de viață). Este posibil acest lucru, chiar și în principiu, din
surse regenerabile de energie? Fiecare metru pătrat de Pământ suprafața locuibilă este
traversat de sau accesibile la o medie de energie flux de aproximativ 500 W (a se vedea
Problema 1.1). Aceasta include energia solară, eoliană, sau alte forme de energie din surse regenerabile
într-o estimare globală. Dacă acest flux este valorificat la doar 4% eficiență, 2 kW
de putere pot fi trase dintr-o zonă de 10m
× 10m, presupunând potrivite
metode. Zonele suburbane rezidențiale orașe au densități ale populației
de aproximativ 500 de persoane km
-2
. La 2 kW pe persoană, cererea totală de energie
de l000 kW/km
2
ar putea fi obținute în acest fel, prin utilizarea doar 5% din
locale suprafață de teren pentru producția de energie, permițând astfel tehnice
TWIDELL PAGINARE.indb 8
01/12/2014 11:35
§1.3 Fundamentele
9
potențialul de RE fiind mai mică decât potențialul teoretic', așa cum este indicat
în Fig.1.2 și §1.5.4. Astfel, furnizare de energie regenerabilă ar putea, în principiu,
să ofere un nivel de trai satisfăcător la nivel mondial, dar numai în cazul în metode
exista pentru a extrage, de a folosi, și de a stoca energie în mod satisfăcător la costuri realiste.
Această carte va lua în considerare atât aspectele tehnice de o mare varietate
de metode posibile și o sinteză de factori instituționali implicați.
§1.3 FUNDAMENTELE
§1.3.1 surse de Energie
Definiții din surse regenerabile de energie și de combustibili fosili și energie nucleară
dat la începutul acestui capitol sunt prezentați în Fig. 1.1. Tabelul 1.1
prezintă o comparație din surse regenerabile și convenționale de energie sisteme.
Există cinci final primar surse de energie utilă:
1
Soarele.
2
Mișcarea și potențială gravitațională dintre Soare, Lună și Pământ.
3
Energia geotermală de răcire, reacții chimice, fizice și
dezintegrare radioactivă.
4
Reacții nucleare de pe Pământ.
5
Reacțiile chimice din surse minerale.
Regenerabile de energie provine continuu din sursele 1, 2, și 3. Rețineți
că biomasa și ocean de căldură sunt în cele din urmă derivă din energia solară, ca
Mediul Natural:
verde
Minat de resurse: maro
Sursa de curent continuu
fluxul de energie
O
C
D
E
F
B
Dispozitiv
Utilizarea
D
E
F
Dispozitiv
Utilizarea
Mediu
Chiuveta
Mediu
Chiuveta
Sursă finită de
energie potențială
REGENERABILE DE ENERGIE
ENERGIE FINITĂ
Fig. 1.1
Contrastul dintre regenerabile (verde) și finite (maro) livrările de energie. Mediu
fluxul de energie ABC, valorificat fluxul de energie DEF.
TWIDELL PAGINARE.indb 9
01/12/2014 11:35
10
Principii de energie din surse regenerabile
Tabelul 1.1
Comparație din surse regenerabile și convenționale de energie sisteme
Furnizare de energie regenerabilă (verde)
Convenționale de energie consumabile (maro)
Exemple
Eoliană, solară, biomasă, maree.
Cărbune, petrol, gaze naturale, minereu radioactiv.
Sursa
Naturale locale de mediu.
Stoc concentrate.
Normal stat
Un curent sau flux de energie. O
"venitul".
Static magazin de energie. Capital.
Medie inițială
intensitatea
Intensitate scăzută, dispersate:
≤ M 300W
-2
.
Lansat la
≥100 kW m
-2
.
Durata de viata de aprovizionare
Infinit.
Finite.
Costul la sursa
Gratuit.
Din ce în ce mai scumpe.
Echipamente de capital
costul pe kW
capacitate
Scumpe, de obicei
≈1000 de dolari.
Moderate, poate 500 de dolari, fără emisii
de control; încă
>$1000 cu emisiile
reducere.
Variație și
control
Fluctuante; cel mai bine controlate de schimbare
de sarcină folosind pozitive feedforward
control sau surse complementare.
Constant, cel mai bine controlată prin ajustarea sursa
cu feedback-ul negativ de control.
Locația de utilizare
Site-ul și societatea specifice.
General și utilizare la nivel mondial.
Scara
La scară mică, de multe ori economice.
A crescut scară de multe ori îmbunătățește alimentarea
costurile; scară largă a favorizat frecvent.
Competențe
Interdisciplinare și variate. Larg
gama de competențe. Importanța bioscience
și agricultură.
Legături puternice cu descărcări electrice și mecanice
inginerie. Gamă îngustă de personal
abilitățile.
Contextul
Ei bine adaptate la mediul rural situații și
descentralizate industrie.
Scară favoruri urban, centralizat industrie.
Dependența
Auto-suficiente sisteme de încurajat.
Sistemele dependente în afara intrări.
De siguranță
Locale pericole posibile în funcțiune:
de obicei, în condiții de siguranță atunci când de acțiune.
Poate fi protejat și închis pentru a reduce
mare pericolele potențiale; cele mai periculoase
atunci când defect.
Poluarea și
mediu
daune
De obicei, puține daune mediului,
mai ales la scară moderate.
Poluarea mediului comun, mai ales
de aer și de apă.
Pericolele de la excesiv de lemn ars.
Eroziunea solului de la excesiv de utilizare a biocombustibililor.
Mare hidro rezervoare perturbator.
Daune permanente comune din
minerit și elemente radioactive intrarea
apei. Defrișările și ecologice
sterilizare la poluarea excesivă a aerului.
Emisiile de gaze de seră care cauzează schimbările climatice
schimba.
Estetica, vizual
impactul
Locale perturbatii pot fi grave,
dar sunt de obicei acceptabile dacă locală nevoie
percepută.
De obicei, utilitare, cu centralizarea și
economia de scară largă.
TWIDELL PAGINARE.indb 10
01/12/2014 11:35
§1.3 Fundamentele
11
indicat în Fig. 1.2, și că nu toate geotermale de energie este regenerabilă în
sens strict, așa cum este explicat în Capitolul 14. Energie finită este derivat din
surse 1 (combustibili fosili), 4, și 5. A cincea categorie este relativ minore, dar
este util pentru baterii primare (de exemplu uscată a celulelor).
§1.3.2 Mediu de energie
Fluxul de energie trece continuu ca surse regenerabile de energie prin
Pământ sunt prezentate în Fig. 1.2. De exemplu, totalul fluxului solar absorbit la
nivelul mării este de aproximativ 1.2
× 10
17
W. Astfel solar flux atingerea Pământului
suprafața este
~20 MW per persoană; 20 MW este puterea de ten foarte mare
diesel generatoare electrice, suficient pentru a furniza toate nevoile de energie ale unui
oraș de aproximativ 50.000 de oameni! Maxim solar densitate de flux
(iradiere) perpendicular pe fascicul solar este de aproximativ 1 kW/m
2

; un foarte util
și ușor să-și amintească numărul. În termeni generali, o ființă umană este capabil
de a intercepta astfel de un flux de energie, fără a afecta, dar o creștere începe să
provoca stres și dificultăți. Interesant, putere densități de flux de vânt,
de apă curenți sau valuri
>1 kW/m
2

începe, de asemenea, de a provoca dificultate fizică


a unui adult.
Fig. 1.2
Natural de curenții de energie de pe Pământ, arătându-sisteme de energie regenerabilă. Nota mare
gama de flux de energie (1:10
5

) și dominația de radiațiile solare și căldură. Unități terawatt


(10
12

W).
Reflectate
în spațiu
50 000
Solar
radiații
De La
Soare
Din
Pământ
De la
planetare
mișcare
120 000
Absorbit pe
Pământ
40 000
80 000
Sensibil
încălzire
Căldură latentă
de apă
evaporare
300
Cinetică
energie
Foton
procese
Geotermale
30
100
Căldură
Gravitation,
mișcarea orbitală
Maree de mișcare
3
Infraroșu
radiații
în spațiu
Încălzitoare de apă solare
Solar clădiri
Solare uscătoare
Ocean de energie termică
Hidroenergie
putere Osmotic
Vânt și val turbine
Biomasa și biocombustibilii
Fotovoltaice
Căldură geotermală
energia Geotermală
Tidal gama de putere
de curent de Maree putere
TWIDELL PAGINARE.indb 11
01/12/2014 11:35
12
Principii de energie din surse regenerabile
Cu toate acestea, la nivel mondial date în Fig. 1.2 sunt de mică valoare pentru practice
aplicații de inginerie, din moment special site-uri pot fi extrem de
diferite medii și posibilități pentru valorificarea surselor regenerabile de energie.
Evident, regiunile plate, cum ar fi Danemarca, au puține șanse pentru
hidroenergie, dar poate fi energia eoliană. Totuși, în regiunile învecinate (de exemplu Norvegia)
poate avea vast potențial hidroenergetic. Pădurile tropicale poate avea biomasă
surse de energie, dar deserturi la aceeași latitudine am nici unul (mai mult decât atât,
păduri nu trebuie să fie distruse, ceea ce ar face mai multe deserturi). Astfel,
practic sisteme de energie regenerabilă trebuie să fie corelate la anumite
locale de mediu, fluxurile de energie care au loc într-o anumită regiune.
§1.3.3 Primare de alimentare la final
Toate sistemele de energie poate fi vizualizat ca o serie de conducte sau circuite
prin care curenții de energie sunt canalizate și transformate pentru a
deveni utilă în piața internă, industriale, agricole și de circumstanțe.
Fig. 1.3(a) și Fig. 1.4 sunt diagrame Sankey de aprovizionare cu energie, ceea ce
arată fluxurile de energie prin sistemul energetic național (adesea numit un
'diagramă spaghetti' din cauza aspectului său). Secțiunile peste un astfel de
diagramă poate fi desenat ca diagrame plăcintă arată aprovizionare cu energie primară și
furnizarea de energie finală (Fig. 1.3(b)). Notă modul în totalul consumului final de energie
este mai mică decât primară de alimentare din cauza pierderilor în transformarea
proceselor, în special în generarea de energie electrică din combustibili fosili.
§1.3.4 planificarea Energetică
Anumite principii comune se aplică pentru proiectarea și evaluarea energetice
de aprovizionare și de a folosi, dacă avem în vedere furnizarea de energie la nivelul
unei națiuni, un oraș, sau o gospodărie.
1
Completați sisteme energetice trebuie să fie analizat și de aprovizionare nu ar trebui să
fie luate în considerare separat de utilizarea finală. Din păcate, precis are nevoie
de energie sunt prea adesea uitat, și consumabile nu sunt bine
adaptate la utilizarea finală. Pierderile de energie și funcționarea neeconomică
, prin urmare, duce frecvent. De exemplu, dacă o dominantă interne de energie
cerință de căldură pentru căldură sau apă caldă, este iresponsabil pentru a
genera rețeaua de energie electrică de calitate de la un combustibil, deșeurile majoritatea
de energie termică ca de emisie din cazan și turbină, de a distribui
energie electrică, cu pierderi, și apoi disipa energie electrică livrată ca
de căldură: o pierdere totală de aproximativ 75%! Din păcate, astfel de ineficiență, de respect pentru
resurse, și inutile de poluare asociate de multe ori apar. Încălzire
ar fi mai eficient și cost-eficient de căldură directă a producției
locale de distribuție. Chiar mai bine este de a combina energie electrică
generație cu producerea de căldură folosind COGENERARE (producere combinată de căldură și energie
(energie electrică)).
TWIDELL PAGINARE.indb 12
01/12/2014 11:35
§1.3 Fundamentele
13
Fig. 1.3
Energie diagrame de flux pentru Austria, în 2010,
cu o populație de 8,4 m. (a) Sankey ('spaghetti') diagrama, cu surse regenerabile de
energie fluxurile umbrită în verde; fluxurile de energie care implică termică energie electrică arătat punctată; (b) pie diagrama de surse;
(c) pie
diagrama de utilizări finale.
Contribuția de hidroenergie și biomasă (lemn și deșeuri) este mai mare decât în majoritatea țărilor industrializate, cum este
utilizarea de căldură produsă de termica de generare de energie electrică ('combinată de căldură și energie', CHP). Consumul de energie
pentru transport
este substanțială și foarte dependente (de import), petrol și produse petroliere; prin urmare, guvernul Austriac încurajează
creșterea utilizării biocombustibililor. Austria consumul de energie a crescut cu peste 50% între 1970 și 2010, deși populația
a crescut cu mai puțin de 10%, indicând nevoia pentru o mai mare eficiență a consumului de energie.
Sursa de date: www.iea.org/statistics.
PRIMARE
DE ENERGIE
CONSUMABILE
(a)
Brut
ulei
Non-consumul de energie
Non-consumul de energie
Termice,
elec. gen
(inc. CHP)
Rafinare
Uleiul de
produse
Cărbune
Fosili
gaz
Biomasa
+
deșeuri
Hidro
200 PJ
Deșeuri
căldură
Districtul
de căldură etc.
De energie electrică
Rezidential
+
comerciale
Agricultura
+
silvicultură
Industria
Transport
ENERGIE
END
UTILIZARE
(b) aprovizionare cu energie Primară
(total: 1410 PJ)
Cărbune
10%
Fosili gaz
25%
Bio + deșeuri
18%
Hidro
10%
Petroliere
37%
(c) consumul final de Energie
(total: 1150 PJ)
Resid + comm ' l
27%
Transport
23%
Industria
21%
Ag + Silvicultură
29%
TWIDELL PAGINARE.indb 13
01/12/2014 11:35
14
Principii de energie din surse regenerabile
2
Eficiența sistemului calculele pot fi cele mai revelatoare și poate
indica pierderi inutile. Aici vom defini "eficiență" ca raportul dintre
energia utilă de ieșire dintr-un proces total de energie de intrare la
acest proces. Ia în considerare de iluminat electric produs de "convențional"
termic generat de rețeaua de energie electrică și lămpi. Succesive de energie
eficiență ar fi: energie electrică generație
De ~30%, distribuția
~90%, și iluminat incandescent (de energie în radiații vizibile, de obicei,
cu o lumină-umbră) de la 4 la 5%. Randamentul total este de aproximativ 1 până la 1,5%.
Contrast cu cogenerare de energie termică utilă și energie electrică
(de eficiență energetică
~85%), distribuție (~90%), și de iluminat moderne
consum redus fluorescente compacte (CFL) lumini (
~22%) sau lumina
emitting diode (LED) lumini (
~80%). Randamentul total este acum de aproximativ
17 la 60%, o îmbunătățire a eficienței energetice de către factorii de 10 până la 40 de ani!
Totalul costurilor pe ciclu de viață de cel mai eficient sistem va fi mult
mai puțin decât în sistemul convențional, în ciuda mai mare pe unitate de capital
costurile, deoarece (i) mai puțin capacitatea de generare și de combustibil sunt necesare, și
(ii) echipamente (mai ales lămpi) dureaza mai mult (a se vedea Problemele 1.2 și
1.3).
3
Managementul energiei este întotdeauna important pentru a îmbunătăți eficiența
și de a reduce pierderile economice. Nu de aprovizionare cu energie este gratuită și regenerabilă
consumabile sunt capital intensive. Astfel, nu există nici o scuză pentru a pierde
energie de orice formă inutil. Eficiența cu finite, combustibili reduce
poluarea; eficiența cu surse regenerabile de energie reduce costurile de capital. Capitolele
15, 16, și 17 conține mai multe detalii și exemple.
§1.4 PRINCIPII ȘTIINȚIFICE DIN SURSE REGENERABILE
ENERGIE
1
§1.1 și Fig. 1.1 indică diferențe fundamentale între
regenerabile (verde) și finite (maro) livrările de energie. Ca o consecință, la
utilizarea eficientă a surselor regenerabile de energie necesită aplicarea corectă a
unor principii; de exemplu, pentru a realiza că potențialul pentru o
anumită sursă de energie regenerabilă la un site depinde în primul rând
înțelegerea și cuantificarea naturale de mediu, fluxurile de energie de la site-ul respectiv
(de exemplu, viteza vântului, radiația solară). Acest lucru necesită, de obicei, cel puțin un
an de măsurare, dar pot fi evaluate din înființată înregistrări
(de exemplu.g. înregistrările meteorologice). Același lucru este valabil și pentru utilizarea de deșeuri
(de exemplu, animal pastă de biogaz). Schematic, in Fig. 1.1 energia
de curent ABC trebuie să fie evaluate înainte de a deviat circulația prin DEF este
stabilit.
TWIDELL PAGINARE.indb 14
01/12/2014 11:35
§1.4 principii Științifice de energie din surse regenerabile
15
§1.4.1 caracteristici Dinamice
Utilizare finală cerințe pentru energia variază cu timpul. De exemplu, energie electrică
cererea de pe o rețea de alimentare cu vârfuri de multe ori în dimineața și seara,
și ajunge la un minim noaptea. Dacă puterea este furnizată de
o sursă finită, cum ar fi ulei, intrare poate fi ajustată în răspuns la
cerere. Neutilizate de energie nu este pierdut, dar rămâne cu sursa de combustibil.
Cu toate acestea, cu sisteme de energie regenerabilă, nu numai utilizare finală variază
incontrolabil cu timpul, dar asa nu prea alimentarea naturală în
mediu. Astfel, un regenerabile de energie dispozitiv trebuie să fie adaptate dinamic la
ambele D și E din Fig. 1.1; caracteristicile va fi, probabil, destul de
diferite la ambele interfețe. Capitolul 15 recenzii multe metode disponibile
pentru o astfel de potrivire, inclusiv de stocare a energiei; analiza acestor dinamic
efecte pentru anumite tehnologii este dat în cele mai multe capitole.
Marile periodice variante de surse regenerabile sunt prezentate în Tabelul 1.2,
dar precis comportamentul dinamic pot fi afectate de neregularitățile locale.
Tabelul 1.2
Intensitatea și frecvența proprietăți din surse regenerabile
Sistemul
Marile
perioade
Variabile majore
Putere
relație
Comentariu
Text
de referință
Direct
soare
24 h, 1 y Solare fascicul de radiație G
b
*
(W/m
2
); Unghi fascicul de
vertical
q
z
P
∝ G
b
*
pentru că
q
z
P
max
. = 1kW/
m
2
Doar in timpul zilei
§2.5
Difuze
soare
24 h, 1 y stratul de nori,
poate că poluarea aerului
P<<G;
P
≤ 300 W/
m
2
Semnificative de energie,
cu toate acestea
§2.8
Biocombustibili
1y
Starea solului, expunere la soare,
apă, specii de plante,
deșeuri
Energia stocată
~10 MJ/kg
Foarte multe
variante; legate de
agricultură și silvicultură
§9.6
Vântul
1y
Viteza vântului u
0

Înălțimea nacelei de mai sus


la sol z ;
înălțime de anemometru
catarg h
P
∝ u
o
3
u
z
/u
h
= (z/h)
b
Extrem de fluctuante
b ~ 0.15
§8.3
§7.3
Val
1y
'Înălțimea semnificativă a valurilor' H
s
val perioada T
P
∝ H
s
2
T
Densitate mare de putere
~50kW/m pe val
fața
§11.4
Hidro
1y
Rezervor de înălțime H
volumul de apă debitul Q
P
∝ HQ
Stabilit de resurse
§6.2
Maree
12 h 25
min
A mareelor R; cuprinse
zona O; estuar lungime L,
adâncime h
Maree-curent de alimentare
P
∝ R
2
O
P
∝ u
o
3
Îmbunătățită de maree
gama dacă L /
√h = 36000 m
1/2

§12.5
§12.3
§
12.4
Ocean
termică
gradient
Diferență de Temperatură constantă
între suprafața mării și
apă adâncă,
DT
P
∝ (DT)
2
Unele tropicale
locațiile au
DT~20°C,
potențial harnessable dar
la o eficiență scăzută
§13.3
TWIDELL PAGINARE.indb 15
01/12/2014 11:35
16
Principii de energie din surse regenerabile
Gama de sisteme foarte variabile (de exemplu, energia eoliană) de la extrem de
previzibil (de exemplu, energia mareelor). Soare poate fi extrem de previzibil, în unele
regiuni (de exemplu, Sudan) dar oarecum aleatoare în altele (de exemplu, marea BRITANIE).
§1.4.2 Calitatea ofertei
Putem defini calitatea ca proporția de o sursă de energie care poate fi
transformată în lucru mecanic. Astfel, energia electrică are de înaltă calitate, pentru că
atunci când este consumată într-un motor electric,
>90% din energia de intrare poate fi
transformată în lucru mecanic, să zicem, pentru a ridica o greutate; pierderile de căldură sunt
, prin urmare, mici: <10%. Calitatea nucleare, combustibili fosili sau biomasa de combustibil într -
o singură etapă, centrala termica este moderată, pentru că doar aproximativ
33% din valoarea calorică a combustibilului este transformată în lucru mecanic
și circa 67% se pierde sub formă de căldură în mediul înconjurător. În cazul în care combustibilul este utilizat
într -
un ciclu combinat stație de putere (de exemplu, gaz metan turbina etapă
urmată de turbine cu abur), calitatea este crescut la
~50%. Este posibil
să analizeze astfel de factori în termeni de variabile termodinamice exergie,
definită aici ca 'teoretic sumă maximă de lucru de obținut, la
un anumit mediu de temperatură, de la o sursă de energie'.
Din surse regenerabile de energie sisteme de alimentare sunt împărțite în trei clase mari.
1
Mecanice consumabile, cum ar fi hidro (Capitolul 6) , vânt (Capitolele 7
și 8), val (Capitolul 11), și a mareelor (Capitolul 12). Cele
mecanice sursa de putere este de obicei transformată în energie electrică la mare
eficiență. Proporția de putere în mediul extrase de
dispozitive este determinată de limitele operaționale ale procesului, legate
de variabilitatea de la sursă, după cum este explicat în capitolele ulterioare. La
proporțiile sunt, de obicei, vânt
De ~35%, hidro ~80%, val ~30%, și
maree (gama)
De ~60%. Aceste proporții se referă la factorul de capacitate și
ore de încărcare de dispozitive (a se vedea §1.5.4 și Tabelul D4 în Anexa D).
2
Căldură consumabile, cum ar fi arderea biomasei (Capitolul 10) și solare
colectoare (Capitolele 3 și 4). Aceste surse oferă căldură la mare
eficiență. Cu toate acestea, proporția maximă de energie termică care poate fi extras
ca lucru mecanic, și, prin urmare, de energie electrică, este dată de legea a doua
a termodinamicii și Carnot Teorema, care presupune
reversibile, infinit de mult transformări. În practică, mecanică maximă
de energie produsă într-un proces dinamic este de aproximativ jumătate care a prezis de
la Carnot criterii. Pentru cazan termic motoare termice și internă
motoarele cu ardere maximă realizabilă calitate este de aproximativ 35%.
3
Foton procese, cum ar fi fotosinteza și fotochimie
(Capitolul 9) și fotovoltaice de conversie (Capitolul 5). De exemplu, solar
fotoni de o singură frecvență poate fi transformat într-mecanice
de muncă cu înaltă eficiență, folosind o pereche de celule solare. În practică,
bandă largă de frecvențe în spectrul solar face de potrivire
dificil și fotonice eficiențe de conversie de 25% sunt considerate bune.
TWIDELL PAGINARE.indb 16
01/12/2014 11:35
§1.4 principii Științifice de energie din surse regenerabile
17
§1.4.3 Dispersate față centralizate de energie
O diferență pronunțată între regenerabile și energie finită
livrările de energie densitatea de flux la inițială de transformare. Regenerabile
de energie frecvent vine cu o densitate de flux de aproximativ 1 kW/m
2

(de exemplu, solar


fascicul de radiație, energia vântului la 10m/s), în timp finit centralizat,
sursele de energie densități de flux care sunt ordine de mărime mai mare.
De exemplu, cazan tuburi în cuptoare cu gaz transfera cu ușurință 100 kW/m
2

,
și într-un reactor nuclear primul perete schimbător de căldură trebuie să transmită
mai multe MW/m
2

. La final, după distribuție, cu toate acestea, livrările de


finite surse trebuie să fie redus foarte mult în densitatea de flux. Astfel, în afară de
marile excepții, cum ar fi de metal de rafinare, utilizare finală sarcini pentru ambele
regenerabile și aprovizionarea cu finite sunt similare. În general, energie finită este cel mai usor de
"produs"
2

la nivel central și este scump pentru a distribui. Energia regenerabilă


este cel mai usor de "produs", în locații dispersate și este scump să
se concentreze.
Astfel, tehnologiile de energie regenerabilă încuraja dispersate și
distribuite sisteme de energie. Acestea sunt instalate de către companii și utilități,
de exemplu, ca fermele eoliene (§8.8), curent de maree plante (§12.4), trestie de zahăr
mills (§9.6), și, de asemenea, ca scară mai mică microgenerare de căldură și/sau
energie electrică de către persoane fizice, întreprinderi mici și comunitățile locale ca
alternative sau suplimente la tradiționala rețea de alimentare centralizată.
Exemple de microgenerare includ rețelele fotovoltaice (§5.3),
combinată de căldură și energie de pe site-uri industriale, și biogaz pe ferme (§10.7).
O lume de beneficii, mai ales în țările în curs de dezvoltare, este că moderne
tehnologii de energie regenerabilă permite de la distanță comunitățile să se bucure de
beneficii și servicii (de exemplu, de iluminat și de telecomunicații) anterior
limitat la populația urbană. Atunci când energiile regenerabile instalații sunt de o
scară mare de 500 MW sau mai mult (de exemplu, parcurile eoliene, hidro
generație, biomasă, centrale termice), apoi speciale de transport și
de energie electrică de înaltă tensiune linii de transmisie sunt necesare; de multe ori aceste livrare
sisteme de alimentare cu energie urbane complexe.
§1.4.4 Complexe (interdisciplinare) sisteme de
Furnizare de energie regenerabilă sunt strâns legate naturale
de mediu, care nu este apanajul doar o disciplină academică, cum ar fi
fizica sau inginerie electrică. Frecvent este necesar să treacă
granițele disciplinare la fel de departe ca, să zicem, fiziologia plantelor electronice
de control de inginerie. De exemplu, moderne, de trestie de zahăr industries produce
nu numai zahăr, dar, de asemenea, de combustibil lichid (etanolului). Procesul complet într-o
societate rurală necesită intrare de științe agricole și sociologie, ca
bine ca chimică, mecanică și inginerie electrică (a se vedea Casetele 9.2
și 10.2).
TWIDELL PAGINARE.indb 17
01/12/2014 11:35
18
Principii de energie din surse regenerabile
§1.4.5 Situație de dependență
Nici un singur sistem de energie regenerabila este universal aplicabilă, deoarece
capacitatea mediului local pentru a furniza energie și la capacitatea
societății de a accepta energie variază foarte mult. Este necesar ca să 'perspectiva'
mediu pentru energia din surse regenerabile ca este de prospectare geologică a
formațiunilor de petrol. Cu toate acestea, evaluarea potențialului de surse regenerabile de energie
resurselor este de obicei mult mai ușor și mai ieftin decât prospecțiuni pentru petrol!
De asemenea, este necesar să se efectueze de energie anchete interne,
agricole, industriale și nevoile comunității locale. Special end-utilizarea
nevoile locale și furnizare de energie regenerabilă pot fi corelate,
sub rezerva economică și de constrângerile de mediu. În acest sens
regenerabile de energie este similar cu agricultura. Special medii și
solurile sunt potrivite pentru unele culturi, și nu pentru alții, și de a trage de piață
pentru vânzarea de produse va depinde de nevoile specifice. Astfel, solar
sisteme de energie în sudul Italiei ar trebui să fie destul de diferite de cele
din Belgia sau într-adevăr, în nordul Italiei. Porumb alcool combustibili poate fi
potrivit pentru agricultori în Missouri, dar nu în New England. Consecința
este că planificarea optimă de surse regenerabile de energie furnizarea și utilizarea tinde să se aplice
în regiunile de la distanță scară
~250 km, dar nu 2500 de km, pentru că pe
distanțe mai mari de aprovizionare opțiuni sunt susceptibile să se schimbe. Din păcate,
urbane mari și societățile industrializate au construit în moduri care
nu sunt potrivite pentru o astfel de flexibilitate și de variație pentru optimizarea surselor regenerabile
de cerere și ofertă.
§1.5 IMPLICAȚIILOR TEHNICE
§1.5.1 Prospectare a mediului
Primul pas este o rapidă evaluare din care surse regenerabile sunt în
cantități suficiente pentru a justifica mai detaliate de monitorizare. Ordinul de
mărime formulele indicate în cauză "tehnologie" capitole este suficient pentru
acest scop. Datorită variațiilor sezoniere în cele mai multe opțiuni de surse regenerabile de energie
(sezonul umed să se usuce sezonul de iarnă sau de vară), resurse (energie
flow) trebuie să fie monitorizate timp de cel puțin un an de la site-ul în cauză.
Pentru proiecte de mare anvergură (de exemplu, hidro de
~100 MW), un deceniu sau mai mult de
date pot fi necesare. Analiza în curs de desfășurare trebuie să se asigure că datele utile sunt
înregistrate, în special cu privire la caracteristicile dinamice ale
sistemelor energetice planificate. Datele meteorologice sunt întotdeauna importante,
dar, din păcate, pe site-uri oficiale stații sunt adesea diferite de
cele generatoare de energie-uri, și metode de înregistrare și analiză
nu sunt ideale pentru energie prospectare. Cu toate acestea, un important utilizarea pe
termen lung a datelor de la stațiile de monitorizare oficiale este ca o bază pentru
comparație cu site-ul local de variante. Astfel, viteza vântului poate fi monitorizată de
mai multe luni la un potențial generatoare de site-ul și în comparație cu datele
de la cel mai apropiat oficial stația de bază. Extrapolarea cu ajutorul mulți ani de
TWIDELL PAGINARE.indb 18
01/12/2014 11:35
§1.5 implicațiilor Tehnice
19
stație de bază de date poate fi posibil. Cu toate acestea, pe termen lung
variații în vreme apar, cu dezbaterea continuă despre cât de mult este
din cauza umane induse de schimbările climatice.
Datele nu au legătură cu măsurători meteorologice normale poate fi
dificil de a obține. În special, fluxurile de biomasă și deșeuri vor
de multe ori nu au fost evaluate anterior, și nu au fost
luate în considerare pentru generarea de energie. În general, prospectare pentru livrările de
energie din surse regenerabile necesită metode de specialitate și echipamente care
cererea de resurse semnificative de finanțare și de forță de muncă. Din fericire
link-uri cu meteorologie, agricultură și științe marine dau naștere la o
cantitate considerabilă de informații de bază.
§1.5.2 End-cerințe privind utilizarea și eficiența
Deoarece nu există alimentare cu energie este ieftin sau apare fără o formă de
deteriorării mediului înconjurător, este esențial să se utilizeze eficient energia (adesea numit
conservarea energiei). Cu sisteme electrice de utilizare finală cerință
este numit de încărcare sau a cererii; dimensiunea și caracteristicile dinamice ale
generație trebuie să fie adaptate la cerințele de sarcină. Așa cum sa explicat
în Capitolul 16, banii cheltuiți pe de conservare a energiei și îmbunătățiri
în eficiența utilizării dau, de obicei, mai bine beneficiile pe termen lung decât banii
cheltuiți pe creșterea producției și furnizării de capacitate. Cea mai mare energie
cerințe sunt, de obicei, pentru căldură și transport. Ambele utilizări sunt
asociate cu stocarea energiei capacitate în masă termică, baterii, sau de combustibil
tancuri, și includerea acestor componente în sistemul energetic design
poate îmbunătăți considerabil eficiența globală.
§1.5.3 corelarea cererii cu oferta: sisteme de energie și
mecanisme de control
După cuantificarea și analiza separată dinamic
caracteristicile end-cereri de utilizare și de mediu opțiuni de alimentare, valoarea totală a
cererii și ofertei sunt unite (integrat) ca un sistem energetic. La
următoarea schiță introduce concepte cheie de sisteme practice
discutate în capitolele ulterioare, inclusiv Capitolul 15 (care se ocupă în mai multe
termeni specifici cu rețelele energetice, de stocare a energiei, energie și
transport). Mai multe dintre sisteme se referă, de asemenea, la evoluțiile de "inteligente"
de tehnologie, care sunt aduse în furnizarea de utilități de energie electrică din
rețelele. O astfel de tehnologie inteligentă este, de asemenea, important pentru autonomă
sisteme.
Principiile se aplică într-o oarecare măsură a livrărilor de energie termică și a combustibililor, dar
sunt imediat aplicabile potrivite cererea și oferta de electricitate.
Prin urmare, în Fig. 1.4, vom folosi electrice simbol al (i) de
neoprit 'curent' (două cercuri împletite) pentru sursele regenerabile de energie surse
de energie în mediul de a fluxurilor de ce au nevoie; (ii) un
TWIDELL PAGINARE.indb 19
01/12/2014 11:35
20
Principii de energie din surse regenerabile
tensiune constantă "baterie" (două linii paralele) pentru finită surse (fosili
combustibili și nucleare); (iii) feedback-ul de control (o cruce diagonală într-un
cerc), cu o
semnul + pentru feedback-ul pozitiv, și un semn – pentru negative
feedback-ul.
Fig. 1.4. (a). Corelarea cererii eficient la sursele regenerabile de alimentare
este importantă pentru că: (i) costul de capital din surse regenerabile de energie generație este
un factor dominant, și deci capacitatea, și, prin urmare, costul, de genuri-
Alimentare cu
dispozitiv de
Alimentare cu
dispozitiv de
Finite
sursa
Alimentare cu
dispozitiv de
Alimentare cu
dispozitiv de
D
E
Feedback-ul de control
Feedback-ul de control
+
+
+
+
+

+

Feedforward
control
Vărsat de energie
Utilizare finală
Utilizare finală
End-utilizarea # 1
(prioritate mare)
Utilizare finală # 2
(low priority)
Utilizare finală
Mediu:
chiuveta
Mediu:
chiuveta
Mediu:
chiuveta
De mediu
sursa
F
D
(a)
(b)
(c)
(d)
E
Fig. 1.4
Potrivire sursă de energie regenerabilă pentru utilizare finală: diagrama schematică simplificată pentru a ilustra mecanismele de
control. Simboluri: OO
energie (curent) sursa,
⊗ dispozitiv de control, -- → - fluxul de energie, - - → - controlul link-ul (electronic sau mecanic).
o
Maxim flux de energie pentru dimensiunea minimă de dispozitiv sau sistem necesită rezistență scăzută la flux la D, E, și F (notă: D, E,
și F corespund aceleași puncte în deviat fluxul de Fig. 1.1).
b
Feedback-ul negativ de control pentru un sistem cu finită surse permite carburantului să fie salvat ca sarcină scade.
c
Feedback-ul negativ pentru un sistem cu pur regenerabile de intrare scurgeri de energie dincolo de faptul că este necesar de sarcina.
d
Pozitiv feedforward sarcina de management de control al ofertei, de separare 'de mare prioritate' și 'low-prioritate' sarcini, astfel încât
totală
de încărcare la E pot fi adaptate la oferta disponibilă la D la toate ori.
TWIDELL PAGINARE.indb 20
01/12/2014 11:35
§1.5 implicațiilor Tehnice
21
tor nu trebuie să fie excesive; (ii) fluxul de energie (putere), deși
, în esență, gratuit de la sursă, trebuie să treacă în mod eficient la utilizarea finală
a cererii; prin urmare, rezistențele la flux de energie în D, E, si F ar trebui să
fie mici.
Fig.1.4(b). Feedback-ul negativ control de la cererea de ofertă este normală
cu energie fosilă sisteme. De exemplu, într-un automobil, conducătorul auto
utilizează accelerație pentru a reduce alimentarea cu combustibil dacă viteza vehiculului este
în creștere prea repede (de exemplu, o relație negativă).
Fig 1.4(c). Pentru o sursă de energie regenerabilă, feedback-ul negativ, ca în (b),
rezultate în potențial utile de energie irosite sau 'varsat'. Acest lucru este pentru că
regenerabile de energie este un flux sau sursă de curent în mediu care pot
fi deturnate dar nu s-a oprit. De exemplu, o turbină eoliană de operare la
mai puțin decât capacitatea maximă poate produce mai multă energie electrică decât sarcina
impune; controlând astfel în mod negativ deșeuri posibilitatea pentru mai
cost-free energy.
(e)
(f)
Alimentare cu
dispozitiv de
Aprovizionare
dispozitiv 2
Aprovizionare
dispozitiv 1
Utilizare finală
#2
Utilizare finală
#1
Utilizare finală
Depozitare
Feedforward
control
Feedback-ul
RE
surse
Finite
sursa (s)
Feedback-ul
Mare sistem grilă
Energie
depozitare
Control
E
1
+

F
2
F
2
F
Mediu:
chiuveta
Fig. 1.4
(cont.)
e
De stocare a energiei permite caracteristicile dinamice ale utilizării finale să fie decuplate de la
alimentarea cu caracteristici.
f
Un mare sistem de grilă pot încorpora atât feedback-ul (de adaptare a ofertei la
cerere) și feedforward (pentru a comuta pe 'low-prioritate' încarcă doar atunci când alimentarea este
adecvat).
TWIDELL PAGINARE.indb 21
01/12/2014 11:35
22
Principii de energie din surse regenerabile
Fig 1.4(d). Aici feedforward sarcina de control de gestiune de
energie din surse regenerabile de alimentare separat de serviciile de înaltă prioritate " și "low-prioritate'
de sarcini. De 'low-prioritate' sarcini sunt dimensionate astfel încât ajustări automate
va permite sarcina totală pentru a se potrivi aprovizionare disponibile la D la toate ori.
Potrivit low-prioritate sarcinile au o capacitate de stocare (de exemplu, un rezervor de apă caldă sau
congelare), sau va tolera întreruptă de aprovizionare (de exemplu, o mașină de spălat rufe).
Furnizorul încurajează utilizatorii să se conecteze astfel de sarcini, oferind
energie electrică la tarife reduse. Această metodă este cunoscută ca 'off-peak' de utilități rețeaua
de energie electrică, și, ca și cu unele sisteme autonome, cum ar fi vântul bazate pe
sistemul de pe mica Insula Scoțiană Fair Isle (a se vedea Caseta 8.2), și cu
niște mini-hidro (§6.6).
Fig. 1.4(e). O modalitate evidentă de a se potrivi cererile și așteptările care au
diferite caracteristici dinamice, și totuși să nu-și piardă altfel
'harnessable de energie, este de a încorpora depozitare (a se vedea Capitolul 15).
Fig. 1.4(f). Marea majoritate a surselor regenerabile de energie energie electrică,
generatoarele sunt conectate la rețea la rețelele de utilități. Aceasta include
microgeneratoarele, astfel încât să permită imediat de import atunci când microgenerator de alimentare este
insuficientă pentru a satisface cererea și exportul imediată atunci când există mai mult
decât suficiente generație. Rezultatul este de a decupla locale de aprovizionare de
cererea locală. De lucru folosind grila pentru atât exportul cât și importul de
energie, grila devine un 'magazin virtual'. Mai mult decât atât, există încă
avantajul de a avea comutare sarcini pentru a optimiza pe site-ul utilizat de
microgenerare.
§1.5.4 Eficiență, factorii de capacitate și potențial de resurse de
regenerabile de energie dispozitive
(a) Eficiența
O întrebare comună din partea publicului este: 'Cât de eficiente sunt regenerabile
de energie dispozitive, de exemplu, turbine eoliene și panouri solare?' Cu toate acestea,
ceea ce pare a fi o simplă întrebare nu se poate răspunde atât de ușor.
Nici nu este ușor să răspundă la întrebări de genul 'Cat de eficient este acest motor
masina?" sau " Cât de eficient este acel sportiv?, sau Cât de eficientă este electric
ceas?' Întrebări ca acestea nu sunt niciodată simple, mai ales ca
petentul înseamnă de multe ori: 'Este acel dispozitiv de cost-eficiente?' A sugerat răspunsurile
la un chestionar sunt prezentate după cum urmează.
Nu, "eficiența" contează? Pare evident la prima vedere că
dispozitive tehnologice ar trebui să fie "eficienți". Cu toate acestea, acest cuvânt înseamnă diferite
lucruri pentru diferiți oameni despre tehnologii diferite, ca în
exemplele de mai jos ilustrează pentru generarea de energie dispozitive. Pentru un frigider, de
exemplu, conceptul de eficiență include de vedere, dacă volumul
este suficient pentru nevoile, dacă ușa se deschide și se închide cu ușurință, și dacă
consumul de energie electrică este acceptabil. În practică, mulți factori sunt
TWIDELL PAGINARE.indb 22
01/12/2014 11:35
§1.5 implicațiilor Tehnice
23
relevante. Deciziile sunt luate prin compararea un dispozitiv cu un alt până la
ultima opțiune este selectat; cu toate acestea, încă avem tendința de a spune că am
ales cele mai eficiente frigider'.
Eficiența tehnică a unui dispozitiv de energie, de obicei, înseamnă utile
energiei furnizate ca o fracțiune din energia de intrare. Prin această definiție, cele
mai bune eficiențe de diverse dispozitive comune sunt după cum urmează:

Centralele electrice (de energie electrică de la rețea/de intrare de căldură; cu nici o utilizare a respins
căldură): cărbune și petrol
De ~35%, turbină cu gaz ~45%, nuclear ~30%.

Masini (motiv de energie/căldură de ardere de combustibil)
~10%.

Biciclist în timp ce curse (rata de motiv energie/rata de alimentare metabolismul)
~7%.

Regulat ciclist peste un an (anual motiv de energie/anual de produse alimentare
metabolismul) <1%.

Generator de energie electrică (energie electrică/putere arbore în)
~95%.

Bicicleta (motiv putere de ieșire/putere pedala in)
~85%.

Incandescente de lumină electrică
3
(vizibil lumina/energie electrică în)
~2.5%.

Diodă emițătoare de lumină (LED) (vizibil lumina/energie electrică în)
~12%.
Din această gamă de răspunsuri este clar că (i) , "eficiența" trebuie să fie
definite foarte atent pentru fiecare întrebare, și (ii) comune importante
dispozitive pot avea mici eficiență, dar acest lucru nu împiedică lor
de utilizare. Mai mult decât atât, energia de intrare, de obicei, nu include energia de a
sechestra combustibil sau alimentare; dacă acest lucru este inclus, eficiență poate deveni
mult mai puțin.
Pentru energia din surse regenerabile dispozitive, vom începe prin utilizarea aceluiași simplu
definiție:
eficiența
= energie utilă furnizată ca o fracțiune din energia de intrare
Astfel:

Centrala hidroelectrică (energie electrică de la rețea/inițială energie potențială
de conducte de apă pe turbine rotor) ~90% (în aval de apă
trece cu foarte puțină energie, deoarece scade departe de turbină în
mari zone deschise).

Turbină de vânt în vânt moderat (energie electrică produsă/energia cinetică a
nerestricționat vânt pe rotor zona)
~45% (nu poate elimina toate cinetică
energie, deoarece aerul trebuie să continue să se mute departe în aval).

Incalzitor Solar de apa, la amiază, cer acoperit de nori, temperatura rezervorului inițial rece
(de căldură pentru apă caldă rezervor/primite expunere la soare)
~60%.

Solar cu panou fotovoltaic, la amiază, cer acoperit de nori
~17% (energie electrică gener-
creat/radiația solară intrare de toate lungimile de undă).

Sobă cu lemne și în camera de ardere-pipe
~85% (de căldură care trece la
camera/căldură de ardere de lemn uscat).
TWIDELL PAGINARE.indb 23
01/12/2014 11:35
24
Principii de energie din surse regenerabile

Arzător de biogaz
~90% (de căldură de la flacără/căldura de combustie de biode-
biodegradabile de intrare).

Biomasa centrala termica
~30% (producția de energie electrică/căldură de com-
de ardere a biomasei de intrare).
Dar sunt aceste valori de eficiență de mare importanță, deoarece intrările
de energie din surse regenerabile dispozitive ajunge, de obicei, fără costuri locale de
mediu sub formă de ploaie, vânt, soare, și a deșeurilor? Dacă vom plăti pentru intrare
(de exemplu, combustibili fosili), apoi dispozitivul de eficiență este foarte important, dar daca intrarile
sunt gratuite, este mai important să se evalueze producția efectivă de
dispozitive speciale la anumite site-uri. Mai mult decât atât, mediu intrări sunt
foarte variabile, astfel încât intrările sunt cele mai bune medie de peste timp, de obicei un an.
Avem nevoie de parametri care permit o producție anuală de un dispozitiv pentru a fi
evaluată, în special site-uri, pentru care termenul de factor de capacitate (Z) sau full
load ore (T
F
) este folosit.
(b) factor de Capacitate (Z) și sarcina maximă ore (T F )
Definițiile sunt după cum urmează:
capacitatea factor
=
energia livrată într-o perioadă specificată
(1.5)
energie livrabil la capacitate maximă
în acea perioadă
În mod normal, perioada specificată este de un an de 365 de zile
(365 d/y
× 24 h/d = 8760 h/an), astfel:
(anual) factor de capacitate
= Z =
de energie livrate pe an
energie livrabil la maxim
capacitate pe an
(1.6)
și, de asemenea,
încărcare completă ore
= T
F
=
de energie livrate pe an
capacitatea nominală
(1.7)
Prin urmare:
anual factor de capacitate
= sarcină maximă ore
365 h
(1.8)
Valoarea acestor parametri pentru un dispozitiv depinde atât la propria
eficiență și asupra climei de la site-ul. Prin urmare, de exemplu:

Hidroelectricitate, continuă de apă, 1% mentenantei
Z
~ 99% (T
F
~ 8700 h/y)

Hidroelectricitate, Scoția, cu 30% apă de disponibilitate
Z
~ 30% (T
F
~ 2600 h/y)
TWIDELL PAGINARE.indb 24
01/12/2014 11:35
§1.5 implicațiilor Tehnice
25

Turbine eoliene în Germania centrală (vânt moderat)
Z
~ 18% (T
F
~ 1600 h/y)

Turbine eoliene în Wellington, Noua Zeelandă ('windy city')
Z
~ 45% (T
F
~ 3900 h/y)

PV de urmărire panou solar în nordul Chile (aproape senin)
Z
~ 40% (T
F
~ 3500 h/an)

PV fix de orientare panou solar, centrul Angliei (de multe ori tulbure)
Z
~ 10% (T
F
~ 880 h/y)

Arderea biomasei pentru centrala termică Z
~ 90% (T
F
~ 7900 h/y)

Mareelor (baraj) putere Z
~ 25% (T
F
~ 2200 h/y)

Val de putere, viguros site-ul (potențialul) Z
~ 30% (T
F
~ 2600 h/y)

Tidal stream (curentul) de energie (potențială) Z
~ 20% (T
F
~ 1800 h/y)

Ocean de energie termică (OTEC potențial) Z
~ 90% (T
F
~ 7900 h/y)
Factorii de capacitate sunt cele mai frecvent discutate în cazul electrice
de generare de energie, dar conceptul poate fi aplicat pe scară mai largă. Rețineți
că dispozitivele solare pot captura doar soare în timpul zilei, așa că, dacă "an"
include pe timp de noapte, atunci Z(solar) este de cel mult 50%, chiar și pentru un perfect
aparat în medie, o zi de 12 ore.
Rețineți, de asemenea, că valorile lui Z și T
F

sunt atât de independente de capacitatea


de dispozitiv, astfel încât, dacă valorile lor pentru un anumit site și tehnologia sunt
considerate mici, a crescut de ieșire poate fi obținută numai prin
creșterea capacității de instalare. În plus, din moment comparație
cu capacitatea maxima a dispozitivului în sine, acești factori nu
se dau informații despre eficiența. O vor face, cu toate acestea, permite
diferite dispozitive de aceeași tehnologie pentru a fi comparate, fie prin tipul de
pe același site sau prin site-ul cu alte dispozitive similare.
Evident, producătorii și proprietarii de dispozitive încercați pentru a maximiza Z și T
F

cu valorile abordare teoretică maxima, dar de obicei există


limitări datorită tehnologiei specifice și site-ul și aplicația.
Tabelul 1.3 încercări de a rezuma acești factori. Tabelul D4 din Apendicele D
indică gama de Z găsit în întreaga lume pentru o gamă largă de tehnologii.
(c) potențialul De resurse
Potențial de resurse regenerabile dispozitiv este energia pe care o poate furniza
pe an. Resursa poate fi estimat la orice geografice la scară, de la
o gospodărie la toată lumea. Există două măsuri comune de
potențial de resurse într-o zonă geografică:
1
De potențial teoretic este derivat de la persoane fizice și climatice (fizic)
parametri (de exemplu totală a radiației solare primite de pe un continent suprafață).
Cele 'naturale curenții de energie' este prezentat în Fig. 1.2 sunt un exemplu la
scară globală. Este limita superioară a ceea ce poate fi produs dintr-o
resursă de energie bazate pe principii fizice și științifice actuale
TWIDELL PAGINARE.indb 25
01/12/2014 11:35
26
Principii de energie din surse regenerabile
cunoștințe. Acesta este punctul de plecare de la care se aplică restricții pentru
amplasarea, pierderile tehnice, de mediu, bariere, etc.
2
La potențialul tehnic este cantitatea de energie din surse regenerabile de ieșire
obținute prin punerea în aplicare integrală a demonstrat tehnologii
Tabelul 1.3
Factorii care influențează capacitatea de factori
Tehnologia
Principalele limitări naturale
Cel mai bun
valorile Z
Util
valorile Z
Comentarii
Incalzitor Solar de apa
Orientarea pe timp de noapte, nori,
încălzit temperatura apei și de utilizare
~ 40%
~ 10%
Doar preîncălzirea apei este util
Energia solară
Orientarea pe timp de noapte, nori,
mediu temperatura (rece cel mai bun), low
soare, umbrire
~ 40%
~ 10%
Hidroelectricitate
De alimentare cu apă și suma
variabilitatea, drop (cap) și lungimea de
conductă forțată
~ 95%
~ 15%
Pompat hidro
depozitare
Înălțimea și volumul de apă
magazin; pierderea prin frecare în conducte
~ 10%
~ 5%
Valoarea pentru rețeaua de energie electrică
este mai mare în perioadele de vârf
de aprovizionare
Turbine eoliene
Viteza medie a vântului, variabilitatea
locale de vânt, de caracteristicile amplasamentului
~ 40%
~ 20%
(A se vedea §7.3)
Biomasa
de ardere de căldură
(de exemplu, soba sau cazan)
Conținutul de apă din combustibil (ar trebui să fie uscat),
secundar de ardere emise de
gaze
~ 90%
~ 40%
Biomasa de aburi
cazan pentru energie electrică
Tipul și continuitatea aprovizionării,
bine-proiectat camera de ardere
~ 30%
~ 20%
cu deșeuri
combustibil
Principalele pierderi sunt intrinseci în
turbine cu abur sau motor
Biogaz căldură
Stabil de intrare de material pentru a
digestor anaerob
~ 90%
~ 50%
Foarte mică pierdere de energie
în digestor (§10.7.2)
Val de putere
Continuitatea de echilibru valuri
~ 30%
~ 10%
[Real experimentat valori
este necesar]
Maree baraj putere
Naturale periodicitatea mareelor,
tidal gama de la site-ul, eficiența turbinei
~ 25%
~ 15%
Ieșire legate de maree
periodicitatea, baraj permite
unele modificări în calendarul
de aprovizionare
Tidal stream putere
Naturale periodicitatea mareelor, vârf de maree
flux de viteza, eficiența turbinei în
open flow (cf. turbine eoliene)
~ 20%
~ 10% de Ieșire este variabil în timp, dar
previzibil (§12.2)
Ocean termică
de conversie în energie
electrică
Mică schimbare de temperatură
între suprafața mării și apă adâncă;
bio-depozite asperități conducte
~ 95%
~ 80%
Foarte mici de căldură–motor
eficiența de conversie a
energiei electrice și mai mult de pompare
atât de scump, continuă
operațiunea este posibil, în
principiu, la o capacitate instalată;
experiență minimă
Energie electrică geotermală Temperatura și presiunea
emise subterane de apă/abur
~ 90%
~ 50%
Motor termic limitări,
funcționarea continuă
este posibil, în principiu,
TWIDELL PAGINARE.indb 26
01/12/2014 11:35
§1.7 implicații Sociale
27
sau practici în regiunea determinată. Nu se face referire explicită la
costuri, politici instituționale, sau alte bariere de om care ar putea limita
de tehnologie. Din potențialul tehnic, mai practic
estimările pot fi făcute pentru a permite alte constrângeri (de exemplu, evitarea
site-uri științifice/valoare ecologică, prioritizarea biomasă pentru uz alimentar
înainte de utilizarea acesteia pentru energie, aplicarea limitele de cheltuieli, etc.).
Vezi Verbruggen et al. (2014) pentru o discuție detaliată a acestor și alte
concepte conexe.
Estimările potențialului tehnic sunt prezentate în capitolele următoare ale acestei cărți.
Numeroase bariere de om sunt discutate în Capitolul 17.
O serie de alți indicatori sunt discutate în Capitolul 15 și prezentate în
Anexa D.
§1.6 STANDARDELE ȘI REGLEMENTĂRILE
Dezvoltarea energiilor regenerabile și echipamente sunt importante aspectele legate de
afaceri și de economii, care, la fel ca atât de mult altceva, beneficiază de
care au acceptat standardele naționale și internaționale și reglementările în vigoare.
Finanțatori (de exemplu, bănci) și asigurătorii solicită ca toate echipamentele îndeplinesc
standardele naționale și internaționale. De exemplu, siguranța este întotdeauna un prim
motiv de îngrijorare, astfel încât există multe cerințe asociate cu proiectarea și
construcția de echipamente de energie regenerabilă (de exemplu, turbine eoliene de frânare
și izolarea electrică a modulelor fotovoltaice). De siguranță și de alte
reglementări guvernamentale fac parte din cadrul instituțional pentru energie
sisteme, care este discutat în Capitolul 17.
IEC (International Electrochimice Comisia) este
organismul internațional care supraveghează multe standarde în toate disciplinele; are o
secțiune specială pentru energia din surse regenerabile (a se vedea
<http://www.iec.ch/renewables/
>), dar, desigur, mai multe standarde comune pentru o gamă mai largă de
tehnologii.
§1.7 IMPLICAȚII SOCIALE
Revoluția Industrială în Europa și America de Nord, precum și
dezvoltarea industrială în toate țările, au afectat profund sociale,
structuri și modele de viață. Influența surselor de energie este o
funcție de conducere pentru o astfel de schimbare. De exemplu, există un istoric al
relației dintre cărbunelui și dezvoltarea industrializate
țări. Norvegia și alte țări similare au fost mult influențate
de energia hidroelectrică. Danemarca și-a găsit o industrie majoră în turbine eoliene de
fabricație. În non-industrializate țări, relativ ieftin ulei de
provizii au devenit disponibile în anii 1950, în același timp, cât mai multe țări
a obținut independența față de colonialism, astfel încât furnizarea de energie pentru lor
de dezvoltare. Astfel, în toate țările, în producerea de energie și utilizarea acesteia au
TWIDELL PAGINARE.indb 27
01/12/2014 11:35
28
Principii de energie din surse regenerabile
a dus la schimbări profunde în bogăție și stilul de viață. La nevoie pentru a asigura
aprovizionarea cu energie este evident, și provizii dintr-o țară cu resurse proprii
de sprijin, cum ar securitate, în special din surse regenerabile de energie tehnologii
aplicabile în fiecare țară.
§1.7.1 Dispersate de viață
În §1.1 și §1.4.3 dispersate și consum redus de energie densitatea de flux de
surse regenerabile a fost discutat. Regenerabile de energie ajunge dispersate în
mediu și este dificil și costisitor să se concentreze. Prin contrast,
finit surse de energie sunt magazine de energie, care sunt ușor de concentrat la
sursă și costisitoare pentru a dispersa. Astfel, rețelele electrice de distribuție de
combustibili fosili și surse nucleare au avut tendința de a radia din europa centrală,
intensiv puncte de distribuție, de obicei, cu ~1000 MW de capacitate. Industria a
dezvoltat pe aceste rețele, cu industria grea cel mai aproape de puncte de
intensiv de aprovizionare. Populațiile au crescut ca răspuns la
ocuparea forței de muncă oportunități de comerț și industrie. Efecte similare
au avut loc cu relațiile dintre miniere de cărbune și oțel
de producție, de rafinare a petrolului și inginerie chimică, și disponibilitatea de gaz
consumabile și urbane complexe.
Acest examinarea fizică a efectului de primar densitatea de flux de energie
surse sugerează că aplicarea pe scară largă a surselor regenerabile de energie
va favoriza dispersate, mai degrabă decât de concentrate, comunități. Link-uri cu
agricultura sunt susceptibile de a fi important. Rețelele de energie electrică în astfel de situații
poate avea intrare la scară mai mică, încorporate generație (de exemplu,
'microgenerare') și la scară mai mare evoluțiile comerciale de eoliene si solare,
ferme, de generație din biomasă și deșeuri, și de energie marină
tehnologie. Pe astfel de grile, fluxurile de energie variabil în ambele direcții,
în conformitate cu locale de generare și de cererea locală. Unele surse regenerabile de energie
surse, în special energia solară, sunt potrivite pentru microgenerare atât în mediul urban cât și
mediul rural. Alții (de exemplu, biomasa) se bazează pe fluxurile de energie care sunt în general
mai accesibile în zonele rurale. Regiunile de lângă marea avea, în practică,
multe oportunități pentru generarea de energie (de exemplu, valurile, mareele, și
fermele eoliene offshore).
Cu toate acestea, mai mult de jumătate din populația lumii este acum trăiesc în
zonele urbane (inclusiv cu cel puțin 40% din populațiile din Africa și Asia,
care erau încă în mare măsură rurală, 30 de ani în urmă), și până în prezent, acest procent
este în continuă creștere. Moderne de energie din surse regenerabile tehnologia poate servi
orașele în care cei mai mulți oameni trăiesc acum, nu numai prin
microgenerare și rețelele inteligente (§15.4.3), dar, de asemenea, prin intermediul scară largă valorificarea
de hidroenergie, energie eoliană, și bioenergie la site-uri unde cei energie
fluxurile sunt abundente și cu mijloace moderne de transmitere a energiei, așa cum
sunt prezentate în Capitolul 15. Astfel, RE va fi important pentru viitorul populației,
atât în mediul urban cât și rural.
TWIDELL PAGINARE.indb 28
01/12/2014 11:35
§1.7 implicații Sociale
29
§1.7.2 Poluare și impactul asupra mediului
Emisiile nocive pot fi clasificate ca substanțe chimice (ca de combustibili fosili și
energie nucleară plante), fizice (inclusiv zgomot acustic și radioactivitate),
și biologice (inclusiv agenți patogeni).
O astfel de poluare de la sisteme de energie este extrem de rezultatul
folosind 'maro' combustibili, atât fosili și energia nucleară. Acest lucru se aplică în special
a gazelor cu efect de seră (GES), care sunt o cauza majora de
potențial periculoase schimbările climatice. După cum a subliniat în §1.2.1 și
discutate mai pe larg în Capitolul 17, reducerea emisiilor de GES este una dintre
principalele forțe motrice în creștere a cererii pentru sursele regenerabile de energie
tehnologii.
Energia regenerabilă este întotdeauna extrase din fluxurile de energie
deja compatibile cu mediul (Fig. 1.1). Energia este apoi
a revenit la mediu, astfel încât nici o poluare termică poate apărea pe
nimic, dar o scară mică. De asemenea, materialul și poluarea chimică în
aer (și, în special, a emisiilor de GES), apă, și refuză tind să fie
minim. O excepție este poluarea aerului din incompletă
ardere de biomasă sau refuză (vezi Capitolul 10). Poluarea mediului
apare în cazul maro energie este folosit pentru materiale și
producerea de energie din surse regenerabile dispozitive, dar acest lucru este de mici pe durata de viață
a echipamentului și va scădea proporțional cu adoptarea de
surse regenerabile de energie.
Majoritatea tehnologiilor regenerabile produce în mod semnificativ mai puțin
convenționale de aer și apă poluante decât combustibilii fosili, dar
cu toate acestea, impactul asupra mediului fiind amplasate în zone mari
de teren, ca, de exemplu, rezervor hidroelectrice (care poate, de asemenea, eliberarea
de metan din vegetația subacvatică) și biocombustibili. Unele surse de energie regenerabile,
în special energie eoliană, nu întrerupeți utilizarea regulată a terenurilor pentru
agricultură sau de recreere. În schimb, combustibili fosili miniere (în special de cărbune
și uraniu) are efecte negative asupra terenului din jur și
utilizarea acestuia.
Acolo poate fi, de asemenea, un impact asupra resurselor de apă. De exemplu,
limitează disponibilitatea de apă pentru răcirea centralelor termice scade
eficiența lor, care pot afecta plantele care funcționează pe cărbune, biomasă, gaz,
energie nucleară, și de concentrare a energiei solare. Nu au fost semnificative
de putere reduceri la energia nucleară și cărbune plante în timpul periodic de secete
în statele UNITE ale americii și Franța. Cu toate acestea, producția de energie electrică din eoliene și
solare FOTOVOLTAICE, necesită foarte puțină apă în comparație cu conversie termică
tehnologii, și nu are impact asupra calității apei.
Impactul asupra mediului a unui sistem de energie regenerabila depinde
în special de tehnologie și de circumstanțe. Considerăm aceste aspecte
în secțiunea finală a fiecărei tehnologii capitol. General instituțional
factori, de multe ori legate de reducerea poluării, sunt luate în considerare în
Capitolul 17.
TWIDELL PAGINARE.indb 29
01/12/2014 11:35
30
Principii de energie din surse regenerabile
§1.7.3 viitor
Știm că multe schimbări în modelele sociale sunt legate de energie
provizii. Ne putem aștepta la modificări suplimentare pentru a apărea ca surse regenerabile de energie
sisteme deveni și mai larg răspândită. Influența moderne
știință și tehnologie asigură că există numeroase
îmbunătățiri tehnologii mai vechi, și, ulterior, standardele de viață poate fi
de așteptat să crească, în special în mediul rural și, anterior, regiunile mai puțin dezvoltate.
Este imposibil de prezis exact efectul pe termen lung de astfel de modificări
în aprovizionarea cu energie, dar natura durabilă de energie din surse regenerabile ar trebui să
producă o mai mare stabilitatea socio-economică decât a fost cazul cu
combustibilii fosili și energia nucleară. În special, ne așteptăm la o mare diversitate
de surse regenerabile de energie pentru a fi asociate cu o varietate similară în
local, caracteristici economice și sociale. Noi cu siguranță de acord cu una din
concluziile majore ale IPCC (2011), și anume:
Există puține, dacă este cazul, limitele tehnice planificate integrarea
de tehnologii de energie regenerabilă în întreaga gamă foarte largă de
prezenta sistemelor de alimentare cu energie la nivel mondial, deși alte bariere
[de exemplu, economice și instituționale] pot exista.
Perspectivele de viitor pentru energia din surse regenerabile sunt discutate în continuare în
secțiunea de concluzii (§17.8) din această carte.
REZUMAT CAPITOL
Energia regenerabilă este energia obținută de la persoane fizice și persistente fluxurilor de energie care au loc în
mediul imediat. Exemple de astfel de fluxuri de energie includ radiația solară, vânt, apă care se încadrează,
biomasă, și mareele oceanice.
Dezvoltarea durabilă înseamnă viață, producătoare și consumatoare într-o manieră care satisface nevoile
prezentului fără a compromite capacitatea generațiilor viitoare de a-și satisface propriile nevoi. Un important
amenințare la adresa dezvoltării durabile este de schimbările climatice cauzate de gazele cu efect de seră emise de combustibili
fosili
combustibili. Aceasta și caracterul finit al fosili și de combustibil nuclear, de materiale, este indispensabil să se extindă
regenerabile în
aprovizionarea cu energie și de a utiliza energia în mod mai eficient.
Comparație de energie necesară pentru fiecare persoană cu naturale fluxurile de energie de la Soare și
alte surse sugerează că furnizare de energie regenerabilă poate oferi un standard satisfăcător
de trai pentru toți, dar numai dacă există metode de a extrage, a folosi, și de a stoca energie în mod satisfăcător la realistă
a costurilor.
Eșecul de a înțelege distinctiv principii științifice pentru valorificarea surselor regenerabile de energie va fi aproape
cu siguranță duce la o slabă inginerie și funcționarea neeconomică. De alimentare cu energie nu trebuie să fie considerate
separat de utilizarea finală. Managementul energiei este importantă pentru a îmbunătăți eficiența și a reduce
pierderile economice. Eficiența cu finite, combustibili reduce poluarea; eficiența cu surse regenerabile de energie reduce capital
cheltuieli de judecată. Cu sisteme de energie regenerabilă, nu numai consumatorilor finali varia incontrolabil cu timpul
dar astfel nu prea mult de aprovizionare natural în mediu. Regenerabile de energie frecvent ajunge la
aproximativ 1 kW/m
2
, întrucât finite surse centralizate au mult mai mare de energie densități de flux; prin urmare,
sursele regenerabile de generare și furnizare va fi răspândit peste dispersate domenii și situații.
TWIDELL PAGINARE.indb 30
01/12/2014 11:35
Probleme
31
ÎNTREBĂRI RAPIDE
Notă: Răspunsurile sunt în textul secțiunile relevante din acest capitol, sau
poate fi ușor dedus din ele.
1
Având în vedere resursele primare, ceea ce distinge regenerabile de energie
din combustibili nucleari și fosili?
2
Altele decât prețul, ce alți factori influențează acceptarea unei
aprovizionare cu energie?
3
Compara pe cap de locuitor consumul de energie din propria țară
cu două țări din alte continente.
4
Numele cinci independent în cele din urmă, sursele primare de energie.
5
Compara consumul de energie pe unitate de utile lumina
incandescent, fluorescent, lumina diodice de lumina.
6
Explica termodinamic "calitate" de o aprovizionare cu energie și cum
acest lucru afectează utilizarea acestuia.
7
Ce este "inteligent" tehnologia și cum poate beneficia de absorbție și utilizare
de energie din surse regenerabile?
8
Ce este factor de capacitate și cum nu se referă la sarcină completă de ore pe
an?
9
Ce este securitatea energetică'? Compara asta pentru combustibilii fosili, nucleari
de putere și energie regenerabilă.
10
Compara impactul asupra mediului (inclusiv zgomotul și poluarea) de
producere a energiei din combustibili fosili, energie nucleară și sursele regenerabile de
energie.
PROBLEME
1.1
(a) Arată că în medie de radiație solară absorbită în timpul 24
ore peste toată suprafața Pământului este de aproximativ 230 de Wm
-2
(a se vedea Fig. 1.2).
Pentru persoanele fizice, moderne tehnologii de energie regenerabilă a încuraja auto-generarea de energie locală și
sisteme (microgenerare). Moderne de energie din surse regenerabile tehnologia poate servi nu numai în zonele rurale, dar
, de asemenea, orașele în care cei mai mulți oameni trăiesc acum, prin microgenerare și rețelele inteligente. Scară mai largă
valorificarea de hidroenergie, energie eoliană, și bioenergie la site-uri în cazul în care aceste fluxuri de energie sunt abundente
utilizeaza mijloace moderne de transmitere și furnizarea de energie urbane complexe și mai mare industrie.
Precedent istoric sugerează că o creștere majoră a surselor regenerabile de energie va influența structurilor sociale și
a economiilor naționale.
Primul pas în proiectarea de o sursă de energie regenerabilă este rapidă evaluare din care surse regenerabile
sunt în cantități suficiente pentru a justifica mai detaliate de monitorizare. Datorită variațiilor sezoniere în cele mai multe RE
curge, bun design de inginerie necesită o monitorizare de resurse pentru cel puțin un an de la site-ul în
cauză.
TWIDELL PAGINARE.indb 31
01/12/2014 11:35
32
Principii de energie din surse regenerabile
(b) Utilizând dispozitive media locală putere accesibile pot fi
a crescut, de exemplu prin înclinarea dispozitive solare spre Soare și de
interceptarea vânturi. Este rezonabil să se afirme că 'fiecare metru
pătrat de Pământ este locuibilă suprafața este traversat sau accesibile
la un flux mediu de aproximativ 500W'?
1.2
Compara costurile directe pentru consumator de a utiliza:
(a) o succesiune de zece 100 W becurile cu incandescență cu o
eficiența energiei electrice, pentru a luminii vizibile de 5%, viață de 1000
ore, pret 0,5 Euro;
(b) o lampă fluorescentă compactă (CFL) a da același illuminati-
tion la 22% eficienta, viata de 10.000 de ore, prețul 3.0 Euro: utilizați
un fix de energie electrică prețul de 0.10 E/kWh;
(c) Calcula timpul de amortizare în ore de iluminat
(b) împotriva (a). (A se vedea, de asemenea, Problema 17.1 care permite pentru
mai sofisticate costuri reduse.)
1.3
Calculul se repetă de problema 1.2, cu prețurile locale de
lămpi și de energie electrică. Atât prețul de Cfl-uri în magazine locale și de
energie electrică variază semnificativ, astfel încât răspunsurile dumneavoastră pot să difere în mod
semnificativ.
Cu toate acestea, este foarte probabil din cauza faptului că cea mai mare durată de viață de economii
va avea loc în continuare.
Următoarele Probleme, marcate cu * sunt deosebit de potrivite pentru clasa de
discuție:
*1.4
Economiștii susțin că și prestările de rezerve de petrol să crească mai puțin,
prețul va merge în sus, astfel încât cererea scade și anterior neeconomică
consumabile va intra în producție. Acest lucru tinde să facă
resurse dura mai mult decât ar fi sugerat printr-un simplu
calcul (bazate pe azi rezervele divizate de azi e folosesc) . Pe
de altă parte, cererea crește condiționată de creșterea economică
de dezvoltare în țările în curs de dezvoltare au tendința de a scurta durata de viață de
rezervă. Discuta.
*1.5
Este stilul tău de viață sustenabil? Dacă nu, ce schimbări ar face
așa?
*1.6
Ne putem aștepta furnizare de energie regenerabilă să fie universal
aplicabile? Clarifica-ti raspunsul prin a-mi explica care sursele regenerabile de energie sunt
cele mai aplicabile în zona de domiciliu.
*1.7
Prezice aprovizionarea cu energie în 30 de ani pentru regiunea în care
locuiți și să explice de ce modificări ar putea avea loc.
TWIDELL PAGINARE.indb 32
01/12/2014 11:35
Bibliografie
33
NOTE
1
Cititorii care se confruntă ocazional un termen tehnic necunoscut pentru ei, care nu este explicat în
textul înconjurător sunt sfătuiți să consulte Indexul pentru îndrumări suplimentare.
2
Deși producția de energie' este industria de-standard de cuvânt, acesta este un drept fundamental principiu fizic care energia
poate fi transformată dintr-o formă în alta, nu a "produs".
3
Eficiența și eficacitatea dispozitivelor de iluminare sunt complexe, cu diverse definiții; stadion adimensional cifrele sunt
prezentate aici; pentru mai multe detalii, a se vedea http://en.wikipedia.org/wiki/Luminous_efficacy.
BIBLIOGRAFIE
Se referă la bibliografie la sfârșitul fiecărui capitol pentru anumite subiecte.
Sondaje de energie din surse regenerabile tehnologia și resursele
Boyle, G. (ed.) (2012, 3rd edn) Energie din surse Regenerabile: Energie pentru un viitor durabil, Oxford University Press,
Oxford. Introducere excelent pentru ambele științifice și non-științifice cititori.
IPCC (2011) O. Edenhofer, R. Pichs-Madruga, Y. Sokona, K. Seyboth, P. Matschoss, S. Kadner, T. Zwickel, P.
Eickemeier, G. Hansen, S. Schlömer și C. von Stechow (eds), IPCC Raport Special privind Energia din surse Regenerabile
Surse și de Atenuare a schimbărilor Climatice, Cambridge University Press, Cambridge. (Text integral disponibil la http://
srren.ipcc-wg3.de/report.) Autoritate revizuire a stat de arta tehnologiilor individuale și de curent
de utilizare și perspectivele de viitor pentru RE. Pe principii, a se vedea în special ch. 1. Include, de asemenea, un util Glosar.
Kishore, V. V. N (ed.) (2009) surse Regenerabile de Energie Inginerie și Practică, Earthscan, Abingdon. Un alt
multiauthor tome (
~900 de pagini); nivel mai avansat decât această carte, ca 'vizează, în ansamblu'.
Kreith, F. și Goswami, D. Y. (eds) (2007) Manual de Eficiență energetică și Energie Regenerabilă, CRC Press,
Londra. Ușa-oprirea multi-autor (
>1000 de pagini!), cuprinzătoare și detaliate, dar încă ușor de citit,
deși, uneori, cu NOI accent.
Sørensen, B. (2011, 4 edn) surse Regenerabile de Energie, Academic Press, London. Deosebit de cel mai bun teoretice
text la nivel postuniversitar, având în vedere energia de la mediu la utilizarea finală.
Tester, J. W., Drake, E., Driscoll, M., Golay, M. și Peters, W. (2012, 2nd edn) Energia Durabilă: Alegerea
între opțiuni, MIT Press, Cambridge, MA. Ample de text, inclusiv capitole nu doar pe individ RE
tehnologii, dar, de asemenea, combustibilii fosili și energia nucleară, și global de mediu, economie și sisteme de energie.
Energie, societate, mediu (inclusiv de "dezvoltare durabilă")
A se vedea, de asemenea, bibliografie pentru Capitolul 17.
Cassedy, E. S. și Grossman, P. G. (2002, 2nd edn) Introducerea de Energie: Resurse, tehnologie și societate,
Cambridge University Press, Cambridge. Bun contul non-tehnic pentru știință și societate' cursuri.
Elliott, D. (ed.) (2010) pentru Energie Durabilă: Oportunități și limitări, Palgrave Macmillan, Basingstoke. Scurt
sondaj de tehnologii, dar mai ample discuții instituționale și societale aspecte; marea BRITANIE se concentreze.
Everett, R., Boyle, G., Peake, S. și Ramage, J. (eds) (2011, 2nd edn) Sisteme de Energie și Sustenabilitate: Putere
pentru un viitor durabil, Oxford University Press, Oxford. Bun contul non-tehnic pentru știință și societate'
cursuri.
TWIDELL PAGINARE.indb 33
01/12/2014 11:35
34
Principii de energie din surse regenerabile
Goldemberg, J. și Lucon, J. (2009, 2nd edn) Energie, Mediu și Dezvoltare, Routledge, Londra.
Ample și ușor de citit expunerea de link-uri între energie și dezvoltarea economică și socială și
sustenabilitate, cu luarea în considerare a capitalului propriu în cadrul și între țările de Brazilian experți.
Houghton, J. T. (2009, 4 edn) Încălzirea Globală: La informare completă, Cambridge University Press, Cambridge.
Simplu și didactic. Armonizează cu oficial Rapoartele IPCC (Houghton a fost președinte al IPCC).
Grupul interguvernamental privind schimbările Climatice (IPCC) al Cincilea Raport de Evaluare (2014). A se vedea, de
asemenea, IPCC de-al Patrulea
Raport de Evaluare (2007), în special Rezumat pentru factorii de Decizie Politică: raport de Sinteză'. Vezi IPCC-ul
enumerate mai jos; raportul complet este în trei mari volume, publicată de Cambridge University Press.
McNeill, J. R. (2000) Ceva Nou sub Soare: Un mediu de istoria secolului al xx-lea, Penguin,
London. Creșterea de combustibili fosili-a tras orașe și impactul acestora asupra apei, aerului, și biosferei.
Von Weizsacker, E., Lovins, A. B. și Lovins, H. (2000) Factor Patru: Dublarea avere, înjumătățirea utilizării resurselor,
Penguin, London. Explorează mai largă de probleme sociale și politice de aprovizionare cu energie, în special cele asociate
cu regenerabilă și nucleară provizii.
Comisia mondială pentru Mediu și Dezvoltare (1987) Viitorul Nostru Comun, Oxford University Press,
Oxford (la 'Raportul Bruntland'). O lucrare, de avertizare cu privire la probleme-cheie într-un limbaj simplu pentru politicieni.
Publicații oficiale (inclusiv statisticile în domeniul energiei și proiecții)
Deasemenea vezi mai jos sub reviste și site-uri web, cât mai multe publicații oficiale, în special cele de natură statistică,
sunt actualizate în fiecare an sau doi.
Agențiile organizației Națiunilor unite a produce o gamă largă de publicații esențiale în ceea ce privește energia, aproape toate
din care sunt
liber downloadable. Acestea sunt deosebit de importante pentru date. De exemplu, vă recomandăm următoarele:
Națiunilor unite Lume Energie Statistică Anuarul, anual. Oferă statistici privind consumul de energie în jurul valorii de
lume, clasificate în funcție de sursă, țară, continent, etc., dar contează doar comerciale de energie (de ex exclude lemne de foc,
etc.). On-line (cu alte date de energie) la http://unstats.un.org/unsd/energy/.
Guvernul publicații sunt întotdeauna importante; a se vedea de exemplu, marea BRITANIE Departamentul de Energie Seria de
Energie Ziare.
Aceste publicații sunt de obicei scrise în mod clar și să includă factorii economici, la momentul de scris. Principiile de bază
sunt acoperite, dar, de obicei, fără detaliile necesare pentru un studiu serios. (Actualizări anuale de multe guvernului și
ONU publicații sunt, de asemenea, disponibile prin intermediul site-urile corespunzătoare.)
Consiliul mondial al Energiei (2001) Studiu ale Lumii de Resursele Energetice. Compilate la fiecare cinci ani de către cnr-CME,
care cuprinde, în principal, energie, companii de utilități din întreaga lume; acoperă atât din surse regenerabile și
resurse neregenerabile.
Agenția internațională a Energiei, World Energy Outlook (anual), Paris. Accentul este pus pe resursele de combustibili fosili și de
a folosi,
bazat pe previziuni detaliate pentru fiecare țară membră, precum și pentru cei non-membru țări care sunt
semnificative în lumea piețelor de energie (de exemplu, OPEC și China). AIE Energie statistici sunt disponibile on-line la www.
iea.org/statistics/.
Do-it-yourself publicații
Există multe publicații disponibile pentru publicul larg și entuziaști, cea mai mare parte axat pe un anumit RE
tehnologie. Nu ignora aceste, dar ai grijă dacă sarcinile sunt făcute să pară ușor. Multe dintre aceste publicații da
stimularea idei și sunt atractive pentru a citi.
TWIDELL PAGINARE.indb 34
01/12/2014 11:35
Bibliografie
35
De referință specifice
Verbruggen, A., Fischedick, M., Herlong, W., Weir, T., Nadai, A., Nilsson, L. A., Nyboer, J. și Sathaye, J. (2010)
'Regenerabile de energie costurile potențiale bariere: aspecte Conceptuale', Politica Energetică, 38, 850-861 (februarie), DOI:
10.1016/j.enpol.2009.10.036.
Reviste, comerț indici, și site-uri web
Regenerabile de energie și, în general, tehnologia energetică și politica sunt continuu avansează. Serios de
studiu este necesar să se adreseze din timp în timp pentru periodice literatura de specialitate (ziare si reviste). Site-uri de
organizațiile-cheie, cum ar fi cele enumerate mai jos, transporta, de asemenea, actualizări de unele referințe cheie, în special cele
de natură statistică.
Vă îndemnăm cititorii pentru a scana grave științifice și de inginerie reviste (de exemplu, New Scientist, Revizuirea Anuală
de Energie și Mediu) și reviste (de exemplu Electrice de Revizuire, Moderne Sisteme de Energie, surse Regenerabile de
Energie Lume). Aceste publicații acoperă în mod regulat proiecte de energie regenerabilă în rândul lor generale art. În
revista din surse Regenerabile de Energie se Concentreze, publicat de International Solar Energy Society, poartă numeroase
articole bine ilustrate pe toate aspectele legate de energia din surse regenerabile. Seria Progreselor în domeniul Energiei
Solare publicat de
American Solar Energy Society, cuprinde volumele anuale de nivel înalt, recenzii, inclusiv toate solare
tehnologii și unele solar derivate tehnologii (de exemplu, energie eoliană și biomasă). Există, de asemenea, multe de specialitate
și jurnale academice, cum ar fi Energia Regenerabilă și Durabilă Recenzii, Energie Solară, Eoliană Inginerie,
Regenerabile de Energie, și de Biomasă și Bioenergie, menționate la capitolele relevante.
Ca surse regenerabile de energie a dezvoltat din punct de vedere comercial, mai mulți indici de companii și produse au fost
produse; cele mai multe sunt actualizate anual (de exemplu Europene Director de furnizare de Energie Regenerabilă și Servicii,
anual,
ed. B. Cruce, James și James, Londra).
www.iea.org
Agenția Internațională pentru Energie (IEA) cuprinde guvernele din aproximativ 20 de țări industrializate; sa
publicații acoperă politici, statisticile în domeniul energiei, și tendințele, și într-o mai mică măsură tehnologii; coordonează, de
asemenea,
și publică mult de colaborare internațională, R&D, inclusiv scris clar aprecieri de stat de arta de
numeroase tehnologii de energie regenerabilă. Publicațiile sale atragă asupra detaliate intrări din țările membre.
www.irena.org
Agenția Internațională pentru energii Regenerabile a fost înființată în 2009 ca o agenție interguvernamentală pentru a promova
energia regenerabilă. Produce multe rapoarte utile.
www.worldenergy.org
Consiliul Mondial al Energiei cuprinde, în principal, energie, companii de utilități din întreaga lume, care cooperează pentru a
produce studii și proiecții de resurse, tehnologii, și prețurile.
www.ipcc.ch
Grupul Interguvernamental privind schimbările Climatice (IPCC) este un panou de 2000 de oameni de știință convocate de
organizația Națiunilor Unite să raporteze cu privire la știință, economie, și de diminuare a gazelor cu efect de seră și schimbările
climatice;
rapoartele lor, emis la fiecare cinci ani sau cam asa ceva, sunt considerate ca fiind de autoritate. Rezumatele sunt disponibile pe
site-ul.
www.practicalaction.org
Cunoscut anterior ca ITDG, Intermediar de Dezvoltare de Tehnologie de Grup. Dezvoltă și promovează simple și
ieftine, dar eficiente tehnologii, inclusiv tehnologii de energie regenerabilă – pentru utilizarea în zonele rurale de dezvoltare
de țări. Ei au o vasta lista de publicații plus online tehnice boxeri'.
TWIDELL PAGINARE.indb 35
01/12/2014 11:35
36
Principii de energie din surse regenerabile
www.ewea.org
Asociația Europeană pentru Energie Eoliană este una din mai multe surse regenerabile de energie asociații, toate din care au utile
site-uri web. Cele mai multe astfel de asociații sunt asociații comerciale', ca finanțate de către membrii în nume din surse
regenerabile
de energie a industriei. Cu toate acestea, ei sunt conștienți de public și de interes educațional, și așa au informații și
dea conexiuni pentru informații de specialitate.
www.ren21.net
În domeniul Energiei din surse Regenerabile în rețea. A se vedea în special lor anual Global Status Report.
TWIDELL PAGINARE.indb 36
01/12/2014 11:35

Radiația solară și efectul de seră


CAPITOLUL

2
CONȚINUTUL
Obiective de studiu
38
§2.1 Introducere
39
§2.2 Extra-terestre radiația solară
40
§2.3 Componentele de radiații
41
§2.4 Geometria Pământ și Soare
42
§2.4.1 Definiții
42
§2.4.2 Latitudine, anotimp, și de zi cu zi
expunere la soare
44
§2.5 Geometria colector
și solar beam
46
§2.5.1 Definiții
46
§2.5.2 Unghiul dintre raza și
colector
47
§2.5.3 orientare Optimă de
un colector
48
§2.5.4 variație Orară a radiației
49
§2.6 transport Atmosferic,
absorbția și reflexia
49
§2.6.1 Reflecție
51
§2.6.2 de Aer–raportul de masă
52
§2.6.3 de Nori temperatura
56
§2.6.4 spectrul Solar a primit de la
suprafața Pământului
56
§2.7 Măsurarea radiației solare
57
§2.8-Ul de estimare a radiației solare
57
§2.8.1 Cerințele
57
§2.8.2 variație Statistic
58
§2.8.3 ore de Soare ca o măsură
de expunere la soare
59
§2.8.4 Geostaționară Operaționale
Mediu
Sateliți (MERGE)
59
§2.8.5 Focusable fascicul de radiații și
Claritatea Index
60
§2.8.6 Efect de colector de înclinare
60
§2.9 efectul de Seră și schimbările climatice
schimba
62
§2.9.1 Radiativ balanța de
Pământul
62
§2.9.2 efectul De seră,
radiativ forțând, iar clima
se schimba
63
§2.9.3 schimbările Climatice: observații
64
§2.9.4 schimbările Climatice: proiecții,
impactul, și de atenuare
67
Rezumat capitol
68
Întrebări rapide
68
Probleme
69
Note
72
Bibliografie
72
Caseta 2.1 Radiații transmise, absorbit
și împrăștiate de Pământ e
atmosfera
55
Caseta 2.2 Unități de concentrație a gazului
65
Caseta 2.3 de Ce știm că recent
crește în CO
2
și în
temperatura se datorează omului
de activitate (antropice)
66
TWIDELL PAGINARE.indb 37
01/12/2014 11:35

www.shahrsazionline.com
38
Radiația solară și efectul de seră
OBIECTIVE DE STUDIU

Apreciez radiația solară are efect asupra
temperaturii Pământului.

Schiță spectrul solar la sursă și la
suprafața Pământului.

Identificarea proceselor cheie din radiația solară
absorbția în atmosferă, și cum acest lucru
implică două spectrale "ferestre" în
atmosfera Pământului.

Schiță de bază și îmbunătățite cu efect de seră
efecte.

Numele metodele de măsurare și
instrumentație pentru radiația solară.

Estimarea radiația solară (Wm
-2
) și de zi cu zi
expunere la soare (MJm
-2
zi
-1
) la orice locație și
sezonul.
LISTA DE FIGURI
2.1
Distribuția spectrală a extra-terestre radiația solară, G*
0
λ
. 40
2.2
Originea fascicul directă și difuză a radiației.
41
2.3
Tehnici pentru a măsura diferitele componente ale radiației solare.
42
2.4
Definiția schiță pentru latitudine
f și longitudine y (vezi textul pentru detalii).
42
2.5
Mișcarea de revoluție a Pământului în jurul Soarelui.
43
2.6
Pământul, văzut dintr-un punct în continuare de-a lungul orbitei sale.
44
2.7
Variația cu sezon de latitudine și de Sec
h
, energia solară (de zi cu zi insolație) a primit pe orizontală
avionul de pe o zi clar.
45
2.8
Secțiuni transversale prin Pământ la amiază solare.
46
2.9
Zenith unghi, unghiul de incidență, panta și unghi de azimut pentru o suprafață înclinată.
47
2.10 (a) Radiația pe o suprafață orizontală (b) variație Tipică de radiația pe o suprafață orizontală pentru
o zi de variabilă cloud.
50
2.11 Efecte produce ca extra-terestre radiatia solara trece prin atmosfera Pământului.
51
2.12 Estimare a Pământului anual și global al bilanțului energetic.
52
2.13 Radiații transmise și absorbită de către atmosferă în funcție de lungimea de undă.
54
2.14 Aer–raportul de masă m
= sec
θ
z
. 54
2.15 distribuții Spectrale de radiație solară.
56
2.16 Fotografii de diferite solare instrumente.
58
2.17 Fracțiune de iradierea difuză a complotat împotriva Claritate Index pentru o gamă largă de detalii
date de teren.
61
2.18 Variația estimată medie zilnică de expunere la soare pe o suprafață la diverse pante.
62
2.19 Observații de gaze cu efect de Seră și efectul lor fizice.
66
TWIDELL PAGINARE.indb 38
01/12/2014 11:35
§2.1 Introducere
39
§2.1 INTRODUCERE
Acest capitol explică modul în care radiația solară link-uri Pământului cu Soarele
și cum atmosfera Pământului controlează acest flux de energie. Mai târziu
capitole arată cum a primit radiații solare (uneori denumită insolație)
puterile regenerabile de energie dispozitive.
Scopul principal al acestui capitol este de a calcula insolație disponibil ca
de intrare de la un dispozitiv solar la o anumită locație, orientare, și de timp.
Radiația solară ajunge la suprafața Pământului la un maxim de densitate de flux
(iradiere) de circa 1,0 kW/m
2
într-o lungime de undă de bandă între 0,3 și
2.5
mm. Distribuția spectrală este determinată de ~6000 K de suprafață
temperatura de Soare; este numit de radiații unde scurte și include
spectrul vizibil. Acest radiația solară la nivelul solului variază de la
aproximativ 3 la 30 MJ/(m
2

zi), în funcție de loc, timp și vreme.


Sa 'termodinamice de calitate' se referă la extreme 'alb cald
temperatura de sursa și așa este mult mai mare decât convenționale de
inginerie surse. Fluxul poate fi folosit atât termic, de exemplu pentru căldură
motoare (a se vedea punctul 4.8) și pentru photophysical și procese fotochimice
și anume fotovoltaice de energie electrică și de fotosinteza de biomasă (a se vedea
Capitolele 5 și 9).
Cum de radiații este transmis printr-un senin atmosferă depinde
pe (a) frecvența radiației, și (b) radiații absorptance de
gaze și vapori prezent. În consecință, gaze (inclusiv a vaporilor de apă)
în atmosferă că suprafața Pământului temperatura crește la
o medie de aproximativ 30°C mai mult cu atmosferă (a se vedea §2.9.1). Aceste
transmisie și caracteristicile de absorbție ale atmosferei au
similitudini cu sticlă, astfel încât suplimentare de încălzire este numit 'efectul de sera',
iar gazele în cauză sunt numite gaze cu efect de seră (Ges).
Efectul de seră este o caracteristica naturala a Pământului, și de
o importanță crucială pentru global susținut de ecologie, pentru că "normal" de
creștere a temperaturii permite cele mai multe de apă de suprafață să fie lichid, mai degrabă
decât solid. Cu toate acestea, amploarea efectului de seră depinde
în mod critic asupra atmosferic concentrația de gaze cu efect de Seră, în special Sec
2
O
și CO
2
.
Cu toate acestea, continuă și rapidă de utilizare a combustibililor fosili în trecut
200 de ani a provocat atmosferice de CO
2

concentrația sa crește
dincolo de nivelurile constatate în ultimele milioane de ani. Astfel pe plan extern
a impus modificări perturba sistemul de Pământ radiația de echilibru
radiativ forțând (de exemplu, un sistem eficient creștere netă în iluminare energetică totală cauzată de o
adăugată atmosferice componentă).
Ca autoritate documentate, de exemplu, de IPCC (2007), acest lucru este
forțând o creștere a temperaturii medii la suprafața Pământului,
atât de precipitare schimbările climatice (a se vedea §2.9). Înlocuirea combustibililor fosili cu
surse regenerabile de energie reduce acest forțând, astfel reducând riscul de
dăunătoare efecte sociale și de mediu (a se vedea §1.2 si Cutie 17.1).
TWIDELL PAGINARE.indb 39
01/12/2014 11:35
40
Radiația solară și efectul de seră
Vom începe prin a discuta despre cât de radiații este disponibil în afara
atmosferei Pământului (§2.2). Proporția în care ajunge un dispozitiv depinde în
cea mai mare parte pe timp de zi, geometrice factori, inclusiv de orientare și latitudine
(§2.4, §2.5), vreme, nori, și de absorbția atmosferică, de exemplu,
de vapori de apă (§2.6). În §2, 7 și §2.8 considerăm instrumentale
de măsurare a radiației solare și cum să utilizați alte meteorologice
date pentru estimarea radiației. §2.9 scurt examinează unele de bază de fizică
și observații de efectul de seră și schimbările climatice. Cele mai multe
informații de bază pentru scopuri de inginerie este conținută în Fig. 2.7 (de zi cu zi
expunere la soare) și Fig. 2.15 (spectrul solar). În plus, Revizuirea R3
descrie multe dintre parametrii radiațiile folosite în acest capitol.
§2.2 EXTRA-TERESTRE RADIAȚIA SOLARĂ
Reacțiile de fuziune nucleară în active de bază ale Soarelui produce interior
temperaturi de aproximativ 10
7

K și, în interior, un flux de radiații de inegale spectrale


de distribuție. Acest lucru interne de radiații este absorbită în exterior pasiv straturi
, care sunt încălzite la aproximativ 5800 K și astfel să devină o sursă de radiație
cu o relativ continuă distribuție spectrală. Strălucirea de la
Soare la Pământ distanța variază de prin anul de ±4%, din cauza
ușor non-circulara traiectoria Pământului în jurul Soarelui. Este, de asemenea, variază în funcție de
probabil ± 0.3%/y din cauza petelor solare; de-a lungul vieții de pe Pământ a
fost, probabil, o persoană fizică declin lent de mult mai puțin anuale de
semnificație (Forster și Ramaswamy 2007). Nici una dintre variante este
semnificativă pentru energie solară aplicații, pentru care considerăm extra-terestre
radiația solară să fie constantă.
Fig. 2.1 arată distribuția spectrală a radiației solare la
Pământ, distanță medie, neinfluențată de orice atmosfera. Notă modul în care această
distribuție este ca de la un corp negru la 5800 K în formă de vârf
de undă, și puterea totală emisă (cf. Fig. R3.10). Aria de sub
curbă este constanta solara G
0
*
= 1366 ±2 Wm
-2

. Acesta este fluxul radiant


densitate (RFD) incident pe un avion direct cu fața la Soare și în afara
atmosferei Pământului la o distanță de 1.496
× 10
8
km de la Soare (de exemplu, la
Pământul înseamnă distanța de la Soare).
Spectrul solar poate fi împărțit în trei regiuni principale:
1
Regiune ultraviolete (
λ < 0.4
mm)
~5% din radiația
2
Regiunea vizibil (0.4
mm < λ < 0.7
mm) ~43% din radiația
3
Infraroșu apropiat) regiune (
λ > 0.7
mm)
~52% a radiației.
Proporțiile indicate mai sus ca a primit de la suprafața Pământului cu
Soarele incidentul de la aproximativ 45 de grade. Contribuția la solar
flux de radiații de lungimi de undă mai mare de 2,5
mm este neglijabil, și toți trei
regiunile sunt clasificate ca solară de unde scurte de radiații.
Pentru a descrie interacțiunile la nivel atomic, ca în Capitolul 5 pentru pho-
tovoltaics și în Capitolul 9 pentru fotosinteza, este util de a portretiza
2000
1000
0
0.3 1
Lungime de undă / m
G
* oλ
/W m
-2
m
-1
µ
2
3
λµ
Fig. 2.1
Distribuția spectrală a
extraterestre radiația solară,
G
*
0
λ
. Aria de sub curbă este egală
1366±2 W/m
2
Sursa datelor: Gueymard 2004.
TWIDELL PAGINARE.indb 40
01/12/2014 11:35
§2.3 Extra-terestre radiația solară
41
radiații individuale fotoni de energie E
= hc/
λ. Apoi gama de
0.3 µm la 2,5 μm corespunde fotoni de energii de 4.1 eV 0.50 eV.
§2.3 COMPONENTELE DE RADIAȚII
Radiației solare incidente pe atmosfera de la direcția de
Soare este solar extra-terestre fascicul de radiatii. Sub
atmosferă, la suprafața Pământului, radiația va fi observabil din
direcția Soarelui disc în fascicul directă, și, de asemenea, de la alte
direcții ca și difuze de radiații. Fig. 2.2 este o schiță a modului în care acest lucru se întâmplă.
Rețineți că, chiar și pe un senin, zi clar, există întotdeauna cel puțin 10%
difuză, radiația de la împrăștierea moleculară, etc. Raportul dintre
fascicul de radiație și iluminare energetică totală, astfel, variază de la aproximativ 0.9 de pe o
zi clar la zero pe un complet acoperit de nori zi. În practică, distincția
între cele două componente este că numai fascicul componentă poate fi
concentrat, astfel încât sistemele care se bazează pe concentrarea energiei solare (§4.8)
funcționează bine doar în locuri în general, cu cer senin și un fascicul puternic
component.
Este important să se identifice diferitele componente ale radiației solare și
de planul pe care radiației este măsurat. Vom folosi indici
cum este ilustrat în Fig. 2.3: b pentru fascicul, d pentru difuze, t de total ore de
planul orizontal, și c pentru avionul de un colecționar. Asteriscul * indică
planul perpendicular pe fascicul. Indicele 0 reprezintă valori în afara
atmosferei în spațiu. Indici c și t se presupune că dacă nu indici
sunt date, deci G (fără indice)
≡ G
tc

.
Fig. 2.3 arată că:
G
bc
= G
b
*
pentru că
θ (2.1)
în cazul în care
θ este unghiul dintre raza și normala la colector
suprafață. În special,
G
bh
= G
b
*
pentru că
θ
z
(2.2)
în cazul în care
θ
z
este (solar) unghiul zenital dintre grindă și verticale.
Totalul de iradiere pe orice avion este suma dintre fascicul și difuze
componente:
G
t
= G
b
+ G
d
(2.3)
Soare
Direct
fascicul
Difuze
Nor de praf și
Fig. 2.2
Originea fascicul directă și difuză a radiației.
TWIDELL PAGINARE.indb 41
01/12/2014 11:35
42
Radiația solară și efectul de seră
A se vedea §2.8 pentru mai multe discutii despre raportul dintre fascicul și difuză
a radiației.

§2.4 GEOMETRIA PĂMÂNT ȘI SOARE


§
2.4.1 Definiții
Este util pentru a marca puncte și avioane pe o sferă, ca în fig 2.4
și 2.5.
Fig. 2.4 arată Pământul. Se rotește în 24 de ore în jurul propriei axe,
care definește punctele de nord și sud poli N și S. axa
polilor este normal să Pământului planul ecuatorial. În Fig. 2.4, C este
centrul Pământului. Punctul P de pe suprafața Pământului este determinată de
sale latitudine
f și longitudine; f este pozitiv pentru puncte la nord de Ecuator,
negativ sud de Ecuator. Prin acord internațional,
y este măsurată
pozitiv spre est de Greenwich, Anglia. Verticală nord–sud
avion prin P locală este meridionali avion. E și G din Fig. 2.4 sunt
puncte de pe Ecuator având aceeași longitudine și P și Greenwich
respectiv.
Fascicul
(a)
(b)
(c)
Perpendicular
pe raza de Soare
Orizontală
=
=
+
+
La panta de
un colecționar
G
b
*
G
t
*
G
bh
G
bc
z
θ
θ
G
dc
G
dh
G
th
G
bh
G
≡ G
tc
G
bc
G
dc
G
dh
Difuze
Fig. 2.3
Tehnici pentru a măsura diferitele componente ale radiației solare. Detectorul este presupus
a fi o suprafață neagră de unitatea de suprafață cu un filtru pentru a exclude longwave radiații. (a) Difuză
blocat. (b) Fascicul blocat. (c) Total.
Meridionali
avion
P
E
φ
G
S
C
N
Ecuatorială
avion
ψ
Fig. 2.4
Definiția schiță pentru latitudine
f
și longitudinea
y (a se vedea textul pentru
detaliu).
TWIDELL PAGINARE.indb 42
01/12/2014 11:35
§2.4 Geometria pământ și soare
43
La amiază solare de timp are loc o dată la fiecare 24 de ore, atunci când meridionali
avion CEP include Soare, ca și pentru toate punctele având ca longitudine.
Cu toate acestea, civile, timpul este definit astfel încât mare parte din țară, care acoperă
până la 15° de longitudine, partaja același oficial de fus orar. Mai mult decât atât,
resetarea ceasuri de 'vara' înseamnă că timpul solar și civile timpul
poate diferi cu mai mult de o oră.
1
La unghi oră
w P este unghiul prin care Pământul s-a rotit
de la amiază solare. Deoarece Pământul se rotește (360°/24h)
= 15°/h, ore
unghiul este dat de:
w = (15°/h
-1
)(t
solar
-12h)
=
(15°/h
-1
)(t
zona
-12h) +
w
eq
+(
y – y
zona
) (2.4)
în cazul în care t
solar
și t
zona
sunt, respectiv, locale solare și civile ori (amm-
ured în ore),
y
zona
este longitudinea unde Soarele este deasupra capului când
t
zona
este după-amiază (de exemplu, în cazul în care timpul solar și civile timp coincid).
w este pozitiv în
seara și negative în dimineața. Mici de corecție pe termen
w
eq

se numește ecuația de timp; ea nu depășește 15 minute și poate fi


neglijat pentru cele mai multe scopuri (a se vedea Duffie și Beckman 2006). Ea apare
deoarece elipticitate a Pământului pe orbită în jurul Soarelui, înseamnă că
acolo nu sunt exact 24 de ore între succesive solare noons, deși
intervalul mediu este 24.0000 ore. (Efectul de elipticitate pe
iradianța este mic: a se vedea Problema 2.6.)
Pământul orbitează în jurul Soarelui o dată pe an, în timp ce direcția axei sale
rămâne fix în spațiu, la un unghi
d
0
= 23.45° departe de normal
Fig. 2.5
Mișcarea de revoluție a Pământului în jurul Soarelui, așa cum este privit dintr-un punct oblic deasupra orbitei (nu
la scară!). Grele linie pe Pământ este ecuator. Adjectivele 'de toamnă, de primăvară
(spring); vara și iarna poate fi folosit pentru a distinge echinocții și solstiții, ca
adecvate pentru sezonul și emisfera. Rețineți că solstițiile de vară și de iarnă
sunt, respectiv, cea mai lungă și cea mai scurtă zi din an, și în câțiva ani să apară pe
cea de-a 22-a zi a lunii mai degrabă decât pe cea de-a 21.
N
21 Dec.
S
21 Martie
Soare
21 Sept.
21 iunie
S
N
δ
ο
δ
ο
δ
ο
δ
ο
TWIDELL PAGINARE.indb 43
01/12/2014 11:35
44
Radiația solară și efectul de seră
planul de revoluție (Fig. 2.5). Unghiul dintre direcția Soarelui
și planul ecuatorial se numește declinație
d, referitoare la sezon
modificări. Dacă linia de la centrul de la Pământ la Soare taie
suprafața Pământului la P în Fig. 2.4, apoi
d este egal cu f, adică declinația este
latitudinea punctului unde Soarele este exact deasupra capului la amiază solare.
Prin Urmare, (Fig. 2.6),
d variază ușor de la +d
0
= +23.45° la midsum-
mer în emisfera nordică, la
d
0
= -23.45° la nord la mijlocul iernii.
Din punct de vedere analitic,
d=d
0
păcat[360°(284
+ n)/365] (2.5)
unde n este ziua în anul (n
= 1 pe 1 ianuarie).
§
2.4.2 Latitudine, de anotimp și de zi cu zi de expunere la soare
De zi cu zi de expunere la soare H este energia totală pe unitatea de suprafață a primit pe o
suprafață într-o zi de Soare:
H
= ∫G
t
dt
(2.6)
Fig. 2.7 ilustrează cât de zi cu zi de expunere la soare variază în funcție de latitudine și
anotimp. Variația sezonieră la latitudini mari este foarte mare. La
cantitatea complotat este cer senin radiației solare pe un plan orizontal. Sale de
variație sezonieră apare de trei factori principali:
1
Variația în lungime a zilei Problemă 2.5 arată că numărul
de ore între răsărit și apus de soare este:
N
= (2/15)pentru
-1
(–tan
f tand) (2.7)
La latitudinea de
f = 48°, de exemplu, N variază de la 16 ore în mijlocul verii
la 8 ore in mijlocul iernii. În regiunile polare (de exemplu, în cazul în care
|
f| > cu 66,5°)
21 Martie
N
N
N
N
S
S
S
S
21 iunie
21 Sept.
21 Dec.
Soarele
radiații
δ
=0
°
δ
=0
°
δ
= 23.5
°
δ
= – 23.5
°
Fig. 2.6
Pământul, văzut dintr-un punct în continuare de-a lungul orbitei sale. Cercurile de latitudine 0°, ±23.5°, ±66.5°
sunt afișate. Notă modul în care declinația
d variază pe parcursul anului, reprezentând extreme la
cele două solstiții și zero când la amiază Soarele este la Ecuator pentru cele două
echinocții (egale zi și noapte la Ecuator).
TWIDELL PAGINARE.indb 44
01/12/2014 11:35
§2.4 Geometria pământ și soare
45
|tan
ϕ tand | poate depăși 1. În acest caz N = 24 h (în timpul verii) sau N = 0
(în timpul iernii) (vezi Fig. 2.6).
2
Orientarea de la primirea de suprafață Fig. 2.8 arată că planul orizontal
la o locație P este orientat mult mai mult spre fascicul solar în timpul verii
decât în timpul iernii. Prin urmare, chiar dacă G
b
*
în (2.2) rămâne aceeași,
factor cos
θ
z
reduce G
bh
în timpul iernii, și, astfel, reduce H
h
. Astfel, curbele
în Fig. 2.7 sunt aproximativ proporționale cu cos
θ
z
= cos(
f – d) (Fig. 2.8).
Pentru expunere la soare pe suprafețe de diferite pante, vezi Fig. 2.18.
3
și vreme de Variație în absorbția atmosferică clear sky
radiații reprezentate grafic în Fig. 2.7 este mai mică decât extra-terestre radiații din cauza
atmosferice atenuare și dispersie. Acest atenuarea crește
cu
θ
z
deci G
b
*

este mai mică în timpul iernii; în consecință, variația sezonieră de


cer senin expunere la soare este mai mult decât datorează efectele geometrice (1) și
(2) (a se vedea §2.6). Mai mult decât atât, cer senin, radiația este oarecum noțională
cantitatea, deoarece condițiile meteorologice, în special în cloud, variază foarte mult și
de multe ori domina primit insolație.
Pentru proiectarea clădirilor, este esențial să realizăm că variația H pe o
verticală sau înclinată a suprafeței (de exemplu o fereastră) este semnificativ diferită de
cea prezentată în Fig. 2.7 (a se vedea §2.8.6 și Fig. 2.18). În consecință iarna
captarea energiei solare de clădiri în mijloc și latitudini mai mari pot fi
semnificative.
30
H
h
/(MJ m
-2
zi
-1
)
25
20
15
10
5
J (Nord)
JASONDJ
FMAM
Luna
Latitudine
0
°
12
°
24
°
36
°
48
°
60
°
J
FMAMJ
J A S O N D (Sud)
Fig. 2.7
Variația cu sezon de latitudine și de Sec
h

, energia solară (de zi cu zi insolație) a primit, la un


plan orizontal pe o zi clar. În timpul verii, H
h

este de aproximativ 25 MJ m
-2
zi
-1

la toate latitudinile.
În timpul iernii, H
h
este mult mai puțin la latitudini mari din cauza mai scurte de seara, mai oblic
incidență, și mai mare atmosferice atenuare. A se vedea, de asemenea, Fig. 2.15, care arată cât de
zi cu zi de expunere la soare variază cu panta de suprafață care, mai ales pentru vertical
suprafețe, cum ar fi windows.
TWIDELL PAGINARE.indb 45
01/12/2014 11:35
46
Radiația solară și efectul de seră
§2.5 GEOMETRIA ȘI COLECTOR SOLAR
FASCICUL
§
2.5.1 Definiții
Pentru înclinat suprafață (colector) este prezentat în Fig. 2.9, următoarele Duffie și
Beckman (2006), vom defini următoarele.
(a) Pentru colector de suprafață
Panta
b: unghiul dintre planul suprafeței în cauză și
orizontală. În ambele emisfere: pentru o suprafață orientată spre
Ecuator 0
<
b < 90°, pentru o suprafață care se confruntă departe de Ecuator 90
°
<
b
<180
°
.
Suprafața unghi de azimut
g: proiectate pe planul orizontal, unghiul
dintre normala la suprafață și locale longitudinea meridianului. În
ambele emisfere, pentru o suprafață orientată spre sud
g = 0°; spre nord g =
180°; spre vest
g = 0° la 180°; spre est, g = 0° -180°. Pentru orice hori-
zontal suprafață,
g = 0°.
Unghiul de incidență
θ: unghiul dintre fascicul solar și suprafața normală.
(b) Pentru fascicul solar
(Solar) unghiul zenital
θ
z
: unghiul dintre fascicul solar și pe verticală.
Rețineți că
θ
z
și
θ nu sunt, de obicei, în același plan.
Solar altitudine
o
s
(
= 90°-
θ
z
): complementul (solar) zenith
unghi; unghi fascicul solar orizontal.
Soare (solar) unghi de azimut
g
s

: proiectate pe planul orizontal, în


unghiul dintre fascicul solar și longitudinea meridianului. Semnează
convenția
g. Deci, pe plan orizontal, unghiul dintre raza și
suprafața este (
g
s
-
g ).
C
S
(a)
(b)
Soarele
razele
P
N
E
N
C
S
P'
δ
δ'
θ
θ'
β
β'
φ
φ'
Fig. 2.8
Secțiuni transversale prin Pământ la amiază solare, care arată relația între latitudine
f, declinația d, și panta b a unui colector la P. θ este unghiul de incidență la nord/
de sud-cu care se confruntă colector. (a) emisfera Nordică în timpul verii:
f, d, b >0. (b) 'Simetric'
exemplu 12 ore mai târziu, în emisfera sudică. (
f' = -f, d = -d, b' = b, θ' = θ).
TWIDELL PAGINARE.indb 46
01/12/2014 11:35
§2.5 Geometria și colector solar beam
47
(Solar) unghi oră
w: ca în (2.4), unghi care Pământul s-a rotit de
la amiază solare (de exemplu, atunci când
g
s
= 0 în emisfera nordică).
§
2.5.2 Unghiul dintre raza și colector
Cu această convenție, geometrie oferă ecuații esențiale pentru solar
modelare:
pentru că
θ = (A-B) sin d + [C sin w + (D+E) cosw]cosd (2.8)
în cazul în care
O = sin
f cos b B
= cos
f sin b cos g
C = sin
b sin g D
= cos
f cos b
E = sin
f sin b cos g
și
pentru că
θ = cos θ
z
pentru că
b + sin θ
z
păcat
b cos(g
s

g) (2.9)
Pentru mai multe geometrii speciale, complicate formula (2.8) simplifică
în mod considerabil; de exemplu, pentru un colector orientat spre ecuator
și cu panta
b egal cu magnitudinea de latitudine f, (g = 0, b = f
în emisfera nordică;
g = 180°, b = -f în emisfera sudică), (2.8)
reduce la
pentru că
θ = cos w cosd
(2.10)
Normal la
orizontală
Zenith
Soare
W
E
N
S
Normală
a înclinat
suprafață
θ
z
θ
γ
β
Fig. 2.9
Unghiul zenital
θ
z
, unghiul de incidență
θ
,
panta
b
și unghi de azimut
g
pentru o suprafață înclinată.
Notă: pentru ca această est-cu care se confruntă suprafață
g < 0.
Sursa: După Duffie și Beckman (2006).
TWIDELL PAGINARE.indb 47
01/12/2014 11:35
48
Radiația solară și efectul de seră
1
2
3
4
5
6
7
8
9
10
11
12
13
14
15
16
17
18
19
20
21
22
23
24
25
26
27
28
29
30
31
32
33
34
35
36
37
38
39
40
41
42
43
44
45
Pentru un plan orizontal,
b = 0 și (2.8) se reduce la
pentru că
θ
Z
= sin
f sin d + cos f cos w cosd
(2.11)
Două atenționări ar trebui să fie menționat despre (2.8) și formule similare:
1
La latitudini mai mari în timpul verii,
θ depășește 90° la începutul până la mijlocul-dimineață,
și de la mijlocul până seara târziu, când Soarele se ridică sau cade să
observatorului orizont (de exemplu, pentru că
θ negativ). Când acest lucru se întâmplă, de
exemplu, pe un sud-cu care se confruntă suprafeței în emisfera nordică,
iradiere va fi pe partea din spate de un colecționar, nu în față.
2
Formulele sunt derivate în mod normal pentru cazul în care toate unghiurile sunt
pozitive, și în special
f > 0. Unele latitudine nordică scriitori acorde
o atenție insuficientă pentru a semna, atât de des formulele lor nu se aplică în
emisfera sudică. Sud cititorii ar trebui să verificați toate având
formule, de exemplu, prin construirea complementare diagrame, cum ar
fi Smochine 2.8(a) și (b), în care
θ' = θ, și verificarea că semnele în
dat formula de acord.
§
2.5.3 Optime de orientare a unui colector
O parabolica de concentrare colector (§4.8.2) trebuie să arate întotdeauna spre
direcția de fascicul solar (de exemplu,
θ = 0). Cu toate acestea, optim
EXEMPLUL 2.1 CALCULUL DE UNGHIUL DE INCIDENȚĂ
Se calculează unghiul de incidență al fasciculului de radiații pe o suprafață situat în Glasgow, Scoția (56°N, 4°W)
la ora 10 dimineața, pe 1 februarie, dacă suprafața este orientată 20° est, de sud și înclinată la 40° față de orizontală.
Soluție
1 februarie este a 32-a zi a anului (n
= 32), deci de la
d
= 23.45° păcat[360°(284 + 32)/365] = -17.5°
Civilă dată în Glasgow iarna este Greenwich Mean Time, care este timpul solar (
±15 min) la longitudinea
f
zona
= 0.
Prin urmare, t
solar
≈ 10 h, deci (2.4) dă
w = -30°.
Astfel
f = +56°, g = -20° și b = +40°, astfel încât, în (2.8)
O
= sin 56° pentru 40° = 0.635
B
= cos 56° sin 40°cos(-20°) = 0.338
C
= sin 40° sin(-20°) = -0.220 D
= cos 56° pentru 40°
=
0.428
E
= sin 56° sin 40°cos(-20°) = 0.500
prin urmare,
pentru că
θ = (0.635–0.338)sin(-17.5°) + [-0.220 păcat(-30°) + (0.428 + 0.500)
cos(-30°)]cos(-17.5°)
= 0.783
Deci
θ = 38.5°
TWIDELL PAGINARE.indb 48
01/12/2014 11:35
§2.6 transport Atmosferic, absorbție și reflecție
49
direcția fixă placă plană colector nu este atât de evidentă. Expunere la soare Sec
c
primite este suma dintre fascicul și difuze componente:
H
G
G
=
pentru că
dt
c
b
*
d


(
)
θ+
(2.12)
În general, colector de orientare se confruntă Ecuator (de ex. spre nord
în emisfera sudică), cu o pantă de aproximativ egală cu
latitudinea, ca în (2.10). Alte considerații pot modifica acest lucru; de exemplu,
orientarea clădirilor existente și dacă mai mult de căldură este în mod
necesar (sau dispoziție) în dimineața sau după amiaza, ierni sau veri.
Cu toate acestea, deoarece pentru
θ ≈ 1 pentru θ < 30°, variații de ±30° în azimut sau
panta de orientare fixe colectoare au un efect redus asupra anuală totală
de energie colectate. De-a lungul unui an, cu toate acestea, la altitudinea de
amiază solare variază considerabil și poate fi sensibil pentru a regla
"fixe" colector de pantă.
§
2.5.4 variație Orară a radiației
Câteva exemple de variație orară a G
h

sunt prezentate în Fig. 2.10(a) pentru


zilele clare și Fig. 2.10(b) pentru o zi noroasă. În zilele senine forma din
Fig. 2.10(a) este:
G
h
≈ G
h
max
sin(
πt' / N ) (2.13)
unde t este perioada de timp după răsăritul soarelui, iar N este durata de lumina zilei pentru
anumită zi clar (a se vedea (2.7) și Fig. 2.10(a)). Integrarea (2.13) de-a lungul
zilei perioadă pentru o zi clar,
H
h
≈ (2N /
π ) G
h
max
(2.14)
De exemplu, la latitudinea
±50° (i) în mijlocul verii, dacă G
h
max
≈ 900 Wm
-2
și
N
≈ 16 h, atunci H
h
≈ 33 MJm
-2
zi
–am
; (ii) în mijlocul iernii, la aceeași latitudine,
G
h
max
≈ 200 Wm
-2
și N
≈ 8h, deci H
h
≈ 3.7 MJm
-2
zi
–am
. La tropice,
G
h
max
≈ Wm 950
-2
dar perioada de zi nu diferă mult de 12 h
pe tot parcursul anului, astfel încât H
h
≈ 26 MJ m
-2
zi
-1
pe toate zilele senine.
Aceste calcule nu fac nici indemnizațiile pentru cloud sau praf, și așa mai
media valorilor măsurate de Sec
h
sunt întotdeauna mai puțin decât cele menționate.
În majoritatea regiunilor, valorile medii ale Sec
h

sunt, de obicei, de la 50 la 70% din cer acoperit


de nori valoare.
§2.6 TRANSPORT ATMOSFERIC, ABSORBȚIE,
ȘI DE REFLECȚIE
Temperaturile din atmosfera superioară a Pământului, la circa 230 K,
și Pământul suprafețe, la aproximativ 260 la 300 K, rămâne în echilibru
la mult mai puțin decât
~6000 K temperatura de Soare. Prin urmare, la
exterior fluxuri de energie radiantă emisă de către atmosfera Pământului și
TWIDELL PAGINARE.indb 49
01/12/2014 11:35
50
Radiația solară și efectul de seră
suprafețe egale în medie de intrare a radiației, ambele
~1 kWm
-2

.
Ieșire de departe-infraroșu cu lungime de undă de bandă are lungimi de undă cuprinse între
aproximativ 5 și 25
mm, numit longwave radiații, ajungand la aproximativ 10
mm (a se vedea legea lui Wien, §R3.5). În consecință, pe unde scurte și
lungi de radiații regiuni pot fi tratate ca fiind destul de diferite una de cealaltă,
care este o puternică metodă de analiză în știința mediului (a se vedea
Fig. 2.13(a)).
Ca radiatia solara trece prin gaze și vapori din
atmosfera Pământului un set complicat de interacțiuni apare, care reduce
densitatea de flux ajunge la suprafața Pământului. Interacțiunile cu
molecule, atomi și particule includ: (i) atmosferice de absorbție (
~19%),
provocând încălzire și ulterior re-emisie de energie ca longwave
radiații; (ii) împrăștierea, lungimea de undă dependentă de schimbare în
direcție, astfel încât, de obicei, nici o suplimentare de absorbție apare și radiația
continuă difuz la aceeași lungime de undă; și (iii) reflecție (
~30%), de la
Fig. 2.10
(a) Radiația pe o suprafață orizontală, măsurată pe trei diferite aproape clar de zile de la
Rothamsted, marea BRITANIE (52°N, 0°W). Notă modul în care atât valoarea maximă a G
h

și lungimea
zilei sunt mult mai puțin în timpul iernii decât în timpul verii. (Sursa: După Monteith și Unsworth
2007). (b) variație Tipică de radiația pe o suprafață orizontală pentru o zi de variabilă cloud.
Notă valorile scăzute în cer acoperit de dimineață, și mari, neregulate variații în
după-amiaza din cauza împrăștiate în cloud.
Iunie
1000
800
600
400
200
(b)
(a)
Timp de zi/h
Timp Solar/h
800
400
G
h
/(W m
-2
)
Iradiere orizontală
G
h
/(W m
-2
)
6
8
10
12
14
16
18
0
2
4
6
8
10
12
14
16
18
20
22
24
Sept
Jan
TWIDELL PAGINARE.indb 50
01/12/2014 11:35
§2.6 transport Atmosferic, absorbție și reflecție
51
particule, nori, și de la suprafața Pământului, care este
independent de lungimea de undă. Deci, chiar și cu cer senin este reflectarea înapoi în
spațiu. Informații de fond este prezentat în Caseta 2.1. Fig. 2.11 descrie
primite solare de unde scurte de radiații. Fig. 2.12(a) și (b) atât arăta
de scurtă și lungă fluxurilor și interacțiunilor, așa cum este descris de două
surse.
În consecință, continuă radiația solară de unde scurte în cer, nor-
mai puțin condiții , la amiază are densitatea de flux redus de la 1,3 kW/m
2
în
spațiu, pentru a
~1.0 kW/m
2
la nivelul solului. Acest maxim de radiație solară de
~1 kW/m
2
este un parametru important să-ți amintești.
§
2.6.1 Reflecție
În medie, aproximativ 30% din extra-terestre intensitatii solare este reflectată
înapoi în spațiu. Cele mai multe dintre reflexia are loc de la picături de apă lichidă
Solară extraterestră
fascicul de radiații
(radiații unde scurte)
A revenit la spațiu ca unde scurte radiația reflectată
Absorbția în atmosferă cauzează încălzire
Eventuala re-emisie ca longwave radiații
Continuă în direcția înainte în
unghiulare con de discul Soarelui
Direct fascicul de radiatii, indice b.
Difuză sau reflectată de directă fascicul de
discul soarelui, încă incident pe suprafața Pământului
Di folosesc radiații, indice d.
Radiația difuză poate fi împărțită
după acel incident din anumite direcții
având o dependența unghiulară
Rămase difuză, radiație având
pic dependența unghiulară
Fig. 2.11
Efecte care apar ca extra-terestre radiația solară trece prin Pământ e
atmosfera.
TWIDELL PAGINARE.indb 51
01/12/2014 11:35
52
Radiația solară și efectul de seră
107
Reflectate solare
radiația
107 Wm
-2
Reflectate de nori,
aerosoli și
atmosferice
gaze
77
Reflectată de
suprafata
30
168
Absorbit de
suprafata
235
Cu efect de seră
gaze
324
Absorbită de suprafața
324
Înapoi
radiații
40
Atmosferice
fereastra
40
30
165
Ieșire
longwave
radiații
235 Wm
-2
Intrare
solar
radiații
342 Wm
-2
Emise de
atmosfera
Emise de nori
Absorbit de
atmosfera
Latentă
78 căldură
350
390
Suprafata
radiații
78

Evapotranspirației
24
Termici
24
67
342
Fig. 2.12
o
Estimare a Pământului anual și global al bilanțului energetic. Pe termen lung,
suma de radiația solară absorbită de Pământ și atmosferă este
echilibrat de Pământ și atmosferă eliberează aceeași cantitate de ieșire
longwave radiații. Aproximativ jumătate din radiația solară este absorbită de
suprafața Pământului. Această energie este transferată în atmosferă prin încălzirea aerului în
contact cu suprafața (termici), prin evapotranspirație și de lungă radiații
este absorbită de nori și de gaze cu efect de seră. Atmosfera în rândul radiază
longwave energie înapoi la Pământ precum și la spațiu.
Sursa: IPCC (2007, FAQ1.1 Fig. 1).
și gheață în nori, cu o proporție mai mică din Pământul e uscat și pe mare
suprafață (mai ales de zăpadă și gheață) (vezi Fig. 2.12). Încă o mică
proporție este de împrăștiere atmosferic. Această reflexie este numit albedo,
și variază în funcție de condițiile atmosferice și de unghiul de incidență.
§2.6.2 de Aer–raportul de masă
Distanța parcursă de fascicul directă prin atmosferă
depinde de unghiul de incidență la atmosferă (unghiul zenital)
și înălțimea deasupra nivelului mării de observator (Fig. 2.14). Considerăm
un cer senin, cu nici un nor, praf sau poluare a aerului. Din partea de sus a
atmosferei nu este bine definit, de o importanță mai mare decât distanța
parcursă este cantitatea de gaze atmosferice și vapori întâlnite.
Pentru fascicul directă la incidență normală trece prin
atmosferă la presiune normală, o cantitate standard ("masă") din atmosferă este
întâlnit. Dacă fasciculul este la unghiul zenital
θ
z

creșterea calea lungime


comparativ cu normalul cale este numită de aer–raportul de masă (sau de masă de aer'),
simbol m.
TWIDELL PAGINARE.indb 52
01/12/2014 11:35
§2.6 transport Atmosferic, absorbție și reflecție
53
Abreviația este folosită pentru aer–raportul de masă. AM0 se referă la zero
atmosferă, adică radiații în spațiu; AM1 se referă la m
= 1, adică Soarele
deasupra capului; AM2 se referă la m
= 2; și așa mai departe.
Din Fig. 2.14, deoarece nu cont este luată de obicei de curbură a
Pământul,
m = sec
θ
z

(2.15)
diferite de aer–raportul de masă întâmpinate din cauza o schimbare în
presiunea atmosferică sau schimbare în înălțime de observator este considerat
separat.
Radiația solară de unde scurte (s. w.)
Longwave radiații infraroșii (am.r.)
16% Absorbită de Sec
2
O,
O
3
, CO
2
,
CH
4
,
alte
GES, particule de aerosoli
4% Absorbit
prin nori
50%, Medie, incidentul ca pe direct
și difuze de iradiere cu unde scurte
6% retroîmprăștiați
de molecule de aer
4% Reflectată de
suprafața pământului
20% Reflectată de
nori
20% la am.r.
50%
6%
20%
44%
Ieșire longwave infraroșu 70%; deci
net primite s. w. = net de ieșire l.w.
20% Net am.r.
emisii de
suprafețele
Alte caldura
în sus de la
suprafețele de 30%
Din absorbit s. w.
ca am.r. emisii
de la H
2
O, O
3
, CO
2
,
CH
4
alte GES, aerosoli
particule și nori
Din care 20%
trece prin
Atmosfera
adică 6% din totalul
Din care 80%
trece prin
Nori etc.
adică 44% din total
30% din Totalul de reflectat
unde scurte solare
Solare de intrare
100%
Fig. 2.12
(cont.)
b
Alternative aproximat reprezentare a radiativ componenta a (a), indicând
procesele fizice implicate
TWIDELL PAGINARE.indb 53
01/12/2014 11:35
54
Radiația solară și efectul de seră
(a)
(b)
(c)
Fig. 2.13
Radiațiile transmise și absorbită de către atmosferă în funcție de lungimea de undă.
o
Monocromatic densitatea de flux radiant
f
λ

pentru downgoing ('unde scurte') solare,


radiații și upgoing termică ('lunga') radiație. (Notă: trase cu vârfuri
normalizat.)
b
Total monocromatice absorptance
o
λ
de atmosferă.
c
Contribuții la
o
λ

de la diferite gaze și alte efecte.


A se vedea textul (Caseta 2.1) pentru mai multe detalii.
Nivelul mării
Unghiul zenital
Atmosfera : cer acoperit de nori
Incidență normală:
unitate calea
θ
z
Calea
crescut cu
un factor de
m
Fig. 2.14
Aer–raportul de masă m
= sec
θ
z

.
TWIDELL PAGINARE.indb 54
01/12/2014 11:35
§2.6 transport Atmosferic, absorbție și reflecție
55
CASETA 2.1 RADIAȚII TRANSMISE, ABSORBIT ȘI ÎMPRĂȘTIATE DE PĂMÂNT E
ATMOSFERA
Fig. 2.13(a) prezinta:
am
În curba la stânga, sus, distribuirea de radiații de la surse din exterior temperatura Soarelui (aici 5525 K).
ii
În curbe la dreapta sus, distribuirea de radiații dintr-o gamă de temperaturi (210 K, 260 K și 310 K)
ușor mai mare decât intervalul de la partea de sus la partea de jos a atmosferei Pământului.
iii
În solide, umplere, la lungimi de undă de solar 'downgoing' radiații ce ajung la suprafața Pământului (de exemplu, expunere la soare), și
de 'upgoing termică radiația infraroșie trece de la partea de sus a atmosferei.
Fig. 2.13(b) arată procentul de absorbție din atmosferă pe deplin solare de unde scurte și termică longwave
regiuni spectrale. La stânga este 'unde scurte fereastra'; aceasta transmite numai "sigure" insolație; adică se absoarbe mai
scurt radiații ultraviolete,
λ < 0.3μm, care altfel ar deteriora mult viața biologică. Rețineți că moleculară
și particule de împrăștiere și absorbție reduce intensitatea razei la nivelul solului chiar și într-un cer senin. Ne
poate rețineți că temperatura medie la suprafața Pământului (aproximativ 291 K
= 14°C) permite cele mai multe suprafață
apa să fie lichidă și reacțiile fotosintetice a progresa.
La dreapta este 'longwave fereastră', care transmite vârf de ieșire radiație infraroșie, dar al cărui abrupt
limitele sunt în mare parte determinate de absorbție de vapori de apă și CO
2

. Rețineți că o mare parte din upgoing


radiația termică de la suprafața Pământului este absorbită în atmosferă. Absorbția la aceste lungimi de undă, de asemenea,
apare în radiativ solar de intrare, dar proporția în totalul fluxului solar este mai mică. Natura selectivă a
longwave absorbție rezultă din modurile de vibrație de gaze și vapori de molecule cu trei sau mai mulți atomi
(H
2
O, CO
2
, CH
4
,N
2

O, etc.). În vigoare, concentrația acestor gaze în atmosferă afectează lățimea (interval


de lungimi de undă) de la fereastră. La o concentrație mai mare, îngustă fereastră, și vice-versa. O largă
fereastră duce la o răcire a suprafeței Pământului; o fereastră îngustă duce la încălzirea.
Cu toate acestea, rolul apei este complicat, pentru că a crescut Pământ temperaturii duce la evaporarea a crescut
și vice-versa. Crescut de evaporare duce la (i) mai mult de cloud care reduce insolationand, prin urmare, se răcește Pământul;
și (ii) a crescut de vapori de apă de concentrare, mai ales la altitudine mare, care îngustează longwave fereastră, care
duce la încălzire. Calcule de modelare indică faptul că rezultanta rolul de vapori de apă pe temperaturii globale
modifica este mai puțin pronunțată decât cea a CO
2
.
Fig. 2.13(c) prezinta separat spectrele de absorbție majore de gaze și vapori de apă în atmosferă, și
efectul de împrăștiere Rayleigh. În longwave termică regiune, vapori de apă și CO
2

absorbi în mod semnificativ la


radiații infraroșii upgoing de la suprafața Pământului și atmosfera inferioară. Vapori de apă concentrația variază foarte mult
în funcție de regiune și de sezon, și ar putea ajunge la circa 4% din volumul atmosferei la nivel local, dar la nivel global medie de
concentrare nu se schimba prea mult. Astfel, fluctuațiile de absorbție de vapori de apă poate fi de o semnificație
în aplicații practice, dar norul este probabil să fie mult mai influent. Această absorbție de longwave radiații
crește temperatura din atmosferă și, prin urmare, la suprafața Pământului, adică provoacă radiativ forțând și
efectul de seră (a se vedea §2.9).
Împrăștierea Rayleigh este imprastierea elastica de lumină sau alte radiații electromagnetice de particule mult mai mici
decât lungimea de undă a luminii. Particulele pot fi individuale atomi sau molecule. Împrăștierea Rayleigh de lumina soarelui
în atmosferă determină radiația difuză a cerului, care este motivul pentru culoarea albastră a cerului și ton de galben
de Soare în sine.
Unde scurte radiații ultraviolete (
λ < 0.3μm) ar afecta multe forme de viață, dar este eliminat din downgoing
radiația cea mai mare parte de ozon (O
3
) în partea superioară a atmosferei și de împrăștierea Rayleigh. Cu toate acestea, chiar și în cantitate mică
de radiații UV transmise (cu 0,3μm
<
λ < 0.4μm) este suficient pentru a provoca arsuri solare severe. Epuizarea atmosferic
de ozon, prin urmare, constituie o amenințare majoră pentru sănătatea oamenilor și chiar mai mult pentru plante; prin urmare,
preocuparea
despre epuizarea ozonului în straturile superioare ale atmosferei, descoperit în anii 1970, care a fost dovedit a fi cauzate de
emisiile de clorofluorocarburi și substanțe înrudite, care sunt făcute de produse chimice industriale. Deși unele
dintre aceste substanțe sunt, de asemenea, gazele cu efect de seră, este de ozon, epuizarea stratului de Ozon este deosebit de puternică în
mare
latitudini la primavara (de 'gaura de ozon'). La Protocolul de la Montreal (1989) este un acord internațional să elimine
producerea și utilizarea unor astfel de substanțe, și s-a dovedit eficientă în acest sens.
TWIDELL PAGINARE.indb 55
01/12/2014 11:35
56
Radiația solară și efectul de seră
Lungime de undă
λ
/
µ
m
G
*
λ
/W
m
-2
µ
m
-1
Extraterestre
Nivelul mării (m = 1)
0.3
0
1000
2000
1
2
3
Fig. 2.15
Distribuții spectrale de radiație solară primită de deasupra atmosferei (curba superioară)
și de la nivelul mării (inferioară a curbei). Aproximativ jumătate din această iradiere cu unde scurte are loc vizibil în
regiune (de la 0,4 la 0,7
mm). Există o scădere treptată a G
b
*

ca
λ crește în infraroșu,
cu goluri în mare nivel de spectru datorită absorbției de Sec
2
O și CO
2

. 'Nivelul mării' curba este


pentru masa de aer m
= 1.
§2.6.3 de Nori temperatura
Aer, vapori de apă, nori, și particule în atmosferă emit în infraroșu
radiații la nivelul solului obiecte în funcție de temperatură și de masă
în transmiterea calea. În consecință, obiecte de la suprafața Terrei
de schimb radiații predominant cu răcitor de aer și vapori de apă de mare în
atmosferă și, dacă este prezent, cu nori. Având în vedere acest schimb în
termeni de §R3.5 (Tabelul C. 5), cerul se comportă ca o cabina de la o medie
de temperatură T
cer
, cu nori temperatura, care, în practică, este întotdeauna mai puțin
decât la nivelul solului temperatura mediului ambiant T
o
. Comun de estimare este:
T
cer
≈T
o
- 6°C (2.16)
deși în cer acoperit de nori regiunile deșertice, pe timp de noapte (T
o
– T
cer
) poate fi la fel de mare ca
25°C. Dacă norii sunt prezenți, "cerul" temperatura crește, dar poate fi
întotdeauna de așteptat să fie mai mică decât nivelul solului temperatura.
Mediu de nori temperatura poate fi măsurată cu ușurință cu un îngust
diafragma termometru cu infraroșu arătând spre cer numai.
2

§2.6.4 spectrul Solar a primit de la suprafața Pământului


Fig. 2.15 arată efectul cumulativ asupra spectrului solar de aceste
absorbtii. Partea inferioară a curbei este spectrul de Soare, văzută prin
aer–raportul de masă m = 1. Aceasta reprezintă radiația primit aproape amiază
la tropice (cu Soarele vertical deasupra observatorului). În
spectrul primit de fapt depinde de gradul de prăfuire și umiditate, chiar și în
absența de cloud.
TWIDELL PAGINARE.indb 56
01/12/2014 11:35
§2.8-Ul de estimare a radiației solare
57
§2.7 MĂSURAREA RADIAȚIEI SOLARE
Pentru măsurarea radiației solare există o gamă largă de instrumente, de exemplu,
Fig. 2.16, multe cu nume confuze! Fundamentale (absolută)
măsurători la standarde laboratoare utilizarea activă cavitatea radiometre; la
solar beam este absorbit pe un negru mat de suprafață din zona O, a căror
creștere a temperaturii este măsurată și comparată cu creșterea temperaturii într-un
identice (umbrită) absorbant de încălzit electric. În principiu, atunci,
aAG
b
*
=P
elec
(2.17)
Geometria este de așa natură încât absorptance
o = 0.999. Importante utilizări sunt
pentru masuratori prin satelit solar constantă, și pentru calibrarea
secundar instrumente.
Selectate meteorologice, stații Meteorologice Mondiale Birou
(OMM) standardizat piranometrele cu o precizie absolută
~3%. În
esență, ele au termocuplu intersecții (o termopilă) sub un negru
suprafata, toate sub un pahar emisfera; creșterea temperaturii cauzate
de absorbit de expunere la soare produce o calibrat tensiune. În practică, ele
sunt proiectate și fabricate cu mare experiență. De bază
de măsurare este total radiația pe o suprafață orizontală G
th

(Fig. 2.3(c)). Alte
măsurători pot fi: (i) de radiații difuze numai, cu radiație directă
a împiedicat de o reglabile umbra inel; (ii) de fascicul de radiatii G
b
*

numai
că intră în colimatorul tub continuu de urmărire a traiectoriei Soarelui (un
pyroheliometer).
Pentru domeniul de utilizare (de exemplu, măsurarea iradianței pe diferite părți ale unei
clădiri) sunt mult mai ieftin de instrumente, de multe ori numit 'solarimeters'
(deși acest termen este de asemenea folosit pentru piranometrele), care sunt de obicei
celule solare calibrat cu o OMM-instrument standardizat. Lor
precizie absolută este, de obicei doar
~15%, ca urmare a răspuns spectral
Si celule (Fig. R4.11 în Revizuire 4), dar pentru comparații lor
reproductibilitatea este probabil să fie mai mult de 5%.
§2.8-UL DE ESTIMARE A RADIAȚIEI SOLARE
§2.8.1 Cerințele
Toate dispozitivele solare utiliza radiației solare de unde scurte; astfel solare
dezvoltarea și utilizarea depind de măsurarea și estimarea atât instant
și integrat de expunere la soare la locul de utilizare. Din fericire, integrat
pe o zi sau mai mult, fără abajur expunere la soare nu este site-ul dependente de peste
regionale distanțe, astfel încât meteorologic regional de date poate fi folosit direct.
(Acest lucru contrastează cu, să zicem, energia eoliană, care este foarte ului dependente.)
Tipice de variație în timp a datelor va aplica, de asemenea, la nivel regional și pot fi utilizate pentru a
simula performanța de dispozitive în timpul dezvoltării. Prin urmare, diurne
și mai mult în medie solara date preluate de la stațiile meteorologice și
TWIDELL PAGINARE.indb 57
01/12/2014 11:35
58
Radiația solară și efectul de seră
sateliții pot fi utilizate în termen de distanta de variație de cel puțin 100 km
și, eventual, 1000 m, după cum se stabilește de către sinoptice vreme
modele. Cu toate acestea, pentru a testa un dispozitiv, de specialitate sunt necesare instrumente pentru a
măsura radiațiile solare la punctul de utilizare. Vezi Fig. 2.3 să reamintească multe
referitoare la parametrii de iradiere solare.
§2.8.2 variație Statistic
În plus evident de zi cu zi și de sezon regulat variante de expunere la soare,
ca în fig 2.7 și 2.10(a), există, de asemenea, semnificative variații neregulate.
De aceste incertitudini, cele mai semnificative pentru scopuri practice în
multe climate sunt de zi cu zi variante, ca în Fig. 2.10(b), deoarece acestea
afectează energia cerințele de stocare, de exemplu, volumul de apă caldă rezervorul de
căldură sau capacitatea bateriei pentru stand-alone de energie fotovoltaică. Astfel, chiar și
(a)
(b)
(c)
(d)
Fig. 2.16
Fotografii de diferite solare instrumente. (a) Kipp & Zonen piranometru (solarimeter) cu două cuarț cupole pentru
standardizate global insolație; (b) o astfel de piranometrele, utilizate pentru măsurarea radiației globale pe un panou solar; (c) colimată
pyroheliometer pentru măsurarea directă a radiației; (d) moderne on-line durata de strălucire a soarelui recorder.
Sursa: (a), (b) și (d) Kipp & Zonen; (c) Prof. Dr. Volker Quaschning de HTW Berlin (www.volker-quaschning.de/fotos/messung/index_e.
php);.
TWIDELL PAGINARE.indb 58
01/12/2014 11:36
§2.8-Ul de estimare a radiației solare
59
o înregistrare completă a trecut de iradiere pot fi utilizate doar pentru a statistic
a prezice viitorul iradiere cu o autonomie estimată de incertitudine.
§2.8.3 ore de Soare ca o măsură de expunere la soare
Toate marile stații meteorologice măsură de zi cu zi 'ore de luminos
soare' n, pentru care înregistrările trebuie să fie disponibile pentru multe decenii.
În mod tradițional, n este măsurată cu un sferice Campbell-Stokes recorder
care încorporează un standard de marcat cardul poziționat în spatele o
lupă. Atunci când soarele este în 'luminos', concentrat direct fascicul arde
o gaură alungită în carte. Observatorul obține n din totalul
ars lungime pe fiecare zi de card. Ore de soare sunt, de asemenea, măsurată prin
dispozitive electronice.
Naționale și internaționale stațiile meteorologice sunt dispus de a schimba
instrumente meteorologice le-au folosit, probabil, pentru zeci de o sută de
ani. Exemple Simple sunt termometre mercur și Campbell-Stokes
soare-ore de înregistrare. Datele pe termen lung se execută sunt importante, mai ales
pentru a analiza schimbările climatice, astfel încât schimbarea instrumentation poate introduce
erori necunoscute. Încă instrumente moderne sunt susceptibile de a fi mai
precis, capabil să înregistreze în format electronic, și mai puțin exigente ale timpului uman.
Deoarece Campbell-Stokes și mai modern soare recordere sunt
simple instrumente, din punct de vedere istoric ele au fost folosite
în întreaga lume pentru a corela ore de soare cu expunere la soare (Sec). Corelația
ecuații sunt de multe ori de forma:
H=H
0
[o + b(n /
N )] (2.18)
în cazul în care (pentru ziua respectivă) H
0

este orizontală radiații cu nici o


atmosferă (de exemplu, spațiu liber echivalență, calculat la fel ca în Problema 2.6)
și N este de "lungime" al zilei în ore (2.7). Cu toate acestea,
coeficienții de regresie o și b variază de la site la site. Chiar și așa, corelația
coeficientul respectiv este, de obicei, doar aproximativ 0,7, respectiv valorile măsurate expunere la soare
sunt foarte împrăștiate de cele prezise de ecuația.
Multe alte climatologice corelații cu insolație au fost
propuse, folosind variabile, cum ar fi latitudine, temperatura mediului ambiant, umiditatea
și nebulozitatea. Cele mai multe au o precizie limitată și gama de aplicabilitate.
§2.8.4 Geostaționară Operațional De Mediu Sateliți
(Se DUCE)
De măsurare și de detectare a parametrilor de mediu, folosind sateliți
au avut un impact profund asupra mediului analiza și
disponibilitatea informațiilor. Cu toate acestea, corelația dintre măsurători la sol
cu masuratorile din satelit nu este simplă. Un exemplu simplu
este prin satelit de măsurare de la nivelul solului expunere la soare. Satelitul poate
măsura separat downcoming unde scurte radiația solară (radiație)
TWIDELL PAGINARE.indb 59
01/12/2014 11:36
60
Radiația solară și efectul de seră
din spațiu, și upgoing unde scurte radiații. La upgoing radiații este
suma dintre (i) radiației reflectate și împrăștiate în sus de
atmosferă și de nor, și (ii) expunere la soare reflectată de la suprafața Pământului și
transmise prin atmosfera (vezi Fig. 2.12). Teren-nivelul de
insolație este downgoing radiației în atmosferă, mai puțin
proporția absorbită în atmosferă. Prin urmare, nu este simplu pentru a
calcula nivelul solului insolație din satelit măsurători fără suplimentare
de măsurare și modelare. Cu toate acestea, masuratorile prin satelit și
hărți sunt de mare importanță, mai ales atunci când calibrat cu
încredere la nivelul solului a datelor meteorologice.
§2.8.5 Focusable fascicul de radiații și Indicele de Claritate
Așa cum sa explicat în §2.3, poate fi focalizat fasciculul componentă de intrare
radiații depinde predominant de nebulozitatea și gradul de prăfuire al
atmosferei. Efectul se referă la Claritate Indicele K
T

, care este
raportul de iradiere pe o suprafață orizontală într-o perioadă (de obicei, o medie
de peste poate o zi sau o lună) la iradiere primite în paralel
extra-terestre de suprafață în aceeași perioadă:
K
T
≈H
h
/
H
0

(2.19)
Chiar și cu un cer acoperit de nori, extra-terestre expunere la soare este redus prin
împrăștiere și aerosoli de absorbție, astfel încât, chiar cu aer–raportul de masă m = 1 (a se vedea
Ecuația (2.15)) o valoare instantanee de K
T

≈ 0.8. Acest lucru implică faptul că chiar și


cu un complet cer acoperit de nori, acolo este difuze semnificative de radiații. Fig. 2.17
este un teren de oră fracțiune de nivelul solului difuze de iradiere la un total de
iradiere împotriva Claritate Index. Din aceste date, putem concluziona
că:

difuze de iradiere este mereu prezent, chiar și cu cer acoperit de nori;

minim difuze fracțiune este de aproximativ 16 la 20% (care nu poate fi
concentrat);

concentrat dispozitive necesită climă, cu o proporție mare de zile
complet senine.
Rețineți că axat sisteme care urmăresc Soarele colecta nu orizontale
fascicul de component H
bh
dar cea mai mare fază normală componentă H
b
*.
§2.8.6 Efect de colector de înclinare
Fascicul de radiație solară măsurată pe un plan (1) poate fi transformat pentru
care a primit pe un alt plan (2). Acest lucru este deosebit de important pentru
transformarea datelor de la planul orizontal la un plan înclinat cu ajutorul Ecuației
(2.8). Prin urmare, pentru grinda componenta:
G
1b
/ cos
θ
1
= G
2b
/ cos
θ
2
= G
b
*
(2.20)
TWIDELL PAGINARE.indb 60
01/12/2014 11:36
§2.8-Ul de estimare a radiației solare
61
Radiația difuză, cu toate acestea, nu poate fi transformat de la un plan la
altul prin astfel de analiza simplă. Motivele sunt după cum urmează:

radiația difuză poate fi independent de nori direcție (izotrop) sau
altfel (anizotrope);

pentru planuri înclinate, punctul de vedere este parțial senin și, parțial, sol, etc., deci,
punctul de vedere factorilor sunt necesare pentru fiecare componentă de vedere;

obiectele din jur pot reflecta atât fascicul și difuze iradiere pe
avionul de interes.
Duffie și Beckman (2006) discuta aceste efecte în detaliu cu asortate
ecuații empirice și diagrame din literatura de specialitate. De exemplu,
Fig. 2.18 prezintă variația zilnică estimată de radiații pe diverse
pante în funcție de perioada din an, la o latitudine de 45°N, și cu
Claritate Indicele K
T

= 0.5. Rețineți că la această latitudine, media de


expunere la soare pe o verticală spre Soare suprafață variază extrem de puțin cu sezonul,
și în timpul iernii depășește 10MJm
-2
zi
–am

. Acest lucru este de două ori expunere la soare pe


o suprafață orizontală în timpul iernii și poate oferi importante de căldură prin
ferestre pentru clădiri; astfel de efecte sunt vitale pentru solară pasivă clădiri la
latitudini mai mari și pentru unele active sisteme de încălzire (§16.4).
Claritate indicele K
T
Difuze fracțiune k
d
0.0
0.2
0.4
0.6
0.8
0.0
0.2
0.4
0.6
0.8
1.0
Fig. 2.17
Fracția difuză iradiere a complotat împotriva Claritate Index pentru o gamă largă de
detalii date de teren.
Sursa: Adaptat de la C. P. Jacovides, F. S. Tymvios, V. D. Assimakopoulos și N. A. Kaltsounides
(2006), 'studiul Comparativ al diferitelor corelații în estimarea detalii difuze fracțiune din solară globală
radiații', Energia din surse Regenerabile, 31, 2492-2504.
TWIDELL PAGINARE.indb 61
01/12/2014 11:36
62
Radiația solară și efectul de seră
20
10
0
Jan. Feb. Mar.
β = 0
°
30
°
45
°
60
°
90
°
φ = 45
°
K
T
= 0.50
δ = 0
°
Apr. Mai Iun. Jul. Aug. Sep. Oct. Nov. Dec.
H
/MJ m
-2
zi
-1
Fig. 2.18
Variația estimată medie zilnică de expunere la soare H pe o suprafață la diferite pante,
b, în calitate de
funcție de perioada din an. Pentru latitudinea de 45°N, cu K
T
= 0.50,
g = 0°, și masă de reflexie
0.20.
Sursa: De la Duffie și Beckman (2006) (cu permisiunea de John Wiley & Sons Inc.).
§2.9 EFECTUL DE SERĂ ȘI SCHIMBĂRILE CLIMATICE
§2.9.1 echilibru Radiativ al Pământului
Dacă raza Pământului este R, cu medie de albedo
ρ
0
și extra - terestre
radiația solară (constanta solara) G
0
, apoi a primit putere în
Pământ cu atmosfera sa este
πR
2
(1
-
ρ
0
) G
0
, deoarece solar beam "vede"
Pământul ca un disc de raza R. (Grosimea atmosferei este
<<R
și nu sunt semnificative aici.) La echilibru termic, acesta a beneficiat de unde scurte
de putere este echilibrat de longwave putere radiată în spațiu din
sferice a Pământului și a atmosferei sale. Acest flux radiat este proporțională
cu puterea a patra de temperatura absolută T (a se vedea §R3.5). Astfel, cu
Pământ albedo
ρ
0
= 0.3, emisivitate
e = 1, Stefan-Boltzmann constanta s și
temperatura medie T
e
cum s-a observat din spațiu,
nR
2
(1
-
ρ
0
)G
0
=4
nR
2
sT
e
4
(2.21)
și, prin urmare,
T
e
≈ 255 K (de exemplu, T
e

≈ -18°C) (vezi Problema 2.1). Această


temperatură este radiația efectivă temperatura din partea superioară a atmosferei,
care este sursa de ieșire longwave radiații.
TWIDELL PAGINARE.indb 62
01/12/2014 11:36
§2.9 efectul de Seră și schimbările climatice
63
Astfel, lungă de radiații de pe Pământ și atmosfera a
aproximativ distribuția spectrală a unui corp negru la 250 K. după Cum
este indicat în Fig. 2.13, ieșire radiație apare într-o lungime de undă de bandă
între 7 și 15
mm, cu un vârf la aproximativ 10 mm (în conformitate cu
legea lui Wien, §R3.5). Rețineți că cele de mai sus calcul nu trebuie să
implice temperatura de pe Pământ suprafețe și atmosfera inferioară.
Reiese din Fig. 2.13 care o clară distincție poate fi făcută
între distribuția spectrală a radiațiilor Soarelui (unde scurte) și
că, din sursele termale de Pământ și atmosferă
(lungă). Infraroșu longwave fluxurile de la suprafața Pământului sunt
ei înșiși mari și complexe. Atmosfera radiaza de la această suprafață
, precum și în sus și în spațiu. Când măsurarea radiațiilor, sau atunci când
determinarea bilanțului energetic de o suprafață de teren sau un dispozitiv, este
extrem de important să fie conștienți de infrarosu invizibil fluxuri în
mediu, care pot fi
~1 kWm
-2
.
§2.9.2 efectul De seră, radiativ forțând,
și schimbările climatice
Pământului suprafața medie temperatură de aproximativ 14°C este de aproximativ 30°C
mai mult decât temperatura din atmosferă. În efect,
atmosfera acționează ca o infraroșu 'patura', din cauza anumitor gaze și
vapori de apă în a absorbi longwave radiații (vezi Caseta 2.1). Acest infraroșu
absorbție se produce atât cu radiația solară în timpul zilei și cu
ieșire radiație de căldură în mod continuu; efectul total produce o mai calde
suprafața Pământului decât în caz contrar. Această creștere în temperatură a suprafeței
(relativ la ceea ce ar fi fără atmosferă) se numește
verdecasa efect, pentru că sticla unei sere horticole (un
efect de seră), de asemenea, (i) absoarbe radiațiile infraroșii, inclusiv cele emise de
obiecte în interiorul serei pentru 24 de ore pe zi; și (ii) permite
primite radiația solară de unde scurte pentru a fi transmis în timpul zilei
(vezi Fig. R3.12). Schimbarea în radiativ net flux de energie, pentru că de
sticlă menține temperatura în interiorul cu efect de seră de mai sus ambiantă
temperatura de afara, care este scopul principal al agricole
sere în mijloc și mai mare latitudine de țări și de sere
clădiri adiacente. (În horticole caz, temperatura este în continuare
crescut de la cabina reduce naturale și vântul-au forțat
convecție pierderea de căldură.)
Fără efectul de seră, pe Pământ mai mult de apă ar fi de gheață,
fotosintetice ratele ar fi mult mai puțin și viața ar fi profund
diferite. Gazele responsabile, în special dioxid de carbon (CO
2
), protoxid de azot
oxid (N
2
O) și metan (CH
4

), sunt numite gaze cu efect de seră (Ges).


Efectul de seră este o caracteristica naturala a Pământului și a
atmosferei, strâns legate de stabilit proceselor ecologice. În
ultimii 200 de ani, mai ales, a omenirii industrii și practici agricole
TWIDELL PAGINARE.indb 63
01/12/2014 11:36
64
Radiația solară și efectul de seră
au condus la schimbări semnificative în ratele de emisii de gaze cu efect de Seră, astfel încât
concentrațiile de gaze cu efect de Seră din atmosferă au atins niveluri
>~30%
mai mult decât cele înregistrate în trecut de 500.000 de ani. Acest lucru este
indus de omenire ('antropic') crește, și este menționată ca îmbunătățită
efect de seră. Utilizarea combustibililor fosili este o cauza majora de acest efect (a se vedea
Caseta 2.3), și pot fi ameliorate prin utilizarea de surse regenerabile sau a energiei nucleare
în loc. Această carte se ocupă cu energie din surse regenerabile.
Unele gaze cu efect de Seră contribuie mai mult decât alții să radiativ forțând-a
îmbunătățit efect de seră. Esențiale fizica este că infraroșu
radiații este absorbită atunci când radiația electromagnetică rezoneaza cu
cele naturale vibrații mecanice ale moleculelor. Cele mai complexe
sunt moleculele, mai multe moduri de vibrație și cu atât mai mare
probabilitatea de absorbție la un anumit radiații de frecvență. Astfel, 1 kg
de CH
4
(cinci atomi per moleculă) care se adaugă la atmosfera are la fel de mult
cu efect de seră impactul 21 kg de CO
2
(trei atomi per moleculă). Asta
comparația cu privire la CO
2
se numește Potențial de Încălzire Globală
(GWP); de exemplu, potențialul de încălzire globală al CO
2
este (prin definiție) 1.000, potențial de încălzire globală de CH
4
21. În mod similar potențial de încălzire globală de N
2

Oana este de 310, în timp ce faptul că de cele mai multe


hidrofluorocarburi (de exemplu, cele utilizate în frigidere) este de peste 1000. Măsurarea
de potențial de încălzire globală de emisiile antropice este complexă pentru că depinde
pe cantitatea de gaze deja prezente și durata lor de viață în
atmosferă. Numai gazele a căror molecule persistă în atmosferă
de zeci de ani sunt considerate a avea un semnificativ efect de seră. De
exemplu, gaz metan are un timp de înjumătățire
~12 ani, CO
2

~100 de ani (Forster și


Ramaswamy 2007). Fără A Aduce Atingere Fig. 2.13, vapori de apă, în general, este
listat ca un GES, deoarece moleculele sale trece în și din
atmosferă într-un timp relativ scurt cadru (
<~1 an). De Gwp citat aici
sunt cele pentru o perioadă de 25 de ani, așa cum este folosit în sensul de la Kyoto
Protocol (a se vedea Capitolul 17).
Astfel de perturbatii la Pământ sistemul radiațiilor sunt adesea
exprimate în termeni de radiativ forțând, adică eficiente creștere netă în
iluminare energetică totală (unde scurte plus longwave) vor provoca.
§2.9.3 schimbările Climatice: observații
Măsurători de gaze prinse în gheața polară arată fără echivoc faptul că
concentrația de gaze cu efect de seră din atmosferă a crescut
semnificativ de la Revoluția Industrială din secolul al 18-lea. Mai multe
informații recente, de asemenea, vine din măsurători directe de "curate" de aer
la posturi, cum ar fi Mauna Loa, în Hawaii și Cape Grim în Tasmania. De
exemplu, media globală a concentrației atmosferice de CO
2
a crescut
de la 280 ppm în anul 1800 la 380 ppm în 2005 (Fig. 2.19(a)), și este încă
în creștere (de trecere 396 ppm în 2013). Mostrele de gheață arată că la nici o altă
dată în ultimii 600.000 de ani a CO
2
concentrația a depășit 300
ppm; într-adevăr, acesta a refuzat să
~190 ppm în fiecare din cele șase epoci de gheață în timpul
TWIDELL PAGINARE.indb 64
01/12/2014 11:36
§2.9 efectul de Seră și schimbările climatice
65
acea perioadă. De aproximativ 8000 de ani în urmă, prin intermediul a aproximativ 200 de ani în urmă,
acolo a fost relativ constant echilibrul în fluxurile de CO
2
la atmosfera
de teren (plante și animale) și mare, și vice-versa, astfel încât CO
2

concentrația în atmosferă păstrate în aproximativ 20 ppm de o


valoare de aproximativ 280 de ppm. Ice core înregistrările indică faptul că clima Pământului
a fost, de asemenea, relativ stabile de-a lungul acestei perioade, care, probabil, are profunde
implicații pentru dezvoltarea civilizațiilor.
CASETA 2.2 UNITĂȚI DE CONCENTRAȚIE A GAZULUI
Concentrația de gaze se măsoară în părți per milion (ppm), deci o concentrație de 300 ppm CO
2
înseamnă 300 de molecule de CO
2
pe de milioane de molecule de gaz din atmosferă (fără vapori de apă).
Totalul "eficace" cantitatea de gaze cu efect de Seră în atmosferă este de multe ori exprimată în ppm CO
2
echivalent
(CO
2
-eq). Astfel 490 ppm CO
2
-eq înseamnă o concentrație de gaze cu efect de Seră, care se combină pentru a produce același
suma de încălzire (radiativ forțând) ca 490 ppm de CO
2
singur ar fi făcut. Aceasta este calculată prin
ponderarea concentrația fiecărui gaz de GWP (a se vedea §2.9.2) și însumarea acestora.
IPCC autoritate de revizuire (2007) estimează că creșterea
concentrațiilor de GES între anii 1750 și 2000 cauzate
radiativ forțând (cu minus) din 2.5 Wm
-2

. Acest pozitive forțându-a fost


parțial compensată de alți factori, de exemplu, o creștere în
antropic reflectorizante particule de aerosoli în atmosferă. De la acest lucru și alte
studii, IPCC concluziona că CO
2
este dominantă antropice
gaze cu efect de seră, și că cele mai multe dintre creșterea CO
2
în atmosferă
se datorează activității umane (a se vedea Caseta 2.3). IPCC găsi că CO
2
este responsabil
pentru
~60% din radiativ forțând datorită gaze cu efect de Seră, urmată de CH
4
la
~20%.
Pozitiv radiativ forțând provoacă o creștere a temperaturii la
suprafața Pământului, adică încălzirea globală. Temperatura medie anuală a
crescut considerabil în ultimii 100 de ani, la aproape toate
observarea stațiile de pe uscat și pe mare. (Luând medii anuale ajută să
statistic descoperi tendința pe termen lung de zi cu zi și variabilitatea sezonieră.)
Fig. 2.19(b) este un grafic al Temperaturii medii Globale la Suprafață (GMST) pentru
ultimii 100 de ani. (Eficient anual GMST este media pentru toate marile
respectarea stații, în funcție de zona care fiecare se servește.) Rețineți că
rata de creștere a GMST a crescut în ultimele decenii, în
răspuns la creșterea globală utilizarea combustibililor fosili.
Creșterea în GMST este un aspect al schimbărilor climatice, care se referă
să tendințe sau alte schimbări sistematice de-a lungul perioadelor
>~30 de ani, fie
starea medie a climatului sau variabilitate (de exemplu, evenimente extreme).
Analiza indică faptul că creșterea regională temperatura poate fi mai mare
la latitudini mari (a se vedea Problema 2.9 pe efectul albedo). Un astfel de efect este
evident în accelerarea scădere rapidă în gradul de gheata Arctica,
mai ales în ultimul deceniu (Fig. 2.19(c)).
TWIDELL PAGINARE.indb 65
01/12/2014 11:36
66
Radiația solară și efectul de seră
Fig. 2.19
Observațiile de gaze cu efect de Seră și efectul lor fizice. (o) a Crescut CO
2

în atmosferă (1800-2005). Datele din 1958 au fost


măsurate direct din atmosferă; datele anterioare sunt din carotele de gheață. (b) Creșterea temperaturii medii globale la suprafață
(1850-2012). Diferitele curbe și eroare trupa reflecta ușor diferite opțiuni de stații și ponderea acestora să includ
în media globală, dar toate includ măsurători pe teren și ocean. (c) a Scăzut gradul de gheata Arctica în
septembrie, de la 1979-2012 (adică minim anual măsură).
Surse: (a) Adaptat de la IPCC WG1 (2007, Cifrele SPM.1); (b) OMM (2013); (c) date de la US National Snow and Ice data Center, cu
proprie a autorului extrapolarea
.
(a)
1800
An
2000
300
350
400
CO
2
concent
ra
tion (ppm
)
1850
1900
1950
2000
An
-0.8
-0.6
-0.4
-0.2
0
0.2
0.4
0.6
(b)
Temperatura medie globală anomalie (°C)
Met office hadley centre și climatice
unitatea de cercetare
NOAA National climatic data center
NASA institutul Goddard pentru studii spațiale
0
1
2
3
4
5
6
7
8
9
(c)
1970
1980
1990
2000
2010
2020
Mi
llion
mp
. m
CASETA 2.3 DE CE ȘTIM CĂ RECENTELE CREȘTERI ÎN CO
2

ȘI TEMPERATURA SUNT DATORATE


ACTIVITĂȚII UMANE (ANTROPICE)
CO
2
Izotopice dovezile arată în mod clar că creșterea recentă în atmosferică CO
2
concentrația este cauzată de
activitățile umane – în special arderea combustibililor fosili.
De Carbon este găsit cu trei izotopi: C
12
este dominantă (98.9%), C
13
(1%), și C
14
(numai 1 parte la 10
12
).
Astfel cantitative de identificare este bine la sensibilitatea de spectrometrie de masă. C
12
și C
13
sunt stabile
izotopi, dar C
14
se descompune la azot cu un timp de înjumătățire de 5700 de ani, așa cum se formează în mod continuu de la
azotul atmosferic de către razele cosmice și, dacă se întâmplă, prin testelor cu arme nucleare.
În fotosintetice difuzie, plantele absorb preferențial bricheta izotop C
12
, astfel încât raportul dintre C
13
/C
12
este
redus în vegetație și, prin urmare, în combustibilii fosili, în comparație cu raportul din atmosferă. Peste
TWIDELL PAGINARE.indb 66
01/12/2014 11:36
§2.9 efectul de Seră și schimbările climatice
67
Ca atmosfera se încălzește în contact cu oceanele, acceptă mai mulți
vapori de apă (a se vedea Fig. 4.3); prin urmare, intensitatea precipitațiilor crește. Crescut
de evaporare din oceanele calde (T
> 28°C) favorizează cicloane tropicale, care
poate fi, prin urmare, de așteptat să crească în intensitate.
§2.9.4 schimbările Climatice: proiecții, efecte, și de atenuare
Autoritate studii prezic că, dacă arderea combustibililor fosili continuă la
curent sau ratele au crescut, schimbările climatice vor deveni mult mai severe
până în 2050 și ulterior, cu dire de mediu și de consecințele sociale
(IPCC Synthesis 2007). Aceste proiecții și efecte sunt prezentate în
Capitolul 17, ca necesitatea de a atenua umane induse de schimbările climatice
și, astfel, pentru a minimiza aceste consecințe este una dintre cele instituționale majore
și de factori sociali încurajează înlocuirea sursele de combustibili fosili
(care emit cantități mari de CO
2
) de resurse regenerabile de energie
(care nu).
vârsta Pământului, atmosferice de CO
2

s-a format: (i) la descompunerea și arderea materialului vegetal


în biomasă și combustibili fosili; și (ii) din vulcanice și alte emisii din subteran Pământ. În cele
din urmă nu există nici preferențiale de creștere în proporție de C
12
și, prin urmare, nici o schimbare în C
13
/C
12
raport.
Cu toate acestea, raportul dintre C
13
pentru C
12
în atmosfera a fost în scădere, arată că suplimentare C
12
provine din arderea combustibililor fosili și pădure de ardere.
C
14
nu este prezent în combustibili fosili datorită relativ scurt de înjumătățire. Înainte de a atmosferice testarea
arme nucleare, scade din valoarea relativă a C
14
a arătat că a crescut C
12

a avut loc de la fosile


de combustibil de carbon se adaugă la atmosfera. În plus, concentrația de oxigen din atmosferă a
scăzut, în timp ce CO
2

de concentrare a crescut, deoarece oxigenul este epuizat ca combustibilii fosili sunt arși
(IPCC 2007; Houghton 2009). Similare analize de izotopi de carbon este folosit pentru a studia emisiile de metan
în atmosferă de la biologic și surse fosile.
Temperatura
Dovezile care leagă cele observate încălzirea globală la un antropice creștere în Seră, mai degrabă
decât la diverse "natural" forcings (cum ar fi energia solară variabilitatea și vulcani), este mai puțin directă decât cea de
origine antropică a crește în CO
2

. La fizica de bază stabilite în Caseta 2.1 este un puternic indicator, dar
cea mai convingătoare dovadă vine de modele climatice globale. Aceste modele numeric urmați
de transport de masă, energie și alte variabile-cheie de-a lungul timpului într-un 3-D grilă reprezintă atmosfera,
cu o atenție specială acordată interacțiunea dintre atmosferă și oceane. Pentru studii climatice, de
modele sunt rulate înainte pe perioade mult mai lungi decât câteva zile pentru care acestea sunt conduse de vreme
de prognoză.
În esență, simulări de o gama larga de modele, începând cu anul 1900, care includ atât
naturale și umane forcings, urmări creșterea observată de
~0,7°C în GMST de c.1950 (Fig. 2.19(b)),
dar aceleași modele "proiect" o scădere de
~0.2°C în cazul în care numai naturale forcings sunt incluse. Pentru mai
multe
discuții de atribuire a încălzirii globale la influența omului, a se vedea IPCC (2007, ch. 9) sau Houghton
(2009). IPCC (2013) susțin această cauzalitate în mai.
TWIDELL PAGINARE.indb 67
01/12/2014 11:36
68
Radiația solară și efectul de seră
REZUMAT CAPITOL
Radiația solară ajunge la suprafața Pământului la un maxim de densitate a fluxului de circa 1,0 kW/m
2
într-o lungime de undă
trupa între 0,3 și 2,5
mm, care include regiunea vizibilă de ~0,4-0,7 mm. Pentru zonele locuite,
acest flux variază de la aproximativ 3 la 30 MJ m
-2
zi
–am

, în funcție de loc, timp și vreme. Spectrale


de distribuție este determinată de 6000K temperatura de suprafață a Soarelui, deci este un flux de energie foarte
mare termodinamice de calitate.
Cei mai importanți factori sunt rezumate în Fig. 2.7 (de zi cu zi de expunere la soare pe o suprafață orizontală H
h
),
Fig. 2.15 (spectrul solar), și Fig. 2.18 (efect de înclinare).
Acolo sunt "globale" baze de date de precizie măsurători meteorologice de radiațiile solare, dar
acestea sunt în mare parte doar de Sec
h.

Răspândirea de măsurare site-uri este neregulat, astfel încât observațiile prin satelit au
un mare potențial. Mai ieftin instrumente (de exemplu, cele bazate pe celule solare fotovoltaice) sunt utile pentru domeniul
de aplicații și pentru monitorizarea unui dispozitiv de performanță relativă.
Geometrice formule calcula cu precizie efectul de înclinare pe iradierea cu fascicul (de exemplu, direct de la
Soare), dar estimarea radiații difuze (componenta împrăștiate prin nori, etc.) este incert.
Pământul emite longwave radiații (
~10
mm) pentru a menține echilibrul termic cu intrare solare de
unde scurte de iradiere. Cu efect de seră gaze în atmosferă absorb mult din această lungă de radiații,
păstrând astfel Pământul mai cald decât ar fi altfel. Umane (antropice) de activitate (în special
combustibili fosili) a crescut cantitatea de astfel de gaze în atmosferă, astfel măsurabil
creșterea temperaturii medii a suprafeței Pământului. Acesta este un simptom mai general clima
se schimba.
ÎNTREBĂRI RAPIDE
Notă: Răspunsurile sunt în textul secțiunile relevante din acest capitol, sau
poate fi ușor dedus din ele.
1
Ce este aproximativă densitatea fluxului de radiație solară (radiație)
în Wm
-2
pe un colector cu fața la Soare de pe suprafața Pământului pe o
zi însorită? Aproximativ, ce procent din această radiație este vizibil
pentru ochii omului?
2
Spectrul solar este declarat a fi împărțit în trei regiuni. Numele
acestor regiuni și să explice semnificația lor.
3
Dacă toată radiația solară cu spectru este descris ca 'unde scurte',
ceea ce este 'lunga' de radiații și de unde vine?
4
Care este semnificația din atmosfera Pământului, având o
' fereastra unde scurte "și un " longwave fereastra'?
5
Distincția între faza lungă, difuză și radiația totală.
6
Explica pe scurt de ce este mult mai dificil de a prezice radiația difuză
decât fascicul de radiație.
7
Ce face un piranometru măsură? Ce este baza fizică de sale de
funcționare?
8
În mijlocul iernii, ceea ce este o expunere la soare pe o suprafață orizontală, la
latitudini de: (i) 18°? (ii) 56°? De ce în mijlocul verii? Ceea ce ar fi o
orientare corespunzătoare pentru un colector fix: (i) în Suva, Fiji (18°E); (ii) la
Glasgow, Scoția (56°N)?
TWIDELL PAGINARE.indb 68
01/12/2014 11:36
Probleme
69
9
Ce este efectul de seră și de ce este important?
10
De cât de mult a Temperaturii medii Globale la Suprafață
s-a schimbat în ultimii 50 de ani? Indica unele efecte fizice care
explica cel mai mult de această schimbare, și să dea dovezi justificative pentru
răspuns.
PROBLEME
2.1
(a) ia în Considerare Soare și Pământ să fie echivalentul a două sfere
în spațiu. Din datele prezentate mai jos, se calculează aproximativ
constanta solar în afara atmosferei Pământului (W/m
2
).
(b) ia în Considerare Pământ după cum este evident din spațiu (de exemplu, delimitată de
atmosfera) să fie un corp negru cu temperatura de suprafata
T. Calcula T. Cum suprafața Pământului temperatura T'
se referă la T și ce variabile de control T'?
Date
Diametrul soarelui 2 R
S
= 1.392
× 10
9
m
Pământ cu diametrul de 2 R
E
= 1.278
× 10
7
m
Soare–Pământ distanța L = 1.498
× 10
11

m
Soare echivalentul negru temperatura corpului = 5780 K.
2.2
Să presupunem că semnul convenții pentru
w (unghi oră) în §2.4.1,
și pentru
b (panta) și g (suprafata azimut) în §2.5.1 sunt corecte pentru
emisfera nordică. Prin luarea în considerare diagramele corespunzătoare
cazuri speciale (de exemplu, Fig. 2.8) să verifice faptul că aceste convenții sunt corecte
, de asemenea, pentru emisfera sudică (de exemplu, un nord-cu care se confruntă colector în
emisfera sudică a
b > 0, g = 180°).
2.3
La Suva (
f = -18°), la ora 9 a. m. pe data de 20, radiației măsurate
pe un plan orizontal a fost G
h
= 1.0 MJh
–am
m
-2
.
(a) să se Determine unghiul
θ
z
între fasciculul de radiații și
verticală, și, prin urmare, a găsi iradiere G* = (G
b
+ G
d
)*
măsurată în direcția fasciculului. (Presupunem că G
d
<< G
b,
ca
poate fi cazul pe o zi clar.)
(b) în aceleași ipoteze ca în (a), determina unghiul
θ
c
între fasciculul și un colector de pantă de 30° se confruntă din cauza
De nord. Prin urmare găsi iradiere G
c
pe colector.
(c) să Presupunem în schimb că difuze radiații G
d
este uniformă în
cer, și că G
dh
= ½G
th
. Acest lucru este realist pentru un cer
zi. Recalcularea G* și G
c
și comentarii cu privire la diferența
între aceste valori și cele obținute în (a) și (b).
TWIDELL PAGINARE.indb 69
01/12/2014 11:36
70
Radiația solară și efectul de seră
2.4
(a) Arată că pierderea de căldură radiativ de la o suprafață la tempera-
ratură T
1
la cer (mod eficient la temperatura T
s
) pot fi
scris ca:
P
r
=
O
1
es (T
1
4
-T
4
s
) (2.22)
(Care este, deriva Ecuația (C. 17) în Apendicele C din prima
principii de R3.)
(b) prin Urmare, arată că:
P
r
=O
1
h
r
(
T
1
-T
o
), cu
h
r
=
es (T
1
2
-T
2
s
)(
T
1
+T
s
)(
T
1
-T
s
)(
T
1
-T
o
) (2.23)
2.5
(a) Din (2.11) găsiți unghiul de oră de la răsărit (atunci când zenith
unghi
θ
z
= 90°). Prin urmare, arată că numărul de ore între
răsărit și apus de soare este dat de (2.7).
(b) Calculați lungimea zi de la mijlocul verii și mijlocul iernii
la latitudini de: (i) 12°; (ii) 60°.
2.6
(a) în Cazul în orbita Pământului erau circulare, apoi radiației pe un
plan orizontal în afara atmosferei ar fi:
G'
0h
=G
*
0
pentru că
θ
z
(2.24)
în cazul în care
G
*
0
este constanta solara.
Dacă
w
s

este unghiul de oră de la apusul soarelui (în grade), arată că


integrată de zi cu zi extra-terestre radiația pe o orizontală
de suprafață este:
H'
oh
=G
*
0
t
s
[sin
f sind + (180 / nw
s
) cos
f cosd sin w
s
] (2.25)
în cazul în care
t
s
este lungimea zi.
Notă: din Cauza ușoară elipticitate a Pământului pe orbită, la
extra-terestre radiația nu este
H'
oh
dar
H
oh
= [1
+ e' cos(360n / 365) H'
oh
(2.26)
în cazul în care
e' = 0.033 și n este numărul zilei (de exemplu, n
= 1 pentru 1
Ianuarie).
(b) Utilizarea (2.26) pentru a calcula H
oh
pentru
f = 48° în mijlocul verii și
mijlocul iernii.
Comparați răspunsurile dumneavoastră cu cer senin radiații dat în
Fig. 2.7.
2.7
Deriva (2.10), adică:
pentru că
θ = cosw cosd
de la primele principii. (Această formulă dă unghiul
θ între
beam și normala la o suprafață având azimut
g = 0, panta dreptei, b =
|latitudine|.)
TWIDELL PAGINARE.indb 70
01/12/2014 11:36
Probleme
71
Sugestie: pentru a Construi o (x, y, z) sistem de coordonate centrat pe
Pământul este centrul cu Polul Nord pe OZ și Soarele în avion
y = 0, și pentru a găsi direcția ambianta din diferite direcții.
Notă: derivarea din plin formula (2.8) este similară, dar
complicat. Vezi Coffari (1977) pentru detalii.
2.8
Este energia de ieșire longwave radiații de Pământ
, egală cu cea de intrare unde scurte radiații de la Soare?
De ce?
2.9
Albedo de gheață este de aproximativ 0,8 și de apă de mare de 0.2.
Dacă unele mare de gheață (de exemplu, gheață plutind pe apa de mare) se topește complet
în timpul verii, expunând astfel apa de dedesubt, ce efect
ar avea asupra (a) temperatura apei în scurt
termen; (b) (re)formarea de gheață în toamnă (toamna) și de iarnă?
Discuta modul în care aceste efecte contribuie la creșterea medie
a temperaturii de-a lungul ultimilor 50 de ani, fiind mai mare în Arctica
decât la tropice.
2.10
Aproximativ 70% din suprafața Pământului este acoperită de ocean, cu o adâncime medie
de aproximativ 4 km.
(a) în Cazul în care temperatura din întregul ocean a crescut cu 1
º

C,
estima de cât de mult nivelul mării ar putea crește din cauza termice
de expansiune. (Ia rază de Pământ
R
E
= 6.4 × 10
6
m, coeffi-
cient de dilatare termică a apei de mare
b = 3 ×10
-4
K
-1

.) A
observat creșterea nivelului mării de-a lungul ultimilor 50 de ani este de numai aproximativ
10 cm; explica de ce. (Indiciu: creșterea nivelului mării este de așteptat să continue
pentru zeci de ani, chiar dacă temperatura suprafeței se oprește în creștere.)
(b) explica pe Scurt de ce topirea gheții Arctice (Fig. 2.19(c))
nu a contribuit la creșterea globală a nivelului mării.
(c) stratul De gheata a Groenlandei are o adâncime medie
~0.5 km
și o zonă
~2 X 10
6
m
2
. Daca asta a fost tot pentru a se topi, de cât de mult
acest lucru ar ridica media nivelului mării, oceanului?
*2.11
(pentru discuție) țara Dumneavoastră (numim X) a depus cel puțin un raport
privind schimbările climatice, ca parte la Convenția-Cadru a ONU
privind schimbările Climatice. Toate aceste rapoarte sunt disponibile publicului la
cconusc.int și să includă un capitol privind impactul potențial al climei
schimbarea pe care țara. Ce v-X e cel mai recent raport evalua
ca principalele efecte ale schimbărilor climatice asupra X? Credeți că acest lucru
este o subestimare sau supraestimare a efectelor probabile în:
(i) 30 de ani?; (ii) 60 de ani? Sugestie: ia în Considerare în special
cantitățile relative de RE și de combustibili fosili pentru a fi utilizate în viitor
(a se vedea Caseta 17.1 și Cutie de 17.5).
TWIDELL PAGINARE.indb 71
01/12/2014 11:36
72
Radiația solară și efectul de seră
NOTE
1
Vizita http://en.wikipedia.org/wiki/Solar_time pentru o excelent 'user-friendly' descriere solare și civile în timp.
2
Vezi datelor NASA proiecte științifice la https://mynasadata.larc.nasa.gov/P18.html.
BIBLIOGRAFIE
General
Duffie, J. A. și Beckman, W. A. (2006, 3rd edn) Solar de Inginerie a Proceselor Termice, Wiley, New York.
Fundația text pentru serioasă analiză tehnică.
Monteith, J. L. și Unsworth, M. (2007, 3rd edn) Principii de Fizica Mediului, Academic Press, London.
Se aplică în special pentru cultură și de creștere a plantelor, animalelor și echilibrul termic. Include o descriere concisă a
mediul de radiații aproape de sol.
Geometrie
Coffari, E. (1977) 'Soarele și pe bolta cerească, în Sayigh, A. A. M. (ed.), Energia Solară Inginerie, Academic
Press, London. Derivarea formule geometrice.
Date și modele de estimare a radiației
ASHRAE (2009) Handbook of Fundamentals, Societatea Americana de Încălzire și Refrigerare și Aer Condiționat
Ingineri. Include tabele de iradiere pentru calculele de energie în clădiri.
Badescu, V. (ed.) (2008) Modelarea Radiației Solare la Suprafața Pământului: progresele Recente, Springer-Verlag, New
York. Multi-autor compilare.
Davies, J. A. și Mackay, d. c. (1989) de Evaluare a selectat modele de estimare a radiației solare pe orizontală
de suprafață', a Energiei Solare, 43, 153-168.
Gueymard, C. A. (2004) 'Soare este totală și iluminare energetică spectrală de energie solară aplicații și radiația solară
modelelor, Energie Solară, 76, 423-453.
Myers, D. M., Emery, K. și Gueymard, C. (2004) 'Revizuirea și validarea iluminare energetică spectrală standarde de referință
pentru fotovoltaice de evaluare a performanței', Journal of Solar Engineering (ASME), 126, 567-574.
NASA (2003) http://science.nasa.gov/science-news/science-at-nasa/
2003/17jan_solcon/. Aștepta ulterioare
actualizări de la constanta solara pe acest site-ul NASA.
Renne, D., Perez, R., Zelenka, A., Whitlock, C. și DiPasquale, R. (1999) Utilizarea de vreme si clima de cercetare
sateliți pentru a estima resursele solare', Progresele în domeniul Energiei Solare, 13, 171.
Vignola, F., Harlan, P., Perez, R. și Kmiecik, M. (2007) Analiza prin satelit derivate fascicul și radiația solară globală
de date', a Energiei Solare, 81, 768-772.
Instrumente
Vignola, F., Michalsky, J. și Stoffel, T. (2012) Solare și Radiațiile Infraroșii Măsurători, CRC Press, Londra.
Manualul detaliază punctele forte și punctele slabe ale instrumentelor utilizate pentru a efectua astfel de solar și infraroșu
radiații măsurători.
TWIDELL PAGINARE.indb 72
01/12/2014 11:36
Bibliografie
73
Organizația Meteorologică mondială (1996) Ghid pentru Instrumente Meteorologice și Metode de Observare,
esp. ch. 7, 'Măsurarea radiațiilor'.
Efectul de seră și schimbările climatice
Forster, P. și Ramaswamy, V. (coord) (2007) Modificări în Componentele Atmosferice și în Radiativ Forțând,
ch. 2 al IPCC (2007).
Hammond, G. P. (2004) 'Inginerie durabilitate: termodinamică a sistemelor de energie și mediu',
revistă Internațională de Cercetare a Energiei, 28, 613-639. O grijuliu lucrarea de revizuire referitoare termodinamicii la
teorii de material și energie durabilă.
Houghton, J. T. (2009, 4 edn) Încălzirea Globală: La informare completă, Cambridge University Press, Cambridge.
Cont de clar pentru studenții de la știința relevante. Mai tehnice și mai plin de viață decât oficial raportul IPCC;
Houghton este un fost președinte al IPCC.
IPCC (grupul Interguvernamental de experți privind schimbările Climatice) (2007) schimbările Climatice 2007: științe fizice de
bază,
Cambridge University Press, Cambrfidge (rezumat disponibil online la www.ipcc.ch). IPCC este convocat de către
Națiunile Unite pentru a oferi o autoritate de revizuire cu privire la starea de cunoștințe științifice cu privire la schimbările
climatice.
IPCC a produce un raport de evaluare actualizat la fiecare șase ani. Acest volum este raportul IPCC -
Grupul de Lucru 1.
IPCC (2013) Grupul de Lucru 1 contribuția la schimbările Climatice 2013: științe fizice de bază: Rezumat pentru
factorii de decizie politică (disponibil online la www.ipcc.ch). Aceasta este prima parte a lansat al IPCC a Cincea Evaluare a
Raportului.
Sinteză al IPCC (2007) schimbările Climatice 2007: raport de Sinteză, grupul Interguvernamental privind schimbările Climatice/
Cambridge University Press, Cambridge. Sintetizează și integrează trei IPCC volume în
științe fizice, impactul și adaptarea la acestea, și de atenuare (disponibil online la www.ipcc.ch).
OMM (2013) OMM Declarație privind Statutul de Clima Globală în 2012, Organizația Meteorologică Mondială,
la Geneva. Aceasta este una dintre o serie anuală disponibile online la www.wmo.int. OMM publica, de asemenea, un Gaz cu
efect de Seră
Buletin de cel puțin anual.
Site-uri și reviste
NASA – cele mai bune informații actualizate de la http://solarsystem.nasa.gov/features/planets/Sun`.
www.astm.org – standard de referință spectrelor de radiație solară la AM0 și AM1.5.
Lumea Radiații data Center – ului susținut de OMM, baze de date măsurate zilnic de expunere la soare pentru 1000 de site-uri
din întreaga lume: wdrc.mgo.rssi.ru.
Fotovoltaice pe Sistemul de Informații Geografice – acoperă numai Europa și Africa, include hărți ale resurselor solare
și de 'optim de colectare unghi': http://re.jrc.ec.europa.eu/pvgis/.
Climat de Monitorizare prin Satelit Cerere Facilitate (date pe diferite radiații și cloud parametri; operațional din
2007, dar are unele date mai vechi în arhiva sa): www.cmsaf.eu/bvbw/appmanager/bvbw/cmsafInternet.
Grupul interguvernamental privind schimbările Climatice – sursa de autoritate privind schimbările climatice: a se vedea de
referință de mai sus. Toate
publicate rapoartele IPCC sunt disponibile pe site-ul asta: www.ipcc.ch.
NOAA – cele mai recente date privind CO
2
în atmosfera: www.esrl.noaa.gov/gmd/ccgg/trends/.
>
[NOI] Național de Date despre Zăpadă și Gheață Center – cele mai recente date privind Arctica și Antarctica ice acoperă:
www.nsidc.org.
TWIDELL PAGINARE.indb 73
01/12/2014 11:36
74
Radiația solară și efectul de seră
Evoluții majore în domeniul schimbărilor climatice știință sunt acoperite de către cele două mari academice generale reviste
științifice,
Natura si Stiinta, ambele din care ar trebui să fie disponibile în aproape fiecare bibliotecă universitară. Există, de asemenea,
multe
reviste de specialitate, inclusiv Natura schimbărilor Climatice, Clima Cercetare, Journal of Geophysical Research, Journal
de Clima, Jurnalul Internațional de Climatologie, etc. Pentru alte aspecte ale schimbărilor climatice, a se vedea bibliografia
de la Capitolul 17.
TWIDELL PAGINARE.indb 74
01/12/2014 11:36

Solare de încălzire a apei


CONȚINUTUL
Obiective de studiu
75
§3.1 Introducere
77
§3.2 Calculul de echilibru termic:
observații generale
79
§3.3 colectoarelor Plane
81
§3.3.1 Estimarea performanței
de o plat-colector
82
§3.3.2 Eficiența plat placa
colector
87
§3.4 Sisteme cu stocare separat
88
§3.4.1 sisteme Active cu
circulație forțată
88
§3.4.2 Sisteme cu thermosyphon
circulația
90
§3.5 Selectivă a suprafețelor
92
§3.5.1 Ideal
92
§3.5.2 Metal semiconductor
suprafața compozite
93
§3.5.3 Fabricarea de selectiv
suprafețele
95
§3.6 Evacuat colectoare
96
§3.7 și aparatură de monitorizare
99
§3.8 aspectele Sociale și de mediu
100
Rezumat capitol
101
Întrebări rapide
102
Probleme
102
Bibliografie
106
Caseta 3.1 temperatura de Referință
T
ref
pentru căldură circuit modeling
86
OBIECTIVE DE STUDIU

Aprecia nevoia de căldură interne și
comerciale situații.

Înțeleg de bază în proiectarea și amenajarea de
instalații solare pentru încălzirea apei.

Utilizarea analizei de încălzitoare de apă solare pentru a
motiva învățarea despre transferul de căldură.

Estimarea parametrilor de performanță solare
colectoare de la primele principii.

Cunoască principiile de absorbție selectivă
a suprafețelor.

Ia în considerare implicațiile practice de
tehnologie.

A stabili o bază pentru studiul în continuare a solar
aplicații.
CAPITOLUL

3
TWIDELL PAGINARE.indb 75
01/12/2014 11:36

www.shahrsazionline.com
76
Solare de încălzire a apei
LISTA DE FIGURI
3.1
(a) Un comun tip de incalzitor solar de apa în Australia. (b) O gospodărie solare de încălzire a apei colector
cu o separat rezervor de stocare de sub acoperiș.
78
3.2
Schiță diagrame de diferite solare de apă, colectoare, cu unele transferuri de căldură indicat.
79
3.3
Transferul de căldură de la radiația solară a unui fluid într-un colector.
80
3.4
(un) Singur-geamuri plate-placă colector (schematică); (b) circuit analogic de (o) utilizate în Muncit
Exemplu 3.1.
83
3.5
Circuitelor analogice pentru ecuația (3.18).
86
3.6
Tipic eficiența curbele de un singur geam colectoarelor plane.
88
3.7
Pe acoperiș colector solar ca prioritate de intrare pentru un sistem de apă caldă.
89
3.8
Colector și rezervor de stocare cu thermosyphon circulație.
91
3.9
Principiul de thermosyphon flux.
91
3.10 caracteristicile Spectrale ale diferitelor suprafețe.
93
3.11 fluxului de Căldură într-un singur tip de suprafață selectivă.
94
3.12 (a) Evacuat colector. (b) Circuitul analog de (- o). (c) Evacuați tub colector cu interne
heat-pipe si colector.
97
3.13 Termice rezistențele de Probleme 3.2.
102
3.14 reflexii Multiple între colector acoperire(s) și placa (pentru Problema 3.9).
104
3.15 secțiune Transversală a unui tub și placă colector (pentru Problema 3.10).
105
LISTA DE TABELE
3.1
Rezumat tipic de performanță pentru diferite tipuri de colectori
78
TWIDELL PAGINARE.indb 76
01/12/2014 11:36
§3.1 Introducere
77
§3.1 INTRODUCERE
O bază utilizarea energiei solare pentru încălzirea fluidelor de aer și apă. De
exemplu, case în frig și climă temperată de multe ori nevoie de aer încălzit
pentru confort, și în toate țările de apă caldă este benefică pentru personal și în
haine de spălat, precum și pentru alte scopuri casnice. Există sunt similare
nevoilor în afaceri, în industrie și agricultură. În consecință, având în vedere
național de aprovizionare cu energie, în marea BRITANIE, aproximativ 30% este folosit pentru astfel de
încălzire
în clădiri, și chiar și în Australia, cu un climat mai cald aproximativ 20%
este folosit pentru incalzire lichide, predominant apa, pentru a "scăzut" temperaturi
(
<100°C). Pentru sisteme de energie solară, dacă expunere la soare este absorbită și
utilizată fără mecanice semnificative de intrare (de exemplu, pentru pomparea sau scade),
sistemul solar este declarat a fi pasiv. Dacă căldura solară este colectat într-un
lichid, de obicei apă sau aer, care este apoi mutat de pompe sau ventilatoare pentru utilizare
sau stocare în altă parte, sistemul solar este declarat a fi activ.
Principiile generale de analiză și de încălzitoare de apă solare se aplică, de asemenea,
la multe alte sisteme care utilizează activ și pasiv mecanisme pentru a
absorbi energia solară ca de căldură (de exemplu, încălzitoare de aer, cultură uscătoare solare putere
turnuri, solare, cazane pentru distilarea apei, solar clădiri). Aceste alte
aplicații vor fi tratate în Capitolele 4 și 16.
Fabricarea de încălzitoare solare de apă a devenit un
industrie în mai multe țări, în special China, Australia, Germania,
Grecia, Israel, Brazilia și Japonia. Mai mult de 200 de milioane de gospodării acum
folosesc colectoare solare pentru apă caldă, în aceste și în multe alte țări, cu
mai mult de jumătate din China. Fig. 3.1 sisteme tipice pentru uz casnic
utilizare.
Partea principală a unui sistem de incalzire solar este colector, unde solare
radiația este absorbită și energia este transferată la lichid. Colectoare
luate în considerare în acest capitol nu se concentrează radiația solară cu oglinzi
sau lentile; ele sunt clasificate fie ca placă plană sau ca evacuate colectoare,
în contrast cu accent colectoare discutate în §4.7. Non-concentrându-se
colectoare absorbi atât fascicul și difuze de radiații, și, prin urmare, încă în
funcție, atunci când fasciculul de radiații este tăiat de cloud. Acest avantaj, împreună
cu ușurința de operare și favorabil cost, înseamnă că non-concentrându-se
colectoare sunt în general preferate pentru încălzire fluide la temperaturi mai puțin
decât aproximativ 80°C.
Fig. 3.2 prezinta diagrame schematice de mai multe tipuri de colectoare utilizate
pentru încălzire solară a apei, și în Tabelul 3.1 sunt prezentate câteva indicatorii lor
de performanță. Acest capitol se concentrează pe geamuri plate-placă și tuburi vidate
colectoare, deoarece acestea sunt comune în întreaga lume; în plus, ele permit
experimente practice în predarea și analiza transferului de căldură oferă o
stepby-step aprecierea fundamentale atât pentru active și pasive de
aplicații.
§3.2 și §3.3 demonstra cum pentru a estima performanța indica-
tor din Tabelul 3.1, folosind metode de Revizuire 3. §3.4 discută
TWIDELL PAGINARE.indb 77
01/12/2014 11:36
78
Solare de încălzire a apei
Fig. 3.1
o
Un tip comun de incalzitor solar de apa în Australia. La acoperită cu sticlă plană a plăcii de colector încălzește apa pentru izolat
rezervor de stocare plasat mai sus (apă caldă se ridică). Un back-up electric de încălzire în rezervorul este disponibil pentru rarele ocazii
când
solare de intrare este insuficientă.
Sursa: Autor foto.
b
O gospodărie solare de încălzire a apei colector cu o separat rezervor de stocare de sub acoperiș. Deosebit de frecvente în zonele cu climă
cu ierni geroase, cu primar lichid antigel circuit furnizarea de căldură printr-un schimbător de căldură pentru apa caldă
rezervorul de stocare. Acest special colector este de tuburi vidate tip.
Sursa: Foto de la www.greenenergynorthwales.com, folosit cu permisiune.
(a)
(b)
Tabelul 3.1
Rezumat tipic de performanță pentru diferite tipuri de colectori
Suprafata
Geam
Figura
r
pa
/m
2
KW
-1
T
p
(m)
/ ºC
Raport de preț și performanță
Negru
Nici unul
0.03
40
Folosit pentru piscine foarte ieftine pentru
de alimentare cu apă caldă
Negru
Single
3.2(a)
0.13
96
Mai ieftin
Negru
Dublu
0.22
140
Creșterea prețurilor mici pentru temperaturi mai mari
Selectiv
Single
3.2 litera(c)
0.40
240
Important de ameliorare la moderată cost suplimentar
Selectiv
Dublu
0.45
270
De îndoielnic beneficii suplimentare
Selectiv
Evacuați
tub
3.2 litera(d)
0.40
300
Important pentru temperatură mai mare, dar mai
scump
Note
1
r
pa

este rezistența la pierderile de căldură prin partea de sus a colectorului pentru T


p
= 90°C, T
o

= 20°C, u = 5 m s
-1
.
2
T
p
(m)

este stagnarea temperatura pentru care o iradiere de 750 W m


-2
doar soldurile de căldură pierdut prin r
pa

. Deoarece
'stagnare' presupune zero, debitul real temperatura de lucru este în mod substanțial mai mică decât aceasta (a se vedea textul).
3
Un colector este eficientă dacă pierderile de căldură sunt mici (de exemplu, mare r
pa

) și dacă temperatura apei este potrivit (adică nu


trebuie să fie mare, pentru uz casnic). În general, cele mai eficiente colectoare mai mare T
p
(m)

.
4
Calculele de r
pa
și T
p
(m)

sunt în Exemple 3.1 și 3.4, și în Probleme 3.3, 3.4 și 3.5.


integrarea de colectori în solare complete sisteme de încălzire a apei. §3.5
și §3.6 examina două rafinațiile, și anume utilizarea unor suprafețe
și evacuat colecționari. §3.8 concluzionează prin conturarea unor socială și
de mediu aspecte de acest benigne tehnologie.
TWIDELL PAGINARE.indb 78
01/12/2014 11:36
§3.2 Calculul de echilibru termic: observații generale
79
Fig. 3.2
Schiță diagrame de diferite solare de apă, colectoare, cu unele transferuri de căldură indicat.
o
De bază incalzitor solar de apa, apa tuburi sudate la placa si tot negru mat, capac de sticlă și inferioare cabina de bine izolate.
Convecție internă pierdere de aer de circulație nu sunt afișate.
b
Transfer de căldură îmbunătățit de la placa de sus a suprafeței de apă cu un inundat placa.
c
Eficiență îmbunătățită cu o suprafață selectivă pe placa care reduce pierderile de căldură cu mai puțin de emisii de longwave infraroșu
radiații.
d
Geam exterior tub cu vid în jurul absorbant pentru a anula pierderea de căldură de interne convecție.
(b)
Negru inundat placa
(transfer de căldură îmbunătățit)
(c)
Selectiv suprafață
(negru de absorbție pentru unde scurte,
strălucitoare non-emițătoare pentru longwave)
(d)
Tub de sticlă
Vid
Tub interior cu suprafață selectivă
conțin fluid de transfer de căldură
de exemplu, apa
Izolare
Expunere la soare
(unde scurte
transmise)
Capac de sticlă (absoarbe longwave
radiații infraroșii și re-emite)
(a)
Tuburi cu apă
Farfurie neagra (absoarbe `radiația solară de unde scurte,
emite longwave radiații)
§3.2 CALCULUL DE ECHILIBRU TERMIC: GENERAL
OBSERVAȚII
Analiza noastră utilizează termeni și concepte continuarea de la Capitolul 2 și
cu transfer de căldură teoria acoperit de Revizuire 3. Analiza noastră folosește căldura
circuit teoria' în modul de 'circuit electric teorie, pentru că acest
TWIDELL PAGINARE.indb 79
01/12/2014 11:36
80
Solare de încălzire a apei
se referă direct la bazele de conducție, convecție, radiație, și
de transport de masă. Mai mult decât atât, diagrame de circuit ajuta la clarificarea fizică
a proceselor implicate.
Toate colectoarele solare includ o suprafață de absorbție, numit aici pe farfurie.
În Fig. 3.3 fluxul radiant izbitoare placa este
τ
cov
O
p
G, unde G este
radiației pe colector, O
p
este expusă zonă a plăcii, și
τ
cov

este
transmitanța orice capac transparent ce poate fi folosit pentru a proteja
placa de vânt (de exemplu, Fig. 3.2). Doar o fracțiune
α
p

din acest flux este


de fapt absorbit. Deoarece placa este mai fierbinte decât împrejurimile sale, se pierde
căldură la o rată (T
p
– T
o
)/R
Am
, unde R
Am
este rezistența termică pierderi de căldură
de la placa (temperatura T
p
) de mediul exterior (temperatura
T
o
). Pe net flux de căldură în farfurie, prin urmare, pot fi analizate de către oricare dintre
următoarele trei ecuații echivalente:
P
net
=
τ
cov
α
p
O
p
G
− [(T
p
− T
o
)/ R
Am
] (3.1)
P
net
= O
p
{
τ
cov
α
p
G
− U
Am
(T
p
− T
o
) (3.2)
P
net
=
η
sp
O
p
G (3.3)
în cazul în care
η
sp
este eficienței de captare (
<1) și U
Am
= 1/(R
Am
O
p
) este generală a
pierderilor de căldură coeficientul'. Fie a (3.1) sau (3.2) este menționată ca
HottelWhillier-Fericire ecuație.
Echivalarea (3.2) și (3.3):
η
sp
=
τ
cov
α
p
− U
Am
(T
p
− T
o
)/G (3.4)
Folosind (3.4), care catalogheaza parametrii pentru un anumit colector sunt
determinată experimental prin trasarea determinate empiric
colector de eficiență în funcție de diferența de temperatură, așa cum se arată în
Fig. 3.6.
Este evident din Hottel-Whillier-Fericire ecuația (3.1) sau (3.2) că
eficiența de încălzire solară a apei depinde de un set de parametri
legate de transmisie, reflexie și absorbție a radiației solare,
și un alt set de parametri legate de păstrarea și mișcarea de
căldură. În acest text vom lua în considerare fiecare proces în mod independent, pentru a forma un total
Fig. 3.3
Transferul de căldură de la radiația solară a unui fluid într-un colector.
G
Capac
Placa
G
GA
p
P
net
α
p
τ
cov
(T
p
– T
o
)/R
Am
τ
cov
P
u
=
η
pf
P
net
Lichid
TWIDELL PAGINARE.indb 80
01/12/2014 11:36
§3.3 colectoarelor Plane
81
de căldură în circuitul de analiză. Cu toate acestea, inginerie tradiționale consideră, de asemenea,
sistem fizic ca o "cutie neagră", pentru a fi analizate din punct de vedere funcțional. Pentru acest lucru,
practic inginerie urmărește 'non-dimensional factori de scală' ca grupuri
de parametri care, ca grup, sunt independente de anumite
circumstanțe; a 'f-chart' metoda este un bine-a folosit de exemplu (a se vedea Duffie și
Beckman (2006) sau Brinkworth (2001)). Cu toate acestea, utilizarea unor astfel de 'catalogheaza
parametru' metode obscure fundamentale de transfer de căldură
procese, care sunt evidente în căldură în circuitul de analiza vom folosi.
În general, doar o fracțiune
η
pf
de P
net
este transferat la fluid la tempera-
ratură T
f
. Într-un bine conceput colector de diferența de temperatură dintre
placa și lichidul ar trebui să fie mici, și deci eficiența transferului
η
pf
este doar puțin mai mică decât 1. În practică,
η
pf

pentru întregul sistem variază


considerabil datorită diferențelor în proiectarea, amplasarea și întreținerea, dar
aici avem în vedere doar colector.
Utile putere de ieșire din colector este:
P
u
=
η
pf
P
net
(3.5)
= mc(dT
f
/dt) dacă o masă statică m de lichid este încălzit
(3.6)
= m.c(T
2
− T
1
) dacă o masă m. curge prin colector
în unitatea de timp
(3.7)
În (3.7), T
1
este temperatura fluidului la intrarea în colector și T
2
frunze de colector.
Aceste ecuații sunt cel mai frecvent utilizate pentru a determina ieșire P
u
pentru o anumită radiație G. Parametrii O,
τ, α comerciale colectoare
sunt de obicei specificate, lăsând R
Am
pentru a fi calculate utilizând metodele de
Revizuire 3. Deși T
p
depinde de P
u

rezonabil de prima estimare poate fi


făcută și apoi rafinat mai târziu, dacă este necesar. Acest lucru este ilustrat în următoarea
secțiune.
§3.3 COLECTOARELOR PLANE
Multe solară a apei-sisteme de încălzire în producția comercială se bazează
pe o plat-colector. Placa și tubul colector (Fig. 3.2(a)), frecvente
încă din anii 1960, este cel mai simplu tip și încă în utilizarea pe scară largă. La
apă este limitat în paralel tuburi sudate sau altfel aderat la o
placă de metal. Este esențial să avem o minimă rezistență termică între
placă și tub, și peste placa dintre tuburi. De obicei
tubul de diametru este
~2 cm, tub spațiere ~20 cm și placa gros-
ness
~0,3 cm. Placa și tuburi sunt fixate într-un cadru cu un pahar
de top și cu o grosime de izolare la părțile laterale și din spate; ansamblul
trebuie să fie complet etanș și să nu permită pătrunderea umezelii (pentru a
evita mucegai, coroziune, și în plus pierderea de căldură); 25 - 30 de ani garantat
de viață este comună.
TWIDELL PAGINARE.indb 81
01/12/2014 11:36
82
Solare de încălzire a apei
§3.3.1 Estimarea performanțelor de o plat-colector
Ecuațiile (3.6) și (3.7) sugerează două tipuri de parametri pentru a măsura
performanța unui colector: (i) stagnare temperatura T
p
(m)

este
temperatura statică a lichidului de umplere colector (de exemplu, în tuburi de o
farfurie si tub colector) și în echilibru termic cu pierderi; și (ii) nivelul
maxim de ieșire temperatura T
2
(m)
de lichid care curge prin colector la
standardizate ale ratei de m..
T
2
(m)
este întotdeauna mai mică decât T
p
(m)

și depinde în mare măsură de debitul de apă


rata m., asa ca ne vom concentra în primul rând pe stagnare temperatura,
ca (pentru standard de condiții externe) se mai caracterizează colector
distinct de sistem ca un întreg (vezi §3.4 pe toată sisteme
, inclusiv lichidul de stocare). Stagnarea de temperatură, de asemenea, oferă un
ghid important pentru gama de aplicații posibile de colector.
De exemplu, un colector cu o stagnare temperatura de 60ºC ar fi
adecvate pentru multe aplicații domestice, dar nu pentru o aplicație industrială
care este necesar de fierbere sau aproape de apa clocotita.
În Exemplul 3.1, estimăm colector de performanță de la primele
principii, folosind căldura circuit teoria de Revizuire 3 pentru a calcula cheia
parametri. Vă recomandăm să lucreze prin acest exemplu pas cu pas
în timp ce verificarea înțelegere a principiilor implicate. Fiecare pas
, nu este dificil, dar întreaga analiză devine complex. Ia-ti timp
si nu te grabi – încălzitoare de apă solare nu sunt dispozitive simple!
EXEMPLUL 3.1 CALCULA TEMPERATURA MAXIMA A APEI DINTR-O INSTALAȚIE SOLARĂ
DE ÎNCĂLZIRE A APEI
Un non-selectivă vopsite în negru plate-placă colector, 1.0 m
× 1,0 m în zonă, are un singur capac de sticlă 3.0 cm
de mai sus și izolare imediat mai jos de 10 cm grosime. Este expus la radiația solară G
= 750
Wm
-2
. Apa este fluidul de lucru la temperatura T
f

. De a face aproximări rezonabile și tratarea


sistem ca un singur obiect compozit, se calculează rezistența la pierderile de căldură din farfuria cu
lichid, și, prin urmare, temperatura maximă a apei atunci când debitul de apă este zero (adică stagnare
temperatura T
p
(m)
).
Date: transmisie de capac de sticlă
τ = 0.9; absorptance "la negru" placa α = 0.9; emisivitate de placa
și sticlă
e
p
=
e
g
= 0.9 pentru longwave radiații; conductivitatea termică a izolației k = 0.035 Wm
-1
K
-1

;
viteza vântului asupra capacul 5.0 m/s; temperatura aerului ambiant 20 ° C.
Soluție
Situația fizică este prezentată în Fig. 3.4(a) cu căldura circuit diagram (b). Ca un prim apropie ation, vom
presupune zero rezistență termică între placă de metal și lichidul, astfel încât placa de temperatură T
p
= T
f

,
temperatura lichid, care se presupune a fi uniform, deoarece debitul este zero, la stagnare
temperatura. Deoarece suprafața este non-selectivă, emisivitate de suprafață
e = α.
Analiza este de un electric analogic care tratează temperatura de tensiune și de energie termică ca și curent,
care trece prin rezistențele la solar de intrare pentru mediu la fix T
ref
. (A se vedea de Revizuire 3 pentru
mai multe detalii pe circuitul de analogie în general.)
TWIDELL PAGINARE.indb 82
01/12/2014 11:36
§3.3 colectoarelor Plane
83
În electrice analogice de căldură în circuitul din Fig. 3.4(b), G este simbolizat ca o sursă de curent continuu în
lichidul, reprezentat de primul nod la 'tensiune' T
f
. Fluid și placă sunt tratate ca și combinate.

La extremitatea din dreapta a circuitului, pierderea de căldură P
b
trece la mediul înconjurător prin conducție prin baza de
mare rezistență R
b
. (Pierderea de căldură printr-un uscat, izolat de bază, altele decât prin conducție este neglijabil.)
Fig. 3.4
(un) Singur-geamuri plate-placă colector (schematică); (b) circuit analogic de (o) cum este folosit în Exemplul 3.1.
Vântul
(a)
(b)
Gratuit
convecție
Forțat
convecție
Radiații
Radiații
Scurt
val
Longwave
radiații
pierderile
Convective
pierderile
u
G
G
τ
T
b
Baza
C
f
R
g
T
g
T
f
T
cer
T
o
T
ref
R
r, pg
R
r, ga
R
b
T
b
P
b
R
v, pg
R
v, ga
T
g
De sticlă (de sus)
T
f
Lichid
T
o
Ambient
T
cer
Cer
Baza de pierderi
T
p
TWIDELL PAGINARE.indb 83
01/12/2014 11:36
84
Solare de încălzire a apei

În centrul de circuit, în paralel pierderea de căldură trece de la lichid la mediul în trei etape.
În primul rând, prin convecție liberă și radiații în paralel la partea inferioară de capacul de sticlă. Al doilea, prin
geamul de conducție prin rezistenta R
g
. Al treilea, la mediul înconjurător prin convecție și radiație în
paralel.

Pe partea stângă a circuitului, capacitate C
f
reprezintă capacitatea de căldură de la lichidul fiind încălzit.
Circuitul analogic permite convecție din partea de sus a capacului fiind a aerului ambiant la temperatura T
o
,
pentru radiațiile din partea de sus a capacului fiind de la cer la temperatura T
cer
și pentru mici pierderi prin
izolare fiind său exterior temperatura T
b

. Aceste temperaturi diferite (cf. tensiuni) sunt menținute


în circuitul analogie de 'baterii', care menține temperatura necesară cu privire la o referință
de temperatură T
ref
mai puțin decât la alte temperaturi. T
ref
este analogul de pământ potențiale în energie electrică, așa cum este
a explicat în continuare în Caseta 3.1. De obicei, o alegere potrivită este T
ref
= 0ºC.
Aceste ipoteze implică dT
f
/dt
= 0 și
η
pf
= 1, deci (3.1), (3.5) și (3.6) se reduce la:
(T
p
(m)
− T
o
)/R
Am
= τα O
p
G (3.8)
Partea de jos rezistență este pur conductiv și calculat cu ușurință din (R3.10):
=
=

R
x
kA
(0.1 m)
(1.0 m )(0.035 Wm K )
= 2.9 K/W
b
2
1
1

(3.9)
Partea de jos rezistență R
b
este mult mai mare decât catalogheaza împreună top rezistență R
pa

, deoarece, în
practică,
este ușor și ieftin de a oferi suficientă izolație sub placă de bază pierderile sunt neglijabile (a se vedea
Problema 3.2). Deci la o primă aproximare, R
b
= ∞.
Luați în considerare mai în detaliu cele trei etape de exterior de transfer de căldură prin capacul de sticlă în
centrul calea Fig. 3.4(b):
1
Gratuit convecție de aer în spațiul transportă căldură de sticlă. În paralel cu aceasta placa radiază căldură
la lungimi de undă
~10
mm. La aceste lungimi de undă, sticla nu este transparent, dar puternic absorbante (vezi Fig.
R3.12). Prin urmare, această radiație este absorbită de sticlă.
2
Căldura ajunge la pahar prin aceste două mecanisme este apoi efectuat la suprafața exterioară a
sticlei.
3
De aici este transferat la împrejurimile gratuit și/sau convecție forțată, și radiații.
Astfel, rezistența generală între partea de sus a plăcii și împrejurimile trei etape în serie:
=
+





 + +
+







R
R
R
R
R
R
1
1
1
1
v pg
r pg
v ga
r ga
pa
,
,
1
g
,
,
1

(3.10)
În Fig. 3.4(b) rezistența R
g
este neglijabil, deoarece sticla este subțire (
~5 mm) și o bună moderat
conductor (k
≈1Wm
-1
K
-1
) (Puteți verifica acest lucru folosind (R3.10)). Prin urmare, diferența de temperatură de peste
sticla este de asemenea neglijabil.
Convective și radiative rezistențele variază lent cu temperaturi în circuit, astfel încât
calculul poate continua cu estimările inițiale pentru aceste temperaturi, de exemplu:
T
p
= 70 ° C și T
g
=
1
2
(T
p
+ T
o
)
= 45ºC
(3.11)
Pentru 1 m
2
colector, convective rezistență R
v,pg
urmează direct de Lucrat Exemplu R3.2: R
v,pg
=
0.52 KW
-1
.
TWIDELL PAGINARE.indb 84
01/12/2014 11:36
§3.3 colectoarelor Plane
85
Radiativ rezistență R
r,pg
este determinat, folosind (R3.45) de Revizuire 3 și (C. 18) din Apendicele C:
R
UN T
(1/
) (1/
)1
4
()
(1/ 0.9) (1/ 0.9) 1
4(1m )(5.67 10 Wm K )(330K)
0.150 KW
p
g
r,pg
3
2
8
2
4
3
1
e
e
σ
=
+

=
+

×
=




(3.12)
Rețineți că, deși acest calcul este cel mai precis la două cifre semnificative, vom efectua trei cifre
cu nerăbdare pentru a evita erorile de rotunjire mai târziu în calcul.
Astfel, totalul placă-la-sticlă de rezistență este dat de:
R
pg
= [(1/R
v,pg
)
+ (1/R
r,pg
)]
-1
= 0.116 KW
-1
(3.13)
Rezistența la pierderea de căldură prin convecție de la capacul de sus este explicat de (R3.15) și (R3.17) ca:
R
v,ga
= 1/(h
v
O)
în cazul în care coeficientul de transfer termic h
v
[unitate W/(m
2
K)] cu o viteză a vântului u este dat de (C. 15) din Anexa C
ca:
h
v
= o + bu = 24.7 Wm
-2
K
-1
pentru valorile date. Deci R
v,pa
= 0.040 K/W
Luând T
cer
= T
o
− 6 K = 287 K, la fel ca în §2.6.3, deci T

= ½ (T
cer
+ T
g
)
= 303 K, rezistența la radiativ de căldură
transferul este, folosind (R3.45) de Revizuire 3 și (C. 17) din Anexa C,
R
r,ga
= 1/{4
e
g
σO(T

)
3
}
= 1/{4(0.9)(5.67 × 10
-8
Wm
-2
K
-4
)(1.0 m
2
)(303K)
3
}
= 0.176 K. W
-1
(3.14)
Prin urmare:
R
ga
= [(1/R
v,ga
)
+ (1/R
r,ga
)]
-1
= 0.0329 K/W
(3.15)
Deci, reamintind că R
g
≈ 0, total (serie) rezistența deasupra plăcii este:
R
pa
= R
pg
+ R
g
+ R
ga
= 0.148 K/W
(3.16)
care este, de asemenea, rezistența totală R
Am
de când ne-am asumat paralel bază de rezistență R
b
este infinit. Apoi
în (3.8) cu
τ = α = 0.9 și G = 750 Wm
-2
oferă:
T
p
(m)
= R
Am
ταO
p
G
+ T
o
(3.17)
= (0.148 KW
-1
)(0.9)(0.9)(1m
2
)(750Wm
-2
)
+ 20°C = 110°C
În practică, cu toate acestea, eficiența transferului
η
pf
≈ 0.85 mai degrabă decât 1.00; vezi §3.3.2. A pune acest lucru în
calcul (se multiplică primul termen pe RHS de (3.17)) randamentele T
p
(m)
= 96°C, cu alte ipoteze
neschimbată.
Cu toate acestea, deoarece apa va fierbe la 100°C, noastre aproximative de calcul corect a arătat că
stagnarea temperatura cu zero debitul de apă în condițiile însorită poate fi suficient de mare pentru a provoca
clocotită.
TWIDELL PAGINARE.indb 85
01/12/2014 11:36
86
Solare de încălzire a apei
Exemplul 3.1 la unele puncte cheie caracteristici de design:
1
Izolatia este util. Aproape orice material care capcane de aer într-o matrice
de mici volume (
≤1 mm) este util ca un izolator pe acest lateral spate
(de exemplu, fibra de sticla, polistiren expandat sau așchii de lemn). Termică
conductivitatea toate aceste materiale este comparabilă cu cea de aer
(k
~ 0.03 Wm
-1
K
–am

); a se vedea Tabelul B. 3. Izolator volume de aer trebuie să


nu fie prea mare, deoarece în caz contrar aerul va de transfer de căldură prin
convecție. Materialul trebuie să fie, de asemenea, uscat, deoarece apa în matrice este un
mult mai bun conductor decât aerul (a se vedea Anexa B). Problema 3.2 arată
că numai câțiva centimetri de izolație sunt necesare pentru a crește
jos rezistență la de zece ori mai sus de rezistență. În ciuda nevoie
pentru o capacului din spate pentru a menține izolația uscat și pentru a preveni daune de
păsări și șoareci, etc., spate de izolare este aproape întotdeauna benefic și
cost-eficiente. [Continue P. 87]
CASETA 3.1 TEMPERATURA DE REFERINȚĂ T
ref
PENTRU CĂLDURĂ CIRCUIT MODELING
În apă solare Lucrat Exemplul 3.1, combinarea (3.6), (3.5) și (3.1) arată că lichidul într-un colector
se încălzește la o rată dată de:
τα
=


T
t
AG T T
R
mc
d
d
(
)/
f
f
o
Am
(3.18)
Fig. 3.5
(a) Analogice de circuit pentru ecuația (3.18) cu pierdere de rezistență R
Am

arătat în mod generic ca un singur


componenta între placă și temperatura mediului ambiant T
o

; (b) mai precis analog


circuitul, cu R
Am
separate în paralel componente pierderea de căldură de diferite, și, eventual,
schimbarea, temperaturi T
o
și T
cer

.
G
Util
T
f
C
f
P
Am
T
p
R
Am
T
o
T
o
T
p
P
Am
T
cer
Cer
T
ref
T
ref
T
ref
Convecție
Radiații
Ambient
Pierderile
Pierderile
(b)
(a)
Căldura circuit pentru această situație este prezentată în Fig. 3.5(a). Pentru a menține circuitul analogie, avem nevoie de o
temperatură de referință T
ref
ca analog al pământului potențiale în energie electrică. T
ref
este un arbitrar dar fixat
temperatura, care este independent de timp, deoarece dT
f
d /t pe partea stângă a (3.18) poate fi înlocuit cu
d(T
f
– T
ref
)/dt dT
ref
/dt
= 0. O alegere convenabilă este T
ref
= 0°C. Numai dacă temperatura ambiantă este constantă
putem seta T
ref
= T
o
și încă păstra analogia între circuitul și bilanțul termic ecuația
(3.18). Simbolul bateriei în brațul drept al analogice de circuit reprezintă T
o
ca o diferență de T
ref
.
Rețineți că, în multe situații, radiatorul temperaturi de convecție și de radiație sunt nu egale.
În general, convecție pierderea este la temperatura aerului ambiant, și radiativ pierderea este de la cer și/sau
radiativ mediu; Fig. 3.5(b) permite atât de diferite, și, eventual, schimbarea, chiuveta de căldură de temperaturi.
TWIDELL PAGINARE.indb 86
01/12/2014 11:36
§3.3 colectoarelor Plane
87
2
Evita o presiune excesivă și alte pericole de apă foarte fierbinte și
de fierbere. Chiar și simpla încălzire a apei solare produc periculoase
condiții dacă elementare măsuri de protecție nu sunt luate (de exemplu, de eliberare a presiunii
supape, semne de avertizare, intrare copil de prevenire). Evitând fierbere este un
motiv pentru care thermosyphon sisteme (§3.4.2) sunt de preferat în
zonele cu climat însorit (când debitul de apă nu este supusă la defectarea pompei).
§3.3.2 Eficiența unei plat-colector
Un colector de eficiență
η
c

și zona
O
p

, expuse la iradiere G (amm-
ured în planul de colector), oferă o putere utilă:
P
u
=
η
c
O
p
G (3.19)
Conform (3.3) și (3.5), randamentul colectorului
η
c

pot fi împărțite în
două etape, eficiența de captare a
η
sp

și eficiența transferului
η
pf

:
η
c
=
η
sp
η
pf
(3.20)
Rezultă din (3.2) în care:
η
sp
=
τ
cov
α
p
− U
Am
(T
p
− T
o
)/G (3.21)
ceea ce arată că placa devine mai fierbinte, pierderile crește până
η
sp

scade la zero la "echilibru" temperatura


T
p
(m)

(de asemenea, numit


stagnare temperatura).
Pentru placa de temperatură T
p

de un colector de operare nu este


de obicei cunoscut, este mai convenabil să se refere utile câștig de energie de
la temperatura medie a fluidului
T

f
.
Prin urmare:
η
c
= P
u
/(AG)
=
η
pf
τ
cov
α
p

η
pf
U
Am
(T–
f
− T
o
)/G (3.22)
Într-un bine conceput colector, diferența de temperatură între
placă și fluid este mică și valoarea de
η
pf

este aproape o (a se vedea Problema


3.8). De obicei,
η
pf

= 0.85 și este aproape independentă de operare


condiții, și, din conducte și rezervoare de stocare ar trebui să fie bine izolate,
T

f

≈ T
p
, colector, temperatura. Prin urmare, U
Am
în (3.22) este numeri-
cally aproape aceeași ca în (3.21). Eficienței de captare
η
sp
(și
prin urmare, de asemenea, randamentul colectorului
η
c
) ar varia liniar cu tem-
dacă temperatura de U
Am
= 1/(O
P
R
Am
) este constantă în (3.21) și (3.22), dar, în practică,
radiativ rezistența scade în mod semnificativ ca T
p
crește. Prin urmare, o
teren de
η
c
împotriva temperatura de operare este curbat, ca în Fig. 3.6.
Performanța unui plate-placă colector, și în special eficiența acestuia
la temperaturi ridicate, poate fi substanțial îmbunătățită prin una sau ambele dintre
următoarele:
1
Reducerea convective de transfer între placă și exterior
capac din sticlă, cu un geam dublu-capac superior (vezi Fig. 5.1(b) și
Problema 3.5).
TWIDELL PAGINARE.indb 87
01/12/2014 11:36
88
Solare de încălzire a apei
2
Reducerea radiativ pierderea de la placa de a face suprafata sa nu
pur și simplu negru, dar selectiv, adică puternic absorbante dar slab emițătoare de lumină
(vezi §3.5).
Rezultat câștiguri în performanță sunt prezentate sintetic în Tabelul 3.1.
Comerciale încălzitoare solare de apă ar trebui să fie de așteptat să aibă selectivă
suprafața plăcilor.
§3.4 SISTEME CU STOCARE SEPARAT
§3.4.1 Activ sistemele cu circulație forțată
Colectoarele în sine (a se vedea Fig. 3.1(a) și (b)) conțin numai un
volum relativ mic de apă, care atunci când este încălzit trece la un izolate
rezervorul de stocare; în cazul în care rezervorul este mai presus de colector, pompa nu este necesar.
Cu toate acestea, în zonele cu climă cu înghețarea iarna și atunci când sunt integrate cu
alte sisteme de încălzire, rezervorul de stocare este în construcție și
, în mod normal, mai jos de colector, deci, o pompă de apă este necesar; Fig. 3.7 prezintă
astfel un sistem integrat cu alte încălzire a apei mecanisme. A
separa circuitul lichidului prin colector permite antifreezing lichide pentru a
fi utilizate. Pentru sisteme naționale, rezervoare cu volume de la aproximativ 100 la
300 de litri poate stoca o zi de furnizarea de apă caldă, cu utilizarea efectivă în funcție de
gama și utilizarea eficientă a apei de spalatoare, dusuri si bai. Astfel
circulație forțată are nevoie doar de o pompa de mici, astfel încât temperatura apei
crește în soare de aproximativ 5°C la 10°C la fiecare inițială trece prin
colecta sau. Acest increment creștere a temperaturii depinde în mare măsură
de radiație solară G și diferența între intrare și ieșire
temperatura de colector. Performanța optimă necesită un controlată
variabila-pompa de viteza, dar, de obicei, mai ieftin imobilizate-viteza pompei este folosit
Fig. 3.6
Tipic eficiența curbele de un singur geam colectoarelor plane. T

f

este temperatura medie


ale fluidului de lucru și T
o

este temperatura mediului ambiant.


Sursa: după Morrison (2001).
0.1
Colector effciency
η
c
Simplu
negru mat
Cu
selective
suprafață
0
0
1
[(T
f
– T
o
)/G]/[m
2
KW
-1
]
TWIDELL PAGINARE.indb 88
01/12/2014 11:36
§3.4 Sisteme cu stocare separat
89
pentru care design creșterea temperaturii are loc doar pentru un singur set de
condiții. Pompa este alimentat de la rețeaua de energie electrică sau de la un mic
panou fotovoltaic alături de colector; acesta este comutat automat pe
și în afara, astfel colector ieșire temperatura
~5°C mai mult de intrare.
Acest lucru previne inutil utilizare a pompei și, în special, prostia
de a pierde căldură de la colector la săraci lumina soarelui și noaptea. O continuare
Fig. 3.7
Pe acoperiș colector solar ca prioritate de intrare pentru un sistem de apă caldă (izolare termică la
rezervor, conducte și colector nu sunt prezentate). Solar pompa funcționează atunci când colectorul de ieșire
este mai fierbinte decât cea de intrare de aproximativ 6°C. Cu intrare de la cea mai rece (de exemplu, cea mai mică) parte din
rezervorul de apă, colector funcționează la eficiență maximă. Alte sisteme de încălzire
sunt doar de rezervă (de exemplu, în timpul iernii sau din cauza excesive de apă caldă de utilizare).
De alimentare cu apă caldă
Energie electrică
boiler
(în mod normal, off)
Cazan (iarnă)
Apă rece
afluxul
Rezervor de apă caldă
Pompa
(
T
h
>
T
c
+
6 °C)
Expunere la soare
Acoperiș
T
h
T
c
Tin
low
EXEMPLUL 3.2 CREȘTEREA TEMPERATURII PRINTR-UN COLECTOR
O plat-colector de măsurare 2 m
× 0,8 m are o pierdere de rezistență r
Am
= 0.13 m
2
KW
-1
și o placă de transfer
eficiența
η
pf
= 0.85. Capacul de sticlă a transmitanței
τ = 0.9 și absorptance de placa este α = 0.9.
Apa intra la o temperatură T
1
= 40°C. temperatura mediului ambiant T
o
= 20°C și iradiere în
planul de colector este G
= 750 Wm
-2
.
(a) Calcula debitul necesar pentru a produce o creștere de temperatură de 4°C.
(b) să Presupunem că pompa continuă să pompeze la noapte din cauza defect de control. Estimarea inițială de temperatură
scădere la fiecare trecere prin colector. Presupune: G
= 0, aceeasi pompa de rata, T
1
= 40°C, T
o
= 20°C.
Soluție
(a) La (3.1) și (3.7), utile de putere pe unitatea de suprafață este:
q
u
=(
ρcQ
/O)(T
2
− T
1
)
=
η
pf
[
ταG − (T
p
− T
o
)/r
Am
] (3.23)
Presupunând Că T
p
= 42°C (temperatura medie a fluidului), se obțin:
Q
= 3.5 × 10
-5
m
3
s
-1
= 130 L h
-1
(b) Din (3.23) cu G
= 0, T
p
= 38°C, iar anterior a calculat valoarea Q,
T
2
– T
1
= -1.3°C
TWIDELL PAGINARE.indb 89
01/12/2014 11:36
90
Solare de încălzire a apei
senzor de temperatura in partea de sus a rezervorului poate fi folosit pentru a preveni clocotită.
Unele țări membre și necesită încălzire suplimentară pentru a fi utilizate
săptămânal sau lunar pentru a crește temperatura rezervorului la aproximativ 55°C pentru a ucide
bacteriile nedorite (de exemplu, cele care ar putea provoca boala legionarilor).
Solare mai eficiente sisteme de încălzire a apei includ rezervor și
mecanisme de transfer de căldură în designul de ansamblu. Generos izolare este
întotdeauna benefică, mai ales ca este ieftin. Introducerea rezervorul de apă pentru a
minimiza lungimea de tevi la cald este, de asemenea, benefic, și mai simplu pentru noi
clădiri cu design integrat. Sensibil reglementările de planificare necesită astfel de
design pentru noi cât și convertite în clădiri (a se vedea Menanteau 2007).
Având în vedere rezervor, design eficient își propune să mențină cea mai tare apă
la partea de sus a rezervorului și permite ca acest lucru să rămână cu stratificare stabilă.
Cu toate acestea, de intrare a apei în colector trebuie să fie de cea mai rece
straturi de rezervor la fundul acesteia pentru cel mai bun colector de eficiență. Reușind
ambele condiții este dificilă și nu frecvent întâlnit, mai ales daca
potabilă (pur) apa nu poate trece prin colector datorită potențialului
de congelare sau de contaminare. De exemplu, bobinele de încălzire prezentate în
Fig. 3.7 iniția convective de amestecare în rezervor, astfel încât prevenirea stabil
de stratificare. În plus, temperatura apei livrate utilizatorului
depinde de înălțimea la care rezervorul este exploatat.
Alte sisteme sunt concepute pentru a promova stratificare, astfel încât cea
mai tare apă este disponibil pentru cât mai mult posibil. Un mod ingenios de a
realiza acest lucru este de a avea apa incalzita de la colector introduce printr -
o conductă verticală cu clape peste găuri distribuite pe verticală de-a lungul conductei.
Apă mai fierbinte este mai puțin dens decât apa rece, astfel încât clapele rămâne închis
până la încălzit apa ajunge la un stratificat layer la propria temperatură.
În această poziție, clapeta se deschide și apa încălzită se alătură cu rezervor de
apă la aceeași temperatură și deci, în general stratificarea este menținută.
§3.4.2 Sisteme cu circulație thermosyphon
Combinarea de stocare a apei într-o singură unitate cu colector, și de stabilire
această unitate pe acoperiș sau la înălțimea acoperișului, este comun în țările cu un
general climat cald (de exemplu, Africa și Australia). Circulația apei
într-un thermosyphon sistem (Fig. 3.8), cu rezervorul de stocare deasupra
colecta sau ca într-un acoperiș unitate, este determinată de diferența de densitate între
apă caldă și rece. Ia în considerare simplu sistemul prezentat în Fig. 3.9,
închis buclă verticală din țeavă umplută cu lichid.
La secțiunea aa
',
gdz
g dz 0
o (stânga)
b
o (dreapta)
b



ρ
ρ

>
(3.24)
Coloana din stânga a lichidului exercită o presiune mai mare la aa
decât dreptul de
coloana, stabilind astfel întreaga buclă de lichid în mișcare. De conducere
de presiune, care este tocmai partea stângă a (3.24), poate fi exprimat
în general ca:
TWIDELL PAGINARE.indb 90
01/12/2014 11:36
§3.4 Sisteme cu stocare separat
91
p
th
=

ρgdz (3.25)
în cazul în care cercul denotă faptul că integrala este luat de pe un închis buclă.
Rețineți că dz în (3.25) este verticală creștere, și nu sporul de
lungime de-a lungul conductei. Ecuația (3.25) poate fi rescrisă ca:
p
th
=
ρ
0
gH
th
(3.26)
unde thermosyphon cap
H
th
=

(
ρ/ρ
0
− 1)dz (3.27)
reprezintă câștigul de energie pe unitatea de greutate a fluidului și
ρ
0

este orice
convenabil densitate de referință. Acest câștig de energie a fluidului poate fi pierdut
prin alte procese și, în special, prin frecare conductă reprezentat de
frecare cap Sec
f

de §R2.6.
Extinderea coeficientul
β = −(1/ρ)dp / dT (3.28)
este de obicei constant, astfel încât (3.27) se reduce la
H
th
=−
βeu
T
=−
β ∫(T − T
0
)dz (3.29)
în cazul în care T
0
este o temperatura de referință. Fluxul este în direcția pentru care
Am
T
este pozitiv.
Fig. 3.8
Colector și rezervor de stocare cu thermosyphon circulație: (a) fizic diagrama;
(b) distribuție de temperatură (a se vedea Exemplul 3.3).
Înălțime : z (m)
1.4
0.7
0.5
0
Stocare
rezervor
Colector
1
2
4
4
1
2
3
3
40
44
Temperatura: T(°C)
(b)
(a)
Fig. 3.9
Principiul de thermosyphon flux.
Rece
(fluid dens)
Fierbinte (mai puțin dens
extins lichid)
Solar
radiații
o'
b'
o
b
TWIDELL PAGINARE.indb 91
01/12/2014 11:36
92
Solare de încălzire a apei
§3.5 SELECTIVĂ A SUPRAFEȚELOR
§3.5.1 Ideal
Un colector solar absoarbe radiațiile cu lungimi de undă în jurul valorii de 0,5
mm (de la
solar sursa de la 6000 K) și emite radiații cu lungimi de undă în jurul valorii de
EXEMPLUL 3.3 CALCULUL THERMOSYPHON FLUXUL
În sistemul de încălzire este prezentat în Fig. 3.8, apa intră în colector la temperatura T
1
= 40°C, este încălzită de
4°C, și se duce în partea de sus a rezervorului, fără pierderi de căldură la T
3
= T
2

= 44°C. Dacă sistemul deține 100 de litri


de apă, se calculează timpul pentru toată apa să circule o dată în jurul sistemului. Presupunem că rezervorul are timp
pentru a realiza o stratificare stabilă.
Soluție
Circulația și izolare a se asigura că apa cea mai rece în partea de jos a rezervorului este la aceeași
temperatură ca și intrarea în colector (de exemplu, T
4
= T
1
).
Integrala

(T
− T
0
)dz în jurul conturul 1234 este doar zona din interiorul curbei (Fig. 3.8(b)). Această zonă este
suma de triunghiuri hașurate plus dreptunghiul din mijloc, adică
Am
T
=
1
2
(0,5 m)(4°C)
+ (0,2 m)(4ºC) +
1
2
(0,7 m)(4ºC)o
= +3,2 m.K
Evident, fluxul merge în direcția 1234.
Având o valoare medie
β = 3.5 × 10
-4
K
-1
în (3.29) dă H
th
= − 0.0010 m.
Această valoare va fi suficient de precisă pentru cele mai multe scopuri, dar o valoare mai precisă ar putea fi derivat de
trasarea unui contur de
ρ(z), folosind Tabelul B. 2 pentru ρ(T) și evaluarea (3.27) în mod direct.
Să se calculeze viteza de curgere, ar echivala thermosyphon cap la frecare cap a i se opune. Cel mai
de frecare va fi în cel mai subtire conducte, și anume coloană tuburi în colector. Să presupunem că există patru
tuburi, fiecare de lungime L
= 2 m și diametrul D = 12 mm. Apoi, în fiecare tub, folosind simbolurile de control 2
(§R2.6)
H
th
= 2gripă
2
/Dg unde u este viteza de curgere în tub și f
= 16
ν/(uD) pentru flux laminar.
Prin urmare:
=
u
gD H
Lv
32
2
th

=
×
×
×




(1.0 10 m)(12 10 m) (9.8 ms )


(32)(2m)(0.7 10 m s )
3
3
2
2
6
2
1

= 0.031 m.s
-1
Verificarea coerenței, vom găsi numărul lui Reynolds uD/
ν = 540, astfel încât curgerea este laminară ca
se presupune.
Debitul volumetric prin cele patru tuburi este:
Q
= 4(u
πD
2
/ 4)
= 1.4 × 10
-5
m
3
s
-1
Astfel, dacă sistemul deține 100 de litri de apă, întregul volum circulă într-un timp de
(100)(10 m )
1
1.4 10 m s
1.0 sec
3.6 10 s
2.0 h
3
3
5
3
1
3

×




×




=




TWIDELL PAGINARE.indb 92
01/12/2014 11:36
§3.5 Selectivă a suprafețelor
93
10
mm (de la o sursă la ~350 K) (a se vedea Fig. R3.10). Prin urmare, un ideal
de suprafață pentru un colectionar ar își maximizeze câștigul de energie și reduce la minimum
pierderile de energie, de a avea o mare monocromatice absorptance
α
am
la
l ~0.5
mm și mici monocromatice emisivitate
e
am
la
l ~10 mm, după cum este indicat
schematic aliat în Fig. 3.10. O astfel de suprafață a
α
scurt
>>
e
mult
în notația
de §R3.5.4. Cu o suprafață selectivă,
α și e sunt ponderate mijloace de α
am
și
e
am
respectiv peste diferite lungimi de undă variază (cf. (R3.27)).
§3.5.2 Metal semiconductor compozit suprafata
Unii semiconductori au
α
am
și
e
am

caracteristici, care seamănă cu


cele ale unui ideal selectivă suprafață. Un semiconductor absoarbe numai acei
fotoni cu energii mai mari decât E
g

; adică energia necesară pentru a promova


un electron de valență în banda de conducție (vezi Capitolul 5). De
critică de energie E
g
corespunde o lungime de undă de 1.1
mm pentru siliciu și
2
mm pentru Cu
2

O; mai scurte lungimi de undă sunt absorbite puternic (Fig. 3.10).


Cu toate acestea, slabă rezistență mecanică, conductivitate termică scăzută și
cost relativ ridicat de semiconductoare suprafețele de a le face improprii pentru
întregul colector de material.
Metale, pe de altă parte, sunt de obicei rezistente mecanic, bune
conductoare și relativ ieftine. Ele sunt, de asemenea, din păcate bun reflectoare
(de exemplu săraci amortizoare) în ambele vizibil și infraroșu. Când lumina (sau alte
radiații electromagnetice) este incident pe un metal, electronii liberi în apropiere de
suprafața vibreze rapid ca răspuns la diferite electromagnetice
domeniu. Prin urmare, electronii constituie o variabilă de curent, care
radiază unde electromagnetice, ca într-un antenei radio. Se pare pentru un
observator din afară că radiația incidentă a fost reflectat. La
putere a undei reflectate este doar puțin mai mică decât cea de la incident
Fig. 3.10
Caracteristicile spectrale ale diferitelor suprafețe. Metal indicat este Cu, semiconductoare este
Cu
2

O.
Semiconductoare
Ideal
selectivă
suprafață
Metal
0
1
0.3
1
3
10
λ
/µm
α
λ
=
λ

TWIDELL PAGINARE.indb 93
01/12/2014 11:36
94
Solare de încălzire a apei
val (Născut și Lupul 1999), atât pentru
l ≥ 1 mm, ρ
am
≈ 0.97 (de exemplu,
α
am
=
e
am
≈ 0.03)
(a se vedea Fig. 3.10)).
Unele metale prezintă o creștere în absorptance mai jos un scurt-val
lungime
am
p
. Pentru cupru
am
p
≈ 0.5
mm (vezi Fig. 3.10). Prin urmare, cuprul
absoarbe lumina albastra mai mult roșu și apare roșiatică în culoare. La
lungimea de undă
am
p
corespunde plasmă frecvență f
p
= c/
am
p

care este
frecvența naturală de oscilație a unui electron strămutate despre un
ion pozitiv. Net de energie trebuie să fie hrănite cu electronii pentru a le face să oscileze
mai repede decât această frecvență, deci
α
am
crește la aproximativ 0,5 pentru frecvențe
mai mult de f
p
(adică lungimi de undă mai mică decât
am
p
).
Prin plasarea unui strat subțire de semiconductor de peste un metal, putem combina
cu caracteristicile dorite de ambele. Fig. 3.11 arată cum primite
cu unde scurte radiații este absorbită de semiconductoare. Absorbit
de căldură este apoi trecut prin conducție de fond al sistemului de metal. Deoarece
conductivitatea termică a unui semiconductor este mic, semiconductoare stratul
trebuie să fie subțire pentru a asigura transferul eficient de metal. Cu toate acestea, ea
nu trebuie să fie prea subțire; în caz contrar, o parte din radiația s-ar ajunge la
metal și de a fi reflectate.
Din fericire absorbția lungime de un semiconductor la
l = 0,6 mm
este, de obicei doar
~1
mm, adică 63% din radiatii primite este absorbit
în top 1
mm, și 95% în primele 3 mm (a se vedea §R3.6). Prin urmare,
la absorptance pentru radiația solară este mare. Radiația emisă este la
lungimi de undă
~l0
mm pentru care emisivitate atât de metal și
semiconductoare este mic (
e ≈ 0.1, ca în Fig. 3.11).
Fig. 3.11
Fluxul de căldură într-un singur tip de suprafață selectivă. Aici un semiconductor (care absoarbe puternic
solare de unde scurte radiații) este depus pe un metal (care este un slab emițător termice
longwave radiații)
Unde scurte
radiații
(puternic absorbit
în
semiconductor)
Longwave
radiații
(slab emise
de
metal)
Absorbția
Căldură bună
conducție
în
metal
Slabă emisie
de metal
și
semiconductor
α
scurt
≈ 0.85
λ
~1
µm
λ
~ 10
µm

mult
≈ 0.1
Semiconductoare
(de exemplu Cu
2
O)
Metal (de exemplu, Cu)
TWIDELL PAGINARE.indb 94
01/12/2014 11:36
§3.5 Selectivă a suprafețelor
95
Rezultatul este un compozit de suprafață, care are mult mai mic radiativ pierdere
decât un simplu vopsite în negru suprafata (care este negru la ambele vizibile și
radiații infraroșii, și, prin urmare, are
α = e ≈ 0.9). La absorptance nu este
destul de la fel de mare ca de un negru pur de suprafață, pentru că
α
am
de selectiv
suprafața scade pentru
l≥ 1 mm (vezi Fig. 3.10), și 30% din radiația solară-
tion este la lungimi de undă mai mare decât 1
mm (vezi Fig. 4.1).
Mici de emisivitate al selectivă suprafața devine mai mult un
avantaj inițial ca temperatura de lucru crește, de la
radiativ pierderi crește ca
eT
4
. De exemplu, la o placă la temperatura de 40°C
cu
e >0.9, pierderile radiative sunt, de obicei, doar 20% din total (de exemplu,
calcula aceste în Exemplul 3.1); cu toate acestea, de la o placă la temperatura
de 400°C-ar fi de 50% dacă
e = 0.9, dar numai 10% dacă e = 0.1 (dar a se vedea
atenție după (4.24) pentru T
> 1000°C). Cu toate acestea, în cazul în care suprafața
temperatura devine extrem de fierbinte, de exemplu, pentru un colector într-un
solară concentrată array (§4.8) de la, probabil, 1000ºC sau mai mult, atunci lungimea de undă
variază atât pentru absorbție (de la Soare) și emisie (de
colector) se suprapun, astfel încât monocromatice absorptance
α
am
și mono-
cromatic emisivitate
e
am

nu mai sunt semnificativ diferite și selective


suprafețe nu pot fi obținute.
§3.5.3 Fabricarea de selectiv suprafete
O metodă pentru prepararea unei reale selectivă suprafață implică scufundare o
foaie de cupru în soluție alcalină, astfel încât un film Cu
2

O (care este
un semiconductor) este format pe ea. Multe alte acoperire a suprafeței tipuri au
fost dezvoltate cu succes, inclusiv negru crom (Cr/CrO
x
), metal
strat din oxid de aluminiu (de exemplu, Ni/ Al
2
O
3

) și oxidat din oțel inoxidabil.


Cele mai multe producția comercială de selectiv suprafețe este acum de pulverizare,
mai degrabă decât prin electrochimice de scufundare. Pulverizare permite prepararea
de apă-gratuit acoperiri compozite în care compoziția chimică,
compoziția granulometrică, metal, particule de dimensiune și volum factor de umplere poate fi
controlată cu atenție. Astfel selectivă amortizoare obținerea
α >0.95
și
e <0.10.
Absorbant peliculă subțire de strat este, de obicei un metal: dielectric compozit,
de multe ori cu gradată a indicelui de refracție crește cu adâncimea. Un favorizat
compoziția este o granulatie fina dispersie de dimensiuni submicronice efectuarea de
particule încorporate într-o izolare matrice de scăzută constantă dielectrică
, care este transparent la radiația infraroșie. Multe procese fizice
pot contribui la mare solare absorptance (de exemplu cu plasmă rezonanță
de electroni liberi (ca în Cu), rezonanță împrăștierea de discrete
efectuarea
particule texturale discontinuități și rugozitatea suprafeței, inter-
trupa
tranziții (ca în
semiconductori), și efecte de interferență).
Modele teoretice de astfel de dispersii folosind ecuațiile lui Maxwell du-te
înapoi la 1904, dar recent au fost rafinate în mediu eficient
teorii care permit
modelare pe calculator pentru a fi utilizate pentru a evalua
TWIDELL PAGINARE.indb 95
01/12/2014 11:36
96
Solare de încălzire a apei
candidatul mass-media și de a optimiza modele (Wackelgard et al. 2001; Hutchins
2003).
Selectiv suprafețe continuă să fie un domeniu activ de cercetare și
dezvoltare (R&D), în calitate de producători de energie solară termică echipamente depună eforturi pentru a
îmbunătăți eficiența, a reduce costul de producție, și de a îmbunătăți
robustețea (în special pentru aplicații la temperaturi
>
~200°C). Mult de acest
R&D se concentrează pe tehnici de producție și pe nanostructura de
suprafață.
§3.6 EVACUAT COLECTOARE
Folosind o suprafață de absorbție selectivă reduce substanțial pierderile radiative
de la un colecționar. Pentru a îmbunătăți în continuare eficiența și de a obține mai mari
diferențe de temperatură (de exemplu, pentru alimentarea cu căldură la temperaturi
>~70°C, pentru care
nu este substanțială a cererii industriale) pierderi convective trebuie să fie, de asemenea,
redus. O metodă de îmbunătățire moderată de o plat-colector
este de a utiliza dublu-vitraj (a se vedea Problema 3.3). Cu toate acestea, fără îndoială, cea
mai bună metodă este să evacueze spațiul dintre placă și sticlă
capacul. Acest lucru necesită o foarte puternică structurale de configurare pentru a preveni
aer de mare presiune, forțele de rupere de sticlă, care este cel mai bine asigurat
cu colector într-o sticlă exterior tub de secțiune circulară. O
mai puțin frecvente metodă pentru colectori plați este de a fi puternic,
transparent 'distanțiere în parțial evacuat spațiu între farfurie și
se acoperă pentru a contracara extern de presiune a aerului.
Un tip de evacuat colector foloseste un tub dublu, așa cum se arată în
Fig. 3.12(a), cu tub interior conțin fie apă potabilă să
fie încălzit direct sau un alt fluid de transfer de căldură. Tubul exterior este
realizat din sticlă, deoarece este transparent pentru solară de unde scurte radiații
, dar nu să termică, longwave, radiații, și pentru că sticla este relativ
puternică în comparație cu plastic transparent materiale. Tubul interior este
, de obicei, realizate din sticlă de sticlă deține un vid mai bine decât cele mai multe
alte materiale. Emisia de gaze rata la cuptor din sticlă Pyrex este astfel
că presiunea poate fi deținute mai puțin de 0,1 N m
-2
timp de 300 de ani, care
este de aproximativ 10
l2

ori mai mult decât pentru un tub de cupru. Tubul interior are o


secțiune transversală circulară. Acest lucru ajută pe cei slabi de sticla rezista la tensiune
forțele produse de diferența de presiune dintre lichidul din interiorul
și vidul exterior. De obicei, tuburile au diametrul exterior
D
= 5 cm și un diametru interior d = 4 cm. De către corespunzător conectarea unei matrice
de aceste tuburi, probabil, cu reflectoare, astfel colectoare primi
ambele radiatiei solare directe si difuze. Alte variante sunt, de asemenea, comercializate
cu succes, mai ales singur tub de sticlă sisteme cu interior
colector-un tub de metal în contact cu o farfurie lung, în spatele căreia este fixată o
pânză țesută wick ca unheat-pipe' (Fig 3.12(c), a se vedea §R3.7.2). Aici
combinate tuburi sunt înclinate și lichidul în wick se evaporă.
Vaporii se condensează în cadrul orizontală a galeriei de un antet de căldură
TWIDELL PAGINARE.indb 96
01/12/2014 11:36
§3.6 Evacuat colectoare
97
schimbător și condensat fluid trece înapoi în jos de asamblare
pentru ciclul să se repete. Fiecare combinate tub este independent și poate fi
extras și înlocuit fără a interfera cu sistemul general. The
heat-pipe-transferuri semnificative de căldură cu neglijabilă rezistența termică.
Procesele de fabricație pentru toate formele de evacuat colectoare
utilizarea sofisticate, echipamente automate. Tuburile trebuie să aibă o lungă
durată de viață, dar sunt sensibile la daune de grindină și vandalism.
EXEMPLUL 3.4 ECHILIBRUL TERMIC AL UNUI EVACUAT COLECTOR
Se calculează pierderea de rezistență a evacuat colector de Fig. 3.12(a) și estimarea de stagnare
temperatura. Ia D
= 5,0 cm, d = 4.0 cm, lungime de tub 1.0 m; longwave (infraroșu) emittances
e
P
= 0.10,
e
g
= 1.0,
e
aer
=1.0; unde scurte (solar) absorptance de placa α
P
= 0.85, transmisie de sticlă
τ
g
= 0.90,
G
= 750 W m
-2
, T
o
= 20°C, T
cov
= T
g
= 40°C; T
p
= 100°C, viteza vântului u = 5,0 m s
-1
.
Soluție
Simbolurile și metodele de control 3 sunt utilizate, împreună cu informații din Apendicele B (Tabelul B. 5)
și C.
Fig. 3.12
(a) Evacuat colector. (b) Circuitul analog de (- o). (c) Evacuați tub colector cu căldură intern-pipe si
colector. Căldura de condensare trece la încălzit apă în partea de sus a schimbătorului de căldură. Multe astfel de tuburi
sunt, în mod similar, conectate în paralel.
Pahar cu
capac
Lichid
Convecție
"Vid"
Radiații
Selectiv
suprafață
de peste
interioară perete de sticlă
d
G
(a)
D

Radiații
Schimbător de căldură
Colector
pentru apă încălzită
Fierbinte
apa
Vaporii
se condensează
renunțarea la
căldură latentă
Interne tub
de absorbție negru
suprafata se incalzeste
asa de lichid
se evaporă la vapori
(c)
în interiorul tubului interior,
lichidul revine
în wick
înainte de vaporizare din nou
exterior transparent
tub de sticlă
(vid interior)
wick interior
interior tub negru
(b)
G
T
f
Radiații
R
r, pg
Radiații
R
r, ga
Convecție
R
v, ga
C
f
T
ref
T
p
=
T
f
Placa, lichid
T
o
Ambient
T
cov
Capac de sticlă
TWIDELL PAGINARE.indb 97
01/12/2014 11:36
98
Solare de încălzire a apei
Circuitul analogic este prezentată în Fig. 3.12(b). Nu are nici convective cale între 'placa' (interior
tub) și sticlă (tub exterior) pentru a vidului. Singura convecție este din geam exterior la
mediu. Ia în considerare o unitate de lungime a tubului. T
p
= 100°C = 373 K, T
g
= 40°C = 313 K. Tratarea două
tuburi cât mai aproape paralel suprafețe, apoi prin (C. 18), (R3.2), (R3.5) și (R3.43) se obține prin algebrice factorizare:
1/r
pg
=
σe
p
e
g
(T
p
2
+ T
g
2
) (T
p
+ T
g
)
= 0.92 Wm
-2
K
-1
Luând caracteristice zonei O
pg
pentru a fi aceea a unui cilindru de lungime 1 m și înseamnă diametru 4.5 cm:
O
pg
= 2π(0.045 m)(1.0 m) = 0.28 m
2
prin urmare:
R
r
O
/
1
(0.92 Wm K )(0.28 m ) 3.88 KW
pg
pg
pg
2
1
2
1
=
=
=


Pentru convective pierdere pe unitatea de suprafață de suprafața exterioară, pentru un răspuns aproximativ, am putea folosi
(C. 15), cu suprafata O
g
=2
π (0.050 m)(1.0 m) = 0.31 m
2
. Astfel convective coeficientul de pierdere pe unitatea de suprafață este
aproximativ
h
v,ga
= o + bu = [(5.7 Wm
-2
K
-1
)
+ (3.8 Wm
-2
K
-1
m
-1
s)
× 5.0 ms
-1
) Wm 25
-2
K
-1
De (R3.45), deoarece
e
g
=e
aer
= 1.0, și F'
12
= 1.0, radiativ pierderea coeficient pentru suprafața exterioară este
h
r,ga
=4
σ [(T
g
+ T
o
)/2]
3
= 6.2 Wm
-2
K
-1

Pierderile prin convecție și radiație la exterior din sticlă pentru mediu sunt în paralel, și de
către (R3.6) h
= 1/r, combinate rezistenta termica este
R
ga
= 1/[(h
v. ga
+ h
r.ga
)O
g
]
= 1/[(24.7 + 6.2)Wm
-2
K
-1
× 0.28 m
2
]
= 1/[8.65 WK
-1
]
= 0,12 KW
-1
și
R
pa
= R
ga
+ R
pg
= (0.12 + 7.7)KW
-1
= 7.8 KW
-1
Notă modul în care rezistenta la radiatii R
pg
domină, deoarece nu există nici convecție la scurt-circuit' asta. Nu este
semnificativ faptul că mixte convecție formula (C. 15) se aplică la o suprafață plană, deoarece acesta va subestima
rezistență de la o suprafață curbă.
Deoarece fiecare 1 m de tub ocupă aceeași zonă colector ca o placă plană din zona 0,05 m
2
am putea spune
că rezistența echivalentă de unitatea de suprafață a acestui colector este r
pa
= R
pa
. 0,05 m
2
= .0.39 m
2
KW
-1
, deși
această cifră nu au aceeași semnificație ca pentru adevărat plăci plane.
Pentru a calcula echilibrul termic pe un singur tub, am act de faptul că aportul de căldură este proiectat zona de
tub interior, întrucât pierderile sunt la tot in afara de cel mai mare tub exterior. Fără căldură eliminat
de un lichid stagnantă, introducere energia solară este egal cu ieșire la pierderi, deci
τ
g
α
p
Gd (1.0 m)
= (T
p
− T
o
)/R
pa
T
p
− T
o
= 0.90 × 0.85 × 750 Wm
-2
× 0,04 m × 1,0 m × 7.8 KW
-1
= 180 K
dându - T
p

= (180 + 293)K= 473K = 200ºC pentru maxim (stagnare) temperatura.


Notă:
Această temperatură este mai mică decât cea enumerate pentru vitraj dublu placă plană în Tabelul 3.1. Cu Toate Acestea, T
p
(m)
și, mai important, temperatură de ieșire a T
2

atunci când nu există debit în tuburi, poate fi crescută prin


creșterea energiei de intrare în fiecare tub (de exemplu, prin plasarea unui alb sau suprafață reflectorizantă spatele tuburi),
care crește radiant de intrare de reflexie radiații unde scurte și reducerea efectului de vânt.
TWIDELL PAGINARE.indb 98
01/12/2014 11:36
§3.7 și aparatură de monitorizare
99
§3.7 ȘI APARATURĂ DE MONITORIZARE
Toate mașinile nevoie de monitorizare pentru a verifica dacă acestea funcționează corect,
care necesită instrumente și dispozitive de avertizare a informa operatorul/
gospodar. Acest principiu este universal acceptat pentru autoturisme și alte
autovehicule, cu o gamă de indicatori în fața șoferului. La fel
principiul se aplică la instalațiile de încălzire solare, deși funcționarea lor este mult
mai simplu. Instrumentele și lămpile de control ar trebui să fie plasate la nivelul ochilor
într-o poziție de multe ori observat de gospodar, de exemplu, pe un perete
adiacente la deschiderea partea de folosit cu regularitate ușă. Ascunse instrumente
sunt inutile. În plus, o copie a originalului instrucțiunile de detalii, planuri
operaționale și notă ar trebui să fie protejate într-o poziție evidentă. Peste
30 de ani sau mai mult durata de viață a dispozitivului, operatorii și gospodari
se va schimba și trebuie să fie re-informat. Odată construite, apă solare
de incalzire sunt dispozitive simple, dar chiar și așa, mulți proprietari nu reușesc să verifice
funcționarea și poate să nu reușească să beneficieze pe deplin de căldură gratuit.
Utile instrumente include următoarele:

Display de temperaturi.*

Senzori de temperatură (de obicei, termistori) pentru a afișa: colector
evacuare,* partea de sus a rezervorului,* jos a rezervorului,* jos a colectorului,
mijlocul rezervorului.

Pompa-ului* on/off lumini colorate.

Pompa de ore rula.

Back-up de încălzire pe* (important mai ales pentru electric immersers pentru
care senzorii și ceasuri de timp poate eșua sau de a fi inoportune, și deci
inutile de energie electrică este utilizat).
* indică cel mai de dorit de instrumente pentru un harnic gospodar.
Defectele (și acțiunile de remediere) care pot apărea în timpul o viață lungă de un
incalzitor solar de apa includ următoarele:

Murdar capacul de sticlă (inspectați cel puțin de două ori pe an și, dacă este necesar,
curat; rețineți că auto-curatare sticla poate fi folosit: a se vedea Caseta 5.1).

Defectarea pompei (indicată de diferența de temperatură pe colector
fiind prea mare).

Eroare a senzorului (de multe ori șoareci și rozătoare ciuguli cablurile!).

Siguranțele cu sufletul la gură (nu în sine o vina, dar indică un posibil defect).

Congelate și țevi de impuls (inadecvate de izolare și/sau poziționarea greșită).

Coroziune de Metal de design săraci folosind o combinație de metale (nu ar trebui să
apară cu design fiabil; anual de inspecție recomandate cu
măsuri de remediere, dacă este necesar).
Foarte puțini încălzitoare de apă solare sunt dotate cu căldură de metri
(proporțională cu produsul dintre debitul de apă și creșterea temperaturii) și
deci valoarea de încălzire nu este de obicei măsurată. Se poate deduce
TWIDELL PAGINARE.indb 99
01/12/2014 11:36
100
Solare de încălzire a apei
cu toate acestea, dacă ratei de pompare este cunoscut și pompa de ore de funcționare sunt
măsurate.
§3.8 ASPECTELE SOCIALE ȘI DE MEDIU
Solare de încălzire a apei este extrem de tumori benigne și acceptabil tech nologie.
Colectoare nu sunt deranjante, mai ales atunci când sunt integrate în acoperiș
de design. Nu există emisii nocive în exploatarea și fabricarea
implică nici deosebit de periculoase, materiale sau tehnici. Instalațiile
pot fi de așteptat pentru a fi eficace, cu foarte puțin costul serviciului pentru cel puțin
25 la 35 de ani. Instalarea necesită agenți să fie instruiți
în mod convențional în instalații și construcții, și a avut un scurt curs în
solar legate de principii. Tehnologia este acum dezvoltat și com-
comerciale în cele mai multe țări, fie pe scară largă (de exemplu, China, Turcia, Brazilia,
Grecia, Cipru, Germania, Israel) sau fără implementarea pe scară largă
(de exemplu, SUA, Franța și marea BRITANIE). Aceasta funcționează cel mai bine peste tot în timpul verii
și mai ales în zonele cu climat însorit (de exemplu, marea Mediterană), și în cazul în care
alternativele, cum ar fi gaz sau electricitate, sunt cele mai scumpe (de exemplu, Israel).
Unitățile proiectate pentru climate mai reci, cu amenințarea de îngheț de condiții
sunt mai sofisticate și mai scumpe decât cele realizate pentru țările
unde congelare nu se produce. La instalațiile solare pentru încălzirea apei discutate în
acest capitol pot fi utilizate la scară largă pentru încălzirea spațiilor, precum și pentru
încălzirea apei, mai ales dacă este legat la scară largă inter-sezon termică
de stocare.
Este important să se sublinieze că, în locații cu relativ mici de zi cu zi
de expunere la soare, fie din cauza latitudine sau nebulozitate (vezi Fig. 2.18), solare de apă
încălzitoare sunt mai benefice pentru preîncălzirea apei (sa zicem, la 30 ° C), atunci când nu
este un al doilea sistem pentru a finaliza încălzire (la, să zicem, 50 ° C). În marea BRITANIE, de
exemplu, un 4 m
2

colector este suficient de aproape 100% de aprovizionare pentru o familie


de două până la patru, cu utilizarea atentă, de la mijlocul lunii aprilie până la sfârșitul lunii septembrie, și va
incalziti în alte luni, deci de economisire pe alți combustibili pe tot parcursul anului.
Plăci de încălzire solară a apei sisteme sunt mai ieftine și oferă util
de căldură, în anumite circumstanțe (de exemplu, pentru ușor creșterea de temperatură a
apei din piscine, și este larg răspândită în statele UNITE ale americii).
În aproape toate cazurile, utilizarea energiei solare pentru încălzirea apei înlocuiește maro
(fosili) energie de la sursă. Acest lucru vă oferă beneficiile de îmbunătățirea
sustenabilității și mai puține gaze cu efect de seră, așa cum este descris în §1.2. Pentru acest
motiv, unele guverne subvenționa parțial de uz casnic achiziționarea de
instalații solare pentru încălzirea apei într-o încercare de a compensa "costurilor externe" de maro
energie (a se vedea Capitolul 17 pentru o discuție generală a costurilor externe și
politica de instrumente). La utilizarea combustibililor fosili înlocuit poate fi directă (de ex. încălzire cu gaz)
sau indirecte (de exemplu, gaz sau cărbune, energie electrică). Mai ales în țările mai reci,
înlocuirea este probabil să fie de sezon, cu 'solar deficit în
lunile mai reci fiind furnizate de încălzire electrică, încălzire centrală cazane,
sau de încălzire centralizată.
TWIDELL PAGINARE.indb 100
01/12/2014 11:36
§3.8 aspectele Sociale și de mediu
101
Până la sfârșitul anului 2011, totalul la nivel mondial de încălzire solară a apei capacitate (geamuri)
a ajuns la 232 de GW (termică), cu net anual plus (adică mai puțin
pensionări) de 50 GW
th
(REN-21 Raport de Stare 2012). (Într-strike instalat
capacitate pentru scopuri statistice, de 1 milion de m
2

de colector este echivalat


la 1GW de capacitate termică; vezi Problema 3.8.) Semnificative anual de
creștere indică faptul că pe termen mediu - pe termen lung perspectivele de apă solare
de încălzire este pozitiv, deși creșterea nu poate fi constantă, din cauza
pe termen scurt schimbări de politică și financiare 'crize'.
Un solar de apă sistem de încălzire poate fi instalat de către practice
gospodari, deși cei mai mulți oameni angajați instruiți și certificați de comercianți.
Colectoare (și pentru unele sisteme de rezervorul de apă, de asemenea) sunt de obicei
fixate pe acoperișuri, de o rezistență suficientă. În cele mai multe situații, o "convențională"
sistem de încălzire a apei este disponibil ca back-up și pentru condiții de iarnă.
Cu toate acestea, timpul de recuperare în economie de combustibil împotriva operaționale
costul unui sistem convențional este, de obicei cinci până la zece ani, care este
substanțial mai mică decât durata de viață a sistemului solar în sine (a se vedea Lucrat
Exemple 17.1 și 17.2).
Încălzitoare solare de apă, chiar și relativ sofisticate, pot fi
fabricate în cele mai multe țări de pe un mic sau de dimensiuni medii, oferind astfel
ocuparea forței de muncă și furnizarea de produse utile. Ei nu au nevoie să fie
importate și există, de obicei, o cerere, mai ales de la mijlocul
clasei și membri ai "verde" organizații. Tehnologia este modular
și pot fi scalate pentru utilizări comerciale, cum ar fi spalatorii si hoteluri.
Experiența timpurie din 1970 incoace condiția piață
stimulent pentru moderne de fabricație. Cea mai mare producția națională de energie solară
de încălzire a apei este în China, care reprezintă mai mult de 75% din
producție. Aici bază la prețuri mici sisteme cea mai mare parte oferi interne
de apă caldă, chiar dacă numai pentru o jumătate de an în climatul de iarnă și de mare
latitudine din China. Mai recent tuburi vidate colectoare formă mult de
Chinezii de producție, care contribuie în mod semnificativ exporturile.
Toate aceste caracteristici sunt exemple ale beneficiilor utilizării surselor regenerabile de energie
în general, în sistemele prevăzute în Capitolul 1.
REZUMAT CAPITOL
Încălzitoare de apă solare sunt utilizate pe scară largă și simplă aplicație a energiei solare, utilizat în peste
200 de milioane de gospodării din întreaga lume. Există relativ puține case și întreprinderi, care ar
beneficia de astfel de instalații, dar utilizarea lor este încă departe de a fi universal. Cele mai multe sisteme folosesc geamuri,
fără concentrare, colecționari, care, de obicei, ridica temperatura apei la 30 la 60°C peste temperatura ambiantă,
în funcție de insolație și de debit. Performanța unui astfel de colecție poate fi estimată folosind
formule standard de transfer de căldură, după cum este demonstrat în acest capitol. Principiile generale și
de analiză care se aplică pentru instalațiile solare pentru încălzirea apei se aplică, de asemenea, la multe alte sisteme care
utilizează activ și pasiv
mecanisme pentru a absorbi energia solară sub formă de căldură. Selectivă a suprafețelor și evacuat colectoare a spori
performanța de colecționari, la un cost acceptabil.
TWIDELL PAGINARE.indb 101
01/12/2014 11:36
102
Solare de încălzire a apei
ÎNTREBĂRI RAPIDE
Notă: Răspunsurile la aceste întrebări sunt în textul de la secțiunea relevantă
din prezentul capitol, sau poate fi ușor dedusă din aceasta.
1
Ce substanță ocupă cel mai mare volum într-o bună
nonevacuated izolator termic?
2
Ce tip de colector este cel mai potrivit pentru furnizarea de apă la (i)
~ + 60ºC; (ii) temperatura de fierbere?
3
Ce tip de incalzitor solar de apa este cel mai economic pentru a da plus
de căldură de la un bazin de înot?
4
De ce cele mai multe colectoare solare include un capac de sticlă? De ce sticlă și
nu de polietilena?
5
Care este diferența între un "activ" și "pasiv" solar
sistem de încălzire a apei?
6
Ce este un selectiv suprafață, și de ce este util în apă solare
de încălzire?
7
Numele dispozitivului utilizat în unele evacuat colectoare solare
care transferă căldura foarte ușor, de exemplu, care are foarte mici termică
rezistență.
8
De ce este eficiența tipică a unui colector solar mai puțin la 80ºC decât la
20ºC?
9
Ce este important de instrumente de bază pentru apă solare
de incalzire si de unde ar trebui să fie plasat?
10
Numele de trei căldură-mecanisme de pierdere prezent în toate încălzitoare de apă solare
și descrie metode pentru a reduce fiecare.
PROBLEME
3.1
Colector de Exemplu Lucrat 3.1 avut o rezistivitate la pierderi
din partea de sus a r
pa
= 0.13 m
2
KW
-1

. Să presupunem că partea de jos a


plăcii este izolat de mediul ambiant (încă) de aer de vată de sticlă
izolație cu k
= 0.034 Wm
-1
K
-1

. Ce grosime de izolare este


necesară pentru a asigura că rezistența la pierderile de căldură în partea de jos
este (a) egală și (b) de 10 ori rezistența de top?
T
g
T
o
T
cer
R
ga
R
pg
R
pa
T
p
Fig. 3.13
Termice rezistențele de Probleme 3.2.
3.2
Într-un adăpost plate-placă colector, transferul de căldură între
placă și aerul exterior de mai sus poate fi reprezentată de
TWIDELL PAGINARE.indb 102
01/12/2014 11:36
Probleme
103
de rețea este prezentat în Fig. 3.13, unde T
p
, T
g
și T
o
sunt
temperaturi de tablă, sticlă și aer, respectiv.
(a) Arată că:
T
g
= T
o
+ (R
ga
/ R
pa
) (T
p
– T
o
)
Verificați dacă, pentru T
p
= 70°C și rezistențele calculate în
Exemplul 3.1, acest lucru implică T
g
= 32°C.
(b) Recalcularea rezistenței implicate, folosind acest al doilea
aproximare pentru T
g
în loc de prima aproximare a
1
2
(T
p
+ T
o
)
= 45°C utilizate în exemplu, și să verifice că efectul
pe rezistența totală r
pa
este mic.
3.3
Un anumit plate-placă colector de-a doua sticlă acoperă. Trage o
rezistență diagramă care arată cum se pierde căldură de la placa la
împrejurimi, și se calculează rezistența (pentru unitatea de suprafață) r
pa

pentru pierderile
prin acoperă. (Presupune condiții standard de Lucrat
Exemplu 3.1.) De ce vor acest colector nevoie de mai groasă posterior de izolare
decât un singur-geamuri colector?
3.4
Calculați partea de sus rezistență r
pa

de o plat-colector cu un singur
capac de sticlă și un selectivă suprafață. (Presupunem standard de
condițiile de Exemplu Lucrat 3.1.) Vezi Fig. 3.2 litera(d).
3.5
Calculați partea de sus a rezistenței de o plat-colector cu
doubleglazing și selectivă suprafață. (Asume din nou standard de
condiții.) Vezi Fig. 3.2 litera(c).
3.6
Berea îmbuteliată este pasteurizat prin trecerea a 50 de litri de apă caldă (la
70°C) pe fiecare sticlă timp de 10 minute. Apa este reciclată, așa
că temperatura minimă este de 40°C.
(a) O fabrică de bere în Kenya propune de a utiliza energia solară pentru a încălzi
această apă. Ce formă de colector ar fi cel mai potrivit
pentru acest scop? Având în vedere că fabrica produce 65.000 de
umplut sticle în 8 ore de lucru pe zi, și că
radiația de la fabrica de bere poate fi presupus a fi întotdeauna cel puțin
20 MJ m
-2
zi
-1
(pe o suprafață orizontală), se calculează minim
colecta sau suprafața necesară, presupunând că nu există alimentare cu căldură pierderi.
(b) Rafina estimare necesar colector zonă, permițând
de obicei pierderile dintr-un singur-geamuri plate-placă colector.
(Face potrivit estimărilor pentru G, T
o
, u.)
(c) Pentru această aplicație, ar fi util folosind colectoare
cu (i) vitraje dublu-izolante; (ii) selectivă suprafață?
Justificați caz ca din punct de vedere cantitativ cât poți.
Sugestie: Utilizați rezultatele sintetizate în Tabelul 3.1.
TWIDELL PAGINARE.indb 103
01/12/2014 11:36
104
Solare de încălzire a apei
3.7
Ce se întâmplă cu un thermosyphon sistem noaptea? Arată că,
în cazul în care rezervorul este în întregime mai sus colector sistemul poate stabiliza
cu H
th

= 0, dar că un sistem cu rezervor inferior (în părți) va


avea loc o inversare a circulației.
Sugestie: construct temperatura de înălțime diagrame așa cum se arată în Fig.
3.8(b).
3.8
În luarea în considerare a capacității instalate în scopuri statistice, de 1 milion de
m
2

de colector este echivalat la 1GW de capacitatea termică. Verificați dacă


acest lucru este rezonabil factor de conversie, folosind insolație datele din
Capitolul 2.
Următoarele probleme implica mai sofisticate de analiză, dar poate fi
potrivit pentru studiu extins, poate că într-o clasă de grup.
3.9
Unele radiații ce ajung la placa de sticlă plate-placă
colector este reflectată de pe placa de sticlă și înapoi în farfurie,
unde o fracțiune
α de care este absorbit, așa cum se arată în Fig. 3.14.
(a) pentru a Permite mai multe reflecții, arată că produsul
τα în
(3.1) și (3.8) ar trebui să fie înlocuit cu
()
1 (1
)
fep
d
τα
τα
αρ
=
−−
în cazul în care
ρ
d
este o reflexie a sistemului de acoperire pentru difuze
lumina.
(b) gradul de reflexie al unei foi de sticlă crește considerabil pentru
unghiuri de incidență mai mari decât aproximativ 45° (de ce?). De
reflexie
ρ
d
poate fi estimată ca valoare de incidență de 60°;
de obicei
ρ
d
≈ 0.7. Pentru
τ = α = 0.9, se calculează raportul (τα)
fep
/
τα,
și observații cu privire la efectul pe echilibrul termic al plăcii.
3.10
Fin eficiență: Fig. 3.15 prezintă un tub și colector. Un
element de placă, zona dxdy, absoarbe o parte din caldura ajunge
Iradianța
Acoperă
Amortizor
placa
G
(1 –
α
)
τ
G
(1 –
α
)
τ
ρ
d
G
τα
(1 –
α
)
ρ
d
G
τα
G
ρ
τ
G
Fig. 3.14
Reflexii Multiple între colector acoperire(s) și placa (pentru Problema 3.9)
TWIDELL PAGINARE.indb 104
01/12/2014 11:36
Probleme
105
de soare, pierde ceva pentru mediul înconjurător, și trece la
restul de conducție de-a lungul plăcii (în x direcția) pentru legătura
regiunea de deasupra tubului. Să presupunem că placa are o conductivitate k și
grosimea
d, și secțiunea de placa de deasupra tubului este la constanta
temperatura T
b
.
(a) Arată că, în echilibru balanța energetică pe elementul de
placa poate fi scris
k
dT
dx
TT
Gr
r
(
)/
pa
pa
2
2
o
d
τα
=−−
(b) Justifica condițiile la limită:
dT
dx
la x
TTx
WD
0
0
(
)/2
b
=
=
=
=

(c) Arată că soluția de (a), (b) este
TT
Gr
T
T
Gr
mx
mWD
cosh
cosh (
)/2
o
pa
b
o
pa
τα
τα
−−
−−
=

în cazul în care
m
2
= 1/(kdr
pa
), și că de căldură care curge în legătură
regiunea din partea este
(W – D)F [
ταG – (T
b
– T
o
)/r
pa
]
în cazul în care fin de eficiență este dată de
F
mWD
mWD
tanh (
)/2
(
)/2
=


(d) Evaluează F pentru k
= 385 Wm
-1
K
-1
,
d = 1 mm, W = 100 mm,
D
= 10 mm.
Bond regiune
W/2
z
y
x
dx
D
W
(T − T
o
)/r
pa
T
b
τ
G
δ
Fig. 3.15
Secțiune transversală a unui tub și placă colector (pentru Problema 3.10).
TWIDELL PAGINARE.indb 105
01/12/2014 11:36
106
Solare de încălzire a apei
BIBLIOGRAFIE
General
Duffie, J. A. și Beckman, W. A. (2006) Solar de Inginerie a Proceselor Termice, 3rd edn. John Wiley and Sons,
New York. Standard de lucru pe acest subiect, inclusiv nu doar colecționari, dar, de asemenea, sistemele din care
fac parte.
Gordon, J. (ed.) (2001) Energie Solară – starea de arta, James & James, Londra. Zece capitole de solare
termice, fotovoltaice si geam experți; plus un singur capitolele privind politica și puterea vântului. A se vedea în special:
Wackelgard, E., Niklasson, G. și Granqvist, C. pe Selectiv solare de absorbție a vopselelor, și Morrison, G. L. pe
'colectoare Solare" și "Solar de încălzire a apei'.
Goswami, D. Y., Kreith, F. și Kreider, J. F. (2000), Principii de Solar Engineering, 2nd edn, Taylor și Francis,
London. Un alt manual standard de la nivel postuniversitar, deși până acum un pic datat.
Laughton, C. (2010) Solare Încălzirea Apei menajere: În Earthscan expert ghid pentru planificare, proiectare și
instalare, Earthscan, Londra. Principiile și aspectele practice.
Martin, C. L. și Goswami, D. Y. (2005) a Energiei Solare de Referință de Buzunar, International Solar Energy Society/
Earthscan, Londra. La îndemână compilație de date și formule, acoperă expunere la soare, proprietățile materialelor, colector
tipuri, PV configurații, și mai mult.
Referințe specifice
Născut, M. și Lup, W. (1999, 7 edn) Principiile Opticii, Cambridge University Press, Cambridge.
Teoria electromagnetică de absorbție, etc. Greu merge!
Brinkworth B. J. (2001) Solar de apă caldă menajeră sistem de performanță corelarea revizuit', Energie Solară, 71 alineatul (6),
377-387.
O revizuire aprofundată de "cutie neagră" analiza comparativă și standardele pentru apă caldă menajeră (ACM) sisteme,
inclusiv depozitare; pe baza de căutare de zboruri non-dimensional grupuri de parametri, care oferă
generalizat metodele de referință de performanță.
Aproape, D. J. (1962) 'performanța de încălzitoare de apă solare cu circulație naturală', Energie Solară, 5, 33-40. O
lucrare de teorie și experimente pe thermosyphon sisteme. Mai multe articole in curs de desfasurare in aceeasi revista a
dezvolta acest subiect.
Hutchins, M. G. (2003) 'Spectral selectiv materiale pentru eficienta vizibile, solare și radiația termică de control', în
M. Santamouris (ed.), Tehnologii solare Termice pentru Clădiri, James & James, Londra, p. 33-63.
Laughton, C. (2010) Solare Încălzirea Apei menajere: În Earthscan expert ghid pentru planificare, proiectare și
instalare, Earthscan, Londra. Principiile și aspectele practice.
Morrison, G. L. (2001) 'colectoare Solare', în Gordon (2001), pp. 145-221.
Peuser, F. A., Remmmers, K-H. și Schnauss, M. (2002) Sisteme Termice Solare, James și James, din Londra,
cu Solarpraxis, Berlin. Predominant în vedere mare solare de încălzire a apei de plante, această carte demonstrează
complexe de învățare curba de experiență comercială în Germania; citiți acest lucru pentru a aprecia inginerie cererile
de succes instalații de mari dimensiuni.
Politica și reglementarea
Menanteau, P. (2007) Măsuri de Politică pentru a Sprijini Solare de Încălzire a Apei: Informații, stimulente și reglementări,
Consiliul Mondial al Energiei și ADEME proiect privind politicile de eficiență energetică (autor la LEPII/ CNRS, Université de
Grenoble).
TWIDELL PAGINARE.indb 106
01/12/2014 11:36
Bibliografie
107
Site-uri și reviste
AIE-SHC (www.iea-shc.org) Solar de Încălzire și Răcire Grup de Lucru al Agenției Internaționale pentru Energie este
o lungă colaborare internațională, care piscine și publică de cercetare. Printre publicații gratuite este un
Tehnologia foaie de Parcurs pentru energia Solară de Încălzire și de Răcire (2012).
REN-21 (ren21.net) În domeniul Energiei din surse Regenerabile de Rețea pentru secolul 21 să publice anual Global Status
Report, care oferă date și observații privind capacitatea instalată (prin tehnologie), politicile naționale privind RE, etc.
Energie Solară. Cercetarea journal of International Solar Energy Society, acum publicat de Elsevier.
TWIDELL PAGINARE.indb 107
01/12/2014 11:36

Alte aplicațiile termice solare


CAPITOLUL
4
CONȚINUTUL
Obiective de studiu
108
§4.1 Introducere
110
§4.2 încălzitoare de Aer
110
§4.3 Cultură uscătoare
112
§4.3.1 vapori de Apă și aer
113
§4.3.2 conținutul de Apă al culturilor
114
§4.3.3 echilibrul Energetic și temperatura
pentru uscare
115
§4.4 Solare termice refrigerare
și de răcire
117
§4.5 desalinizare a Apei
120
§4.6 Solare sare-gradient de iazuri
122
§4.7 Solare concentratoare
123
§4.7.1 Bază
123
§4.7.2 Termodinamic limita la
raport de concentrare
125
§4.7.3 Derivare: Performanță
liniar de concentrație
128
§4.7.4 Parabolic concentrator bowl
129
§4.7.5 Fresnel de concentrare
lentile și oglinzi
130
§4.7.6 Non-imaging concentratoare
130
§4.8 solară termică Concentrată puterea
(CSTP) pentru generarea de energie electrică
132
§4.8.1 Introducere
132
§4.8.2 CSTP tipuri de sisteme de
135
§4.8.3 Adăugarea de depozitare, astfel de potrivire
solar de introducere a cererii de energie electrică 138
§4.8.4 Termochimice în buclă închisă
depozitare
139
§4.8.5 scară Mică CSTP
microgenerare
140
§4.9 Combustibil și chimice de sinteza din
energia solară concentrată
140
§Introducerea 4.9.1
140
§4.9.2 de producere a Hidrogenului
141
§4.10 aspectele Sociale și de mediu
141
Rezumat capitol
142
Întrebări rapide
142
Probleme
143
Note
148
Bibliografie
148
Caseta 4.1 Solare desicant de răcire
120
OBIECTIVE DE STUDIU

Apreciez multe utilizări ale termică solară
energie.

Înțeles solare de concentrare.

Prin urmare, să ia în considerare producerea de energie electrică.
TWIDELL PAGINARE.indb 108
01/12/2014 11:36

www.shahrsazionline.com
Lista de figuri
109
LISTA DE FIGURI
4.1
Două modele de aer de încălzire.
112
4.2
De căldură în circuitul de încălzire a aerului din Fig. 4.1. (a).
113
4.3
Psychrometric diagramă.
114
4.4
(a) schema de principiu a unui frigiderului cu absorbție; (b) absorbție Solară sistem de răcire;
(c) Solar desicant de răcire.
118
4.5
(a) fluxurile de Căldură într-un solar încă (b) O scară mică plutind încă în situații de urgență și la mare.
121
4.6
Un solar de sare-gradient de iaz.
122
4.7
(o) Fotografie și schiță a unei oglinzi parabolice (b) O parabolice liniare concentrator
(c) vedere a capătului de linie concentrator.
125
4.8
Parametrii geometrici utilizate în analiza matematică de maximă teoretică raport de concentrare.
127
4.9
Secțiunea transversală a unei lentile Fresnel.
131
4.10 Solare mașini de gătit cu reflexie colectoare la satul de El Didhir, nordul Somaliei.
131
4.11 Aspect tipic CSTP sistem.
133
4.12 harta Lumii directe normale de expunere la soare.
134
4.13 Principalele tipuri de CSTP colecționari.
137
4.14 Schema de variație în timp a CSTP generație, cu utilizarea de stocare a energiei termice și
energia auxiliară.
138
4.15 Două din cele patru CSTP sisteme (fiecare de 50 MWe) la Solnova stație de putere în Spania.
139
4.16 Disociere și sinteza de amoniac, ca un mediu de stocare de energie solară.
140
4.17 Pentru Problema 4.2: (a) bloc străpuns de tuburi paralele; (b) pori într-un pat de cereale; (c) volumul de
cereale pat.
144
4.18 Pentru Problema 4.5. O propunere de griji n trator sistem pentru generarea de energie.
146
TWIDELL PAGINARE.indb 109
01/12/2014 11:36
110
Alte aplicațiile termice solare
§4.1 INTRODUCERE
Radiația solară are multe alte aplicații decât încălzirea apei, astfel încât în
acest capitol ne progres treptat prin analiza unor alte termică
aplicații, folosind principiile de bază ale transferului de căldură și de stocare de
Revizuire 3. Solar clădiri sunt, probabil, cele mai importante astfel de
cerere, dar ca se integreze solare de încălzire și de răcire cu utilizarea eficientă
a energiei, acestea sunt incluse ca un aspect cheie din Capitolul 16.
Solar de aer-incalzire, §4.2 sunt baza de solar cultură uscătoare (§4.3). De mult
a lumii recolta de cereale este pierdut la atacul fungilor, care este împiedicat
de uscare corespunzătoare. Uscarea culturilor necesită transferul nu numai de căldură, dar
, de asemenea, de vapori de apă; astfel nu prea solare distilare de soluție salină sau apă salmastră
impur apă pentru irigare si de baut (potabila) de apă, §4.5. Absorbția
frigidere folosesc căldura pentru a produce frig; utilizarea lor în solar de refrigerare și
de răcire este explicat în §4.4. O metodă interesantă de a capta caldura solara
este solare sare-gradient de iaz, §4.6. Cu toate acestea, aplicarea în practică a
acestor trei aplicații a fost foarte limitată, în mare parte datorită costurilor
relativ la alternativele, nu mai puțin de refrigerare și de desalinizare condusă de
energie electrică solară din celule fotovoltaice.
Intensitatea maximă pe Pământ de radiațiile solare, fără a concentra-
tion este de aproximativ 1 kW/m
2

; această intensitate este compatibil cu procesele de viață,


dar insuficientă pentru centrale termice de intrare pentru mașini sau pentru produse chimice de
prelucrare. Prin urmare, în clar-nori climate ne concentrăm fascicul de radiații cu
concentrarea oglinzi de factori de până la
~100 în concentratoare liniare
și până la
~3000 din punct de concentratoare, în timp ce în același timp, creșterea
temperatura la care accentul să maxima de
~750°C în liniar concentra-
tor și
~De 3.500°C, punct de concentratoare. Cu toate acestea, un dispersate se concentreze
care se raspandeste energie peste un receptor de suprafață pentru a maximiza captarea energiei
funcționează la o mai mică temperatură. §4.7 consideră că aceste concentratoare
și §4.8 explică modul în creșterea fluxului de energie și creșterea
de temperatură sunt utilizate pentru motoarele de căldură pentru generare de energie (
concentrarea de energie termica solara). Rețineți că, concentrându-se colectoare de §4.7
avea numeroase alte utilizări, inclusiv fotovoltaice în generarea de energie electrică
(Capitolul 5) și sinteza de substanțe chimice și combustibili, cum ar fi hidrogen
(§4.9).
Capitolul se încheie cu o scurtă trecere în revistă a unora dintre cele sociale și
de mediu aspecte de tehnologiile discutate.
§4.2 ÎNCĂLZITOARE DE AER
Aerul cald este necesar să se încălzească clădiri (§16.4) și uscată (§4.3). Solar
încălzitoare de aer sunt similare la boilere solare descrise în Capitolul 3
pentru un lichid este încălzit prin contact cu un iradiate de suprafață într-un
colector. În special, efectele de orientare și mecanismele de
pierderile de căldură sunt foarte similare.
TWIDELL PAGINARE.indb 110
01/12/2014 11:36
§4.2 încălzitoare de Aer
111
Două modele tipice sunt prezentate în Fig. 4.1(a), cu o practică
de aplicare este prezentat în Fig. 4.1. (b). Rețineți că încălzitoare de aer sunt ieftine, deoarece acestea
nu trebuie să conțină lichide grele; prin urmare, ele pot fi construite de lumina,
materiale locale, și nu necesită protecție la îngheț. Revizuire 3 consideră că
transferul de căldură în detaliu; cu toate acestea, principiul de încălzitoare de aer este
simplu. Pentru aer de densitate
ρ, capacitate de căldură specifică c, debitul volumetric
Q fiind încălzită de la temperatura T
1
pentru a T
2
, utile fluxului de căldură în
de aer este:
P
u
=
ρcQ(T
2
− T
1
) (4.1)
Din tabele de date B1 și B2(a), rețineți că densitatea aerului este
≈1/1000
de apă, și capacitatea termică specifică
~1/4 din apa; deci, pentru
aceeași energie de intrare și de diferențele de temperatură, aerul are o mult mai mare
debitul volumetric Q. Cu toate acestea, deoarece conductibilitatea termică a aerului este
mult mai mică decât cea a apei de circumstanțe similare, transferul de căldură
de la placa de fluid este mult mai redusă. Prin urmare, încălzitoare de aer de
tipul prezentat în Fig. 4.1 ar trebui să fie construit cu rugoasă sau canelat
multistrat sau plăci poroase grile, pentru a mări suprafața și turbulențe
disponibile pentru transfer de căldură de la aer.
O analiză completă de interne de transfer de căldură într-un încălzitor de aer este complicată,
pentru că aceleași molecule transporta energie termică utilă și convective
pierderea de căldură, adică curge " în " la placa si de la placa de
acoperire sunt cuplate, cum este indicat în Fig. 4.2. De obicei prima aproximare
este de a ignora această cuplare și de a analiza încălzitoare de aer în modul de
încălzire a apei; vezi §3.2 și §3.3.2. În cazul în care componenta de radiație solară
incident perpendicular pe colector este G
c
pe zona, colector efi-
motivul este:
η
ρ
=
=

P
GO
cQ T T
GO
(
)
c
u
c
c
2
1
(4.2)
Căldura utilă P
u

este diferența dintre absorbit de căldură și


pierderile de căldură. Absorbit de căldură este o fracțiune f a radiației ajunge la
colector, de absorptance
o
p
prin capacul transparent de
transmisie
t
cov
. Dacă U
c

(pierderea de căldură factor) este pierderea de căldură pe unitatea de


colector de suprafață pe unitatea de diferența de temperatură dintre colector
de suprafață la T
p
și aerul înconjurător la T
o
, atunci:
P
u
= f O G
c
t
cov
o
p
− U
c
O (T
p
− T
o
) (4.3)
În cea mai simplă de modelare putem presupune o singură valoare pentru a colecta sau
placa de temperatură T
p
; în mai sofisticate de modelare colector este
clasificate ca fiind, cu aer care trece de la o zonă la alta.
Standard empirice de evaluare a sistemului de caracteristici este de a
măsura fluxurile de aer și temperaturi ca radiația solară G
c
variază.
Apoi
η este obținut din (4.2) și a complotat împotriva (T
p
− T
o
) /G
c
, așa cum se arată
TWIDELL PAGINARE.indb 111
01/12/2014 11:36
112
Alte aplicațiile termice solare
în Fig. 3.6. Mai multe informații sunt obținute folosind (4.1) pentru a obține P
u
și
complot împotriva (T
p
− T
o
), ca în (4.3). Dacă proprietățile materialului
t
cov
și
o
p
sunt cunoscute, atunci pierderea globală factor U
c

și colectarea fracțiune f
sunt obținute din panta și ordonata de interceptare.
§4.3 CULTURĂ USCĂTOARE
Cereale și multe alte produse agricole trebuie să fie uscate înainte de depozitare;
în caz contrar, insecte și ciuperci, care prospera în condiții umede, distruge-le.
Exemplele includ grâu, orez, cafea, ulei de copra (pulpă de nucă de cocos), anumite
T
2
Dur, negru
suprafață de absorbție
Capac de sticlă
(a)
(b)
T
1
Fig. 4.1
Două modele de aer de încălzire.
o
cu aer care trece peste o suprafață neagră;
b
un colector solar în Minnesota, statele UNITE ale americii, construit ca un negru grilă prin care aerul este
aspirat, astfel încălzire, și trece apoi în casă.
TWIDELL PAGINARE.indb 112
01/12/2014 11:36

§4.3 Cultură uscătoare


113
fructe, și, într-adevăr, din lemn. Vom considera uscare de cereale, dar în alte
cazuri sunt similare. Apa are loc în straturile exterioare ale bobului și
, de asemenea, în cadrul structurii celulare; aceasta durează mult mai mult timpul
de uscare a difuza departe decât fostul. Toate formele de cultură uscare implică
transferul de apă de la cultură la aerul din jur, așa că mai întâi trebuie să
se determine cât de mult de apă din aer se poate accepta ca vapori de apă.
§4.3.1 vapori de Apă și aer
La umiditatea absolută (sau 'concentrației de vapori')
c este masa de apă
de vapori în prezent 1.0 m
3
de aer la o anumită temperatură și pres-
sigur. Acest lucru devine maximă la saturație, deci, dacă am încerca să crească
c
dincolo de saturație (de exemplu, cu aburi), lichid de apă se condensează. La
saturație de umiditate
c
s
depinde puternic de temperatură (Tabelul B. 2(b)). O
teren de
c (sau unele legate măsură de umiditate) împotriva T este numit un
psychrometric diagramă (Fig. 4.3). Raportul
c/c
s

este umiditate relativă,
care variază de la 0% (aer complet uscat) la 100% (aer saturat).
Alte măsuri de umiditate poate fi de asemenea utilizat (Monteith și Unsworth
2008).
Ca un exemplu de utilizare a o diagramă psychrometric, luați în considerare Fig. 4.3.
Aer de la punctul a (30°C, 80% umiditate relativă și 25 g/m
3
absolut umed-
itate) este încălzită de la punctul B la 47°C. umiditatea relativă a
Radiații
(placa sau
grile)
Ieșire
Placa
Ambient
Transport
de încălzit
aerul de
încălzire
Convecție
Convecție
Conducere
Sticla
Radiații
T
cer
T
o
T
2
T
g
T
p
C
placa
T
ref
G
ta
Fig. 4.2
De căldură în circuitul de încălzire a aerului din Fig. 4.1. (a). Notă modul de circulație a aerului în termen de încălzire
face ca la ieșire temperatura T
2
mai mică decât placa de temperatură T
p

. Simbolurile sunt ca în
Capitolul 3; vezi, de asemenea, lista de simboluri de la începutul cărții.
TWIDELL PAGINARE.indb 113
01/12/2014 11:36
114
Alte aplicațiile termice solare
a scăzut de la aproximativ 35% (umiditatea absolută, adică masa de apă
conținut. rămâne aceeași). Aerul este apoi folosit pentru a usca o cultură, astfel încât
umiditatea relativă a aerului crește până la 100% pe măsură ce se răcește până la 30°C, acum
cu creșterea conținutului de apă reprezentat de creșterea absolută
umiditate de aproximativ 30 g/m
3
. Acest proces este considerat mai târziu a Lucrat în
Exemplul 4.1.
§4.3.2 conținutul de Apă al culturilor
Procentul de conținut de umiditate (bază uscată) w a unui eșantion de
cereale este definit prin:
w
= (m − m
0
)/m
0
(4.4)
unde m este masa totală a eșantionului "așa cum este" și m
0
este masa
substanță uscată în eșantion (m
0

poate fi determinat prin uscare într-un cuptor


(de exemplu, pentru lemn prin uscare la 105°C timp de 24 de ore)). În această carte vom
folosi, în general, această definiție a conținutului de umiditate ('greutatea uscată bază), care este
standard în silvicultură. În alte domenii ale agriculturii, conținutul de umiditate pe o
'greutate umedă baza', w', pot fi utilizate:
w '
= (m − m
0
)/m
= w/(w + 1)
(4.5)
10
20
O
C
B
30
40
50
60
40
100%
80%
60%
40%
25%
Relativă
umiditate
30
20
10
0
Temperatura bulb uscat/°C
Umiditatea absolută/gm
-3
Fig. 4.3
Psychrometric grafic (pentru presiune standard de 101,3 kN m
-2

).
TWIDELL PAGINARE.indb 114
01/12/2014 11:36
§4.3 Cultură uscătoare
115
Determinarea exactă a m
0
necesită îngrijire, și în mod ideal ar trebui să fie
măsurată într-un laborator în conformitate cu procedurile standard pentru fiecare
cultură sau produs. Pentru măsurarea de rutină la mai puțin de precizie, o varietate de
relativ ieftin instrumente pot fi utilizate, de exemplu, în funcție de
rezistența electrică a materialului. Este de asemenea important să realizăm că
există temperaturi maxime de uscare culturi pentru depozitare, astfel încât
produsul nu crapă și permite bacteriilor și a altor microbi pentru a intra,
producătoare de degradare și de toxine. Detalii suplimentare sunt în secțiunea de referință
la sfârșitul acestui capitol.
Dacă este lăsat timp suficient de lung, umed cereale va da apă la
aerul din jur până la cereale ajunge la conținutul de umiditate de echilibru
w
e
. Valoarea w
e
depinde parțial și de cultură, dar mai ales pe
temperatura și umiditatea din aerul înconjurător. De exemplu, orez în
aer la 30°C și 80% umiditate relativă (tipic de orez-zone de creștere) a
w
e
≈ 16%.
Rețineți că rata de uscare nu este uniformă. Mult de umiditate
în materialul de cultură este liberă în apă a avut loc în celulă porii,
care, după recoltare difuzează relativ ușor la suprafețele și
se evaporă (de exemplu, de la dispersate cereale răspândit în uscat). Toți ceilalți parametri
rămânând constantă, conținutul de umiditate se reduce la o rată constantă
ca acest vag loc de apă este eliminat. Restul de apă (de obicei 30
la 40%) este obligat să peretii celulelor prin legături de hidrogen, și, prin urmare, este
mai greu pentru a elimina; această umiditate este pierdut la un tarif redus. Este
important ca cerealele să fie uscate cât mai repede posibil, fără fisuri (de exemplu,
în câteva zile de la recoltare) la aproximativ 14% la 16% conținut de umiditate
pentru a preveni creșterea de ciuperci care se dezvolta in zonele umede sau parțial umed cereale.
Chiar dacă ciuperci muri, deșeurile chimice care rămân pot fi
otravitoare pentru vite și oameni. Odată uscate, cereale trebuie să rămână uscat, în
ventilate de stocare.
Combustibil-lemn este cel mai bun uscate in stive cu husa de ploaie, dar prin care aerul
poate trece cu ușurință. Uscare cherestea la conținutul de umiditate de echilibru, fără
încălzire, de obicei, durează între unul și doi ani.
§4.3.3 echilibrul Energetic și temperatura de uscare
Dacă nesaturate aerul este trecut peste materialul umed, aerul va prelua apa
din materialul așa cum este descris în secțiunea anterioară. Această apă trebuie să
fie evaporat, și de căldură pentru a face acest lucru provine din aer și material.
Aerul este astfel răcit. În special, dacă un volum V de aer este răcit la
T
1
pentru a T
2
în procesul de evaporare o masă m
w
de apă, apoi
m
w
L = ρcV(T
1
− T
2
) (4.6)
în cazul în care
L este căldura latentă de vaporizare a apei și
ρ și c sunt
densitatea și căldura specifică de " aer " (adică inclusiv vapori de apă) la
TWIDELL PAGINARE.indb 115
01/12/2014 11:36
116
Alte aplicațiile termice solare
EXEMPLUL 4.1
Orezul se recoltează la o bază uscată, conținut de umiditate de w
= 0.28. Condițiile ambientale sunt 30°C și 80% față
umiditate, la care conținutul de umiditate de echilibru de orez (bază uscată) este dat ca w
e

= 0.16. Calcula cât de
mult aer la 45°C este necesară pentru uscare 1000 kg de orez, în cazul în care condițiile sunt așa cum se arată în Fig. 4.3.
Soluție
Din (4.4), m/m
0
= w + 1.
Pentru 1000 kg de orez de la w
= 0.28, masa uscată este
m
0
= m/(w+1) = 1000 kg/1.28 = 781 kg.
La w
= 0.28, masa de apa = 1000 kg − 781 kg = 219 kg.
La w
= 0.16, masa de cultură m este dat de 0.16 = (m − 781 kg)/781 kg,
deci, m
= 1.16 × 781 kg = 906 kg, iar apa prezent = (906 − 781) kg = 125 kg.
Masa de apă de evaporat este, prin urmare, (219-125) kg
= 94 kg , care este egal cu 94/220 = 42% din
total de apă prezent.
Rețineți că aerul umed este mai puțin dens decât uscător de aer la aceeași temperatură și presiune. Acest lucru este pentru că
moleculele de apă au mai puțin de masă decât oxigen sau molecule de azot (și acesta este motivul pentru care aerul umed se
ridica pentru a
forma nori). Vom neglija acest efect mic.
Putem obține umiditatea absolută,
c, din aerul înconjurător intrarea în uscător în două moduri:
o
din Fig. 4.3 (punctul A), în cazul în care pe ordonată scară dă
c≈ 25 g/m
3
b
mai precis din Tabelul B. 2(b) la 30°C, pentru aer saturat
c = 30.3 g/m
3
, deci la 80%
c = 0.8 × 30.3 g/m
3
= 24.2 g/m
3
.
Umiditatea absolută de același aer după încălzire la 45°C (punctul B) are aproximativ 35% umiditatea relativă a aerului, ca de
dat din Fig. 4.3. Acest aer trece prin orez si extracte de 94 kg de apă, deci creșterea absolută
umiditate și apoi răcire la 30°C temperatura ambiantă (punctul C).
Atunci din (4.6):
V
m
cTT
(
)
(94kg)(2.4 MJkg )
(1.15 kg.m ) (1.0 kJkg K ) (45 30) C
13 000 m
w
1
2
1
3
1
1
3
ρ
=
Am

=

°
=



în cazul în care căldura latentă de vaporizare a apei la această temperatură este de 2,4 MJ/kg; alte date provin
din Anexa B.
presiune constantă de la temperatura medie, pentru temperatură moderată
diferențe.
Provocarea de bază în proiectarea unei culturi uscător este de a avea o adecvată T
1
și V pentru a elimina o anumită cantitate de apă m
w
. Temperatura
T
1

nu trebuie să fie prea mare, care ar sparge cereale, dar trebuie să fie
suficientă pentru a preveni condiții de umiditate relativă ridicată de durată pentru
perioade suficient de lungi pentru creșterea microbiană.
Calcul Exact ar lua în considerare variațiile parametrilor
L, ρ
și c ca aerul trece prin cultură pat. Cu toate acestea, pe ansamblu,
concluzia ar fi aceeași; uscare necesită volume relativ mari de
aer cald, uscat pentru a trece prin cultură în uscător. Uscare cu aer fortat
debitul este stabilit și tehnologice subiect, producătoare de produse sigure
TWIDELL PAGINARE.indb 116
01/12/2014 11:36
§4.4 Solare termice, frigorifice și de răcire
117
pentru piețele mari. Uscarea culturilor fără a forțat fluxul de aer este comun în
unele țări pentru producția de acasă, dar poate prezenta riscuri pentru sănătate; ea
este mult mai complexă pentru a analiza decât cu fortata a fluxului de aer, mai ales dacă uscarea
ori și temperaturile sunt limitate.
§4.4 SOLARE TERMICE REFRIGERARE
ȘI DE RĂCIRE
Căldura solară pot fi folosite nu numai pentru căldură, dar, de asemenea, să se răcească. Un
dispozitiv mecanic capabil de a face acest lucru este frigiderului cu absorbție (Fig. 4.4).
Mecanic, frigidere si instalatii de racire depinde de răcit material
renunțarea la caldura pentru a evapora un lichid de lucru. Într-un convențional electrice
(sau de compresiune) cu motor de frigider, fluidul de lucru este
recondensed de schimb de căldură la creșterea presiunii aplicate de motor. Într -
o absorbție frigider, căldura de la răcit materialul se evaporă
un agent frigorific care cicluri rotund sistemul 'alimentat' de o sursă externă de căldură
sursă. Absorbția de refrigerare, condus de o lampă cu flacără, a fost o dată
norma pentru off-grid locații, dar, odată cu apariția solare fotovoltaice
de putere, acționate cu energie solară electric-motorizate de refrigerare este acum comun.
Absorbția procesul de refrigerare depinde circulă două
componente: un agent frigorific și un absorbant, cu fiecare având propriile sale
interconectate circuit. Luați în considerare cele mai simple amoniac ciclu
de absorbție frigider, care are amoniac ca agent frigorific și apă lichidă
, ca absorbant. Aceste componente circula în doua bucle
(Fig. 4.4(a)).
1
Bucla 1: ciclul de agent Frigorific. În absorbant, amoniacul ca
agent frigorific se dizolvă în apă și căldura de reacție este eliberat în
mediu. Concentrat lichid trece la generator,
care este încălzită din exterior de către o flacără sau de către un colector solar. Aici
vaporii de agent frigorific fierbe off și trece în bucla 1 la crescut
presiunea la condensator, unde amoniac condensează lichid într-un
schimbător de căldură, cu căldură emise în mediul înconjurător. Sub presiune
înainte, lichidul de amoniac trece printr-un îngust
de extindere/restrângere a supapei de la care a "clipește" să apară ca un lichid/vapori de amestec
la presiune redusă. Curge mai departe, acest amestec trece prin
evaporator, unde căldura este eliminată din interiorul frigiderului,
provocând răcire. Cu un plus de căldură, fluxul de agent frigorific continuă,
predominant sub formă de vapori de amoniac, care trece mai departe la absorbant
în cazul în care ciclul se repetă.
2
Bucla 2: Absorbant ciclu. În absorbant, apa ca absorbant
se dizolvă în amoniac a vaporilor, și se amestecă trece de-a lungul combinate
secțiune cu bucla 1. În generator, amoniacul este fiert off pentru a
trece prin bucla 1, dar apa lichidă trece în bucla 2 înapoi la
absorbant pentru a repeta ciclul.
TWIDELL PAGINARE.indb 117
01/12/2014 11:36
118
Alte aplicațiile termice solare
Efectul net este: (a) căldură a fost eliminat prin condensator din
interiorul frigiderului; (b) de căldură a fost absorbit de la generator
la sursa de căldură (de exemplu, expunere la soare sau o flacără); (c) de căldură este emisă la
mediu, atât la condensator și absorbant.
Alte combinații de agent frigorific și absorbante sunt, de asemenea, utilizat (de exemplu,
NH
3
/H
2

/apa, sau, apa/LiBr). Căldură pentru generatorul poate fi de la o


flacără, de altfel căldură reziduală, de la un incalzitor electric sau de la solar
de energie. Absorbție, răcitoare și frigidere sunt simplu de utilizat, dar
poate avea nevoie de specialist de întreținere.
"Eficienței" de refrigerare este măsurată prin coeficientul de per-
formance (COP):
=
POLIȚIST
căldură eliminat din spațiu răcoros
energiei furnizate în mod activ din sursă externă
(4.7)
Pentru o absorbție mai rece, energie furnizate în mod activ la exterior
sursa este căldura aplicată "generator"; pentru refrigerare alimentat
Căldură
să-împrejurimi
(a)
Condensator
Amoniac
vapori
Căldură de externe
sursa de exemplu, solar
Generator
Bucla
2
Bucla
1
Apa
De înaltă presiune
Slab
soluție
Amoniac
vapori
Apa
absorbant
Absorbant
De joasă presiune
Căldură
să-împrejurimi
Mare
presiune
Low-
presiune
Evaporator
Căldură
eliminat
din interiorul
frigiderului
Supraîncărcarea
supapa
Puternică
soluție
Fig. 4.4
o
Diagrama schematică a unui absorbție frigider. Zig-zaguri aici reprezintă căldură
schimbătoare (nu rezistențe). Bucla 1: ciclul de agent frigorific; bucla 2: absorbant ciclu.
(Continuare p120)
Căldură
să-împrejurimi
(a)
Condensator
Amoniac
vapori
Căldură de externe
sursa de exemplu, solar
Generator
Bucla
2
Bucla
1
Apa
De înaltă presiune
Slab
soluție
Amoniac
vapori
Apa
absorbant
Absorbant
De joasă presiune
Căldură
să-împrejurimi
Mare
presiune
Low-
presiune
Evaporator
Căldură
eliminat
din interiorul
frigiderului
Supraîncărcarea
supapa
Puternică
soluție
TWIDELL PAGINARE.indb 118
01/12/2014 11:36
§4.4 Solare termice, frigorifice și de răcire
119
(b)
Outlet:
aer rece
Apa
Umed
de evacuare a aerului
Deshidratant roata
Cald
admisie
aer
Bariera de
încălzire Solară
cu aer
Cald
dehumified
aer
Prin evaporare
cooler
(c)
Fig. 4.4
(cont.)
b
Solare absorbție sistem de răcire instalată în 2004 pe acoperișul unei clădiri de birouri
în Madrid, Spania. Căldură pentru "generator" vine de la solar cu tub vidat
colectoare în partea de sus a fotografiei. Vizibile (de la stânga la dreapta în fotografie) sunt 'de răcire
turn, 105 kW chiller unitate, și rezervoare tampon de apă rece și caldă, respectiv.
Sistemul include un total de 72 m
2
de colectori (nu toate prezentate aici), și realizează o
cameră temperatura de 19°C chiar și la temperaturi ambiante
~40°C.
c
Solar desicant de răcire (vezi Caseta 4.1). Adsorbție pe volan preia umiditatea
din aerul de admisie (mai sus de barieră) și pierde umiditate (sub bariera) pentru fluxul
de încălzire solară cu aer.
TWIDELL PAGINARE.indb 119
01/12/2014 11:36

120
Alte aplicațiile termice solare
de motoare electrice, aceasta este energia electrică utilizată. Electrice
de refrigerare este norma întreaga lume, cu astfel de frigidere si racitoare, în mod normal,
având POLIȚIST
>1.5. Absorbție, răcitoare, în practică, COP ~0.7, și deci
sunt mai puțin eficiente din punct de vedere tehnic. Valoarea lor, cu toate acestea, nu este POLIȚIST, dar
utilizarea lor în situații în care nu există alimentare cu energie electrică sau, la o mai mare
amploare, unde low-cost de căldură poate fi folosit la intrare în ciuda lor relativ
mare a costurilor de capital (vezi de ex. Fig. 4.4(b)).
CASETA 4.1 SOLARE DESICANT DE RĂCIRE
Desicanți sunt materiale care absorb umezeala din aer și apoi eliberați de umiditate atunci când este încălzit;
gel de siliciu este un exemplu comun. În exemplul dat aici, căldura solară este folosită pentru uscare. Vrem
aer curat într-o cameră care este mai uscată decât în afara; Fig 4.4(c) arată o metodă. Aerul trece mai întâi
printr-o încet de cotitură roata în secțiunea care conține uscate desicant. Ca aerul umed trece
în jurul valorii de adsorbție, căldura latentă de absorbție (similar cu condensare) este eliberat, astfel încât parțial încălzire
aer, care, totuși, a avut semnificative de vapori de apă eliminate. Secțiunea în desicant
roata se rotește încet și viitoare trece printr-un flux fierbinte de încălzire solară cu aer, astfel încât să devină din nou uscat
('regenerata'), gata de a repeta ciclul. Între timp, cald, uscat (deshidratat) aerul trece printr -
o evaporare mai rece și apoi în cameră. Sistemul funcționează pentru că umezeala adăugat în
răcire prin evaporare este mai mică decât cea a eliminat în primele uscare prin adsorbție.
Alte solar legate de metodele de refrigerare și de răcire
În practică, energia electrică din surse convenționale bazate motorizate frigidere
si racitoare de a domina piața, cu off-grid funcționarea din ce în ce mai
alimentat de panouri solare fotovoltaice (Capitolul 5). Pentru clădiri în hot
climate, de răcire este un aspect integral al solare pasive de proiectare (cf. §16.4.3).
Putem nota că răcire prin evaporare în cald, climat uscat este simplu
și de încredere în cazul în care apa este disponibil pentru evaporare. Chiar și în cald,
climat umed, de răcire prin evaporare pot fi eficiente în cazul în care împreună cu solare,
deshidratarea (vezi Caseta 4.1).
§4.5 DESALINIZARE A APEI
Pentru comunitățile în regiunile aride și condițiile de deșert, o potabilă (în condiții de siguranță pentru a bea)
de alimentare cu apă este esențial. În plus, îmbunătățite de apă pot fi necesare
pentru culturi și scopuri generale. De exemplu, multe deșert regiunile
din apropiere provizii de apă de mare sau apă sărată care poate fi mai ieftin pentru a
purifica la nivel local decât la transportul în apă dulce. Aceleași regiuni, de obicei,
au încredere radiației solare de desalinizare tehnologie.
O astfel de tehnologie este solare distilare. Fig. 4.5(a) indică
fluxurile de căldură care poate fi descris prin căldură circuit. O reclamă
exemplu este la scară mică plutind încă, concepute pentru a naufragiat
mariners (Fig. 4.5(b)). La baza sa este un intern înnegrit "bazin" care
poate fi umplut cu o adâncime superficială de sărată/apă salmastră. Peste asta
TWIDELL PAGINARE.indb 120
01/12/2014 11:36
§4.5 desalinizare a Apei
121
este un mod transparent, vapori-capac etanș complet care cuprinde spațiul
de deasupra bazinului. Capacul este înclinat spre canal de colectare. În
funcționare, radiația solară trece prin acoperă și se încălzește apa,
ceva de care apoi se evaporă. Vaporii de apă difuzează și se mută
convectively în sus, unde se condensează pe cooler acoperi. La
condensat picături de apă, apoi glisați în jos capacul în bazinul
jgheabului.
Exemplul 4.2 arată că substanțială zone de sticlă (sau
plastic) sunt necesare pentru a produce suficientă apă proaspătă chiar și pentru o
comunitate mică, având în vedere că Organizația Mondială a Sănătății
recomandă minim băut zilnic cerința de
~3 L/(persoană/
zi). (Permițând apă pentru gătit, spălat, etc. ridică de zi cu zi
cerința de a
~30 L/persoană/zi.)
T
w
T
g
T
o
q
e
q
v
q
r
q
ga
G
G
Fierbinte
Cool
X
τG
ρ
(a)
Fig. 4.5
o
Fluxurile de căldură într-un solar încă; simboluri ca și mai înainte, cu indici: b de bază, e de evaporare,
v convecție, r radiație, w apă și - un ambient.
b
O scară mică plutind încă în situații de urgență și la mare.
(b)
TWIDELL PAGINARE.indb 121
01/12/2014 11:36
122
Alte aplicațiile termice solare
Economia de desalinizare depinde de prețul de alternativă
surse de apă proaspătă. Într-o zonă de mare sau precipitații moderate (
>40 cm/y) acesta
este aproape sigur mai ieftin de a construi o apă sistem de stocare decât orice solare
dispozitiv. Dacă la distanță de desalinizare este necesar (de exemplu, în zonele foarte uscate), apoi
o abordare alternativă utilizând energia fotovoltaică este acum, de obicei, mai ieftin decât
solare distilare. În această abordare, apa este purificata prin osmoza inversa,
cu apă pompat împotriva presiunii osmotice peste speciale
membrane care împiedică fluxul de material dizolvat.
1

Solare
fotovoltaice de energie pot fi utilizate pentru a conduce pompe, inclusiv orice este necesar pentru a
ridica apa din subteran.
§4.6 SOLARE SARE-GRADIENT DE IAZURI
Pentru cantități mari de temperatură scăzută de căldură (
<100°C),
convenționale colectoare descrise în Capitolul 3 sunt scumpe. O alternativă
de luat în considerare este o sare solare-gradient de iaz (de obicei abreviat 'solar
iaz') este o scară largă de colector, în mod eficient, folosind apă proaspătă ca partea de sus sa
acopere, sare de apă de mai jos pentru stocare de căldură și un fund negru suprafata ca
absorbant solar. Toată această apă este transparent. Dimensiunile unei mari
solar iaz poate fi de aproximativ 1 hectar, în suprafață și profunzime
~2 m, deci
conțin
~10.000 m
3

de stocate apă caldă. De construcție este de


convenționale lucrări de terasament, cu fund negru - captuseala folie de plastic si instalat
conducte; toate la un nivel relativ scăzut pe unitatea de cost.
Inițial iaz este umplut cu mai multe straturi de apă sărată, cu cel
mai dens strat mai mic (Fig. 4.6), la aproximativ 2 până la 3 m adâncime. Soare este
absorbită pe linie neagră peste partea de jos a iazului; prin urmare, cel
mai mic strat de apa este încălzită cel mai mult. Într-o obișnuită omogen
iaz, apa calda va fi atunci mai ușor decât împrejurimile sale și
s-ar ridica, care transportă căldura la aer de mai sus prin convecție liberă (cf.
§R3.4). Cu toate acestea, într-un iaz solar, stratul de jos este facut de mult
mai sărată decât cea de mai sus, astfel că, densitatea sa scade pe măsură ce se încălzește,
acesta rămâne în continuare mai dens decât stratul de mai sus; de asemenea, straturile de mai sus
că. Astfel convecție este suprimată, iar cel mai mic strat rămâne la
partea de jos, devenind mai caldă decât straturile de mai sus. De obicei "sare" este
NaCl, dar altele (de exemplu, MgCl
2
) au o mai mare saturație densitatea și deci ar putea
oferi o mai mare stabilitate atunci când iazul este fierbinte.
EXEMPLUL 4.2 DE IEȘIRE DINTR-UN IDEAL SOLARE ÎNCĂ
Expunere la soare într-un loc uscat, zonă însorită este de obicei de 20 MJ m
-2
zi
-1
. Căldura latentă de evaporare a apei este
2.4 MJ kg
-1
. Prin urmare, dacă toate căldura solară este absorbită de evaporare, și toate s-au evaporat apa este
colectate, de ieșire de la încă este:
20 MJm zi
2.4 MJ kg
8.3 kg zi m
2
1
1
1
2
=





(4.8)
Absorbție de căldură de jos
Foarte sărat
Sărat
Proaspete
Pierderile de căldură
Fig. 4.6
Un solar de sare-gradient de iaz
(schematică); convectie este
suprimat din cauza mai dens
straturile inferioare. Straturile inferioare
magazin de căldură de la Soare.
O cabina de polietilenă poate fi
folosit pentru a crește temperaturile
de suprafață și pentru a preveni turbulențe
de vânt.
TWIDELL PAGINARE.indb 122
01/12/2014 11:36
§4.7 Solare concentratoare
123
Desigur, stratul de jos nu se încălzește pe termen nelimitat, dar
echilibrează la o temperatură determinată de căldura pierdută prin conducție,
în principal, prin apă staționare de mai sus. Calculul arată că
rezistența la această pierdere de căldură este comparabil cu cel convențional placă
colector (Problema 4.3). Echilibru temperaturi de 90°C sau mai mult au
fost realizate în cel mai de jos strat. Completarea unui iaz solar poate dura mai multe
luni, pentru că dacă straturile superioare sunt adăugate prea repede rezultate
turbulențe stârnește straturile inferioare și distruge dorit de stratificare.
Cu un mare iaz solar, termică capacitate și rezistență poate
fi suficient de mare pentru a reține căldură în stratul de jos de la vară
la iarnă (Problema 4.3). Astfel interseasonal de stocare de căldură poate fi folosit
pentru încălzirea clădirilor. Solar iazuri au multe aplicații potențiale în
industrie ca o sursă constantă de căldură la moderat temperatură ridicată,
și este posibil să se producă energie electrică de la un iaz solar folosind un
speciale "temperatură joasă" motor termic cuplate la un generator electric
(a se vedea Caseta 13.1). Prototip solar iazuri fost primul construit în India;
un iaz solar la Beit Ha'Harava în Israel a produs o constantă și de încredere
de 5 MW(e) de furnizare de energie electrică la un cost nivelat de aproximativ 30 de USc/kWh
(Tabor și Doron 1990); un iaz solar, El Paso, Texas, a câștigat
mai mulți ani de experiență operațională, producerea de căldură, electricitate și
apă desalinizată pentru o apropiere de fructe fabrica de conserve (Lu et al. 2004).
Dezvoltarea continuă în mai multe proiecte internaționale, dar solare
iazuri nu au obținut o largă utilizare comercială pentru că laborios
de construcție și de lungă perioadă de timp-constante de funcționare necesită mulți ani de
experiență pentru a obține rezultate satisfăcătoare.

§4.7 SOLARE CONCENTRATOARE


§
4.7.1 Bază
Multe aplicații de căldură solară necesită temperaturi mai calde decât cele
realizabile de către chiar cel mai bun colectoarelor plane. De exemplu, un lucru
fluid la
~500°C poate conduce un motor termic convențional pentru a produce deteriorare me-
cal de lucru pentru producerea de energie electrică. Chiar temperaturi mai calde (la
~2000°C)
sunt utile pentru producerea de materiale refractare din pură condiții. The
scopul este de a colecta expunere la soare pe o suprafață mare și să se concentreze acest flux pe un
receptor mic; în practică, oglinzile sunt folosite pentru a concentra solare directe
bârna din senin condiții. Deoarece numai fascicul de radiatii pot fi
concentrate, concentratoare sunt utile numai în locuri cum ar fi California de Nord și
Africa, care au perioade lungi de soare luminos; energie solară
applicacondițiile în încețoșat climate cum ar fi Europa de Nord sau 'tropice umede trebuie să
se bazeze pe colectoarelor plane și panouri fotovoltaice cu nr de concentrare.
Teoria de astfel de energie solară de concentrare inițial derivat din
imagine optic dispozitive (de exemplu, telescoape); cu toate acestea, moderne de cercetare
și dezvoltare a dezvoltat non-imaging concentratoare ca mai
TWIDELL PAGINARE.indb 123
01/12/2014 11:36
124
Alte aplicațiile termice solare
benefic pentru energia solară de concentrare pentru ambele termice și
fotovoltaice de putere (a se vedea Roland lui Winston publicații de la Universitatea din
California). Aplicații au nevoie de o anumită temperatură ridicată, care nu este prea
mare și nu prea mici, iar o mare flux de energie; prin urmare, non - imaging
solare concentratoare sunt concepute pentru a oferi temperatura dorită cu
fluxul de energie în mod uniform în absorberul cu pierderi minime.
Un obiect ascuțit optic de imagine de calitate nu este de dorit. Solar aragaz prezentată în
Fig. 4.10 este o astfel de aplicatie.
Un concentrator cuprinde un colector care direcționează fasciculul de radiații pe
un receptor, în cazul în care radiația este absorbită și convertită în alte
forme de energie. Deci, în acest text:
concentrator
= colector (indice c) + receptor (indice r)
Concentratoare au colecționari care se concentrează pe un singur domeniu de interes, fie
pe "punct" de intrare pentru o carie, sau pe o linie de conducte. Fostul sunt
punctul concentratoare (Fig 4.7(a)); acestea din urmă sunt concentratoare liniare (Fig.
4.7(b)). Punctul concentratoare trebuie să fie orientate după Soare în două
dimensiuni: est/vest și nord/sud. Linie concentratoare rotește în jurul
orizontalei nord/sud axa receptor să urmeze altitudine de
Soare peste zi; de urmărire într-o singură dimensiune este mecanic
mai simplu și mai ieftin decât de urmărire în două dimensiuni.
La receptor este de absorbție de căldură componente. Acest lucru este, de obicei, un
recipient prin care un lichid trece la transportul de energie de la un foc
motorul. Cu toate acestea, R&D progresează pe alternative de receptoare, cum ar fi o
scădere flux de particule (de ex. nisip) care trece prin axat
fascicul, devine încălzit, poate 1000°C și apoi trece într-o căldură
container de transfer de la care căldura este transportată de la un motor. La
beneficiul este mai mare temperatura de transferul de energie și o mai mare
eficienta a motorului termic.
Energia solară concentrată de energie termică (§4.8) poate fi cel mai proeminent
aplicarea solare concentratoare.
Cu toate acestea, unele aplicații solare nevoie de un mare flux de energie la o
temperatură care nu este prea mare, dar semnificativ mai mult decât ambiant; aceste
non-imaging solare concentratoare (§4.7.6) sunt adecvate și mai ieftin decât
concentrându-se concentratoare, mai ales pentru că atinge suficient
de concentrare ( X
~5) fără mecanice de complexitate pentru urmărirea Soarelui.
Diafragma de colector are zonă de O
c
și iradiată zonă a
receptorul este Un
r
. În zona de concentrare raportul X
o
este definit ca raportul dintre O
o
la absorbție zona O
r
de receptor:
X
o
= O
c
/A
r

(4.9)
cu toate Acestea, în practică, aceste zone nu sunt ușor de definit cu exactitate,
mai ales de când fascicul concentrat nu va fi uniformă pe receptor
și de structuri de sprijin interveni. O mai semnificativ parametru este
de flux raport de concentrare
2
X
f
, fiind raportul dintre densitatea de flux la
TWIDELL PAGINARE.indb 124
01/12/2014 11:36
§4.7 Solare concentratoare
125
De urmărire
am
(b)
Solar
beam
(a)
Receptor
Receptor
suport
Punctul
concentrator de
structura de sprijin
Parabolic
oglindă
concentrator și
suport mecanic
structura
Fig. 4.7
o
Schiță a unei oglinzi parabolice cum colector pentru un punct concentrator; schita
explică concentrându-se.
b
Un parabolice liniare concentrator, arată receptor ca o țeavă orientate nord/sud,
de-a lungul focus; bare de sprijin pentru absorbție de receptor și de colectare oglindă sunt, de asemenea,
trase.
receptor pentru că, la concentrator. Pentru un ideal colector, X
o
= X
f
. Deoarece
X
o
este aproximativ egal cu X
f

, termenul de raport de concentrare (X) este adesea


folosit pentru ambele fara precizare.
§4.7.2 Termodinamic limită pentru concentrația
Temperatura de receptor, ca distinctă de puterea de intrare, nu poate
fi majorat pe termen nelimitat pur și simplu prin creșterea X
f
, deoarece, prin Kirchhoff
legile (§R3.5.4), receptorul temperatura T
r
nu poate depăși tempera-
tura T
s
de Soare. Termodinamice limitele maxime de flux concen-
administrarea raport X
f
pot fi calculate pentru: (a) punctul concentratoare, și (b) linie
concentratoare.
TWIDELL PAGINARE.indb 125
01/12/2014 11:36
126
Alte aplicațiile termice solare
(a) Punctul concentrator limita
Lasă Soarele să-i fie raza R
S
la distanța de la Pământ D
S
. Unghiul
subîntins de Soare este raza Pământului este
θ
S
, în cazul în care păcatul
θ
S
,
= R
S
/D
S
.
Sferice Soare, de suprafață 4
pR
S
2
, emite radiații de densitate de flux
la suprafața sa
f
s
=
sT
4
s
în cazul în care
s este Stefan-Boltzmann constanta, la fel ca în
§R3.5.5. Total flux emise de Soare este, prin urmare, 4
pR
S
2
sT
S
4
. La
Soare–Pământ distanta D
S,
acest total flux trece printr-o suprafață de arie
4
pD
S
2
cu densitatea de flux G
E
date de:
ps
p
s
s
θ
=
=
=
G
RT
D
TR
D
T
(4
) / (4
)
(
/
)
păcat
E
S
S
S
S
S
S
S
S
2
4
2
4
2
4
2
(4.10)
Acum ia în considerare un punct concentrator de la Pământ cu o deschidere de intrare
O
c
concentrarea radiației și așa încălzire un receptor de suprafata O
r

. La
concentrator este controlat să urmeze traiectoria Soarelui și concentrația este în
două dimensiuni pe un punct. De Kirchhoff, Legea lui (§R3.5.4) valoarea maximă a
temperaturii de receptor este temperatura de Soare, T
S

. La acest
maxim, receptorul este în echilibru termic, emite la fel de mult ca
o primește de la Soare, adică T
r
= T
S
. Prin urmare, radiațiile
receptor este:
s
s
s
θ
=
=
=
UN T
UN T
OG
UN T
(
)
(
)
păcat
r
r
r
S
cE
c
S
S
4
4
4
2
(4.11)
deci, concentrația maximă posibilă raport X
max
este
θ
=
=
=
=






X
O
O
R
D
D
R
1
păcat
1
(
/
)
f
c
r
S
S
S

S
,max
2
2
2
(4.12)
Folosind datele prezentate în Tabelul B. 7:
=
×
×
=
X
(150 de 10 m / 700 10 m)
46 000
f , max
9
6
2
(4.13)
Argumentul de mai sus presupune că receptorul este un corp negru și că
nici unul dintre radiația solară care este "pierdut" de la Soare la receptor
(de exemplu, prin absorbția în atmosferă), și că receptorul pierde căldură
numai prin radiație. Deci, în practică, X
f
< 46000.
(b) Linie concentrator limita
Linia de concentrare este numai în Soare/Pamant plan perpendicular
liniare receptor. Termodinamice argument acum trebuie să fie pe o singură
dimensiune, nu doi, ca pentru punctul de concentrator. Pe creștere
∆x în
lungime de colector, în modul a (4.11), dar acum, în singur
avion și cu plane insolație' G
'
E
.
(2
p R
s
dx ) s T
4
s
= G '
E
(2p D
s
d x) (4.14)
deci
G
T
R
D
T
.
păcat
E
S
S
S
S
4
4
s
s
θ
=
=
'
(4.15)
TWIDELL PAGINARE.indb 126
01/12/2014 11:36
§4.7 Solare concentratoare
127
Scut
Receptor
D
Oglinda
(colector)
D'
2
s
ψ
ζ
θ
Fig. 4.8
Parametrii geometrici utilizate în analiza matematică de maximă teoretică
raport de concentrare. Radiația solară este axat pe un receptor de pe suprafața Pământului
(nu la scară). Diagrama este de a fi interpretat ca un plan (bidimensional) pentru o linie de
receptor, dar rotită cu 360 ° în jurul Soare–Pământ axa pentru un punct de receptor.
La radiativ echilibru termic cu Soarele, receptorul temperatura
este egal cu T
S
. Apoi am echivala flux de intrare pe receptor la acest
maximă de temperatură cu ieșire flux la acest echilibru. Dacă d
c
este
"înălțimea" de diafragma liniare colector și d
r
"înălțimea" de
receptorul (presupus a fi perfect izolate pe partea din spate), apoi:
ds
d
ds
s
θ
d
='
=
d
T
Gd
d
T
T
d
(
x)
(
x)
deci (
x)
sin (
x)
r
r
E
c
r
r
S
c
4
4
4
(4.16)
Rearanjarea, valoarea liniară a concentrației raport d
c
/d
r
devine:
θ
=
=
=
×
×

X
d
d
1
sin
150 de 10 m / 700 10 m 210
f
c
r
9
6
(4.17)
Prin urmare termodinamice limită a concentrației de flux raport X
f

de un liniar concentrator este de numai 0,5% din care pentru un punct de concentrator.
Cu toate acestea, temperaturile extrem de ridicate nu pot fi necesare pentru
energie capturat și aplicații practice trebuie să fie cost-eficiente;
concentratoare liniare sunt mult mai ieftin de fabricat și opera decât
punctul concentratoare, astfel încât acestea au tendința de a fi opțiunea preferată.
O complicație în calculele de mai sus de X este că Soarele
radiază raza este dificil să se definească exact. Aplicațiile practice au
mulți alți factori care afectează concentrator de performanță. Unele dintre acestea
sunt acoperite în următoarea subsecțiune; vezi Lovegrove și Stein (2012) sau
Winston et al. pentru mult mai multe analize extinse. În Capitolul 5
utilizarea concentratoare solare fotovoltaice cu celule matrice este discutat pe scurt
(vezi Fig. 5.24).
TWIDELL PAGINARE.indb 127
01/12/2014 11:36
128
Alte aplicațiile termice solare
§4.7.3 DERIVARE: PERFORMANȚA DE CONCENTRATOARE LINIARE
Fig. 4.8 prezinta un tipic liniar de jgheab concentrator. Colector trase are o secțiune parabolică ca oglindă
de lungime L cu absorbție receptor de-a lungul axei sale. Pentru a înțelege acest lucru, mental extinde felie (subțire
avion) tot drumul înapoi la Soare, ca în derivarea X
f
(max)
de mai sus. Numai radiații în acest felie subțire este
axat pe o lungime de colector.
Axa este aliniat nord/sud, și jgheab este rotită automat în jurul axei sale, astfel încât în urma
Soarele în înclinați numai. Dacă am personifica Soarele, ne dăm seama că pentru un perfect aliniate concentrator, Soarele pare
la colector și vede doar imaginea de negru absorbant de umplere colector zona. Prin urmare,
puterea absorbită de absorbție a tubului este:
ρ-o
=
P
OG
abs
c
cb
(4.18)
în cazul în care
ρ
c
este o reflexie a concentrator,
o este absorptance de absorbant,
θ
=
O
DL cos
c
este
colector zona ca "văzut" de către Soare,
θ este unghiul de incidență (definit în Fig. 2.9) și G
b

este fascicul
de
radiație de la direcția de Soare. (Aceste simboluri sunt ca și în Capitolul 2 și Revizuire 3, și sunt în
lista de simboluri la începutul cărții, pentru că de la urmărire, la prânz solare atunci când jgheab de puncte
pe verticală,
θ este egal cu unghiul de la care Soarele este mai jos pe verticală.)
Scutul este prezentat în Fig. 4.8 reduce pierderile de căldură de la receptor. Se elimină, de asemenea, de intrare pentru unele
iradierii directe, dar acest lucru este nesemnificativă în comparație cu concentrate reflectă radiații absorbite
de la colector. Receptorul pierde radiații numai în direcții neprotejat de scut, adică
radiativ pierdere este o fracțiune (1
− ζ/p) din care ar fi emise din întreaga sa suprafață 2prl.
Prin urmare, puterea radiată pierderea de receptor este:
es
p
ζp
=

P
T
rL
(
) (2
)(1
/)
r
rad
4
(4.19)
în cazul în care T
r
,
e, r și L sunt, respectiv, temperatura, emisivitate, rază și lungime de tubul de receptor,
și
ζp
/
este în radiani fracțiune pentru care scutul previne radiativ pierdere. Pentru fiecare reflectate ray,
unghiul de reflexie este egal cu unghiul de incidență, astfel încât să reflecte Soarele subîntinde același
unghi ca Soarele vizualizate direct, adică 2
θ
S
. Pentru a minimiza pierderile de la receptor vrem mici r, dar
pentru a obține complet absorbit puterea P
abs
tubul trebuie să fie cel puțin la fel de mare ca imaginea Soarelui.
Prin urmare, pentru temperaturi ridicate, vom alege
θ
='×
rD
S
(4.20)
în notația din Fig. 4.8. Cu toate acestea, D' variază de la l
f
la nodul de la oglindă la 2l
f
(la nivel de puncte cu
receptorul) în cazul în care am
f
este distanța focală a parabolei.
În principiu, alte pierderi de căldură pot fi eliminate, dar radiativ pierderi nu pot. Prin urmare, prin stabilirea P
rad
=
P
abs
vom găsi stagnare temperatura T
r
:
T
G
D
r
pentru că
2
1
/
r
cb
1/ 4
1/ 4
ap
θ
es
p
ζp
(
)
=











(4.21)
în cazul în care
o este absorptance de receptor, egale pentru un non-selectivă suprafață de emisivitate e.
T
r
este un maxim atunci când scut permite radiații exterior doar la oglindă, adică
ζ → p − ψ. În practică,
jgheab de colecționari, de obicei, au subîntins jumătate de unghi
ψ ≈ p/2, astfel că distanța medie de la oglinda
accentul este

D
D
' 0.3
și
ζp
≈/2
. Cu aceste valori pentru D' și
ζ, și folosind (4.20) pentru a substitui r, la
geometrice termen în al doilea suport de (4.21) devine:
TWIDELL PAGINARE.indb 128
01/12/2014 11:36
§4.7 Solare concentratoare
129
p
ζp

p
θ
θ


'
'

× 0.3

D
r
D
D
2 (1
/)
/ 0.3
2(
)(1/ 2)
1
(
)
1
S
S
S
(4.22)
Deci maxim obținut de temperatură pentru condiții tipice în soare strălucitor G
b
pentru că
θ = 700 W m
-2
,
ρ
c
= 0.8,
a/e = 1, θ
s
= 1/210 (din (4.17) ) și
s = 5.67 × 10
-8
Wm
-2
K
-4
, este:
T
G cos
1200K
r
cb
S
(max)
1/ 4
ap
θ
esθ
=






=
(4.23)
Deși (4.21) sugerează că T
r

ar putea fi crescută și mai mult prin utilizarea selectivă suprafață cu


o /e >1, această abordare depinde de o și e fiind medii peste diferite regiuni ale spectrului
(cf. §R3.6). De §R3.5, definițiile lor sunt:




o
of
am
f
am
e
ef
am
f
am
=
=
ll
am
ll
am




d
d
d
d
,
,
în
0
în
0
B
0
,B
0
(4.24)
Deci, ca T
r
crește, corespunzătoare spectru de corp negru
f
l,B
(T
s
) de emițător își apropie negru
corpul spectru de Soare,
f
f
=
am
am
T
()
în
B
s
,
,
. Deoarece Kirchhoff, legea lui (§R3.5.4) prevede că
o
am
=
e
am
pentru fiecare
eu,
(4.24) presupune ca ca T
r
→ T
s
atunci
un/e →1.
T
r
= 1200K este o temperatură mult mai mare decât se poate obține din colectoarelor plane (cf. Tabelul 3.1). În
practică, obținute temperaturile sunt mai mici decât
T
r
(max)
pentru două motive principale:
1
Practic jgheaburi nu sunt perfect parabolic, astfel încât solar imagine subîntinde unghiul
R
Am
/
s
s
s
θ
θ
'>
=
.
2
Termică utilă P
u
este eliminat prin trecerea unui fluid prin absorbție, deci, evident, de răcire receptor.
Cu toate acestea, util de intrare de căldură poate fi obținută pentru un motor la
~700°C în bune condiții (a se vedea
Problema 4.5).
§4.7.4 Parabolic concentrator bowl
Concentrația poate fi realizată în două dimensiuni prin utilizarea unui
bowlshaped concentrator. Acest lucru necesită o mult mai complicat de urmărire
aranjament decât unul-dimensional jgheab, similară cu cea necesară pentru
'ecuatorială montare' de un telescop astronomic. Ca cu un liniar
colector, cel mai bun focalizarea este obținută cu o forma parabolică, în acest caz,
un paraboloid de revoluție'. Performanțele sale pot fi găsite prin repetarea
calculelor de §4.7.3, dar de data asta Fig. 4.8 reprezintă o secțiune
prin paraboloid. Receptorul este presupus a fi sferic. La
maxim absorbant temperatura este găsit în limita
ζ → 0, ψ → p/2 și
devine
T
G sin
4
r
c
s
(max)
un 0
2
2
1/ 4
ap t
ψ
esθ
=





(4.25)
Comparând acest lucru cu (4.23), vom vedea că concentrator acum pe deplin piese
Soare, și
θ
s
a fost înlocuit de (2
θ
s
/sin
ψ)
2
. Astfel
T
r
(max)
crește
TWIDELL PAGINARE.indb 129
01/12/2014 11:36
130
Alte aplicațiile termice solare
în mod substanțial. Într-adevăr, pentru cazul ideal, pacat
ψ=o=ρ
c
=
t
o
=
e = 1, lim-
vizită la temperatura T
r
= T
S

≈ 6000 K. În practică, temperaturile se apropie de


3000K poate fi atins, în ciuda prezenta imperfecțiuni în
urmărire, modelarea oglindă, și proiectarea receptorului.
§4.7.5 Fresnel concentrarea lentile și oglinzi
Principiul de o lentila Fresnel este ilustrat în Fig. 4.9. Pe dreapta este un
convenționale 'plano convex' obiectiv în care paralel de raze de lumină care intră
din dreapta sunt concentrate la un punct de pe stânga pe suprafața curbată.
Zona hașurată este solidă de sticlă, care pe o scară largă face ca obiectiv
foarte grele. Ingenios obiectiv pe stânga, proiectat de Fresnel în
1820, se obține același efect optic cu mult mai puțin de sticlă, având
curbura fiecare din cele curbate segmente (pe partea dreaptă a
schemei) precizie de potrivire de curbură a secțiunii corespunzătoare
de lentilă groasă.
O Fresnel oglinda este un aranjament de aproape plat reflectând
segmente, fiecare segment de potrivire curbura corespunzătoare
concentrându-se în oglindă în trei sau două dimensiuni, acesta din urmă fiind un liniar
oglindă. Fresnel liniar oglindă în Fig. 4.9 prezinta caracteristici cheie:

Oglinda ca un întreg este într-un singur loc.

Fiecare bandă segment de oglindă poate fi individual axat, în acest
caz pe liniar de absorbție receptor.

Banda segmente poate fi lung, oglinzi plane, deoarece linia exactă de focalizare
nu este dorit și non-imaging răspândit în medie pe receptor
este mai bine.

Viteza vântului la nivel scăzut este mai mic decât mai mare deasupra solului, astfel încât vântul
prejudiciul este mai puțin probabil decât o oglindă parabolică.

De curățare, cu banda segmente verticale, este mult mai ușor decât pentru o
oglindă parabolică.
Prin urmare, oglinzi Fresnel au unele beneficii importante pentru mari
concentratoare de peste oglindă convențională concentratoare, ca vor fi explicate în
§4.8.2 pentru concentrarea energiei solare.
§4.7.6 Non-imaging concentratoare
Secțiunile anterioare descrie cât de mare coeficienții de concentrare poate fi
realizat cu precizia geometrică și precisă de urmărire. Cu toate acestea,
concentratoare cu concentrație mai mică raport și cu non-imaging
caracteristici pot fi preferat, de exemplu, pentru a evita "puncte fierbinți" de
excesivă sau neuniformă intensitate pe module fotovoltaice (a se vedea Capitolul 5)
și pentru a simplifica urmărirea. De exemplu, acesta poate fi mai ieftin și la fel de
satisfăcătoare pentru a utiliza un 5 m
2
zona de non-de urmărire concentrator de concentra-
TWIDELL PAGINARE.indb 130
01/12/2014 11:36
§4.7 Solare concentratoare
131
Fig. 4.10
Solar mașini de gătit cu reflexie colectoare la satul de El Didhir, nordul Somaliei. Aici
solare mașini de gătit se substituie cărbune incendii, astfel, reducerea defrișărilor și ajutându -
post-tsunami reamenajare. (A se vedea http://solarcooking.org/Bender-Bayla-Somalia.htm și
http://solarcooking.wikia.com/wiki/Solar_Cookers_World_Network)
tion raport 5 cuplat la un 1 m
2
solare fotovoltaice cu celule matrice, decât să folosească 5
m
2
de static module fotovoltaice cu nr de concentrare.
Nu au de a urmări este un avantaj major pentru cele mai multe aplicații generale
la o scară mai mică, în ciuda astfel de instalații fiind mai puțin eficiente termic.
Fig. 4.9
Secțiune transversală a unei lentile Fresnel (stânga) și echivalentul său convențional plano-convexe de sticlă
lentile (dreapta). Notă cât de curbura la aceeași distanță de linia mediană este același
pentru ambele obiective. Același principiu poate fi aplicat la oglinzi, de exemplu, ca în Fig. 4.13. (b).
A se vedea, de asemenea, Fig. 5.24(c).
1
2
TWIDELL PAGINARE.indb 131
01/12/2014 11:36
132
Alte aplicațiile termice solare
De exemplu, low-cost, de înaltă temperatură, non-urmărire solare termice
colector de sisteme au fost dezvoltate de la Universitatea din California
Solare Avansate Institutul de Tehnologii. Evacuat solare termice
amortizoare sunt aliniate cu non-imaging reflectoare care se concentrează atât directe, cât
și indirecte expunere la soare pe tuburi vidate termice receptoare, astfel încât
capturarea 40% a fluxului solar de căldură la 200°C.
O singură aplicație pentru non-imaging concentratoare este în solar mașini de gătit, așa cum
se arată în Fig. 4.10. De exemplu, astfel de mașini de gătit sunt deosebit de utile
în taberele de refugiați, unde a crescut rapid populației devansează pe
lemn de foc de aprovizionare.
§4.8 SOLARĂ TERMICĂ CONCENTRATĂ PUTEREA
(CSTP) PENTRU GENERAREA DE ENERGIE ELECTRICĂ
§4.8.1 Introducere
Această secțiune analizează modul în care radiația solară este concentrată pentru a produce
suficientă căldură la temperatura necesară pentru producerea de energie electrică
de la motoare termice. Pentru că aceasta este comun pentru a apela de energie electrică "putere",
astfel de sisteme solare sunt numite concentrat (sau concentrare) solare în
energie termică (CSTP, sau CSP fără cuvântul 'termic'). Concentrația
poate beneficia, de asemenea, generarea de energie electrică din module fotovoltaice în
locații care sunt, de obicei, cloud gratuit, astfel încât autorii pot distinge între
CSTP (concentrarea energie termică solară) și CCA (concentrare solare
energie fotovoltaică), deși aceste abrevieri nu sunt comune.
După cum este indicat în Fig. 4.11, diverse componente pentru CSTP sunt
următoarele:

clima cu dominantă, cerul e limpede, prin urmare, solar fascicul de radiatii să
se concentreze;

solar câmp de colectori de concentrare solar fascicul de radiatii;

absorbție receptoare;

fluide de transfer de căldură;

schimbătoare de căldură;

turbine;

generatoare;

sistemele de răcire;

"opțional" auxiliare de putere/energie magazin;

substație electrică.
A se vedea, de asemenea, Fig. 4.13 și Fig. 4.15.
Având integrat de stocare termică este o caracteristică importantă a societății CSTP
plante; de asemenea, cele mai multe au putere de combustibil rezervă de capacitate. Cu aceste extra
caracteristici CSTP poate genera energie în mod continuu și după cum este necesar într-un utilitar de
rețea de distribuție (de exemplu, de echilibrare de ieșire din alte surse regenerabile,
cum ar fi variabile fotovoltaice și eoliene).
TWIDELL PAGINARE.indb 132
01/12/2014 11:36
§4.8 solară Concentrată de energie termică (CSTP) pentru generarea de energie electrică
133
Rețineți că nu este practic în cele mai multe aplicații solare pentru a utiliza pahar mare
de lentile, fie convenționale sau Fresnel design, din cauza greutății de
sticlă de care au nevoie. Oglinzi pe de altă parte, necesită doar o foarte mică
cantitate de materiale de suport pentru a oferi stabilitate mecanică la curbe
suprafață, și așa sunt de preferat deoarece solare concentratoare.
Randamentul unui motor termic (puterea la arborele de ieșire/intrare de căldură) îmbunătățește
ca temperatura sursei de căldură crește, așa cum sa explicat de către
Carnot teorie (a se vedea Caseta 16.1). Prin urmare, este extrem de important pentru a
furniza energie la temperatura cea mai ridicată compatibil cu energie
flux necesare și materialele folosite. Dintre toate sursele de căldură, concentrat
de radiație solară oferă cele mai ridicate temperaturi; într-adevăr, este teoretic
posibil să se producă temperatura de Soare într-un fascicul focalizat (punctul 4.7).
Prin urmare, dacă avem materiale si sisteme care pot tolera foarte mare de
temperaturi, un fascicul concentrat de radiație solară poate alimenta un motor
și, prin urmare, de a genera energie electrică foarte eficient. Rețineți că ultimul fascicul
trebuie să fie răspândit uniform pe toată zona de recepție a unui absorbant,
și nu ar trebui să fie concentrat brusc. Prin urmare, optica este non-imaging,
care este subtil diferit de imagistica cerința de telescoape.
Pentru generarea de energie electrică, puterea arborelui de motorul termic unități
generatoare cu foarte puțin mai departe pierderea de eficiență, astfel încât în ansamblu
eficiența este definită ca 'de energie electrică de ieșire/intrare de căldură'. "Motor" care poate
Fig. 4.11
Aspect tipic CSTP sistem (schematică), arată subsisteme pentru energia solară
de captare, de schimb de căldură, de stocare termică și electrică generație. Tipurile și
modalitățile de concentrator, fluide de transfer de căldură, schimbătoare de căldură, magazine de energie,
turbine și auxiliare de putere, toate pot varia. (Diagrama indică faptul că, în acest caz
generație este alimentat de o turbină cu abur.). A se vedea, de asemenea, Fig. 4.13, Fig. 4.14 și Fig. 4.15.
Sursa: Adaptat din AIE CSP foi de Parcurs Tehnologie (2010).
Câmp Solar
De stocare termică
Generarea de energie electrică
Căldură
de schimb
Căldură
de schimb
TWIDELL PAGINARE.indb 133
01/12/2014 11:36
134
Alte aplicațiile termice solare
fi o turbină cu abur, sau poate un motor Stirling.
3
În condițiile în direct
cer acoperit de nori iradiere, CSTP stații atinge eficiența de
~45%, care este
mai bună decât cea a cărbunelui și nucleare (
~30%) și asemănătoare
turbina cu gaze a centralelor electrice.
Rețineți că, în definiția clasică de stație de putere de eficiență
(energie electrică/căldură) nu am luat în considerare utilizarea
căldurii reziduale din centralele termice, la fel ca în 'combinată de căldură și energie
(CHP). Acest cel mai sensibil strategie îmbunătățește eficiența totală dacă este definită
ca puterea la arborele plus de energie termică utilă/de intrare de căldură'. CHP tehnologie poate fi
utilizată în CSTP, în special pentru instalații solare de multe ori pot fi ușor
amplasate aproape de facilități industriale capabil de a utiliza în caz contrar pierdere de căldură.
Fig. 4.12 identifică regiunile unde clima este potrivit pentru prelungit
solară concentrată aplicații. Multe dintre zone sunt deserturi, care
au avantajul de terenuri ieftine și dezavantajele de acces redus,
limitat de alimentare cu apă și, cu mici densități ale populației din apropiere, la
necesitatea de a exporta energie pe distanțe lungi. Pentru Europa, numai
potrivit zonă este sudul Spaniei; prin urmare, în continuare atragerea de CSTP în
Africa de Nord transmiterea de energie electrică în întreaga Mediterană. La
Orientul Mijlociu și Australia au o oportunitate majoră–, dar încă nu au
această oportunitate ca ei au, de asemenea, de importante resurse de petrol sau de cărbune. Acolo
este un potențial considerabil în sud-vest de statele UNITE ale americii.
Este important să realizăm că, motoare cu abur și turbine necesita o
mare cantitate de apă, chiar dacă cea mai mare parte din apa este reciclată. Evident
semnificative de alimentare cu apă nu este disponibil în general în zonele deșertice; în astfel de
situații aplicatii solare nu necesită semnificative de aprovizionare cu apă sunt
favorizate (de exemplu, energie fotovoltaică).
Fig. 4.12
Harta lumii directe normale de expunere la soare (DNI, adică focusable radiația solară), cu scară
în kWh/(m
2

y). Mai ușoară umbrire reprezintă cea mai mare expunere la soare. DNI de cel puțin
1800 kWh/(m
2

y) (5kWh/(m
2
zi)) este necesar pentru CSTP. Rețineți că această hartă nu
arăta de alimentare cu apă necesară pentru CSTP instalații.
TWIDELL PAGINARE.indb 134
01/12/2014 11:36

§4.8 solară Concentrată de energie termică (CSTP) pentru generarea de energie electrică
135
CSTP centrale au progresat rapid de la R&D, proiecte la
scară comercială investiții. Stații individuale (plante) au
capacitatea de generare de energie electrică din
~10 kW (microgenerare) la mai mult de
200 MW. Lumea instalată de generare a energiei electrice capacitatea de CSTP
este semnificativă, în creștere de la 1.600 de MW în 2009 la 10.000 MW (10 GW)
de la aproximativ 25 de multi-megawatt (a capacității de energie electrică) de plante la nivel mondial
până în 2012 (conform European Solar Thermal energie Electrică de Asociere).
CSTP este acum adoptat ca o formă de utilitate 'mainstream generație,
în special în Spania de sud și California. În unele țări, CSTP
este deja costuri competitive în perioadele de vârf ale cererii de energie electrică. Cu
creșterea în continuare a scară de utilizare și succesive elementare de tehnologie
îmbunătățiri, CSTP este de așteptat să devină o sursă competitivă de
energie în vrac în vârf și intermediare sarcini până în 2020, și de sarcina normală
de alimentare până în 2025-2030.
§4.8.2 CSTP tipuri de sisteme de
Există patru tipuri principale de CSTP a sistemelor în dezvoltare comercială
și de a folosi, ca nume de tipul lor de colector: (1) jgheab parabolic;
(2) liniar Fresnel; (3) receptor central (turn de putere); și (4) vas
(paraboloid) (vezi Fig. 4.13). În plus, există mai multe variante, de exemplu,
pentru structuri, sisteme de control, fluide de transfer de căldură, căldură
schimbătoare, cuplaje și operaționale temperaturi. Caracteristici suplimentare pentru a
prelungi timpul de generație includ: (5) auxiliare de energie și căldură
de stocare.
(a) jgheab Parabolic, line-a concentrat CSTP
Jgheab Parabolic concentrator roti colector și receptor împreună
prin intermediul a doua zi pe orizontală fixă axa nord – sud, adică există
doar o singură axă de orientare în comparație cu cele două axe de un fel de mancare
sau sistem de turn. Sistemul de deschidere este, prin urmare, nu perpendicular
pe fascicul (cu excepția de două ori pe an la Tropice), și așa mai extins
oglindă zone sunt necesare în comparație cu feluri de mâncare. Mișcările sunt
însă simple și robuste. Oglinzile se concentreze solar fascicul de radiații
pe o centrală receptor liniară, care este în esență o țeavă conținând o căldură
fluid de transfer (de obicei un ulei, deși folosirea sistemelor de apă/abur, topit
săruri sau cu aer comprimat au fost dezvoltate). În practică receptorul
are o structură sofisticată a reduce pierderile de căldură, de natură să includă un
evacuate spațiu interior, izolatoare și reflectorizante scuturi, și, probabil,
selectiv suprafețe. Transferul de lichid trece la o poziție centrală în cazul în care
căldura este transferată de la un motor, care puteri de energie electrică
generator. În cazul în care transferul de lichid este abur, acesta poate fi folosit direct într-un Rankine
ciclul de turbină cu abur (Caseta 13.1), sau dacă un ulei mineral, căldura poate fi
transferat la căldură abur indirect, ca și în Kramer Junction solar power
station este prezentat în Fig. 4.13(a).
TWIDELL PAGINARE.indb 135
01/12/2014 11:36
136
Alte aplicațiile termice solare
(b) Liniară Fresnel CSTP
Lentile Fresnel si oglinzi (Fig. 4.9) sunt segmentate pentru a avea un profil plat,
menținând în același timp efecte optice convenționale de lentile și oglinzi.
Ele sunt ușoare și low-cost. Fig. 4.13(b) arată o Fresnel de reflexie
concentrator axat pe un receptor liniară; concentratorul este format
de a avea timp segmente orizontale plate sau ușor curbate oglinzi. Fiecare
oglinda este rotit în mod independent, să se concentreze pe fix centrală
orizontală receptor. Deoarece non-imagistica optica este de preferat, Fresnel metoda
permite reglajul fin pentru a obține dispersate concentrare pe receptor.
Receptorul este în staționare, în permanență, de-jos cu care se confruntă (atât în caz contrar
izolate) și fixe separat de concentrator (deci care nu necesită
articulații mobile pentru transfer termic lichid; un avantaj în practică peste
multe parabolice concentrându-se sisteme). Deoarece oglinzile sunt într-un
plan orizontal aproape de sol, de întreținere și de curățare sunt mai ușor
decât pentru concentratoare parabolice, și vântul perturbatii și daune
ar trebui să fie redus în mod semnificativ.
Din optica acestora sunt echivalente, analiza unui jgheab parabolic
concentrator de la punctul 4.7.3 se aplică, de asemenea, pentru o Fresnel oglindă concentrator.
(c) receptor Central (turn de putere)
Fig. 4.13(c) prezinta o serie de oglinzi (heliostate) de reflexie solară
directă a fasciculului într-un ridicat receptorul cavitate în partea de sus a unui turn central.
Oglinzile sunt de obicei plate (pentru simplitatea și ieftinătate), dar poate fi
ușor parabolic, subliniind punctele de fixare. Fiecare oglindă trebuie să fie
controlat individual în două axe să se concentreze pe cavitatea cum apune Soarele
poziție se mișcă prin zile și ani. Strict procedurile de siguranță și
proiectarea atentă sunt necesare pentru a preveni accidentale concentrându-se pe alte
componente sau personal. Cu timpul, există multe variații de
temperatură, vânt și alte condiții de mediu, astfel încât de proiectare și
întreținere provocare este considerabilă. Un avantaj al centrală
deschidere mică receptor este că la temperaturi foarte ridicate
≥1000°C pot fi
obținute; de exemplu, în mod direct puterea de turbine cu gaz, pentru ciclu combinat
sisteme (de exemplu, cu prima etapă pierdere de căldură pentru a fi folosit pentru a alimenta o
secvențială turbina sau pentru testarea materialelor).
(d) Antena CSTP
Antena CSTP sisteme individuale de receptor, motor și
generator situat permanent la centrul de fiecare paraboloid oglindă, așa cum
se arată în Fig. 4.13(d). Ca cu turnuri de putere, receptor temperaturile
pot fi foarte ridicate ( >
~
1000 C
o

). Motorul este, de obicei, un Motor Stirling,


deoarece aceasta poate fi alimentat de la o sursă de căldură externă și este, în
principiu, simplu și eficient. Eficiența globală este probabil să fie cu siguranță
mai bine decât linia concentratoare și în practică mai bine decât turnuri de putere.
Cu toate acestea, utilaje grele trebuie să fie suspendat de la centrul de
fiecare oglindă, astfel încât necesită un puternic și scump structură dinamică.
TWIDELL PAGINARE.indb 136
01/12/2014 11:36
§4.8 solară Concentrată de energie termică (CSTP) pentru generarea de energie electrică
137
(c)
(d)
Fig. 4.13
Principalele tipuri de colectoare CSTP
o
Jgheab Parabolic. Imaginea arată partea de instalație de la Kramer Junction, California,
care are o capacitate totală de 354 MWe și acoperă mai mult de 400 de ha. Cu un raport anual
de ieșire
~660 GWh/an, 24-ore factor de capacitate este de 21%. Fiecare urmărire colector are cinci
segmente și este de aproximativ 10 m de-a lungul său de două-dimensional de urmărire axa. (Compara
dimensiunea de tehnicieni în fotografie.) Fluidul de lucru este încălzit la 400°C în receptor
absorbție conducta de la focus de fiecare jgheab parabolic. Construit între 1984 și 1991,
acest complex cuprinde cele mai multe dintre CSP din lume până în anul 2005.
b
Liniar Fresnel reflector sistem. Partea a 5 MW Kimberlina stația de la Bakersfield,
California, construit in anul 2008. Fluidul de lucru este de aburi la 400°C.
c
Turn de putere. Fotografie prezinta PS10 de lângă Sevilla, Spania. Comandat în anul 2000, este pe
lume prima putere comercială turn. Acesta este evaluat la 11 MW, este de 115 m de mare, are 624 de
heliostate fiecare de 120 m
2

, pentru a produce abur saturat la 275°C în receptor. Acesta


include 1 oră de stocare a energiei ca apa sub presiune.
d
Fel de mancare. Fotografie prezinta 500 m
2

mare vas de la Australian National University, construit


în 2010, care este format din 384 de oglinzi mici, fiecare cu o reflectivitate de 93%.
În comparație cu un prototip timpuriu, construit în 1994, și-a îmbunătățit sistemele de urmărire,
oglinzi, și de receptor.
(a)
(b)
TWIDELL PAGINARE.indb 137
01/12/2014 11:36
138
Alte aplicațiile termice solare
Este avantajos în anumite privințe, că fiecare fel de mâncare sistemul este
independent, astfel încât sistemul este modular'. Încă multe, relativ mici,
cuplate motoare și generatoare necesare pentru scara de utilitate planta cerere
semnificativă de întreținere.
§4.8.3 Adăugarea de depozitare, astfel de potrivire solare de intrare la energie electrică
cerere
Presupunând că sistemul de urmărire solare concentratoare este de lucru
în mod corespunzător (acestea sunt foarte fiabile și precise) și nu este continuă,
cer acoperit de nori, atunci puterea de intrare pe un soare de urmărire CSTP colector într-o
locație favorabilă va rămâne aproape de maxim la aproximativ 9 a. m.
la 5 p. m. de-a lungul anului. Încorporează stocare a căldurii vă permite
generație în seara (Fig. 4.14). Cu toate acestea, o schimbare în claritate de
aer sau cloud va scădea această putere pe timp de noapte și există, desigur,
previzibil intrare zero. Prin urmare, ca cu cele mai multe surse regenerabile de energie sup-
straturi, există o variabilitate considerabilă, dar cu CSTP aceasta este cea mai mare parte complet
previzibil. Cererea de energie electrică în multe caldă, însorită locuri vârfuri în
după-amiaza, atunci cand aparatele de aer conditionat sunt de lucru mai greu. Acest lucru coincide
cu vârf de ieșire din CSTP, care ajută viabilitatea comercială pentru că
valoarea aprovizionării cu energie electrică în orele de vârf este mai mare. Cererea este
încă mare în seara, deci o reclamă CSTP generatoare de companie va
crește veniturile prin stocarea de energie de zi pentru a genera energie electrică
mai târziu. În practică, extinderea generație de aproximativ trei sau patru ore până la
8-9 p.m. folosind energia stocată poate fi util. Cele mai multe
plante comerciale au, de asemenea, auxiliare generarea de energie disponibile din combustibili fosili
sau biocombustibili, folosind un motor convențional generator în paralel cu
CSTP (așa cum se arată în Fig. 4.11), acoperind perioade ocazionale de
nebulozitate, sau care să permită ieșirea pentru a continua toată noaptea, dacă din punct de vedere comercial
util.
Fig. 4.14
Schema de variație în timp a CSTP generație, cu utilizarea de stocare a energiei termice și
de alimentare auxiliară.
0
50
40
30
20
10
0
6
2
4
8
10
14
16
20
22
Firma
capacitate de linie
La depozitare
Din
depozitare
12
18
24
Combustibil de rezervă
Solare directe
Timp de zi/h
Putere/MW
TWIDELL PAGINARE.indb 138
01/12/2014 11:36
§4.8 solară Concentrată de energie termică (CSTP) pentru generarea de energie electrică
139
În Fig. 4.11, excesul de căldură de la colector de transfer fluidul trece
printr-un schimbător de căldură unde chimice sarea s-a topit si lichidul
de sare a avut loc într-un rezervor izolat. Atunci când generatorul are nevoie de un plus de intrare (de exemplu,
în seara), sare topită trece înapoi prin schimbătorul de căldură pentru a
stimula temperatura fluid de transfer. În acest fel, energia electrică poate fi
generat pe timp de noapte, așa cum se arată în Fig. 4.14. Fig. 4.15 este o fotografie de o parte de
o mare CSTP sistem în Spania, care include aproximativ șase ore termice
de stocare.
§4.8.4 Termochimice în buclă închisă de stocare
O alternativă și mai sofisticate procedura are fluid de transfer
o termochimice mediu de stocare, de exemplu, amoniac disociat, după cum
este ilustrat în Fig. 4.16. Astfel de sisteme, propus inițial de către Carden și cu
dezvoltarea ulterioară (Dunn et al. 2012), au avantajul de care căldura absorbită
de la un receptor este imediat stocate în formă chimică, fără ulterioare
pierderea in transmisie termica la o centrală termică a motorului. Chimice
, prin urmare, devine o energie store, care pot fi stocate și transmise
pe distanțe lungi inainte de energie este în cele din urmă eliberată sub formă de căldură,
poate pentru generarea de energie.
În original procedură (a se vedea Fig. 4.16), expunere la soare este colectat într-un punct
concentrator și s-a axat pe un receptor în care amoniacul gaz (la mare
presiune,
~300 de atmosfere) este disociat în hidrogen și azot.
Această reacție este endotermă, cu
∆H = − 46 kJ (mol NH
3
)
-1

obținută
din căldura solară. Disociate de gaze trece la o instalație centrală, unde
H
2
și N
2

sunt parțial recombinat în sinteza camera, folosind


un catalizator. Căldura din această reacție poate fi folosit pentru a conduce un extern
Fig. 4.15
Două din cele patru CSTP sisteme (fiecare de 50 MWe) la Solnova stație de putere în Spania.
TWIDELL PAGINARE.indb 139
01/12/2014 11:36
140
Alte aplicațiile termice solare
motor termic sau alt dispozitiv. Fluxul de sinteza camera
este separat de răcire, astfel încât amoniac se lichefiază. Analiza numerică este
prezentată în Probleme 4.5 și 4.6.
§4.8.5 scară Mică CSTP microgenerare
La scară mică solare concentratoare utilizate pentru gătit sunt total nepotrivite
pentru generarea de energie din cauza de urmărire, control și
powergeneration echipamentele necesare. Cu toate acestea, acest lucru nu înseamnă că
profesional construit planta nu poate opera pentru a microgenerării în
consum de energie în rețele sub supravegherea personalului instruit.
Un astfel de echipament a fost cercetat și dezvoltat de către un număr mic
de companii, dar până în prezent nu a fost semnificativă pe piața
de impact. Companiile care au succes pe piață mică termice solare
concentratoare și de urmărire pentru livrările de apă caldă, căldură de proces și
de desalinizare, etc. s-au adaptat sistemele lor termică pentru producerea de energie electrică
(Lovegrove și Stein 2012). Cu toate acestea, până în prezent se pare că o astfel termică
generație a fost dovedit a fi mult mai costisitoare și supărătoare decât
fotovoltaice echivalente.
§4.9 COMBUSTIBIL ȘI CHIMICE DE SINTEZA DIN
ENERGIA SOLARĂ CONCENTRATĂ
§Introducerea 4.9.1
Temperaturile ridicate și curat, nepoluat condiții de solar
de concentrare permite prelucrarea chimică pentru cercetare și în cele din urmă pentru
Fig. 4.16
Disociere și sinteza de amoniac, ca un mediu de stocare de energie solară. Ca
a propus și dezvoltat inițial de către Carden în 1970.
Motor termic
NH
3
(+N
2
+H
2
)
N
2
+H
2
N
2
+ tH
2
N
2
+H
2
NH
3
NH
3
Pentru
alte
oglinzi
Separator
Schimbător de căldură (30
ºC)
Schimbător de căldură
Oglinda
Dissociator
(700
ºC)
Sintetizator
(450
ºC)
TWIDELL PAGINARE.indb 140
01/12/2014 11:36
§4.10 aspectele Sociale și de mediu
141
produse și procese comerciale (Konstandopoulos et al. 2012).
Fabricarea combustibililor este un obiectiv principal, astfel captarea și stocarea
tranzitorii energie de la Soare. Astfel de procese termochimice general sunt
endoterm (absoarbe energie) și trece mai repede la temperaturi ridicate;
prin urmare, beneficiul de concentrare solară.
Principalii combustibili sunt: (1) hidrogenul din termochimice
disociere de apă; și (2) de carbon a combustibililor de la termochimice disociere
a deșeurilor la CO și H
2

, cu producția ulterioară de lichid sintetic


de combustibili. Amoniac de sinteză/disociere (Fig. 4.16) este un caz special, în
care reacția chimică este folosit ca un magazin cu energie.
§4.9.2 de producere a Hidrogenului
Un singur pas (direct) apă împărțirea în hidrogen și oxigen într-o singură etapă
necesită temperaturi
>2000°C, și poate fi periculos. Prin urmare, un
proces cu mai multe etape este de preferat; de obicei, un proces în două etape, folosind metal
oxizi de ca 'sistemul de catalizatori (de exemplu, substanțe chimice care sunt esențiale reactanților,
dar care ciclu continuu în procesul dintre un reduse (de exemplu,
SnO) și o oxidat de stat (de exemplu, SnO
2
)). În astfel de moduri de reacție tem-
de aceea este temperaturile mai poate fi
<1500°C și mai ușor de atins. O gama foarte mare
de oxizi de metal (MOs) au fost cercetate în solar hidrogen
sinteza. Cele două reacții simplifica, de exemplu, pentru:
Cel mai mic "temperatură înaltă" MO
+H
2
O
→ MO
2
+H
2
+ căldură
(4.26)
La mai mare 'temperatură mai mare' 2MO
2
+ caldura → 2MO + O
2

(4.27)
R&D până în prezent a explorat astfel de procese operaționale solare
concentratoare. Europene soluțiilor coloidale program a demonstrat concept
în continuă creștere cântare, și cu diverse multi-camera solare
reactoare cu scopul de producție continuă, inclusiv a soluțiilor coloidale-2
100-kilowatt instalație pilot la Plataforma Solar de Almeria, Spania,
operațional din 2008. (A Se Vedea Bibliografia.)
§4.10 ASPECTELE SOCIALE ȘI DE MEDIU
Solar cultură uscătoare și concentrarea de energie termica solara (CSTP) poate avea
beneficii pe scară largă în climă corespunzătoare. Alte tehnologii solare
examinate în acest capitol (apa de distilare, absorbție, frigidere,
saltgradient iazuri, combustibil și chimice de sinteză) sunt mai puțin frecvente. În cazul în care
aceste tehnologii sunt adecvate, pot contribui la curat,
economii durabile. Cu toate ca sursele regenerabile de energie, cu un impact benefic este să substitut
pentru combustibilii fosili, și deci reduce poluarea, atât la nivel local cât și la nivel global. Mai multe
de tehnologii pot folosi potențial dăunătoare sau periculoase substanțe chimice,
astfel încât procedurile stabilite în convențional industrii pentru sănătate și siguranță
trebuie să fie respectate. Radiației solare concentrate este un pericol serios pentru
prejudiciu personal și pentru foc, astfel încât site adecvat procedurile de siguranță sunt esențiale.
TWIDELL PAGINARE.indb 141
01/12/2014 11:36
142
Alte aplicațiile termice solare
Dat aride/semi-aride natura de medii potrivite pentru CSTP, o
provocare-cheie, de obicei, este obținerea de apă de răcire pentru motoare termice.
În caz contrar, uscat sau hibrid uscat/umed de răcire trebuie să fie utilizate, dar cu pierderea de
eficiență. O altă limitare pentru CSTP plante este distanța între
zone potrivite pentru producția de energie electrică și marile centre de consum.
Mai multe propus scară largă de propuneri (de exemplu, Desertec, care își propune să
folosească CSTP în Africa de Nord pentru a furniza europei Centrale cererea), prin urmare, să
includă low-loss, foarte înaltă tensiune, de transport de energie electrică.
Unele aplicatii solare sunt atât de evident benefic pe care se pare
inutil să subliniem aici; exemple sunt beneficiile
panourilor solare clădirilor (§16.4), haine de uscare și sare de producție (prin
evaporarea apei sărate din mare sare de bucătărie). Este regretabil faptul că în multe bogate
gospodării și organizații, se spală hainele sunt de obicei uscate de căldură
de energie electrică în uscătoare de haine, mai degrabă decât folosind soare ori de câte ori
este posibil. Chiar și atunci când uscat rufe reutilizarea căldurii recuperate de la vapori de apă
de condens, astfel încălzită electric de uscare este frecvent necesară.
REZUMAT CAPITOL
Analiza de bază este explicat aici pentru mai multe aplicatii solare, altele decât apa de încălzire (Capitolul 3) și
clădiri (Capitolul 16). Uscarea culturilor după recoltare este vitală pentru securitatea alimentară și este foarte mult ajutat de solare
de energie. Închis cultură uscătoare sunt încălzitoare de aer, care funcționează pe principii similare la boilere solare, dar sunt
de obicei mult mai simplă construcție. Energia solară concentrată de energie termică (CSTP) a progresat rapid
de la R&D proiecte la scară comercială investiții, în special în Spania de sud și California. Pentru că
numai directe (fascicul) de radiații pot fi concentrate de factori
>10, CSTP este aplicabilă numai în soare, aproape
nor-gratuit locații. Integrat de stocare termică este o caracteristică importantă a societății CSTP plante, și deci, astfel de
sisteme extinde generație în seara, și, de asemenea, pot avea putere de combustibil rezervă de capacitate. Astfel, CSTP
ofera firma, flexibil electrice capacitatea de producție la utilități, și este deja costuri competitive la ore de
vârf a cererii de energie electrică, mai ales în timpul verii. Principalele limitări pentru extinderea CSTP plante sunt
accesarea necesar de apă de răcire și distanța între aceste zone și consum mare
de centre. CSTP sisteme pot folosi oricare din cele patru tipuri principale de concentrare colector: (1) parabolic
jgheab; (2) liniar Fresnel; (3) receptor central (turn de putere); și (4) 'antena' (paraboloid). Tipuri (1) și (2)
evidența Soare într-o singură dimensiune (est–vest) și de obicei atinge temperaturi receptor
~400°C, dar
sunt mecanic mai simplu și mai ieftin decât tipuri (3) și (4), care a urmări Soarele în două dimensiuni. Cele
din urmă, cu punctul de concentrare, pot atinge temperaturi maxime de
~2000°C, pentru producția de produse
chimice
și materiale de testare. Alte tehnologii solare examinate în acest capitol (apa de distilare, absorbție,
frigidere, sare-gradient de iazuri, solar, mașini de gătit), toate au avantaje specifice în caz climate și
locații, dar nu au, în general, înlocuit de tehnologia convențională.
ÎNTREBĂRI RAPIDE
Notă: Răspunsurile la aceste întrebări sunt în textul de la secțiunea relevantă
din prezentul capitol, sau poate fi ușor dedusă din aceasta.
1
De ce este important să se usuce culturile imediat după recoltare?
2
Care este diferența dintre umiditatea absolută și relativă
umiditate?
TWIDELL PAGINARE.indb 142
01/12/2014 11:36
Probleme
143
3
De ce nu o absorbție cooler nevoie de căldură pentru a fi adăugate?
4
Cât de mult de apă proaspătă pe zi ar putea fi produs de un solar încă de
10 m
2
zona?
5
Ce este un solar de iaz'?
6
De ce este selectivă suprafață de important pentru un incalzitor solar de apa, dar nu
pentru un CSTP receptor?
7
Numele a trei tipuri de concentrator solar. Care dintre aceste realizează
cea mai mare temperatura de ieșire? Care este cel mai simplu de a păstra
curat?
8
Schița de principiu a unui Fresnel oglindă.
9
De ce cele mai multe comerciale CSTP sisteme includ unele de energie
de stocare? Care este cea mai comună formă de depozitare?
10
Lista de trei constrângeri cu privire la locația de CSTP plante.
PROBLEME
4.1 Teoria de fum
Un coș de fum vertical de înălțime h ia aer cald, la temperatura T
h

de la o sursă de căldură. Prin evaluarea integralei (3.25) în interiorul și în afara


coșului de fum, se calculează thermosyphon presiune p
th

pentru următoarele
condiții:
(o) T
o
= 30°C, T
h
= 45 °C, h = 4 m (corespunzător la un solar de cultură uscător).
(b) T
o
= 5°C, T
h
= 300°C, h = 100 m (corespunzător unei industriale
coș de fum).
4.2 Fluxul printr-un pat de cereale
Fluxul de aer printr-un pat de cereale este analog de curgere a fluidului printr-o
rețea de conducte.
(a) Fig. 4.17(a) prezintă o secțiune transversală a unui bloc solid străpuns de n paral-
lel tuburi, fiecare dintre raza - o. Ca o este mic, curgere este laminar (de ce?),
caz în care poate fi demonstrat că, în conformitate cu legea Poiseuille lui, la
volumul de fluid care curge prin fiecare tub este:
p
m
=






Q
o
dp
dx
8
1
4
(4.28)
în cazul în care
m este vâscozitatea dinamică (a se vedea de Revizuire 2) și dp/dx este
gradient de presiune de conducere fluxul. Utilizați acest lucru pentru a arăta că cea mai mare parte
viteza de curgere a fluidului prin bloc solid de secțiune transversală O
0
este:
e
=
=
v
Q
un dp
dx
O
8
total
0
2
TWIDELL PAGINARE.indb 143
01/12/2014 11:36
144
Alte aplicațiile termice solare
Fig. 4.17
Pentru Problema 4.2:
o
bloc străpuns de tuburi paralele;
b
porii într-un pat de cereale;
c
volumul de cereale pat.
Zona
O
0
o
(a)
(b)
(c)

x
O
0
în cazul în care porozitatea
e este o fracțiune din volumul de bloc care
este ocupat de fluid, și Î
total
este volumul total de curgere prin
bloc.
(b) pat de cereale într-un uscător solar are un volum total de V
pat
= O
0
∆x
(Fig. 4.17(c)). Uscătorul este de a fi proiectat să dețină 1000 kg de cereale de
volum V
pat
= 1,3 m
3
. Bobul este de a fi uscate în patru zile (
= 30
ore de funcționare). Arată că acest lucru necesită un debit de aer de cel puțin
Q
= 0.12 m
3
s
-1
(indiciu: vezi exemplul 4.1).
(c) Fig. 4.17(b) arată cum un pat de cereale poate fi, de asemenea, considerate ca fiind
un bloc din zona O
0

străpuns de tuburi, al cărui diametru este comparabil


(sau mai mic decât) raza de cereale. Cea mai mare parte viteza de curgere este
redusă cu un factor k(
<1) din care a prezis de către (o), din cauza
neregulate și sinuos tuburi. Dacă la presiunea este
∆p, arată că
grosimea
∆x , prin care debitul Q poate fi menținută este:
e
m
∆=







x
koVp
Q
8
2
1/2
Pentru un pat de orez,
e = 0.2, o = 1 mm, k = 0.5 aproximativ. Luând Q
de la (b), și
∆ p la Problema 4.1(a), se calculează ∆x și O
0
.
4.3 solar iaz
O idealizat solare iaz măsuri de 100 m
× 100 m × 1,2 m. Partea de jos
20 cm (depozitarea layer) are un efectiv de absorptance
a = 0.7. La
TWIDELL PAGINARE.indb 144
01/12/2014 11:36
Probleme
145
1.0 m de apă de mai sus (stratul izolator) are un factor de transmisie
t=
0.7, și densitatea crește în jos, astfel încât convecție face
să nu apară (vezi Fig. 4.6). Designerul speră să mențină depozitare
strat la 80°C. temperatura De la suprafața iazului este de 27°C (zi
și noapte).
(a) se Calculează rezistența termică a stratului izolator, și compara
cu partea de sus de rezistență tipic plate-placă colector.
(b) se Calculează rezistența termică a unui strat similar de apă proaspătă,
pot să convecție liberă. Comparați această valoare cu care în (a), și
comenta cu privire la orice îmbunătățiri.
(c) densitatea de soluție de NaCl crește cu 0,75 g/litru pentru fiecare 1.0
g de NaCl adăugat la 1.0 kg H
2
O. O soluție saturată de NaCl con-
perdele de aproximativ 370 g NaCl / kg H
2
O. volumetric coeficientul de
dilatare termică de soluție de NaCl este de aproximativ 4
× 10
−4
/K. Calcula
concentrația minimă C
min

de NaCl necesară în depozit strat


pentru a suprima convecție, presupunând că stratul de apă de la partea de sus a
iazului nu conține sare. Cât de ușor este de a realiza această concentrație în
practică?
(d) se Calculează la o scară de timp caracteristică pentru pierderea de căldură din depozitarea
strat, prin rezistența stratului izolator. Dacă
temperatura de depozitare strat este de 80°C la apus de soare (6 p. m.) care este
temperatura la răsărit (6 a. m.)?
(e) Difuzarea unei solut depinde sale moleculare difuzibilitate D,
analog cu difuzibilitate termică
κ §R3.3. Pentru NaCl în apă,
D
= 1.5 × 10
-9
m
2
s
-1
.Iaz este configurat cu depozitarea strat
de două ori concentrația critică de NaCl, adică dublul valorii C
min

calculat în (c). Estima timpul de difuzie moleculară a reduce


concentrația C
min
.
Notă: difuzie Moleculară de sare (solvat) dintr-o regiune de puternică
concentrare la o slabă concentrare este analog cu cel de difuzie
a căldurii dintr-o regiune de înaltă temperatură de la o regiune de joasa
temperatura prin conducție, așa cum este descris în §R3.3. În fapt, aceleași
ecuații se aplică cu moleculară difuzibilitate D în loc de difuzivitate termică
κ și concentrația C în loc de temperatura T.
(f) în Funcție de răspunsurile la (c)–(e), discuta despre practicabilitatea
de a construi un iaz, și utilizări posibile la care ar putea
fi puse.
4.4
Fig. 4.18 prezintă caracteristica cheie a unui sistem pentru utilizarea pe scară largă
a energiei solare similar cu cel implementat în California în anii 1980.
Lumina solară este concentrată pe o conducta perpendicular pe planul de
diagramă și este absorbită selectiv de suprafață pe partea exterioară a
TWIDELL PAGINARE.indb 145
01/12/2014 11:36
146
Alte aplicațiile termice solare
țeavă. Fluidului in conducta prin urmare, este încălzit la o temperatură T
f

de
aproximativ 500°C. lichidul trece apoi printr-un schimbător de căldură unde se
produce abur pentru a conduce un abur convențional turbina, care la rândul său
conduce un generator electric.
(a) de Ce este de dorit pentru a face T
f
la fel de mare ca posibil?
(b) să Presupunem că țeava interioară este de 10 m lungime și o grosime de 2.0 mm si are o
diametru de 50 mm, și că lichidul este obligat să furnizeze 12 kW de
energie termică a schimbătorului de căldură. În cazul în care conducta este realizat din cupru,
arată că diferența de temperatură de peste conductă este mai mică de
0.1°C. (se Presupune ca temperatura fluidului este uniformă.)
(c) să Presupunem că selectivă suprafață are o/
e = 10 la operare
temperatura de 500°C. ceea Ce este factorul de concentrare necesar de
obiectiv (sau oglinda) pentru a atinge această temperatură cu ajutorul evacuat
colector arătat? Este acest lucru din punct de vedere tehnic de două-dimensional
system?
(d) să Presupunem că teava de cupru nu a fost protejat de sistemul de vacuum
dar a fost expus direct la aer. Presupunând zero, viteza vântului,
calcula pierderea de căldură prin convecție pe secundă de la conducta.
(e) să Presupunem că întregul sistem este de a genera 50 MW de electrice de putere.
Estimarea captatorului este necesar.
(f) să discutăm pe Scurt despre avantajele și dezavantajele unei astfel de
schema, în comparație cu (i) un ulei centrală electrică pe bază de similare
Fig. 4.18
Pentru Problema 4.4. O propunere de concentrator de sistem pentru generarea de energie.
Concentrator
(lentila Fresnel)
Fereastra
Vid
Lichid
Reflectând
suprafață
Selectiv
suprafață de absorbție
TWIDELL PAGINARE.indb 146
01/12/2014 11:36
Probleme
147
capacitate, și (ii) la scară mică utilizări ale energiei solare, cum ar fi piața internă
de încălzire a apei.
4.5
Să presupunem că sistemul prezentat în Fig. 4.16 este de a fi utilizate pentru a furniza o
medie de 10 MW de energie electrică.
(a) Estimarea totală a captatorului acest lucru va necesita. Compara acest lucru cu
un sistem folosind celule fotovoltaice.
(b) explica pe Scurt de ce o substanță chimică (sau alte) depozitul de energie este necesar,
și de ce oglinzile trebuie să fie îndreptată spre Soare. Cum ar putea acest lucru
fi aranjate?
(c) Pentru a asigura un mod corespunzător de mare rata de disociere, de dissociator este de a fi
menținut la 700°C. Sanitare considerente (Problema 4.6) necesită
ca dissociator are un diametru de aproximativ 15 cm. Presupunând (pentru
simplitate) este de formă sferică, să se calculeze puterea a pierdut de fiecare
dissociator de radiații.
(d) Fiecare oglinda are o deschidere de 10 m
2
. Într-o radiație solară de 1 kW/m
2
,
ce este radiația de la receptor? Arată că aproximativ 2,5 g/s de NH
3
pot fi disociate în aceste condiții.
4.6
Sistemul prezentat în Fig. 4.16 necesită 2,5 g/s de NH
3
pentru a trece la fiecare
concentrator (a se vedea Problema 4.5). Să presupunem că NH
3
este la o presiune de
300 de atmosfere, în cazul în care acesta are densitatea
ρ = 600 kg m
-3
și viscozitate
m = 1.5 × 10
-4
kg m
-1
s
-1
.
Amoniac trece printr-o conductă de lungime L și diametru d.
Pentru a menține frecare la un nivel scăzut acceptabil, este necesar pentru a menține
numărul Reynolds
R
< 6000.
(a) Calculați: (i) diametrul d; (ii) energia a pierdut la frecare de pompare
2.5 g de amoniac pe o distanță L
= 50 m.
(b) Compara această pierdere de energie cu energia efectuate. De ce este
amoniac-a menținut la o presiune de
~300 de atmosfere, mai degrabă decât de
~1 atmosferă? (Indiciu: a estima dimensiunile unui sistem de lucru
la
~1 atmosferă.)
4.7
Fig. 4.5(a) este o schiță a fluxului de căldură, temperaturile și alte aspecte
de o simplă solare în continuare pentru obținerea de apă proaspătă de apă salmastră.
Folosind parametrii indicate prin simbolurile de pe schiță, și orice
alți parametri de care ai nevoie, face un flux de căldură în circuitul de sistem ca un
analog al unui circuit electric.
TWIDELL PAGINARE.indb 147
01/12/2014 11:36
148
Alte aplicațiile termice solare
NOTE
1
A se vedea §13.8 pentru o schiță de osmoză'.
2
Acest lucru poate fi, de asemenea, numit 'optică raport de concentrare'.
3
La explicație excelentă cu diagrame dinamice pot fi găsite la http://en.wikipedia.org/wiki/Stirling_engine
(accesat la 14 septembrie 2011).
BIBLIOGRAFIE
General
Duffie, J. A. și Beckman, W. A. (2006, 3rd edn) Solar de Inginerie a Proceselor Termice, John Wiley & Sons,
New York. Textul clasic, în special pentru energia termică solară teorie și aplicații. Acoperă cele mai multe dintre subiectele din
acest
capitol prin empirice de analiză tehnică.
Gordon, J. (ed.) (2001) A Energiei Solare
− Starea de arta, James & James, Londra. Zece capitole de către experți în
solare termice, fotovoltaice și geamuri.
Goswami, D. Y., Kreith, F. și Kreider, J. F. (2000, 2nd edn) Principiile de Solar Engineering, Taylor și Francis,
London. Un alt manual standard pe sistemele termice solare.
Reviste
Cel mai consacrat jurnal, care acoperă toate aspectele legate de solar (soare) de energie, este Energia Solară, publicat de
Elsevier, în cooperare cu International Solar Energy Society (ISES).
Încălzitoare de aer și uscarea culturilor
Brenndorfer, B., Kennedy, L., Bateman, C. O., Mrena, G. C. și Wereko-Brobby, C. (1985) Uscătoare Solare: rolul Lor
în post-recoltare, prelucrare, Commonwealth Secretariat, Londra.
Monteith, J. și Unsworth, K. (2008, 3rd edn) Principii de Fizica Mediului, Academic Press, London.
Include o discuție plină de umiditate.
Desalinizarea apei
Delyannis, E. și Belessiotis,V. (2001) 'energia Solară și de desalinizare', Progresele în domeniul Energiei Solare, 14, 287. Util
review cu fizica de bază afișate; constată că aproape toate mari de stat-of-the-art de fotografii au fost demontate'.
Rizzutti, L., Ettouney, H. și Cipollina, A. (coord) (2007) Solare de Desalinizare pentru Secolul 21, Springer, New
York. Acțiunea unei conferințe sponsorizate de NATO.
Solare absorbție de răcire
Wang, R. Z. (2003) Solar de refrigerare și de aer condiționat de cercetare în China', Progresele în domeniul Energiei Solare, 15,
261.
Explicație clară a principiilor; constată că au fost câteva aplicații comerciale încă.
TWIDELL PAGINARE.indb 148
01/12/2014 11:36
Bibliografie
149
Solar iazuri
El-Sebaii, A. A., Ramadan, M. R. I., Aboul-Enein, S. și Khallaf, A. M. (2011) 'Istoria bazine solare: o revizuire
studiu', Energia Regenerabilă și Durabilă Recenzii, 15, 3319-3325. Ceea ce sugerează și titlul.
Leblanc,
J., Akbarzadeh, A., Andrews, J., Lu, H. și Golding, P. (2011) Căldura metode de extracție din
salinitygradient solare iazuri și introducerea unui sistem nou de căldură de extracție pentru o eficiență îmbunătățită', Energie
Solară,
85(12), 3103-3142.
Lu, H., Swift, A. H. P., Hein, H. D. și ross, J. C. (2004) 'Progresele în gradient de salinitate solare iaz tehnologie
pe baza șaisprezece ani de experiență operațională', Journal of Solar Energy Engineering, 126, 759-767. Atent
de revizuire, inclusiv detalii tehnice și experiență operațională.
Tabor, H. și Doron, B. (1990) La Beit Ha'Harava 5 MW(e) solar iaz', Energie Solară, 45, 247-253. Descrie
cea mai veche și mai mare în timp – operaționale solare iaz.
Concentratoare și aplicațiile lor
Dunn, R., Lovegrove, K. și Burgess, G. (2012) O revizuire pe bază de amoniac termochimic de stocare a energiei pentru
Concentrarea energiei Solare', IEEE Journal, 100(2), 391-400.
Soluțiilor coloidale: succesive finanțat de Uniunea Europeană, programele de Cercetare și dezvoltare pentru producția de
hidrogen din apă. De
exemplu, a se vedea http://160.40.10.25/hydrosol/ (accesat Martie 23, 2013).
Konstandopoulos, A. G. și Lorentzou, S. (2010) 'Roman monolit reactoare pentru solar termochimică a apei
împart, în L. Vayssieres (ed.), Pe Solar Hidrogen și Nanotehnologie, John Wiley & Sons, Singapore.
Konstandopoulos, A. G., Pagkoura, C. și Lorentzou, S. (2012) Solar de combustibili și industriale solar chimie', ch. 20
în Lovegrove și Stein (2012). Excelent studiu din punct de vedere chimic.
Lovegrove, K. și Stein, W. (eds) (2012) Concentrarea energiei Solare Tehnologie: Principii, evoluții
și aplicații, Woodhead Publishing, Cambridge. Cuprinzătoare și de autoritate integrată capitole de
experți; include solar, fizice, chimice, economice și de producție recenzii.
Winston, R. (2011) Termodinamic eficient concentrator solar tehnologie, seria de conferințe, UC Davis. Disponibil
la http://solar.ucdavis.edu/files/education/2011-minicourse-winston.pdf (accesat în Martie 2013).
Winston, R., Minano, J. C. și Beniez, P. G. (2005) Nonimaging Optica, Elsevier Academic Press, San Diego, CA.
A se vedea, de asemenea, la Universitatea din California Solare Avansate Institutul de Tehnologii. Centrul de cercetare non-
imaging
concentratoare de condus de Profesorul Winston. Vezi http://ucsolar.org/research-projects/solar-thermal.
Termică solară energie electrică
Carden, P. O. (1977) 'Energie co-radiații folosind reversibile amoniac reacția', Energie Solară, 19, 365-378.
Primul dintr-o lungă serie de articole; a se vedea, de asemenea, Luzzi, A. și Lovegrove, K. (1997) Un solar termochimice
centralei
folosind amoniacul ca o opțiune atractivă pentru reducere a emisiilor de gaze cu efect de seră', Energie, 22, 317-325. A se vedea,
de asemenea, Dunn et al.
(2012).
Mills, D. R. (2001) Termice Solare de energie Electrică, în Gordon (2001), pp. 577– 651.
Iarna, C. J., Sizmann, R. L. si Vant-Coca, L. L. (eds) (1991) centrale electrice Solare: Fundamentele, tehnologii,
sisteme, economie, Springer Verlag, Berlin. Inginerie de revizuire.
TWIDELL PAGINARE.indb 149
01/12/2014 11:36
150
Alte aplicațiile termice solare
Site-uri
Există nenumărate site-uri de-a face cu aplicații în energie solară, unele excelente și multe dintre dubioase
valoare academică. Utilizați un motor de căutare pentru a localiza acestea și să dea cele mai crezare la site-urile oficiale ale
organizațiilor,
ca și cu exemplele citate mai jos.
ESTELA (European Solar Thermal energie Electrică de Asociere)
<www.estelasolar.eu/> Site-ul include ESTELA/
Kearney Raportul din iunie 2010, energia Termică Solară energie Electrică în 2025.
International Solar Energy Society (ISES)
<www.ises.org>. Cel mai mare, mai veche, și cea mai mare autoritate
profesională, organizație care se ocupă cu tehnologia și punerea în aplicare a energiei solare. ISES sponsorizează
researchlevel jurnalul Energiei Solare, care acoperă toate subiectele din acest capitol.
AIE Solare de Încălzire și Răcire Program
<www.iea-shc.org> Rapoarte de cercetare internaționale și proiecte în aceste
domenii.
AIE program pe Energie Solară Concentrată
<www.solarpaces.org> Rapoarte de cercetare internaționale și proiecte
în acest domeniu, împreună cu contururi de tehnologii relevante. Publicații disponibile pe acest site includ CSP
foi de Parcurs Tehnologie (2010-2050).
TWIDELL PAGINARE.indb 150
01/12/2014 11:36

Fotovoltaice (PV) de alimentare tehnologia


CONȚINUTUL
Obiective de studiu
152
§5.1 Introducere
153
§5.1.1 Fundal
153
§5.1.2 Folosește și o creștere rapidă
154
§5.1.3 Bază
155
§5.1.4 Capitolul secțiuni
156
§5.2 Fotovoltaice circuit proprietăți
156
§5.3 Aplicații și sisteme
161
§5.3.1 aplicații Stand-alone
161
§5.3.2 Grid-connected systems
162
§5.3.3 Echilibru a sistemului (BoS)
componente
165
§5.4 Maximizarea eficienței celulelor solare (Sicells)
167
§5.4.1 Top-suprafața electrice-contact
obstrucție zona (intrinseci
pierdere de ~3%)
168
§5.4.2 pierderi Optice, de sus și de
spate suprafete
169
§5.4.3 Foton cu energie mai puțin
decât band gap (pierderea ~23%)
172
§5.4.4 Excesul de energie fotonica (pierdere de ~33%) 172
§5.4.5 eficiența de Captare (pierderea ~0.4%) 172
§5.4.6 eficiență de Colectare
173
§5.4.7 Tensiune factorul F
v
(pierdere de ~20%)
173
§5.4.8 factor de Umplere (curba factor) F
c
(intrinseci pierdere de ~12%)
174
§5.4.9 Idealitate factorul O (pierdere de ~5%)
174
§5.4.10 rezistență Serie (pierderea ~0.3%) 175
§5.4.11 Shunt resistance (neglijabil
pierderea ~0.1%)
175
§5.4.12 Livrate putere
175
§5.5 celule Solare și module
fabricarea
176
§5.6 Tipuri și adaptări ale
fotovoltaice
179
§5.6.1 Variații în Si material
180
§5.6.2 Variații în intersecția geometrie 183
§5.6.3 Alte materiale de substrat;
chimice grupele III/V și II/VI
184
§5.6.4 Alte materiale semiconductoare
mecanisme, clasificări
și terminologii
186
§5.6.5 Variație în sistemul aranjament 189
§5.7 Sociale, economice și
aspecte de mediu
191
§5.7.1 Preturi
191
§5.7.2 Grid-connected systems
192
§5.7.3 sisteme Stand-alone
193
§5.7.4 PV pentru electrificarea rurală,
mai ales în țările în curs de dezvoltare 193
§5.7.5 de impact asupra Mediului
196
§5.7.6 Outlook
196
Rezumat capitol
197
Întrebări rapide
197
Probleme
198
Notă
200
Bibliografie
200
Caseta 5.1 de Auto-curatare sticla pe
modulul PV acoperă
167
Caseta 5.2 absorbția radiațiilor Solare
la joncțiune p–n
171
Caseta 5.3 Fabricarea de silicon
celulele cristaline și module
176
Caseta 5.4 Un exemplu de un sofisticat
Si celule solare
185
CAPITOLUL

5
TWIDELL PAGINARE.indb 151
01/12/2014 11:36

www.shahrsazionline.com
152
Fotovoltaice tehnologie
LISTA DE TABELE
5.1 Aproximativă a limitelor de eficiență într-un singur strat (homo-joncțiune) cristalin Si celule solare (se referă la
§5.4 pentru explicații de fiecare proces)
169
5.2 Exemple de celule solare și parametrii lor de bază, în condiții standard* (date aproximative de
o serie de surse; există o îmbunătățire constantă în cel mai bun randament cu experiență de cercetare
și experiență de fabricație)
1
81
5.3 Performanță selectate celule solare sub concentrate "răsărit de soare" (măsurată în simulatoare solare). 191
LISTA DE FIGURI
5.1
O parte din fermă solară de 13 MW capacitate, la Nellis Air Force Base, în apropiere de Las Vegas, Nevada, statele UNITE ale
americii.
153
5.2
Creștere de capacitatea instalata la nivel mondial (GW) din celule fotovoltaice.
154
5.3
(a) Schematică ('micro-view) portret de PV generație (b) Conturul de celule fotovoltaice
într-un circuit ('macro-view).
155
5.4
Circuitul echivalent al unei celule solare.
157
5.5 (a)
I–V caracteristice tipic 36 de celule Si module. (b) putere Maximă curbă și I-V caracteristice. 158
5.6
Tipic aranjamente comerciale Si celule solare.
160
5.7
Tipic aplicații stand-alone de celule fotovoltaice.
162
5.8
Exemple de conectate la rețea a instalațiilor fotovoltaice.
163
5.9
Schema (nu diagramele de cablare) de: (a) ~5 kW fotovoltaice microgenerare (b) o mare MW
de capacitate solara.
164
5.10 schema de principiu a unui stand-alone sistem fotovoltaic.
165
5.11 structura de Bază de joncțiune p–n de celule solare.
168
5.12 Antireflex film subțire.
170
5.13 suprafețele de Sus pentru creșterea absorbției după inițială de reflecție.
171
5.14 Indicativ teren de solar iluminare energetică spectrală împotriva foton de energie pentru a ilustra foton de absorbție.
171
5.15 nivelul de Energie într-o celulă cu suprafața de teren' (BSF).
174
5.16 Un cristal metode de creștere.
177
5.17 Etape în fabricarea de module solare.
178
5.18 estimările privind costurile și eficiența a trei generații de celule solare.
180
5.19 nivelul de Energie de diferite celule solare cu joncțiune tipuri.
183
5.20 La SJ3 NREL/solar intersecția cu mai multe straturi de celule.
184
5.21 PERL celulă (pasivizat emițător, spate local difuză).
185
5.22 Un colorant sensibil la celule solare.
187
5.23 Verticale multi-joncțiune celule.
188
5.24 Un concentrator de sisteme.
190
5.25 reduceri de Costuri de PV în aplicare.
192
5.26-O progresie de iluminat solare kituri.
194
5.27 aprecierea Publicului și înțelegerea sunt critice pentru succes.
196
OBIECTIVE DE STUDIU

Știu cum electricitatea este generată de soare
strălucească.

Apreciez distinctiv tehnologia de PV.

Ia în considerare cererile, cu exemple.

Înțeleg electrice proprietăți și utilizări.

Conturul solid-state proprietăți și de a dezvolta
-
mente.

Se referă la Revizuirea 4 pentru siliciu semiconductor
cristalin teorie.

Ia în considerare procesele de fabricație.

Cunosc posibilitățile și tendințele de eficiență
îmbunătățiri.

Ia în considerare alte PV mecanisme.

Să fie conștienți de mediu și economice
aspecte.
TWIDELL PAGINARE.indb 152
01/12/2014 11:36
§5.1 Introducere
153
Fig. 5.1
O parte din fermă solară de 13 MW capacitate, la Nellis Air Force Base, în apropiere de Las Vegas, Nevada,
statele UNITE ale americii. Modulele fotovoltaice sunt fixate la aproape 6000 de Soare-de urmărire rame, la care
de curent continuu este transformat (inversat) în AC convenționale alimentare de la rețea pentru energia electrică
de-a lungul Bază, ajungând la 13 MW. De energie electrică solară se integrează cu alimentare
de Baza este generatoare diesel, dând astfel o reducere considerabilă a consumului de combustibil.

§5.1 INTRODUCERE
§5.1.1
Fundal
Există doar două metode de a genera semnificative de energie electrică.
Prima este electromagnetice generarea dinamică a descoperit de Michael
Faraday în anul 1821 și în producția comercială de anul 1890; aceasta este
metoda dominantă astăzi, care necesită mișcarea relativă dintre un câmp magnetic
și un conductor alimentat de la o sursă externă de motor sau turbina.
Cea de-a doua metodă este fotovoltaice generație fără piese în mișcare
, folosind celule solare (din punct de vedere tehnic celule fotovoltaice), care produce
energie electrică de absorbție a radiațiilor electromagnetice, mai ales lumina,
predominant în materiale semiconductoare, așa cum se arată în Fig. 5.1.
Fotovoltaice (PV) efect
1

a fost descoperit de Becquerel în 1839, dar


nu a fost dezvoltat ca o sursă de energie până în anul 1954 de către Chapin, Fuller și
Pearson folosind dopat semiconductor de siliciu; mecanismul fizic
este explicat în Revizuire 4. Există mai multe tipuri diferite de celule PV, dar
pentru aplicarea în practică nu este esențial pentru a înțelege lor interne de
funcționare, deoarece acestea pot fi descrise de externe circuit electric
caracteristici (§5.2).
TWIDELL PAGINARE.indb 153
01/12/2014 11:36
154
Fotovoltaice tehnologie
§5.1.2
Folosește și o creștere rapidă
Putere PV este una dintre cea mai rapidă creștere tehnologii energetice: instalat
capacitatea a crescut exponențial de la ~200 MW în 1990 la mai mult de 80.000
MW (80 GW) în 2012, cu o creștere similară a ratei de așteptat să continue (Fig.
5.2). Factorii tehnici de conducere a cererii sunt universal aplicabilitate
(radiația solară este disponibil peste tot, deși energia de intrare și
, prin urmare, producția de energie sunt mai mari în mai însorite locuri), modular
caracter (permite utilizarea la toate scările de la câțiva wați până la zeci de mw),
fiabilitate, durată lungă de viață, ușurința de utilizare, și lipsa de zgomot și emisii. De
creștere a pieței se referă la politicile de susținere în multe țări,
în special 'tarifele feed-in' că incurajam utilizatorii de energie electrică pentru a instala
rețeaua de alimentare conectate la sistemele FOTOVOLTAICE pentru puterea lor, cu exces exportate și
vândute prin rețeaua de utilități. Rezultate cererii încurajează producătorii
la scară de producție, care la rândul său face costul unitar mai ieftin
pe plan internațional.
Înainte de anul 2000, cele mai multe celule fotovoltaice au fost în sisteme stand-alone,
progresează de la sateliți spațiu de iluminat, de pompare a apei, de refrigerare,
de telecomunicații, solar, case, proprietate bunuri mobile și sau
izolate echipamente (de exemplu, bărci mici, lumini de avertizare, parcare metri). Începând cu
anul 2000 FOTOVOLTAICE conectate la retea putere (de exemplu, încorporate în clădiri sau
în mare free-standing tablouri: Fig. 5.1) a devenit dominantă
aplicație ca un acceptată "mainstream" formă de generatoare de energie electrică pentru
secolul 21.
Evident generație apare numai în timpul zilei și variază în funcție de expunere la soare,
deci de stocare a energiei electrice (de exemplu, baterii) sau grilă de legătură este de obicei (a se vedea punctul
5.3).
Astfel de mecanisme, de asemenea, a netezi mai rapidă variabilitate de ieșire
în timpul zilei. Fosta fabrica de cost pe unitatea de capacitate a scăzut la
$US1/W pentru film subțire de celule în 2009 și pentru siliciu cristalin celule
în 2011, cu o inflație-adaptarea prețurilor în scădere deoarece fabricarea
Fig. 5.2
Creștere de capacitatea instalata la nivel mondial (GW) din celule fotovoltaice. Linia punctată este "mediu"
scenariu de EPIA (2014).
Data Sursa: European Asociația Industriei Fotovoltaice
50
100
150
200
250
300
350
0
2000
2005
An
Lumea FOTOVOLTAIC instalat/GW
2015
2020
2010
TWIDELL PAGINARE.indb 154
01/12/2014 11:36
§5.1 Introducere
155
și know-how extinde (vezi §5.7). Generarea de energie PV este rapid
devenind o tehnologie de masă pentru integrarea în națională la scară energie electrică
de alimentare.
Pentru stand-alone de energie electrică la un rezonabil loc însorit de expunere la soare
20 MJ/(m
2

zi), pe termen lung generate de putere este de obicei semnificativ


mai ieftin decât cel din generatoare diesel. Pentru a fi răspândite în rețeaua de alimentare
în zonele cu climat însorit, PV este puterea costuri competitive cu zi vârf rețeaua
de energie electrică. Dacă poluante forme de generație fost taxat pentru
costurile externe (a se vedea Caseta 17.3), atunci PV și alte surse regenerabile de energie ar fi
chiar mai eficiente.
Comerciale module solare cu dovedit încapsulare da fără probleme
de serviciu, atât timp cât elementare abuz este evitată. Vieți de cel puțin
20 de ani sunt din punct de vedere comercial garantat, cu așteptări foarte mult
mai operațiune de succes.
§5.1.3
Elementele de bază
Fotovoltaice de generare a puterii este cauzată de către fotoni de
radiație electromagnetică de separare pozitive și negative purtători de sarcină într -
un material absorbant (Fig. 5.3). Dacă un câmp electric este prezent, aceste taxe pot
produce un curent într-un circuit extern. Astfel de domenii exista permanent
+
+
+
+











+
O
B
+
+
+
+
+
X
+
Electrice
sarcina
Curentul de electroni
Lumina soarelui
e

(a)

+
Sarcina
Fotonii
n-tip
silicon
p-tip
silicon
Junction
Lumina soarelui
(b)
Fig. 5.3
o
Schematică ('micro-view) portret de PV generație de fotoni de lumina soarelui
absorbit în apropierea unei intersecții între straturi semiconductoare a și B cu diferite
doping. A se vedea de Revizuire 4 pentru explicații suplimentare proceselor fizice, și Figurile
5.11, 5.21 și 5.22 pentru mai puțin schematice de celule solare de construcție. Rețineți că
convenționale (direct) fluxurile de curent de la (+) la (-), adică în direcția opusă la
curentul de electroni.
b
Schiță de celule fotovoltaice într-un circuit ('macro-view). Diagrama prezinta multe celule
asamblate într-un modul.
TWIDELL PAGINARE.indb 155
01/12/2014 11:36
156
Fotovoltaice tehnologie
la intersecțiile în celule FOTOVOLTAICE ca 'built-in' câmpuri electrostatice care asigură
diferența de tensiune (CEM) pentru util de producție de energie. Generarea de energie
este obținută din celule adaptate la radiații cu lungimi de undă de
infraroșu (
l ≈ 10 μ m) la ultraviolete (l ≈ 0.3 μm); cu toate acestea, cu excepția cazului în oth-
erwise declarat, considerăm celule adaptate la radiații solare (
l ≈ 0.5 μm).
Built-in domenii de comun semiconductoare celule produc potențiale
diferențe de aproximativ 0,5 V și densități de curent de aproximativ 400 A/m
2
în mod clar
nori de radiație solară de 1,0 kW/m
2
. Mai multe detalii de interne fizica
de celule solare sunt date în Revizuire 4.
Comerciale celulele fotovoltaice au eficiență între 12%
și 25% în ordinare soare, în funcție de tipul și prețul. În
mirrorconcentrated soare, eficiența poate fi de aproape 50%. Comerciale
celulele sunt interconectate și a fixat termen de intemperii încapsulare ca
module; în funcție de numărul de celule în serie, modul în circuit deschis
tensiuni sunt, de obicei, între 15 și 30 V. curent de
celule este în mod inerent curent continuu (DC); invertoare electronice sunt folosite pentru a
schimba acest lucru în curent alternativ (AC). Pentru un anumit comerciale module
într-un optimă poziție fixă, ieșire de zi cu zi pe unitate colector de suprafață depinde de
la clima, dar poate fi de așteptat să fie de aproximativ 0,5 la 1,0 kWh/(m
2

zi).
De ieșire poate fi crescut de utilizarea dispozitive de urmărire și solare concentratoare.
§5.1.4
Capitolul secțiuni
§5.2 și §5.3 examinați circuitul de caracteristicile și utilizările de putere PV
surse. Cititorii interesați în principal în aplicații se pot concentra pe
aceste secțiuni și pe §5.7, care examinează economice, sociale și
de mediu, legate de utilizarea și producția de celule fotovoltaice. §5.4
prezintă unele limitări intrinseci pentru eficiența energetică a celulelor solare,
folosind celule solare de siliciu ca un exemplu, și bazându-se pe solid-state
teoria prezentată în Revizuire 4. §5.5 consideră modul in care celulele sunt construite.
Variații în celula material, inclusiv în formă cristalină și
dezvoltarea de celule din alte materiale decât Si, sunt discutate în §5.6.
Pentru această a treia ediție a Resurselor Regenerabile de Energie ne-am stabilit
fizica solid-state de forma dominantă de celule fotovoltaice, silicon
cristal de celule, în Revizuire 4. Acest lucru nu minimalizează o înțelegere a
proceselor interne, dar recunoaște specialitate ale subiectului. Alte
tipuri de PV trebuie legate de proprietăți interne.
§5.2 FOTOVOLTAICE CIRCUIT PROPRIETĂȚI
Cu celule fotovoltaice, cu toate regenerabile de energie dispozitive,
condiții de mediu oferi o sursă de curent de energie.
Circuitul echivalent (Fig. 5.4) descrie esențiale macroscopic
caracteristici pentru generarea de energie PV, inclusiv interne seria
de rezistență R
s
și rezistența șunt R
sh
.
TWIDELL PAGINARE.indb 156
01/12/2014 11:36
§5.2 Fotovoltaice circuit proprietăți
157
Din circuitul echivalent,
Am
= Am
Am
– Am
D
– Am
sh
(5.1)
în cazul în care am
Am
este lumina-curent indus și eu
sh
= (V – IR
s
)
/ R
sh
deci
=--
-
EU AM
Am
V IR
R
(
)
Am
D
s
sh
(5.2)
În discutarea celule fotovoltaice ca generatoare de putere în funcționare (de exemplu,
atunci când iluminate), este de obicei pentru a lua dispozitivul curent am fi pozitiv, atunci când
curge de la borna pozitivă a celulei (de exemplu, generatorul) prin
sarcina extern. Aceasta este convenția cu toate generatoare de curent continuu,
inclusiv baterii, dar este opusă analizei fundamentale de flux de electroni
derivate din fizica de o simplă diodă (după cum este explicat în §R4.1.9).
În Fig. 5.5(a) pentru o anumită iluminare, curba caracteristică este de
V
= 0 (scurt-circuit, cu curent am
sc
) pentru a V
= V
oc
(tensiunea de circuit deschis, cu
Am
= 0). Tensiunii în circuit deschis V
oc
crește doar ușor cu iradiere,
spre deosebire de curentul de scurt-circuit - am
sc
care este proporțională cu absorbită
expunere la soare. Puterea produsă este produsul dintre m și V, (P
= IV );
putere maximă la fiecare tip de iluminare este indicat de putere de vârf de linie.
Fig. 5.5(b) parcele generate de celule de putere împotriva tensiune pentru o valoare de
expunere la soare.
Condiția de maxim de putere într-un circuit extern este că
sarcină externă R
Am
este egală cu rezistența internă a sursei R
int
. Cu toate acestea,
R
int
depinde de absorbit fotonul flux și așa se schimbă cu insola-
tion, atât de bine puterea de potrivire într-o celulă solară necesită R
Am

cum s-a văzut de


PV matrice, să se schimbe în relația cu radiația solară. Această
potrivire este efectuat în mod automat printr-o interfață electronică unitate de legătură
între matrice și circuite externe. Pentru grid-connected systems
această putere de vârf de potrivire este integrat electronic cu un invertor
de la AC DC energie electrică, care este găzduit într-o cutie de control'.
Pentru constanta de insolație, o creștere în celula material temperatura
θ afectează performanța prin reducerea V
oc
și creșterea eu
sc
, cu
Fig. 5.4
Circuitul echivalent al unei celule solare. Simbolurile sunt ca în (5.1) și (R4.22), unde: m - este
curent în sarcină externă, am
Am
lumina-curent indus, am
D

dioda de curent de întuneric, R


sh
șuntul rezistenta, am
sh

șunt de curent, R
s
seria de rezistență, și V celula de ieșire
tensiune.
Am
Am
Am
D
Am
sh
Am
Extern
de încărcare
V

+
Am
R
sh
R
s
TWIDELL PAGINARE.indb 157
01/12/2014 11:36
158
Fotovoltaice tehnologie
3
Eu kW m
-2
0.8
Putere de vârf de linie
V / Volt
0.6
0.4
0.2
2
1
0
Am/Amp
5
Tipic de încărcare
gama obișnuită de
12 V acumulator
10
15
/
sc
V
oc
(a)
Fig. 5.5
o
I–V caracteristice tipic 36 de celule Si module. Rețineți că, chiar și fără maximă
de încărcare de putere de control, puterea de vârf linia de maximă IV produsul este un meci bun, cu
tensiune de încărcare de intervalul nominal 12 V baterii.
b
Putere maximă curbă și I-V caracteristice, cu puterea P
= A IV-a complotat împotriva V. A
punctului de putere maximă (MPP) este indicat.
3.5
0.5
0.1
0.2
0.3
0.4
0.5
0.6
0.7
1.0
Celula curentă / A
Celule tensiune / V
Celula de putere / W
V
MPP
PUTERE
PUTERE MAXIMĂ
LITERA (MPP)
2.0
3.0
1.5
2.5
1.4
0.2
0.4
0.8
1.2
0.6
1.0
CURENT
(b)
TWIDELL PAGINARE.indb 158
01/12/2014 11:36
§5.2 Fotovoltaice circuit proprietăți
159
caracteristic schimbă în consecință. În Mod Eficient, R
sh
(adesea luate ca infi
nit mari) și R
s
(a făcut cât mai mic posibil) scad odată cu creșterea
în temperatură. Relații empirice pentru aceste efecte de la 1 kW m
-2
expunere la soare pe Si material și de temperatură Celsius
θ sunt:
V
V
o
()
( )[1
(
)]
oc
oc 1
1
θ
θ
θθ
=
-
-
(5.3)
Am
Am
b
()
( )[1
(
)]
sc
sc 1
1
θ
θ
θθ
=
+
-
(5.4)
în cazul în care
θ
1
= 25°C este o referință de temperatură și tempera-
ture coeficienții sunt o
= 3.7 x 10
-3
(°C)
-1
, b
= 6.4 x 10
-4
(°C)
-1
. Notă, cu toate acestea,
că la temperatură constantă, V
oc
crește ușor cu expunere la soare.
Efectul net de o creștere a temperaturii la o valoare constantă a radiației este de a
reduce puterea P. O relație empirică pentru cristalin Si material este:
P(
θ ) = P(θ
1
)[1
- c(θ - θ
1
)] (5.5)
în cazul în care c
= 4 × 10
-3
(
°C)
-1
. Astfel, o bază de siliciu cristalin modul de operare la
65
°C (destul de posibil într-un deșert soare mediu) pierde aproximativ 16%
din nominală a acestuia putere; aceste module sunt cele mai eficiente la rece
temperaturi. Capacitatea de module solare, și, prin urmare, celule, să-și piardă absorbit căldură
în principal prin convecție și radiație infraroșie este, prin urmare, o importantă
provocare de design, dar de multe ori neglijat.
Restul de cerințe pentru o bună producție de energie sunt evidente
din circuitul echivalent, și anume:
1
Am
Am
ar trebui să fie un maxim, după cum se consideră în §5.4 (de exemplu, la suprafață,
minim electrice în zona de contact și minimă reflecție optică).
2
Am
D
ar trebui să fie un minim (de exemplu, prin optimă dopant de concentrare).
3
R
sh
ar trebui să fie mare (de exemplu, cu curat margine de celule formarea).
4
R
s
ar trebui să fie mici (de exemplu, prin asigurarea scurte căi pentru curenți de suprafață a
contacte electrice, și prin utilizarea rezistență scăzută contacte și de a conduce).
5
R
sarcina
= R
interne

= V/m pentru putere optimă de potrivire.


Celule solare tablouri sunt adesea asamblate dintr-o combinație de individ
module de obicei conectate în serie și în paralel. Fiecare modul este în sine
o combinație de celule în serie. Fiecare celulă este un set de elemente de suprafață
conectate în paralel (Fig. 5.6). Pentru un 36-modulul de celule, maxim
opencircuit potențial poate fi ~22 V, cu maximă a curentului de scurtcircuit, la
modul terminale ~5.5-O în condiții standard. Aceste module au fost
dezvoltate inițial pentru încărcare '12 Volt' baterii. Module mai mari sunt
acum comune, deoarece acestea sunt mult mai cost-eficiente pentru grid-connected utilizare
(de exemplu.g. 72-module de celule de la 100 la 160 W la aproximativ 32 V în plin soare,
deschide circuitul).
Când celulele sunt în serie, dificultăți vor apărea dacă o celulă sau un element
de o celulă devine defect, sau dacă o matrice este egal iluminate de umbrire
sau de inegale concentrare de lumină, pentru că o celulă care nu este
iluminat corespunzător, se comportă ca o diodă redresoare (vezi fig R4.7 și R4.10).
TWIDELL PAGINARE.indb 159
01/12/2014 11:36
160
Fotovoltaice tehnologie
Prin urmare, curentul generat într-un mod corespunzător iluminate de celule PV încearcă pentru a
trece la următoarea celulă umbrită în direcția în care este acum blocat, din
care umbrită de celule se comportă ca o diodă "în întuneric". Astfel, atunci când celulele sunt
conectate în serie cu una umbrită, puțin curent trece (analogia este
pas cu pas pe o apă furtun: un blocaj oriunde pe conducta se oprește
fluxul de apă). În consecință, umbre nu ar trebui să li se permită să cadă
pe module FOTOVOLTAICE. Dacă umbrire este inevitabilă, apoi conectat șiruri de
module ar trebui să fie aranjate astfel încât fiecare șir, fie rămâne în
soare sau este umbrit.
Mai mult decât atât, este posibil ca o umbră sau defect celula devine
supraîncălzit – un 'hot spot'. Astfel de defecte pot avalanșă, cu excepția cazului de protecție bypass
diode sunt prezentate în paralel cu o serie legate de celulă sau grup de celule. Deci,
atunci când un defect celula devine rezistiv, diferența de tensiune pe această
celulă sau grup de celule inversează și dioda în paralel devine
conductoare, astfel încât reducerea actual în defect de celule. În practică, o astfel de protecție
nu este instalat pentru fiecare celulă în cadrul unui modul, dar toată module sau linii
de module vor fi astfel protejate. În plus, celulele pot fi conectate în
paralel în cadrul mini-blocuri, deci, dacă o celulă nu, o alternativă calea de curent
există. Mini-blocuri pot fi apoi conectate în paralel, așa cum se arată în
Fig 5.6(d).
Spate
de contact
Fața de contact
grila
(a)
+

(b)
~ 4A
~ 15V
M
*
M
M
(c)
(d)
Fig. 5.6
Tipic aranjamente comerciale Si celule solare: (a) celulă; (b) modulul de 36 de celule; (c) matrice;
(d) modul cu fir în blocuri pentru a minimiza efectul de nu a reușit celula indicată (de cruce).
Nu de protecție diode sunt trase.
TWIDELL PAGINARE.indb 160
01/12/2014 11:36
§5.3 Fotovoltaice circuit proprietăți
161
§5.3 APLICAȚII ȘI SISTEME
PV aplicații sunt de două tipuri: (1) autonom (independent)
echipamente; și (2) sisteme interconectate cu utilități a rețelelor de energie electrică.
Un beneficiu de a folosi energie electrică PV de la single module este în siguranță din cauza
la tensiune scăzută și lipsa de deteriorare a modulului în cazul în care producția
shortcircuits. Prin urmare, PV de putere de la o singură celulă sau module oferă
esențiale hands-on experiență educațională, atât în interior cât și în aer liber.
Cu toate acestea, este nevoie de multă atenție, cu tablouri interconectate de module
din terminalele lor sunt "live" la lumina zilei; chiar și electricieni cu experiență
pot uita acest lucru.
§5.3.1 aplicații Stand-alone
Modulele fotovoltaice sunt foarte fiabile, nu au piese în mișcare și necesită
întreținere sau alimentare cu combustibil, altul decât un flux de energie solară.
Astfel fotovoltaice oferă una dintre cele mai bune soluții din punct de vedere tehnic pentru
a aduce pe scară largă de energie moderne rurale și îndepărtate de
țările în curs de dezvoltare, în cazul în care sistemele de doar câteva zeci de Wați poate oferi
de iluminat și de telecomunicații, care sunt de mare beneficiu social. Acest lucru este
cu atât mai mult, deoarece costul lor scade. Cu toate acestea, succesul în aceste
aplicații depinde cel putin la fel de mult pe factorilor sociali și instituționali ca pe
tehnologia (vezi §5.7 și §17.2.2).
Aceleași avantaje tehnice a însemnat că, în general, primele
utilizări semnificative de PV au fost în aplicații în cazul în care o cantitate mică de electrice
de putere a fost esențială, dar în cazul în care a fost dificil sau scump pentru a aduce în
combustibil pentru generatoarele convenționale. Primul exemplu important a fost pentru spațiu
sateliți, ceea ce a dus la considerabile de dezvoltare timpurie. Multe alte
utilizări, de obicei, în legătură cu baterii pentru stocarea energiei electrice și a tensiunii de
regulament, au beneficiat de acest lucru; exemplele includ solar de case, izolate
comunități, la distanță de centre medicale (în special pentru frigidere pentru
medicamente farmaceutice), de măsurare meteorologice, marin avertizare
lumini, telecomunicații, stații repetoare (Fig. 5.7), torțe, portabile,
aparate de radio și alte dispozitive electronice, de trafic și de semne de avertizare, parcare
metri, etc. Dacă un sistem stand-alone nu are nevoie de baterie de stocare (de exemplu,
pentru pomparea apei), apoi o sarcină-de potrivire și de reglare a tensiunii
de interfață este important. Aplicații Stand-alone de multe ori funcționează în mod automat,
dar au nevoie de curățare periodică și întreținerea bateriei de către personal instruit
(dar a se vedea Caseta 5.1, §5.3.3, despre auto-curatare sticla).
Deoarece costul de sisteme FOTOVOLTAICE a scăzut, așa că a distanța de la
rețeaua electrică la care instalațiile sunt costuri competitive. De exemplu,
este de multe ori mai ieftin, și întotdeauna mai sigur, pentru a instala semne de circulație, parcare
metri sau de iluminat pentru trotuare ca stand-alone alimentat cu energie solara sisteme
decât să instalați conectare la rețea și de măsurare pentru cantități mici de
energie necesara. Mai mult decât atât, cele mai recente dispozitive electronice, inclusiv LED-uri
TWIDELL PAGINARE.indb 161
01/12/2014 11:36
162
Fotovoltaice tehnologie
Fig. 5.7
Tipic aplicații stand-alone de celule fotovoltaice. (a) Alimentează un semnal de cale ferată cutie într
- o zonă izolată din Australia. (b) modul PV alimentează luminile pentru o casă în Solomon
Islands.
(a)
(b)
(light emitting diode) de iluminat, întotdeauna au tendința de a utiliza mai puțină energie decât
predecesorii, deci PV este puterea chiar și mai susceptibile de a fi utilizate.
§5.3.2 Grid-connected systems
Începând cu anul 2000, conectate la rețea, sisteme, așa cum se arată în fig 5.1 și 5.8, au
fost cea mai mare și cea mai rapidă creștere, utilizarea de celule fotovoltaice. În general,
există două clase de astfel de grilă 'distribuite/încorporat' generation:
1
'Microgenerare la sau de pe o clădire, prin care PV de alimentare se conectează
la partea de consum de utilitate metri, cu exces de putere exportate
la rețeaua locală de distribuție linii.
2
'Fermele solare de o gamă largă de module conecta direct la
scalate în mod corespunzător rețelei de linii.
Fig. 5.9(a) prezinta fluxurile de energie și conexiunile tipice
microgenerare la o clădire. Solar module (panouri) sunt fixate sau integrate
cu acoperiș, sau pe un drum liber-în picioare-cadru în apropierea clădirii. La
microgenerated putere, de obicei se conectează la partea de utilizator de utilitate
metri, deoarece proprietarii beneficia, în principal, la folosirea propria lor putere și
deci reducerea importate (cumpărate) de putere. Excesul de putere fluxurile de departe de
clădire, ca și de export în utilitarul de distribuție linii de grilă, pentru care
proprietarii se așteaptă să fie plătit. În împrejurare fericită că plata
per unitate pentru exportate de putere este mai mare decât prețul unitar al importate
putere, microgenerated putere ar trebui să fie conectate la utilități
partea de utilitate metri pentru a maximiza veniturile. O combinație de utilitate
metri și proprietarilor de metri permite cel puțin trei fluxurile de energie să fie
măsurate: (i) generate de putere, (ii) exportate putere; și (iii) importate
de putere. Dispozițiile financiare pentru import si export de energie
(de obicei numit un 'feed-in tariff', §17.5.1) variază foarte mult în funcție de țară și de utilitate.
TWIDELL PAGINARE.indb 162
01/12/2014 11:36
§5.3 Fotovoltaice circuit proprietăți
163
(a)
Fig. 5.8
Exemple de conectate la rețea a instalațiilor fotovoltaice.
o
Apartamente în Freiburg, Germania; PV tablouri de 11 module furnizează atât umbra pentru
windows și energie electrică pentru fiecare apartament.
Foto: autorul.
b
Un serviciu de transport de stație în Australia cu PV de acoperiș.
Fotografie prin amabilitatea de BP Solar.
(b)
În unele locuri, feed-in tariff include chiar și unele de credit pentru
'abatedcarbon" și "curat-putere'.
Fig. 5.9(b) arată legătura dintre un megawatt solare la scară de fermă (de exemplu,
care este prezentat în Fig. 5.1) la rețeaua de distribuție; în general, nu există nici un local
TWIDELL PAGINARE.indb 163
01/12/2014 11:36
164
Fotovoltaice tehnologie
sarcina de orice semnificație. Modulele sunt montate aici pe fix-orientarea
cadrelor, optimizat în pantă și de direcție pentru locația; cu toate acestea,
fermele solare din senin regiuni sunt susceptibile de a utiliza de soare de urmărire
cadre. Interconectarea liniilor electrice sunt amplasate subteran. Tot
site-ul poate fi de mai multe hectare. Intermediar invertoare transforma
DC puterea de a AC (probabil la ~500 V) pe linii care duce la o
transformare de unde transformatoare trece puterea de a utilitarul linie de
la, probabil, 15 kV. Stația include toate de metri, monitorizare și
electrice, echipamente de siguranță. Supravegherea este de interogare de la distanță de
monitorizare și de inspecții periodice.
Toate aceste sisteme folosesc invertoare de a transforma DC energie electrică la
tablouri FOTOVOLTAICE în curent ALTERNATIV, compatibil cu utilitatea rețelelor electrice (a se vedea
§R1.3).
Conectate la rețea (grid-tie) invertoare sunt diferite de stand-alone
invertoare; ei folosesc predominant linie-tensiune de frecvență pe imediată
utilitate linie de grilă ca un parametru de control, astfel încât sistemul PV de ieșire
devine sincronizate cu rețeaua. Puterea este exportat din PV
sistem atunci când invertorul de tensiune de ieșire devine mai mare decât linia
de tensiune; acest lucru se întâmplă deoarece solare-energie generată de forțe în sine în


Fotovoltaice
module de
~ 5 kw
Clădirea sarcini
Utilizare metri kwh
Siguranța
Siguranța
AC
ieșire
Controller
cu
invertor
DC
circuit
intrare
Export
de metri
Export de metri
kwh
Import
metri
240/120v
AC
Utilitate
distribuție
linii de grilă
~100 kW matrice
~100 kW matrice
~15 kV
AC
Utilitate
distribuție
linii de grilă
Substație
și
transformator
AC
AC
DC
DC
Transformator
~ 15 kV
AC
~ 300 V
Intermediar
controler/
invertoare
(a)
(b)
Fig. 5.9
Schema (nu diagramele de cablare) de: (a) ~5 kW fotovoltaice microgenerare conectate
în circuite electrice dintr-o clădire cu conectare la rețea, (b) o mare MW de capacitate solara
ferma cu multe ~100 kW liber-în picioare tablouri conectat printr-un site de transformare de la un
utilitar de rețea de distribuție rețea (a se vedea, de asemenea, Fig. 5.1 pentru o astfel de fermă solară).
TWIDELL PAGINARE.indb 164
01/12/2014 11:36
§5.3 Fotovoltaice circuit proprietăți
165
linia de alimentare. O astfel de linie comutată' invertoare deconecta automat dacă
utilitarul de putere eșuează, așa că nu neprevăzute și potențial periculoase
"rogue" tensiune apare pe linie de grilă. La puterea de vârf maximă
trackere (MPPTs) sunt încorporate cu invertoare, ca parte a
conexiunii/unități de control.
§5.3.3 Echilibru a sistemului (BoS) componente
Un sistem FOTOVOLTAIC este mult mai mult decât celule și module, în ciuda lor de
sofisticare. Alte echipamente și sisteme de fixare sunt numite 'echilibru de
sistemul' (BoS) componente.
(a) BoS pentru sisteme stand-alone
Fig. 5.10(a) arată schematic cum matrice de Fig. 5.6 poate fi
conectat la un DC încărca într-un sistem stand-alone. PV matrice este prezentat
configurat pentru a nominal 'de 12V bateriile și sarcini, dar alte tensiunea
configurații sunt posibile în funcție de aparatul de rating (de exemplu 24 V).
Module și tablouri de module au un circuit echivalent și I-V
caracteristici ca Smochine 5.4 și 5.5, dar cu valori numerice în mod corespunzător
extinsa.
Puterea maximă este obținută prin controlul V și nu să se întindă pe
linie de tensiune maximă, ca a primit insolație și rezistența de sarcină variază
(vezi Fig. 5.5). În practică, temperatura de operare, de obicei, crește cu
iradiere; aceasta modifică tensiunea și curentul de la fix
caracteristici de temperatură, așa cum rezultă de (5.3) și (5.4). Efectul net este că
puterea de vârf de linie este mai aproape de verticală decât cele indicate în Fig. 5.5(a).
Tensiunea la borne a unui electrice baterie de stocare (uneori numit
un acumulator') rămâne aproape constantă indiferent de curentul de încărcare,
dar crește odată cu creșterea stării de încărcare. Prin urmare, de potrivire
MPPT
Invertor
240 Vac
aparat
M
M
Controller
Motor
(a)
(b)
12 Vdc
aparat
De depozitare
a bateriei
Matrice solare

+
+15V
Fig. 5.10
Schema de principiu a unui stand-alone sistem fotovoltaic;
o
Nominal 12 V DC sistem cu baterie controler de încărcare, cu posibile 240 V (sau
110 V) AC aparate;
b
sistemul cu puterea de vârf maximă tracker (MPPT).
TWIDELL PAGINARE.indb 165
01/12/2014 11:36
166
Fotovoltaice tehnologie
matrice optim de tensiune și tensiunea nominală a bateriei, care V/am linia de încărcare
de încărcare a bateriei poate fi asortat aproape la maxim puterea de linie din
matrice dacă o mai sofisticate operatorul nu este încorporată. Într-un asemenea
caz, bateria reglează tensiunea.
Aproape toate stand-alone, sisteme FOTOVOLTAICE necesita baterie de stocare a energiei electrice,
cel mai evident pentru iluminat pe timp de noapte, dar, de asemenea, pentru a face față cu sarcina de valuri,
astfel
ca pentru transmisie radio. Bateriile sunt discutate în §15.5. Durata de viață
și fiabilitatea unui sistem FOTOVOLTAIC sunt îmbunătățite folosind un scop-proiectat
adâncime de descărcare de gestiune solar de baterie și nu un vehicul baterie. Un controler
oferă maxim specificat rata de încărcare și adâncimea de descărcare de gestiune, și
este aproape esențial pentru o funcționare fiabilă. Chiar și cu un controler, baterie
durata de viață este, de obicei, doar trei la șase ani – mult mai puțin decât modul de
viață, și de multe ori mai puțin decât sistemul de designeri implica! Operatorul
poate să includă un maxim de putere de vârf tracker (MPPT) într-o singură unitate.
Direct sarcinile electrice nu reglementează în mod direct de tensiune și de curent,
la fel ca o baterie. Prin urmare, un intermediar controler este folosit pentru a
separa (decuplarea) tensiunea și curentul de optimizare a PV matrice de
tensiunea cerința de sarcina. Operatorul poate încorpora un
electronic MPPT astfel DC tensiune și curent din matrice sunt
controlate, astfel că puterea maximă este atinsă după expunere la soare modificări
(Fig. 5.10(b)). MPPTs sunt adesea construite în stand-alone solar de pompare
sisteme cu nume ca 'maximizer" sau "liniară de curent de rapel' și poate
permite 95% din cea maximă de ieșire pentru a ajunge la pompa de apa sub
diferite conditii solare.
Pentru a opera AC aparate (240 V/50 Hz sau 110 V/60 Hz) de la un DC PV
de aprovizionare necesită un invertor, așa cum se arată pe partea stângă a Fig. 5.9(a). Un
invertor autonom utilizează o frecvență internă a generatorului și de comutare de circuite
pentru a transforma joasă tensiune de curent continuu la tensiune mai mare de alimentare AC. În
forma de AC de undă poate fi un val pătrat (ieftin invertor) sau
aproape pure sine wave (sofisticat solid-state electronice invertor). De
invertor trebuie să fie dimensionate pentru creșterea curenților asociate cu
motorde pornire (dacă este cazul), dar nu atât de mare că acesta funcționează în mod normal la o mică
fracțiune a sa putere nominală (să zicem, <15%) și, prin urmare, la eficiență slabă
(<85%). Solid-state invertoare electronice sunt disponibile în comerț, cu
fiabilitate excelentă și 95% la 99% eficienta la un cost rezonabil.
(b) BoS FOTOVOLTAICE conectate la retea
Pentru microgenerare la clădiri, în plus față de fixare și de cablare,
balanța de echipamente de sistem este o unitate de control pentru
conexiunea la rețeaua de alimentare, linii suplimentare de contorizare, fitiluri de siguranță și
switch-uri. Unitatea de control include în mod normal una sau mai multe invertoare de a
transforma PV DC la AC din clădire rețeaua de energie electrică, care
la rândul său este conectat la utilități locale liniile de alimentare de la de obicei 110V/60 Hz
sau 240 V/50 Hz. Invertorul este întotdeauna legat-grilă, astfel încât producția
menține sincronismul cu rețeaua de energie electrică; în cazul în care grila de aprovizionare eșuează, atunci
TWIDELL PAGINARE.indb 166
01/12/2014 11:36
§5.4 Maximizarea eficienței celulelor solare (si celule)
167
invertor imediat taie, dar va reduce în mod automat atunci când
rețeaua de alimentare se întoarce. Proprietarii pot adăuga un plus de echipamente de monitorizare pentru
înregistrări de performanță și de informații pentru a face uz de pe site-ul
puterea.
Modulul acoperă poate necesita curățare, mai ales în medii uscate,
dar acoperă, de auto-curatare sticla' se acumulează mai puțin depozite și
selfclean în ploaie (vezi Caseta 5.1).
CASETA 5.1 DE AUTO-CURATARE STICLA DE PE MODULUL PV ACOPERĂ
Așa-numita auto-curățarea de sticlă este folosit pentru partea din față înclinată capacul din mai multe module. Acesta este fabricat
cu o ~25 nm monostrat de dioxid de titan (TiO
2

) pe sale orientate spre exterior, de suprafață, care are asociate


două
efecte benefice pentru relaxarea organice murdărie și praf: (1) un efect catalizator organic care se descompune murdăria
în ultraviolete solare (fotocataliză); și (2) o reducere a tensiunii superficiale și de suprafață unghiul de contact al
apei de pe coperta (hidrofil efect), care permite de ploaie sau un furtun de apă pentru a rula ca o foaie de film, așa că transportă
departe descompus murdărie. Procesul a fost dezvoltat și patentat de către Pilkington Sticlă.
§5.4 MAXIMIZAREA EFICIENȚEI CELULELOR SOLARE (SI CELULE)
Eficiența și eficacitatea costurilor de celule fotovoltaice sunt
îmbunătățite continuu de cercetare, dezvoltare și producție,
know-how, dar de multe variabile și tipuri de celule, acest subiect
extrem de complex. În această secțiune, vom explica cea mai mare parte aspecte-cheie de
forma dominantă de Si celule, ocazional, referindu-se la fizica de bază
de celule fotovoltaice în Revizuire 4. Cel puțin, această secțiune ar trebui să
indice rafinament extrem de fabricarea de celule solare.
Celulele fotovoltaice sunt limitate în eficiența prin multe pierderi; unele dintre
acestea pot fi evitate, dar altele sunt intrinseci sistemului. Unele limite
sunt evidente și pot fi controlate în mod independent, dar altele sunt complexe
și nu pot fi controlate fără a produce efecte interdependente. De
exemplu, creșterea dopant de concentrare poate avea atat avantaje, cat
si efecte nocive. Tabelul 5.1 descrie tipic pierderile comerciale Si
joncțiune p–n singur cristal celule solare în AM1 iradiere, luate în ordine
de sus a celulei la partea de jos (vezi Fig. 5.11). Din păcate, nu
este nici un standard de convenții pentru denumiri de pierderea factori, care va
fi considerat mai târziu.
Rețineți că cele mai semnificative ineficiențele sunt intrinseci nepotrivire
de spectrul solar la un singur strat band gap (Caseta 5.2, §5.4.2). O
strategie de a reduce aceste ineficiențe este de a avea multistrat
(heterojunction) dispozitive cu straturi potrivite pentru diferite regiuni ale solară
cu spectru (§5.6.2); astfel de îmbunătățiri de eficiență, de obicei, permite celulei să
fie subțire, pentru ~2
mm grosime, mai degrabă decât ~200mm, reducând astfel cantitatea de
materiale scumpe, și, astfel, costul de celulă și puterea de ieșire.
TWIDELL PAGINARE.indb 167
01/12/2014 11:36
168
Fotovoltaice tehnologie
Echilibrul între costurile, complexitatea și eficiența este un delicat
comerciale judecată atât pentru producători și utilizatori de celule solare.
Într-adevăr, reducerea costurilor ($/Watt) de energie electrică PV este un factor major pentru
dezvoltarea a numeroase forme de celule PV, altele decât cristaline Si
(a se Vedea §5.6, Fig. 5.18 și Tabelul 5.2).
În general, o mai mare eficienta celulelor permite tablouri de un anumit total
de energie să aibă suprafață mai mică cu mai puțin de încapsulare, transport
și costuri de instalare; astfel o eficiență crescută este un factor major, dar nu
la costul de mare. Există câteva specialist aplicații, cum ar fi energia solară masina
de curse sau o călătorie în spațiu, în cazul în care utilizatorii caută cea mai mare eficiență cu
suficientă durabilitate, aproape indiferent de cost. În practică, dominantă
factor furnizarea de produse de cost mai mic este crescut și automatizat,
capacitate de producție, condus de un puternic și de creșterea cererii. În plus,
având să îndeplinească standardele internaționale de testare și certificare
oferă o mai bună calitate și satisfacția consumatorilor.
În următoarele subcapitole considerăm o bază cu un singur strat Si solar
mobil, care este încă dominant material din punct de vedere comercial. Pierderile sunt
indicate ca un procent aproximativ de expunere la soare în această etapă,
inițial AM1 = 100%. Efectele sunt descrise în ordine de sus
în bază de celulă, așa cum se arată în Tabelul 5.1, în cazul în care eficiența
factori indică proporția din restul de iradiere, care este util
absorbit în această fază fotovoltaice în generarea de energie electrică. Unele
pierderile sunt intrinseci (nu pot fi evitate) și unele pierderi pot fi reduse
prin superior fabricarea. Până în 2013, cel mai bun laborator de "campion"
singlelayer Si celule a ajuns la aproximativ 25% eficiență și cea mai bună reclamă
celule de aproximativ 20%.
§5.4.1 Top-suprafața electrice-contact obstrucție zona
(intrinseci pierdere de ~3%)
Curentul electric frunze suprafața superioară de o rețea de contacte metalice
aranjate pentru a reduce rezistență serie de pierderi în suprafață (a se vedea §5.4.10).
Acoperiri antireflex
n 0.2–1.0 m
p 250-400 m
p
+
pentru BSF: 0,2 m
La spate contact
de următoarea serie de celule
De sus grilă de
celula anterioară
<~15 cm
Spate/spate contact
Fața de contact
Conexiune de bandă
µ
µ
µ
Fig. 5.11
Structura de bază de joncțiune p–n de celule solare. Nu sunt indicate de acoperire (sticlă sau, uneori,
de plastic) de mai sus celulei și de umplere între capac și celula. BSF: spate suprafata de teren.
TWIDELL PAGINARE.indb 168
01/12/2014 11:36
§5.4 Maximizarea eficienței celulelor solare (si celule)
169
Contactele sunt de obicei format dintr-un ecran-procesul de imprimare, ca pentru
dispozitive microelectronice; procesul este similar, în principiu, la care a folosit
pentru imprimarea de pânză și imagini. Aceste contacte au un finit suprafață
zonă și astfel încât acestea să acopere partea de altfel suprafeței active; această pierdere
de domeniu nu este întotdeauna reprezentat în calculele de eficiență. Laser-cut
caneluri în care contactele electrice sunt plasate permite suprafață
obstrucție a fi redus având în același timp suficient de contact electric.
§5.4.2 pierderi Optice, de sus și din spate suprafete
(a) Reflectarea reducere la suprafață (pierdere de ~3%)
Fără măsuri speciale de precauție, în partea de sus-suprafața de reflexie de
semiconductori este mare, la aproximativ 40% din radiația solară incidentă. Din fericire,
acest lucru poate fi redus dramatic de film subțire tratament de suprafață (de exemplu, cu
grosimea de film controlate pentru a produce interferențe constructive
a reflectat grinzi: Fig. 5.12). Avem în vedere trei caracteristici ale
problemă.
Caracteristică 1: ordinare suprafața de reflexie. Pentru intensitatea de reflecție,
ia în considerare trei materiale (aer, capac, semiconductor) de indice de refracție
n
0
, n
1
și n
2
. Pentru dielectric electric, materiale izolante, reflecta-
ance la aer/coperta de interfață, prima este:
r=
-
+
n
n
nn
(
)
(
)
refl
0
1
2
0
1
2
(5.6)
Tabelul 5.1
Aproximativ limite de eficiență într-un singur strat (homo-joncțiune) cristalin Si celule solare
(vezi §5.4 pentru explicații de fiecare proces)
Text §
Cauza de pierdere
Puterea
pierdere/câștig
(aproximativ) %
Elementare
de eficiență schimba
pe proces
Energia
rămasă
%
5.4.1
Top contact obstrucție
-3
0.97
97
5.4.2
Suprafață de reflecție cu
antireflex film în loc
-3
0.97
94
5.4.2
Suprafata din spate reflecție
+3
1.03
97
5.4.3
Nici fotovoltaice de absorbție: hv < E
g
-23
0.77
75
5.4.4
Excesul de fotoni de energie sub formă de căldură:
hv > E
g
-33
0.67
50
5.4.5
Eficiența de captare a
-0.1
0.99
49
5.4.6
Eficiență de colectare
-10
0.90
44
5.4.7
Tensiune factor de eV
B
<E
g
-20
0.8
35
5.4.8
Factor de umplere = (max. putere)/ am
sc
V
oc
-12
0.88
31
5.4.9
Factorul de idealitate O, recombinare pierderi
-5
0.95
29
5.4.10
Rezistență serie
0.3
0.97
26
5.4.11
Rezistența șunt
0.1
0.99
25
5.4.12
Livrate putere
25
TWIDELL PAGINARE.indb 169
01/12/2014 11:36
170
Fotovoltaice tehnologie
De exemplu, cu nici o interferență, pentru aer (n
0
= 1) de plastic (să zicem, n
1
= 1.6),

r
refl

= 0.36/6.76 = 5.3%. Pentru aer Si, situația este mult mai complexă


ca semiconductori au un indice de refracție reprezentat de un complex de
număr, deoarece acestea sunt o parte conductoare. Si reflexie, prin urmare, este
dependentă de frecvență, și variază în mărime de peste activă a spectrului,
cu o medie de o magnitudine de aproximativ n
2

≈ 3.5 pentru Si. Cu nici un film subțire de acoperire,


înlocuind în (5.6) oferă 31% pentru Si reflexie în aer, care este mult prea
mare.
Caracteristică 2: interferențe distructive. Fig. 5.12 explică cum un film subțire
reduce reflexia dacă principale reflectate componente o și b sunt (i) de egală
intensitate și (ii) diferă în faza de
p radiani (l /2 calea diferenta). Pentru
reflexie la fiecare suprafață să fie egală, n
1
=
√(n
0
n
2
), și pentru interfer-
ence grosimea filmului ar trebui să fie t =
l /(4n
1

). Există doar o singură lungime de undă


pentru care această condiție este îndeplinită exact; cu toate acestea, de-a lungul solară
cu spectru de bandă largă de reflexie este considerabilă cu o peliculă subțire care acoperă de
n
1
= 1.9, grosimea t = 0.08
mm, pentru care în bandă largă de reflexie
de tip "sandwich" este redus la ~6%. Mai multe straturi subțiri pot reduce
în bandă largă de reflexie a <3%.
Caracteristică 3: texturare. O altă metodă de a reduce partea de sus a suprafeței de reflexie
pierderi utilizează configurații geometrice, texturare, care reflectă fasciculul
pentru o a doua oportunitate de absorbție (a se vedea schița diagrame și
legende în Fig. 5.13(a) și (b)).
(b) suprafața Posterioară reflecție și lumină capcane
Fotoni care trec prin strat semiconductor fără absorbție
pot fi reflectate de suprafața din spate pentru o a doua trecere. Acest lucru
permite semiconductoare strat să fie mai subțire și reduce costul materialelor.
Dacă acest din spate de reflexie este inegală, atunci o mare parte a radiației reflectate
devine prins de studiu randomizat interne reflexie de la suprafața superioară.
o
b
n
0
n
1
t
n
2
Fig. 5.12
Antireflex film subțire.
TWIDELL PAGINARE.indb 170
01/12/2014 11:36
§5.4 Maximizarea eficienței celulelor solare (si celule)
171
CASETA 5.2 RADIAȚIEI SOLARE ABSORBȚIE LA JONCȚIUNE P–N
Proprietățile detaliate ale radiației solare au fost considerate pe deplin în Capitolul 2. Fig. 5.14 arată solar
spectrum (reprezentate în termeni de fotoni energia (mai degrabă decât lungimea de undă) (Fig. R4.11 în Revizuire 4 arată
la fel, împreună cu parcele similare cu lungime de undă
l, și numărul de fotoni ca axa orizontală). Astfel de
transformări matematice schimbare vârfuri de curbe, dar nu zona de sub ele, care este
adecvat iluminare energetică totală G.
Pentru generarea de energie fotovoltaică într-un tipic de celule solare (de exemplu Si material), factorii esențiali este indicat
în Fig. 5.14 sunt după cum urmează:
1
Spectrul solar include frecvențe prea mici pentru sistemele fotovoltaice de generare (h
n<E
g
) (O regiune).
Absorbția acestor joasă frecvență (lungime de undă) fotoni produce căldură, dar nu de energie electrică.
λ (µm)
1
2
3
4
5
1.24
500
400
300
200
O
B
C
100
Infraroșu
Vizibil
Ultraviolete
0.62 0.41 0.31 0.25
h
(eV)
ν
[d
G
/d(h
)]/(
W
m
-2
eV
-1
)
ν
Fig. 5.14
Indicativ teren de solar iluminare energetică spectrală împotriva foton de energie pentru a ilustra foton
de absorbție pentru producerea de energie electrică într-un singur nod Si celule solare.
Rețineți cele trei regiuni în diagramă:
O
Fotonii au energie hv mai puțin decât benzii interzise E
g
și nu sunt absorbite.
B
Reprezintă proporția de iluminare energetică spectrală, care este transformată în energie electrică.
C
Reprezintă proporția de iluminare energetică spectrală, care este disipată sub formă de căldură în
materialul pentru h
n > E
g

.
Fig. 5.13
Suprafețele de sus pentru creșterea absorbției după inițială reflecție; scară de la 10 la 100 mm: (a)
idealizat texturate forma (de exemplu, prin gravură chimică); (b) forma structurata (de exemplu, prin laser
de prelucrare).
(a)
(b)
TWIDELL PAGINARE.indb 171
01/12/2014 11:36
172
Fotovoltaice tehnologie
2
La frecvențe al benzii de absorbție (h
n >E
g
), excesul de energie fotonica (h
n - E
g
) se pierde sub forma de caldura
(regiunea C).
3
Prin urmare, nu este optimă a benzii de absorbție pentru a se potrivi un spectru solar pentru maximum de energie electrică
de producție (Fig. R4.12). Distribuția spectrală (și iluminare energetică totală) variază cu adâncimea, prin
Atmosferă și cu nebulozitate, umiditate, poluare, etc. (A se vedea §2.6.2 privind aerul raportul de masă, adică AM0
în spațiu, AM1 de la zenith, AM2 la zenith unghi de 60°; AM1.5 condiții sunt de obicei considerate ca standard
pentru celule solare de proiectare.)
4
Numai energia în regiunea B din Fig. 5.14 este potențial disponibile pentru fotovoltaice de putere într-un singur nod
de celule solare. Proporția maximă de total de energie [B/(O
+ B + C)], unde Un, B, C sunt zonele regiunilor
a, B, C, este de aproximativ 47% pentru Si, dar suma exactă variază ușor cu distribuție spectrală. Nu toată această
energie poate fi generat la fel de utile putere, din cauza celula de tensiune V
B
fiind mai mică decât banda interzisă E
g
(a se vedea
Fig. R4.3 și §5.4.7); deci utile de putere, de la curent am, este V
B
Nu, nu E
g
Eu. Prin urmare, în practică, cu V
B
/E
g

≈ 0.75, doar un maxim de aproximativ 35% ( = 75% 47%) din radiația solară este potențial disponibile pentru
conversie de energie electrică, cu o singură bandă de celule fotovoltaice – așa-numitul 'Shockley-Queissner' limita.
Prin urmare, căutarea pentru mai multe benzii celule și alte sisteme sofisticate, care pot ocoli această limită.
Efecte similare se aplică pentru orice semiconductor. Luați în considerare de ieșire a unei celule solare; cu o mai mare decalaj
trupa, la
ieșire are tensiune mai mare dar mai mic de curent, deoarece mai puține fotonii au energie suficientă, și deci puterea
se reduce. În schimb, cu un mic decalaj trupa, curentul crește (mulți fotoni se califica) dar tensiunea este
mai mică. Undeva în între, puterea de ieșire maximizează. Pentru spectrul solar la AM1.5, acest vârf este
la o trupa decalaj de aproximativ 1.6 eV (vezi Fig. R4.12).
§5.4.3 Foton cu energie mai mică decât banda interzisă (pierderea ~23%)
Referindu-se la Fig. R4.1, fotoni de energie cuantică h
n < E
g
nu poate con-
tribut direct fotovoltaice de generația actuală. Pentru Si (E
g
≈ l.l eV) astfel
inactiv de undă au
l >1.1 mm și include 23% din AM1 iradiere
(a se vedea Caseta 5.2). Dacă aceste mai mare lungime de undă a fotonilor (sub pragul
de frecvență) sunt absorbite în dispozitiv, încălzire are loc cu o temperatură de
creștere, care reduce producția de energie din active, mai scurtă lungimea de undă
a fotonilor. Strategii pentru a depăși această ineficiență includ: (i) eliminarea
de lungime de undă a fotonilor din fasciculul incident de filtre (puțin probabil să
fie o soluție practică); (ii) utilizarea de căldură într-un combinat de căldură solară și
PV sistem de putere (sensibil, dar nu comun); și (iii) fotochimice
'up-conversie', prin care grupuri de mai multe mai mare lungime de undă a fotonilor
combina intr-un fotochimice substrat pentru a emite o lungime de undă scurtă
active de fotoni (de cercetare).
§5.4.4 Excesul de energie fotonica (pierdere de ~33%)
Așa cum sa explicat de către Fig. R4.1 și Caseta 5.2, excesul de energie de active
fotoni (h
n – E
g
), de asemenea, apare sub formă de căldură.
§5.4.5 eficiența de Captare (pierderea ~0.4%)
Fotonii cu energie quanta h
n > E
g
ar trebui să producă perechilor electron-gol,
deci, crearea de dispozitivul curent. Fracțiunea de aceste "activ" fotoni
TWIDELL PAGINARE.indb 172
01/12/2014 11:36
§5.4 Maximizarea eficienței celulelor solare (si celule)
173
producerea perechilor electron – gol este eficiența de captare a', care de obicei se
apropie de 100%, deoarece fie (i) semiconductoare de grosime este
suficientă pentru absorbția cu o singură trecere (§R4.2), sau (ii) reflectă straturi în partea
din spate a celulei reveni transmise de radiații pentru o secundă sau mai multe treceri.
Aceasta din urmă este lumina capcane, la fel ca în cu pereți subțiri celulele de siliciu depus pe un
sprijin substrat de sticlă.
§5.4.6 eficiență de Colectare
Eficiență de colectare este un termen vag folosite în moduri diferite de către diferiți
autori. Acesta poate fi aplicat pentru a include pierderile descrise în §5.4.3 și
§5.4.4, sau, de obicei, ca aici, să electric de colectare de taxe după purtător
generație. Eficiență de colectare, prin urmare, este definită ca proporția de
radiații generate perechilor electron-gol care produc curent în
circuit extern. Pentru 10% eficiența globală celule, eficiența de colectare este
de obicei de aproximativ 0,7, dar pentru 0.9 20% eficienta celulelor; deci eficiență de colectare
de îmbunătățire este un important design-țintă.
Există mulți factori care afectează eficiența colectorului. O îmbunătățire
este suprafața de teren (BSF). Un strat de crescut concentrația de dopant este
format ca un strat suplimentar dincolo de joncțiune p–n (de exemplu, 1
mm a p
+
pe p
pentru a produce o nouă joncțiune de ~200 kV m
-1
(Fig. 5.15)). Electron minore-
itate transportatorii format în p strat aproape de acest p
+

regiunea sunt "reflectată" jos


un gradient de potențial înapoi spre principalele joncțiune p–n, mai degrabă decât de sus
în gradient de la spate de metal de contact. De recombinare Electron – gol la
spate de contact este prin urmare redus.
Similare diodă-cum ar fi straturi, prezentat aici ca un n pe p mobil, poate fi adăugat
la suprafața frontală (de exemplu, n
+

pe n) pentru a produce aceleași beneficii pentru a reduce


recombinarea purtătorilor minoritari, care prevede că absorbție optică este
nesemnificativă; acest efect este numit passification. Sub suprafața frontală
contacte metalice, chiar mai puternic dopat regiuni (de exemplu, n
++

) reduce
recombinare și de a reduce rezistența de contact.
§5.4.7 Tensiune factorul F
v
(pierdere de ~20%)
Fiecare foton absorbit produce perechilor electron – gol, cu
o diferență de potențial electric de E
g
/e (l.eu V în Si). Cu toate acestea, doar o parte (V
B

)
de acest potențial este disponibil pentru EMF de un circuit extern. Acest lucru este
făcut clar în Fig. R4.3, în cazul în care deplasarea benzilor de peste
junction într-un circuit deschis produce pe banda potențial V
B
. Tensiunea
factor este F
v
= eV
B
/E
g
. Pentru Si, F
v
variază de la ~0.6 (pentru 0.01
W m material)
la ~0,5 (0,1
W m material), astfel încât în Si V
B
≈ 0.66 V-0.55 V. În GaAs,
F
v
este de ~0.8.
A 'disparut' EMF (
f
n
+
f
p

) în Fig. R4.3 se produce din cauza o întrerupere a


circuitului de nivelul Fermi pe jonctiune echivaleaza la dopant n și
p niveluri, și nu la strămutate conducere-pentru-benzii de valenta niveluri.
TWIDELL PAGINARE.indb 173
01/12/2014 11:36
174
Fotovoltaice tehnologie
A crescut dopant crește concentrația F
v
(0.01
W m Si are o mai mare
V
B
și V
oc
de 0,1
W m Si), dar alte efecte limita maximă de dopant
concentrațiile în Si la ~10
22
m
-3
de 0.0 l
W m materiale.
Atunci când se produce curent de sarcină, mișcarea de transportatori conform
înainte părtinire produce căldură ca rezistiv impedanță internă de încălzire. Acest lucru
poate fi inclus ca tensiune factor de pierdere, ca idealitate factor de pierdere (§5.4.9) sau, ca
aici, prin seria de încălzire cu rezistență (§5.4.10).
§5.4.8 factor de Umplere (curba factor) F
c
(intrinseci pierdere de ~12%)
Puterea maximă produsă de o celulă este nu produs am
sc
V
oc
dar
cantitate mai mică P
max

la punctul de putere maximă. Acest lucru este pentru că


I–V caracteristică este puternic influențată de p–n polarizarea diodei
caracteristice (Fig. R4.6).
Astfel ca celula solară tensiunea de ieșire este ridicată spre V
oc

dioda
devine din ce în ce mai înainte părtinitoare, astfel încât creșterea internă
recombinare curent am
r
pe jonctiune. Acest lucru este necesar comportament este tratat
ca un drept fundamental pierderi în sistem, măsurată prin factor de umplere:
=
FP
AM V
/(
)
sc oc
max
(5.7)
Valoarea maximă a lui F în Si este de 0.88.
§5.4.9 Idealitate factorul O (pierdere de ~5%)
În practică celula caracteristică să nu urmeze exact ecuația (R4.23),
derivat din proprietăți diodă, și este mai bine reprezentată de (R4.2.4):
=
-
-
EU AM
Am
eV AkT
[exp(
/
) 1]
0
Am
(5.8)
în cazul în care aici am
Am
și , prin urmare, nueste considerat pozitiv pentru celule PV.
Idealitatea factorul O (>2 pentru mai multe celulele comerciale) permite
Grila fata
e–
h+
n
p
p+
h+
e–
e–
Contact Metal
Fig. 5.15
Nivelul de energie într-o celulă cu suprafața de teren' (BSF) a indicat ca p
+

la partea din spate de metal


de contact. Acest strat suplimentar reduce difuzia scurgere de electroni curent transportatorii de la spate
de celule, prezentat aici ca un 'n pe p' mobil.
TWIDELL PAGINARE.indb 174
01/12/2014 11:36
§5.4 Maximizarea eficienței celulelor solare (si celule)
175
de recombinare electron-gol pierderi în joncțiune. Acest efect tinde, de asemenea,
pentru a schimba V
oc
și 1
0
, astfel încât, în general optimă de ieșire ar avea loc numai dacă
O
= 1.
Nedorite de recombinare electron-gol a fost deja menționat, pentru
suprafata de teren (§5.4.6). În interiorul celulei, recombinare este diminuat dacă:
1
Difuzie drumurile sunt lungi (în Si din ~50 ~100
mm). Acest lucru necesită
timp minoritate purtător de vieți (în Si pana la 100
ms).
2
Intersecția este aproape de suprafață (în 0.15
mm, mai degrabă decât 0.35
mm ca în normal Si celule).
3
Materialul are câteva defecte, altele decât dopant.
Suprafața de recombinare efecte sunt influente din cauza defecte și
imperfecțiuni introdus la crystal feliere sau la materialul de depunere.
§5.4.10 rezistență Serie (pierderea ~0.3%)
Celula solară curent trece prin materialul în vrac pentru față și spate
de contact. La spate contact zona poate acoperi întreaga celulă și
contribuția la seria ohmice rezistența este foarte mic. Cu toate acestea, partea de sus
de suprafață ar trebui să fie expuse la suma maximă de lumina cu
partea de sus a zonei de contact reduse la minimum, astfel provocând relativ lungă calea de curent de
lungimi semnificative cu rezistență serie. Îmbunătățiri au fost
făcute la fața contacte (de exemplu, de a avea cu laser îngust tăiat canale
în care contactele pot fi formate), și prin amenajarea de contact
layout pentru a minimiza rezistența la ~0.1
W într-o celulă de rezistență de ~20 W
putere de vârf.
§5.4.11 Shunt resistance (pierderi neglijabile ~0.1%)
Șunt rezistență în paralel cu o rezistență mai mare parte este cauzata de
defecte structurale întreaga suprafață și la marginea celulei. Îmbunătățit
tehnologia a redus aceste pentru un efect neglijabil, astfel încât rezistența șunt
poate fi considerat infinit într-un singur cristal Si celule. Acest lucru nu poate fi atât în
celulele policristaline, cu toate acestea.
§5.4.12 Livrate putere
Pentru 'înaltă eficiență' cristaline Si celule, dupa pierderile enumerate în
secțiunile de mai sus, Tabelul 5.1 estimează procentul de putere de 25% din
incidentul de expunere la soare. Aceasta presupune sarcină optimă de potrivire în plină
expunere la soare, fără supraîncălzire, pentru a produce puterea de vârf pe care I–V
caracteristice. Rețineți că pierderile referitoare la intrinsecă nepotrivire de solar
radiații cu o singură bandă decalaj setați o limită teoretică pentru eficiența
chiar și un "perfect" Si celule de aproximativ
-
×
-

(100 33)% (100 23)% 50%
.
Prin urmare, o modalitate evidentă de creștere a eficienței este de a avea multistrat
TWIDELL PAGINARE.indb 175
01/12/2014 11:36
176
Fotovoltaice tehnologie
celulele, cu fiecare strat potrivite pentru o regiune diferită de solar
spectrum, la fel ca în §5.6.2.
§5.5 CELULE SOLARE ȘI MODULE FABRICAREA
Majoritatea celulelor solare fabricate la nivel mondial este cu celule de siliciu,
deci avem prima schiță construcția de single standard-cristal Si celule și
fabricarea lor în module. Există mai multe variante, și
concurența comercială produce o îmbunătățire continuă de tipul de celulă și
de metode de fabricație. Un general de proiectare Si celule este prezentată în Fig.
5.11, cu schema de module și matrice ansambluri prezentată în Fig. 5.6.
§5.5.1 Generale criterii de proiectare
1
Inițial materialele trebuie să fie de excelenta puritate chimică în concordanță cu
proprietățile.
2
Celula de proiectare ar trebui să îmbunătățească eficiența de generare a electricității.
3
Celulele sunt produse în masă, cu costuri minime; deci, în practică, ele
trebuie să fie mai subțire (mai puțin material) și de suprafață mai mare (mai puține conexiuni
și mai puțin goale modulul de zona), cu manufacturarea rapidă viteză (mai mult
manu factură pe unitate de forță de muncă și cheltuieli de regie) folosind 'robotizate de control
al proceselor și precizie excelentă (de înaltă eficiență celule), adică
mai subțire, mai mare, mai rapid, mai ieftin.
4
Testate și clasificate celulele sunt interconectate și apoi încapsulate ca
module.
5
Design-ul trebuie să permită unele defecte să apară fără eșec de
sistem complet. Astfel redundante contactele electrice sunt utile și
module pot fi conectate în paralel siruri de caractere, astfel încât dacă un șir nu reușește,
există încă o generație.
6
Modulele sunt de obicei garantat pentru cel puțin 20 de ani. Design
special pentru potențiale daune de transport și de pe site-ul de
construcții de clădiri, și de expunere în medii ostile, cu
modificări semnificative de temperatură (chiar și fără concentrare solară,
celula temperatura poate varia între -30 și +100°C). Electrice
de contact trebuie să supraviețuiască și toate formele de coroziune evitate, în special
apa nu trebuie să intre în modul.
Caseta 5.3 oferă o descriere mai detaliată a procesului de producție.
CASETA 5.3 FABRICAREA DE SILICIU CRISTALIN CELULE ȘI MODULE
Pasul 1:
Materii prime pentru policristaline lingouri
"Pure" SiO
2
nisipul este redus la metalurgice clasa Si (~puritate 98,5%, respectiv <1.5% impurități) în cocs (carbon)
cuptoare, și apoi purificat în continuare în nici scump electronic-clasa a Si (<10
-7
% impurități) sau mai ieftin
solar-clasa a Si (<10
-3
% impurități). Deșeurile electronice de calitate Si este folosit pentru PV fabricarea, dar limitat
TWIDELL PAGINARE.indb 176
01/12/2014 11:36
§5.5 celule Solare și module fabricarea
177
disponibilitatea a condus la industria solară produce propriul material de bază la fel de mare policristaline lingouri (de exemplu,
de ~1 m x 1 m x 0,5 m dimensiuni: Fig. 5.16). După ce a obținut un material pur, topit în lingouri pot
fi măsurate cantități mici de trivalent (de exemplu, bor) sau pentavalent (de exemplu, fosfor) elemente de adăugat pentru a
face, respectiv p sau de tip n material de bază.
Pasul 2:
Creșterea cristalelor
În policristaline Si sunt mici cristale unice de mm dimensiune. Acestea ar putea fi eliminate pentru a deveni 'semințe de
cristale' pentru a forma cristale mari. Metoda standard este metoda Czochralski, dar alte metode sunt
, de asemenea, utilizate.
o
Tehnica Czochralski mari cristale unice. Mici de semințe de cristal este fixat la capătul de jos
o tijă detașabilă și muiată în topită electronice - sau solar-material de calitate (Fig. 5.16(a)). Dopant este
adăugat pentru a topi dacă nu este prezent anterior. Încet cristal este mecanic tras în sus în afară de
topi, acum, cu o mare de cristal cilindric (la ~15 cm diametru), în creștere de la semințe. Acest cristal este
apoi se taie fie în (i) napolitane subtiri, care sunt folosite direct pentru a face celule FOTOVOLTAICE individuale, sau (ii) mai
multe semințe de
cristale de producție în paralel a mari cristale unice în metru scară topit în lingouri.
b
Zona de rafinare. Policristaline material este format ca un rod. Un topit zona este trecut de-a lungul tijei
de încălzire cu o frecvență radio bobina sau cu lasere (Fig. 5.16(b)). Acest proces atât purifică
material și formează un singur cristal, care poate fi folosit ca o sămânță de cristal sau felii de celule ca și pentru alte
tehnici.
c
Panglică de creștere Această metodă evită feliere și, în consecință, deșeuri de creștere continuă subțire
benzi de singur cristal de până la 10 cm lățime și 300 μm grosime, așa cum se arată în Fig. 5.16(c).
Pasul 3:
Cristal lingouri taie în plachete,
lingouri sunt tăiate în ~300 μm-gros plachete de una sau mai multe operațiuni cu extrem de precise diamond
saws. Poate ~40% din material cristalin care poate fi pierdut în timpul acestui proces, care reprezintă o gravă
pierdere.
Topit Si
Topit si
Singur cristal
Policristaline
Topit zona
Frecvență Radio
Mare singur
cristal
Încet clemă rotativă
tras în sus
Original semințe de cristal
Single-cristal panglică
Marginea definirea plăci
(a)
(c)
(b)
Fig. 5.16
Un cristal metode de creștere:
o
Czochralski;
b
zona de recristalizare sau cu laser încălzire;
c
panglică.
TWIDELL PAGINARE.indb 177
01/12/2014 11:36
178
Fotovoltaice tehnologie
Tedlar
EVA
(b)
De înaltă puritate silicon
(a)
Napolitana Procesul De Producție
Lingou de creștere
Lingou
Caramida feliere
Lingou cuadratura
100% napolitana inspecție
Fig. 5.17
Etape în fabricarea de module solare.
o
Napolitana producția de lingouri mari în continuă producției în fabrică. După
automatizate aderarea la celulele, modulele sunt de obicei atent asamblat manual.
b
Structura unui modul PV, arată celulelor încapsulate în straturi de etilen
vinil acetat (EVA), cu exterior partea de sus pentru geamurile din spate și sprijin structural. Marginea
de legare sau cadru (nu este prezentat) este garantat pentru a preveni umezeala, vapori și
gaze de intrare pentru cel puțin 20 de ani în toate climatele.
TWIDELL PAGINARE.indb 178
01/12/2014 11:36

§5.6 Tipuri și adaptări ale fotovoltaice


179
§5.6 TIPURI ȘI ADAPTĂRI ALE FOTOVOLTAICE
Deși plate-placă Si celule solare a fost dominantă comercială
a produsului, există o mare varietate de tipuri alternative și construcții.
Acestea au scopul de a îmbunătăți eficiența și/sau pentru a reduce costul
energiei electrice produse prin reducerea costurilor de capital. Această secțiune rezumă un
complex și în continuă schimbare de scenă.
Un mod util de a clasifica diferitele tipuri de celule este în prima, a doua
și a treia generație (Fig. 5.18). "Prima generație" de celule sunt cele bazate
pe cristalin Si un singur nod de celule, așa cum este descris în §5.4 și §5.5; aceste
domina curent instalații. Costurile de producție sub 1$/watt
sunt fezabile de reducere pe unitate de cost de fabricație cu scară mai mare
a producției și îmbunătățirea eficienței către un singur nod final
limită de aproximativ 31%. Această limită (prezentate în Caseta 5.2, §5.4.2) depinde de
material semiconductor și trupa sa decalaj, și este numit
ShockleyQueisser limită (a se vedea, de asemenea, §R4.3).
"A doua generație" celulele folosesc peliculă subțire de celule tehnologie pentru o singură joncțiune
celule bazate pe depunerea de straturi subțiri de materiale fotoactive pe
sprijinirea substraturi, sau superstrates, care sunt, de obicei, foi de
Pasul 4:
Felie de tratament și de dopaj
200
mm - 400 mm-grosime plachetele sunt gravate chimic. Un strat foarte subțire de n-tip material este
format prin difuzie de donatori (de exemplu, fosfor) în suprafață de top. O metodă este de a încălzi felii la
1000°C într-o cameră de vid, în care este trecut P
2
O
5
, dar cel mai adesea felii sunt încălzite în azot
cu adaos de Poch
3

. Photolithographic metode pot fi folosite pentru a forma rețeaua de contacte electrice.


Prima, Ti-ar putea fi depozitate, pentru a forma o rezistență scăzută contact cu Si; în al doilea rând, un foarte subțire Pd strat
pentru a
preveni o reacție chimică de Ti cu Ag; și a treia, finală Ag depozit pentru a transporta curent de grilă. Alte
metode depinde de ecran de imprimare și de galvanizare.
Straturile antireflex sunt atent depuse de vid tehnici sau proprietăți similare de texturate
suprafețe sunt produse doar de gravură chimică. Suprafața din spate poate fi difuză cu Al pentru a face o
suprafata de teren de p
+

pe p (a se vedea §5.4.6). Pe aceasta este prevăzută spate electrice de contact de metal ca un relativ
gros de ansamblu strat.
Pasul 5:
Module și matrice
De celule individuale, de dimensiuni ~ (10 cm x 10 cm), sunt apoi conectate și montate în module (Fig. 5.17).
În mod tradițional, cele mai multe module avut aproximativ 36 de celule în serie pentru a furniza o tensiune de încărcare nominal
12 V baterii. Dar multe mai târziu tipuri de module au un număr mai mare de celule în serie de mari tensiuni
mai compatibil cu eficienta invertoare pentru AC grid-connected systems.
Celulele sunt comasate într-un material de umplutură inert între clară față acoperă, de obicei, plastic rezistent la ultraviolete,
și o placă de susținere (Fig 5.17(b)). Încapsularea într-un cadru trebuie să fie etanșe în toate condițiile,
inclusiv de stres termic. Placa din spate trebuie să fie puternic și încă o mică rezistență termică de
răcire. Placa din față este de obicei monostrat (temperat) fier-free pahar de excelenta transmisie. De obicei,
modulele produce curent continuu, dar unii producători pot include grid-tie mini-invertoare în cadrul fiecărui
modul pentru conexiune imediată într-o tensiune de alimentare de rețea.
TWIDELL PAGINARE.indb 179
01/12/2014 11:36
180
Fotovoltaice tehnologie
de sticlă. Această metodă folosește mult mai puțin din cel mai scump material
(semiconductor), astfel încât, în ciuda celule care au o eficiență limitată, film subțire
de celule și module sunt mai ieftine pe unitate de capacitate ($/Watt), după cum este indicat în
Fig. 5.18. Semiconductoare pot fi amorfe Si sau una din celelalte
materiale prezentate în Tabelul 5.2 și discutate mai târziu în această secțiune.
A treia generație de celule nu sunt limitate la un singur nod operare;
de exemplu, acestea includ multijunction/heterojunction tandem celule
concepute pentru a absorbi o gamă mai largă din spectrul solar decât
singlejunction celule și astfel au potențialul de eficiență >30%, fără solare
de concentrare, și >40% cu concentrația (a se vedea §5.6.2). Presupunând că
tehnologia thin film cu costurile de producție pe unitatea de suprafață asemănătoare cu
a doua generație de celule, dar au o eficiență mai mare, proiectat
scăderea costurilor suplimentare (Fig. 5.18). Concepte pentru alte a treia generație de
celule includ intermediar trupa celule, multi-excitonul generație de celule și
fierbinte purtător de celule; aceste subiecte sunt discutate în publicațiile de specialitate.
Tabelul 5.2 liste ambele variante în Si celule solare și unele dintre celelalte
tipuri descrise în această secțiune, împreună cu unele dintre parametrii cheie
și eficiență atins.
§5.6.1 Variații în Si material
1
Singur cristal. Celulele descrise până acum-și asume singur cristal
(omogene) material de bază obținute prin metodele prezentate în §5.5,
mai ales scalate-up Czochraski procese. Resturi de cele mai bune calitate Si
microelectronica materiale sunt disponibile relativ ieftin, dar
Fig. 5.18
Estimările privind costurile și eficiența a trei generații de celule solare: (I) "Prima generație" –
singur-joncțiune celulele cristaline Si. (II) "a Doua generație" – film subțire singură joncțiune
celulele Si sau alte semiconductori. (III) a Treia generație de celule cu o mai mare eficiență
(de exemplu, folosind "stive" de mai multe semiconductori).
TWIDELL PAGINARE.indb 180
01/12/2014 11:36

Tabelul 5.2
Exemple de celule solare și parametru de bază
s, condiții standard* (appro
ximate de date dintr-o varietate de surse; acolo
este un echilibru impro
face bine în cel mai ef
ficiency cu experiență de cercetare
h și experiență de fabricație)
Material de bază
înainte de dopaj
Band gap†
E
g
/ eV
V
oc
/
volt
Grup în
Periodice
Tabelul
Directă (D)
sau
Indirecte (I)
foton
absorbția
Exemplu de celulă
Eficiența
cel mai bun
comerciale
celulele ‡
(c.2013)
Eficiența
de
cel mai bun
laborator
celulele ‡
(c.2013)
Ge
0.67
-
IV
Am
Din punct de vedere chimic active, de aceea sunt folosite numai în
multistrat celule
-
-
Si (singur cristal)
1.1
0.71
IV
Am
Semnificative comerț
≈ 17 %
≈ 25%
Si (multi-cristal)
1.1
0.66
IV
Am
Mai ieftin comercial mobil
≈ 13%
≈ 20%
Si (nanocrystal) (thin-film)
0.54
IV
În dezvoltarea
-
≈ 10%
Si (amorfe) (thin-film)
1.1
0.89
IV
Comerciale ca film subtire sau panglică
≈ 7%
≈ 12%
a-Si/ nc-Si (multistrat)
1.4
IV
În dezvoltarea
-
≈ 12%
GaAs (thin-film)
1.4
III-V
D
p/n
≈ 28 %
CdTe (thin-film)
1.5
1.4
II-VI
D
În practică, multistrat cu Cd-uri
≈ 8%
» 17%
Cd-uri
2.4
II-VI
D
Folosit numai în celule multistrat
-
-
GaInP/cu Arsenură/Ge (thin-film)
2.7
(III-V)/IV
D
Multistrat cu GaAs bază
n/a
≈ 18 %
Cu(InGa)Se
2
(CIGS) (film subțire) #
0.71
(I/III)-VI
D
Multistrat
≈ 15%
≈ 20 %
GaInP/GaAs
III-V
Multistrat, concentrator. NREL
record 25/6/13
31.1%
'Se diluează nitrură de proprietar
Multistrat, concentrator: IQE și
Solar Intersecția Corpului 25/8/13
44.1%
Note
* AM1 pentru band gap, etc. Optimă a benzii în AM1 radiații este cuprinsă între 1.4 și 1.5 eV (Fig. R4.12).

Date aici pentru temperatura mediului ambiant (~25
°C). Benzii interzise scade cu creșterea temperaturii (de exemplu Si 1.14 eV (30°C), 1.09 eV (130°C)). Notă: în circuit deschis
tensiune
V
oc
o celulă este întotdeauna semnificativ mai mică decât banda interzisă de bază (nedopat) material.
#
Compoziția este, de fapt, Cu(În
1–x

Ga
x
)Se
2

, cu banda de decalaj (de la 1.1 V la 1.6 eV) și eficiența în funcție de x. Cel mai comercial CIGS celulele au x

0.3,
De exemplu
≈ 1.2
V.

Eficiența măsurată ca energie electrică de ieșire împărțit de radiație solară pe mobil, în condiții standard (1000 Wm
-2

, 25
°C, AM1.5 spectru).
TWIDELL PAGINARE.indb 181
01/12/2014 11:36
182
Fotovoltaice tehnologie
cantitățile sunt suficiente pentru moderne industriei FOTOVOLTAICE, care
din ce în ce mai produce propriul material de bază. Acesta din urmă trebuie să fie mai puțin pur
decât pentru microelectronică și poate fi taiat din metru-scară lingouri cu
multi-seminte de segmente din care atât de singur și policristaline
plachetele pot fi obținute.
2
Amestecat cristaline (neregulate juxtapunere de un singur cristal de cereale într -
un solid). Industriei FOTOVOLTAICE utilizează o serie de Si material, descris de
creștere cristal dimensiune de cereale ca: microcristalină <~1 mm,
policristaline <~1 mm, policristaline <~3 cm, și singur cristal de o
mare de cereale. Cu toate acestea, frecvent cuvântul 'policristaline' include
toate formele, altele decât singur cristalină. Astfel policristaline material este
mai ieftin și mai ușor pentru a obține decât un singur cristale și nu este neapărat
din punct de vedere structural slab. Cu toate acestea, fotovoltaice curenți sunt reduse atunci când
perechilor electron-gol se recombina pe plan intern, la cereale limite, astfel încât
reducerea eficienței globale. Având tipic granulație dimensiune
cel puțin egală cu grosimea stratului de celule, devine puțin probabil ca
actualul traversează o limita de cereale, deci nu este o pierdere de eficiență.
Prin urmare, mai subtire celulele sunt mai ieftin de a avea mai puțin material și poate fi
proiectat pentru o eficiență îmbunătățită.
Rețineți că controlate de creștere a cristalelor la micron (
mm) scara este un aspect
de nanotehnologie, așa cum ar microcristalină celule poate fi numit
nanocristalin.
3
Amorf. Materialelor amorfe sunt solide cu rază scurtă de ordinul a
numai un număr relativ mic de atomi și, prin urmare, nu sunt cristaline (de exemplu solid
de sticlă). Siliciu amorf (
α-Si) poate fi produsă de film subțire
de depunere cu Si vapori de metode de depunere și își păstrează poziția de bază
tetraedrice proprietăți semiconductoare; în special n și de tip p dopanți
permite fotovoltaice intersecții să fie format ca și în material cristalin.
Cu toate acestea, structura amorfa produce o proporție foarte mare
de neatașat 'suspendati' legăturile chimice care capcana de electroni și găuri
curent transportatorii, reducând astfel drastic fotovoltaice eficiență.
Pentru a contracara acest lucru, material amorf este format inițial într-o
atmosferă de silan (SiH
4

), astfel încât atomii de hidrogen se lega chimic


de la anterior neatașat site-uri, astfel reducand numarul de
electron-gol capcane. Amorf Si este folosit în film subțire de celule solare de low
- cost cu o grosime totală de semiconductor de aproximativ 1 μm (adică ~1/100 de
grosimea unui convenționale singur cristal de celule). Trupa decalaj de
α-Si
este de 1,7 eV, comparativ cu cristalin Si de 1,1 eV, care este mai
potrivit să spectrul solar (a se vedea Fig. R4.12). De dezvoltare cu mai multe
intersecții în care 1 μm are o eficiență crescută la aproximativ 10%.
O dificultate practică poate fi redus eficiența cu vârsta, mai ales în
primii ani de funcționare. Un avantaj este că producția de
α-Si
celulele nu se modifică semnificativ cu o creștere de temperatură.
TWIDELL PAGINARE.indb 182
01/12/2014 11:36
§5.6 Tipuri și adaptări ale fotovoltaice
183
§5.6.2 Variații în intersecția geometrie
(a) o Singură joncțiune (homojunction)
Dacă baza material semiconductor rămâne aceeași peste p–n
intersecție, și singurele modificări sunt în tip sau concentrația de dopant, este
un homojunction. Si celulele discutat până acum sunt singur astfel de intersecții.
Trupa diferența este constantă de-a lungul joncțiune (Fig. 5.19(a)).
(b) Heterojunction (multistrat, tandem, etc.)
Dacă materialul de bază se schimbă cu adâncimea, de exemplu, de creștere a straturilor
de un cristalin semiconductor pe un alt cristalin
semiconductoare, trupa decalaj de intersecția schimbări cu adâncime (de exemplu, așa cum se arată în
Fig. 5.19(b)).Avantajul este că foton de absorbție la trupa diferența este de la
două sau mai multe frecvențe. Acest lucru crește proporția totală de fotoni
care pot fi absorbite, și deci scade excesul de foton pierderea de energie
(h
n – E
g

). În mod normal, mai larg decalaj banda de material pe suprafața superioară, astfel încât
cei mai puțin energici (neabsorbite) de fotoni continua pentru absorbție în
bandă mai îngustă, diferența de material. Multistrat celulele sunt un tip de 'a treia
generație de celule.
Alternativ, o continuă scădere band gap cu profunzime (
clasificate benzii de celule) este posibil, dar dificil de fabricație (de exemplu
Ga
1– x
Al
x
Ca, în cazul în care x se schimbă cu adâncimea de la 1,0 (cu E
g
= 2.2 eV),
la 0.0 (cu E
g
= 1.4 eV). Pentru acest material, curentul de scurt-circuit este
Fig. 5.19
Nivelul de energie de diferite celule solare cu joncțiune tipuri: (a) Homojunction: materialul de bază și
trupa diferența constantă de-a lungul joncțiunii. (b) Heterojunction: materialul de bază și band gap schimba
peste joncțiune.
(a) Homojunction
(b) Heterojunction
p
p
n
Contact
n
Contact
Contact
Contact
E
g
E
g1
E
g2
E
g
TWIDELL PAGINARE.indb 183
01/12/2014 11:36
184
Fotovoltaice tehnologie
relativ mare, deoarece fotonii sunt absorbiți în mod eficient, dar
opencircuit tensiune este relativ mic din cauza la cel mai mic profunzime mici benzii interzise.
(c) celule cu strat Subtire (sau peliculă subțire de celule)
Acesta este un termen generalizat pentru celulele de ~20 µm grosime, mai degrabă decât la ~200
m m grosimea standard Si celule de cristal. Exemple de strat subțire de celule
amorfe si și CIGS (a se vedea §5.6.3). De obicei, film subțire de activ
material este depus pe un substrat de sticlă sau alte materiale pentru a oferi
sprijin mecanic. În practică multistrat celulele sunt de obicei subțiri, în mod semnificativ cu
cantități reduse de material scump.
(d) Directe și indirecte band gap
Semiconductori se comportă pe plan intern în diferite moduri. În special indirecte
benzii de material (de exemplu, Si) are o mai mică dispariție (absorbție optică)
coeficient decât directă a benzii de material (de exemplu GaAs), deci necesită mai gros
de celule (a se vedea §R4.2 și dispozitivul de texte pentru explicații suplimentare).
§5.6.3 Alte materiale de substrat; chimice grupele III/V
și II/VI
Siliciul este un element din Grupa a IV-a a Tabelului Periodic, ceea ce înseamnă că
fiecare atom are patru electroni în învelișul exterior. În general, atomii formează
un grajd carcasă exterioară de opt electroni prin schimbul de electroni – lipire –
cu alți atomi. Legare covalentă cu patru mai apropiat vecin atomi
într-o configurație tetraedrică forme de cooperare stabil exterior
scoici în siliciu, germaniu (care este, de asemenea, un semiconductor), și
Fig. 5.20
La SJ3 NREL/solar intersecția cu mai multe straturi de celule are trei straturi semiconductoare cu
succesiv mai mici band gap (InGaP 1.9 eV, GaAs 1.4 eV, GaInNAs 1.0 eV). Acesta are o
eficiență de 43.8% la 418 soare concentrată expunere la soare.
Mobil de Top
Contact
Anti–reflexie
Tunnel junction
Tunnel junction
Mijloc de celule
De jos de celule
InGaP
1.9 eV
GaAs
1.4 eV
GalnNAs
1.0 eV
substrat de GaAs
Contact
TWIDELL PAGINARE.indb 184
01/12/2014 11:36
§5.6 Tipuri și adaptări ale fotovoltaice
185
de carbon (diamant). O altă consecință este că Si formelor tetraedrice
cristale într-un corp centrat cubic lattice, cu fiecare atom în centrul
unui cub cu patru mai apropiat vecini (a se vedea Fig. R4.13). Acest
tetraedrice structura apare, de asemenea, în anumite doi-element (binar) materiale din
Grupele III și V (de exemplu, arseniură de galiu, GaAs) și de Grupuri a II-a și a VI-a
(de exemplu cadmiu telluride CdTe), și în trei-element (ternar)
materiale (de exemplu, de Grupuri (I/III)/VI, cum ar fi CuInSe
2

) în cazul în care legare covalentă


permite, de asemenea, opt comun de electroni în învelișul electronic. Mult mai complexe, dar
'reglabil' compus materialele utilizate ca materiale fotovoltaice sunt
Ga
x
În
1–x
Ca
y
P
1– y
și Senzuală
x
Ga
1–x
Se
2
(CIGS), unde x și y variază între
unu și zero.
Toți acești compuși sunt, de asemenea, semiconductori, cu un cristal
cu structură și formație de muzică electronică structură comparabilă cu Si (a se vedea §R4.4). Astfel de
'look-alike' tetraedrice compus semiconductori pot fi "adaptate" pentru
trupa dorit structură, proprietăți, folosind disponibile și acceptabile
elemente (a se vedea Tabelul 5.2 pentru exemple).
CASETA 5.4 UN EXEMPLU DE SOFISTICAT Si CELULE SOLARE
Fig. 5.21
PERL celulă (pasivizat emițător, spate local difuză).
degetul
oxid de
silicon
spate de contact
oxid de
p
+
p
+
p

n
+
n
'inversat' piramide
p
+
p
+

Acest tip de celule, dezvoltat la Universitatea din New South Wales, este una dintre cele mai eficiente folosind
cristalin Si, cu un randament de 24%. Celulele de acest lucru și structuri similare au fost făcute în
semicommercial cantități pentru aplicații specializate. Sale complicate structura ilustrează complexitatea și
indică costul de realizare a unei astfel de înaltă eficiență. Acesta dispune de detaliate atenție pentru a maximiza
absorbția luminii de către atent fabricarea de o texturate de sus a suprafeței în formă de piramide inversate cu
lățime de ~10 μm. Stratul de oxid de la partea din spate reflectă de cele mai multe rămase neabsorbite lumina înapoi în
celulă, sporind astfel și mai mult absorbția, ca are un strat anti-reflexie (a se vedea §5.4.2). În plus,
oxid de straturi în partea de sus și de jos 'pasiva' transportatorilor, și anume de a reduce ratele de recombinare la aceste suprafețe
cu un minim de dopaj. Contacte electrice folosesc laserul grove tehnica, care crește zona de contact,
pentru o rezistenta scazuta, dar nu reduce diafragma este zona de top.
TWIDELL PAGINARE.indb 185
01/12/2014 11:36
186
Fotovoltaice tehnologie
§5.6.4 Alte semiconductoare mecanisme, clasificări
și terminologii
Până acum am considerat PV generație de semiconductori cu
tetraedrice structura (de exemplu, si și GaAs), deoarece acestea sunt cele mai
comune PV materiale. Cu toate acestea, există și alte sisteme și
configurații. Exemple sunt după cum urmează.
(o) 'PV termic' colectoare
Acest nume este utilizat pentru construcții, care combină de energie electrică PV
generație cu producerea de căldură (de exemplu, apă caldă). Ar trebui avantaje
includ: (i) PV de eficiență este crescut în cazul în care PV materialul este răcit; (ii)
o mai bună utilizare este fabricat din zona de colectare; și (iii) de construcție și
costurile de instalare sunt mai mici decât echivalentul sisteme separate. Cu toate acestea,
în ciuda acestor avantaje, sisteme mixte de acest fel sunt neobișnuite. În
practică, bine stabilit SĂRUT principiu funcționează (păstrați-l simplu,
prost).
(b) Organice fotovoltaice (OPV)s
Aceasta este comun pentru lumina pentru a fi absorbit în anumiți compuși organici deci
producerea separată de electroni și goluri la fel de incantati membre de
structura moleculară, dar asociat destul de aproape pentru a forma un legat stat ca
'excitons'. Structura moleculară are adesea o dimensiune de un relativ puține
repetate molecule, adică de un oligomer, spre deosebire de un polimer. Astfel de
procese și oligomerii sunt baza de fotosinteza (Capitolul 9). De
esența unei OPV dispozitiv este de a permite electroni și găuri de excitonul
să fie separate și să treacă la un circuit extern. Acest lucru necesită două straturi de
diferite efectuarea materiale care au un câmp electric intrinsec între
ele, adică o tensiune. Un exemplu timpuriu este un strat de oxid de indiu și un
strat de low-muncă-funcția de metal (de exemplu, Al), cu materiale organice între
aceste straturi (de exemplu macromoleculare colorant compus ftalocianină).
Vastele cunoștințe de chimie organică și posibil
ieftinătate de materiale organice face evoluții în acest domeniu de mare
interes. Eficiență de 10% au fost atinse (Green et al. 2012).
(c) Quantum-dot dispozitive
Punctele cuantice sunt nanocristale semiconductoare (de exemplu, Si, de cu diametrul de aproximativ
5
mm (5 x 10
-6

m). Solară absorbită de fotoni creați unul sau mai mulți electroni –


găuri perechi ('excitons') în nanocrystal care sunt 'quantum limitat și
doar posibilitatea de a se recombina cu emisie de fotoni cu lungimea de undă
definite de nanocrystal dimensiune. Prin urmare, puncte cuantice de aceeasi
dimensiune toate luminesce la aceeași frecvență. Luminescența apare atunci când
solare, fotonii sunt absorbite, ceea ce duce la emisia de unul sau mai mulți
fotoni cu mai puțin discrete de energie cel mai mare lungime de undă. Prin
care conțin materiale luminescente într-un strat subțire de sticlă "rezervor", cele mai multe dintre emise
TWIDELL PAGINARE.indb 186
01/12/2014 11:36
§5.6 Tipuri și adaptări ale fotovoltaice
187
fotoni poate fi pe plan intern reflectat pe un perete de capăt acoperit de un PV
de celule. Sistemul, prin urmare, devine un statice fotovoltaice concentrator
de directe și difuze expunere la soare. Dispozitivul de eficiență este potențial
mai mare decât homojunction semiconductori (de exemplu, un singur Si strat), cu
posibilitatea de a crescut producția de energie electrică pe unitatea de suprafata de colector și
de costul mai ieftin per unitate de energie electrică produsă.
(d) Colorare de celule sensibile (photoelectrochemical celule Grätzel)
Această formă de celule solare seamănă cu fotosinteza în funcționarea acestuia. Mai degrabă
decât a soarelui fiind absorbit într-un semiconductor, celula absoarbe
lumina în moleculele de colorant conține ruteniu ioni. Coloranți sunt distincte în
absoarbe lumina la lungimi de undă diferite. Astfel moleculele de colorant sunt acoperite
pe întreaga afara suprafețelor de nanocristale de o bandă interzisă largă
semiconductor, de obicei TiO
2

, așa cum se arată în Fig. 5.22. Mecanismul


de absorbție de fotoni și ulterior de electroni 'excitonul de transfer de la un
'centru de prelucrare' seamănă cu procesul fotosintetic (a se vedea Capitolul
9 și Fig. 9.6). Lumina foton de absorbție prin soare se confruntă cu suprafata
de transparente conductoare de oxid (TCO) excită electronii în vopsea pentru a o
energie în cazul în care acestea sunt injectate în banda de conducție adiacente
Fig. 5.22
Un colorant sensibil la celule solare. Vopseaua acoperă suprafețele TiO
2

nanocristale. TCO:
transparente conductoare de oxid.
electrolit
sticla
sticla
lumina
Pt
TCO–acoperire
TCO acoperire
vopsea pe TiO
2
nanocrystal
TWIDELL PAGINARE.indb 187
01/12/2014 11:36
188
Fotovoltaice tehnologie
n-tip TiO
2

și de acolo la suprafața frontală și circuitul exterior. La


curentul de electroni trece prin sarcină externă la spate electrod,
unde reduce tri-iodură de la iod, care apoi difuzează prin
electrolit pentru a reduce foto-oxidat moleculele de colorant înapoi la
starea inițială. Eficiență de 11% au fost realizate în
laborator (Green et al. 2012). Astfel de tehnologii, dar folosind infraroșu de absorbție a
coloranților, au potențialul de a produce 'vizual transparent' module care
ar fi de mare interes comercial ca generatoare de electricitate windows
în clădiri. Procese similare bazate pe lichide oferi perspectiva de
scară largă și relativ ieftine producția de masă.
(e) tranziții Intermediare (fosforescente)
În principiu, suprafața frontală a unei celule fotovoltaice ar putea fi acoperite cu
un fluorescente sau fosforescente strat pentru a absorbi fotoni de energie
semnificativ mai mare decât banda interzisă (h
n
1
>> E
g
). Cu toate acestea, emise
fotonii ar trebui să fie absorbit în mod activ (h
n
2
≤ E
g
). Astfel
excesul de energie original fotoni (h
n
1
- hn
2

) ar fi disipată în
suprafață, sperăm, cu mai puțin creșterea temperaturii din celulă. Alte,
idei similare au fost luate în considerare, fie pentru a elibera două active fotoni
de fiecare originale foton, sau de a absorbi două inactiv fotoni (h
n < E
g

) pentru a
produce un activ foton într-o manieră ce amintește de fotosinteză.
(f) Verticale multijunction celule (VMJs)
Celulele sunt formate astfel încât lumina intră la margini (Fig. 5.23):
am
Seria legate. Aproximativ 100 similare p–n intersecții sunt realizate într-o grămadă
(Fig. 5.23(a)). Lumină este incident pe margini, deci relativ mare
potențial de ieșire (~50 V) este suma dintre numeroasele intersecții în serie.
Curentul este legată de expunere la soare doar pe marginea zone, și deci
nu este mare.
ii
Paralel legate. Aceasta este o formă de grilaj mobil, de obicei, realizate cu
scopul de a absorbi fotoni mai eficient în zona de joncțiune
(Fig. 5.23(b)).
Fig. 5.23
Verticale multi-joncțiune celule (VMJs): (a) legate în serie; (b) legate în paralel.
Contactele metalice
Metal de contact pentru a p
(a)
(b)
p
p
p
p
p
p
p
n
n
n
n
n
n
p
p
p
n
n
n
n
Metal ohmice
de contact la n
100
µ
m
10
µ
m
300
µ
m
TWIDELL PAGINARE.indb 188
01/12/2014 11:36
§5.6 Tipuri și adaptări ale fotovoltaice
189
(g) Termo-fotovoltaice
Aceste dispozitive produc energie electrică după absorbția de longwave
radiații infraroșii și din surse de la, să zicem, aproximativ 1000 de
°C. Mic decalaj banda
semiconductoare sunt utilizate (de exemplu, GaSb cu 0.7 eV band gap), care sunt
în mare parte în cadrul laboratorului de dezvoltare. Utilizări posibile includ generarea
de energie electrică de altfel deșeuri de căldură (de exemplu, la turnătorii de metale). O altă
opțiune este de a concentra radiația solară pe o neagră suprafață de absorbție,
care apoi re-radiaza de la un termo-fotovoltaice dispozitiv. Eficient
de vârf frecvența radiației solare concentrate este mutat în
infraroșu pentru a obține o mai bună potrivire cu un mic decalaj trupa fotovoltaice
celule.
(h) Nanotehnologie
Ca cu solid-state dispozitive electronice, PV procese depinde atomică
și moleculară scară procese, la o scară corespunzătoare de aproximativ 1 la
100 de nanometri (10
-9
la 10
-7

m) . Materialele pot fi "văzut" la această scară


de microscoape electronice; în special, suprafețele și straturile de suprafață pot fi
investigate la scară atomică, folosind o serie de electronic cu scanare
microscoape. Astfel de instrumente au facilitat foarte precis 'inginerie' de PV
dispozitive (de exemplu, depunerea de straturi semiconductoare și contactele de pe lângă
scară atomică ca anti-reflexie de suprafață straturi). Procesele de fabricație
la o astfel de precizie poate fi operat precisă replicat producția de
milioane de produse (de exemplu, producerea pe scară largă a nano-dispozitivelor la scară).
(i) Apa de divizare pentru hidrogen și oxigen de producție
Activ de cercetare în photoelectrochemistry încearcă să se folosească de lumină solară
pentru a produce comerciale de hidrogen de la "directa" apă-divizarea proceselor.
Un exemplu este o comună scară nanometrică structura de hematit (Fe
2
O
3

) cu un
colorant sensibil fotovoltaice strat atasat (un 'hematit photoelectrode'),
care în efect produce o tensiune suficientă pentru a electrolyze apă în
anii 'tandem' structura.
§5.6.5 Variație în sistemul de aranjament
(a) Concentratoare (vezi Fig. 5.24 și §4.8)
Beneficiile de concentrare a radiației solare pe celulele fotovoltaice sunt:
(i) mai puține celule sunt necesare, prin urmare, reducerea costurilor pe unitatea de putere
generate; (ii) prin urmare, celulele care sunt utilizate pot fi cel mai bun disponibil (posibil
să fie mai multe straturi de celule cu, probabil, 40% eficiență); (iii) mai puțin zona de site
este necesar; (iv) total cadre și poate reduce costurile de construcție.
Dezavantajele sunt: (i) perioade lungi de cer senin sunt esențiale; (ii) să urmeze
Soarele, concentrator este costisitoare și necesită întreținere; (iii)
eficiența celulelor este redusă la temperatură crescută, atât de activ sau pasiv
este nevoie de răcire (cu toate acestea, căldură eliminată în active de răcire poate fi
util).
TWIDELL PAGINARE.indb 189
01/12/2014 11:36
190
Fotovoltaice tehnologie
La concentrația X este raportul dintre concentrator de intrare
diafragma la suprafața celulei; concentrația efectivă este de obicei de aproximativ
90% din aceasta. Sisteme cu X
≤ 5 nu urmări, de obicei, Soarele prin
a doua zi, dar poate fi ajustat lunar; ele absorb direct și unele
radiații difuze. Cu X
> 5, Soarele de urmărire este de obicei, dar numai sensibil în
regiunile cu o proporție mare (>70%) de radiație directă. Concentratoare
sunt bazate pe lentile (de obicei Fresnel plat avion lentile), oglinzi și,
ocazional, alte metode (de exemplu, reflecție internă: Fig 5.24(d)).
Cu concentrate insolația celule PV este mic în comparație cu
concentrarea structura; prin urmare, cel mai bine este de a utiliza cele mai eficiente,
și, prin urmare, mai scumpe de celule (vezi Tabelul 5.3). Impresia că
utilizarea de concentrate de expunere la soare îmbunătățește eficiența în sine este oarecum
fals, deoarece scump celulele folosite sunt la fel de eficiente în "obișnuită"
insolație.
(b) împărțirea Spectrală
Separate de celule solare cu creșterea benzii poate fi pus de-a lungul un solar
spectrum (să zicem, de o prismă, și variind de la infraroșii la ultraviolete)
pentru a obține o mai bună frecvență de potrivire. Ca cu multistrat celule,
Fig. 5.24
Un concentrator de sisteme. Atenție: extrem de inegale iluminare de celule sau module
poate cauza deteriorarea celulelor: (a) Compound parabolic concentrator: poate fi construit ca un
bloc solid din plastic transparent. (b) Partea reflectoare. (c) lentila Fresnel. (d) Quantum-dot
de asamblare, arătând quantum-dot nanocristale încorporat într-un mediu transparent;
capacul superior transmite expunere la soare și laterali reflecta intern secundar
luminescente radiații pe final de perete celulă PV; coeficientul de concentrare este raportul dintre
partea de sus suprafață pentru suprafața de celule PV.
(a)
Celula sau celulele
Curbură echivalente
lentile convexe
(c)
(b)
Module
Expunere la soare
Marginea
oglinzi
(d)
Închise
colector
Quantum dot
nanocristale
Celule PV primește
secundar luminescente
pentru quantum dot
placilor
TWIDELL PAGINARE.indb 190
01/12/2014 11:36
§5.7 Sociale, economice și de mediu
191
dominantă pierderile din nepotrivire de fotoni de energie și band gap într-un
singur nod mobil poate fi, prin urmare, a scăzut foarte mult. Spectral divizarea
poate include, de asemenea, concentratoare. Final eficiență de ~40% au fost
obținute în procesul sisteme.
§5.7 SOCIALE, ECONOMICE ȘI DE MEDIU
ASPECTE
§5.7.1 Preturi
Tehnologia și comerciale aplicații de energie fotovoltaică
a crescut rapid începând cu anii 1980, când ex-fabrică costurile au fost inițial
~$US40/W, dar până în 2013 s-a redus la ~$US1/W. Atât reducerea
costurilor și de creștere a capacității instalate în întreaga lume sunt dramatice (Fig
5.2 și 5.25); aceste două efecte sunt strâns legate, fiind exemple de
'curbe de învățare (cf. Fig. 17.2(a), §17.8, care prezinta aceste două
cantități complotat împotriva celuilalt). Până în 2013, costul per unitate de energie
generată ajuns la paritatea de rețea , în unele regiuni, și anume costul pentru un
utilizator de energie electrică pentru auto-generarea de egalat prețul la import de putere de utilitate (cum ar
calcule depind de valoarea banilor, durata de
instalare, și de ora din zi, care afectează utilități preț; vezi §17.6).
Asociat factori includ: (i) continuă îmbunătățire a eficienței în
tehnologie și de fabricație; (ii) acceptarea de către public; și (iii) minim
impactul asupra mediului. De o deosebită importanță a fost puternic
cererea pentru instalații FOTOVOLTAICE în țările cu sprijin instituțional
mecanisme, cum ar fi tarifele feed-in (de exemplu, Germania). Aceste mecanisme de piață
se referă la politicile de reducere a schimbărilor climatice, emisiile generate de combustibilii fosili
și de a crește securitatea energetică (a se vedea Capitolul 17). Rezultate cerere
încurajat producătorii la scară de producție, care la rândul său
a făcut sistemelor FOTOVOLTAICE ieftine, inclusiv pentru utilizatorii din alte țări –
și, prin urmare, încurajate în continuare de vânzări într-un curs pozitiv
buclă de feedback. Creșterea ușoară a modulului de preț în anul 2006 pentru anul 2007 a fost
pentru aprovizionare Si pentru celulele solare nu a putut ține pasul cu creșterea
cererii înainte de noi Si turnatorii au fost deschise în răspuns. Industria
observatorii se așteaptă ca prețurile modul de a continua să scadă, deși cu
ocazionale 'sughiț așa în 2006 la 2007 (EPIA 2012; IRENA 2012).
Tabelul 5.3
Performanță selectate celule solare sub concentrate "răsărit de soare" (măsurată în
simulatoare solare).
Material
Tip
Intensitatea ('sori')
Eficiența
Si
singur cristal
92
28%
GaAs
film subțire
117
29%
GaInP/GaAs/GaInNAs
multi-joncțiune
418
43%
Sursă: Datele colectate prin Green et al. (2012)
TWIDELL PAGINARE.indb 191
01/12/2014 11:36
192
Fotovoltaice tehnologie
Costurile în Fig. 5.25 sunt exprimate în dolari SUA per watt peak
($SUA/Wp). Aceasta este o masura standard de cost referitoare la ieșire în
lumină de densitatea de flux radiant 1000 W/m
2
cu un standard spectrale
de distribuție ( corespunzătoare la Soare, la 48 de grade de la verticală, adică AM1.5)
și cu temperatură de pe panoul fixat la 25
°C. cu toate Acestea, un complet
iluminate panou evaluat la (să zicem) 80 Wp va produce, probabil, mai mult de 80 de W
(i) de radiație este mai mică de 1000 W/m
2
și/sau (ii) op-
ing temperatura este mai mare de 25
°C. costul De capital pe vârf watt a
instalat sisteme este de două până la trei ori mai mult decât fosta fabrica de costul de
module din cauza 'echilibru a sistemului de costuri pentru alte componente și
de instalare.
De obicei, la fel de important ca costul de capital pe Wp de un nou sistem este
costul per kWh de energie electrică produsă, de exemplu, la un neumbrit locație fixă
în California un tablou evaluat la 1 kWp poate produce 1800 kWh/y, totuși, în
marea BRITANIE această ieșire poate necesita o putere nominală de 2 kWp.
§5.7.2 Grid-connected systems
Creșterea cererii pentru PV a fost pentru grid-connected systems,
care a crescut de la <30% din totalul la nivel mondial capacitatea instalată în 1995
la ~97% în 2012 (REN21 2012; AIE-PVP 2013). De exemplu,
sunfacing suprafață de acoperiș majoritatea de case de tramvai din Europa, atunci când
cea mai mare parte acoperite în conectate la rețea fotovoltaică, generează anual un
Fig. 5.25
Reduceri de costuri de PV în aplicare. Curbe de la partea de sus sunt pentru costul total al instalată o
rețea conectate la sistem. În curba de la partea de jos este prețul franco fabrică de module (în
vrac). Diferența este costul de echilibru dintre componentele sistemului și de instalare. Notă:
Prețul pentru sistem FOTOVOLTAIC per watt capacitate a scăzut de la ~50%, peste 13 ani, condus de o
și mai mare scădere a prețurilor pentru module.
Sursa: Date de la D. Feldman et al., Fotovoltaice Tendințele de stabilire a Prețurilor: Istoric, Recent, și pe Termen scurt,
Previziuni, Laboratorul National pentru Energie Regenerabila, statele UNITE ale americii (iunie 2013).
$14
$12
$10
$8
$6
$4
Instalat sistem de Preț și
Global Modulul de Preț (2012US$/W
DC
)
$2
$0
1998
1999
2000
2001
2002
2003
2004
2005
2006
2007
2008
2009
2010
2011
2012
Global modulul de preț
Instalare An
Sistemul de preț pentru utilizator
(Valori Medii)
>100 kW
10-100 kW

10 kW
TWIDELL PAGINARE.indb 192
01/12/2014 11:36
§5.7 Sociale, economice și de mediu
193
cantitatea de energie electrică egală cu 50 până la 100% de uz casnic de energie electrică a
cererii. Astfel gospodari folosi propria lor microgenerated de energie electrică
în timpul zilei, în timp ce vânzarea de orice exces a rețelei de utilități, apoi, la noapte
vor cumpăra importate de putere. Grila, astfel, acționează ca o "stocare virtuală".
Pentru uz casnic consumul de energie electrică este neregulat și expunere la soare variază, o
ruleof-degetul mare este că 50% din microgenerated putere este folosit în clădire
și 50% este exportată către rețea. Același principiu se aplică pentru
afaceri și clădiri comerciale; cu toate acestea, în cazul în care sarcinile sunt mari și continu-
ous, o proporție mult mai mare de microgenerated putere este folosit pe
site-ul. Guvernul instituționale mecanisme de sprijin ajuta
microgeneratoarele stabili cost-eficiente sisteme de una sau mai multe dintre: (a) impunerea
de utilități să plătească pentru microgenerated de energie electrică la tarife preferențiale (feed-in
tarifelor și obligațiile legale); (b) subvenționarea capital inițial costul de
matrice solare; și (c) stabilirea unor plăți pentru carbon de reducere a emisiilor de credit
obținute în proporție de energie din surse regenerabile generate. Modular
natura PV generație și ușoare de statică module face
astfel distribuite (embedded) de generatie relativ ușor de instalat, fie
pe nou-construit și a stabilit clădiri, sau pe structuri independente.
Economia și ușurința de construcție sunt îmbunătățite prin
dezvoltarea de "structurale" panouri FOTOVOLTAICE integrate în material exterior a
clădirilor și a acoperișurilor, cu lor de instalat, costuri reduse de economii la
materialele convenționale. Este rezonabil să ne așteptăm că în câteva decenii
PV va deveni la fel de încorporată în standardul structuri de acoperiș de sticlă este
în windows acum.
§5.7.3 sisteme Stand-alone
Sisteme Stand-alone care depind de stocare baterii sunt de obicei
de două ori mai scump pe unitatea de capacitate ca grid-connected systems, din cauza
la un cost suplimentar de baterii. Pentru aplicații stand-alone, cea mai
importantă măsură este costul relativ al serviciului livrat la un anumit
site (de exemplu, compararea unui PV-alimentat de lumina de o anumita intensitate a luminii cu
o lampă alimentată de lumina de intensitate similară). În ceea ce privește
eficiența-ofuse de energie electrică solară, există un trade-off între sistemul de
componente (de exemplu, mai bine aparate eficiente energetic necesită panouri mai mici și
mai puțin balanța de sistem cost), astfel încât investițiile în eficiența energetică aproape
întotdeauna dă pe termen lung reduceri în viață cheltuieli, după cum
a subliniat în Capitolul 16.
§5.7.4 PV pentru electrificarea rurală, mai ales în curs de dezvoltare
țări
PV utilizarea și cererea au continuat pentru off-grid de electrificare rurală

vital
pentru dezvoltarea economică și socială, în special în zonele rurale din
țările în curs de dezvoltare, unde miliarde de oameni trăiesc fără acces la rețea
TWIDELL PAGINARE.indb 193
01/12/2014 11:36
194
Fotovoltaice tehnologie
Fig. 5.26
O progresie de iluminat solare kituri. Sistemele cu mai eficiente lumini (Led-uri)
in figura (a) și (b) sunt mult mai ieftine decât cele cu Cfl-urile prezentate în (c), care au fost
de obicei până de curând.
o
O bază de felinar solar', cu celule evaluat la 0.3 Wp.
b
Un copil de 11 W sistem cu 4 lămpi cu LED-uri. Fiecare lampă are o valoare nominală de eficiență de 23%
(cf ~5 până la 10% pentru Cfl-uri) și are o 60 kJ baterie Li-ion construit în ea, și o reglabile
setare de luminozitate care permite pentru a rula timp de până la 12 ore. 'Baioneta' conectori
permite o instalare ușoară.
c
Solar sistem home fi instalat pe scară largă în țările în curs de dezvoltare, care costa ~1000$,
în 2013.
(a)
(b)
(c)
de energie electrică (a se vedea §17.2.2). Înainte de apariția de PV solar/baterie de putere,de
obicei, astfel de oameni s-a bazat pe lămpi cu kerosen și lumânări pentru iluminat
și scumpe uscat-baterii pentru radio și telefoane mobile, sau mai mari pentru
sarcini pe generatoare diesel.
În zonele rurale, la preț de vânzare cu amănuntul de benzină și alți combustibili similari, ca folosite
pentru iluminat, este, de obicei, cel puțin dublu orașul prețul și disponibilitatea poate
fi neregulat. Acest lucru prezintă o oportunitate pentru energie electrică solară și sisteme de
baterii pentru a furniza lumină. Tub și luminile fluorescente compacte (Cfl)
au fost utilizate pe scară largă ca ei au fost de aproximativ cinci ori mai eficiente decât
lămpile cu incandescență, Astfel de instalații, cu trei sau patru Cfl-uri, și
TWIDELL PAGINARE.indb 194
01/12/2014 11:36
§5.7 Sociale, economice și de mediu
195
de obicei, de asemenea, alimentarea încărcătoare pentru telefoane mobile și radio, sunt numite
sisteme solare casnice (vezi Fig. 5.26(c)). De obicei acestea costa ~1000$,
instalat. Din 2008, lămpi cu LED-uri au devenit disponibile pe scară largă,
care produc aceeași cantitate de utila luminos (lumeni) pentru o treime din
energia electrică (kWh), astfel încât un sistem echivalent poate folosi mult mai mici
panouri FOTOVOLTAICE pentru aceeași cantitate de lumină (Fig. 5.26(b)). Împreună cu
scăderea prețurilor pentru module aceasta și-a redus substanțial costul de
un solar sistem home pentru 300 de dolari, care este mai ușor de finanțat. Astfel de
sisteme de <
~10 Wp sunt numite Solare Pico Sisteme. Acestea includ port-
capabil de lanterne solare (Fig. 5.26(a)), care avea electronice și o baterie Li-ion
baterie built-in și costa doar ~US$10 la$40, în funcție de calitate
și de ieșire de lumină, ceea ce le face accesibile chiar și pentru cei foarte săraci;
până în 2013, de circa 0,5 milioane de LED-uri lanterne au fost vândute în Africa. Ca cu
toate tehnologie, a crescut pe piețele permite îmbunătățit suport tehnic și
, prin urmare, mai durabile sisteme (AIE-PVP 2013).
Provocările-cheie în a face astfel de sisteme durabile nu mai sunt
tehnice, dar instituționale și financiare. Soluții adecvate pentru aceste
non-tehnice provocări depinde de cultura locală și factori sociali (de exemplu,
dacă oamenii funcționează ca persoane fizice sau în cooperare într-o comunitate,
gradul de educație și aptitudini practice (inclusiv pentru lucrări de întreținere),
venituri în numerar, și ușurința de transport pentru furnizori) (Chaurey și Kanpal
2010). Uneori, în cazul în care o comunitate are o cooperativă de infrastructură
și cultură, un sistem centralizat 'microgenerare' mini-sistem grilă servește un
sat întreg.
De multe ori cu sprijin de la bănci multilaterale și bilaterale de donatori,
de multe milioane de sisteme solare casnice (SHS) au fost instalate în întreaga lume,
în special în Africa, Orientul Îndepărtat și America de Sud, împreună cu piața
structuri pentru difuzarea în continuare. Mai multe țări în curs de dezvoltare au
modele de afaceri inovatoare pentru PV microgenerare bazate pe taxa pentru
service', 'pay as you go" sau "prepaid contorizare' a îmbunătăți
accesibilitatea. Guvernul Chinei a distribuit aproximativ 400.000 de SHS între
2005 și 2011, completând 2800 MW de FOTOVOLTAICE conectate la retea. Într-o
perioadă similară, de aproximativ 1.3 milioane de SHS au fost distribuite în Bangladesh de
30 de organizații partenere, cu finanțare de la Banca Mondială și alte
agenții (REN21 2012).
În țările bogate, sisteme solare casnice sunt utilizate la distanță
de gospodarii, etc., prea departe de grilă pentru a justifica legătură; astfel de sisteme
, de obicei, au capacitate instalată mai mare (>5 kW) pentru mai multe electrice
aparate decât sunt accesibile în țările mai sărace.
Ca piețe de desfacere pentru micii surse regenerabile de energie crește, diferențele
între stand-alone și conectate la rețea sistemelor de microgenerare sunt
mai puțin contrastante; este evident benefic cazul în care, ca multe componente
sunt posibile în comun, astfel încât prezentarea unui totale mai mari de piață și mai puțin
diferențiere între curs de dezvoltare " și " dezvoltate regiuni.
TWIDELL PAGINARE.indb 195
01/12/2014 11:36
196
Fotovoltaice tehnologie
§5.7.5 de impact asupra Mediului
În funcțiune, fotovoltaice sunt inofensive pentru mediu, fără
emisii și fără zgomot, deși fabricarea implică unele pe deplin controlate de
substanțe chimice nocive și folosește energie. Modul de viață garantat de către
producători este de obicei de cel puțin 20 de ani, dar cele mai multe module vor genera
acceptabil pentru foarte mult timp, poate la ~100 de ani pentru module
cu celulele cristaline în bună încapsulare. La sfârșitul ciclului de viață, module
ar trebui să fie returnate de specialitate pentru reciclare; din păcate, astfel de instalații nu sunt
(încă) frecvente timpul pentru un anumit tip de panouri FOTOVOLTAICE pentru a genera electricitate
egală în energie pentru că a folosit în fabricarea acestuia (acesteia de recuperare a energiei)
depinde de site-ul insolație și metoda de fabricație. Pentru o
tipice de climat temperat, această energie timp de recuperare pentru un singur cristal
de siliciu încapsulate module este de aproximativ două până la trei ani (a se vedea
referințele de la sfârșitul acestui capitol); pentru film subțire tehnologii și pentru
mai însorite locuri este mai puțin.
§5.7.6 Outlook
Producția de masă de module FOTOVOLTAICE continuă să crească dramatic în
scară și de calitate, asociate cu scădere în preț, deci, un viitor în care
majoritatea dintre noi acoperișuri la clădiri genera energie electrică este previzibil.
Ca un mecanism pentru o astfel de producere de energie electrică, de energie PV este inegalabil

nu există emisii, nu există nici un zgomot, aproape nici costurile de funcționare,
durata de viață este de cel puțin câteva decenii, noile costuri sunt reducerea și se păstrează în
ploaie!
Fig. 5.27
Aprecierea publicului și înțelegere este esențială pentru succes.
o
Gospodari cu o casă mică-sistem de iluminat în Bhutan. Training local de persoane
în întreținere de bază a acestor sisteme este vital pentru succesul lor.
b
Membrii Westmill Solare de Cooperare în marea BRITANIE la deschiderea lor de 5 MW
de energie electrică-instalație de generare; tot mai multe cooperative acționarii au aceleași
drepturi, indiferent de investiții.
(a)
(b)
TWIDELL PAGINARE.indb 196
01/12/2014 11:36
Întrebări rapide
197
ÎNTREBĂRI RAPIDE
Notă: Răspunsurile la aceste întrebări sunt în textul de la secțiunea relevantă
din prezentul capitol, sau poate fi ușor dedusă din aceasta.
1
Ce este numele de o entitate de cuantificat lumina?
2
Ce este aproximativă de tensiune circuit deschis-o Si joncțiune p–n?
3
Aveți opțiunea de a scurt-circuitului, fie o baterie sau un
iluminat de module fotovoltaice; care sunt periculoase, care sunt în condiții de siguranță,
și de ce?
REZUMAT CAPITOL
Celulele solare produc energie electrică din fotovoltaice (PV) în vigoare, adică absorbția luminii în
materiale semiconductoare. Un cont de solid-state fizicii care stau la baza acestui proces este dată în Revizuire 4.
Avantajele tehnice ale PV includ său universal aplicabilitate (deși producția de energie este mai mare în
mai însorite locuri), modular caracter (permite utilizarea la toate scalele de ~1 W la ~100 MW), fiabilitate și
durată lungă de viață (pentru că nu există piese în mișcare), ușurința de utilizare, și lipsa de zgomot și emisii. Principalele
tehnice dezavantaj este faptul că generarea de energie electrică este numai în timpul zilei. Prin urmare, de stocare a energiei
electrice
(de exemplu, baterii) sau grilă de legătură este de obicei. Astfel de mecanisme, de asemenea, a netezi mai rapidă variabilitate de
ieșire în timpul zilei.
PV a fost puternic încurajat în mai multe țări de politici economice, cum ar fi tarifele feed-in.
Rezultate cererii încurajat producătorii la scară de producție, care la rândul său a făcut sistemelor FOTOVOLTAICE
mai ieftin din întreaga lume, inclusiv pentru sisteme stand-alone în zonele rurale din țările în curs de dezvoltare – și, astfel,
a încurajat în continuare de vânzări într-o continuă buclă de feedback pozitiv. În consecință putere PV este una dintre
cea mai rapidă creștere tehnologii de energie: capacitatea instalată a crescut exponențial de la ~200 MW în 1990
la mai mult de 80.000 MW (80 GW) în 2012 (din care 97% a fost grid-connected systems), similare cu o
rată de creștere de așteptat să continue. În zonele cu climat însorit, PV de putere este acum costuri competitive cu zi
vârf rețeaua de electricitate.
Eficiența și eficacitatea costurilor de celule fotovoltaice sunt îmbunătățite continuu de R&D
și de experiență de fabricație. 'Prima generație de celule bazate pe cristalin sau multi-cristalin Si
singlejunction celule domina prezenta instalații. 'Generația a doua' celule utilizare film subțire de celule tehnologie pentru
un singur nod de celule, dar bazat pe depunerea de straturi subțiri de materiale fotoactive pe un sprijin
substrat (de exemplu, sticlă). Prin utilizarea mult mai puțin din cel mai scump material (semiconductor), peliculă subțire
de celule și module sunt mai ieftine pe unitate de capacitate ($/Watt). Unele dintre ele folosesc mult mai complex chimic
semiconductori, cum ar fi Senzuală
x
Ga
1–x
Se
2

(CIGS) sau anumite organice oligomeri. Intrinseci nepotrivire de


spectrul solar la band gap limitează eficiența single-band celulele fotovoltaice pentru a <35%. 'A treia
generație de celule nu sunt limitate la un singur nod operare; de exemplu, acestea includ multi-joncțiune/
heterojunction tandem celule concepute pentru a absorbi o gamă mai largă din spectrul solar decât o singură joncțiune
celule și deci, având potențialul de eficiență >30%, fără solare de concentrare, și >40% cu
concentrația.
Comerciale celulele fotovoltaice au acum eficiență de aproximativ 12 până la 25% în ordinare soare. Celulele
FOTOVOLTAICE
sunt de obicei vândute ca intemperii module, cu circuit deschis de tensiuni între aproximativ 15 și 30 V.
curent de celule este în mod inerent curent continuu (DC); invertoare electronice sunt folosite pentru a schimba acest lucru pentru
curent alternativ (AC), dacă este necesar. Ieșire de zi cu zi este de obicei ~0.5 ~1.0 kWh/( m2 zi), în funcție de
climat.
TWIDELL PAGINARE.indb 197
01/12/2014 11:36
198
Fotovoltaice tehnologie
4
Pe măsură ce temperatura crește constant expunere la soare, nu-i putere PV
crește sau descrește?
5
Module FOTOVOLTAICE generează curent continuu, dar cele mai multe grile sunt AC; cum pot fi
conectate?
6
Numele trei componente ale unui PV de echilibru a sistemului (BoS).
7
Cum poate absorbția radiațiilor într-o celulă PV fi crescut?
8
Ce este factorul dominant care limitează eficiența de un single-band
gap celule PV?
9
Numele într-un fel în care intrinseci lipsa de eficiență al QQ 8 pot
fi depășite.
10
Dați două motive pentru modul PV costul a scăzut dramatic
în ultimii 20 de ani.
PROBLEME
5.1
Trupa decalaj de GaAs este de 1,4 eV. Calcula optim de
undă a luminii pentru sistemele fotovoltaice de generare într-o GaAs celule solare.
5.2
(a)
Da ecuației pentru m–V caracteristic de o joncțiune p–n
diodă în întuneric.
(b)
Dacă saturația curentă este 10
-8
Sunt
-2
, calcula și de a trage
I–V caracteristic ca un grafic la 0,2 V.
5.3
(a)
Ce este aproximativă foton densitate de flux (fotoni s
- am
m
-2
)
pentru AM1 radiației solare la 0,8 kW m
-2
?
(b)
AM1 expunere la soare de 0,8 kW m
-2
este incident pe un singur Si celule solare
din zona l00 cm
2

. Presupunem că 10% din fotoni cauza electron – gol


separarea peste intersecția care duce la o sursă externă de curent.
Ce este curentul de scurt-circuit - am
sc
de celula? Schița I–V
caracteristice pentru celula.
5.4
Un mic uz casnic sistem de iluminat este alimentat de la un nominal
8 V (adică 4 celule, la 2 V) baterie de stocare având o 30 Ah aprovizionare atunci când
încărcat. Iluminatul este folosit pentru 4.0 sec fiecare noapte la 3.0 A.
Un Design potrivit fotovoltaice sistem de putere care va încărca
bateria de la un aranjament de Si celule solare.
(a) Cum vei aranja celulele?
(b) Cum va circuitul fie conectat?
(c) Cum va testa circuitul și performanță?
5.5
(a)
Calcula timpul aproximativ pentru un singur cristalin PV
module pentru a genera energie electrică egală în termeni de energie în
energie primară utilizate în fabricarea acestuia. Ia în considerare: un tipic
Japonez climatice cu 1450 kWh/(m
2
y) de expunere la soare; module
TWIDELL PAGINARE.indb 198
01/12/2014 11:36
Probleme
199
de 15% a eficienței; pe 1.0 kWp putere de vârf de module. În
total directe de producție și prelucrare de energie (include
rafinare, etc.) este de 1350 kWh/kWp, plus o suplimentare 1350 kWh/kWp
de fabrica aeriene de energie', cu 90% atât la energie electrică
(generate termic la 55% eficienta la combustibili în combinat
ciclu de generație).
(b) Simplă de recuperare a energiei de (a) își asumă electric-
itate are aceeasi "valoare" sau "valoare" ca de căldură. Este acest lucru corect?
Ce rezultat ar putea fi obținut în cazul în celule și module PV au fost
fabricate în întregime din, să zicem, hidroelectricitate? Cum poate
fabricarea fi mai eficiente energetic?
5.6
Ceea ce este cel mai bun de orientare fixe pentru producerea de energie de la un
modul fotovoltaic situat la Polul Sud?
5.7
(A) band gap intrinseci Si la 29°C este de 1.14 eV. Calcula
probabilitatea funcția exp(
-E
g
/(2kT) pentru electroni să treacă complet
benzii de excitație termice.
(b) Dacă nivelul Fermi în tip n Si este de aproximativ 0,1 eV sub con-
ducerea trupa, calcula probabilitatea funcție de electroni să
fie termic entuziasmat în banda de conducție.
Comparați răspunsurile dumneavoastră pentru (a) și (b).
5.8
Einstein a câștigat premiul Nobel Fizică în 1905 pentru a explica
efectul fotoelectric, în care lumina incident pe o suprafață poate duce
la emisia de electroni de la suprafața cu energie
E
= h
n-Φ
unde h
n este energia unui foton de lumină și Φ este o proprietate de
suprafața.
(a) care sunt principalele diferențe și asemănări între
efectul fotoelectric și efectul fotovoltaic?
(b) Discuta despre cum, dacă la toate, efectul fotoelectric ar putea fi utilizate pentru
randament util de energie.
5.9
Un modul fotovoltaic este evaluat la 50 W cu izolatie 1000 W/m
2

,
pentru ca vârf de expunere la soare pe Pământ. Ce-ar fi apogeul ieșire de pe
Marte? (Notă: adică distanța de la Soare la Pământ 1.50 x 10
11
m
și de pe Marte 2.28 x 10
11
m; nu există nici o atmosferă semnificativă
de pe Marte.)
5.10
De diferențiere a (R4.28) de către părți, a dovedi (R4.29).
5.11
O matrice solare evaluat la 1 kW produce aproximativ 1800 kWh anual în
California. Ce este capacitatea sa de factor?
TWIDELL PAGINARE.indb 199
01/12/2014 11:36
200
Fotovoltaice tehnologie
NOTĂ
1
Cele fotovoltaice efect nu ar trebui să fie confundat cu fotoelectric efectul prin care electronii sunt emiși
de pe suprafețe, așa cum sa explicat de Einstein în 1905 (vezi Problema 5.8).
BIBLIOGRAFIE
Cuprinzătoare de cărți
Goetzberger, A. Hoffmann, V. U. (2005) Solare Fotovoltaice de Generare a Energiei, Springer Serie Optică
Science, Springer. Excelent comentariu de PV de dezvoltare, generație principii, fabricarea, instalarea și
piață de implementare. Cantitative și informații amănunțite, dar non-matematice. Bine face referire.
Luque, A. și Hegedus, S. (eds) (2011, 2nd edn) Manualul de Fotovoltaice Știință și Inginerie, Wiley, New
York. Referință cuprinzătoare care acoperă fizica, construcție, testare, sisteme, aplicații, economie și
implicațiile pentru dezvoltarea rurală. Include un excelent și ușor de citit de ansamblu de către editorii de stat de
artă din toate cele de mai sus, și în profunzime capitole pe fiecare dintre principalele dispozitiv tehnologii.
McEvoy, A., Markvart, T. și Castañer, L. (coord) (2011) Ghid Practic de celule Fotovoltaice: Fundamente și
aplicații, Elsevier, Oxon. Un multi-autor sondaj similar Luque și Hegedus (2011).
În principal dispozitiv fizica
Verde, M. A. (1998) Celule Solare: principii de Funcționare, tehnologia și sistemul de aplicare, Prentice-Hall, New York.
Retipărit de la Universitatea din New South Wales, Australia. Un text de bază de aproape primele principii. Excelent
text, cu revizuirile ulterioare, de către un cercetător remarcabil. Vezi Wenham et al. (2011) pentru o companie de text aplicată.
Verde, M. A. (2001) Fotovoltaice Fizică și Dispozitive, în J. E. Gordon (ed.), Energia solară: starea de arta,
James & James, din Londra, pp. 291-355. Concis și cuprinzător de revizuire într-un excelent general, solar text.
Verde, M. A., (2006), A Treia Generație De Celule Fotovoltaice, Springer, New York. Se concentrează pe tipuri avansate încă de
la
conceptual sau la începutul etapa de laborator.
Islamul, S., Woyte, A., Belmans, R., Heskes, P., Rooij, P. M. și Hogedoorn, R. (2006) Costul efectiv de-a doua
generație AC-module: elaborarea și testarea aspecte', a Energiei, 31(12), 1897-1920. Studiu cuprinzător,
inclusiv opțiuni tehnologice și costurilor.
În principal aplicații
Boxwell, M. (2012, 6 edn) energie Electrică Solară Manualmai multe Publishing, marea BRITANIE. Ei bine descrise de
subtitrare, Un simplu ghid practic pentru energie solară: cum de a proiecta și instala solare fotovoltaice sisteme electrice'. A se
vedea
, de asemenea, www.SolarElectricityHandbook.com.
Chaurey, A. și Kandpal, T. C. (2010) de Evaluare și de evaluare a FOTOVOLTAICE pe bază descentralizată rurale
electricification: o privire de ansamblu', Energia Regenerabilă și Durabilă Recenzii, 14, 2266-2278. Excelent revizuire la nivel
global,
concentrându-se pe cele sociale și aspecte instituționale și diverse moduri folosite pentru a le îndeplini în diferite părți ale
lumii.
AIE-PVP (2013) Pico Sistemele FOTOVOLTAICE Solare pentru Casele de la Distanță. Rezumat util, de asemenea, inclusiv o
istorie de
'solar home systems pentru țările în curs de dezvoltare; disponibil de la www.iea-pvps.org.
TWIDELL PAGINARE.indb 200
01/12/2014 11:36
Bibliografie
201
Krauter, S. (2006) Solare De Generare De Energie Electrică, Springer, New York. Utile pe bază ale circuitelor etc.; de
asemenea, unele
de modelare de utilizări.
Labouret, A. și Viloz, M. (2011) a Energiei Solare Fotovoltaice, La institutul de Inginerie și Tehnologie,
de la Londra. Tradus din 4 edn de accesibil franceză de muncă.
Loos, G. și van Hemert, B. (eds) (1999) Stand-alone Aplicații Fotovoltaice: Lecții învățate, International
Energy Agency, Paris, și James & James, Londra.
Weir, T. și Prasad, S. (2012) Adoptarea de clima-smart technologies: cazul rurale solare de energie electrică în
Insulele Pacificului', disponibil la www.climate2012.de.
Wenham, S. R., Verde, M. A., Watt, M. E., Corkish, R. și Sproul, A. (2011, 3rd edn) Aplicate Fotovoltaice,
Routledge, Oxon. Scris de experți cu experiență pentru universitate și studenți, dar cu teoria de bază și
aplicarea detaliu.
Referințe specifice
European Photovoltaic Industry Association (2012) , perspectiva Pieței Mondiale pentru energia Fotovoltaică până în 2016,
disponibil
la www.epia.org.
Feldman, D. et al. (2013) Fotovoltaice Tendințele de stabilire a Prețurilor: Istoric, recent, și pe termen scurt,
previziuni, National
renewable Energy Laboratory, SUA (iunie).
Verde, M. A., Emery, K., Hishikawa, Y., Warta, W. și Dunlop, E. D. (2012) 'celule Solare eficiența tabele (versiune
39)', Prog. Photovolt:Res. Appl., 20, 12-20. Continuând seria de rapoarte; a se vedea de exemplu http://onlinelibrary.wiley.com/
doi/10.1002/pip.2163/plin.
IRENA (2012) surse Regenerabile de Energie analiza Costurilor Seria: Problema 4, Fotovoltaice, Internațional de Energie
Regenerabilă,
Agenția, www.irena.org.
Knapp, K. și Bufon, T. (2001) 'de investigații Empirice de timpul de recuperare a energiei pentru module fotovoltaice',
Energie Solară, 71, 165-172. A se vedea, de asemenea, cele mai optimiste estimări de recuperare a energiei ori de Alsema, E.
(1998)
Energia regenerabilă și Durabilă Recenzii, 2, 387-415; de asemenea, Fthenak, V. et al. (2005) Progresele înregistrate în celule
Fotovoltaice,
13, 713-723, și (2008) Mediu, Știință și Tehnologie, 42, 2168-2174.
REN21 (2012) Global Status Report 2012. Acesta este un studiu anual al RE-utilizarea și politici din întreaga lume,
disponibil la www.ren21.org. Raportul pe 2012 include un capitol special privind rurală din surse regenerabile de energie'.
Reviste și site-uri web
Progresul în celule Fotovoltaice, bilunar de Wiley, Chichester. Un important jurnal cu liderii mondiali în
energia fotovoltaică de pe Placa de Editori.
www.solarbuzz.com. Include statistica industriei și știri, contururi de tehnologii și statutul lor, etc.
www.solarserver.com/. International Solar Energy information. Are rapoarte excelente pe tehnologie și de absorbție.
www.nrel.gov/pv/. NE-Centrul Național pentru energia Fotovoltaică. Puternic pe R&D, NE-programe guvernamentale, și cazul
de studii.
www.iea-pvps.org. Site-ul web al AIE Operativ asupra Sistemelor Fotovoltaice; are multe rapoarte utile,
inclusiv rapoarte anuale privind starea de industrie și aplicații.
TWIDELL PAGINARE.indb 201
01/12/2014 11:36

Hidroenergie
CAPITOLUL
6
CONȚINUTUL
Obiective de studiu
202
§6.1 Introducere
204
§6.2 Principii
208
§6.3 Evaluarea resurselor
209
§6.3.1 Măsurarea cap Sec
210
§6.3.2 Măsurare a debitului Q
210
§6.4 Impulse turbine
212
§6.5 Reacție turbine
217
§6.6 sisteme Hidroelectrice
220
§6.6.1 Puterea de reglementare și control:
grid-connected
222
§6.6.2 Puterea de reglementare și control:
sisteme stand-alone
223
§6.6.3 eficiența Sistemului
223
§6.6.4 domeniul de Aplicare pentru upgrade-uri de tehnologie 224
§6.7 Pompat de stocare a energiei hidro
224
§6.8 aspectele Sociale și de mediu
225
Rezumat capitol
227
Întrebări rapide
228
Probleme
228
Bibliografie
231
Caseta 6.1 Măsurarea debitului Q:
principii descrise pentru
sistemele mici
210
Caseta 6.2 "viteză Specifică"
216
Caseta 6.3 barajul celor Trei Defileuri, Yangtze
Râu, Provincia Hubei, China:
cea mai mare hidrocentrala de
instalare
221
OBIECTIVE DE STUDIU

Aprecia mare măsură din întreaga lume
generarea de energie electrică de apă care se încadrează.

Înțeleg cum energia transformări
apar.

Efectuați calculele fundamentale.

Estimarea potențialului hidroenergetic de la un site.

Ia în considerare aplicații la scară mică și astfel să fie
capabil de a efectua experimente practice și
studii de teren.

Aplica scalarea legi care extinde laborator de
studii pentru aplicarea pe scară largă. Aprecia
impactul asupra mediului.
TWIDELL PAGINARE.indb 202
01/12/2014 11:36

www.shahrsazionline.com
Lista de tabele
203
LISTA DE FIGURI
6.1
Creșterea lume generarea de energie hidroelectrică (TWh/an) și capacitate (GW).
204
6.2
Aspect tipic de o centrală hidroelectrică.
207
6.3
Măsurarea debitului de apă.
211
6.4
Diagrama schematică a unei Pelton roata turbina cu impuls.
213
6.5
Fotografie de roată Pelton.
213
6.6
Viteza de ceașcă și fluid
214
6.7
Metode de creștere a puterii de la o anumită dimensiune de mașină, care lucrează în aceeași apă
sub presiune.
218
6.8
Ilustrative de vârf în materie de eficiență.
219
6.9
Aspect de micro hidrocentrale de sistem folosind o roată Pelton.
220
6.10 Trei Defileuri baraj hidroelectric în China: cel mai mare din lume.
221
6.11 aspectul Tipic al unui pompat hidro sistem de stocare a energiei.
225
6.12 Un U-weir.
229
LISTA DE TABELE
6.1
Potențial hidroenergetic, de capacitate și de ieșire în funcție de regiune și de eșantionul de țări (2008). Notă
de variație în factor de capacitate de țară, și un potențial semnificativ de dezvoltare din Africa și
Asia. 205
TWIDELL PAGINARE.indb 203
01/12/2014 11:36
204
Hidroenergie
§6.1 INTRODUCERE
Termenul de hidroenergie înseamnă valorificarea apă care se încadrează pentru a produce
putere – de obicei sub formă de energie electrică (de exemplu, hidroenergie). Din punct de vedere istoric
hidrocentrale a fost de asemenea utilizat pentru măcinat cereale sau pentru pomparea apei.
Alte surse hidraulice (apa) putere sunt valuri și maree (Capitolele
11 și 12).
Hidroenergie rămâne cel mai stabilite, utilizate pe scară largă și
pe termen lung de resurse regenerabile pentru producerea de energie electrică. Hidroenergie
instalații sunt adesea combinate cu alte utilizări, inclusiv de control al inundațiilor, de
aprovizionare cu apă, și cu acumulare prin pompare de apă pentru ulterioare
hidroelectrice. Este de apreciat la orice scară, de la foarte mari (~GW) la foarte mic
(~kW) capacitate; cu toate acestea, oportunitățile depinde în mod crucial de topografie
și precipitații de a oferi debitul de apă suficientă și toamna (cap).
Lumea e mai vechi de distribuție a energiei electrice în 1881 derivate din
hydroturbines de kW scară capacitate. Până în 2008 hidroelectrice de capacitate a ajuns la
aproximativ 874 GW, nu inclusiv ~130 GW de pompat hidro-depozitare. La
o capacitate totală de instalații în întreaga lume continuă să crească de la aproximativ
2% pe an, cu hidroelectricitate furnizarea de aproximativ 16% de la nivel mondial
de energie electrică (a se vedea Fig. 6.1). Acest procent poate crește, determinată de
considerente de securitate energetică națională și atenuarea efectelor climatice
schimba (a se vedea Capitolul 17). Cu toate acestea, de mediu și sociale
sunt de multe ori cele mai mari provocări a continuat implementare; prin urmare, ai grijă
de management este esențială (a se vedea §6.8).
Fig. 6.1
Creșterea lume generarea de energie hidroelectrică (TWh/an) și capacitate (GW). Acumulare prin pompare
de capacitate nu sunt incluse. Datele reale din US Energy Information Agency. Liniile punctate sunt
extrapolare liniară din ultimii 10 ani. Bara verticală la dreapta indică mijlocul de jumătate din
peste 100 de estimările prognozate generație în 2020 (TWh/an), astfel cum a fost revizuit de către IPCC SRREN
(2011).
5000 TWh/an
GW
4000
3000
2000
1000
0
1970
0
800
1200
400
1980
1990
2000
2010
2020
2030
An
TWIDELL PAGINARE.indb 204
01/12/2014 11:36
§6.1 Introducere
205
Hidroelectrice de ieșire depinde de cantitatea anuală de precipitații, de captare a apei și,
desigur, capacitatea instalată. Tabelul 6.1 recenzii potențialul hidroenergetic și
de generare de continent și regiuni pentru diferite țări; în general, nu
este semnificativ potențial nevalorificat, în special în Africa. În Norvegia, Venezuela,
Brazilia, Canada și hidroelectrice produce mai mult de jumătate din totalul de energie electrică.
Ca o țară se dezvoltă, site-uri cu cea mai mare capacitate, de obicei, sunt valorificate
în primul rând, așa național de creștere din capacitatea totală de generare tinde să se diminueze
cu timpul. Prin 1940, în vârstă țările industrializate au exploatat
lor mai bune site-uri, prin urmare, relativ mare proporție utilizate'
procentele prezentate în Tabelul 6.1. Acum mai mult de creștere prezentat în Fig. 6.1 este
în noi țări industrializate, în special China, Brazilia și India.
Cu toate acestea, național-scară estimări pot fi înșelătoare pentru locale
hidroelectrice de planificare, deoarece aplicații la scară mică (~10 kW la 1 MW)
sunt adesea neglijate de evaluări, în ciuda site-uri pentru astfel de
Tabelul 6.1
Potențial hidroenergetic, de capacitate și de ieșire în funcție de regiune și de eșantionul de țări (2008). Notă
variația factor de capacitate de țară, și un potențial semnificativ de dezvoltare din Africa și Asia
O
Regiune/de exemplu
țară
B
Brut
potențialul
TWh/an
C
Tehnice
potențialul
TWh/an
D
Real
generație
TWh/an (2008)
E
= D/C
Proporția de
tehnice
utilizate
%
F
Instalat
capacitate
(2008)
GW
G
Factor de capacitate
D/(F
× 8760 h/an)
%
LUME, total
39842
15955
3194.0
20
874.0
42
AFRICA
3909
1834
96.0
5
22.0
50
Congo (Dem Rep)
1397
774
7.3
1
2.4
35
Egipt
125
50
15.5
31
2.8
63
Etiopia
650
260
3.4
1
0.7
55
AMERICA de Nord
5511
2416
694.0
29
168.0
47
Canada
2067
820
377.0
46
73.4
59
Statele UNITE ale americii
2040
1339
255.0
19
77.5
38
AMERICA de Sud
7541
2843
643.7
23
131.6
56
Argentina
354
169
30.6
18
10.0
35
Brazilia
3040
1250
365.0
29
77.5
54
Peru
1577
395
19.0
5
3.2
67
Venezuela
731
261
86.7
33
14.6
68
ASIA
16618
5590
985.0
18
306.8
37
China
6083
2474
580.0
23
171.0
39
India
2638
660
114.8
17
37.8
35
Indonezia
2147
402
11.5
3
4.5
29
Japonia
718
136
74.1
54
27.9
30
Pakistan
475
204
27.7
14
6.5
49
Filipine
47
20
9.8
49
3.3
34
Turcia
433
216
33.2
15
13.7
28
Vietnam
300
123
24.0
20
5.5
50
TWIDELL PAGINARE.indb 205
01/12/2014 11:36
206
Hidroenergie
instalațiile fiind cele mai numeroase. Sondaje de multe ori nu reușesc să recunoască
beneficiile pentru proprietarii de mici dimensiuni site-uri pentru a compensa scumpe importate
putere și instala pe termen lung a activelor de capital. Astfel, potențialul hidro
generație de la run-de-râu scheme (de exemplu, doar cu foarte mici baraje)
este de multe ori subestimat. Factori sociali și de mediu sunt, de asemenea,
important, iar acestea nu prea poate fi judecat de sondaje la nivel mondial, dar numai prin
evaluarea condițiilor locale. Împreună cu direct costurile de construcție,
acești factori reprezintă pentru "potențialul tehnic" pentru studiu global
de energie hidroelectrică în Tabelul 6.1 fiind considerat doar aproximativ jumătate din brut
potențial evaluate în funcție de regiune.
Hidro instalații și plante (vezi de ex. Fig. 6.2) sunt de lungă durată cu
întreținere de rutină (de exemplu, turbine de 50 de ani și mai cu minore
de renovare, baraje și pe căile navigabile pentru, probabil, 100 de ani). Timp
turbina viață se datorează continuă operațiune de echilibru fără de înaltă
temperatură sau alt stres. Turbinele au un răspuns rapid pentru putere
generație și deci puterea ar putea fi folosite pentru alimentarea cu atât sarcina de bază și de
vârf a cererii de cerințele de pe o rețea de aprovizionare; rețineți că țările care utilizează
energia hidroelectrică, în principal, de cererea de vârf au capacitate relativ scăzută de factori
Tabelul 6.1
(continuare)
O
Regiune/de exemplu
țară
B
Brut
potențialul
TWh/an
C
Tehnice
potențialul
TWh/an
D
Real
generație
TWh/an (2008)
E
= D/C
Proporția de
tehnice
utilizate
%
F
Instalat
capacitate
(2008)
GW
G
Factor de capacitate
D/(F
× 8760 h/an)
%
EUROPA
4919
2762
714.8
26
220.7
37
Franța
270
100
59.3
59
21.0
32
Italia
190
65
41.6
64
17.6
27
Norvegia
600
240
140.0
58
29.5
54
Fed
2295
1670
180.0
11
49.7
41
Spania
162
61
17.8
29
16.0
13
Suedia
200
130
68.4
53
16.2
48
Elveția
125
43
38.9
90
13.5
33
Marea BRITANIE
35
14
5.1
36
1.6
36
ORIENTUL MIJLOCIU
690
277
27.7
10
11.5
27
OCEANIA
654
233
38.3
16
13.7
32
Australia
265
100
14.9
15
7.8
22
Noua Zeelandă
205
77
22.1
29
5.4
47
Note
o
Brut potențial de precipitații doilea tur de scrutin și de cartografiere, potențialul tehnic de construcții, experiența, capacitatea reală
instalată și producția efectivă.
b
Capacitatea și producția figuri exclude stațiile care sunt în principal sau pur pompat hidro.
Sursa: Date de la Consiliul Mondial al Energiei (2010), Studiu de Resurse Energetice.
TWIDELL PAGINARE.indb 206
01/12/2014 11:36
§6.1 Introducere
207
(Tabelul 6.1). Mai mult decât atât, turbinele pot fi proiectate pentru operațiunea inversă ca
pompe de acumulare prin pompare sisteme (§6.7).
Sistemele hidroenergetice au printre cele mai bune eficiențe de conversie
de toate cunoscute surse de energie (pana la 90% eficienta, apa de sârmă). La
relativ scumpe investiția inițială este compensat de durată lungă de viață, împreună
cu low-costul de exploatare și întreținere. În consecință, nivelat
costul de producție de energie electrică din hidrocentrale (de exemplu, costul de
generare medie pe durata de viață a proiectului: a se vedea Capitolul 17) poate fi la fel de
ieftine ca la 3 la 5 cenți SUA/kWh, în condiții bune, în comparație cu
utilitate prețurile de vânzare către public a ~la 15 la 20 de cenți SUA/kWh.
Unul aproape caracteristică unică a energiei hidroelectrice este continuu
gama de scale aplicabile, de la mai puțin de 1 kW la mai mult de 500 de MW.
Adjectivele "mic", "mare", etc. folosit pentru a descrie proiecte depind
în organizație sau persoană implicată; nu există un cod internațional.
Cu toate acestea, putem distinge patru mari cântare; Mare (>100 MW),
Mediu (de la 15 la 100 MW), Mini (0,1 la 15 MW) și Micro (<100 kW).
Dezavantajele majore ale hidroelectrice sunt asociate cu efecte
decât alte generatoare de echipamente, în special pentru sistemele mari. Aceste
Fig. 6.2
Aspect tipic de o centrală hidroelectrică. Apa este stocată în spatele unui baraj (în apropiere de partea de sus din
foto), curge pe conducte (mijloc de fotografie) la turbinele (în carcase partea de jos
a fotografiei), și este apoi eliberat din aval. Fotografie prezinta 1500 MW Tumut 3 putere
stație în Australia, cu cap de 150 m. În acest special de instalare, producția de apă
trece printr-un tunel (nu apare in foto) pentru a spori de irigații, agricole – astfel
să ilustreze cum hidro instalații poate avea mai multe societale și beneficii economice.
Această stație poate fi de asemenea folosit ca un pompat hidro sistem de stocare a energiei (a se vedea punctul 6.7).
TWIDELL PAGINARE.indb 207
01/12/2014 11:36
208
Hidroenergie
include posibile efecte negative asupra mediului, efectul de pește, colmatarea
barajelor, coroziune de turbine în anumite condiții de apă, impactul social
de deplasare a oamenilor din rezervor site-ul, pierderea de potențial
teren productiv (de multe ori echilibrat de beneficii de irigare de pe alte terenuri),
și relativ mari costurile de capital, comparativ cu cei fosili putere
stații. De exemplu, a existat ample dezbateri internaționale cu privire la
beneficiile și dezavantajele Barajului de la Assuan pentru Egipt și Trei
Chei proiect pentru China. Toate aceste probleme sunt discutate în continuare în §6.8.
Acest capitol tratează aspecte fundamentale de hidroenergie și
nu încearcă să fie complete în astfel de dezvoltat subiectul. În
special, am considerat aplicații la scară mică, deoarece elevii pot
utiliza în condiții de laborator și de teren pentru învățarea practică. Ne
referim cititorii să bibliografia pentru lucrări ample la înființată
de inginerie nivel.
Ecuația fundamentală (6.1) este suficientă pentru estimarea hidroelectrice
potențial la o anumită locație; metodele descrise în §6.3 oferi o
evaluare mai precisă. Turbinele sunt de două tipuri: (un) impuls
de turbine, în cazul în care fluxul este turbina ca un avion într-un mediu deschis,
cu puterea care derivă din energia cinetică a fluxului (vezi §6.4);
și (b) reacție de turbine, în cazul în care turbina este complet încorporat în
lichid și alimentat de la o cădere de presiune pe dispozitivul (a se vedea §6.5).
Mecanice turbine apoi conduce mașini (istoric de utilizare) sau
generatoare de energie electrică (utilizarea dominantă). Reacția generatoare de turbine poate fi
inversat, astfel încât apa este pompată la un nivel ridicat pentru pompaj și
generațiile ulterioare (§6.7), la o eficiență totală de ~70%. §6.6
considerare și alte aspecte tehnice sistemelor hidroelectrice, și §6.8 recenzii
aspectele sociale și de mediu de hidroenergie. La eResource
și Bibliografie la sfârșit de capitol oferi informații suplimentare cu privire la
cererile, inclusiv cele pur mecanice, hidraulice ram pompa.
§6.2 PRINCIPII
Apă de volum pe secundă Q și densitatea
ρ cade pe o pantă. La
masa care se încadrează pe unitatea de timp este
ρQ, iar rata de energie potențială de pierdut
care se încadrează de lichid este
ρ
=
P
QgH
0
(6.1)
unde g este accelerația gravitațională și H este verticală
componentă de calea de apă.
Turbinele converti această putere la arbore putere. Spre deosebire de energie termică
surse, nu există nici o fundamentale de termodinamică sau dinamic motiv pentru care
puterea de ieșire a unui sistem hidro ar trebui să fie mai mică decât puterea de intrare
P
0

în afară de pierderile prin frecare care poate fi în mod proporțional, foarte mici. Pentru
un site cu un rezervor de apă, H este fix și Q este reglabil. Prin urmare,
puterea de ieșire este controlat rapid la, sau mai mult, design-ul de ieșire,
TWIDELL PAGINARE.indb 208
01/12/2014 11:36
§6.3 Evaluarea resurselor
209
cu condiția că nu există suficiente de alimentare cu apă. Rețineți că densitatea de
apă proaspătă la temperatura ambiantă este de 1000 kg/m
3
și de aer numai despre
1,2 kg/m
3
, care este un motiv major pentru diferența de diametru între
apă și turbine eoliene de aceeași valoare nominală putere de ieșire.
Principalul dezavantaj al energiei hidroelectrice este clar, de asemenea, din (6.1): site-ul
trebuie să aibă suficientă Q și H. În general, acest lucru necesită o precipitațiilor > ~ 40
cm/y dispersate prin an, potrivit ridicat de captare sau râu
(dacă este posibil, cu apă de stocare) și o finală de căderea de apă pe
turbine. Această combinație de condiții nu este comun, deci hidroenergetic
este departe de a fi universal disponibile. Cu toate acestea, în cazul în care sunt disponibile, hidroenergie
este aproape cu siguranță cele mai potrivite generatoare de electricitate sursă, după cum
a sugerat în Exemplul 6.1.
Cu toate acestea, considerabil inginerie civilă (în formă de baraje,
conducte, etc.) este întotdeauna necesar pentru a direcționa fluxul prin
turbine. Aceste lucrări civile de multe ori costa mai mult decat cele mecanice si
componente electrice. Cu toate acestea, pentru o mare, mare șef, hidroenergie, tunel
tehnologia s-a îmbunătățit foarte mult ca urmare a introducerii din ce în ce mai
eficiente echipamente. În consecință, costurile de excavare s-au redus cu
25% în ultimii 30 de ani. Rețineți că costul pe unitatea de putere a turbinelor
tinde să crească cu Q. Prin urmare, costurile per unitate de putere de ieșire de
highhead instalații sunt mai puțin decât low-cap, cu excepția cazului conductelor costurile deveni
excesivă. Pentru instalațiile mici de la vechea moara de apa, site-uri, de conversie pentru
generarea de energie electrică poate fi foarte cost-eficiente.
§6.3 EVALUAREA RESURSELOR
Să presupunem că avem un flux de disponibil, care poate fi util pentru
hidroenergie. La început, doar date aproximative, cu o precizie de circa ±50%,
EXEMPLUL 6.1
Apa dintr-un moderat de dimensiuni râul curge la o rată de 100 m
3

/s pe un perfect netedă țeavă, care se încadrează 50 m


într-o turbină.
(a) Cât de mult puterea este disponibil? (b) Dacă, în practică, 10% din putere se pierde prin frecare, transformare și
distribuție, cât de multe case având medie consumul de energie electrică de aproximativ 0,5 kW (adică 12 kWh/zi) ar putea
alimentare?
Soluție
Din (6.1),
P
(1000kg/m ) (100m /s) (9,81 m/s ) 50m
49 10 kg.m/s
49MJ/s 49MW
0
3
3
2
6
3
=
×
×
×
=
×
=
=
Numărul de case este (49,000 – 4,900) kW/(0,5 kW pe casa)
≈ 88.000 de case, adică un oraș mare, cu o
populația de aproximativ 220.000.
TWIDELL PAGINARE.indb 209
01/12/2014 11:36
210
Hidroenergie
sunt necesare pentru a estima potențialul de putere al site-ului. Dacă acest studiu
dovedește promițătoare, apoi o investigație detaliată va fi necesar
implică date, de exemplu, precipitații luate pe parcursul mai multor ani. Este clar din (6.1)
care a estima puterea de intrare P
0
trebuie să măsurați debitul
Q și vertical disponibil toamna H (de obicei numit cap). De exemplu,
cu Q
= 40 l/s și H = 20 m, maxim puterea disponibilă la sursa
este de 8 kW. Acest lucru ar putea fi foarte potrivit pentru o sursa de uz casnic.
§6.3.1 Măsurarea cap Sec
Pentru aproape verticală cade, trigonometrice metode de anchetă (poate chiar
folosind lungimi de umbre) sunt adecvate; întrucât, pentru mai blând
pante, nivelul și polul topografie este simplă. Rețineți că puterea
de intrare a turbinei nu depinde geometrice (sau 'total') cap Sec
t
ca
chestionate, dar pe disponibile cap Sec
o
:
H
H
H
o
t
f
=

(6.2)
unde pierdere cap Sec
f

permite pentru pierderile prin frecare în conducte și


canale care duce de la sursă la turbina (a se vedea §R2.6). Cu potrivit
conducte
f
H
<
~ H
t
/3; cu toate acestea, prin (R2.11) H
f

crește proporțional cu
lungimea totală a conductei, astfel încât cele mai bune site-uri pentru hidroenergie au pante abrupte.
§6.3.2 Măsurare a debitului Q
Fluxul prin turbina produce energie, iar acest flux va
fi, de obicei, mai puțin decât debitul în flux. Cu toate acestea, fluxul în
flux variază în funcție de timp, de exemplu, între secetă și inundații
perioade. Pentru generarea de energie noi, de obicei, vrei să știi minim
(sezonul uscat) de curgere, deoarece o turbina potrivite pentru acest lucru va produce energie
pe tot parcursul anului, fără supracapacitate de mașini. Astfel de date sunt, de asemenea,
necesar pentru impactul asupra mediului (de exemplu, menținerea unui debit minim
pentru mediul acvatic). De asemenea, am nevoie să știu maximă de curgere și de inundații
de niveluri pentru a evita deteriorarea instalațiilor.
Măsurarea Q este mai dificilă decât măsurarea H.
Caseta 6.1 prezintă câteva metode posibile. Metoda aleasă va
depinde de dimensiunea și viteza de flux în cauză. Pentru mari
instalații, sofisticata metoda este mereu folosit.
CASETA 6.1 MĂSURAREA DEBITULUI Q: PRINCIPII DESCRISE PENTRU SISTEMELE MICI
La fel ca în § R2.2,
Q
t
t
debitul
(volumul trece în timp
)/
=


(6.3)
u
O
(viteza medie ) (aria secțiunii transversale )
=
×
(6.4)
=
∫ u d O (6.5)
TWIDELL PAGINARE.indb 210
01/12/2014 11:36
§6.3 Evaluarea resurselor
211
unde u este streamwise viteza (normal să elementare zona d- O ). Metodele de măsurare
referitoare la fiecare ecuație numim bază (6.3), rafinat (6.4) și sofisticate (6.5). O altă metodă poate fi
utilizată în cazul în care apa cade liber pe un pervaz sau weir.
Fig. 6.3
Măsurarea debitului de apa: (a) metoda de bază; (b) rafinat metoda (i); (c) rafinat
metoda (ii); (d) metode sofisticate; (e) weir metodă, a se vedea, de asemenea, Fig. 6.12.
Oprire ceas
Viteza u
Zona dO
d

x
h
Notch
Stream
Baraj
(e)
(d)
(c)
(b)
0
(a)
Butoi
Suprafata
Diversiune
țeavă
1
2
3
4
y
y
1
z
1
z
2
z
3
Stream
TWIDELL PAGINARE.indb 211
01/12/2014 11:36
212
Hidroenergie
(a)
Metoda de bază (Fig. 6.3(a)). Întregul flux este fie oprit de un baraj sau deviat într-o conțin
volum. În orice caz, este posibil pentru a măsura debitul de volum prins (6.3). Această
metodă face presupuneri despre flux, este exactă și este ideal pentru mici fluxuri, cum ar fi cele
de la o foarte mică cascadă.
(b)
Rafinat metoda (i) (Fig. 6.3(b)). Ecuația (6.4) definește viteza medie
u
a fluxului. Deoarece
viteza de curgere este zero pe partea de jos a fluxului (datorită vâscos fricțiune), viteza medie va fi
ușor mai mică decât viteza u
s
la suprafața superioară. Pentru o secțiune transversală dreptunghiulară, de exemplu, a
a fost constatat că
u
≈ 0.8u
s
. u
s

poate fi măsurată prin plasarea un flotor (de exemplu, o frunza) pe suprafața


de măsurare și timpul necesar pentru a merge la o anumită distanță de-a lungul pârâului. Pentru cele mai bune rezultate de
măsurare ar trebui să fie făcut în cazul în care fluxul este în mod rezonabil drept și uniforme în secțiune transversală.
Aria secțiunii transversale - O poate fi estimată prin măsurarea adâncimii la mai multe puncte de pe
stream și integrarea pe flux în mod obișnuit (Fig 6.3(b)):
O
yz
(y
y )(z
z)
(y
y )(z
z)
(y
y )z
1
211
1
2
2
1
1
2
1
2
3
2
2
3
1
2
4
3
3

+

+
+

+
+

(6.6)
(c)
Rafinat metoda (ii) (Fig. 6.3(c)). Un rafinament care evită nevoia de sincronizare exactă poate fi
util pe fast-curge fluxuri. Aici un flotor (de exemplu, o minge de tenis de masa) este eliberat de un standard adâncime
sub suprafața. Timpul pentru el să se ridice la suprafață este independent de la o mișcare pe orizontală și poate
fi ușor de calibrat în laborator. Măsurând distanța orizontală necesară pentru float să crească
oferă viteză în mod obișnuit. Mai mult decât atât, ceea ce este măsurat este în medie de viteză (deși medie de
peste profunzime, mai degrabă decât pe secțiune transversală: diferența este mică).
(d)
Metodă sofisticată (Fig. 6.3(d)). Aceasta este metoda cea mai precisă pentru mari fluxuri și este utilizat
de către profesioniști hidrologii. În esență, viteza de înaintare u este măsurată cu un debit mic metri la
punctele de o grilă bidimensională care se extind dincolo de flux. Integrala (6.5) este apoi evaluată
prin însumare.
(e)
Folosind un weir (Fig. 6.3(e)). Dacă Q este să fie măsurată de-a lungul anului pentru același flux,
măsurarea se poate face prin construirea unui baraj cu o formă specială de calibrare notch. Un astfel de baraj
este numit un weir. Înălțimea de curgere prin crestătura dă o măsură a debitului. Sistemul este
calibrat cu un model de laborator având aceeași formă de crestătură. Real calibrări sunt
prezentate în standard manuale. Problema 6.2 arată modul în care acestea sunt derivate.
§6.4 IMPULSE TURBINE
Impulse turbine sunt mai ușor de înțeles decât o reacție de turbine. Ne
considerăm mai întâi un anumit turbina cu impuls: la roată Pelton turbine
(Smochine 6.4 și 6.5).
Energia potențială a apei în rezervor este schimbat cinetică în
energie de una sau mai multe jeturi. Fiecare jet apoi lovește o serie de cupe sau cesti'
plasate pe perimetrul de o roata verticala, ca schițat în Fig. 6.4. În
rezultat deformarea de lichid constituie o schimbare în ritmul de
lichid. Cupa a exercitat o forță de lichid, și, prin urmare, fluidul are
, de asemenea, exercită o forță pe cupa. Acest tangențială forța aplicată la
volan provoacă-l pentru a roti.
Deși ideal eficiența turbinei este de 100%, în practică, valorile
gama de 50% pentru unități mici de 90% pentru prelucrate cu precizie mare
TWIDELL PAGINARE.indb 212
01/12/2014 11:36
§6.4 Impulse turbine
213
sistemele comerciale. Designul practic roata Pelton (schițat în
Fig. 6.4) are ca scop ideal de performanță descrise. De exemplu, duzele
sunt ajustate astfel încât jeturile de apă a lovit în mișcare cupe perpendicular la
optim viteza relativă maximă de transfer de impuls. Idealul
nu poate fi realizat în practică, deoarece o intrare jet-ar fi
deranjat atât de reflectat jet și prin următoarea ceașcă de tip revolving în loc.
Pelton au făcut mai multe îmbunătățiri în turbine din timpul său (1860), pentru a
depăși aceste dificultăți. Crestături în partea de sus a cupelor-a dat jeturile
acces mai bun la turbina cupe. Forma de cupe incorporate-un
splitter central secțiune, astfel încât jeturile de apă au fost reflectate de
apa de intrare.
6.4
Diagrama schematică a unei Pelton roata turbina cu impuls.
Sursa
Alimentarea
conductei
(area O) Jet
(zona un
j
)
Roata
Control
Cupa
volumul
Duza
Total
cap Sec
t
Disponibile
cap Sec
o
p = atmos.
p = atmos.
Fig. 6.5
MW scară roata Pelton turbine pe un singur ax, de conducere generator (dreapta). Acoperă
demontat pentru întreținere.
TWIDELL PAGINARE.indb 213
01/12/2014 11:36
214
Hidroenergie
DERIVAREA 6.1 PUTERE DE IEȘIRE ȘI DIMENSIUNI DE O TURBINA CU IMPULS
Fig. 6.6
Viteza de ceașcă și lichid: în (a) laborator de cadru; (b) cadrul cupei.
(a)
Jet de apă
Jet de apă
Cupa
(b)
Q
u
j
u
c
u
c
u
r1
= u
j
– u
c
u
r2
= u
2
– u
c
u
2
Fig. 6.6(a) arată un avion, de densitate
ρ și debitul volumic Q, lovind-o cupa cum se vede în "laborator"
(de exemplu pe pământ) cadru. Cupa se deplasează la dreapta cu viteză constantă u
c
și de intrare viteza de reacție este u
j
.
Fig. 6.6(b) prezinta un cadru de cupa cu relativă viteza jet u
j
− u
c

; deoarece lustruit cupa este buna,


frecarea este neglijabilă, și așa jet este deviat lin prin aproape 180°, cu nici o pierdere de viteză.
Forțele
Astfel, în cadrul cupei, schimbarea în impuls pe unitatea de timp, și, prin urmare, forța F cu experiență
de cupa, este
F
=2
ρQ
j
(u
j
− u
c
) (6.7)
(Această forță este în direcția jetului.) Puterea P
j
transferat la un singur pahar este
P
j
= Fu
c
=2
ρQ
j
(u
j
− u
c
)u
c
(6.8)
unde Q
j
este fluxul prin jet. Prin diferențiere cu privire la u
c
, aceasta este un maxim pentru constanta
u
j
, atunci când
uu
/
c
j
1
2
=
(6.9)
Deci înlocuind pe u
c
în (6.8):
P
Qu
j
jj
1
2
2
ρ
=
(6.10)
(de exemplu, puterea de ieșire este egal cu puterea de intrare, și acest ideal turbina are eficiență de 100%). Pentru acest ideal
cazul în laborator cadru, viteza de apa lăsând cupa are zero componentă în direcția
de avion. Prin urmare, apa de la orizontală jet cade vertical de la cupa.
Ideal eficiența poate fi de 100%, deoarece lichidul atenteaza la turbina într-un constrâns de intrare debit
(jet), și puteți să plecați cu o cale separată; acest lucru contrastează cu situații de extins flux (de exemplu, de vânt pe
o turbină eoliană), de unde poate fi extras de energie este limitată în mod semnificativ (a se vedea §8.3).
Jet velocity și duza de dimensiunea
indicată în Fig. 6.4, presiunea atmosferică, atât la partea de sus a conductei de transport și la jet. Deci,
din teorema lui Bernoulli (§R2.2) și ignorând frecare în conducte,
u
gH
2
j
2
t
=
. Cu toate acestea, conducta de frecare poate
fi incluse prin înlocuirea totală cap Sec
t
de disponibil cap Sec
o
definit prin (6.2), deci
u
gH
2
j
2
o
=
(6.11)
TWIDELL PAGINARE.indb 214
01/12/2014 11:36
§6.4 Impulse turbine
215
În practică, dimensiunea de conducte este ales astfel încât u
j
este independent de duza zona. Dacă există n
duze, fiecare zona o, atunci debitul total din toate jeturi este
Q
= nau
j
= nQ
j
(6.12)
Dacă eficiența de transformare jetul de apa, energie mecanică, energie de rotație este
η
m
,apoi
puterea mecanică de ieșire P
m
la turbina cu n jeturi, din (6.9) și (6.10),
P
nP
n
Qu
n
au u
na
gH
(
)(
)(
)(
)
(2
)
m
m
j
m
j
m
j
j
m
o
1
2
2
1
2
2
1
2
3/2
η
η
ρ
η
ρ
η
ρ
=
=
=
=
(6.13)
Acest lucru arată importanța de a obține suma maximă disponibilă cap Sec
o
între turbină și rezervor.
Puterea de ieșire este proporțională cu totalul jet aria secțiunii transversale O
= n
o. Cu toate acestea, o este limitată de
dimensiunea cupei, deci, dacă o este de a fi crescută, o mai mare turbina este nevoie. De obicei, este mai ușor pentru a crește
numărul
de duze n decât pentru a crește dimensiunea totală a turbinei, dar aranjamentul devine unworkably
complicat pentru n
≥ 4. Pentru roți mici, n
= 2 este cea mai comună.
Desigur, debitul total Q prin turbina nu poate fi mai mare decât debitul în flux Q
stream
. Folosind
(6.11) și (6.12),
na
Q
gHa
/ (2
)
j
stream
1/2

(6.14)
Viteza unghiulară și turbină de dimensiuni
Presupunem că am ales mărimea duzei și numărul în conformitate cu (6.12) și (6.13), pentru a da
puterea maximă disponibilă. Duza dimensiune a stabilit dimensiunea de cupe, dar nu dimensiunea totală a
roții. Acesta din urmă este determinată de constrângerile geometrice, și, de asemenea, de viteza de rotație. De
generare a energiei electrice, variabilele de ieșire (de exemplu, de tensiune, de frecvență și de eficiență) depinde unghiulară
viteza de generator. Cele mai multe generatoare electrice au cea mai mare eficiență în mare viteză de rotație
(frecvență), frecvent ~1500 rpm. Pentru a evita complicat și cu pierderi de transmisie, este important ca turbina
ar trebui să opereze la o astfel de mare viteza; la roată Pelton este deosebit de potrivit în acest sens.
Dacă roata are raza R și se transformă de la viteza unghiulară
ω, de (6.7) și (6.8),
P
= FR
ω (6.15)
Astfel, pentru o anumită putere de ieșire, cea mai mare roata cel mai mic viteza unghiulară. Deoarece u
c
= Rω, și
u
c
= 0.5 u
j
de (6.9), și folosind (6.11),
R
gH
0.5(2
)
o
1/2
ω
=
(6.16)
Duzele da, de obicei cu secțiune circulară jeturi de zona o și rază r.
Deci o
=
π r
2
și de
(6.13)
,
r
P
n gH
(
)
2
m
jm
o
3/2
ηρπ
=
√2
(6.17)
Combinarea (6.16) și (6.17), vom găsi
η
ω
ρ
η
=




=


rR
n
P
gH
n
/
0.68(
)
(
)
0.68(
)
m
1/2
m
1/2
1/2
o
5/ 4
m
1/2
S
(6.18)
TWIDELL PAGINARE.indb 215
01/12/2014 11:36
216
Hidroenergie
În (6.18)
S
este un non-dimensional măsură de condițiile de operare,
a sunat la numărul forma de turbina:
ω
ρ
=
P
gH
(
)
m
1/2
1/2
o
5/ 4
S
(6.19)
Astfel de non-dimensional factori sunt funcții puternice în inginerie (de exemplu,
permițându-rezultatele de măsurare și optimizare de laborator
modele fizice să fie aplicată la scară largă a plantelor).
Din (6.18), mecanice eficiența
η
m

în orice moment este o funcție de:


(i) geometrie fixă de un anumit roata Pelton (măsurată prin
nondimensional parametrii r/R și n), și (ii) non-dimensională de formă
numărul Modelului, care caracterizează condițiile de operare la acel moment.
EXEMPLUL 6.2
Determina dimensiunile unui singur jet roata Pelton de a dezvolta 160 kW
sub un cap de: (i) 81 m, și (ii) 5.0 m. Ce este viteza unghiulară de la care aceste roți va efectua
cel mai bine?
Soluție
Să presupunem că apa este fluidul de lucru. Să r fi duza rază și R raza roții. Este dificil să se
opereze o roată cu r > R/10, deoarece cupe ar fi atunci atât de mare încât acestea ar putea interfera cu fiecare
CASETA 6.2 'VITEZA SPECIFICĂ'
Feriți-vă! În loc de adimensional de forma număr de de (6.19), unele inginerie texte folosi o dim en -
sioned caracteristic numit 'specifice de viteză', N
s,
definit de variabilele P,
ν (= ω/2π) și H
o
:
N
P
H
s
o
1/2
5/ 4
ν
=
(6.20)
"Viteză specifică" nu include g și
ρ, deoarece acestea sunt în mod eficient constante. În Consecință, N
s

are
dimensiuni și unități, și așa, tulburator, sa valoare numerică depinde de anumite unități de folosit. În
practică, aceste unități variază între statele UNITE ale americii (de exemplu rpm, ax cai putere, ft) și Europa (de exemplu rpm,
metrice
de cai putere, m), cu o versiune standard pentru unități SI încă să devină comune.
Implicit în (6.18) este raportul dintre viteza de piese în mișcare
u
c
și viteza de jet u
j
. Dacă raportul u
c
/u
j

este aceeași pentru două roți


de dimensiuni diferite, dar aceeași formă, atunci întregul flux de model este
, de asemenea, același pentru ambele. Rezultă că toate non-dimensional măsuri
de performanță hidraulică, cum ar fi
η
m
și S, sunt aceleași pentru impuls
turbine cu același raport de u
c
/u
j

. Mai mult decât atât, pentru o anumită formă de


roată Pelton (specificat aici de r/R și n), există o anumită
combinație de condiții de funcționare (specificate prin S ) pentru o eficiență maximă.
TWIDELL PAGINARE.indb 216
01/12/2014 11:36
§6.5 Reacție turbine
217
Comparând cazurile (i) și (ii) în Exemplul 6.2, Pelton roți la low șefii
ar trebui să se rotească încet și au rază mare. Astfel de instalații ar
fi greoaie și costisitoare, mai ales că dimensiunea de cadru și
de locuințe crește cu dimensiunea de turbina. În practică, prin urmare, Pelton
roți sunt utilizate predominant pentru high-cap și relativ mici-fluxul de
instalații.
§6.5 REACȚIE TURBINE
Low-cap situații (6.1), necesita un flux mai mare de Q prin turbina
decât pentru high-cap. De asemenea, având în vedere forma număr de de (6.19),
pentru a menține același
ω și P cu mici H, avem nevoie de o turbina cu
mare S. De exemplu, prin creșterea numărului de duze pe un Pelton
roata: vedea (6.18) și Fig. 6.7(a). Cu toate acestea, conducte devine
în mod nejustificat complicat dacă n > 4, iar eficiența scade deoarece
multe jeturi de apă interfera cu fiecare alte.
Pentru a menține un flux mai mare printr-o turbina, modificările de design sunt
necesare, ca și în Francis reacție turbina de Smochine 6.7(b) și 6,7(c). În
efect, întreaga periferie a roții este de făcut într-o mare 'slot'
jet de apă să intre și apoi rândul său, într-un vortex de a împinge împotriva rotor
cu palete. Astfel de turbine sunt numite reacție mașini pentru lichidul
alte debit; prin urmare, presupunem că r
= R/12 (6.18), și din Fig. 6.8 cu condiții optime de funcționare
η
m
≈ 0.9, forma caracteristică număr în (6.18) devine:
S = 0.11
(i) Cu H
o
= 81 m, (6.17) din viteza unghiulară pentru cea mai bună performanță este
ω
ρ

=
×
=




gH
P
(
)
0.11(10 kgm ) [(9.8 ms )(81m)]
(16 10 W)
36 rad s
1
1/2
o
5/ 4
1/2
3
3 1/2
2
5/ 4
4
1/2
1
S
Din (6.11)
u
gH
(2
)
40 ms
j
o
1/2
1
=
=

Prin urmare:
ω
=
=
R
u/
0,55 m
1
2j
(ii) în mod Similar, cu H
o
= 5 m,
ω
ω
=
=
=
=


u
R
(5 / 81)
1.1 rad s
10 ms
4.5 m
2
1
5/ 4
1
j
1
TWIDELL PAGINARE.indb 217
01/12/2014 11:36
218
Hidroenergie
împinge (sau 'reactioneaza') continuu împotriva lame. Acest lucru contrastează cu
impulsul de mașini (de exemplu, Pelton roți), în cazul în care lamele (cupe) de a primi
o serie de impulsuri. Pentru o reacție turbina, roata, numit runner,
trebuie să fie adaptate astfel încât lichidul intră radial perpendicular pe
turbine cu axa, dar se întoarce și pleacă paralel cu această axă. În consecință,
viteza fluidului are o componentă radială în plus față tangențială
a vitezei, ceea ce complică analiza (a se vedea manualele în Bibliografia de la
sfârșitul acestui capitol).
O mai mare debit de apă pot fi obținute prin efectuarea de intrare a apei
jet aproape la fel de mare în secțiune transversală ca roata în sine. Acest concept
duce la o turbină în formă de elice, cu fluxul principal de-a lungul
axei de rotație (de ex. turbina Kaplan este prezentat în Fig. 6.7(d)).
Fig. 6.7
Metode de creștere a puterii de la o anumită dimensiune de mașină, care lucrează la aceeași
presiune a apei.
o
Un patru-jet roată Pelton, puterea de care este de patru ori mai mare decât cea dintr-o
onejet roata de aceeași dimensiune și viteză. (Pentru simplitate nu toate cupele sunt afișate.)
b
O turbină Francis vazut de sus; jeturile furnizarea apei în rotor acum există toate
în jurul circumferinței ca un slot.
c
Turbină Francis (cut-away vedere laterală) cu apă de la intrarea la presiune înaltă din
stânga și dreapta de intrare tuburi jurul axei verticale cu turbină, înainte de a intra în spațiile
dintre palete, și în scădere pe verticală în jos prin centru.
d
O elice (Kaplan) turbină; aici forma mare număr, S, se obține dacă jetul este făcut
aceeași dimensiune ca rotorul și nu există nici un flux radial de-a lungul rotorului.
Rotirea
paletele
Staționare
palete de ghidare
Scroll
caz
Afluxul
Pelton (impuls)
Francis (impuls)
(b)
(a)
Ghid de
palete
Palele turbinei
Turbină de
generator de arbore
(d) Kaplan (elice)
(c) Francis (vedere laterală)
Apa
fluxul
Ghid de
palete
Palele turbinei
Turbină de
generator de arbore
(d) Kaplan (elice)
(c) Francis (vedere laterală)
Apa
fluxul
TWIDELL PAGINARE.indb 218
01/12/2014 11:36
§6.5 Reacție turbine
219
În practică, un beneficiu de reacție turbine este considerabilă
modificare a presiunii într-un lichid ce trece prin carcasa, sigilat din
aerul exterior. Lui Bernoulli ecuația (R2.2) poate fi folosit pentru a arăta că
mai mici de apă presiunea din sistem va fi mult mai puțin decât
atmosferice. Într-adevăr, cea mai mică presiune poate fi chiar mai mică decât a vaporilor
sub presiune de apă. Dacă acest lucru se întâmplă, bulele de vapori de apă se va forma
în interiorul lichidului – un proces numit cavitație. În aval de aceasta,
presiunea apei ar putea crește brusc spre atmosferic, astfel provocând
bula să se prăbușească. Rezultate vigoare de la năvală de apă lichidă
poate provoca considerabile deteriorări mecanice din apropiere piese mecanice.
Aceste efecte crește cu viteza de curgere și capul, și așa axial masini
sunt limitate în practică a scăzut Sec. În plus, performanța de reacție
turbine, în general, și turbina cu elice în special, este foarte
sensibil la schimbările în rata de curgere. Eficiența scade rapid dacă debitul
scade, deoarece mai lent fluxul nu mai lovește lama la
unghiul corect. Este posibil pentru a permite acest lucru prin ajustarea automată
lama unghi, dar este complicat și scump. Elice,
turbine cu automat reglabil lama de teren au fost din punct de vedere istoric
considerat util doar în instalații de mari dimensiuni (de exemplu, Kaplan, turbina).
Cu toate acestea, mai mici elice, turbine cu lamele reglabile sunt acum
disponibile în comerț la scară mică operație.
Funcționarea unui Pelton roata nu este atât de sensibil la condițiile de curgere
ca o elice turbina.
Ca un ghid pentru alegerea corespunzătoare turbina pentru dat Q și H,
Fig. 6.8 prezinta gama de forma număr de peste care este posibil
de a construi un sistem eficient de turbina. În plus, pentru fiecare tip de turbina nu
va fi o relație între numărul forma S (caracterizarea
condiții de operare în care turbina realizează cel mai bine) și un alt
non-dimensional parametru ce caracterizează forma de turbina. Un
astfel de parametru este raportul r/R de (6.18). Fiind non-dimensional, aceste
Fig. 6.8
Ilustrativ vârf de eficiență, aici fiind cuprinse între 85% și 95%, de turbine diverse
tipuri, în raport cu forma numărul.
Sursa: Adaptat de la Çengel și Cimbala (2010).
0
100
90
Francis
Elice
Pelton roata (1, 2, 3, 4 jeturi)
Banki
1234
80
1
2
3
4
5
η
/%
TWIDELL PAGINARE.indb 219
01/12/2014 11:36
220
Hidroenergie
relații pot fi stabilite atât teoretic, cât și experimental,
și sunt folosite pentru a optimiza design. Detalii sunt prezentate în recomandat
de texte și manuale de inginerie de la sfârșitul acestui capitol.
§6.6
HIDROELECTRICE, SISTEME DE
O majoritate covârșitoare, hidroenergie generează energie electrică, deși foarte
ocazional apă-mills, lifturi hidraulice și ram pompe utile
puterii mecanice (a se vedea referințele de la sfârșitul acestui capitol). Toate
hidroelectrice sisteme, chiar dacă la scară mare (ca în Fig. 6.2 sau Caseta 6.3) sau mică
scară (Fig. 6.9), trebuie să includă o sursă de apă, delimitată de presiune a conductei
(conductă forțată), controlul fluxului, turbină, generator electric, electrice, de control,
și reticulare (cabluri și cabluri) de distribuție a energiei electrice.
La barajul asigură o aprovizionare constantă de apă, fără fluctuații, și,
cel mai important, permite stocarea energiei în rezervor. De asemenea, poate
oferi beneficii decât alte generatoare de energie electrică (de exemplu, controlul inundațiilor, apa
de alimentare, un drum de trecere). Mici run-of-the-râul sisteme de la un mod rezonabil
flux constant poate solicita doar un zid de sprijin de înălțime redusă (de exemplu, suficient
pentru a menține stăvilar pe deplin cufundat), dar acest lucru nu oferă stocare.
Conducta de alimentare (stăvilar) este, de obicei, un relativ construcții majore
de cost. Este mai ieftin dacă cu pereți subțiri, scurte și de diametru mic; dar aceste
condiții sunt rar posibile. În special diametrul D nu poate fi
acoperit din cauza excesive pierdere cap Sec
f
∝D
-5

(a se vedea Problema 6.7). Cea mai mare


costul de o mai mare conductei trebuie să fie comparată cu pierderea continuă de putere
folosind o țeavă mică. Un compromis comun este de a face H
f
≤ 0.1 Sec
t
. Pentru
sistemele mai mari, pe 'teava' poate include tuneluri subterane.
Material de conductă forțată trebuie să fie atât de netedă (pentru a reduce
frecare) și puternic (pentru a rezista la presiunile statice, și considerabil
Fig. 6.9
Aspect de micro hidrocentrale de sistem folosind o roată Pelton. Rețineți că această diagramă nu
nu indica apa în cap Sec necesară.
Electric
generator
Centura
Turbina
Duza
Baraj
Țeavă
Supapa
Ecran
Apa
TWIDELL PAGINARE.indb 220
01/12/2014 11:36
§6.6 sisteme Hidroelectrice
221
mai dinamic, cu ciocan de apă' presiunile la schimbări bruște de debit).
Pentru instalațiile mici, din plastic PVC este potrivit pentru principalele lungimea
conductei, poate cu o scurtă de oțel de secțiune în partea de jos pentru a rezista la
presiuni mai mari acolo. Un ecran este necesară în partea de sus de pe conducta de alimentare a
intercepta gunoi (de exemplu, frunze) înainte de a se blochează țeavă. Acest ecran trebuie să
fie verificate în mod regulat și curățată de moloz; cu toate acestea, utilizand ecrane
Fig. 6.10
Cele Trei Chei baraj hidroelectric în China: cel mai mare din lume. A se vedea Caseta 6.3 pentru o
descriere detaliată.
CASETA 6.3
BARAJUL CELOR TREI DEFILEURI, YANGTZE, PROVINCIA HUBEI, CHINA: LUMEA E
CEA MAI MARE CENTRALA DE INSTALARE
Finalizat în 2012, barajul este de 2335 m lungime și se ridică 181 m deasupra albiei (rock). Nivelul normal
de apa din baraj este de 170 m deasupra debit scăzut nivel (vezi Fig. 6.10). De captare a zonei ~1.000.000 de km
2
și se extinde ~600 km în amonte. Rezervor de apă de suprafață zona de ~1000 de km
2
.
Încorporat în baraj sunt 32 de turbine Francis, fiecare 700 MW (cu diametrul de 10 m; cap 81 m; debit max
950 m
3
/s). Există un generator pe turbine, fiecare cu o putere nominală de 700 MW la 20 kV, maxim
generator electric eficiența 96.5%. Prin urmare, capacitatea electrică totală instalată este de 22.500 de MW (de exemplu,
≈30%
din marea BRITANIE capacitatea totală a tuturor formelor de energie electrică).
Putere de ieșire depinde de debitul râului, care variază puternic cu sezonul. De ieșire este de obicei
≤ 5000
MW în noiembrie pentru a Putea sezon uscat; când nu este suficient de curgere (de obicei iulie-septembrie)
puterea de ieșire este limitată de plante-capacitate de generare a 22.500 de MW. Anual estimat generație
de ieșire este de 100 TWh, ceea ce înseamnă un factor de capacitate de (100 X 10
3
GWh/ 8760 h)/(22.5 GW)
= 50%.
Alte beneficii: controlul inundațiilor, pe tot parcursul anului de transport maritim și barja de trafic de mai sus de baraj, ascensoare
pentru nave (încuietori), alături de
baraj; 190 milioane t/an de carbon fosil potențial redus în comparație cu clădirea centrală pe cărbune și 10 milioane de tone/an
cu ajutorul transportului fluvial în loc de drum.
Efectele Negative includ: 1300 milioane de persoane strămutate; 1300 situri arheologice s-au înecat (cu toate acestea,
unele au fost repoziționate); îndepărtarea mâlului crește probabilitatea de inundare în aval; construit pe un
cutremur regiune (risc potențial imens daca se rupe barajul).
Sursa: http://en.wikipedia.org/wiki/Three_Gorges_Dam.
TWIDELL PAGINARE.indb 221
01/12/2014 11:36

222
Hidroenergie
pe efect Coanda (tendința unui jet fluid de a fi atras de o apropiere
de suprafață) sunt mai puțin supărătoare.
Sistemele mici (~10 kW) folosesc, în general, off-the-shelf generatoare sau
motoare asincrone funcționează ca generatoare (a se vedea §R1.6). Dacă turbina
viteza nu este suficient de mare pentru a se potrivi generator, atunci transmisie este folosit.
Un V-belt este o comună gearing mecanism, care, din păcate, pot da
pierderi de putere de la 10 la 20% în foarte mică de sisteme. Sistemele mari de obicei
au mai multe turbine, fiecare cu una sau mai multe scop-construit generatoare
de funcționare de la același ax ca turbina, care reduce atât
costurile de construcție și pierderile de putere.
§6.6.1 Puterea de reglementare și control: grid-connected
Cu orice hidro instalare alimentarea de energie electrică într-un utilitar de transmisie de
rețea, este important ca tensiunea și frecvența de ieșire meci
că de restul rețelei. Deși primare generație este întotdeauna la
un nivel relativ joasă tensiune, cu tensiune de curent ALTERNATIV poate fi ușor crescut de
transformatoare, atât pentru a se potrivi cu tensiunea de rețea și pentru a minimiza am
2

R pierderi în
transmisie. Este important ca tensiunea și frecvența fie controlate
pentru a menține standarde comune și electrice cerințe de dispozitiv. (A se vedea
discuțiile legate de rețelele de electricitate (§15.4), eoliene conectate la rețea
putere (§8.8) și sistemul de control al principiilor (§1.5.3.)
Cu hidroenergie, acest lucru se face în mod tradițional prin procedee mecanice feedback-ul
sistemelor de control al fluxului prin turbină, astfel încât se menține
constantă frecvența ("viteza"). De exemplu, cu o roată Pelton, o suliță
supapa este făcut să se mute în și la ieșirea din duză (așa cum este indicat în Fig. 6.9),
care reglementează astfel Q. Pentru turbinele cu elice ar putea fi posibil pentru a regla
unghiurile lamă în plus față de rata de curgere. Toate aceste sisteme mecanice
sunt relativ complicate și costisitoare, mai ales pentru scară mai mică
aplicație.
Hidroelectrice sisteme, odată turbinele și generatoarele sunt stabilite
de tensiune și frecvență, sunt ușor să se integreze într-o rețea electrică.
Într-adevăr, ei au mai multe avantaje semnificative fata de alte generatoare de
sisteme:
1
Dacă zona de captare, precipitații și îndiguite rezervor sunt suficient de mari
(sau debitul râului este foarte consistent), aproximativ constantă, puterea poate fi
generat zi și noapte ca 'importanta' de plante.
2
Cu mai puțin de disponibilitatea apei, stocate hidroenergetic este probabil să fie mai
valoros ca "de vârf" de putere de utilitate atunci când cererea este mare (de exemplu dimineața
și seara). Rapid timp de pornire (~1 minut) și relativ ușor
de control sunt cele mai benefice în comparație cu producerea de energie termică
a plantelor.
3
Ele oferă flexibilitate în operare, începe generatoare cu un preaviz foarte scurt
și cu minim de cost de start-up, și de a oferi schimbări rapide în generată
de putere într-o gamă largă menținând în același timp excelent full si part-load
TWIDELL PAGINARE.indb 222
01/12/2014 11:36
§6.6 sisteme Hidroelectrice
223
eficiența. Într-un nationala de electricitate în rețea, pompat
hidroelectrice de stocare (§6.7) este un avantaj pentru inflexibil putere de plante, cum ar fi
energia nucleară, și variabile de plante, cum ar fi energia eoliană.
§6.6.2 Puterea de reglementare și control: sisteme stand-alone
Autonom, autonom, hidro sisteme (de exemplu, pentru alimentarea cu energie electrică de la un
sat sau ferma), de asemenea, nevoie de viteză și de putere regulament; cu toate acestea,
dispozitivele de putere, cum ar fi lumini și motoare electrice de mici dimensiuni în
frigidere, în general tolera variații ale tensiunii și frecvenței de ±10%.
Mai mult decât atât curenții implicate sunt usor de trecut de electronice de putere
dispozitivele, cum ar fi tiristoare. Acest lucru vă oferă posibilitatea de a o mult mai ieftin
de control decât convenționale sistemele mecanice.
Cu o sarcină electronică de control sistem, modificări majore în ieșire se
realizează prin comutarea manuală duze complet în sau în afara, sau de
manual de control debitul total prin turbina. Control mai bun este
realizată de un dispozitiv electronic de feed-forward control care împărtășește de ieșire
a generatorului între principalele sarcini (de exemplu, casa de lumini) și un balast
(sau 'off-peak') circuit de încălzire care pot tolera o variabilă sau intermitent
de alimentare (vezi Fig. 1.4(d)). Generatorul astfel vede întotdeauna o constantă totale
de sarcină (
= principal + balast); prin urmare, se poate rula la putere constantă de ieșire, și
așa prea pot turbina de la care vine puterea. Fluxul prin
turbina, prin urmare, nu trebuie să fie în permanență în mod automat
ajustat, care simplifică foarte mult construcția sa.
§6.6.3 eficiența Sistemului
Chiar dacă eficiența fiecărui individ puterea de transformare pas
este mare, nu există încă o semnificativă pierdere de energie în trecerea de la original
potențialul de putere P
0
de apă, electrice de ieșire P
e

de la
generator, prin mecanică (m), generarea de energie electrică (g) și
de transmisie (t) etape. Aceste considerații se aplică, desigur, toate formele de
generare a energiei. Având în vedere succesive transformări de energie,
aproximativ sisteme eficiențe poate fi:
Mici sisteme autonome:
P
P
(0.8)(0.8)(0.8) 0.5
gt
e
0
m
ηη
=


η
(6.21)
Mare a sistemelor de utilități:
ηη
=


η

P
P
(0.95)(0.95)(0.95) 0.86
gt
e
0
m

(6.22)
TWIDELL PAGINARE.indb 223
01/12/2014 11:36
224
Hidroenergie
§6.6.4 domeniul de Aplicare pentru upgrade-uri de tehnologie
Moderne de mare hidroelectrice turbine sunt aproape de limita teoretică pentru
eficiență, cu până la 96% eficiență posibilă în condiții optime,
dar a continuat cercetarea este necesară pentru funcționarea eficientă pe o mai largă
gamă de debite. În vârstă de turbine poate fi mai mic randament din cauza
inadecvate de proiectare sau de eficiența redusă din cauza la coroziune și cavitație
daune.
Potențial, prin urmare, există pentru a crește producția de energie prin modernizarea cu
echipamente noi a îmbunătățit eficiența și poate crește capacitatea.
De exemplu, o estimare pentru statele UNITE ale americii este că o creștere de 6% în ieșire
(TWh/an) ar putea fi realizat la îmbunătățirea eficienței în cazul hidrocentralelor,
fabricate în anii 1970 sau anii anteriori, având o capacitate totală de 30 GW, sunt
înlocuite (Kumar et al. 2011).
Nu este mult de cercetare în curs de desfășurare cu scopul de a extinde raza de operare
in conditii de cap și de descărcare de gestiune, și, de asemenea, la îmbunătățirea mediului
de performanță și fiabilitate și de a reduce costurile. Cele mai multe dintre noile tehnologii
în curs de dezvoltare vizează utilizarea low-cap (<15 m) sau foarte mici-cap
(<5 m) site-uri, astfel încât să permită mai multe site-uri pentru hidroenergie.
Computational fluid dynamics (CFD) facilitează turbina design de înaltă
eficiență pe o gamă largă de debite. Există, de asemenea, domeniul de aplicare pentru
hydrokinetic dispozitive concepute inițial pentru a mareelor aburi putere (Capitolul 12) pentru a
fi utilizate în interiorul atât de liber-râuri curgătoare și în inginerie navigabile,
cum ar fi canale și tailraces existente de alimentare cu apă, baraje. Aceste
evoluții pot crește în mod semnificativ potențialul tehnic pentru hidroenergie
în unele țări. De exemplu, în 2004 potențialul de cost-eficiente
nou mini-hidro (de exemplu, <10 MW), în Norvegia a fost calculat a fi de ~25 TWh/an
(T. Jensen, www.HydroWorld.com, accesat la 31 August, 2012).
§6.7 POMPAT DE STOCARE A ENERGIEI HIDRO
Pentru furnizarea de utilități companii, hidroelectricitate oferă un extrem de
flexibil și de încredere metodă de generare a energiei electrice, limitată doar de lipsa
de precipitații. Caracteristica esențială este că puterea poate fi crescut sau a scăzut
rapid, în decurs de câteva secunde pentru a ajusta echilibrul de putere pe o grilă. Dacă
hidroenergetic este deconectat, acesta poate fi adus pe deplin on-line în câteva minute de la
un 'start'. Dacă este deconectat, nici o resursă irosită.
Un alt beneficiu de hidroenergie este că un sistem alimentat de apă
într-un rezervor și de alimentare cu apă într-un râu sau lac poate fi inversat.
În acest fel, excesul de putere în rețea (de exemplu, de la parcurile eoliene și la
noapte de la centralele nucleare) pot fi folosite pentru a pompa apa de sus a
rezervorului. Mai târziu, când vârful este nevoie de energie electrică, apă pot fi
returnate la vale, pentru a genera necesare "extra" de putere. De multe ori
aceleași mașini sunt utilizate ca ambele turbine și pompe. Aceasta este o formă
de stocare a energiei electrice – dar, de obicei, pe o scară mult mai mare decât alte
forme de stocare electric (a se vedea §15.6). Fig. 6.11 prezinta structura
TWIDELL PAGINARE.indb 224
01/12/2014 11:36
§6.8 aspectele Sociale și de mediu
225
un astfel de sistem. Raportul dintre energia de energie în (de exemplu, eficiența de
stocare) este de aproximativ 70%.
Partea de sus a rezervorului de acumulare prin pompare schema poate (a) să accepte precipitații
într-un bazin hidrografic (ca cu convenționale hidroenergie), și (b) de a primi
de apă pompat de la cea mai mică 'rezervor'. De obicei, (b) domină. Astfel
energie electrică de la pompat componentă de stocare nu ar trebui să fie
tratate ca surse regenerabile de energie, cum ar fi; primar generarea de etichete
clasificare (de exemplu, dacă din fermele eoliene apoi regenerabile, dacă de la energia nucleară
și/sau de combustibili fosili, atunci non-regenerabile). Astfel carbon de reducere a emisiilor de
pompat de stocare nu este evident, și de eficiența de stocare este
de aproximativ 70 la 80% la cele mai multe, aproximativ 20 la 30% din energia de intrare este irosit.
De pompaj este un utilizator net de energie electrică (necesită energie electrică
pentru a pompa apa la cel mai mare rezervor de stocare), depinde puternic
diferențele în prețul de piață al energiei electrice, între low și de vârf de
cerere, pentru viabilitatea sa financiară.
§6.8 ASPECTELE SOCIALE ȘI DE MEDIU
Hidroenergia este o tehnologie matură, în întreaga lume. Aproximativ 16% din
energia electrică este energia hidroelectrică; peste 90% în Norvegia. Hidrocentrala nu este
din punct de vedere termic a subliniat și operează în mod constant; prin urmare, este de lungă durată, cu
relativ reduse de întreținere cerințe: multe sisteme, atât mari și
mici, au fost folosite continuu timp de peste 50 de ani și ceva mai devreme
instalații funcționa în continuare după 100 de ani. Relativ mare de capital inițial
Fig. 6.11
Aspectul tipic al unei pompat hidro sistem de stocare a energiei.
De comutație
la grila
Principalul tunel de acces
Val camera
Nivelul apei mai jos
Turbina/pompa de camera
Întrerupătoare
Transformator seif
Rezervor
Aportul
TWIDELL PAGINARE.indb 225
01/12/2014 11:36
226
Hidroenergie
costul a fost mult de scris off, cu nivelat' costul de
energie electrică produsă (de exemplu, costul per kWh în medie pe durata de viață a sistemului)
mult mai puțin decât alte surse, în special în centrala termică care necesită
cheltuieli pentru combustibil și înlocuirea mai frecventă a mașinilor. Dacă
costurile externe sunt interiorizate (a se vedea Capitolul 17), non-surse regenerabile
deveni chiar mai scump. Pentru o hidrocentrală cu o rezervă suficientă de
apă, debitul poate fi controlat pentru a produce fie o bază sau rapid
ajungând putere ca a cerut; dacă alimentarea cu apă este limitată, atunci vânzarea de
energie electrică la numai vârfului de cerere este ușor și cel mai profitabil. Cu toate acestea,
capitalul inițial costul de hidroenergie este întotdeauna relativ mare, astfel încât acesta a
fost observat că toți producătorii de energie doresc au investit în
hidrocentrale acum 20 de ani, dar, din păcate, nu își poate permite să facă acum – și
- au spus acelasi lucru acum 20 de ani!'
Complicațiile sistemele hidroenergetice apărea în cea mai mare parte de
asociat baraje și lacuri de acumulare, în special pe proiecte de mare anvergură. Cele mai multe
râuri, inclusiv râuri mari cu continental-scara bazinelor hidrografice, cum ar fi
Nilul, Zambesi și Yangtze, au mari fluxurile de sezon, ceea ce face
inundațiile o caracteristică majoră. Prin urmare, cele mai mari baraje (adică acelea >15 m
de mare) sunt construite pentru mai multe scopuri: generarea de energie electrică, apă
potabilă de irigare și alimentare, controlul debitului râului, atenuarea viiturilor,
și furnizarea de punctele de trecere rutiere, activități de agrement, de pescuit, etc. Socială și
dezvoltarea economică necesită întotdeauna de energie electrică și de alimentare cu apă,
astfel de proiecte de mare anvergură apel către politicieni și finanțatorii care solicită
centralizat național de dezvoltare, care este din punct de vedere conceptual și administrativ
"simplu". Cu toate acestea, investiții enorme și impactul pe scară largă
a energiei hidroelectrice au făcut baraje mari aprig disputate probleme în
dezvoltarea durabilă (Comisia Mondială a Barajelor 2000). Combaterea
beneficii mari hidro, menționate mai sus, sunt, cu siguranță, efecte adverse;
exemple sunt povara datoriei (baraje sunt de multe ori cea mai mare investiție unică
proiect într-o țară), depășiri de costuri, de deplasare și de sărăcirea
oamenilor, distrugerea ecosistemelor și a resurselor de pescuit, și
inechitabil de partajare a costurilor și beneficiilor. De exemplu, peste un milion de oameni
au fost strămutați de construcția barajului celor Trei Defileuri din China
(Caseta 6.3), care are o capacitate de peste 22500 MW; totuși, aceste strămutate
oameni ar putea să nu ia în considerare faptul că acestea sunt, pe sold, beneficiari a
crescut capacitatea de putere și industrializare. Unele diguri au fost
construite pe notorietate nămol-laden râuri, ducând la depleția de
volum rezervor. Diverse marile agenții de finanțare (inclusiv Banca Mondială)
și grupuri de părți interesate (de exemplu, UNEP, IEA, IHA) au urmat Lume
Comisia a Barajelor, prin dezvoltarea propriilor orientări privind
dezvoltarea durabilă, care să sperăm că va limita astfel de greșeli în viitor.
Hidroenergie, ca toate sursele regenerabile de energie, reduce emisiile
de gaze cu efect de seră CO
2

prin înlocuirea combustibililor fosili, care altfel ar


fi fost folosit. Cu toate acestea, în unele proiecte de baraje, într-un efort de a salva
timpul de construcție și de cost, vegetația în putrefacție (mai ales copaci) a fost
TWIDELL PAGINARE.indb 226
01/12/2014 11:36
§6.8 aspectele Sociale și de mediu
227
a lăsat în loc ca barajul se umple, ceea ce duce la emisii semnificative
de metan, un alt gaz cu efect de seră. Chiar și așa, aproape toate estimările de
ciclul de viață gaze cu efect de seră (GES) impactul sistemele hidroenergetice sunt
mai puțin de 40 gCO
2
-eq/ kWh, care este un ordin de mărime mai mică decât pentru
fosili sisteme (vezi tabelul D3 în Anexa D).
În multe țări industrializate din punct de vedere tehnic cele mai atractive site-uri
au fost dezvoltate de zeci de ani în urmă și deci construirea de baraje de mari dimensiuni a toate
, dar a încetat. Mai mult decât atât, în statele UNITE ale americii, baraje au fost dezafectate
, pentru a permite creșterea ecologică a beneficia de protecția mediului flux prin
ecosistemelor din aval. Încă hidroelectrice de capacitate poate fi mărită
prin adăugarea de turbine generatoare pentru alimentarea cu apă a rezervoarelor și, mai vechi
hidrocentrale, instalarea de turbine suplimentare și/sau înlocuirea vechi
turbine cu mai eficiente sau de capacitate mai mare instalație modernă. Astfel de
evoluții au un impact pozitiv asupra mediului, cu nr nou negativ
de impact, și este un exemplu de utilizare a unui altfel de 'pierdut' fluxul de energie
(cf. §1.4). De asemenea, instalarea de mic 'run-de-râu' hidroelectrice
sisteme, doar cu foarte mici baraje, în general, este considerat un rezultat pozitiv
de dezvoltare; de exemplu, producția de astfel de sisteme în China este mai mare
decât totalul de hidrocentrale de capacitate de cele mai multe alte țări.
REZUMAT CAPITOL
Hidroenergia este cea mai stabilite, utilizate pe scară largă și de lungă durată de resurse regenerabile pentru energie electrică
generație. Acesta furnizează aproximativ 16% de la nivel mondial de energie electrică. Sistemele hidroenergetice în utilizare
variază de la foarte
mare (~GW) la foarte mic (~kW) capacitate.
Hidroenergie necesită topografie și precipitații care pot oferi suficiente debitul de apă Q și toamna (cap Sec).
O prima estimare de putere potențial disponibil la un site-ul este
P
0
=
ρQgH
în cazul în care
ρ este densitatea apei și g este accelerația gravitațională.
Sistemele hidroenergetice excelent de eficiență energetică (până la 95%, apă pentru sârmă de mare de plante comerciale).
Relativ scump investiția inițială este compensat de durată lungă de viață (turbine și generatoare de ~40 y,
baraje >100 y), împreună cu low-costul de exploatare și întreținere.
Turbinele sunt de două tipuri: (un) impuls de turbine, în cazul în care fluxul este turbina ca un avion într-o deschide
mediu, cu puterea care derivă din energia cinetică a fluxului; și (b) reacție de turbine,
în cazul în care turbina este complet încorporat în lichid și alimentat de la o cădere de presiune pe dispozitiv.
Impulse turbine sunt de obicei folosite atunci când H este mare, chiar dacă Q este scăzut. Reacția turbine sunt de obicei atunci
când H este
relativ scăzut. Alegerea este guvernat de un non-dimensională de formă numărul de forma
ω
ρ
=
P
gH
(
)
1/2
1/2
5/ 4
S
.
Toate hidroelectrice sisteme trebuie să includă o sursă de apă, delimitată de presiune a conductei (conductă forțată), controlul
fluxului,
turbină, generator electric, electrice, de control, și reticulare (cabluri și cabluri) de distribuție a energiei electrice.
Barajul asigură o aprovizionare constantă de apă, fără fluctuații, și, cel mai important, permite energiei
TWIDELL PAGINARE.indb 227
01/12/2014 11:36
228
Hidroenergie
ÎNTREBĂRI RAPIDE
Notă: Răspunsurile la aceste întrebări sunt în textul de la secțiunea relevantă
din prezentul capitol, sau poate fi ușor dedusă din aceasta.
1
Apa dintr-un rezervor curge de la 2 m
3
/s pe o netedă și verticală
stăvilar țeavă de lungime 10 m înainte de intrarea în turbină generator de
eficiență globală 80%. Ce este puterea maximă generată?
2
Ce factori afectează lungimea, diametrul și materialul de un stăvilar
țeavă?
3
Ce este o diferență esențială între formă numărul și
viteza specifică de hidro turbina?
4
Explica diferența în modul de funcționare între un impuls
de turbine și o turbină de reacție.
5
Explica pe scurt 'cavitație' care pot apărea într-o turbină de reacție.
6
Hidroelectrice se poate apropia de 100% eficiență, întrucât
producerea de energie termică este de aproximativ 30 până la 45%. De ce există o astfel de
diferență?
7
Listă de cel puțin trei avantaje ale centralelor hidroelectrice electric
de putere de utilitate.
8
Listă de cel puțin trei comune impact negativ asupra mediului de
hidroenergie.
9
De ce utilitati electricitate în Norvegia se tem de o iarnă foarte rece?
10
Care tehnologii generatoare de energie electrică sunt cele mai susceptibile de a beneficia
de stocare cu pompe pe o rețea națională, și de ce?
PROBLEME
Notă: *indică o "problemă", care este deosebit de potrivit pentru clasa de
discuție sau grupul de tutoriale.
*6.1
Utilizați un atlas pentru a estima potentialul hidro din țara dumneavoastră sau
de stat, după cum urmează:
(o) Numesc locul în cauză X. ceea Ce este cea mai mică altitudine din X?
Ce zona X se află mai mult de 300 de metri deasupra celui mai jos
depozitare in rezervor. Acesta poate oferi, de asemenea, beneficii decât alte generatoare de energie electrică (de exemplu,
controlul inundațiilor,
aprovizionarea cu apă, un drum de trecere).
Pompat hidro sisteme sunt utilizate de către utilități de mari dimensiuni de stocare a energiei. Excesul de putere pe grilă la
un moment dat de cererea scăzută este folosită pentru a pompa apa în sus într-un rezervor. Mai târziu, când cererea de energie
electrică este mai mare,
apa este returnat la vale, pentru a genera necesare "extra" de putere.
La dezavantaje majore de hidroenergie sunt asociate cu alte efecte decât cele generatoare de
echipamente, în special pentru sistemele mari. Acestea includ posibil impact negativ asupra mediului de
efectele pe pește, colmatarea barajelor, coroziune de turbine în anumite condiții de apă, impactul social al
deplasarea de persoane din rezervor site-ul, și pierderea de potențial productiv a terenurilor. În consecință,
rolul de baraje de mari dimensiuni în promovarea dezvoltării durabile este aprig contestată.
TWIDELL PAGINARE.indb 228
01/12/2014 11:36
Probleme
229
nivel? Cât de mult ploaia cade pe an pe această parte de mare de X?
Ce-ar fi potențialul de energie pe an, având în vedere de către această
masă de apă, dacă tot a fugit în jos la cel mai mic nivel? Expres
acest lucru în megawați.
(b) Rafina această putere de estimare care să permită următoarele:
(i) nu toată ploaia care cade apare ca scurgerile de suprafață; (ii) nu
toate scurgerile apare în fluxuri care sunt în valoare de îndiguiri;
(iii) dacă coborârea este la o adâncime prea mica panta, conducte dificultăți
limita disponibil cap.
(c) Dacă o centrală hidroelectrică a fost instalat la X, compara
răspunsul tău cu o capacitate instalată de X, și comentariu la
orice diferențe mari.
6.2
Curgerea peste un U-weir poate fi idealizat în forma prezentată în
Fig. 6.12. În regiunea 1, înainte de weir, fluxul viteza u
1

este
uniformă cu adâncimea. În regiunea 2, după weir, viteza de curent
crește cu adâncimea h în apă.
(a) să Utilizeze teorema lui Bernoulli pentru a arăta că, pentru eficientizarea
trecand peste weir, la adâncimea h sub suprafata,
u
g
hu
g
(2 ) (
/2)
h
1/2
1
2
1/2
=
+
Sugestii:
presupunem că p
h
în apă
= presiunea atmosferică,
deoarece aceasta este presiunea de deasupra și de sub apă. Presupune
, de asemenea, că u
1
este suficient de mic ca p
1
este hidrostatică.
(b) prin Urmare, arată că de descărcare de gestiune de-a lungul idealizat weir este
Q
g
LH
(8 / 9)
th
1/2
3/2
=
(c) Prin experiment, efective de descărcare de gestiune este dovedit a fi
Q
exp
= C
w
Q
th
6.12
Un U-weir: (a) față de elevație; (b) vedere laterală a idealizat flux (u
h

este viteza de apa


de-a lungul weir în cazul în care presiunea este p
h

).
H
L'
b
Aer p = p
o
Regiunea 2
Regiunea 1
(a)
(b)
Aer (p = p
o
)
Am
h
u
h
u
1
TWIDELL PAGINARE.indb 229
01/12/2014 11:36
230
Hidroenergie
în cazul în care
C
w
≈ 0.6. (Valoarea precisă a C
w
variază cu H/L' și
L/b.) Explica de ce C
w
<1.
(d) se Calculează Q
exp
pentru cazul L'
= 0,3 m, L = l m, b = 4 m, H = 0.2
m. Calcula, de asemenea, u
1
și justifica ipotezele despre u
1
folosit
în (a) și (b).
6.3
Verificați dacă S definit prin (6.19) este adimensional. Care sunt
avantajele de a prezenta datele de performanță pentru turbine în
formă adimensională?
6.4
O elice turbina a forma numărul S = 4 și produce 100 kW
(mecanic) la un cap de lucru de 6 m. Eficiența sa este de aproximativ 70%.
Calcula:
(a) debitul.
(b) viteza unghiulară a arborelui.
(c) raportul De transmisie necesară în cazul în care arborele este de a conduce un patru poli alter-
nator pentru a produce un echilibru de 50 Hz.
6.5
O roată Pelton cupa este atât de în formă ca ieșirea fluxului face un unghi
θ cu incidentul jet, așa cum se vede în cupa cadru. Așa cum se arată în
Fig. 6.6, u
c

este viteza tangențială de cupa, măsurată în


laborator cadru. Energia pierdută prin frecarea dintre apă
și cana pot fi măsurate printr-o pierdere coeficient k , astfel încât
u
u
k
(1
)
r
r
1
2
2
2
=
+
Arată că puterea transferată este
PQuuu
k
(
)1
pentru că
(1
)
c
j
c
ρ
θ
=

+
√+




Obține eficiență mecanică
η
m.
Ce este scăderea eficienței de ideal atunci cand
θ = 7°,
k
= 0.1? Ce este unghiul de deviere văzut în laborator
cadru?
6.6
O roată Pelton este de a fi instalat într-un site cu H
= 20 m, Q
min
=
0,05 m
3
s
-1
.
(a) Neglijând frecarea, găsim: (i) viteza cu jet; (ii) valoarea maximă
putere disponibilă, și (iii) raza de duze (presupunând că
există două duze).
(b) Presupunând că roata a forma numărul
P
gH
(
)
0.1
1
1/2
1/2
5/ 4
ω
ρ
=
=
S
TWIDELL PAGINARE.indb 230
01/12/2014 11:36
Bibliografie
231
în cazul în care
P
1

este puterea pe duza, găsim: (iv) numărul de


cupe; (v) diametrul roții, și (vi) viteza unghiulară
a roții în funcțiune.
(c) Dacă conducta principală (la stăvilar) a avut o lungime de 100 m,
cum ar fi răspunsurile la (a) și (b) să fie modificat prin
frecare fluid folosind: (vii) teava din PVC cu un diametru de 15 cm;
(viii) comune plastic furtun cu un diametru de 5 cm? În
fiecare caz se determine numărul Reynolds în țeavă.
6.7
O țeavă de oțel de diametru D și lungime L este de a transporta un debit Q.
Presupunând că conducta de coeficientul de frecare f variază lent
cu numărul Reynolds, arată că șeful pierdere ca urmare a frecare
este proporțională cu D
-5
(pentru fix L și Q). (Indiciu: se Referă la §R2.6.)
*6.8
Sudanul este o țară plat cu foarte putine precipitatii, dar cu mijloace proprii
de alimentare este dominat de hidroelectricitate. Cum poate fi aceasta? Ce
înseamnă acest exemplu de Sudan diferă de cea a (la fel de plat)
țara de Danemarca?
BIBLIOGRAFIE
General articole și cărți pe hidroenergie
Kumar, A., Schei, T., Ahenkorah, A., Caceres Rodriguez, R., Devernay, J-M., Freitas, M., Sala, D., Killingtveit, Å. și
Liu, Z. (2011) 'Hidroelectrice', în O. Edenhofer, R. Pichs-Madruga, Y. Sokona, K. Seyboth, P. Matschoss, S. Kadner,
T. Zwickel, P. Eickemeier, G. Hansen, S. Schlömer și C. von Stechow (eds) IPCC Raport Special privind Regenerabile
Surse de Energie și de Atenuare a schimbărilor Climatice, Cambridge University Press, Cambridge. Autoritate de revizuire de
stat de artă și potențialul de utilizare mai largă, inclusiv probleme de sustenabilitate (adesea menționată ca SRREN).
Ramage, J. (2004, 2nd edn) 'Hidroelectricitate', ch. 5 în G. Boyle (ed.), Energie regenerabilă: Energie durabilă pentru
viitor, Oxford University Press, Oxford. Non-studiu tehnic, cu multe fotografii și ilustrații.
Mecanica de turbine
Cele mai multe manuale de inginerie mecanica fluidelor include un capitol referitor la turbomasini; de exemplu:
Çengel, Y. A. și Cimbala, J. (2009, 2nd edn) Mecanica Fluidelor: Fundamente și aplicații, McGraw-Hill,
New York. Explicații clare și detaliate cu accent pe principii fizice. Foarte student-friendly, cu un
exemplar care însoțește învățarea sida.
Dixon, S. L. B. și Hall, C. ( 2010, 6 edn) Mecanica Fluidelor și Termodinamică de Turbomasini,
ButterworthHeinemann, Londra. Stabilit un manual pentru ingineri profesionale.
Massey, B. Ward-Smith, J. (2011, 9 edn) Mecanica Fluidelor,, CRC Press, Londra. Mai mult cont de
turbomasini decât alte generală manuale, dar încă la nivel de elev.
Wagner, H-J. și Mathur, J. (2011) Introducere în Hidro Sisteme de Energie: noțiuni de Bază, tehnologia și funcționarea,
Springer, New York. Manual pentru studenți.
TWIDELL PAGINARE.indb 231
01/12/2014 11:36
232
Hidroenergie
La scară mică (mini), hidroenergie (~1 la 100 kW)
Bhatti, T. S., Bansal, r. C. și Kothari, D. P. (eds) (2004) Hidrocentrale de Mica Putere Sisteme, Dhanpat Rai & Co, Delhi,
India. Principii generale și luminoase studii de caz, bazat pe experiența din India.
Khennas, S. și Barnett, A. (2000) cele mai Bune Practici pentru Dezvoltarea Durabilă a Micro-Hidrocentrale în
Țările în curs de Dezvoltare, ITDG, Londra. Disponibil pe web la www.microhydropower.net; a se vedea, de asemenea, (mult
mai scurte) ITDG tehnice scurtă pe micro hidrocentrale de la www.itdg.org.
Moniton, L., Le Nir, M. și Roux, J. (1984) Micro Hidrocentrale, Wiley, Chichester. Traducere
a unui text francez din 1981.
Tong Jiandong et al. (1996) Mini-Hidrocentrale, UNESCO/ Wiley, Chichester. Rezumat util al vastă
experiență Chinez.
Departamentul de Energie al SUA (1988) la scară Mică Sistemele Hidroenergetice, NCIS, Washington, DC. Non-tehnice
cont cu multe desene.
Apa de Alimentare și de Construcție a Barajului (1990) 'Micro hidro: practica actuală și viitoare de dezvoltare', Scoțian
Seminar, număr special.
Pompat sistemelor hidroedilitare
Deane, J. P., O'Gallachoir, B. P. și McKeogh, E. J. (2010) 'Tehno-economic review existente și noi pompat
energia hidro, instalații de depozitare', Energia Regenerabilă și Durabilă Recenzii, 14, 1293-1302.
Mecanice utilizează: berbec hidraulic
Berbec hidraulic este un dispozitiv mecanic care utilizează un flux cu mare Q și low Sec pentru a pompa apa pentru o mai mare
site (cu mai Sec și mai mici Q).
Inverson, A. R. (1978) Berbec Hidraulic Pompa, Voluntari în Asistență Tehnică, Maryland, statele UNITE ale americii, Tehnice
Buletinul nr. 32. Planurile de construcție a ram-ul in sine.
Jeffrey, T. D., Toma, T. H., Popescu, a. V., Glover, P. B. și Fântână, P. D. (1992) Berbec Hidraulic Pompe: Un ghid
pentru etansare pompa de sisteme de alimentare cu apă, ITDG Publishing, marea BRITANIE. A se vedea, de asemenea, ITDG
tehnice scurtă 'berbec hidraulic pompe',
on-line la www.itdg.org.
Krol, J. (1951) 'automate hidraulice ram', Procedurile de Institutul de Inginerie Mecanică, 165, 53-65.
Teoria matematică și unele sprijinirea experimente. Stângace scris face analiza părea dificil.
Twidell, J. și Weir, T. (2006, 2nd edn) Resurse Energetice Regenerabile,, Taylor & Francis, Oxon. §8.7 oferă o
scurtă cont de berbec hidraulic și este reprodus în on-line de materiale suplimentare pentru această carte. Vezi
eResource S6.1 pe site-ul editorului: www.routledge.com/books/details/9780415584388.
Watt, S. B. (1975) Un Manual de Berbec Hidraulic pentru Pomparea Apei, Tehnologia Intermediară Publicații,
Londra. Planuri pentru o alternativă de proiectare de home-made ram, plus detalii de instalare și operare.
Instituționale și problemele de mediu
Agenția internațională a Energiei (1993) Hidroenergie, Energie și Mediu. Conference proceedings, dar
cu o privire de ansamblu utilă. Se concentrează pe implicațiile de upgrade-uri pentru a instalațiilor existente.
TWIDELL PAGINARE.indb 232
01/12/2014 11:36
Bibliografie
233
Moreira, J. R. și Poole, A. D. (1993) 'Hidroenergetic și constrângerile sale', în T. Johansson et al. (eds), surse Regenerabile de
Energie: Surse de combustibili și energie electrică, Earthscan, Londra, pp. 71-119. Bun sondaj de probleme globale și
potențial, cu accent pe social și de constrângerile de mediu și studii de caz din Brazilia.
Comisia mondială a Barajelor (2000) Baraje și Dezvoltare: Un nou cadru pentru luarea deciziilor (de la www.
dams.org). Comisia a fost înființată de către Banca Mondială și Uniunea Internațională pentru Conservarea
Naturii să revizuiască eficacitatea de baraje mari în promovarea dezvoltării economice și sociale și de a dezvolta
noi criterii de evaluare a propunerilor pentru astfel de baraje.
A se vedea, de asemenea, Kumar et al. (2011), enumerate mai sus în General articolelor.
Reviste și site-uri web
Puterea de apă și Construcția Barajului, lunar, Cadranul Casa, Sutton, marea BRITANIE. Jurnal General, inclusiv producția de
informații, rapoarte de conferințe, articole, etc.
Consiliul mondial al Energiei (2010) Studiu de Resurse Energetice 2010 (capitolul pe hidroenergie). Disponibil pe web la
www.worldenergy.org/wec-geis/publications/reports/ser/hydro/hydro.asp. Datele privind capacitatea instalată și
potențialul tehnic pentru numeroase țări, compilate de către utilități și energie agenții; publicarea acoperă alte tipuri de energie
resurse, precum și, inclusiv fosili și chiar OTEC (de obicei, actualizat la fiecare trei ani).
http://en.wikipedia.org/wiki/Hydroelectricity (rezumat excelent cu link-uri importante).
http://hydroelectric-energy.blogspot.com/ Știri pe noi hidro evoluții.
http://www.eia.gov/ US Energy Information Administration oferă vast rezervor de energie statistici, inclusiv
hidrocentrale, din întreaga lume.
www.Indexmundi.com. Are unele convenabil trasează grafice bazate pe date EIA.
TWIDELL PAGINARE.indb 233
01/12/2014 11:36

Resursele de vant
CAPITOLUL

7
CONȚINUTUL
Obiective de studiu
234
§7.1 Introducere
236
§7.2 World wind
237
§7.2.1 efecte Globale
237
§7.2.2 strat limită Planetar
și turbulențe
240
§7.2.3 Regionale de energie eoliană
evaluarea resurselor
240
§7.3 Caracteristicile vântului
242
§7.3.1 Bază de date meteorologice
și viteza vântului serii de timp
242
§7.3.2 Variația cu înălțimea
243
§7.3.3 viteza Vântului analiza, probabilitatea
și predicție
248
§7.3.4 viteza Vântului probabilitate
distribuții: Weibull
și Rayleigh
248
§7.3.5 viteza și direcția Vântului:
variația cu timpul și distanța 254
§7.4 Vânt de instrumentație de măsurare
și instrumente de calcul pentru predicția 258
§7.4.1 Tradiționale stabilite
instrumente
258
§7.4.2 Instrument turnuri
259
§7.4.3 viteza și direcția Vântului
instrumente pentru comerciale
și de cercetare
259
§7.4.4 Alți indicatori și
instrumente
262
§7.4.5 instrumente de Calcul pentru
evaluarea potențialul eolian 262
§7.4.6 pe termen Scurt, previziuni
263
Rezumat capitol
264
Întrebări rapide
264
Probleme
265
Bibliografie
266
OBIECTIVE DE STUDIU

Apreciez cum vântul apare și modul în care este
măsurat.

Apreciez variație a vitezei vântului:
– de-a lungul timpului pe scale de ani, luni,
ore și secunde;
– de la regiune la regiune și de la site-ul pentru a
site-ul într-o regiune (de exemplu, efectul locale
de teren și obstacole);
– cu înălțimea.

Aprecia probabilitatea de distribuție de vânt
de viteză, inclusiv Weibull și Rayleigh
distribuții.
TWIDELL PAGINARE.indb 234
01/12/2014 11:36

www.shahrsazionline.com
Lista de tabele
235
LISTA DE FIGURI
7.1
Circulația Atmosferei Pământului (schematică).
237
7.2 vitezei Medii a vântului în întreaga lume, în ianuarie și iulie.
239
7.3
Regionale viteza vântului harta de statele UNITE ale americii.
241
7.4
Viteza vântului în timp și frecvență parcele.
246
7.5
Wind rose de la datele acumulate.
247
7.6
Viteza vântului variația cu înălțimea; vântul de forfecare'.
247
7.7
Distributia de probabilitate a vitezei vântului împotriva punct.
250
7.8
Probabilitatea de viteze ale vântului mai mari decât o anumită viteză u'.
251
7.9
Distribuția de putere în vânt, de exemplu, North Ronaldsay.
251
7.10 Putere pe unitatea de suprafață în vânt împotriva probabilitatea de viteze ale vântului mai mari decât o anumită
viteza u'.
252
7.11 Weibull curbelor de distribuție.
253
7.12 Unele instrumente pentru măsurarea vitezei vântului și/sau de direcție.
260
LISTA DE TABELE
7.1
Viteza vântului relații bazate pe scara Beaufort.
244
7.2
Viteza vântului analiză pentru North Ronaldsay.
250
TWIDELL PAGINARE.indb 235
01/12/2014 11:36
236
Resursele de vant
§7.1 INTRODUCERE
Extragerea de energie electrică din vânt cu turbinele moderne și energie,
sisteme de conversie este stabilită o industrie la nivel mondial. Acest capitol
se concentrează pe energie eoliană de resurse și măsurarea acesteia; de
tehnologie pentru a extrage această putere urmează în Capitolul 8. Suntem deosebit de
interesați în medie și peste medie și viteza vântului, pentru că puterea
vântului este proporțională cu cubul vitezei vântului, așa cum se arată
la punctul 8.3.1. Suntem de asemenea, interesat (a) cum vântul are loc; (b)
măsurament; (c) variația cu timpul, pentru că de ieșire fluctuații de energie;
(d) creșterea vitezei vântului cu înălțimea deasupra solului, deoarece lama
sfaturi de masini foarte mari ar putea fi de 200 de metri de mare; (e) turbulență și
rafale de vânt; (f) site-ul local de condiții și obstacole, inclusiv alte turbine,
care afectează generate de ieșire, și (g) de predicție, astfel încât rețeaua de energie electrică,
operatorii pot planifica.
Vântul rezultate din extinderea și convecție de aer ca radiația solară este
absorbită de pe Pământ. Pe o scară globală, aceste efecte termice combina cu
efecte dinamice de rotație a Pământului de a produce predominantă a vântului
modele. Energia cinetică stocată în vânturi este de aproximativ 0,7
× 10
21

J, și
acest lucru este disipată prin frecare, în principal în aer, dar, de asemenea, prin contact cu
pământul și marea. Aproximativ 1% din radiația solară absorbită, 1200 TW
(1200
× 10
12

W), este disipată în acest fel. În plus față de acest general


sinoptice comportament de Atmosfera de acolo este considerabilă regionale și locale
variație cauzată de factorii geografici si de mediu. În general,
viteza vântului crește cu înălțimea, cu componentele orizontale
semnificativ mai mare decât componentele verticale.
Viteza vântului variază în mod semnificativ în timp pe perioade de la câteva secunde
la anotimpuri și ani, și pe distanțe ~1 km, mai ales în zonele colinare
teren. Prin urmare, este important să se facă măsurători la
- ului nominalizat la mai multe înălțimi de cel puțin 12 luni și să compare aceste
oficiale date meteorologice și de vânt atlas informații. La
informații permite predicția de generare a energiei electrice din nominalizat
turbine pentru site-ul.
Un important criteriu de proiectare pentru turbine este nevoie pentru a proteja aparatul
împotriva deteriorării în foarte puternic și turbulent vânturi, chiar dacă astfel de
vânturile puternice sunt relativ rare. Vântul forțe au tendința de a crește
ca piata de viteza vântului și amplitudinea turbulent
variație a crește în mod similar. Prin urmare, oboseala, pot aparea daune, mai ales
legate de lamele și de antrenare; deci, viteza vântului variație de un
minut și mai puțin trebuie să fie înțeleasă în întreaga zonă a rotorului turbinei.
Din fericire, există alte industrii și servicii pe care trebuie să știți
despre condițiile de vânt și astfel de informații pot fi partajate; aceasta include
servicii meteorologice, agricultură, aeronave și aeroporturi, clădiri și
construcții de poduri și siguranța rutieră.
TWIDELL PAGINARE.indb 236
01/12/2014 11:36
§7.2 World wind
237
§7.2 WORLD WIND
§7.2.1 efecte Globale
Turbinele eoliene operează numai în cazul în care și atunci când este vânt! Cu toate acestea, acest
adevăr evident este adesea uitat, chiar în energetică națională de planificare. În această
secțiune, vom lua în considerare cum vântul are loc la nivel global.
Aerul este transparent la radiația solară și deci nu este încălzit până la
radiație este absorbită în sol și sol incalzeste aerul de mai sus.
Aerul încălzit aproape de sol se extinde, devine mai puțin dens și se ridică
prin aerul mai rece de deasupra. Efectul de încălzire este mai puternic în apropiere de
Ecuator. Acest lucru provoacă buclă curenți de convecție în partea inferioară a
atmosferei (troposfera) la înălțimi ~15 km. Fig. 7.1(a) descrie acest
scenariu a produce o pereche de celule de aer care circulă, ca prima avute în vedere de către
Hadley în secolul al 17-lea.
În viața reală acest 'o singură celulă de circulație nu poate fi susținută pe
distanțe lungi (~9000km) între Ecuator și Poli, într-un termen
relativ superficial atmosferă (~15 km). Circulația se desparte în
trei celule în fiecare emisferă, așa cum se arată în Fig. 7.1(b). Creșterea aerul de la
Ecuator coboară în jurul valorii de latitudinea de 30°, continuă spre Polul aproape de
suprafață până la latitudinea de 60°, apoi se ridică înainte de a continua spre
Pol în atmosfera superioară.
Această imagine simplă este complicată de rotație a Pământului. În
regiunile tropicale, un "colet" de aer de lângă suprafața Pământului este împins
Fig. 7.1
Circulația Atmosferei Pământului (schematică). Rețineți că 'grosimea' din
Atmosferă este exagerat de către un factor de ~100.
o
Valoarea noțională
nord–sud circulație într-o pereche de celule, în funcție de Hadley (c.1670).
b
Aproximativ reale de circulație în trei perechi de celule. De asemenea, sunt indicate
puternic mid-
latitudine vânturile de vest și mai slab tropicale 'vânt'.
Nordic
vânturi de suprafață
Ecuator
Polul nord
Polul sud
Rece
Rece
Rece
Rece
Cool
(a)
Cool
Cool
Cool
Cald
Cald
Cald
Cald
Cald
Cald
Sud
vânturi de suprafață
(b)
Polar easterlies
Westerlies
60 N
Polar celule
Creșterea
aer
Creșterea
aer
Scufundarea
cu aer
Ferrel
mobil
Hadley
celule
30 N
30 S
0
Vânturile comerciale
Westerlies
Polar easterlies
60 S
TWIDELL PAGINARE.indb 237
01/12/2014 11:36
238
Resursele de vant
spre Ecuator către termică circulație, mișcare, astfel, într-o regiune
unde viteza de rotație a Pământului este mai mare decât cel în care
aerul venit de la, astfel încât "colet" de aer este lăsat în urmă de pe suprafața
de dedesubt și ajunge la un punct la vest de unde ar fi fost dacă
s-a mutat pur de la nord la sud. Astfel, "wind" (adică de aer
de circulație în apropiere de suprafața de nivel) apare pentru un observator de pe suprafața
Pământului să fie venind dinspre nord-est (în emisfera nordică) sau
din sud-vest (în emisfera sudică) (vezi Fig.7.1(b)). Acest
obicei moderată predominantă NE sau SE vânt este numit un 'vânt', ca urmare
a utilizării navele de navigatie facute de ea. În mod similar, polar mobil seturi puternice
vânturi estice.
În mijlocul latitudini, în cazul în care suprafața de aer este de gând spre Poli,
se deplasează în regiunile mai lent viteze de rotație, și este, prin urmare, să
călătoresc mai repede decât noua suprafață. Un observator simte
vântul de vest – așa-numitul westerlies (o vest, vântul
este unul care vine din vest). La vest componentă a acestui
midlatitude vântul este mai puternic decât cea de est componentă în
comerțul vânt, pentru că diferența în viteza de rotație pe latitudine este
mai mare; prin urmare, numele de 'roaring forties'.
Hărțile în Fig. 7.2 arată că cele mai puternice vânturi dominante
apar peste ocean în anii 'roaring forties și slăbi peste continente.
Succes valorificarea energiei eoliene necesită puternică, constantă vânturi și
o populație cu o cerere de energie. Hărțile indică faptul că
nord-vestul statele UNITE ale americii, Europa de vest (inclusiv marea Britanie și Irlanda), Noua
Zeelandă și Chile sunt toate aceste favorabile regiuni. Fig. 7.2 ilustrează, de asemenea,
că, în vara anului 'roaring forties' sunt mai departe spre Pol, și în
timpul iernii se mută în mijlocul latitudini; aceasta deoarece declinația
Pământului (§2.4) implică faptul că regiunea de aer în creștere în apropierea Ecuatorului
se mută de la nord la sud cu anotimpurile.
Înțelegerea vânt în toată disciplina de meteorologie este
un important provocare analitică în asociere cu cuprinzătoare
de măsurare și înregistrarea datelor (a se vedea în acest capitol bibliografie pentru continuarea
și în profunzime informații). În plus față de circulația globală a descris
până în prezent, există o multitudine de alte efecte cu impact semnificativ de sezon,
regionale și locale de variație.
(a) Efectul de oceane și continente
Radiația solară încălzește pământul repede, dar oceane încet; cu toate acestea, termică
capacitate de ocean aproape de apă de suprafață este mare și deci contrastează cu
continental teren de masă. Relativă efecte sunt sezoniere solare înclinare
și alte efecte schimba pe parcursul anului. Extreme de vânt apar în
uragane (cicloane tropicale) și musoni, ca umiditate, cea mai mare parte luate din
ocean, se condensează în apă (ploaie), cu eliberare de căldură latentă.
Efecte similare, dar mai puțin extreme, apar în toate ciclonic vreme ca aerul nu
circula numai, dar se mișcă în sus și în jos. Aceste mișcări
TWIDELL PAGINARE.indb 238
01/12/2014 11:36
§7.2 World wind
239
Fig. 7.2
Vitezei medii a vântului în întreaga lume, în ianuarie și iulie. Cel mai puternic mediu de vânturi
(afișate ca albe) sunt vânturi de vest în Oceanul Sudic și Atlanticul de Nord.
Sursa: http://earthobservatory.nasa.gov/IOTD/view.php?id
=1824; nota: acest site are, de asemenea, o lună de
luna de animație (accesat la 1 octombrie 2013).
Viteza vântului (m/sec)
Iulie
Ianuarie
0
7
14
Viteza vântului (m/sec)
Iulie
Ianuarie
0
7
14
TWIDELL PAGINARE.indb 239
01/12/2014 11:36
240
Resursele de vant
sunt cunoscut ca vântul. Pe o scară mai mică, la încălzirea neuniformă a terenului
și marea produce local diurne briza marii.
(b) Efectele de teren de forma
Pe teren complex de dealuri și munți deviază și concentrate de aer
de circulație, cu efect semnificativ asupra vântului. Variație zilnică a acestor vânturi
apare ca urmare a inegale solare absorbție și diferențe de înălțime, și cu
concentrare, ca și în văi. Circulație a aerului de peste munți poate duce
la depunerea de ploaie pe windward părți și aer încălzit prin creșterea
de presiune pe partea de sub vânt (Fohn vânt).
(c) Efecte de anotimp și de ora
În marea majoritate de locații, viteza medie a vântului și direcția
depinde de sezon în an, și în multe locații de pe timp de
zi. Serviciile meteorologice cunosc aceste efecte de bine și pot face
în mod rezonabil de încredere previziuni. Cu toate acestea, nu există aproape completă
ignoranță despre prezicerea variație de la an la an, ceea ce pentru energia eoliană
poate provoca variații semnificative care afectează economia de instalații.
Astfel de cunoștințe este necesar pentru a fi capabil de a alege
productive turbine eoliene site-uri și de a prezice condiții de vânt și, prin urmare,
vânt generate de putere, ca, de exemplu, în vânt atlase, așa cum este descris în
§7.2.3.
§7.2.2 Planetare strat limită și turbulență
Turbulența este o schimbare de viteza vântului și direcția vântului, atât în
plan orizontal și în direcția verticală. Meteorologii vorbesc de
strat limită planetar ca fiind cea mai mică regiune din Atmosfera Pământului,
acolo unde este marcată de turbulențe cauzate de frecare cu solul
și tulburări de obstacole, cum ar fi copaci, clădiri mari, orașe
și dealuri. Acest strat limită variază în grosime de la aproximativ 250 m deasupra
mării, la aproximativ 500 m peste orașe și abrupt țară. Mai sus de limita
strat, mișcarea aerului este buna (laminar), dacă nu există furtuni
sau uragane; o caracteristică neobișnuită sunt fluxurile de aer la înălțimi cuprinse între
aproximativ 10 până la 15 km, care afectează caracterul și avioane, dar nu a vântului
turbinele direct. Cele mai mari turbine eoliene avea de top lama-sfat înălțime de
aproximativ 150 m, deci cu siguranta toate turbinele eoliene funcționează în cadrul planetare
strat limită și întotdeauna experiența turbulențe în vânt. Acest lucru are
un impact semnificativ cu privire la proiectarea turbinelor, mai ales în ceea ce privește
puterea și duritatea de lame și drive-train componente.
§7.2.3 Regionale de energie eoliană resurse evaluare
Există multe publicații, site-uri și instrumente software pentru a ajuta la
a determina potențialul eolian de țări, regiuni și zone locale.
TWIDELL PAGINARE.indb 240
01/12/2014 11:36
§7.2 World wind
241
Fig. 7.3
(a) Sudul Franței; Notă localizate regional windiness din valea Ronului și de vest coasta Mediteraneană.
Credit: Eoliene Europene Atlas, DTU Energie Eoliană, fostă RISO Național de Laborator.
(b) Regionale viteza vântului harta de statele UNITE ale americii. Notă distinctivă vânturi puternice asociate cu muntele și valea
regiunile de
vest central. Contururile reprezintă medie a vântului de putere (proporțională cu viteza vântului la cub; vezi §8.3), cu verde închis
cea mai mare (u->7/m/s).
Sursa: http://rredc.nrel.gov/wind/pubs/atlas/maps/chap2/2-01m.html.
> 6,0
Adăpostit de teren

Resurse eoliene la 50 de metri mai sus de nivelul solului


pentru cinci tipuri diferite de condițiile topografice m/s.
(a)
(b)
STATELE UNITE ALE AMERICII MEDIU ANUAL DE ENERGIE EOLIANĂ
În plan deschis
La o coasta mării
Deschide mare
Dealuri și creste
< 3,5
5,0–6,0
4,5–5,0
3,5–4,5
> 7,5
< 4,5

6,5–7,5

5,5–6,5

4,5–5,5
> 8,5
> 9,0
< 5,0
< 5,5
7,0–8,5

8,0–9,0

7,0–8,0

5,5–7,0
> 11,5
< 7,0

10,0–11,5

8,5–10,0

7,0–8,5

6,0–7,0

5,0–6,0
Vântul
puterea de
clasa

100

150

200

250

300

400
1000
Creasta creasta estimări (locale de relief > 1000 ft)

Clase de vânt
densitatea de putere
0

4.4

5.1

5.6

6.0

6.4
7.0
9.4

9.8

11.5

12.5

13.4

14.3

15.7

21.1

200

300

400

500

600

800
2000

5.6

6.4

7.0

7.5

8.0

8.8
11.9

12.5

14.3

15.7

16.8

17.9

19.7

26.6
Energia eoliană
W/m
2
Energia eoliană
W/m
2
10 m (33 f
t)
50 m (164 f
t)
Viteza
m/s
mph
Viteza
m/s
mph
0
100
200
300
mile
0
100
200
300
kilometri

Alaska
Directorul
Hawaiian
Islands
0
100
200
mile
0
100
200
kilometri
0
100
200
mile
0
100
200
kilometri
0
100
200
mile

PUERTO RICO
0
100
200
kilometri

TWIDELL PAGINARE.indb 241


01/12/2014 11:37
242
Resursele de vant
De bază importanță sunt energia eoliană atlase asociate cu software-ul de
instrumente, dintre care cele mai internaționale sunt cele produse de Viespe
program (a se vedea www.windatlas.dk). Atlasele și software specifica și de a
obține viteza vântului sezoniere medii care pot fi aplicate la nivel regional
zone cu diferite caracteristici geografice, în general, de
exemplu, la adăpost de teren' (de exemplu, lângă pădure), 'deschis', 'sea coast',
'open sea" și "dealuri și creste'. A se vedea §7.4.5.
Fig. 7.3 arată hărți eoliene din sudul Franței și Statelor Unite
de la astfel de atlase. Ambele arată puternic viteza vântului locații mai mici la
sol, aproape de munți, și/sau în cazul în care vântul este canalizată prin sinoptice
meteorologice și/sau diurne solare de încălzire oferă vânt puternic cu viteze
în văi de munte, și/sau în apropierea unor mari întinderi de apă deschisă. Astfel de
efecte pot fi extrem de important pentru regionale de energie eoliană.
§7.3 CARACTERISTICILE VÂNTULUI
§7.3.1 Bază de date meteorologice și de vânt
viteza serii de timp
Toate țările au serviciile naționale de meteorologie, care înregistra și publica
legate de vreme datelor, inclusiv viteza vântului și direcții. Metodele
sunt bine stabilite și coordonate în Meteorologică Mondială
Organizația de la Geneva, cu scopul principal de a furniza continuu se execută
de date pentru mai mulți ani. Datele tind să fie înregistrat la un număr relativ mic de
personal permanent oficial stații folosind robust și de încredere de echipamente.
Din păcate pentru energia eoliană de predicție, oficial măsurători de vânt
viteza tind să fie măsurată numai la înălțimea standard de 10 m,
și la stațiile de aproape de aeroporturi sau orașe unde protectie de vant
poate fi o caracteristică naturală a site-ului. Astfel de date sunt, cu toate acestea, este important
ca bază 'ancore' pentru computerizată modelare vânt, dar nu sunt
potrivite pentru a aplica direct pentru a prezice vânt condițiile de putere la un anumit site.
Standard meteorologică de vânt datele de cea mai apropiată stație oficiale sunt
utile doar ca primul-pentru estimări; ele nu sunt suficient de detaliate
de planificare, în special în zone de deal (complex) de teren. Măsurători la
- ului nominalizat la mai multe înălțimi sunt necesare pentru a prezice puterea produsă
de special turbine. Astfel de măsurători, chiar și pentru câteva luni, dar
cel mai bine de un an, sunt, comparativ cu standard de date meteorologice, astfel încât
pe termen scurt comparație poate fi folosit pentru mai mult de predicție pe termen; la
tehnica se numeștemăsură de-a corelat-prezice'. În plus, informațiile
deținute în specialitate de energie eoliană bănci de date care sunt obținute de la aeronave
măsurători, vânt instalații de putere și modelare matematică,
etc. Astfel organizat și accesibil de informații este din ce în ce mai disponibile
pe Internet. Energia eoliană modele de predicție (§7.4.5) (de ex. la
proprietate Viespe modele dezvoltate în Danemarca) permite detaliate vânt puterea
de predicție pentru viitorii turbine eoliene site-uri de relativ rare locale
de date, chiar și în teren deluros.
TWIDELL PAGINARE.indb 242
01/12/2014 11:37
§7.3 Caracteristicile vântului
243
Clasificarea de viteze ale vântului de birourile meteorologice este legat de
istoric pe scara Beaufort, care se referă la observații vizuale.
Tabelul 7.1 oferă detalii împreună cu relația dintre diverse
unități de viteza vântului.
Un standard de măsurare meteorologice din punct măsuri
de "lungime" sau " run " de vântul care trece-o 10 m de mare cupa anemometru
în 10 minute. Astfel de măsurători pot fi luate pe oră, dar, de obicei,
mai puțin frecvent. Astfel de date oferă puține informații despre fluctuații în
viteza și direcția vântului este necesar pentru precizie
estimarea turbine eoliene de performanță. Continuu lectură anemometre sunt
mai bine, dar acestea vor avea un anumit timp de răspuns. Un tipic
citirea continuă urmă (Fig. 7.4(a)) arată rapidă și fluctuațiile aleatorii
care apar. Transformarea acestor date în domeniul de frecvență oferă
gama și importanța acestor variații (Fig. 7.4(b)).
Direcția vântului se referă la compas din care
vine vântul. Datele meteorologice sunt de obicei prezentate ca un vânt
rose (Fig. 7.5(a)), care arată viteza medie a vântului în anumite
intervale de direcție. Este de asemenea posibil pentru a arăta distribuția de viteze
de la aceste direcții pe o roză a vânturilor (Fig. 7.5(b)). Astfel de informații este
de o mare importanță atunci când amplasarea o mașină de vânt în zone deluroase, lângă
clădiri, sau în tablouri de mai multe masini de protectie ar putea apărea.
Schimbări în direcția vântului poate fi numit 'vânt'; de 0,5 rad/s (30°/s) este
o schimbare rapidă (de exemplu.g. în teren deluros). Astfel de modificări pot afecta un vânt
turbina mai mult decât schimbări extreme de viteza vântului.
§7.3.2 Variația cu înălțimea
Viteza vântului variază considerabil cu înălțimea deasupra solului; acest lucru este menționat
ca vântul de forfecare. O mașină cu o inaltime de (sa zicem) 30 m deasupra alte
obstacole va experimenta mult mai puternic decât vânturile de o persoană de la sol
nivelul. Fig. 7.6 se prezintă sub formă de viteza vântului variația cu înălțimea z în
apropiere de-la-sol strat limită până la aproximativ 100 m. La z
= 0 aer
viteza este întotdeauna zero. În înălțime locale obstacole, viteza vântului
schimbă haotic, rapidă și direcționale pot exista fluctuații puternice
de vânt. Mai sus de acest neregulat regiune, înălțimea/punct de profil este dat
prin expresii de forma
z
− d = z
0
exp(u
z
/ V ) (7.1)
Prin urmare,
u
V
zd
z
în
z
0
=





(7.2)
Aici d este zero avion deplasare cu magnitudinea un pic mai puțin decât
înălțimea locale obstacole, termenul z
0
se numește rugozitate lungime
și V este o caracteristică de viteză. În Fig. 7.6 funcția este extrapolată la
TWIDELL PAGINARE.indb 243
01/12/2014 11:37
Tabelul 7
.1
Viteza vântului relații bazate pe Beaufor
t scară
Beaufort
numărul
Viteza vântului variază de la 10 m inaltime
Descriere
Turbine eoliene
efecte
Generarea de energie
posibilitatea daca medie
viteza vântului a menținut
Efecte observabile
(ms
-1
)
(km h
-1
)
(mi h
-1
)
(nod)
pe teren
la mare
0.0
0.0
0.0
0.0
Fumul se ridică
vertical
Oglinda netedă
0




Calm
Nici unul

0.4
1.6
1
0.9
0.4
1.6
1
0.9
Fum galerii dar
paletele neafectat
Valuri mici
1




Lumina
Nici unul

1.8
6
4
3.5
1.8
6
4
3.5
Vânt-am simțit
Clar valuri
2




Lumina
Nici unul
Inutil
peste piele;
3.6
13
8
7.0
Frunzele se amestecă;
Paletele neafectat
3.6
13
8
7.0
Start-up de
De pompare a apei;
Frunze în
Ocazional val
3




Lumina
turbine pentru lumina
vânturi, de exemplu
de pompare
minore electrice
putere
circulație; steaguri
începe să se extindă
creasta pauză,
sticlos
aspectul de
întreaga mare
5.8
21
13
11
5.8
21
13
11
Utile electrice
Ramuri mici
Valuri mai mari,
4
↓ 8.5
↓ 31
↓ 19
↓ 17
Moderat
generarea de energie
producția de energie
muta; praf
ridicat; pagini de
cărți ridicat
alb vreme
comune
8.5
31
19
17
putere
Extrem de bun
Copaci mici în frunze
Alb vreme
5
↓ 11
↓ 40
↓ 25
↓ 22
Proaspete
generație
perspectivele pentru putere
sway, vânt
vizibil pentru
peste tot
comentariu
11
40
25
22
Gamă nominală la plin
Doar pentru cel mai puternic
Ramuri mari
Valuri mai mari
6




Puternic
capacitate
masini
sway, telefon
apar, de spumare
14
51
32
28
linii de fluier
vreme și extinse
14
51
32
28
Capacitate maximă
Viața nu merită trăită
Copaci întregi în
Spuma începe să
TWIDELL PAGINARE.indb 244
01/12/2014 11:37
7




Puternic
ajuns
aici
mișcare
pauza de la
vreme și în
17
63
39
34
dungi
17
63
39
34
Shutdown sau auto-
stagnare a inițiat
Ramuri se rup.
Dens dungi
de suflat
spuma
8




Gale
De mers pe jos dificil
21
76
47
41
21
76
47
41
Toate mașinile taci
Ușoară structurale
Aruncat spuma
9




Gale
jos sau blocat
prejudiciul, de exemplu
extinse
25
88
55
48
cosuri de fum
25
88
55
48
Criterii de proiectare
Copaci dezrădăcinați.
Mult structurale
daune
Valuri mari
cu mult timp
rupere
vreme
10




Puternic
împotriva daunelor
29
103
64
56
Gale
Mașini de închis
jos
29
103
64
56
Numai
consolidat
Pe scară largă
11




Puternic
mașini ar
daune
34
121
75
65
Gale
supraviețui
Daune grave
anumite excepția cazului în
pre-colaps
Apare numai în
cicloane tropicale
Rural
devastat.
Dezastru
condiții.
Navele ascunse
în val
jgheaburi.
Aerul umplut
cu spray
12
>34
>121
>75
>65
Uraganul
1 m/s
= 3,6 km/h
= 2.237 km/h
= 1.943 nod
0.278 m/s
= 1 m/h
= 0.658 km/h
= 0.540 nod
0.447 m/s
= 1.609 km/h
= 1 km/h
= 0.869 nod
0.515 m/s
= 1.853 km/h
= 1.151 km/h
=1 nod
Tabelul 7
.1
(continuare)
TWIDELL PAGINARE.indb 245
01/12/2014 11:37
246
Resursele de vant
Fig. 7.4
Viteza vântului în timp și frecvență parcele: (a) Continuă anemometru lectură. O scurtă
secțiune de un record de orizontală viteza vântului, vertical viteza vântului și a temperaturii la
o înălțime de 2 m de la meteorologică domeniu, Universitatea Reading, marea BRITANIE. Notă pozitivă
corelații între vertical viteza vântului și temperatura, și corelații negative
între orizontală și verticală viteze ale vântului. (b) în domeniul Frecvență, variația spectrului
(după Petersen, 1975). Graficul este o transformare de multe serii de măsurători în
Danemarca, care au fost folosite pentru a găsi pătrat de deviația standard (variația)
vitezei vântului u din viteza u–. Astfel, graficul se referă la energia în vânt
viteza fluctuații în funcție de frecvența lor; uneori este numit un 'Van der Hoven'
spectrului.
6
5
4
3
2
1
0.5
-0.5
-1
19
18
17
16
0
5
10
10
-7
10
-6
10
-5
10
-4
10
-3
Frecvență/Hz
10
-2
10
-1
10
0
10
1
15
20
25
30
35
40
Timp/s
1 an
5
4
3
2
1
0
1 zi
Cu 1 oră 10 min
10 s
1s
0.1 s
Perioada
Informații care ar putea fi pierdute
de către o medie de peste mai mult de
o oră
(b)
(a)
30 de zile
Aer
temperatura
°C
0
Vertical
viteza vantului
ms
-1
Orizontală
viteza vantului
ms
-1
d(
u

u
)
2
/d
f
m
2
s
-2
/Hz
TWIDELL PAGINARE.indb 246
01/12/2014 11:37
§7.3 Caracteristicile vântului
247
Fig. 7.5
Wind rose de la datele acumulate: (a) Direcție. Stația de pe insula Scoțiană Tiree în
Outer Hebrides. Liniile radiale dau procente din an în care bate vântul
la fiecare 16 direcții. Valorile sunt de 10 ani înseamnă și se referă la un sistem eficient de înălțimea
deasupra solului de 13 m. (b) Direcția și distribuția de viteză. Malabar Hill pe Lord Howe
Island, New South Wales. Cele mai gros secțiuni reprezintă proporția de timp în care vântul
viteza este între valorile specificate, în termen de 16 direcționale sectoare.
Sursa: După Bowden et al. (1983).
N
2%
W
S
E
(a)
4%
6%
8%
(b)
N
> 20 m s
-1
5-10 m s
-1
10-15 m s
-1
15-20 m s
-1
Fig. 7.6
Viteza vântului variația cu înălțimea; vântul de forfecare, vezi ecuația (7.1).
d
Viteza vântului u
Aproximativ
scară de
obstacole locale
Înălțimea
deasupra
solului,
z
z
0

valorile negative de u pentru a arăta forma de expresie. Cititorii ar trebui să


consulte texte despre meteorologie și micrometeorology pentru a corecta detaliu
și înțelegerea viteza vântului strat limită profile. Cu toate acestea,
cel mai important aspect practic este nevoie de a plasa o turbina cu mult peste
înălțimea locale obstacole pentru a se asigura că turbină disc primește
un puternic vânt uniform flux întreaga zonă fără fluctuații neregulate.
TWIDELL PAGINARE.indb 247
01/12/2014 11:37
248
Resursele de vant
Cele mai bune site-uri pentru energie eoliană sunt în partea de sus a buna, domul în formă de
dealuri, care sunt poziționate clar de alte dealuri. În general, vântul ar trebui să
fie incident peste suprafețe de apă sau netede teren de câteva sute de
metri, adică nu ar trebui să fie un bun aport. Cele mai multe turbine eoliene operează
la inaltimi intre 5 m (încărcătoare de baterii) și 100 m (mare,
gridlinked). Cu toate acestea, este comun pentru standard meteorologice, viteza vântului
măsurători u
s
pentru a fi luate la o înălțime de 10 m. Un aproximative
expresie des folosită pentru a determina viteza vântului u
z
la înălțimea z este
=




'
u
u
z
10m
z
s
b

(7.3)
Se afirmă adesea că b'
= 1/7 = 0,14 pentru a deschide site-uri în 'non-deluros de la țară.
Site-uri bune ar trebui să aibă valori mici ale b' pentru a evita modificări în sens opus
vitezei vântului de peste turbină disc, și valori mari ale vitezei medii a vantului
u– pentru a crește puterea de extracție. Mare grijă trebuie să fie luate cu această
formulă, mai ales pentru z > 50 m. Problema 8.11 arată că (7.3) indică faptul
că o creștere a înălțimii turnului dincolo de circa 100 m este de scădere a
beneficia de turbine eoliene din țară.
§7.3.3 viteza Vântului analiza, probabilitatea și predicție
Implementarea energiei eoliene necesită cunoștințe de viitor viteza vântului
la turbina site-uri. Astfel de informații sunt esențiale pentru proiectarea de
mașini și sisteme de energie, și pentru economie. La
aparent la întâmplare, natura vântului și site-caracteristici specifice face
astfel de informații provocatoare, încă mai poate fi făcut de statistică
analiza de corelație de măsurare serii de timp și de
meteorologie. Dezvoltarea energiei eoliene a condus la o mare rafinament
în asociat analiză, care implică în special de date-tehnici de manipulare
și modelare pe calculator. A lucrat Exemplu Tabelul 7.1 și 7.2 ilustrează
pas cu pas metoda de analiză, arată cum puterea disponibilă
de vânt poate fi calculată de la foarte de bază de date măsurate pe
distribuția vitezei vântului de la un anumit site. Comerciale de măsurare
tehnici sunt mult mai sofisticate cu on-line de achiziție de date și
de analiză, dar principiile sunt aceleași.
Analiza de Lucrat Exemplu 7.1 este în întregime în termeni de
probabilitate de vânt caracteristici; în esență, am considerat o frecvență
domeniu de analiza si nu de 'domeniu'. Timpul de domeniu, inclusiv
turbulențe și gustiness, este considerat în §7.3.5.
§7.3.4 viteza Vântului distribuții de probabilitate:
Weibull și Rayleigh
Analiza de Lucrat Exemplu 7.1 depins exclusiv pe date de teren și
repetitive de calcul numeric. Ar fi extrem de util în cazul în care
TWIDELL PAGINARE.indb 248
01/12/2014 11:37
§7.3 Caracteristicile vântului
249
EXEMPLUL 7.1 VITEZA VÂNTULUI ANALIZĂ PENTRU INSULA NORTH RONALDSAY,
ORKNEY, SCOȚIA
10 minute de run-of-the-vânt' anemometru a fost instalat de la 10 m înălțime pe o deschide site-ul de aproape o propunere de
turbine eoliene. Cinci lecturi au fost înregistrate în fiecare zi la ora 9 a. m., ora 12, la 3 p. m., 6 p. m. și 9 p. m.,
pe tot parcursul anului. Folosind metoda de 'lari', Tabelul 7.2 oferă o selecție din totalul de date și de analiză,
cu coloane numerotate de mai jos.
1
Citirile au fost clasificate în intervale de
Du = 1 m/s, adică 0.0 la 0,9; 1,0 la 1.9, etc. Un total de N = 1763
citirile au fost înregistrate, cu 62 lipsesc din cauza unor defecțiuni aleatorii.
2
Numărul de apariții de lecturi în fiecare clasă a fost numărate pentru a da
DN(u)/Du, cu unități de
număr pe gama de viteze (dN/du în Tabelul 7.2).
Notă:
DN(u)/Du este un număr pe intervalul de viteză, și așa este numit un frecvenței de distribuție a vitezei vântului.
Ai grijă, însă, să precizeze intervalul de viteza gama
Du (în acest caz 1 m/s, dar de multe ori o mai mare
interval).
3
[
DN(u)/
Du] /N = F
u
este un normalizat funcție de probabilitate, de multe ori numit distribuție de probabilitate de vânt
viteză.
F
u
este complotat împotriva u în Fig. 7.7. Unitatea de
F
u
este inversul vitezei interval, în acest caz
(1 m/s)
-1
.
F
u
Du este probabilitatea că viteza vântului este în clasa definită prin u (adică u a u + Du). Pentru
un an
SF
u
Du = 1.
4
Valoarea totală cumulată a valorilor
F
u
Du este intabulat pentru a da de probabilitate, F
u>u'
de viteze mai mari
decât o anumită viteză u
'. Unitățile sunt numărul pe gama de viteza înmulțită cu viteza variază, adică
adimensional. Această funcție este reprezentată în Fig. 7.8, și poate fi interpretată ca proporția de timp în
anul în care u depășește u
'.
5
Medie sau viteza medie a vântului u
m
(adesea notată cu u– ) este calculată de la
u
m
(
≡u– ) = (SF
u
u) / (
SF
u
)
(7.4)
Medie de viteza u
m
= 8.2 m/s este indicat în Fig. 7.7. Observați că u
m
este mai mare decât cea mai probabilă
viteza vântului de 6.2 m/s la această distribuție.
6
Valorile de u
3
sunt determinate, ca un pas către evaluarea putere în vânt în (9) de mai jos.
7
F
u
u
3
permite valoarea medie u

3
pentru a fi determinată ca




=
F
F=
F
F




u
u
uu
u
d
d
u
u
u
u
3
3
0
0
3
0
0
(7.5)
Rețineți că u

3
se referă la puterea medie în vânt.
8
Puterea pe unitatea de suprafață de vânt secțiune transversală este P
0
=
1
2
ru
3
. Deci, dacă, să zicem,
r = 1,3 kg/m
3
apoi P
u
= Ku
3
unde K
= (0.65 × 10
-3
)Wm
-2
(m/s)
-3
.
9
P
u
F
u
distribuția puterii în vânt (Fig. 7.9). Observați că valoarea maximă a P
u
F
u
apare pe
North Ronaldsay la u
= 12,5 m/s, aproximativ de două ori mai probabil punct de 6.2 m/s.
10
În Cele Din Urmă, Fig. 7.10 parcele puterea pe unitatea de suprafață în vânt la u' împotriva
F
u
>u'
, pentru a indica probabilitatea de
obținerea special nivele de putere.
Notă: atunci când u = O,
F
u
>u'
= 1 și P
0
= O, atunci când u este foarte mare,
F
u
>u'
este mic, dar puterea este mare.
important funcția
F
u

, distribuția de probabilitate a vitezei vântului, ar putea


fi dat o formă algebrică care a montat exact datele. Două
avantaje urmează: (1) mai puține date trebuie să fie măsurat; și (2) analitice
de calcul a vântului mașină de performanță ar putea fi încercat.
TWIDELL PAGINARE.indb 249
01/12/2014 11:37
250
Resursele de vant
Tabelul 7.2
Viteza vântului analiză pentru North Ronaldsay. Aceasta este o selecție a datelor complete de
Barbour (1984), pentru a arăta metoda de calcul. Coloane numerotate ca punctele de
Exemplu Lucrat 7.1.
1
2
3
4
5
6
7
8
9
u
'
dN/du
F
u
F
u
>u'
F
u
u
u
3
F
u
u
3
P
u
P
u
F
u
ms
-1
(ms
-1
)
-1
(ms
-1
)
-1
(ms
-1
)
3
(ms
-1
)
2
kW m
-2
(W m
-2
)(ms
-1
)
-1
>26
1
0.000
0.000
0.000
17576
0.0
11.4
0.0
25
1
0.001
0.001
0.025
15625
15.6
10.2
10.2
24
1
0.001
0.002
0.024
13824
20.7
9.0
9.0
23
2
0.002
0.004
0.046
12167
18.3
7.9
15.8
22
4
0.002
0.006
0.044
10648
21.3
6.9
13.8


















8
160
0.091
0.506
0.728
512
46.6
0.3
27.3
7
175
0.099
0.605
0.693
343
340
0.2
19.8
6
179
0.102
0.707
0.612
216
22.0
0.1
10.2
5
172
0.098
0.805
0.805
125
12.3
0.1
9.8
4
136
0.077
0.882
0.882
64
4.9
0.0
0.0


















0
12
0.007
0
0
0
0
Totaluri
1763
1.000
8.171
1044.8
Comentariu
Vârfuri la
u
m
=
(u

3
)
1/ 3
6.2 ms
-1
8.2 ms
-1
= 10.1 ms
-1
0.00
0.02
0.04
0.06
0.08
0.10
0.12
0
2
4
6
8 10 12 14 16 18 20 22 24
Viteza vântului u (m
s
-1
)
Π
u
/ (ms
-1
)
-1

Observat
Rayleigh
Vârf
Medie
Fig. 7.7
Distributia de probabilitate a vitezei vântului împotriva punct. Date pentru North
Ronaldsay de la Barbour (1984).
date măsurate (Tabelul 7.2);
- - - distribuție Rayleigh echipate pentru a se potrivi viteza medie u– . Rețineți că valoarea medie a
vitezei vântului (8.2 m/s) depășește cele mai probabile viteza vântului (6.2 m/s); a se vedea
Exemplul 7.1, (7.28) și (7.29).
TWIDELL PAGINARE.indb 250
01/12/2014 11:37
§7.3 Caracteristicile vântului
251
1.0
0.9
0.8
0.7
0.6
0.5
0.4
0
2
4
6
8 10 12 14 16 18
Viteza vântului u'/m s
-1
Φ
u
> u'
20 22 24 26
0.3
0.2
0.1
Fig. 7.8
Probabilitatea de viteze ale vântului mai mari decât o anumită viteza u', de
exemplu, North Ronaldsay.
60
maxim maxim
55
50
45
40
35
30
25
20
15
10
0
2
4
6
8 10 12
8.2
6.2 m s
-1
Φ
u
12,5 m, s
-1
10.1
1
3
u
m
(u
3
)
14 16 18
Viteza vântului u
P
0
Φ
u
ms
-1
20 22 24 26
5
(W m
-2
) (m s
-1
)
-1
P
0
Φ
u
Fig. Comentarii: 7,9
Distribuția de putere în vânt, de exemplu, North Ronaldsay.
TWIDELL PAGINARE.indb 251
01/12/2014 11:37
252
Resursele de vant
0
0
0.2
0.4
0.6
0.9
1.0
1.2
1.4
Putere medie
viteza vântului 8.2 m s
-1
1.6
1.8
2.0
2.2
2.3
2.4
2.6
2.8
3.0
3.2
3.4
0.1 0.2 0.3 0.4 0.5 0.6
Φ
u > u'
0.7 0.8 0.9 1.0
P
0
kW m
-2
Fig. 7.10
Putere pe unitatea de suprafață în vânt împotriva probabilitatea de viteze ale vântului
mai mari decât o anumită viteză u'.
Folosind simbolurile din secțiunea anterioară,
uu
u
( )d
1
d
uu
u
u
u
uu
0


F
=
F
=−
F
>'
'
='

(7.6)
Prin urmare, de principiile de calcul,
u
d
d
uu
u
F
'
= −F
>'
(7.7)
Pentru site-uri, fără perioade lungi de zero vântului (de exemplu la site-uri mai promițătoare
pentru energie eoliană, de obicei, cu u– > 5m/s), două parametru exponențială
funcție poate fi, de obicei, strâns montate pentru a măsura viteza vântului date. O
astfel de funcție, de multe ori folosite în viteza vântului analiză, este funcția Weibull
prezentat în Fig. 7.11, obținute din
u
c
exp
uu
k
F
=

'










>'
(7.8)
TWIDELL PAGINARE.indb 252
01/12/2014 11:37
§7.3 Caracteristicile vântului
253
deci (Weibull):
F=−
F
'
=



−











>'

u
k
c
u
c
u
c
d
d
exp
u
uu
k
k
1
(7.9)
Pentru multe site-uri este adecvată pentru a reduce (7.9) pentru un parametru de
distribuție Rayleigh (numit, de asemenea, distribuția chi-pătrat), prin stabilirea
k
= 2. Deci (Rayleigh):
u
c
u
c
2
exp
u
2
2
F=
−









(7.10)
0
0.1
0.2
0.3
0.4
0.5
0.6
0.7
0.8
0.9
1
0
0.5
1
1.5
2
2.5
3
u/u
Weibull c = 1.128u
Φ
u

k = 1.5
k = 2.0
k = 2.5
0
0.2
0.4
0.6
0.8
1
1.2
c = 0.8u
c = 1.128u
c = 1.4u
0
0.5
1
1.5
2
2.5
3
Φ
u
u/u
Weibull (k = 2)
(b)
(a)
Fig. 7.11
Weibull curbelor de distribuție: (a) Variabile k: curbe de
F
u
pentru c
= 1.128 u– și k = 1.5, 2.0, 2.5.
Curba pentru k
= 2, c / u– = 1.128 este o distribuție Rayleigh. (b) Diferite c: curbe de F
u
pentru
k
= 2 și c / u– = 0.80,1.128,1.40. Deoarece acestea sunt normalizate distribuții de probabilitate, de
aria de sub fiecare curbă este egal cu 1.0.
TWIDELL PAGINARE.indb 253
01/12/2014 11:37
254
Resursele de vant
Pentru o distribuție Rayleigh, ca derivate în (7.21),
c
u
2/
p
=
(7.11)
Astfel încât distribuție Rayleigh (adică distribuția Weibull cu k
= 2)
devine
u
u
u
u
2( )
exp
4
u
2
2
p
p
F=
−









(7.12)
Fig. 7.11 prezinta sub forma de
F
u

pentru diferite valori ale lui k în jurul 2.0. Pentru


site-uri care sunt cel puțin moderat vânt, astfel de curbe de multe ori se potrivesc experimentale
date cu k între 1,8 și 2,3 și c aproape de viteza medie a vântului pentru
site-ul. A se vedea, de asemenea, Fig. 7.7, care compară datele reale pentru North Ronaldsay
, cu o distribuție Rayleigh (adică k
= 2). Adimensional parametrul k se
numește factor de formă pentru că schimbarea de k determină o schimbare de formă,
așa cum se arată în Fig. 7.11(a). Parametrul c, unitate de m/s, este de asemenea, numit
factorul de scară pentru a crește de c se referă la mai repede și atât de puternică a vântului
(vezi Fig.7.11(b)). Notă din (7.8) că, atunci când
F
u>u'
= 1/e = 0.37, (u'/c)
k
= 1,
u'/c
= 1, deci c poate fi obținut ca fiind egală cu viteza vântului de măsurare la
acel moment (a se vedea Problema 7.2).
În distribuție Rayleigh (7.12) este deosebit de util pentru preliminară
de analiză a energiei eoliene dacă singurele date disponibile sunt valorile de
viteza medie a vântului, u–, ca acesta este singurul parametru necesar pentru a se potrivi o Rayleigh
distribuție. (Acest lucru este mult mai ușor decât montarea unui distribuția Weibull, după cum
a subliniat în Derivarea 7.1.) Acest lucru duce la o evaluare preliminară a prezis
producerea de energie electrică din turbine și, prin urmare, economice preliminare
de analiză. Ecuații (7.23), (7.26), (7.28), (7.29), derivat de mai jos, și mai ales
(7.27), care estimează puterea de ieșire direct de la viteza medie,
sunt utile în astfel de analize.
§7.3.5 viteza și direcția Vântului: variația cu timpul
și la distanță
Rapid viteza vântului complot în Fig. 7.4 arată importanța
fluctuații la intervale de ~10 s și mai puțin. Rețineți că încercarea de a extrapola
sau o corela 'instantanee' punct cu una mai târziu își pierde credibilitatea
după aproximativ 10 secunde. Din punct de vedere matematic, de autocorelare a seriilor de timp
a devenit zero la acest 'integrantă scară de timp', așa cum se întâmplă cu
turbulențe. Dacă viteza vântului se corelează dincolo aproximativ 10 s, schimbarea este
de obicei descris ca un 'gust'. Schimbări dinamice în turbulență și rafale
poate duce la deteriorarea stres cicluri pe lame și utilaje de wind
turbine, și, prin urmare, înțelegerea ei este important.
O măsură de toate aceste variațiile în timp este non-dimensional turbu-
lence intensitatea (I ), egală cu abaterea standard de instantanee
TWIDELL PAGINARE.indb 254
01/12/2014 11:37
§7.3 Caracteristicile vântului
255
DERIVAREA 7.1 PROPRIETĂȚI MATEMATICE ALE WEIBULL ȘI RALEIGH DISTRIBUȚII
În general, viteza medie a vântului este


=
F
F


u
uu
u
d
d
u
u
0
0
(7.13)
Pentru distribuția Weibull (7.9), aceasta devine


[
]
[
]
=






u
uu
uc
u
u
uc
u
exp ( / ) d
exp ( / ) d
k
k
k
k
1
0
1
0
(7.14)
Să (u/c)
k
= v, deci dv = (k/c
k
) u
k
-1
du. Ecuația (7.14) devine
u
cv
v dv
v dv
exp
exp
k
1/
0
0



[]
[]
=




(7.15)
Numitorul este unitatea, și numărătorul este în formă de un standard integrantă numește factorial sau
funcția gamma, adică
z
z
ve
v
1
!
d
z
v
v 0


(
)
Γ+=
=

=

(7.16)
Rețineți că funcția gamma este, din păcate, de obicei, scris, ca aici, ca o funcție de (z + 1) și nu z
(consultați Jeffreys și Jeffreys 1966).
Astfel
uc
k
c
k
(1 1/ )
[(1/ )!]
=Γ+
=
(7.17)
Folosind standard de matematică din funcția gamma, valoarea medie a u
n
se calculează, în cazul în care n este un
număr întreg sau un număr fracționar, deoarece, în general, pentru funcția Weibull
u
c
nk
(1
/)
n
n
=
Γ+
(7.18)
De exemplu, valoarea medie a u
3
devine
u
c
k
(1 3 / )
3
3
=
Γ+
(7.19)
din care înseamnă putere în vânt este obținut.
Toate formulele de mai sus se aplică în mod egal pentru cazul special de distribuție Rayleigh (7.12), care este doar
o distribuție Weibull cu k
= 2 și
p
=
c
u
2/
.
Există mai multe metode pentru a obține valorile pentru c și k pentru orice special empirice vânt de distribuție (de exemplu,
a se vedea Rohatgi & Nelson 1994; Justus et al. 1977; Manwell et al. 2010); unele exemple sunt după cum urmează:
1
Se potrivesc de distribuție pentru măsurători meteorologice. De exemplu, dacă u– și u

3

sunt cunoscute, atunci (7.17)


și (7.19) sunt ecuații cu două necunoscute. Moderne de colectare a datelor și analiza online
metode permite valori medii pentru a fi acumulat continuu, fără a stoca înregistrări individuale, astfel încât u–
și u

3
sunt ușor de măsurat.
2
Măsura u– și deviația standard a u despre u–, să dea (u

2
− (u–)
2
) și, prin urmare, a obține u

2
.
3
Complot naturale log a log natural din
F
u"u'
în (7.8) împotriva ln u; panta este k, și, prin urmare
intercepta dă c.
TWIDELL PAGINARE.indb 255
01/12/2014 11:37
256
Resursele de vant
DERIVAREA 7.2 PROPRIETĂȚI MATEMATICE DE DISTRIBUȚIE RAYLEIGH
În (7.17) cu k
= 2,
[
]
(
)
=Γ+
=
uc
c
1 1/ 2
(1/ 2)!
(7.20)
Aici, prin definiție,

()
=


ue
u
1/ 2 !
d
u
1/2
0
De un standard integrantă
p
()
=√
1/ 2 !
/2
Prin urmare, din (7.20) pentru distribuție Rayleigh,
p
=
√=
c
u
u
2/
1.13
(7.21)
Astfel distribuție Rayleigh devine
p
p
F=
−









u
u
u
u
2
exp
4
u
2
2
(7.22)
și de (7.6)

p
F
=
F
=

'










>'
='

u
u
u
d
exp
4
uu
u
uu
2
(7.23)
De asemenea,



p
p
=
F
F
=
− 












u
uu
u
u
u
u
u
u
d
d
2( )
exp
4
d
u
u
3
3
0
0
2
4
2
0
(7.24)
Prin standard integrale din funcția gamma acest lucru reduce la
=
u
Ku
()
3
3
(7.25)
unde K se numește model de energie factor'. Pentru distribuție Rayleigh a (7.22), K
= (6/p) = 1.91; a se vedea
Problema 7.4(c).
Prin urmare,
=
=
u
u
u
()
(1.91)
1.24
3 1/3
1/3
(7.26)
Un foarte util relația dintre viteza medie a vântului și mediu anual de energie în vânt per unitate de suprafață
după cum urmează:
r
r
()
=

P
O
u
u
1
2
0
3
3
(7.27)
TWIDELL PAGINARE.indb 256
01/12/2014 11:37
§7.3 Caracteristicile vântului
257
viteza vântului împărțită la valoarea medie a vitezei vântului, adică (cu rms
fiind root mean square):
uu
am
am
N


=<−>
=

Am
uu
u
N
u
/
{
1
(
)}/
rms
2
=1
1/2
(7.30)
Turbulențe intensitatea este o măsură utilă de peste intervale de timp de câteva
minute; turbine eoliene convenției este de a măsura amplitudinea
orizontală viteza vântului la ~1s intervale de timp și apoi se calculează eu pentru perioade de
10 minute. Valorile de am de aproximativ 0,1 implică un "buna" de vânt, ca de-a lungul
mării, și valori mai mari de aproximativ 0,25 implică un furtunos, mari turbulențe,
vânt, ca și în locații montane. Turbulențe intensitatea coeficientului am
poate fi de așteptat pentru a reduce cu înălțimea deasupra solului. Deși
coeficientul este de obicei mai mare pentru viteza redusă decât vânturile de mare viteză,
vânturi, amplitudine de variante crește odată cu viteza vântului. Există
expresii similare pentru variația de direcția vântului cu timp,
uneori numit 'vânt'.
Prin diferențiere pentru a obține valorile de u pentru maxima în
F
u
(probabilitatea de a avea viteza vântului u) și
F
(u3)
(probabilitatea de u
3
referitoare la putere în vânt), și din nou, folosind standardul integrantă
relațiile dintre funcția gamma (a se vedea Problema 7.4):
p
F
=
=
u
u
u
(max)
(2 / )
0.80
apare la
u
1/2
(7.28)
și
p
(
)
F
=
=
u
u
u
u
(max) apare la
2(2 / )
1.60
u
3
1/2
(7.29)
Notă: Ne cerem scuze pentru complicată notație folosit pentru viteza vântului analiza la ori, dar vom încerca
în această carte să urmeze cele mai frecvente formulare. Este nevoie de multă atenție, cu lungimea de
overbars! Ajută dacă ai "citit" simbolurile în capul tău, de exemplu, u

3
citește medie de cub de
viteza vantului'.
EXEMPLUL 7.2 DISTRIBUȚIE RAYLEIGH DOTATE CU DATELE MĂSURATE
Aplica rezultatele de Derivare 7.2 datelor din North Ronaldsay în Exemplul 7.1.
Soluție
Pentru North Ronaldsay u–
= 8.2 m/s. Prin urmare, prin (7.28), F
u
(max) e la (0.80)(8.2 m/s)
= 6.6 m/s. La
valoarea măsurată de la Fig. 7.9 este de 6,2 m/s.
De (7.29), (
F
u
u
3
)(max) este la (1 .60)(8.2 m/s)
= 13 m/s. Valoarea măsurată de la Fig. 7.9 este de 12,5 m/s.
De (7.26), (u

3
)
1/3
= (1.24)(8.2 ms
-1
)
= 10.2 m/s. Valoarea măsurată de la Fig. 7.9 este 10.1 m/s.
A se vedea, de asemenea, Fig. 7.7.
Comentariu: Împreună cu 'ocular' fit prezentată în Fig. 7.7, aceste comparații numerice. sugerează că o
Distribuție Rayleigh montate la u–
= 8.2 m/s este destul de un bun formularea unui punct de distribuție la acest
vant-ul.
TWIDELL PAGINARE.indb 257
01/12/2014 11:37
258
Resursele de vant
O turbină eoliană, în special medii și mari dimensiuni, nu va răspunde
suficient de repede, sau au proprietăți aerodinamice, să "urmeze" rapide
schimbări în viteza și direcția vântului. Prin urmare, de energie în vânt
turbulențe și de schimbare nu poate fi capturat, dar acest lucru este un avantaj dacă oboseala
prejudiciul este astfel diminuat.
Corelația dată este momentul pentru modificări similare să fie evidente la
site-uri separate; produsul dintre viteza vântului și a timpului de corelare
perioadă se numește coerență distanță. Pentru perioade scurte de timp, să zicem, 10 s,
coerența distanța va fi de obicei mult mai puțin decât o "lungime" de un
parc eolian, astfel încât astfel de variații sunt în medie de. Pentru perioade de aproximativ 30 de
minute, corelația distanță poate fi de aproximativ 20 km; în cazul în care
vântul a producției agricole dispersate pe distanțe de ordinul a 100 km va
, de asemenea, nu s-a corelat, cu variații de la putere nu sunt vizibile pe întreaga
rețea. Numai atunci când coerența distanța devine mai mare decât scara
de grila sunt fluctuații nu netezite prin diversitatea de site-ul
locații. Prin urmare, cele mai multe turbine eoliene și parcurile eoliene sunt
dispersate pe o rețea națională rețea, cu atât mai puțin corelate sunt pe termen scurt
(~ orare) variații și cu atât mai ușor este de a accepta creșterea capacității de
energie eoliană.
§7.4 VÂNT INSTRUMENTE, DE MĂSURARE,
ȘI INSTRUMENTE DE CALCUL PENTRU PREDICȚIE
Standardizate de instrumentație și măsurare a forma baza de ambele
oficial de servicii meteorologice și energie eoliană funcționare. Fostul
au adăugat sarcinile referitoare înregistrări istorice pentru prezent și viitor
înregistrări, și de a lucra într-un cadru internațional, astfel încât orice schimbare
în instrumente și metode de măsurare trebuie să fie considerat mai
cu atenție. Acestea din urmă sunt părtinitoare în primul rând spre comerciale resurse
evaluare la anumite locații și, ulterior, față de on-line de măsurare
în asociere cu operare turbine. Stabilit 'mecanic'
instrumente rămâne pentru continuitatea cu trecutul înregistrări și internaționale
de standardizare; cu toate acestea, moderne, robuste și fiabile instrumente
din ce în ce depinde de proprietățile de fizica solid-state (de exemplu, rezistența
și termometre cu termocuplu), emisie și recepție acustice
și electromagnetice interacționează grinzi (de exemplu, sonar și lasere),
comunicare wireless a datelor și de analiză computerizată și înregistrare.
Toate instrumentele lor specifice inexactități, și necesită calibrare
și factorii de corecție specificate de Standardele Autoritățile și
producătorii.
§7.4.1 Tradiționale stabilite instrumente
Organizația Meteorologică Mondială recomandă oficial
stații ar trebui să măsoare orizontală viteza vântului și direcția vântului la 10 m
TWIDELL PAGINARE.indb 258
01/12/2014 11:37
§7.4 Vânt instrumente, de măsurare, de calcul și instrumente de predicție
259
înălțimea la site-uri cu obstacole în termen de cel puțin 100 m (OMM 2008).
Recomandate instrumente și metode de înregistrare sunt după cum urmează:

cupa anemometre pentru viteza vântului

palete eoliene pentru direcția vântului

înregistrările pot fi scrise, pe înregistratoare de diagramă sau cu date digitale magazine.
Ambele tipuri de instrumente au fost atât de folosit de peste 100 de ani și articole
pot fi achiziționate de la standard modele replicabile. De rotație rata de un
standard cupa anemometru cu o cârpă curată și neatinsă de rulment este direct
proporțională cu viteza vântului; și astfel ambele seturi de instrument dau liniar
lecturi de variabila măsurată. Cu toate acestea, cupa anemometre
trebuie să fie calibrat inițial și cel anual sau bianual intervale, deoarece
rulment poate fi defect sau uzat. OMM recunoaște că alte
instrumente demente există, dar este dispus să facă schimbări de dragul internaționale
de cooperare și coerență cu înregistrări istorice. Ambele tip de
instrument poate fi pe același turn, așa cum se arată în Fig. 7.12.
§7.4.2 Instrument turnuri
Viteza și direcția vântului se măsoară la partea de sus a lamei-sfat
înălțimea destinat turbinelor eoliene. Cu toate acestea, o mare turbină poate fi
mai mult de 150 m de mare, care înălțimea este nerealist pentru un instrument
turn, de obicei, limitată de costul și site-ul la cel mult între 50 m și
100 m de mare. În larg, un instrument turn necesită un independent sub
sea foundation, lumini de avertizare, etc., ceea ce face extrem de costisitoare.
Seturi de viteza vântului, direcția vântului și senzori de temperatură vor fi
plasate la înălțimi succesive până la turnul de la aproximativ 15 m de intervale.
Conexiune pentru înregistratoare de date poate fi de la cablu, dar mult mai probabil prin
wireless. Turnul structura deformează fluxul de vânt în anumite direcții, și
atât de corecțiile sunt necesare.
§7.4.3 viteza și direcția Vântului instrumente comerciale
și de cercetare a folosi
Scara de utilitate turbine eoliene, în general, au un anemometru, palete eoliene
și termometru situat pe partea de sus și din spate a turbinei
nacela.
(a) instrumente Mecanice
Mecanice cupa anemometre și palete eoliene montate pe turnuri
sunt comune, dar mecanic cu elice anemometre sunt adesea
favorizate, care au trei elice în unghi drept; analiza online
calculează viteza vântului, direcția vântului și a componentelor verticale de vânt
viteza.
TWIDELL PAGINARE.indb 259
01/12/2014 11:37
260
Resursele de vant
(b)
Fig. 7.12
Unele instrumente pentru măsurarea vitezei vântului și/sau de direcție.
o
Cupa anemometru și direcția vântului vane montate ca un singur ansamblu
b
convențional sonic anemometru;
c
acustic–doppler SODAR echipamente pentru viteza vântului de măsurare de la aproximativ 30m la 200m înălțime; câmp de operare
powered by solare FOTOVOLTAICE.
d
LiDAR folosit pentru a detecta variațiile de sens opus vântului pentru control prealabil de o turbină (schiță de www.tuvnel.com/
tuvnel/article_measuring_flow_regimes_around_large_wind_turbines_using_remote_sensing_techniques/ de la NEL,
filiala TuV SuD, hq Germania).
(c)
(d)
(a)
(b)
TWIDELL PAGINARE.indb 260
01/12/2014 11:37
§7.4 Vânt instrumente, de măsurare, de calcul și instrumente de predicție
261
(b) instrumente Sonic
Acestea includ următoarele (vezi Fig. 7.12(b)):

Cele cu ultrasunete (sonic) anemometru care are un set de trei strămutate
sunet emițători și un set separat de trei recei vers. Timpul de
zbor al sunetului emis de impulsuri între asociat emițători și receptori
depinde de viteza și direcția de aer care trece prin aparat.

La rezonanță acustică anemometru, care utilizează, de asemenea, cu ultrasunete
valuri, dar aceste forme într-un val în picioare model în poziție orizontală,
diferența dintre cele două discuri. Vântul care trece prin golul denaturează
în picioare model de val și vânt viteza și direcția pot fi obținute
de la faza mutat producția de receptoare. Acustic
rezonanță anemometru este un instrument robust, astfel încât este potrivit pentru fixarea pe
de exploatare turbine eoliene.
(c) Doppler spate-efect de dispersie fascicul de instrumente (vezi Fig. 7.12(c)
și (d))
De detectare și, astfel, măsurarea radiațiilor electromagnetice (fie vizibil
sau infraroșu) sau suna înapoi împrăștiate de praf sau temperatura
neomogenități în aer oferi mai multe metode de măsurare viteza vântului
și direcția. Comune metode sunt bazate pe Efectul Doppler, deoarece
ambele acustice și electromagnetice grinzi sunt schimbate în frecvență dacă
acestea sunt reflectate de un obiect în mișcare. Instrumentele acustice sunt
numit Sonic Detection and Ranging (SODAR), și infraroșu/vizibil-roșu
instrumente cu laser Light Detection and Ranging (LiDAR). La
instrumente sofisticate propaga semnalele de la nivelul solului (nu neapărat
întotdeauna verticală, dar capabil de a scana în direcții diferite) și se înregistrează
în spate-scatter' semnalul de praf, care este mereu în aer. În timpul
zborului între emisia unui puls și returnarea acestuia, împreună cu
unghiul față de verticală, dă distanța de eșantionare. Frecvența
de schimbare de spate-împrăștiate fascicul se referă la viteza relativă de praf
(de exemplu, aer), și este măsurată prin interferență cu neperturbat
de ieșire fascicul. Echipamentul poate fi transportat (și furat!) relativ
cu ușurință. Astfel de instrumente este indispensabil la mare, deoarece viteza vântului
și direcția de măsurare la înălțimi de aproximativ 250 m poate fi folosit pentru a
monitoriza din sens opus vântului pentru o turbină eoliană pentru a controla lama
setări și alte variabile în avans. Rata de eșantionare poate fi foarte
mare; de exemplu, SODAR de eșantionare la 200 ms intervale permite aerului
turbulență să fie măsurată. Beneficiile sunt lipsa de turnuri, mare
gama de măsurare înălțimi și poziții unghiulare, și
rapiditate de date analizate. Cu toate acestea, astfel de echipamente este costisitoare și
necesită expert de calibrare. LiDAR instrumente este utilizat de la sateliți
pentru a produce hărți, așa cum se arată în Fig. 7.2.
TWIDELL PAGINARE.indb 261
01/12/2014 11:37
262
Resursele de vant
§7.4.4 Alți indicatori și instrumente
Există multe alte moduri de a evalua viteza vântului, dintre care unele
pot fi utile pentru educație și studenților lucrări practice (a se vedea Problema
7.5). Beaufort scala vizuală în Tabelul 7.1 este o metodă. Instrumentale
metode includ: (i) tub Pitot utilizat pentru a măsura viteza de aeronave; (ii)
hotwire senzor (temperatura și, prin urmare, rezistența variază în funcție de pierderea de căldură
în curentul de aer); (iii) Tela zmee cu calibrate lega vigoare; (iv) tatter steaguri
(stânga pentru luni și comparații și ratele de 'distrusă' a remarcat, așa cum este folosit
de către silvicultori pentru a identifica locuri protejate pentru plantare); (v) trageți sfere,
e.g. în tuneluri de vânt. Mai puțin formale, dar revelatoare metode includ: (a)
bile de pingpong pe siruri de caractere ca trage sfere, și (b) de funcționare de la direcția vântului deține
o batistă care devine verticală atunci când rulează la vânt
viteza (mare distracție, nu a uitat).
§7.4.5 instrumente de Calcul pentru evaluarea energiei eoliene
potențialul
Imposibilitatea de cuprinzătoare masuratori de vant pentru o întreagă
regiune sau țară, înseamnă că computerul modelare (simulare) este singura
metodă de evaluare și predicție, cu model calibrat de la
relativ puține seturi de date măsurate. De asemenea, pentru complicată vânt
proprietăți de site-ul real, inclusiv în cadrul unui parc eolian. Două exemple
sunt după cum urmează:

NOABL (Numerice de Analiză Obiectivă Boundary Layer) de modelare. La
modelul de bază menține constanței masă de aer ca modelate
de vânt fronturi muta pe teren complex. Prin urmare, relativ puține
puncte de calibrare permite medie a vântului să fie prezis în orice altă
poziție în rețea; pentru un exemplu, a se vedea www.rensmart.com.
Rețineți, totuși, că perturbări locale (de exemplu, copaci și clădiri) nu este
considerat, care este un handicap major.

WAsP (Vânt Atlas Analiză și Program de Aplicație) este un
utilizat pe plan internațional set de pachete de programe (modele) pentru Pc-uri produse
de către Departamentul de Energie Eoliana (ex Riso) la Universitatea Tehnică
din Danemarca. Metoda de bază este de a accepta încredere datele de vant
de la un înființată stația meteorologică a elimina efectele
topografiei locale și elevație, transfera aceste date necesare
site-ului, se adaugă efectele altitudinii, relieful și obstacole (de exemplu,
clădiri) și în cele din urmă prezice vânt legate de parametrii (de exemplu, medie
viteza vântului lună de lună). Un set complet de programe oferă multe
mai simulate aspectele legate de energia eoliană.
O astfel de modelare este esențial. Cu toate acestea, aveți grijă să experiență reală de vânt
condiții, în special forța de gale-forța vântului. Computerul modelate
de vânt nu va speria; real vântul va.
TWIDELL PAGINARE.indb 262
01/12/2014 11:37
§7.4 Vânt instrumente, de măsurare, de calcul și instrumente de predicție
263
§7.4.6 pe termen Scurt, previziuni
Cunoștințe de viitor viteza vântului este necesar pentru o varietate de scopuri și
într-un interval de ori înainte. Cu semnificative de energie eoliană instalată
a capacității, astfel de predictii sunt necesare: (i) de minute înainte furnizarea de informații
pentru individuale de turbine eoliene, sisteme de control; (ii) de ore înainte pentru
rețeaua electrică a operatorilor de planificare pe termen scurt piețe; (iii) zile înainte
pentru operatorii de rețele de putere stație de programare; (iii) luni înainte pentru
puterea programarea de întreținere; (iv) ani înainte de fermă eoliană de finanțare.
De asemenea, au nevoie de previziuni de ore pentru lunile următoare sunt financiară
a operatorilor de vânzare și cumpărare de energie electrică pe piața energiei electrice.
Corelarea lunare și anuale viteze ale vântului între un vânt de putere
- ul și standard de stații meteorologice (așa cum este arătat în §7.4.5) permite
estimările să fie făcute de generare a energiei electrice și rezultă finanțe; acest
obicei satisface nevoile proprietarilor și a finanțatorilor.
Cu toate acestea, rețeaua de energie electrică, operatorii de rețea au nevoie de informații pentru a planifica
înainte de intrări de producție pentru a garanta că producția totală consumabile
la cerere variate (§15.4). Astfel de rețele de acoperire mare, regionale, naționale
și internaționale zone, atât de semnificativă în medie apare (a se vedea Caseta 15.5).
În practică, rețeaua nevoie este de a prezice intrare de la mai multe vânt
ferme distribuite pe domenii de aproximativ 300 m x 300 m sau mai mare. Multe
informații despre condițiile de vânt de la ore la câteva zile înainte este
disponibil de la serviciile meteorologice; aceasta este, de obicei disponibil ca hărțile de
pe site-urile gratuit. Aceleași informații pot fi achiziționate
în format digital și online de intrare pentru analiza de calculator și, dacă este necesar,
de control. Cu, să zicem, de la 10 la 100 de ferme eoliene, repartizate pe o mare
regiune, numărul total de turbine eoliene, operațiunea este foarte de încredere și de încredere,
spre deosebire de oferta de la o centrală stație de putere care pot fi decupate
dintr-o dată, de exemplu, conectarea la rețea defecte și defecțiuni operaționale. De
incertitudine cu privire la astfel de energie eoliană nu este starea masini,
dar starea de vânt.
De servicii meteorologice folosi foarte mare capacitate de calcul pentru
prezicerea vremii din ore la mai multe zile înainte, folosind stabilit
modele. Astfel de modele variază de la post-predicție corelații cu trecut
a înregistrat viteza vântului (de exemplu, folosind auto de regresie medie mobilă
(ARMA), matematică, și rețele neuronale artificiale teorie (ANN)) a
modelelor hidrodinamice calcularea cât aer diferențele de presiune și căldură
intrări că masele de aer pentru a muta. O astfel de analiză oferă, în general,
foarte fiabile informații pentru zonele acoperite de rețeaua rețelelor.
TWIDELL PAGINARE.indb 263
01/12/2014 11:37
264
Resursele de vant
ÎNTREBĂRI RAPIDE
Notă: Răspunsurile la aceste întrebări sunt în textul de la secțiunea relevantă
din prezentul capitol, sau poate fi ușor dedusă din aceasta.
1
Pentru energia eoliană, ce gama de viteze de vânt este cel mai productiv?
2
Indica populate regiuni din lume cu cele mai bune de energie eoliană
de resurse.
3
Ce alți factori beneficia de energie eoliană de resurse?
4
De ce este vântul pe turbine eoliene întotdeauna clasificate ca turbulent?
5
Este creșterea vitezei vântului cu înălțimea liniar? Dacă nu, ceea ce este
relația?
6
Descrie efectele de copaci și clădiri în apropiere de turbine eoliene,
și, prin urmare, rezuma sfaturi despre amplasarea de turbine eoliene.
7
Care este rolul Organizației Meteorologice Mondiale privind
energia eoliană?
8
Este imposibil de a prezice cu exactitate viitorul viteza vântului de timp, așa
cum este posibil să se prevadă anual de energie eoliană energie electrică de vânzări, astfel
că o rentabilitate financiară pot fi oferite investitorilor?
REZUMAT CAPITOL
Pentru a evalua în mod corespunzător puterea susceptibile de a fi produse de anumite turbine, în special site-uri necesită
măsurători atente la care site-ul pe cel puțin 12 luni de la mai multe înălțimi. Acest lucru este pentru că viteza vântului
variază puternic cu timpul pe perioade de la câteva secunde la anotimpuri și ani, și pe distanțe ~1 km
, în general, și ~100 m în zone de deal (complex) de teren. Predicție a acestor vânturi este posibil, cu informații de la
oficial stațiile meteorologice cu ajutorul măsură de-a corelat-prezice metodologia, și de la utilizarea Vântul
Atlas de tehnici de calcul. În practică, anuală a vitezei vântului de evaluare poate duce la incertitudinea de cel
puțin 20% de la an la an, datorită o combinație de "natural" de variație și schimbările climatice.
Un indicator-cheie este viteza medie a vântului
u
; un site bun pentru energia eoliană va avea
u
>
~
5 m / s la 10 m
înălțime. Publicat de vânt atlase da un ghid preliminare pentru site-uri bune. La nivel global, astfel de site-uri sunt deosebit de
abundente la latitudini ~40° în cazul în care vânturile sunt puternice (de exemplu, America de Nord vest, Nord-vest
Europa, inclusiv marea Britanie și Irlanda și Noua Zeelandă).
Distribuția de probabilitate a vitezei vântului este important, deoarece puterea vântului este proporțională cu
viteza vântului tocata (u
3

) și pentru că mai sus-medie vânturile contribuie în mod disproporționat mai multă putere. Pentru
o anumită viteza medie a vântului, relativ simple funcții matematice (de Weibull sau Rayleigh distribuții)
dă un nivel acceptabil se potrivesc pentru distribuția de probabilitate la cele mai multe site-uri, permițând astfel o analiză
preliminară a
lunar și anual de energie eoliană potențialul.
Serviciile meteorologice au măsurătorile de rutină de teren pe bază de viteza și direcția vântului peste mulți
ani, folosind cupa anemometre, dar de multe ori la site-uri care nu sunt adecvate pentru energie eoliană (de ex. aeroporturi) și la
înălțimi
mai mici decât cele de multe turbine (viteza vântului crește semnificativ cu înălțimea de la sol în
înălțime regiuni de rotoare de turbină). Offshore informații nu este atât de bine stabilit, astfel încât în continuare de măsurare
este necesar, de obicei, necesită mai sofisticate instrumente (de exemplu, cu sonic radar și înapoi-scatter),
ca fac măsurători la scurt (~al doilea) intervale de timp, ca și pentru analiza de turbulențe. Astfel sofisticate
instrumente vă permite apropierea câmpului de vânt se apropie de o turbină să fie măsurate pentru turbina de control.
Modelele de calculator (de exemplu, Viespe) sunt utilizate pe scară largă pentru a interpola de la câteva site-uri de măsurare a
propus turbina site-uri din apropiere. Vântul predicție este important, în special pentru energie electrică, operatorii de rețea.
TWIDELL PAGINARE.indb 264
01/12/2014 11:37
Probleme
265
9
De ce sunt mai multe site datele necesare pentru a se potrivi o distribuție Weibull decât
o distribuție Rayleigh? În ce condiții are asta de a face
distribuție Rayleigh deosebit de util?
10
Care sunt instrumente de suflat tradiționale pentru stațiile meteorologice
și ce metode instrumentale au fost adăugate la acestea pentru
industria energiei eoliene?
PROBLEME
Notă: *indică o "problemă", care este deosebit de potrivit pentru clasa de
discuție sau grupul de tutoriale.
7.1
Folosind ecuația (7.3), cu b'
= 1/7 = 0.14, compara
creșteri proporționale în viteza vântului așteptat prin adăugarea de 50 m, la 50 m
și 100 m-turn înalt.
7.2
Consultați Tabelul 7.2 coloana 4, și Fig. 7.8. Explica cum un grafic
de
uu
F
>'
împotriva u' este obținut din datele de teren, folosind on-line
de colectare a datelor. Atunci din (7.8) și (7.9) dovedesc că, atunci când
e
1/
1/ 2.72 0.368
uu
F
=
=
=
>'
apoi
u
c
'=
, unde c este scara
factor.
7.3
Fluxul de aer în vântul va fi turbulentă dacă numărul Reynolds
R
≥ 2000 (a se vedea §R2.5). Calcula viteza maximă a vântului pentru
flux laminar în jurul valorii de un obstacol de dimensiuni 1.0 m. Este laminar
flow realist pentru turbine eoliene?
7.4
Pentru o viteză a vântului model urmărind o distribuție Rayleigh, dovedi
că:
(a) Cele mai probabile viteza vântului este
u
0.80
.
(b) Cele mai probabile de putere în vânt are loc la o viteză a vântului
de
u
1.60
.
(c)
u
u
6
()
3
3
p
=
în cazul în care
u

3

este de u
3
și
u

este de u , și așa
u
u
()
1.24
3 1/3
=
Notă: Necesită matematică la nivel de Derivare 7.2.
*7.5
Experiment cu metode mai ieftine de măsurare a vitezei vântului, cum ar
fi: (i) tatter steaguri (stânga pentru luni și comparații și ratele
de 'distrusă' a remarcat); (ii) o masă de ping-pong mingea agățat pe un șir de caractere în
vânt ca un exercițiu de învățământ pentru a calibra; (iii) rularea direcția vântului
ține o batistă pe verticală atunci când rulează la vânt
viteza (mare distracție, nu a uitat).
TWIDELL PAGINARE.indb 265
01/12/2014 11:37
266
Resursele de vant
BIBLIOGRAFIE
General
Manwell, J. F., McGowan, J. G. și Rogers, A. L. (2010, 2nd edn) Energia Eoliană a Explicat, John Wiley & Sons,
New York. Are un capitol întreg pe vânt caracteristicile și resursele.
Rohatgi, J. S. și Nelson, V. (1994) Vânt Caracteristici: O Analiză pentru Generarea de Putere, Burgess
Publishing, Edina, MAMĂ. Un seminale text.
În mod special referire
Barbour, D. (1984) Studiul Energiei de Insula North Ronaldsay, Orkney, MSc tezei de doctorat, Universitatea din Strathclyde.
Bowden, G. J., Barker, P. R., Shestopal, V. O. și Twidell, J. W. (1983) La funcția de distribuție Weibull și vânt
puterea statistică, Inginerie de Vânt, 7, 85-98.
Jeffreys, H. Jeffreys, B. (1966) Metodele Fizicii Matematice, Cambridge University Press, Cambridge.
Cu atenție a prezentat textul de avansate de matematică pentru ingineri, etc.
Justus, C. G., Hargreaves, W. R., Mikherl, A. S. și Morminte, D. (1977) Metodele de estimare a vitezei vântului
frecvența de distribuție', Journal of applied Meteorologie, 17, 673-678.
Petersen, E. L. (1975) Pe Energia Cinetică a Spectrului de Mișcări Atmosferice în Stratul Limită Planetar,
Raportul 285 de Vânt site-ul de Testare, Riso, Danemarca.
Organizația Meteorologică mondială (OMM) (1981) de Meteorologie Aspectele legate de Utilizarea de Vânt ca o Energie
Sursă, Notă Tehnică Nr. 175, WMO, Geneva, Elveția.
OMM (2008) Ghid pentru Instrumente Meteorologice și Metode de Observare, Meteorologică Mondială
Organizația, Geneva, Elveția.
Global climatologie de vânt
Chatfield, R. (2000) 'mișcări Atmosferice și efectul de seră', în G. W. Ernst (ed.), Sistemelor De Pe Pământ –
Procese și probleme, Cambridge University Press, Cambridge.
Lagoa rj, J. P. și Oort, A. H. (1992) Fizica Climatice, Institutul American de Fizica, New York.
Reviste și site-uri web
Vântul atlase; fundal, metode și disponibilitatea (www.windatlas.dk/).
TWIDELL PAGINARE.indb 266
01/12/2014 11:37
CONȚINUTUL
Obiective de studiu
268
§8.1 Introducere
269
§8.2 tipuri de Turbine și termeni
272
§8.2.1 ax Orizontal turbinele eoliene
(HAWTs)
274
§8.2.2 turbine eoliene cu ax Vertical
(VAWTs)
276
§8.2.3 Concentratoare, difuzoare și
capace
277
§8.3 impulsului teoria
277
§8.3.1 extragerea de Energie; Lanchester-
Betz-Zhukowsky teoria
277
§8.3.2 Axială (forța axială) pe vânt
turbine
281
§8.3.3 Cuplu
283
§8.3.4 Drag masini
285
§8.4 cinetic teoria
286
§8.4.1 Concepte
286
§8.4.2 Cuplu, putere și sfat-viteza
raportul de vedere angular
momentum
287
§8.5 Dinamic de potrivire
289
§
8.5.1 Optime viteza de rotație; sfat-viteza
raport
λ
289
§8.5.2 Extensii ale impulsului
teoria
293
§8.6 Lama element de teoria
295
§8.6.1 Calcul de ridicare și trageți
forțele pe o lamă element
295
§8.6.2 Calculul forțelor și de cotitură
cuplul pe un întreg lama
298
§8.6.3 Implicații
298
§8.7 Putere de extracție de o turbina
299
§8.8 generarea de energie Electrică
303
§8.8.1 Bază
303
§8.8.2 Clasificare de energie electrică
sisteme care utilizează energie eoliană
305
§8.8.3 Eoliene: interioare și
offshore
308
§8.8.4 aspecte Tehnice pentru grid-connected
turbine eoliene
311
§8.8.5 energie Eoliană contribuția la
naționale de energie electrică 311
§8.8.6 scară mai Mică, sisteme și
independent de proprietari
312
§8.9 putere Mecanică
314
§8.9.1 transport Maritim
314
§8.9.2 morărit
314
§8.9.3 de pompare a Apei
315
§8.9.4 producerea de Căldură prin frecare
316
§8.10 Sociale, economice și
considerente de mediu
316
Rezumat capitol
318
Întrebări rapide
318
Probleme
319
Note
322
Bibliografie
322
Caseta 8.1 Confruntă cu lift si
forțele de frecare
290
Caseta 8.2 Multimode wind power sistem
cu load-control de gestiune
la Fair insula, Scoția
313
CAPITOLUL
8
Tehnologia energiei eoliene
TWIDELL PAGINARE.indb 267
01/12/2014 11:37

www.shahrsazionline.com
268
Tehnologia energiei eoliene
OBIECTIVE DE STUDIU

Identificați principalele tipuri de turbine eoliene.

Înțeleg motivele fizice ce
puterea vântului este proporțională cu cubul
vitezei vântului, dar nu mai mult de 60% din
putere în care se apropie de vânt poate fi
extras printr-o turbina.

Înțelege sensul cut-in speed, evaluat la
putere, și cut-out viteza de o turbina.

Apreciez motivele din spatele rapidă
creștere a instalat sisteme eoliene,
în special în parcuri eoliene.

Apreciez potențialul de mai avansate
metode pentru o mai bună modelul turbinei.
LISTA DE FIGURI
8.1
Creșterea în energie eoliană: lumea capacitate instalată (GW) și anual de energie electrică (TWh).
269
8.2
Vitezelor și forțelor de la o secțiune de o rotație de palete de turbine.
272
8.3
Clasificarea de vânt mașini și dispozitive: (a) axa orizontală; (b) axa verticală; (c) concentratoare.
275
8.4
Putere în vânt.
278
8.5
Lanchester-Betz-Zhukowsky modelul de expansiune airstream prin rotorului turbinei, modelat
ca un dispozitiv de acționare disc.
278
8.6 coeficientului de Putere C
p
în funcție de factor de inducție - o. 280
8.7
Tracțiune pe turbine eoliene.
281
8.8
Cuplul coeficientul C
G
vs sfat-raportul de viteza
λ. 285
8.9 (a) Un yacht cu vele, folosind drag vigoare a acesteia spinnaker să navigheze în direcția vântului. (b) Yacht sailing în
vântul utilizarea lift vigoare (c) Idealizat trage masina cu balamale clape pe o rotație a centurii.
285
8.10 Acționare disc, arată unele dintre parametrii-cheie.
287
8.11 (o) Parte din fluxul de aer de o mașină în mișcare. (b) Ridicați și trageți forțe pe o suprafață netedă aripă de avion.
290
8.12 Definiție schiță de unghiuri și forțele, în căutarea de-a lungul unei turbine eoliene lama de la vârful său în timp ce se rotește
într-un plan perpendicular pe departe din sens opus (în amonte) vânt.
290
8.13 Turbina viteză (frecvență) și puterea de captare.
292
8.14 schiță Orientativă a coeficientului de putere C
p
ca o funcție de tip-raportul de viteza
λ pentru o serie de vânt
tipuri de turbine.
293
8.15 coeficientului de Putere C
P
comparativ cu factor de inducție un
= 1 – u
1
/u
0
, ca dată de către impulsului model. 294
8.16 (a) Element de secțiune a unui lama (b) Ridicați și forțele de frecare (c) Arată lama poftă de mâncare. (d) Vector triunghi
de viteze la lama element. (e) Comerciale turbina.
296
8.17 Modelat airspeeds la rotor la distanța r de la axa.
299
8.18 de turbine Eoliene de putere, curba de operare regiuni și putere de performanță.
300
8.19 coeficientului de Putere C
p
. 302
8.20 Microgenerare.
305
8.21 Unele opțiuni de alimentare pentru sisteme stand-alone.
306
8.22 Vânt/diesel aprovizionare moduri.
307
8.23 Parte din Buffalo Ridge eolian din Minnesota, statele UNITE ale americii.
309
8.24 fermele eoliene Offshore.
310
8.25 pompare a Apei de mecanica direct pe link-ul de la un multi-blade turbina.
315
LISTA DE TABELE
8.1
Tipic turbine eoliene generatoare de caracteristici la puterea nominală de peste 12 m/s viteza vantului.
271
8.2
Comparație de aripă de avion cu lama de o turbină eoliană
289
8.3 O clasificare de turbine eoliene de energie electrică sisteme
305
TWIDELL PAGINARE.indb 268
01/12/2014 11:37
§8.1 Introducere
269
1,000
900
800
700
600
500
TWh
400
300
200
100
100
0
1985
1990
1995
2000
2005
2010
2015
2020
0
50
150
200
250
GW
300
350
400
450
500
Fig. 8.1
Creșterea în energie eoliană: lumea capacitate instalată (GW) (curba superioară) și anual de energie electrică
generație (TWh) (inferioară a curbei).
Sursă: Datele pentru anul 2012 de la BP. Analiză statistică; extrapolarea liniară dincolo.
§8.1 INTRODUCERE
Capitolul 7 considerat vânt; acum vom studia tehnologia pentru
valorificarea resurselor pentru lucru mecanic (de exemplu, de pompare a apei) și
pentru energie electrică (adesea numit doar "putere"). Turbine eoliene de energie electrică
generatoare, abreviat la 'turbine eoliene', sunt dominante masini,
fabricate la nivel mondial, cu capacități variind de la zeci de wați
să se apropie de zece megawați, cu diametre de aproximativ 1 m, la aproximativ
150 m. Cu toate acestea, în unele zone, mecanice-doar masini sunt încă
vitale pentru pomparea apei. Astăzi, turbinele eoliene sunt acceptate ca
bază de generare de energie pentru rețeaua de utilități a rețelelor din țările cu
potențialul eolian (e.g. în Europa, statele UNITE ale americii, și părți din India și
China); alte țări sunt în mod constant în creșterea lor, capacitatea de energie eoliană.
Mai mici turbine eoliene sunt comune izolate și autonome puterea
de producție.
Creșterea rapidă din întreaga lume turbina capacitatea de generare de energie este
prezentată în Fig. 8.1. Între 2000 și 2010, rata medie de creștere anuală
rata a fost de 27% (compus), care este remarcabil de mare. Începând cu anul 2002,
mai multă capacitate de producție este instalat pe mare în offshore
wind farms, unde adâncimea este de < ~50m.
Analiza noastră în următoarele secțiuni prezintă bază de turbine eoliene teorie;
un aspect cheie este de a determina adimensional factori de scalare care sunt atât de
importante în inginerie (de exemplu, pentru aplicarea rezultatelor din experimente
pe modele fizice de un tunel de vânt pentru proiectarea și operarea foarte
mare de structuri). De exemplu, a se vedea §8.3; o turbina interceptarea o
secțiune Un vânt de viteză u
0
și densitatea
r produce o putere nominală
maxim în funcție de
rp
=
P
D
uC
(
/ 4)( )
T
P
1
2
2
0
3
(8.1)
TWIDELL PAGINARE.indb 269
01/12/2014 11:37
270
Tehnologia energiei eoliene
Aici C
p
este adimensional factorul de eficiență numit coeficientului de putere.
Rețineți că puterea P
T
este proporțională cu O și cu cubul vitezei vântului
u
0

. Astfel, întrucât dublarea O poate produce de două ori la putere, o dublare


a vitezei vântului produce de opt ori potențialul de putere. Puterea
coeficientul C
p

de asemenea, variază în funcție de viteza vântului de mașini individuale. Din


punct de distribuție este oblică (vezi Fig. 7.11), în orice moment viteze
mai puțin decât media sunt mai multe sanse decat viteze mai mari decât media.
Prin urmare, design optim de dimensiuni de rotorul și generatorul la un anumit
site-ul depinde de necesarul de putere, fie pentru a maximiza generate
de energie pe an sau pentru a oferi putere frecvente. După cum este evident din (7.27)
pentru comune viteza vântului distribuții, rata medie anuală de energie de la o
turbină eoliană aproximează la
P
COu
()
T
P
0
3
r

(8.2)
în cazul în care
u
0
este viteza medie a vântului.
1

Ansamblul cuprinde rotorului, sa compensată de energie electrică


generator și alte echipamente este de obicei numit o turbină de vânt, ca în această
carte.
2

Maximă putere nominală capacitate de o turbină eoliană este dat


pentru o anumită 'evaluat' viteza vântului, de obicei de aproximativ 12 m/s. La această
viteză, producția de energie electrică de aproximativ 0,3 kW/m
2
de secțiune transversală ar
fi de așteptat cu putere coeficienții C
p

între 35 și 45%. Cele
optime viteza de rotație depinde de raportul dintre lama-sfat viteza la
viteza vântului, astfel încât mașini mici se rotesc rapid și mașini mari
se rotească încet. Tabelul 8.1 dă contur detalii de dimensiune mașină. Mașinile
sunt de așteptat să dureze cel puțin 20 la 25 de ani și costa aproximativ 1200 Euro
(aproximativ $US1500) per kW capacitate nominală, ex-fabrică. Atunci când este instalat în
locații cu vânt și financiar dat de credit pentru a nu polua, putere
de producție este competitiv cu cele mai ieftine forme de altă generație
(vezi Anexa D).
Energia eoliană în scopuri mecanice, adică de frezat și de pompare a apei,
a fost stabilit pentru multe sute de ani. Energie eoliană
generatoare data din jurul anului 1890, cu cele mai timpurii de dezvoltare de la
1930 la aproximativ 1955. În acest moment dezvoltarea aproape a încetat din cauza
disponibilitatea de petrol ieftin, dar interesul s-a deșteptat și a crescut
rapid din 1973. Câteva dintre mașinile mai vechi păstrate de operare pentru
mai multe zeci de ani (de exemplu, la Gedser 100 kW, 24 m diametru mașină
în Danemarca, construit în 1957). Creșterea producției din anul 1980, a
beneficiat în mare măsură de utilizarea de solid-state electronice, compozite,
materiale, proiectare asistată de calculator și optimizarea site-ului.
Un important criteriu de proiectare este nevoie pentru a proteja aparatul împotriva
daune în vânturi foarte puternice, chiar dacă astfel de vânturile puternice sunt
relativ rare. Vântul forțe au tendința de a crește odată cu pătratul
vitezei vântului. De la 1 la 50 de ani gale viteza va fi de cinci la zece ori
viteza medie a vântului, considerabil asupradesign trebuie să fie încorporată
TWIDELL PAGINARE.indb 270
01/12/2014 11:37
§8.1 Introducere
271
Tabelul 8.1
Tipic turbine eoliene generatoare de caracteristici la putere nominală P
T
în 12 m/s viteza vantului.
Datele calculate presupunând coeficientului de putere C
P
= 30%, densitatea aerului
r = 1,2 kg/m, vârful de viteză de raportul λ = 6. Nominală
putere
r

()
(
)
p
=
1
2
C
P
D
u
/4
T
2
0
3
p
. Prin Urmare D
= (2.02 m)
√(P/1 kW), T = (0.0436 s m
-1
)D.
Clasa
Mici
Intermediar
mare
Putere nominală P
T
/kW
10
50
100
250
500
1000
3000
6000
Diametrul D/m
6.4
14
20
32
49
64
110
160
Perioada T/s
0.3
0.6
0.9
1.4
2.1
3.1
4.8
6.8
pentru rezistența structurală. În plus, viteza vântului variază, deci,
considerabil oboseala daune pot apărea, în special legate de lame și conduce
trenul, de la stres frecvente cicluri de greutate de încărcare (aproximativ 10
8

cicluri de
peste 20 de ani de funcționare de 20 m diametru, ~100 kW de putere nominală a turbinei,
mai puțin pentru mașini mai mari) și de fluctuațiile și turbulențe în
vânt. Ca la aparate sunt construite în creștere dimensiunea, cuplul la
ax principal devine un factor de limitare.
Contribuția energiei eoliene pentru alimentarea cu energie electrică este în mare măsură con-
amendat pentru locuri cu
u
0

5 m/s, care sunt cele mai frecvente în la mijlocul anului latitudine


țări, așa cum este indicat în Fig. 7.2. În 2012, din totalul eoliană instalată
capacitate de putere ale lumii, 39% a fost în Europa (mai ales în Germania și
Spania), 27% în China, și 21% în SUA; țările cu cea
mai mare capacitate eoliana pe cap de locuitor
3
au fost Danemarca (750 W/
persoane), urmată de Spania (485 W/persoană).
Final lume folosesc de energie eoliană nu poate fi estimată din
un mod semnificativ, deoarece este atât de dependentă de succesul și
acceptarea de mașini și adecvată consumului final de energie sisteme. Cu toate acestea,
fără a sugera schimbări majore în infrastructura electrica,
estimările oficiale din potențialul eolian pentru alimentarea cu energie electrică din
Marea Britanie sunt cel puțin 25% din totalul ofertei, o proporție
atins acum în Danemarca. Cu modificările în sistemele informatice (de exemplu, având
răspândită de management de încărcare și conexiune cu hidro depozitare),
semnificativ mai mare de penetrare este posibil. Autonome de energie eoliană
sisteme au un mare potențial ca înlocuitori pentru uleiul folosit la încălzire sau
pentru generarea de energie electrică de la motoarele diesel. Aceste sisteme
sunt deosebit de aplicabil pentru distanță și comunități de pe insulă, și au tendința
de a folosi aceleași utilaje ca și pentru parcurilor eoliene conectate la rețea și pentru
microgenerare.
Mult din acest capitol descrie fizica de bază de turbine eoliene și
cât de mult puterea pe care o pot lua de vânt. §8.3, §8.4 și §8.6
conține analiză matematică, cea mai mare parte cu algebra elementară. O astfel de
analiză este marcat ca 'Derivatii', astfel încât textul nemarcat de cheie
rezultate și fizice interpretare poate fi citit în mod continuu.
<
TWIDELL PAGINARE.indb 271
01/12/2014 11:37
272
Tehnologia energiei eoliene
§8.2 TIPURI DE TURBINE ȘI TERMENI
Numele a diferite tipuri de turbine eoliene depind de
geometria de construcție, și aerodinamica de vântul care trece în jurul valorii de
lamele; de asemenea, numit aripi sau elice. De bază aerodinamica este
descris în Revizuire 2 (de exemplu, Fig. R2.6), deoarece, în ciuda aparențelor, în
mișcare relativă de aer cu o turbina lama secțiune este în esență același
ca și cu o aripă de avion secțiune. Fig. 8.2 prezintă o lamă secțiune de o
axă orizontală de turbine eoliene lama; aceleași principii se aplică vertical
axa turbine. Pentru Fig. 8.2(c) imaginați-vă uita în jos pe o secțiune
de o lama verticală de rotație. Secțiunea se rotește aproximativ
perpendicular îndepărtat din sens opus vântului de viteza u
0

. Din cauza naturii sale


proprii de circulație, lama secțiunea experiențe de aer din sens opus în raport
viteza v
r.
Comparația poate fi făcută cu o aripă de avion secțiune
întorcând pagina, astfel încât Fig. 8.2(c) a relativă viteza aerului v
r
orizontală.
Ca aerul este perturbat de lama, o forță acționează în care este rezolvată în
două componente:

De forța de frecare F
D
este componenta în linie cu viteza relativă v
r.

La lift forță F
Am
este componenta perpendiculară F
D
. Utilizarea
cuvântul "lift" nu înseamnă că F
Am
este neapărat în sus, și derivă
din forța echivalentă pe o aripa de avion.
Vedere din față
(secțiunea indicat)
Hub
v
v
u
o
Vedere în perspectivă
(a)
(b)
Ridicare la putere
α
φ
φ
γ
Direcția de
rotație a rotorului.
Sfat viteza R
Forța de frecare
F
forța de tracțiune
F
roti
În amonte de vânt
viteza u
o
v
r
În căutarea " jos " pe o lamă secțiunea de
sfat, de o rotație a turbinelor eoliene lama.
Lungimea lamei R = raza rotorului
rata de Rotație (radiani/s)
(c)
W
Ω
Fig. 8.2
Vitezelor și forțelor de la o secțiune de o rotație a turbinei lama: (a) vedere din Față de axa orizontală de palete de turbine, rotirea
secțiunii
viteza v; (b) vedere în Perspectivă, arată neperturbat viteza vântului u
0
. (c) Detaliu de fluxul de aer vitezelor și forțelor la un vânt
turbine blade secțiune. Neperturbat viteza vântului u
0
; raport de viteza vântului v
r

; lama de setare/unghi de înclinare


g; unghiul de atac o; afluxul
unghi
f. Lama este rotirea la W radian/s (360/2p grade/s) în unghi drept spre amonte direcția vântului. Vârful lamei
se mută la sfat-viteza v
= R
W în planul de rotor la unghiuri drepte față de amonte direcția vântului.
TWIDELL PAGINARE.indb 272
01/12/2014 11:37
§8.2 tipuri de Turbine și termeni
273
Următorii pași sunt pentru a rezolva ("split"), ambele F
Am
și F
D
:
1
De-a lungul axei rotorului, cu suma de rezolvat componente
fiind răsturnare axială F
forța de tracțiune

. Pentru că trageți este mic din cauza


la buna suprafețelor aerodinamice, și lift este maximizată de
forma paletei, net vigoare în planul de rotor, F
roti
, este în
direcția de rotație.
2
În planul de rotor, cu suma algebrică (de fapt a lor,
diferența în mărime, deoarece rezolvate componente au opus
direcții) fiind forța de rotație F
roti
. Forța F
roti

transformă
arborele turbinei și permite puterea de a fi extras din
generator conectat.
Poate părea ciudat că rotorul se transformă într-o direcție față de
intrare relativă a aerului. Cu toate acestea, aceeași situație s-a întâlnit cu curse de iahturi;
iahturi poate naviga într - un vânt cu o viteză mai repede decât viteza vântului
datorită rezoluției de forțele de ridicare pe vele. Dacă iahtul naviga cu
vântul din spate, de exemplu, cu o vela vela, apoi barca poate
nu merge mai repede decât vântul (vezi §8.3.4).
Alți factori care afectează interacțiunea între lamă și
vântul includ următoarele:
1
Nevăzut de ochi, mișcarea de rotație a aerului apare ca
airstream curge în jurul lamei. În consecință, distincte vartejuri și
vârtejuri (vârtejuri de aer) sunt create în apropierea suprafeței; vortex vărsare
apare ca aceste maselor în mișcare de rupere în aer liber de la suprafață
și pentru a muta departe, încă de rotație, cu acest airstream. În plus,
semnificativ cinetic este împărtășită la lama, deci, egală și
opusă cinetic este dat de fluxul de aer, care circula
în direcția vântului ca se trezeste. Aceste tulburări sunt disipate după ce a călătorit
aproximativ 10 până la 30 de turbine cu diametre de direcția vântului.
2
Aerul este deranjat de lama de circulație și de rafale de vânt, și
fluxul devine neregulat și tulburat. Această turbulență (a se vedea §R2.5)
are loc înainte și după lame rotative, astfel încât fiecare individ lama
poate fi de multe ori se deplasează în turbulențele create de alte lame.
3
Caracteristicile aerodinamice ale paletelor sunt cruciale; rugozitatea
și proeminențe ar trebui să fie evitate. Rețineți că predominant
twodimensional fluxul de aer peste o aripă de avion devine tridimensional,
și, prin urmare, mult mai complex, pentru o rotație de turbine eoliene lama.
Caracteristicile unui anumit tip de turbine eoliene sunt descrise de
răspunsuri la o serie de întrebări (a se vedea Fig. 8.3). Teoretic
o justificare pentru aceste criterii vor fi prezentate în secțiunile următoare.
1
Este axa de rotație paralel sau perpendicular pe fluxul de aer? Cel
dintâi este o axă orizontală mașină, acesta din urmă, de obicei, o axă verticală
mașină într-un cross-vânt de configurare.
TWIDELL PAGINARE.indb 273
01/12/2014 11:37
274
Tehnologia energiei eoliene
2
Este forța predominantă ridica sau trage? Trageți masini poate avea nici o parte
se deplasează mai repede decât vântul, dar ridica masini pot avea lama secțiuni
în mișcare în mod considerabil mai repede decât viteza vântului. Acest lucru este similar cu o
chila barca care poate naviga mai repede decât vântul.
3
Ce este rezistența? 'Duritate' este raportul dintre suprafața totală de lamele
la un moment dat, în direcția de fluxul de aer spre zona de măturat
peste airstream. Pentru mai multe turbine acest lucru este descris prin acordarea
număr de lame. Mare soliditate masini (multe lame) începe cu ușurință
cu inițială mare a cuplului, dar în curând ajunge la puterea maximă la mică
frecvența de rotație. Mici soliditate dispozitive pot necesita de pornire, dar
ajunge la puterea maximă la viteze de rotație de frecvență. Astfel o mare
soliditate mașini sunt folosite pentru pomparea apei chiar si in lumina vânturi. Mici
soliditate turbine sunt utilizate pentru generarea de energie electrică, deoarece rapid axul
de rotație frecvență este nevoie.
4
Care este scopul de turbina? Istoric cereale morile de vânt și
waterpumping turbinele eoliene produc energie mecanică. Marea majoritate
moderne de turbine eoliene pentru producerea de energie electrică; în general mare
(>2,5 MW) pentru reteaua de utilitati putere și intermediare sau mici pentru
autonomă, stand-alone de putere și de rețea legate de microgenerare.
5
Este frecvența de rotație constantă, sau nu variază cu viteza vantului?
O turbină de vânt a căror generatorul este conectat direct la o puternică AC
rețeaua electrică va roti doar la aproape frecvență constantă. Cu toate acestea,
o turbina de frecvență variabilă poate fi compensată în mod mai eficient la
diferite viteza vântului decât o frecvență constantă mașină, dar acest lucru
necesită generatoare speciale cu o conexiune indirectă printr-o
putere–interfață electronică (a se vedea de Revizuire 1).
O clasificare de vânt mașini și dispozitive pot fi acum dat în
asociere cu Fig. 8.3. Aceasta include principalele tipuri, dar numeroase
alte modele și adaptări apar.
§8.2.1 ax Orizontal turbinele eoliene (HAWTs)
Două - și trei pale HAWTs sunt de departe cele mai comune pentru
generarea de energie electrică (a se vedea Fig. 8.3(a)), cu rotor format din ambele
hub și lamele. Trei pale rotoare funcționeze mai lin și,
în general, mult mai liniștit decât două rotoare cu palete. Vizual, trei pale
de turbine roti fara probleme, dar două pale de turbine pot apărea la "clătina".
Single-rotoare cu palete, cu o contragreutate, au fost testate la întreaga
scară, dar asimetria produs prea multe dificultăți pentru comerciale
perspective. Angrenare și generatoare sunt, de obicei, la partea de sus a turnului într
- o nacelă. Multi-blade rotoare, având mare cuplu de pornire în lumina vânturi,
sunt folosite pentru pomparea apei și alte joasă frecvență putere mecanică.
Toate turbinele eoliene au lame similare în funcțiune pentru a aripi de avion
(și, de asemenea, dar mai puțin, avion, elice). Dominant forță motrice
TWIDELL PAGINARE.indb 274
01/12/2014 11:37
§8.2 tipuri de Turbine și termeni
275
se ridica, așa cum se arată în Fig. 8.2(c). Lame pe rotor poate fi față în față (direcția vântului)
sau în spate (direcția vântului) a turnului (vezi Fig. 8.3(a)). Vântul își schimbă direcția
în mod frecvent într-un plan orizontal, și rotorul trebuie să se întoarcă în orizontală
avion (girație) să urmeze vânt, fără oscilație. Direcția vântului turbinele nevoie
Cupa
anemometru
(b)
Savonius
rotor
Furled
Musgrove
De operare
Darrieus
'ou-rabla'
12345
Singlebladed
Direcția vântului cu
fan coada pasiv
de direcție
Direcția vântului cu
direcție activă
din partea rotoare
Direcția vântului cu
activ alimentat
director schimbat
de vânt cu palete
Direcția vântului
auto-orientate
sau puterea de condus
Două-Trei pale-pale
Multi-lamă
(a)
Augmenter
(c)
Difuzor
Concentrarea structurilor
Fig. 8.3
Clasificarea de vânt mașini și dispozitive: (a) axa orizontală; (b) axa verticală;
(c) concentratoare.
TWIDELL PAGINARE.indb 275
01/12/2014 11:37
276
Tehnologia energiei eoliene
o coada sau un alt mecanism de răsucire, cum ar fi motor electric drive, pentru a
menține orientarea. Direcția vântului turbinele sunt, în principiu, de auto - orientare,
dar sunt mult mai afectate de turn, care produce vânt și umbra
suplimentar turbulențe în lama calea. Perturbatii de acest fel cauza
ciclic subliniază cu privire la structura, zgomot suplimentar și fluctuații ale producției.
Direcția vântului și direcția vântului masini de diametrul rotorului de mai mult de aproximativ
10 m folosesc motoare electrice pentru a controla de girație.
§8.2.2 axa Verticală turbine eoliene (VAWTs)
Prin rotirea în jurul unei axe verticale, o mașină poate accepta vântul din orice
direcție, fără ajustare, întrucât o axă orizontală mașina
de girație (de exemplu, rândul său, în plan orizontal pentru a face față vântului). O așteptare pentru
axa verticală generatoare de turbine eoliene este de a avea cutii de viteze și
generatoare de la nivelul solului. Exemple, de la cele mai mici dispozitive, sunt schițate
în Fig. 8.3(b):
1
Cupa anemometru. Acest dispozitiv se rotește de forța de frecare. Forma de
cupe produce o relație aproape liniară între frecvența de rotație
și viteza vântului, astfel încât măsurarea numărului de rotații pe
perioada de timp corelat cu viteza medie a vântului în perioada respectivă. La
aparat este un standard anemometru pentru datele meteorologice (§7.4.1).
2
Rotorul Savonius (turbo masina). Există un complicat mișcare de
vânt prin și în jurul celor două curbe foaie de aripi. Conducere
forță este în principal drag. Construcția este simplă și necostisitoare.
Mare soliditate produce cuplu de pornire mare, deci Savonius rotoarele
pot fi utilizate pentru pomparea apei.
3
Rotor Darrieus. Acest lucru are două sau trei subțire palete curbate cu o paletei
secțiune. Rotorul forma este un catenar, cu scopul de rotație
lame fiind doar a subliniat de-a lungul lungimea lor.
4
Musgrove rotor. Lamele de această formă de rotor vertical normal pentru
generarea de energie, dar sfat sau rândul său, despre o orizontală de punctul de control sau
de oprire. Există mai multe variante, care sunt toate proiectate pentru a avea
avantajul de siguranță de oprire în condiții de vânt puternic.
Pentru Darrieus și Musgrove rotoare, conducere forțe din vânt sunt ridicați,
cu maximă a turbinei loc atunci când o lama se mișcă de două ori pe
rotație în direcția vântului, astfel încât impulsuri pe rotație. Utilizări sunt pentru energie electrică
generație. Rotorul nu este de obicei auto-pornire, deci poate fi inițiat cu
generatorul funcționează ca un motor.
Un avantaj major al axei verticale mașini este de a elimina
gravityinduced stress/strain de cicluri pe lame (care are loc la fiecare rotație în
lame de turbine cu axa orizontală); astfel, în principiu, axa verticală lame
poate fi foarte mare. Pentru mașini mici, angrenaje și generatoare pot fi
cuplate direct la verticală a arborelui principal la nivelul solului. Cu toate acestea, pentru
mașini mari, acest lucru ar necesita un timp arborelui principal transmiterea de foarte
TWIDELL PAGINARE.indb 276
01/12/2014 11:37
§8.3 impulsului teoria
277
cuplu mare, adică arborele va fi lung și gros, și, prin urmare, foarte
scumpe. Soluția este de a avea un generator de ridicat la punctul central de
rotație, și, prin urmare, similar cu o axă orizontală mașină. Principalele
dezavantaje ale VAWTs sunt: (1) multe axe verticale mașini au
suferit de oboseala eșecuri cauzate de mulți naturale rezonanțe în
structura; (2) cuplu de rotație de vânt variază periodic
în cadrul fiecărui ciclu, și, astfel, nedorite putere periodicities apar la
ieșire; (3) ancorat turn de sprijin este complex. Ca urmare, marea majoritate
de mașini de lucru sunt axă orizontală, nu verticală.
§8.2.3 Concentratoare, difuzoare și capace
Turbine trage puterea din interceptat de vânt, și, în principiu,
ar fi avantajos să pâlnie sau concentrat de vânt în turbină
din exterior rotorul secțiune. Diverse sisteme au fost dezvoltate sau
sugerat pentru turbine cu axa orizontală:
1
Lama sfaturi. Lama modele și adaptări au încercat să
tragă aer în rotor secțiune, și, prin urmare, valorifica puterea de la o
secțiune mai mare decât zona rotorului; cu toate acestea, orice avantaj a fost
pierdut din cauza complexității și costurilor. (A nu se confunda cu înclinat
bladetips care reduce vortex vărsare, deci, îmbunătățirea eficienței, ca și în unele
avioane.)
2
Structuri (de exemplu, Fig. 8.3(c)). Pâlnie concentratoare și alte forme de
deflectoare fix în jurul turbinei se concentreze vânt în rotor, dar
întreaga structură poate fi de girație. Astfel de complicații și costuri
înseamnă că astfel de concentratoare nu sunt utilizate pentru activități comerciale de mașini.
§8.3 IMPULSULUI TEORIA
În această secțiune vom obține ecuațiile de bază pentru putere, tracțiune și
torsiune de operare turbine eoliene. Analiza se bazează pe legile de
conservare a impulsului și a energiei. Mai riguros
tratament vor fi prezentate în secțiunile următoare. Energia eoliană dispozitivele sunt plasate în
gamă largă, extinsă, a fluxurilor de circulație a aerului. Aerul care trece printr-o
turbina eoliana nu poate fi, prin urmare, deviat în regiuni unde nu există
aer deja (spre deosebire de apa pe o turbina de apă: Fig. 6.4) și deci nu sunt
distinctive limite de vânt mașină de eficiență. În esență, aerul trebuie să
rămână cu suficientă energie pentru a muta departe în aval de turbina.
§8.3.1
Extragerea de energie; Lanchester-Betz-Zhukowsky
teoria
4
În Fig. 8.4-o coloană de vânt în amonte de turbină, cu secțiune transversală
zona O
1
de turbină disc, a energiei cinetice, care trece în unitatea de timp de:
TWIDELL PAGINARE.indb 277
01/12/2014 11:37
278
Tehnologia energiei eoliene
P
Ouu
Ou
0
1
2
10
0
2
1
2
10
3
r
r
(
)
=
=
(8.3)
Aici
r este densitatea aerului și u
0
neperturbat de viteza vântului. Aceasta este
putere în vânt cu viteza u
0
.
Densitatea aerului
r depinde slab pe inaltime si conditiile meteorologice.
Viteza vântului crește, în general, cu înălțime, este afectată de către autoritățile locale
topografie, și variază foarte mult cu timpul. Aceste efecte sunt luate în considerare pe deplin în
§7.3, și pentru prezent considerăm u
0
și
r constant cu timp și
peste zona coloanei de aer. Astfel fluxul de incompresibil este explicat
in Comentariu 2 mecanica fluidelor. O valoare tipică pentru
r este de 1,2 kg/m
3
la mare
nivel (Anexa Tabelul B. 1). Astfel, de exemplu, dacă u
0
~ 10 m/s, atunci (8.3)
arată că P
0
= 600 W/m
2
și în galeș, u
0
~25 m/s, deci P
0
~ 10.000 W/
m
2
; rețineți că cubi relația dintre putere și viteza vântului este puternic
non-linear.
Lanchester-Betz-Zhukowsky teoria calculează maxim
de putere care pot fi extrase din vânt, folosind un model simplu de o
viteză constantă, fluxul de aer care trece prin și în jurul turbinei în
asumat flux laminar (Fig. 8.5). Rotorul este tratat ca o 'acționare disc'
(care s-ar putea gândi ca un 'magic disc'!) de-a lungul căreia presiunea aerului
se schimbă ca energia este extrasă. În consecință, impulsului și
energia cinetică a vântului scădere; este această pierdere a impulsului
și a energiei cinetice, care este acum analizat. Într-o simplificare grosieră,
momentul cinetic nu este considerată, în ciuda turbină rotativă și
se trezește și vartejuri care apar în fluxul de aer. Modelul presupune, de asemenea,
nici o pierdere de energie prin frecare. Totuși, în ciuda acestor simplificări, modelul
este extrem de util.
O
0
u
0
u
1
u
2
O
1
O
2
În amonte
Turbina
(ca de acționare
disc)
În aval
Fig. 8.5
Lanchester-Betz-Zhukowsky modelul de expansiune airstream prin rotorului turbinei,
modelat ca un dispozitiv de acționare disc.
O
u
0
Masa de coloana
ρ
Au
0
, energie cinetică (
ρ
Au
0
)u
0
2
2
1
Fig. 8.4
Putere în vânt.
TWIDELL PAGINARE.indb 278
01/12/2014 11:37
§8.3 impulsului teoria
279
DERIVAREA 8.1 IMPULSULUI TEORIE: CALCUL
DE u
1
ȘI P
T
Pas 1: se Calculează u
1
de conservare a energiei
Rata debitului masic de aer în coloană este
m
.
. Acest aer în mișcare se aplică o forță
F a rotorului turbinei și, prin legea a treia a lui Newton, în sine experiențele pe un egal
și o forță opusă, astfel încât încetinirea din u
0
pentru a u
2
. De doua lege a lui Newton, F
este egal cu reducerea ritmului de aer pe unitatea de timp:
=

F
mu
mu
.
.
0
2
(8.4)
Puterea P
T
(energie pe unitatea de timp) extras de vânt și trece
în turbină este
=
=

P
Fu
mu
uu
.
(
)
T
1
0
21
(8.5)
În consecință, aer incetineste, deoarece energia cinetică este eliminat de vânt, la o
rata (putere) P
w
date de:
P
mu
u
.
(
)
w
1
2
0
2
2
2
=

(8.6)
De conservare a energiei, puterea extras din vânt este egală cu
puterea dobândită de turbină; deci echivalarea (8.5) și (8.6):
mu
uu
mu
u
mu
uu
u
.
(
)
.
(
)
.
(
)(
)
0
21
1
2
0
2
2
2
1
2
0
2
0
2

=

=
+

(8.7)
Prin urmare:
=
+
u
u
u
(
)
1
1
2
0
2
(8.8)
Rețineți că, în conformitate cu acest impulsului teorie, viteza aerului prin
sistemul de acționare disc nu poate fi mai mică decât jumătate din viteza vântului neperturbat.
Pasul 2: Știind u
1

, calcula puterea extras de vânt


Masa de aer care curge prin disc pe unitatea de timp este dat de:
r
=
m
Au
.
11
(8.9)
Astfel că, în (8.5),
r
=

P
Ouu
u
(
)
T
11
2
0
2
(8.10)
Substituind pentru
=

u
u
u
2
2
1
0
din (8.8) în (8.10) vă oferă:
r
r
=


=

P
Ouu
u
u
Ouu
u
[
(2
)] 2
(
)
T
11
2
0
1
0
11
2
0
1
(8.11)
Cele axiale un factor de inducție (de asemenea, în unele texte, numite 'axială interferențe
factor) este fracționată viteza vântului scădere de la turbina. Astfel,
=

o
u
u
u
(
)/
0
1
0
(8.12)
Considerăm aer se apropie și trece prin și de turbina
disc. Zona O
1
este rotorul zona de măturat, și zone cu O
0
și O
2
anexați
fluxul de aer care trece printr - O
1
. O
0
este poziționat în sens opus vântului
fata neafectat de turbină; O
2
este de la poziția minimă a vântului
viteza direcția vântului înainte de vânt față de reforme. O
0
și O
2
poate fi localizat
TWIDELL PAGINARE.indb 279
01/12/2014 11:37
280
Tehnologia energiei eoliene
experimental pentru viteza vântului determinare. Astfel măsurare la Un
am

este
posibilă datorită unor lame rotative.
Rețineți că modelul prezice:
1
când
o
= 1/3, apoi u
1
= 2 u
0
/3 și u
2
= u
0
/3
2
când
o
= 0.5, u
1
= u
0
/2 și u
2

= 0 (ceea ce ar implica zero curgă


de la turbina, dar, de fapt, indică o schimbare în modul de curgere, ca și
discutate în §8.5.2).
Rețineți, de asemenea, că doar aproximativ jumătate din puterea vântului este extras,
pentru că aerul trebuie să aibă o energie cinetică pentru a lasa turbina regiune.
Criteriul (8.18) pentru o putere maximă de extracție (C
p
max
= 16/27) este
de obicei, numit Betz criteriu, și pot fi aplicate la toate turbinele set
Rearanjarea (8.12) pentru a obține u
1
și folosind (8.8) se obține:
=−
=

u
ou
u
u
(1
)
(
)
1
0
1
2
0
2
(8.13)
așa că:
=

o
u
u
u
(
) / (2 )
0
2
0
(8.14)
Valorile reale pentru o, și alte astfel de parametri de model, sunt obținute prin
compararea predicțiilor făcute de modelarea teoretica cu
măsurători pe turbine în tunel de vânt și condiții de câmp (de exemplu, a se vedea
§8.5.4).
Din (8.11) și (8.14),
r
r
r
r
=

=

−−
=

=

P
Ouu
u
O
ouu
ou
o
o
Ou
Au o
o
2
(
)2
(1
)
[
(1
)]
[4 (1
)](
)
2
(1
)
T
11
2
0
1
1
2
0
2
0
0
2
1
2
10
3
10
3
2
(8.15)
Pas 3: se Calculează fracțiunea de energie eoliană extrase: a coeficientului de putere C
P

Este de obicei pentru a exprima turbina de putere P


T
ca:
P
T
= C
P
P
0
(8.16)
în cazul în care P
0
este puterea în neperturbat de vânt pe o suprafață egală cu
rotorul zona O
1
și C
P
este fracțiunea de energie extras, putere
coeficient. Comparând (8.16) și (8.15) arată că:
C
P
= 4o(1 − o)
2
(8.17)
[Analiză ar putea fi procedat în ceea ce privește raportul b
= u
2
/u
0
uneori
numit, de asemenea, un factor de interferență (a se vedea Problema 8.2).]
Valoarea maximă a C
P
apare în model, atunci când un
= 1/3 (a se vedea Problema
8.1 și Fig. 8.6):
C
P
max
= 16/27 = 0.59
(8.18)
0.1
0.2
0.3
0.1 0.2 0.3 0.4
o
0.5
0.4
C
P
0.5
0.6
Fig. 8.6
Coeficientului de putere C
p

ca o
funcție de factor de inducție - o.
Ca și în text, C
P

= 4o(1 − o)
2
;
o
= (u
0
− u
1
)/u
0

; (C
p
)
max

= 16/27
= 0.59.
TWIDELL PAGINARE.indb 280
01/12/2014 11:37
§8.3 impulsului teoria
281

P
F
O
z
(a)
(b)
O
1
p
0
u
0
u
1
u
2
p
d
p
u
Presiune
Fig. Comentarii: 8,7
Tracțiune pe turbine eoliene: (a) axiale F
O

, diferența de presiune
Dp; (b) înălțimea z, fluxul de aer
de viteză u, cu corespunzătoare presiunilor p afișate sub presiune.
într-un extins jet de fluid. Astfel, acesta se aplică la putere de extracție din
mareele și curenții râului (a se vedea Capitolul 12). Cu convenționale hidroenergetice
(Capitolul 6) apa ajunge la turbină într-un loc închis și nu este
extins în flux, astfel încât alte criterii se aplică.
În practică, o reclamă de turbine eoliene pot avea o
putere maximă coeficientul de aproximativ 0,4, cum s-a discutat în §8.4. Acest lucru
poate fi descris ca având o eficiență relativă la Betz criteriu de
0.4/0.59
= 68%.
A coeficientului de putere C
P

este în vigoare eficienta de a extrage energie


din masă de aer în care ar trebui flux tub care trece prin
acționare disc, zonă O
1
. Acest incident aerul trece prin zona O
0
în amonte
de turbina. Puterea extras pe unitatea de suprafață a Unui
0
în amonte este
mai mare decât per unitate de suprafață de O
1
, deoarece O
0
< O
1
. Acesta poate fi demonstrat (vezi
Problema 8.3) că puterea maximă de extracție pe unitate de Un
0

este 8/9 de
la putere în vânt, și deci turbina are o eficiență maximă de
89%, atunci când sunt considerate în acest mod. Efecte de acest fel sunt importante pentru
matrici de turbine eoliene într-un parc eolian serie de turbine.
§8.3.2 Axială (forța axială) pe turbine eoliene
O turbină eoliană trebuie să nu fie suflat de vânturi puternice. Pentru o
axă orizontală mașină, forța este centrat pe turbine cu axa și este numit
în axiale F
O
(a se vedea Fig. 8.7(a)). Acest axial produce o răsturnare
de cuplu care a rezistat de turnul fundație de o mare armat
bloc de beton încorporate în sol. Vezi Derivare 8.2.
TWIDELL PAGINARE.indb 281
01/12/2014 11:37
282
Tehnologia energiei eoliene
Termenul
Au
1
2
10
2
r
în (8.25) este în vigoare oferite de acest model pentru vânt
lovind un obiect solid din zona frontală O
1
. Fracțiunea de această forță expe-
enced de real turbina este forța axială (sau tracțiune) coeficientul C
F
, deci
care:
F
C Ua
O
F
1
2
10
2
r
=
(8.26)
Comparând (8.26) și (8.25) prezinta:
C
o
o
4 (1 )
F
=

(8.27)
în cazul în care axial factor de inducție - o este, din (8.12) și (8.14):
ou
uu
u
u
u
(
)/
(
)/2
0
1
0
0
2
0
=

=

(8.28)
DERIVAREA 8.2 AXIALE
Noi folosim lui Bernoulli ecuația (R2.2) pentru a calcula forța orizontală, adică
forța pe rotor modelat ca un dispozitiv de acționare disc în raționalizate flux, ca
în §8.3.1. Efectul turbinei este de a produce o presiune măsurabilă
diferenta
Dp între lângă direcția vântului (indice u) și lângă direcția vântului
(indice d) piese de flux (Fig. 8.7(b)). Deoarece nu este neglijabil de schimbare
în z și
r, vom aplica (R2.2) separat în amonte și în aval, dar în
același flux tub limita:
în amonte
r
r
+
=
+
p
u
p
u
u
u
0
1
2
0
2
1
2
2
(8.19)
în aval
r
r
+
=
+
p
u
p
u
d
d
0
1
2
2
2
1
2
2
(8.20)
Debitul masic este continuu prin disc subțire, astfel încât u
u
= u
d
. Prin urmare, sub-
propriile noastre eforturi (8.20) din (8.19) vă oferă:
r
D=

=

pp
p
u
u
(
)
u
d
1
2
0
2
2
2
(8.21)
Dp se numește presiune statică diferență, și termenii în
ru
2

sunt
dinamic presiuni. Conform (8.21), valoarea maximă a static
diferența de presiune apare ca u
2
se apropie de zero, ceea ce corespunde unei
suprafață solidă. Astfel:
r
D
=
p
u
(max)
1
2
0
2
(8.22)
și maximă forța axială (forța de tracțiune) pe o suprafață solidă este:
r
=
D
=
F
O
p
Ou
O
(max)
1
(max)
1
2
10
2
(8.23)
La axială este egală cu rata de pierdere a momentum de fluxul de aer:
=

F
mu
u
.
(
)
O
0
2
(8.24)
Folosind (8.9), (8.12) și (8.14),
r
r
r
=
=

=

F
Au
uo
O
ouuo
Au
o
o
(
)(2
)
(1
) (2
)
(
)4 (1
)
O
11
0
1
0
0
1
2
10
2
(8.25)
1
2
TWIDELL PAGINARE.indb 282
01/12/2014 11:37
§8.3 impulsului teoria
283
De model, valoarea maximă a C
F
= 1.0 când o = ½, echivalentul a
u
2
= 0 (de exemplu, vântul s-a oprit). Compara puterea maximă extrac-
tion, care, de Betz criteriu, apare atunci când un
= 1/3 (Fig. 8.6 și
(8.17)), corespunzătoare C
F
= 8/9 = 0.89.
În practică, valoarea maximă a C
F

pe un solid disc nu este de 1.0, dar aproximativ 1,2


datorită efectelor de margine. Cu toate acestea, impulsului teoria arată
că turbina apare în vânt ca o apropiere-solid disc atunci când
extragerea de energie. Este destul de înșelătoare pentru a estima forțe pe o rotație
a turbinelor eoliene de imaginat vântul trece nestingherit prin golurile
dintre lamele. Dacă turbina este extragerea de energie eficient, aceste
lacune nu sunt evidente la vânt și extrem de mare tracțiune forțele apar.
De turbine eoliene forțele de tracțiune crește ca
Ou
10
2

în (8.26), controlul
strategii sunt folosite pentru a proteja mașinile, la viteze ale vântului de mai
mult de aproximativ 15 până la 20 m/s; acestea includ: (1) pentru a activa (girație) rotorul afară
de vânt; (2) pentru a reduce puterea de extracție și, prin urmare, forța de tracțiune de pitching
lamele sau extinderea strica clape; (3) dacă blade pitch este fix, lamele
sunt concepute pentru a deveni ineficiente și auto-stagnare în mare viteză a vântului;
(4) pentru a opri rotirea cu lama de pitching și/sau frânare. Metoda (3) este
, probabil, cel mai sigur și mai ieftin; cu toate acestea, auto-trage de lamele au o
reducerea coeficientului de putere și nu dau putere optimă de extracție sau
buna putere de control. Prin urmare, metoda (2) este de preferat pentru mari
comerciale utilaje cu lama de tangaj (nu strica clape), deoarece puterea
de performanță pot fi optimizate și controlată în condiții de vânt puternic, și rotația
oprit, dacă este necesar. În zonele predispuse la uragane, turbinele pot fi
turnuri speciale, care poate fi înclinat (redus) la pământ și deci, din
vânt; un cost suplimentar de acestea, mai mult decât răsplătește ceea ce altfel ar fi
pierderea de toată energia eoliană sistem.
§8.3.3 Cuplu
Anterior calculul axiale pe o turbină eoliană oferă o
oportunitate de a introduce definiții pentru cuplu provocând axul să se rotească. La
această etapă este făcut nici o încercare de a analiza unghiulare impuls de schimb
între aer si turbina. Cu toate acestea, este evident că, dacă turbina
se transformă într-un fel de aer trebuie să întoarcă și celălalt; analiză completă trebuie să, eventual,
luați în considerare vartejuri de aer care circulă în aval de turbina (a se vedea §8.4).
Maxim posibil de cuplu,
G
max,

pe o rotorului turbinei s-ar produce dacă


maximă de împingere ar putea cumva fi aplicate în planul de turbină cu
lame la lama-vârful cel mai îndepărtat de axa. Pentru o elice turbina de
rază R, acest "de referință" de criterii ar fi:
G
= F R
max
max
(8.29)
Pentru o mașină de lucru care produc un real arbore de cuplu
G, cuplu
coeficientul C
G
este definit prin referire la criteriul de cuplu
G
max
:
G
G
=
G
C
max
(8.30)
TWIDELL PAGINARE.indb 283
01/12/2014 11:37
284
Tehnologia energiei eoliene
În practică, pentru o reclama de turbine eoliene în condiții normale de funcționare,
C
G
0.3.
Deci, prin această simplă analiză, pentru idealul turbina, C
G
este panta
la C
P
:
λ caracteristice. În special, cuplu de pornire ar fi
panta în origine. Cu toate acestea, este important să realizăm că, cu un real
rotor, este posibil, în practică, de a urmări în mod empiric curba de
C
P
vs
λ (a se vedea Fig. 8.8).
Rețineți că ambele C
P
și C
G
sunt puternice funcții de variabilă
λ și
, prin urmare, nu constant, cu excepția cazului în care rotorul are viteza variabila pentru a menține
constantă
λ. De Betz criteriu (8.18) valoarea maximă a C
P
este 0.59,
deci, în cele "ideale", caz
C
C
la
0.59 /
P. max
λ
()
=
G
(8.37)
Fig. 8.8 prezinta caracteristicile cuplului de practică turbine.
Largesolidity turbine funcționeze la valori mici ale sfat-raportul de viteza si au mare
cuplu de pornire. Dimpotrivă mici-soliditate mașini (de exemplu, cu îngustă
douăsau trei pale rotoare) au cuplu de pornire mic. La valori mari de
λ,
coeficientul de cuplu, și, prin urmare, cuplul, scade spre zero și
turbine 'roata libera'. Astfel, cu toate turbinele există o maximă de rotație
de frecvență în condiții de vânt puternic în ciuda faptului că există mari și poate deteriora
>
~
DERIVAREA 8.3 CUPLU COEFICIENTUL
Ignorarea direcția pentru moment, (8.23) sugerează că cea maximă
de tracțiune disponibilă pe turbină este:
r
=
F
Ua / 2
max
10
2
(8.31)
Deci, vom lua ca referință:
G
r
= Au R / 2
max
10
2
(8.32)
Cum va fi discutat în §8.4 și §8.5, vârful-raportul de viteza
λ este definit ca
raport de exterior lama sfat-viteza v
t
la neperturbat viteza vântului u
0
:
λ
W
=
=
vu
R
u
/
/
t
0
0
(8.33)
unde R este exterior lama rază și
W este frecvența de rotație.
Din (8.32), înlocuind pe R:
G
r
λ
W
λW
=
=
Ouu
P
(
)/2
/
max
10
2
0
0
(8.34)
în cazul în care P
0
este puterea vântului din (8.3). Expresii algebrice pentru
G
decurg din aceasta; a se vedea Problema 8.3(b).
Arborele de putere este puterea derivată din turbină P
T
, astfel:
GW
=
P
T
(8.35)
Acum, din (8.16) P
T
= C
P
P
0
. Egalarea celor două expresii pentru P
T
și
substituind pentru
G din (8.30) și (8.34), avem o relație importantă
între trei non-dimensional scară factori:
λ
=
G
C
C
P
(8.36)
TWIDELL PAGINARE.indb 284
01/12/2014 11:37
§8.3 impulsului teoria
285
0.1
1
2
3
4
5
6
7
8
0.2
0.3
Ideal
C
T
H-S
L-S
am
0.4
0.5
0.6
0.7
Fig. 8.8
Cuplul coeficientul C
G
vs sfat-raportul de viteza
λ, schițat de mare soliditate H-S, low-soliditate L-S,
și "ideal" de criteriu.
(a)
(b)
U
0
(c)
υ
Fig. 8.9
o
Un yacht cu vele, folosind drag vigoare a acesteia spinnaker să navigheze în direcția vântului.
b
Iahtul naviga în vânt utilizand forta de ascensiune.
c
Idealizat trage masina cu balamale clape pe o rotație a centurii.
axială. Rețineți că, cuplul maxim și puterea maximă de extracție
sunt nu de așteptat să apară la aceleași valori ale
λ. Relația vitală de
coeficientului de putere C
p
la sfat-raportul de viteza
λ este discutat în §8.4.
§8.3.4 Drag masini
Iahturi cu vele poate fi spus să aibă două tipuri de plecare. În Fig. 8.9(a)):
velier cu două catarge și brațe de macara, care în principal utiliza ridicare la putere, așa că, atunci când în
unghi în
TWIDELL PAGINARE.indb 285
01/12/2014 11:37

286
Tehnologia energiei eoliene
vântul, iaht poate deplasa mai repede decât viteza vântului. În Fig. 8.9(b),
simetric spinnakers în principal de a utiliza forța de frecare pentru care navighează în direcția vântului,
dar la nu mai mult de viteza vântului. Aceasta este o comparație utilă pentru
realizarea beneficia de ridicare la putere, în comparație cu drag vigoare pentru wind
turbine.
O idealizat drag mașină constă dintr-un dispozitiv cu wind-driven
suprafețe sau clape se deplasează paralel cu netulburată vânt de viteză u
0
(Fig. 8.9(c); compara cupa anemometru în Fig. 7.12. (a).
Pentru un lambou de secțiune transversală O se deplasează cu o viteză v, viteza relativă
este
u
v
(
)
0

și deci (8.23) implică faptul că valoarea maximă forța de frecare pe
suprafata este:
r
=

F
Ou
v
(
)
max
1
2
0
2
(8.38)
Adimensional coeficientul aerodinamic C
D
, definite în §R2.7, este folosit pentru a
descrie dispozitivele cu plecare din ideal, astfel încât forța de frecare devine:
r
=

F
COu
v
(
)
D
1
2D
0
2
(8.39)
Este simplu de a arăta că, pentru acest sistem idealizat, maximul
coeficientului de putere este dat de:
C
C
(4 / 27)
P
max
D
=
(8.40)
(A Se Vedea Problema 8.13.) Valorile de C
D

variază de la aproape zero, pentru o discuție


obiect, la un maxim de aproximativ 1,5 pentru o formă concavă fi utilizate în
standard anemometre. Astfel teoretică maximă a coeficientului de putere pentru
o trageți aparatul este:
()

=
=
C
4
27
1.5
6
27
22%
P
max
(8.41)
Acest lucru poate fi comparat cu Betz criteriu pentru un "ideal" de mașină (drag
sau lift), de C
p
= 16/27 = 59% (8.18).
În practică, trageți mașini au C
P
<5% (Kragten 2009). În §8.3.1
am menționat că cel mai bun lift-forța turbinele de putere coeficienții
de ~40% și mai mult. Prin urmare, trageți-numai dispozitive au putere
numai cele mai bune la aproximativ 10% din lift-forța de turbine cu aceeasi zona
de secțiune transversală. Singura modalitate de a îmbunătăți drag mașini este să
integreze forțele de ridicare, cum se întâmplă în unele forme de rotor Savonius.
În caz contrar, 'drag masini sunt oarecum inutile, cel puțin pentru putere
generație.
§8.4 CINETIC TEORIA
§8.4.1 Concepte
Pierderea liniară de impuls din amonte flux de vant este
în general mecanismul prin care energia vântului este transformată în
TWIDELL PAGINARE.indb 286
01/12/2014 11:37
§8.4 cinetic teoria
287
DERIVAREA 8.4 TANGENȚIALĂ FACTOR de INDUCȚIE o'
De acționare disc concept, prezentat în §8.3 și Fig. 8.10, este acum dezvoltat în continuare. Conceptul este de
o regiune a spațiului în care energia este extrasă din aer în mișcare; a 'disc' nu ar trebui să fie vizualizat ca
, integral sau parțial solide. Modelul nu încearcă să descrie calea de moleculele de aer, dar propune
procese de schimb de energie și impuls. Scopul este de a modela cum rotorul obține unghiulare
(de rotație) impuls, și cum egală și opusă cinetic este dat de trecerea aerului
flux, astfel formând în aval de vârtejuri în trezesc.
Fig. 8.10 descrie viteze și conceptualizat 'streamtube' de vântul care trece prin acționare
disc. Acționare disc este modelat ca un set de inele înguste (annuli) având raza r și mici radială lățimea
dr. Aerul intră disc cu zero, viteza unghiulară și frunze la viteza unghiulară w și, prin urmare, liniar cu
viteza r
w în direcție tangențială (adică perpendicular pe planul din Fig. 8.10(a) și în planul de Fig
8.10(b). Aici
w pot fi diferite de la lama unghiulară de rotație rata W. zona inelului cu care se confruntă
vântul este
d
pd
=
O
r
r
2
.
2
.
De asemenea, la disc este rotorului turbinei, care, prin conservarea momentului cinetic, se rotește în opus
direcție la urma cu magnitudine egală a introdus cinetic. Din punct de vedere conceptual, atât în aer
și rotorul ocupă aceeași regiune a spațiului de acționare disc.
(a)
O
1
p
0
u
0
u
1
u
2
p
d
p
u
Presiune
δr
r
(b)
Fig. 8.10
Acționare disc, arată unele dintre parametrii-cheie:
o
privit din lateral,
b
privit 'end-on'(nu la aceeași scară)
energie mecanică, și apoi, de obicei, în energie electrică. Liniare
impuls teoria §8.3 este foarte de succes în stabilirea dispozitivului de acționare
disc concept și parametrii de bază de turbine eoliene, inclusiv puterea
coeficientul C
P
, (axial) factor de inducție o, (axial) force/forța de tracțiune coeficientul C
F
,
și Betz 59% criteriu pentru putere maximă de extracție.
Cu toate acestea, mai multe analize fundamentale de turbine eoliene, folosind
sistemul de acționare disc analogice trebuie să aibă în vedere cinetic. Astfel
modelarea a fost dezvoltat pentru aeronave, elice și a ajuns la
o complexitate considerabilă și variații. Aici vom introduce conceptele de wind
turbine, dar o întreagă teorie este complicată și de specialitate, așa cum este descris
în mai avansat literatura de specialitate (de exemplu, Burton et al. 2011; Manwell et al. 2010).
§8.4.2 Cuplu, putere și sfat-viteza raportul de vedere
momentul cinetic
TWIDELL PAGINARE.indb 287
01/12/2014 11:37
288
Tehnologia energiei eoliene
Din (8.13) modelată viteza axială a aerului ajunge la discul este u
1
[
= (1–o)u
0
], astfel încât masa de aer care trece în unitatea de timp prin fiecare annulus din zona
d O
este:
d
rd
rd
=
=

m
Au
O
ou
.
.
.
.(1
)
.
1
0
(8.42)
Modelul conceptual este că aerul lăsând de acționare a discului de la r de la axa a vitezei unghiulare
w și
viteza tangențială r
w în planul de disc. De asemenea, la disc este rotorului turbinei cu viteza unghiulară W și
viteza tangențială r
W.
Cea tangențială factor de inducție a' este definit ca:
w
'=
W
o
/ (2 )
(8.43)
Deci
r
m
r
m
o
rata de schimbare a momentului cinetic de aer
= (momentul de inerție)
(schimbare în viteza unghiulară)
=(
.
)
(
.
)(2
)
2
2
dw
d
×
=
W'
(8.44)
Această rată de variație a momentului cinetic oferă un opus creștere de cuplu
dG a rotorului,
de unde, folosind (8.42) și (8.44),
(8.45)
Legate de creștere de putere este:
d
d
rd
=WG=

W'
P
r
O
ou
o
[
(1
) ](2
)
2
0
2
(8.46)
Cu toate acestea, din (8.15) prin impulsului teorie, avem pentru acest element de suprafață
dO:
d
rd
=

P
Ouo
o
2.
.
(1
)
0
3
2
(8.47)
Echivalarea (8.46) și (8.47), deoarece cele două modele să nu se contrazică reciproc:
r
O
ou
o
Au o
o
[
(1
) ](2
)2.
.
(1
)
2
0
2
0
3
2
rd
rd

W'=

(8.48)
și deci:
λ

'
=W



=
o
o
o
r
u
(1
)
o
r
2
2
(8.49)
în cazul în care
λ
r
este numit locale sfat-raportul de viteza la raza r.
La lama-sfat, r
= R, unde R este raza rotorului (în vigoare lama lungime), astfel:
λ

'
=
W






=
o
o
o
R
u
(1
)
o
2
2
(8.50)
în cazul în care
λ este sfat-raportul de viteza. Această importantă non-dimensional factor este, de asemenea, evidentă în (8.36), Fig. 8.8 și
§8.5.1.
dG = rata de schimbare a momentului cinetic de aer
= (r
2
dm) (2Wun')
= [r
2
(
rd)(1−o)u
0
](2
Wo')
TWIDELL PAGINARE.indb 288
01/12/2014 11:37
§8.5 Dinamic de potrivire
289
Tabelul 8.2
Comparație de aripă de avion cu lama de o turbină eoliană
Aripa de avion
Lama de o turbină eoliană
Miscari aer de pe aripă de avion pentru
motoarele propulsa avionul înainte.
Aerul se mișcă pe un operator de turbine eoliene lama pentru
(i) lama este de cotitură, și (ii) de aer abordări în vânt.
Avionul rămâne pentru că forțele de ridicare pe
aripi depăși gravitația.
Rotorului turbinei se transformă pentru componentele
(așa-numitele) forțele de ridicare transforma lame.
O simplă diagramă a forțelor pe orizontală
aripa explica ridicați și trageți (de exemplu, Fig. R2.6 sau
Fig. 8.11(b)).
Echivalentul forțelor pe o lamă rotativă trebuie să fie
rezolvate de două ori pentru a distinge transformarea forțelor de
tracțiune forțe (de exemplu, înțelegerea rezoluție de forțe
pe o turbina lama este mai dificilă decât pe o aeronavă
cu aripă).
Rezumând această introducere pentru momentul cinetic teorie, ne-am
modelat aer vartejuri in urma prin introducerea tangențială
un factor de inducție', legate de acest flux axial factor de inducție-o, și a
arătat relația cu sfat-raportul de viteza
λ. Toți cei trei parametri sunt
adimensionale, astfel încât să permită măsurarea lor în tunele de vânt cu
scară mică modele fizice.
§8.5 DINAMIC DE POTRIVIRE
§8.5.1 Optime viteza de rotație; sfat-raportul de viteza
λ
La Betz criteriu oferă un standard acceptat de 59% pentru
maxim extractibil putere. Sale fizice de bază este că aerul trebuie să-și păstreze
suficientă energie cinetică pentru a muta departe în aval de turbina, dar
derivarea §8.3.1 nu ne spune nimic despre dinamica de rotație a stat de
o turbina necesar pentru a ajunge la acest criteriu de eficiență maximă. Această
secțiune explorează această dinamică cerință.
Primul pas este să dau seama că aerodinamica a unei turbine eoliene
lama sunt în esență aceleași ca aripă de avion prezentată în Fig.
8.11. (b). Având în vedere un avion în zbor orizontal, comparația este prezentată
în Tabelul 8.2. Experimente practice cu model de aripi și lame sunt
singurul mod de a experimenta ceea ce se întâmplă; se recomandă să
încercați (de exemplu, Caseta 8.1).
§R2.7 explică cum aerul care trece pe o pernă de aer, ca o aripă de avion,
creează lift și forțele de frecare. Pentru fiecare paletei există un optim
unghiul de atac (
o
opta

) pentru ridicare maximă vigoare; acest unghi este de obicei de aproximativ 5°


(vezi Fig. R2.7 și Fig. 8.11(b)). Aceeași condiție este necesară pentru un vânt
de palete de turbine, în cazul în care unghiul de atac
o este prezentată în Fig. 8.12.
Vectorul diagrama prezentată în Fig. 8.12 este pentru o turbină cu lama de
lungime R rotește la o viteză unghiulară
W. relativă A aerului este la fluxul
unghi
f și lama-setare unghi este g. Au g ~ 5°
permite ridicare la putere
la vârf "înainte", care oferă o forță componentă în planul de rotație,
TWIDELL PAGINARE.indb 289
01/12/2014 11:37
290
Tehnologia energiei eoliene
Forța de frecare
În amonte de vânt
viteza u
o
Ridicare la putere
Direcția de
rotație a rotorului.
Sfat viteza R
Ω
În căutarea " jos " pe o lamă secțiune
Lungimea lamei R = raza rotorului
rata de Rotație
Ω (radiani/s)
din vârful unui rotație a turbinelor eoliene lama.
v
r
α
γ
F
forța de tracțiune
F
roti
f
f
Fig. 8.12
Definiția schiță de unghiuri și forțele, în căutarea de-a lungul unei turbine eoliene lama de la vârful său ca acesta
se rotește într-un plan perpendicular pe departe din sens opus (în amonte) vânt.
CASETA 8.1 CONFRUNTĂ CU LIFT ȘI FORȚELE DE FRECARE
Într-o locație sigură pe un drum privat, ca un pasager pe bancheta din spate de o mașină care călătoresc la aproximativ 50 km/h,
ține-ți brațul pe fereastră, cu mâna întinsă, ca și în fotografie (Fig. 8.11(a)). Pe măsură ce rotiți
mâna ta va simți forța de frecare și, cu mâna în unghi de aproximativ 5° față de orizontală veți simți
brațul tău în creștere cu liftul vigoare. De ridicare și de forțele de frecare pe o suprafață netedă aripa (a se vedea diagrama,
Fig.8.11(b)) sunt
similare, dar mult mai eficient!
(a)
Lift, L
Drag, D
Centrul de Lift
Coardă linie de la vârful la vârful
Direcția de zbor
u
0
Situație similară pentru o
aripa de avion
(b)
Fluxul de aer
o
Pentru o aripa de avion (o paletei),
ridicați și trageți forțe L și D sunt funcții de unghiul de atac o.
Nu trebuie să fie mare
D trebuie să fie mici
Fig. 8.11
o
Parte din fluxul de aer de o mașină în mișcare.
b
Ridicați și trageți forțe pe o suprafață netedă aripă de avion.
TWIDELL PAGINARE.indb 290
01/12/2014 11:37
§8.5 Dinamic de potrivire
291
deci rotirea rotorului. Relativă a aerului este la un unghi de atac
o lama,
într-o manieră similară cu aer întâlnire de aripa de avion (Fig. 8.11(b)).
Viteza de vârf este R
W într-o direcție în unghi drept față de prezent
din sens opus vântului de viteza u
0
; prin urmare, afluxul unghi
f este dat de:
R
u
cotan
cotan(
)
/
0
f
og
λ
=
+=W
=
(8.51)
În (8.51) parametru adimensional
λ = RW/u
0

este sfat-raportul de viteza.
Acest parametru important a apărut în momentul cinetic teoria de
acționare disc (§8.4), și poate fi înțeles în raport cu
unghiul de atac.
Maximă de ridicare la putere, ca și cu o aripă de avion, apare atunci cand unghiul
de atac este constantă la
o
opta
(de obicei, aproximativ 5° de ridicare cu mici trageți
departe de stand: a se vedea Fig. R2.7(a)); aceasta este condiția pentru funcționarea optimă a discului de
eficiență. Putem concluziona deci că pentru o turbină eoliană pentru a funcționa
eficient, rotorul trebuie să se rotească la o viteză unghiulară astfel că unghiul
de atac
rămâne constantă la cea optimă. Acest lucru implică faptul ca
în amonte viteza vântului u
0
schimbă, la fel ar trebui să viteza de rotație
W
schimba, astfel încât
o, și, prin urmare, f, rămân constante. Lama-stabilirea
unghi (de exemplu, teren) la sfat,
g, este de obicei menținută constantă (dacă nu s-a schimbat
pentru a opri sau altfel controla masina), astfel încât condiția de optim
puterea de captare este că non-dimensional sfat-raportul de viteza
λ fi controlate
constant și optimizat ca în amonte viteza vântului se schimbă. Aceasta este
starea de dinamică optimă de potrivire a turbinei de vânt.
În (8.51), dacă, de exemplu,
un ≈ 5° și b ≈ 3°, apoi
λ = cotan f = cotan 8° = 7.1; deci, păstrând unghiul de atac constant la
vârful necesită
λ rămâne constant la această valoare. Astfel, dacă în amonte
viteza vântului u
0
crește, atunci rata de rotație
W trebuie să crească pentru a
obține maximum de energie de captare, și vice-versa. (8.51) explică de ce, în
același punct, cu rază mare rotoare transforma mai lent decât în mică
rază de rotoare de geometrie similară; fiecare are aceeași valoare de R
W/u
0
, adică
același sfat-raportul de viteza.
Noastre de bază în analiza de mai sus a considerat doar o lama individuale. În
practică prin deplasarea rotorului, paletele se mute în poziție ocupată
anterior de către alte lame. În termeni simpli: (i) dacă rotorul se rotește încet, mai mult
aer se pot deplasa prin planul de rotor, fără nici o interacțiune cu
o lamă și nu atât de transfer de energie, și (ii) dacă rotorul se transformă foarte rapid,
rotorul pare a vântului mai mult ca un disc solid, și din nou energie
nu sunt transferate în mod eficient. Acest lucru este prezentat în Fig. 8.13.
Un ordin de mărime de calcul pe baza Fig. 8.13 (Problema 8.5)
corect sugerează că, pentru o turbină cu n lame de raza R: (i)
viteza de rotație a
W
m
pentru putere maximă de extracție este invers proporțională
cu n; și (ii) semnificative de perturbare de sens opus fluxului de aer începe
numai la o destul de distanță scurtă (<
~R ) în amonte de rotor. Problema 8.5
sugerează, de asemenea, că puterea maximă a coeficientului de o trei pale de turbină
are loc la
λ ≈ 4.
TWIDELL PAGINARE.indb 291
01/12/2014 11:37
292
Tehnologia energiei eoliene
În practică, însă, cu atenție definite elice, vânt comercial
turbinele au tendința de a fi optimă sfat-raportul de viteza
λ pentru putere eficientă
generație în intervalul de la 6 la 8 (vezi Fig. 8.14), care este un util indicativ
schiță pentru a arăta tendințele în coeficientului de putere, C
p,

și sfat-raportul de viteza,
λ, pentru diferite tipuri de turbine eoliene. Cu toate acestea, consulta
datele producătorului pentru mai multe valori specifice. Fig. 8.14 prezinta
LanchesterBetz-Zhukowsky criteriu pentru puterea maximă a coeficientului de aproape 60%
(§8.3.1), și, de asemenea, indică faptul că C
P
(max)
crește ca
λ crește, așa cum este dat
de cele mai sofisticate teoria Glauert, 1930 aerodynamicist,
după cum este explicat mai avansate de texte (de exemplu, Burton et al. 2011).
Sfat-raportul de viteza
λ este, probabil, cel mai important parametru de un vânt
turbina, deoarece se referă la unghiurile de atac de relativă viteza vântului
Rotație
(a)
(b)
(c)
Deranjat
airstream
Netulburată
airstream
Fig. 8.13
Turbina de viteza (frecvența) și de puterea de captare.
o
Frecvența de rotație prea lent: vântul trece nestingherit prin acționare disc.
b
Frecvența de rotație optimă; tot airstream afectate.
c
Frecvența de rotație prea repede: energia este disipată în aer turbulent de mișcare și vortex vărsare.
TWIDELL PAGINARE.indb 292
01/12/2014 11:37
§8.5 Dinamic de potrivire
293
pe lama paletei. Este o funcție de trei variabile importante:
lama măturat de raza, viteza vântului și a rotorului de frecvență. Fiind
adimensional, devine un element esențial factor de scalare în proiectare și analiză.
§8.5.2 Extensii ale impulsului teoria
Fig. 8.15 prezinta un grafic al coeficientului de putere C
P
împotriva factor de inducție
o în intervalul 0 <o <0.5, ca date de simplu impulsului teorie.
Astfel, din (8.17),
=

C
o
o
4 (1
)
P
2
(8.52)
în cazul în care, din (8.28),
o
uu
1
/
1
0
=−
(8.53)
Extensiile de la simplu model teoretic extinde analiza în alte
regiuni ale inducție/interferențe factor, și link-ul de turbină
performanță cu elice de aeronave caracteristici. În Fig. 8.15, jeturilor de aer
sunt schițate pe grafic pentru regiuni specifice care pot fi asociate cu
debitul real de aer condiții, așa cum este indicat în mici diagrame de flux (a)–(d):
o
0 <o <0.5, C
P
pozitive și ajungând. La un
= 0, u
1
= u
0
și C
P

= 0;
turbina se rotește liber în vânt și nu este cuplat la o sarcină
de a efectua munca. Ca o sarcina este aplicată, puterea este captată, așa C
P
crește ca u
1
scade. Puterea maximă este eliminat din
Betz criteriu 59%
Glauert criteriu
Axa verticală
Tip Darrieus
Grid-connected,
axa orizontală
C
p
λ
Multiblade pompa de vânt
Rotorul Savonius
1
2
3
4
5
6
7
8
9
0.6
0.5
0.4
0.3
0.2
0.1
0
Fig. 8.14
Indicativ schiță a coeficientului de putere C
p

ca o funcție de tip-raportul de viteza


λ pentru o serie de
turbine eoliene tipuri.
TWIDELL PAGINARE.indb 293
01/12/2014 11:37
294
Tehnologia energiei eoliene
fluxul de aer atunci când o
= 1/3 și u
1
= 2u
o
/3 (Fig. 8.6). La un
= ½, De bază
impulsului teoria modelelor solid disc, prin estimarea maximă
de tracțiune pe turbina (8.26) cu forța axială coeficientul C
F
= 1.
b
0.5 <o
< 1, C
P
scăderea la zero. Din (8.28), un
= (u
0
− u
2
)/2u
0
. Când
o
= 0.5, modelul are u
2

= 0; adică modelat de vânt iese perpendicular


pe fluxul de intrare. În practică este posibil să se ia în considerare această regiune ca
fiind echivalent cu debutul de direcția vântului turbulent de aer de mișcare. Acesta este echivalent
cu o turbina de operare în extreme, viteza vântului atunci când puterea
eficiență de extracție scade, din cauza unei nepotriviri de rotație, frecvența
și viteza vântului. La un
= 1, C
P

= 0, turbina este de cotitură și provoacă


ample de turbulențe în fluxul de aer, dar nici o putere nu este extras. Real
turbine poate ajunge la această stare excesivă stand condiție.
c
o
< 0, C
P
negativ. Aceasta descrie o elice de avion acțiune în cazul în care
puterea se adaugă la fluxul de a obține propulsia. În acest fel
Putere
generație
de vânt
(a)
(c)
(b)
(d)
Maxim de
putere
Lama trage de stat
Aeronave
elice
membre
0.6
0.3
0.5
-0.5
1.0
1.5
o
C
p
-0.6
Fig. 8.15
Coeficientului de putere C
P

comparativ cu factor de inducție un


= 1 – u
1

/u
0

, ca dată de către liniară


modelul impuls. Rezultatele sunt legate de experiența practică de aer de mișcare și
turbină/elice membre de mici diagrame de flux:
o
energetic normal abstracție de o turbină eoliană, așa cum se arată în Fig 8.5;
b
turbulent trezire reduce eficiența, așa cum se întâmplă cu extreme viteze ale vântului;
c
fluxului normal de aer a unei aeronave cu elice: energie se adaugă la fluxul de aer;
d
echivalent cu elice de aeronave inversorul de presiune de frânare la aterizare.
TWIDELL PAGINARE.indb 294
01/12/2014 11:37
§8.6 Lama element de teoria
295
elicele se trage în intrare airstream și propulsa
avionul înainte, într-o rulotă.
d
o >1. Acest lucru implică negativ u
1

și este îndeplinită dacă o elice de avion inversează


tracțiune prin schimbarea lamei pas la aterizare (de exemplu, C-130 Hercules).
Intens vortex vărsare apare în fluxul de aer ca aerul trece
elice. În avion, energie suplimentară se adaugă la
fluxul de aer și este evident în vârtejuri, dar efectul total este un revers
forța de tracțiune pentru a crește de frânare.
§8.6 LAMA ELEMENT DE TEORIA
Secțiunile anterioare au stabilit o înțelegere de bază de energie eoliană
mașini și dinamica lor; în special, ne-am definit
adimensional de scalare a parametrilor coeficientului de putere C
P,
cuplul coeficientul C
G
,
și sfat-raportul de viteza
λ. Cu toate acestea, nu am analizat modul în care, pentru fiecare
secțiune de o lamă, relativă, viteza vântului și forțele de ridicare și trageți
varia de-a lungul lamei. O astfel de analiză se numește lama teoria elementelor (de asemenea,
numit flux-tub teoria'), de care vom da doar o schiță în această
secțiune. Vezi mai avansate manuale (de exemplu, Hansen 2007) pentru mai multe
detalii.
§8.6.1 Calcul de ridicare și de forțele de frecare pe o lamă element
Vom începe prin luarea în considerare lama elemente (secțiuni) și cilindrii de
fluxul de aer (stream-tuburi) se deplasează pe rotor, așa cum se arată în Fig. 8.16.
Fiecare lama element din Fig. 8.16(a) este asociat cu un standard paletei
secțiune transversală. De ridicare și de forțele de frecare pe cele mai comune paletei forme
au fost măsurate și tabulare, în special de către NASA,
5
ca o funcție
de relativă viteza aerului v
r
și unghiul de atac
o. Din aceste date, pentru fiecare
element, forța rotirea rotorului poate fi calculată prin integrarea de-a lungul
fiecare lamă.
Cele mai eficiente turbine eoliene au lame răsucite, cu "twist"
cel mai pronunțat în apropierea butucului (Fig. 8.16(c) și Fig. 8.16(d)). În
funcțiune, aceasta permite unghiul de atac
o de-a lungul lamei pentru a fi mai aproape de
optimă, deoarece v
r
crește și
f scade cu distanța de la hub
(a se vedea Fig. 8.16(c)). La start-up r
W este zero și, în practică, u
0

≥ 4 m/s, deci
cu cea mai mare poftă de mâncare lângă hub ofera un unghi de atac care rezultă
în suficientă accelerarea cuplu pe rotor pentru a începe rotația. Odată
rotative, contribuția la cuplul de părțile interioare ale lamelor este
minim, deci mare poftă de mâncare, lângă butuc este atunci lipsit de importanță.
Rotorul cu pale se rotește cu frecvența (viteza unghiulară)
W; acolo-
prim-plan elementar secțiune se mișcă la viteză liniară v
= r
W în planul de
rotorul. Viteza liniară de vânt orizontal departe în amonte este u
0

. De
acționare disc teorie (Fig. 8.5), ca fluxul de aer se extinde pe orizontală
viteza vântului în modelul devine u
1
= u
0
(1–o) la rotorul de avion; a se vedea
TWIDELL PAGINARE.indb 295
01/12/2014 11:37
296
Tehnologia energiei eoliene
poftă de mâncare
(a)
(b)
(c)
(d)
(e)
r
δ
r
Ridicare la putere
α
φ
φ
φ
γ
γ
γ
Direcția de
rotație a rotorului.
Sfat viteza R
Ω
Forța de frecare
F
forța de tracțiune
F
roti
În amonte de vânt
viteza u
o
v
r
În căutarea " jos " pe o lamă secțiunea de
sfat, de o rotație a turbinelor eoliene lama.
Lungimea lamei R = raza rotorului
rata de Rotație
Ω
(radiani/s)
Cele mai multe lamele au o poftă de mâncare pentru a îmbunătăți: (i)
o
de-a lungul lamei, (ii) start-up
Blade Root
Lama Sfat (zero poftă de mâncare)
Blade twist de sfat
r
Ω
R
Ω
u
0
(1 – a)
u
0
(1 – a)
θ
α
α
Linie Acord
Linie acord la sfat
Linie acord la sfat
u
1
= (1 – o) u
0
u = r
Ω
v
r
v
r
v
r

Fig. 8.16
o
Element de secțiune a unui lama, latime
d ar la distanța r de la hub.
b
Ridicați și trageți forțe pe lamă element, cum se vede
privind în jos de la vârful lamei.
c
Similar cu (b), dar arată lama poftă de mâncare.
d
Vector triunghiul de viteze la lama
element.
e
Comerciale turbine de 1,5 MW turbine GE la NREL), arată blade twist: lame mai aproape de fața vânt de intrare
la hub decât la vârful lamei.
Rețineți că unghiurile
f și o varia cu r (și de multe ori nu atât de g ). Diagrama (c) indică, de asemenea, coardă linie, respectiv linia de
posterior de extremitatea lamei secțiunea de la marginea de conducere.
TWIDELL PAGINARE.indb 296
01/12/2014 11:37

§8.6 Lama element de teoria


297
(8.13). Combinarea acestor viteze ca vectori (Fig. 8.16(d)), a rezultat
viteza vântului v
r
este:
v
r
= √ {u
0
2
(1
− o )
2
+ (r
W)
2
} (8.54)
Fig. 8.16(b) arată sporul de ridicare la putere, și de forța de frecare, pe
elemental secțiune. Scopul este de a integra din punct de vedere matematic aceste
trepte de-a lungul lungimea lamei pentru a obține generale de ridicare și de forțele de frecare pe
lamă rotativă. Comentariu 2 (§R2.7) explică faptul că ascensorul și forțele de frecare
pe un obiect într-un flux de lichid (în acest caz de aer) se referă la a secțiunii transversale
zonă de obiect O; viteza relativă de obiect în fluid v
r
și
adimensional coeficienții (C
Am
pentru ridicare, C
D

pentru drag). Aceste constante sunt


evaluate la experimente pe obiecte în tunele de vânt și centralizate în
tabele standard. În cazul nostru a lamei elementar secțiune:
F
vCcr
(
)
Am
r
Am
1
2
2
r
d
D=
(8.55)
F
vCcr
(
)
D
r
D
1
2
2
r
d
D
=
(8.56)
Aici empirice coeficienții C
Am
și C
D
sunt definite pentru zona c
d ar (c, a
marginea anterioară a trailing edge, lungime cablu,
dr elementar sectionale
grosime de-a lungul lungimea lamei).
Fig. 8.16(b) indică cât de ridicare vigoare pot fi rezolvate în compo-
nent
DF
L, R
în planul de rotor, și componenta
DF
EU,UN
de-a lungul axei de
rotație. De asemenea,
DF
D
pot fi rezolvate în componenta
DF
D,R
în avion
de rotație, și
DF
D,O
de-a lungul axei
.
. (Aceste componente nu sunt marcate
pe diagrama pentru a preveni de la a deveni în mod nejustificat aglomerat.)
Rețineți că, teren de lama (
g ) și unghiul de atac (a ) sunt astfel
asta
DF
Am
este în unghi "înainte". În plus, forța de frecare este mic din cauza
buna lame. Prin urmare,
DF
L,R
>
DF
D,R
și astfel forța rezultată
DF
R
încearcă să accelereze lama. Prin adăugarea a două seturi de componente:
În planul de rotor
f
f
D
=D
−D
=D
−D
F
F
F
F
F
păcat
pentru că
R
LR
DR
Am
D
,
,

(8.57)
și de-a lungul rotor axa
f
f
D
=D
+D
=D
+D
F
F
F
F
F
pentru că
păcat
O
AM O
DO
Am
D
,
,

(8.58)
Substituirea de (8.55) și (8.56) pentru
DF
Am
și
DF
D
în (8.57) și (8.58)
randamente:
F
vcC
C
r
( păcat
cos )
R
r
Am
D
1
2
2
r
f
fd
D
=

(8.59)
și:
F
vcC
C
r
( cos
sin )
O
r
Am
D
1
2
2
r
f
fd
D
=
+
(8.60)
TWIDELL PAGINARE.indb 297
01/12/2014 11:37
298
Tehnologia energiei eoliene
§8.6.2 Calculul forțelor și de cotitură cuplu pe un întreg
lama
Integrarea (8.59) și (8.60) cu privire la r de la butuc (r
= r
hub
)
pentru a
vârful lamei (r
= R) dă totalul de cotitură forța F
R
și total axiale
F
O
. Așa, de exemplu:
F
vcC
C
r
( cos
sin )
O
r rhub
rR
r
Am
D
1
2
2


r
f
fd
=
+
=
=
(8.61)
Contribuția fiecărui lama de cotitură cuplu
G a rotorului este
obținut prin integrarea produs de
DF
R
și r de la hub la
lama sfat:
vcC
C
r dr
( păcat
cos ) .
rhub
R
r
Am
D
1
2
2


r
f
f
G=

(8.62)
Integrarea formală a (8.62) de calcul este puțin probabil, deoarece toate v
r
, c și
f
variază cu r (adică sunt funcții de r ) (și deci, nu C
Am
și C
D

dacă paletei forma


variază cu r, deoarece este destul de comună). În practică, prin urmare, integrarea
se efectuează la fel ca un computer însumare pentru N elemente (probabil 50 sau
mai multe secțiuni individuale de-a lungul lamei), de exemplu:
vcC
C
r
r
( păcat
cos ) .
n
nn
rnn
Am
n
D
nn
1
2
1
,
2


r
f
f
G=

D
=
=
(8.63)
în cazul în care
Dr = (R–r
hub
)/N cazul în etape egale sunt luate (deși
D ar poate fi, de asemenea,
variat, cu r).
Evaluarea factor de inducție "a"
Un parametru esențial al impulsului teorie este factor de inducție - o.
De exemplu, (8.25) evaluează forța axială pe o turbină eoliană cu rotor fi:
F
Au
o
o
(
)4 (1 )
O
1
2
10
2
r
=

(8.64)
Lama element de teoria calculează, de asemenea, F
O

prin (8.61); acest evaluate rezultat


folosind parametrii cunoscuți de special lame poate fi egală cu
(8.64) pentru a obține valorile pentru factor de inducție - o.
§8.6.3 Implicații
Pentru avioane, relativă viteza vântului incident pe o aripa rămâne
constantă de-a lungul marginea de conducere, fiind aplicate viteză de avion
relativă la vânt naturale.
Cu toate acestea, pentru axa orizontală turbine eoliene, relativă viteza vântului v
r

crește spre vârful lame rotative (Fig. 8.17). În consecință,


contribuția majoră la cotitură cuplu vine din părțile exterioare ale
lamei, așa că este aici, că performanțelor aerodinamice este cel mai important
și unghiul de atac
un relativă a aerului trebuie să fie aproape de optim,
valoare, cum s-a discutat în §8.5. Viteza triunghi prezentată în Fig. 8.16(d)
arată că r crește,
f va tinde să scadă.
TWIDELL PAGINARE.indb 298
01/12/2014 11:37
§8.7 Putere de extracție de o turbina
299
0
10
20
30
40
50
0
5
10
15
20
25
r/m
sp
fed
/(
m/
s)
u
1
r
Ω
v
r
Fig. 8.17
Modelat airspeeds la rotor la distanța r de la axa (mult verde liniuțe) u
1

= u
0
(1–a)
=
constantă; (scurta verde liniuțe ) r
W; (solid negru)
=√

+W
v
u
o
r
[
(1
)
()]
r
0
2
2
2
.
Cazul prezentat are u
0
= 10 m/s, o = 0.30, W = 1.78 rad/sec = 17 rot/min, lama raza
R
= 25 m.
Prin urmare, pentru a menține
aproape constant, unghiul de teren
g (= f − o) trebuie
să scadă față de vârful lamei. Acest lucru se face prin răsucire lama, așa cum
este descris în Fig. 8.16(c) și Fig. 8.16(e).
Impuls teoria și lama element de teoria oferi modele pentru
turbine eoliene de analiză. Toate aceste modele fac presupuneri; succesul lor
depinde de comparații cu rezultatele empirice de teste practice și
operarea cu real turbine. Real comparațiile sunt în general bună
(să zicem, în
± 10%), care permite mai mult rafinament în mai avansate
modele și, de asemenea, adăugarea de îmbunătățiri, cum ar fi descompunerea
flux laminar în turbulențe în vârful lamei (de exemplu, la marginile
ipoteza de acționare disc) și formarea trezește în fluxul de aer
în aval de turbine. Manual de Burton et al. (2011) este excelent
pentru mai multe astfel de studii avansate.
§8.7 PUTERE DE EXTRACȚIE DE O TURBINA
Producătorii sunt de așteptat să furnizeze un măsurată de operare curba de putere
pentru fiecare tip de turbine eoliene furnizate, în forma prezentată în Fig. 8.18.
Acest lucru are două scopuri principale: (1) pentru pre-construcție analiza financiară
cu ajutorul vântului previziuni pentru a determina puterea generata (a se vedea Problema
8.10 și a Lucrat Exemplu 17.3); și (2), ca o referință pentru măsurarea
ulterioară a eficienței operaționale. Majoritatea producătorilor de utilitate scară
mașini oferă o gamă largă de curbele de putere legate de diferite de putere
de control scenarii și acustică emisii de zgomot.
Fracțiunea de energie extras din vânt de o turbina este
coeficientului de putere C
P
, ca definit prin (8.16). C
P
este cea mai dependentă de
sfat-raportul de viteza
λ, care se referă la unghiul de atac o de lame
(Fig. 8.14 de §8.5.1). Strategia de potrivire o mașină de la un anumit
regimul eolian variază între obiectivele (1) maximizarea total de energie
TWIDELL PAGINARE.indb 299
01/12/2014 11:37
300
Tehnologia energiei eoliene
de producție în cursul anului (de exemplu, pentru vânzarea către un utilitar de energie electrică în rețea);
și (2) asigurarea unui minim de alimentare, chiar și în vânt moderat (de exemplu, pentru apă
de pompare la o vite jgheab sau încărcarea unei baterii pentru iluminat). În
plus, echipament secundar, cum ar fi generatoare sau pompe, trebuie să fie
cuplat la turbina, deci puterea sa-potrivire de răspuns trebuie să fie legate
la turbina caracteristice. Obiectul de extracție de putere este, prin urmare,
complexe, care încorporează mai mulți factori, și în practică o serie de
strategii și tipuri de sistem vor fi utilizate în funcție de diferite tradiții
și nevoile.
Această secțiune consideră putere extras de turbina, care va avea
o putere nominală capacitate P
R

de obicei, egală cu capacitatea de generator


care poate fi menținută continuu fara supraincalzire. Fracțiunea de
energie în vânt capturat de turbina este C
P
a coeficientului de putere (u),,
definit de (8.16), care este o funcție de viteza vântului u:
r
=
Cu
P
Ou
()
/(
)
P
T
1
2
1
3

(8.65)
Rețineți că, pentru simplificarea notației în această secțiune, vom folosi uneori
simbolul u pentru neperturbat (în amonte) viteza vantului, notată cu u
0
în §8.3. La fel ca în §7.2.3, să
Φ
u

denotă normalizat probabilitate pe vânt


viteza intervalul că neperturbat viteza vântului va fi în intervalul
u a (u + du ), adică
Φ
u

du este probabilitatea de viteza vântului între u și


(u + du ). Atunci puterea medie extras prin turbina de rotor zona
O
1
de aer de densitate
r pe intervalul de viteza vântului u este:

r
Φ
=

P
O
uCuu
=
( )d
1
2
T
u
u
1
3
P
0
(8.66)
Să E fi energia totală extrase în perioada T, și să E
u

fi
corespunzătoare de energie extras pe unitatea de viteză a vântului între vânt
cu viteze de u și (u + du). De factor de capacitate Z este definit, în general, pentru toate
Turbina
de ieșire
de putere P
T
P
R
Nominală
putere
Se taie în
u
ci
u
R
Viteza vântului u
u
R
Viteza vantului nominala
Taie
Fig. 8.18
Turbine eoliene de putere, curba de operare regiuni și putere de performanță. Valori tipice
sunt cut-in viteza vântului u
ci

≈ 5m/s, viteza vantului nominala u


R
≈ 13 m/s, cut-out viteza vântului
u
co
≈ 30 m/s.
Caracteristicile Standard; necesită exact blade pitch control.
- - - - Operare real caracteristicile de multe masini, inclusiv stand regulament.
TWIDELL PAGINARE.indb 300
01/12/2014 11:37
§8.7 Putere de extracție de o turbina
301
generarea de energie a plantelor în §1.5.4(b), ca energie generată efectiv în
perioada de timp T, de obicei un an, ca procent din energia pe care ar fi
produs dacă turbina generate în mod continuu, la putere nominală:
Deci, în acest caz:


r
=
=
=
=
Φ
=

Z
E
PT
E du
PT
P
P
O
uCuu
P
( )d
1
2
R
u
R
T
R
u
P
u
R
1
3
0
(8.67)
în cazul în care
=
P
ET
/
T

este puterea medie produsă în perioada T. Astfel,


factorul de capacitate depinde puternic de regimul eolian. Pentru un site
cu puternică, vânt constant (de exemplu, pe coasta de vest a Noii Zeelande) Z
poate fi la fel de mare ca 40%. Pentru site-uri cu mai slab, dar încă viabile de vânt (de exemplu,
părți ale Germaniei), Z este de obicei în intervalul 15-25%. (A se vedea Tabelul D. 4 din
Anexa D.)
Acesta este, de obicei, consideră că există patru distincte, viteza vântului regiuni
de funcționare (a se vedea. Fig. 8.18):
1
u
0
mai puțin de cut-in speed u
ci
E
u
= 0 pentru u
0
< u
ci
(8.68)
Nu există nici o putere pentru că generatorul este fie staționar
sau rotativ prea încet pentru semnificativă putere de ieșire; în practică,
mai ales pentru mașini mari, rotorul este frânat automat pentru a preveni
'rock' mișcările care provoacă uzura în arbori și cutii de viteze.
Accidente din cauza unei deblocat rotor început să rândul său, sunt prevenite dacă
automată sau manuală de blocare apare.
2
u
0
mai mare decât viteza nominală u
R
dar mai puțin decât
cut-out viteza u
co
În acest interval turbina produce putere constantă P
R
,
deci:
E
u
= (Φ
u > u
R

−Φ
u > u
co

) P
R
T (8.69)
în cazul în care
Φ
u > u'
este probabilitate a vitezei vântului depășește u' (ca în
Fig. 7.8 din Exemplul 7.1. §7.3.4), și P
R
este puterea nominală
de ieșire și T este timpul evaluării.
3
u
0
mai mult de cut-out viteza u
co
Prin definiție de cut-out viteza,
E
u
= 0 pentru u
0
> u
co
(8.70)
Cu toate acestea, în practică, multe masini nu pe deplin taie în vânt mare
viteză pentru că de stand regulament, dar continuă să funcționeze la foarte mult
eficiența redusă în mod rezonabil de mare putere.
4
u între u
ci
și u
R
Turbina de putere de ieșire P
T

crește cu u într-un mod care depinde


de condițiile de operare și tip de mașină. Pentru mai multe masini,
P
T
în acest interval poate fi montat de către o ecuație de forma:
P
T
≈ ou
3
0
− bP
R
(8.71)
TWIDELL PAGINARE.indb 301
01/12/2014 11:37
302
Tehnologia energiei eoliene
în cazul în care
o și b sunt constante ce pot fi determinate de la putere
curba determinată în termeni de u
ci
, u
R
și P
R
.
În practică, turbine va fi de multe ori operează în regiune între cut-in
și de ieșire nominală, și este o risipă de energie potențială dacă aparatul este
limitat în mod nejustificat, la mari viteze ale vântului. Există două extreme teoretice
condiții de funcționare (vezi Fig. 8.19):
o
Variabila cu rotor de viteză constantă sfat-raportul de viteza
λ, de unde constanta C
P.

Fig. 8.19(b) descrie acest lucru, cel mai eficient mod de funcționare, și


care captează mai multă energie. Vezi Problema 8.12 (și răspuns)
pentru detalii de calcul a energiei de captare. Viteză variabilă turbine
de obicei cut-în, la viteze ale vântului mai mică decât pentru o viteză constantă de turbine,
care, de asemenea, crește captarea energiei. Mari, moderne conectate la rețea
(a)
0.4
0.3
0.2
0.1
3
Sfat raport viteza
λ = R Ω/u
0
Constanta sfat raport viteza
Cutin
Nominală
Viteza vântului u
0
Nominală
Viteza vântului u
0
Pedeapsa pentru
nu funcționează
la constanta
sfat raport viteza
Frecvență constantă
6
9
0.4
0.3
0.2
0.1
0.4
0.3
0.2
0.1
C
p
(C
p
)
λ
(C
p
)
Ω
(c)
(b)
λ=6
λ=3
Fig. 8.19
Coeficientului de putere C
p

: (o) versus sfat-raportul de viteza; (b) în funcție de viteza vântului la constanta


sfat-raportul dintre viteza și așa variabilă viteza rotorului; (c) în funcție de viteza vântului la constanta turbina
frecvență, în comparație cu viteză variabilă la sfat-raportul de viteza de la 6 la 7.
TWIDELL PAGINARE.indb 302
01/12/2014 11:37
§8.8 generarea de energie Electrică
303
turbine eoliene, în special pentru parcuri eoliene, sunt în mod normal automat
controlat individual la optimizate cu viteză variabilă.
b
Constante (fixe) turbină cu frecvența de rotație, prin urmare, diferite grade C
P

Fig. 8.19(c) descrie acest lucru. Deși mai puțin eficient decât viteza variabila
turbine, utilizarea standard low-cost generatoarele de inducție permite
ușor de conectare la rețea (mici frecvență alunecare de generatoarele de inducție
nu este semnificativă, astfel încât mașinile sunt descrise ca 'constant' sau "fixe"
viteza). Cele mai multe turbine eoliene construite înainte de anul 2005, funcționează la fix
viteze cu conectat direct bază generatoarele de inducție. De
operare la frecvență constantă nu este o pierdere de energie posibil de extracție.
Acest lucru poate fi deosebit de grave pentru anual de generare de energie dacă există
o nepotrivire de performanță optimă la mai mari viteze ale vântului.
§8.8 GENERAREA DE ENERGIE ELECTRICĂ
§8.8.1 Bază
Energia electrică este un excelent vector de energie pentru a transmite capturat
puterea mecanică de o turbină eoliană. Generația este de obicei ~95% eficient, și
de transmisie pierderile ar trebui să fie mai mică de 10%. Avantajele generale
de electricitate ca un vector de energie sunt discutate în Capitolul 15, împreună
cu o discuție extinsă a rețelelor de energie electrică și de integrare de
variabile surse regenerabile în astfel de grile (§15.4). De bază
de inginerie detalii de energie electrică și transport (distributie) sunt
prezentate în Revizuire 1.
Există multe eoliene comerciale/sisteme de energie electrică, inclusiv o
gamă largă de generatoare de specialitate, sisteme de control și analizoare de date.
Cercetarea și dezvoltarea continuă puternic pentru îmbunătățiri suplimentare
ca vantul generat de putere este consolidat ca o forma importanta de energie electrică
de alimentare. Grid-connected turbine și ferme eoliene de expediere puterea de a fi
integrate cu alte forme de generare (de exemplu, centralele termice,
energie solară și hidro). Consumatorii folosesc energie electrică la
aproape tensiune constantă și frecvență, ca controlată de grila
operatorilor pentru sistemul de transmisie a puterii. Cu toate acestea, cantitatea de energie
din vânt variază în mod semnificativ în timp și oarecum la întâmplare
, în ciuda tot mai exacte prognoze pentru vant. Cu toate acestea, dacă puterea
de vânt într-o rețea este nu mai mult de 20% din puterea totală la
un moment dat, apoi variațiile sunt de obicei acceptabile în cadrul
mereu in schimbare condiții de sarcinile de consum, așa cum am discutat în Capitolul 15.
Pentru aplicații stand-alone, frecvența și tensiunea de
transmisie nu trebuie să fie atât de standardizate, deoarece utilizarea finală cerințe pot varia.
De încălzire, în special, pot accepta largi variații în frecvența și tensiunea.
În toate aplicațiile în care va fi necesar pentru a se potrivi cu atenție mașina
caracteristicile locale, regimul eolian. Evident, perioade lungi de
zero sau light vântul va limita aplicații eoliene. În special, site-uri
TWIDELL PAGINARE.indb 303
01/12/2014 11:37
304
Tehnologia energiei eoliene
cu o viteza medie a vântului mai mică de 4 m/s la 10 m înălțime, de obicei,
au inacceptabil perioade lungi de timp la care generație nu ar avea loc,
deși de pompare a apei în apa de stocare poate fi încă posibil. De obicei,
cazul în care viteza medie anuală a vântului la 10 m înălțime este de 5 m/s sau mai mult,
generarea de energie electrică din centrale eoliene este benefic.
De caracteristici distinctive ale vântului/sisteme de generare a energiei electrice sunt:
1
Turbine variază în dimensiune de la foarte mari (de exemplu ~130 m diametru, ~5 MW)
pentru utilitate generație, la foarte mic (de exemplu, ~1 m diametru, ~50 W) timp de
încărcare a bateriei.
2
Există întotdeauna perioadele fără vânt. Cele mai multe turbine de rețea
conectat, astfel încât furnizarea la consumatorii continuă din generație în generație. Pentru
relativ puține, dar importante, sisteme stand-alone, turbine eoliene
trebuie să fie legată de stocare a energiei și/sau paralel generație (de exemplu,
generatoare diesel). Capitolul 15-a mai multe detalii.
3
Eficienta turbinei eoliene este mai mare dacă frecvența de rotație variază pentru a
menține constantă sfat-raportul de viteza, dar alimentarea cu energie electrică este aproape la
frecvență constantă. Prin urmare, interfața electronică este necesar, cu excepția cazului
în care turbina eoliană funcționează mai puțin eficient la viteză fixă.
4
Control mecanic al unei turbine cu lama de teren sau alte
control mecanic crește eficiența, dar, de asemenea, crește complexitatea și
cheltuieli. O metodă alternativă, de obicei, mai ieftin și mai eficient
, dar rar făcut, este de a varia sarcina electrică de pe turbina pentru a controla
frecvența de rotație.
5
Optimă de rotație frecvență de o turbina (sa "viteza"), în
special viteza vântului scade cu creșterea razei, în scopul de a
menține constantă sfat-raportul de viteza (§8.5). Astfel, doar mici (~2 m raza)
turbine pot fi cuplate direct la convențională patru sau șase pole-pereche
de generatoare. Mașini de capacitate mai mare nevoie de echipamente suplimentare, care pot
sau nu pot include: (i) o cutie de viteze pentru a crește de acționare a generatorului
de frecvență; (ii) speciale multipole sau de două ori alimentate de generatoare; sau (iii) interfață
electronică ca redresoare și invertoare (se referă la Revizuirea 1). Cutii de viteze
sunt relativ scumpe și greu; ei au nevoie de întreținere regulată
și poate fi zgomotos. Generatoare speciale și interfețele electronice sunt, de asemenea,
scump, dar devin mai ieftine și mai fiabile în producția de masă.
6
Pe termen foarte scurt, dar utile, 'rotorul inerție' de stocare a energiei care apare
netezește vânt turbulențe. Chiar și furnizarea de un soft de cuplare', folosind
teetered lame, amortizoare sau alte mecanisme mecanice este
util pentru a reduce socuri electrice și mecanice tulpina.
7
De multe ori, cele mai bune energie eoliană site-uri sunt în zonelor rurale, insulare sau marine
zone. Locale, necesarul de energie la astfel de locuri sunt distincte, și
aproape sigur nu necesită mult mai mare de energie electrică din
zonele urbane și industriale complexe. Unele dintre aceste locații sunt grila
conectat la nivel regional sau național de rețele și unele nu sunt. Cele
tehnice legate de aceasta sunt prezentate în §8.8.6.
TWIDELL PAGINARE.indb 304
01/12/2014 11:37
§8.8 generarea de energie Electrică
305
§
8.8.2 Clasificare a sistemelor electrice folosind energie eoliană
Există trei clase de turbine eoliene de energie electrică, în funcție de
capacitatea relativă de generator cu turbină eoliană, P
T
și alte elec-
tricity generatoare sau bateriile conectate în paralel cu aceasta, capacitate de P
G

(Tabelul 8.3). Copleșitoare fabricație este de mare, grid-connected


turbine, dar mai mici turbine sunt pentru utilizări speciale, inclusiv
microgenerare (Fig. 8.20) și off-grid de alimentare cu energie electrică (Fig. 8.21).
(a) Clasa a:
capacitate de turbine eoliene dominant, P
T
> ~5P
G
De obicei, acest lucru este un autonom autonom (adică nu este conectată la rețea)
turbina. Folosește sunt: (i) comunicarea la distanță, de iluminat, pentru marină lumini, etc.,
cu un 'foarte mic' turbină de capacitate P
T
De maximum 2 kW; (ii) de uz casnic și de muncă-
magazin de consumabile, inclusiv de căldură, P
T
~10 kW. Baterie de stocare (Capitolul 15) este
aproape sigur să fie încorporate.
Opțiuni de Control au fost discutate în §1.5.3 și sunt de o extremă
importanță pentru sisteme eficiente (Fig. 8.21). O opțiune este de a avea foarte
puțin rotor de control astfel încât rezultatul este de tensiune variabilă (și, dacă AC, variabilă
frecvență) pentru direct rezistive de încălzire și de încărcare a bateriei (Fig. 8.2(a)).
DC sarcini pot fi alimentate direct de la baterie, și puterea necesară
la 240 V/50 Hz sau 110 V/60 Hz poate fi obținută folosind DC/AC invertoare.
Prin urmare, de înaltă calitate, energia electrică este obținută prin 'piggy-backing pe o dominantă
de aprovizionare de calitate mai proastă (de exemplu, încălzire) și a costat numai împotriva
marginale extras din baterie si invertor.
De uz casnic,
ferma etc.
~
Export
import
metru
Rețea de alimentare
Fig. 8.20
Microgenerare: grid-legat de turbine eoliene muncit într-un sistem mare.
Tabelul 8.3
O clasificare de turbine eoliene de energie electrică sisteme
Clasa
O
B
C
P
T
: capacitate de turbine eoliene
P
T
>> P
G
P
T
~ P
G
P
T
<< P
G
P
G
: legate de capacitatea de producție
Exemplu de P
G
Iluminat baterie
Generator Diesel
Central power station
Exemplu de sistem
Autonome
Vânt/diesel
Grila încorporat
Comun generator cu turbină eoliană
tip
Inducție
Inducție
(a) de două ori de inductie
(b) multi-pol cu AC/
DC/AC interfață
TWIDELL PAGINARE.indb 305
01/12/2014 11:37
306
Tehnologia energiei eoliene
Cu toate acestea, poate fi de preferat să aibă energie electrică direct de la
controlată de frecvență. Există două opțiuni extreme pentru acest lucru:
1
Control mecanic de palele turbinei. Ca vântul se schimbă viteza,
teren de lamele sau sfaturi lama este ajustat pentru a controla frecvența
de rotație a turbinei (Fig.8.21(b)). Dezavantajele sunt că puterea în
vânt este 'varsat' și, prin urmare, a pierdut (a se vedea §1.5) și metoda de control
poate fi costisitor și nesigur.
2
Sarcina de control. Ca vântul se schimbă viteza, sarcina electrică este schimbat
de către rapidă de comutare, astfel încât turbina frecvență este controlat (Fig. 8.21(c)).
Această metodă face o mai mare utilizare a puterii în vânt prin
optimizarea sfat-raportul de viteza
λ. Mai mult decât atât, controlul local de electronice moderne
metode este mai ieftin și mai fiabil decât controlul mecanice
componente expuse în medii nefavorabile.
Magnet Permanent multipolar generatoare sunt comune pentru întreprinderile mici
mașini. Sistemele de curent continuu pot fi atenuate și energia stocată în
baterii. Sistemele de curent ALTERNATIV pot fi sincrone generatoare de producerea
necontrolată variabilă frquency de ieșire de căldură, sau de ieșire controlat de
mecanică sau de control a sarcinii. AC generatoarele de inducție poate fi auto-excitat
cu un condensator banca la pământ, sau poate funcționa cu un ralanti
generator sincron ca un compensator. (A se vedea de Revizuire 1 pentru mai multe detalii despre
generator types.)
(a)
Tensiune variabilă
Încălzire rezistivă
De încărcare a bateriei
AC controlate
~
~
DC/AC
invertor
Controller
Feedback-ul de la
blade pitch
(b)
(c)
O sarcină prioritară
Controller
Feedforward
pentru o gama
de sarcini
Controlate
fixe de tensiune
și frecvență
și/sau frecvență
Fig. 8.21
Unele opțiuni de alimentare pentru sisteme stand-alone cu turbină de vânt dominant
de aprovizionare.
TWIDELL PAGINARE.indb 306
01/12/2014 11:37
§8.8 generarea de energie Electrică
307
(a)
(b)
Diesel
M2
M1
M
Metri
Preț unic de energie electrică
Pret redus de energie electrică
, de exemplu pentru căldură
Prețul ridicat de energie electrică
, de exemplu lumini
Diesel
~
Energia
magazin
Fig. 8.22
Vânt/diesel aprovizionare moduri: (a) modul single; (b) modul multiplu.
(b) Clasa B:
capacitate de turbine eoliene
≈ alte generator de capacitate, P
T
≈ P
G

Aceasta este o caracteristică comună din zonele îndepărtate, și anume mici sisteme de rețea.
Prima presupune că 'alte generator de capacitate P
G

este alimentat
de un motor diesel, probabil alimentat cu biodiesel. Scopul principal
al turbinei eoliene este probabil să fie de economisire a combustibilului. Generator diesel va
fi singura alimentare în perioadele fără vânt și va poate spori
turbină eoliană în perioadele de vânt slab. Există două moduri extreme de
funcționare:
1
Single-modul de alimentare cu energie electrică de distribuție. Cu un singur set de
cabluri de distribuție (de obicei în trei faze de alimentare, care are o singură fază
pentru locuințe pe piața internă), sistemul trebuie să funcționeze într-un singur modul la
fix tensiune de 240 V sau 110 V, legate de utilizare (Fig. 8.22(a)). O recepție de 24 de ore
de alimentare menținută fără sarcină control de gestiune va depinde în continuare
foarte mult (cel puțin 50%, de obicei) pe diesel de generație, deoarece vântul este de multe ori
nu este disponibil. Diesel este fie păstrat rulează continuu (frecvent
pe sarcină ușoară, chiar și atunci când energia eoliană este disponibilă) sau oprit
atunci când energia eoliană este suficientă. În practică, o cantitate mare
(uneori peste 70%) de vânt generate de puterea de a fi aruncat în
afara rezistor bănci din cauza de nepotrivire a cererii și ofertei
in conditii de vant.
2
Mai multe-modul de distributie. Scopul este de a folosi toate vântului generat de
putere, oferind energie electrică ieftină pentru multe utilizări în condiții de vânt
(Fig. 8.22(b)). Ca viteza vântului scade, mai ieftin deservite
de sarcini sunt dezactivate automat la scăderea cererii, și
invers. Același sistem poate fi folosit pentru a controla rotație a
turbinei eoliene. Atunci când nu energia eoliană este disponibil, numai încărcările de pe
cele scumpe de aprovizionare sunt activate pentru alimentarea de la generator diesel.
TWIDELL PAGINARE.indb 307
01/12/2014 11:37
308
Tehnologia energiei eoliene
Pragmatic avantaj economic de succes în mai multe-modul de
funcționare este faptul că întregul capital valoare de vânt mașină este utilizat în
toate timpurile, și de la putere inițiale în vânt este gratuit, maxim
beneficiu se obține. Este, de asemenea, avantajoasă în folosind mai puțin combustibil cu
reducere de poluare și zgomot.
(c) Clasa C: grila legate, turbine eoliene integrate într-un sistem mare,
P
T
≤ 0.2 P
G

Acesta este cel mai comun acord pentru a mari (>~1 MW), mediu (~250
kW) și mici (~50 kW) masini unde o utilitate publică sau alte
largecapacity rețea este disponibilă. În acest caz P
G

este, de obicei, de la relativ foarte mare


(>~500 MW) centrale, stații electrice care controlează frecvența întreaga
rețea.
.

Cea mai mare parte dintre noi capacitatea de energie eoliană este pentru ferme eoliene, în care
un număr (de la 10 la 1000) de turbine într-un grup de alimentare în rețea (§8.8.3). Pentru
sisteme mai mici, proprietarul (microgenerator) pot folosi puterea vântului
direct și vinde (export) exces de la rețea, cu energie electrică achiziționate
(importate) de la rețea în perioadele de slabă sau fără vânt (Fig. 8.20).
Comentariu 1 consideră generarea de energie electrică în mai multe detalii. Cel
mai ieftin tip de generator este un generator de inducție conectat direct la
rețea. Turbina trebuie să funcționeze la aproape frecvență constantă, în termen de
maximum alunecare, de obicei, mai puțin de 5% față de rețea legate de
frecvență; aceasta este de obicei numit 'fix viteza'. În vânt slab, nu este un
automatic cut-out pentru a preveni automobilism. Dezavantajul de a o direct
cuplat generatorul este ca turbina frecvența nu poate schimba
suficient pentru a menține chiar și aproximativ constantă sfat-raportul de viteza.
Cu toate acestea, există mai multe moduri în care sistemul poate fi făcut
pentru a produce energie electrică la destul de frecvență constantă, în timp ce permițându -
variație în turbina de frecvență. Acestea includ: (1) mai multe (de obicei două)
combinație înfășurări într-un generator de inducție pentru a conecta mai multe pole-perechi
în vânt slab pentru mai mici de rotație frecvență; (2) un intermediar
scară mașini folosesc două generatoare din aceeași nacela, să zicem, 5 kW si
de 22 kW, pentru conectarea automată la o două viteze în lumină și
vânturi puternice; (3) utilizarea unui generator sincron și rectificarea acestuia de ieșire
pentru curent continuu și apoi producerea prescris de curent alternativ
frecvența rețelei de alimentare cu invertor; (4) creșterea eficientă alunecare pe un
generator de inducție de schimbare activă de curent și de fază la
generator rotorului (de exemplu, într-un dublu alimentat generator de inducție); acest lucru necesită
alimentare externă conexiune la înfășurarea rotorului prin inele colectoare și perii.
§
8.8.3 Eoliene: interioare și offshore
(a) de Ce fermele eoliene?
Comerciale turbinele eoliene sunt o stabilit "mainstream" formă de
generarea de energie electrică în rețeaua de distribuție și rețelelor de transport, cu
cel mai de capacitate în mai multe turbine eoliene, în principal pe uscat (așa cum se arată
TWIDELL PAGINARE.indb 308
01/12/2014 11:37
§8.8 generarea de energie Electrică
309
Fig. 8.23
O parte din Buffalo Ridge eolian din Minnesota, statele UNITE ale americii, cu activitate agricolă
continuă sub și în jurul turbinelor. Turbinele sunt prezentate unele dintre cele
143 Zond Z-750s instalat în 1998, fiecare dintre înălțime de 78 m și evaluat la 750 kW. Mai multe
turbine au fost instalate pe Buffalo Ridge ulterior.
în Fig. 8.23), dar cu o proporție tot mai larg (așa cum se arată în
Fig. 8.24). Creșterea la nivel mondial capacitatea de generare a fost, și
continuă să fie, aproximativ 20%/an, ceea ce este remarcabil pentru inginerie
structuri (a se vedea Fig. 8.1).
Mașini multi-megawatt capacitatea de a opera cu succes, cu
durate de 20 la 25 de ani, și mai mult cu renovarea. Mai multe numere
de mașini instalată în ferme eoliene (de obicei cu 10 la 100 de turbine pe
teren, și de la 50 la 300 de offshore) face convenabil și ușor de gestionat de unități
distribuite generație în regional și național a rețelelor de energie electrică.
Gruparea mașini în acest mod permite economii în aplicatii de planificare,
costurile de construcție (de exemplu, având specializate, macarale, etc. pe site-ul), rețeaua
de conexiune (mai puține posturi de transformare și de rețea interfață transformatoare), comună
de management și de întreținere. Fermele eoliene sunt cel mai probabil în țările
cu (1) guvernamentale angajamentul să durabilă, cu emisii scăzute de carbon, energie,
consumabile; (2) nesatisfacuta de alimentare cu energie electrică are nevoie de, și (3) se deschide, rurale sau
marine în
zonele de coastă cu o viteză medie a vântului de 6 m/s la 10 m înălțime.
(b) fermele eoliene Offshore
Parcurile eoliene au crescut rapid în importanță, deoarece aproximativ
2000. Deși, în general, mai scumpe de instalat și de operat decât
de parcuri eoliene pe uscat, ele sunt favorizate în țările cu coastelor marine
, care au limitat terenuri disponibile pentru parcuri eoliene și/sau există lobby
împotriva energiei eoliene pe uscat. Mai mult decât atât, viteza vântului este de obicei mai
larg decât pe uscat. Europa domină turbine eoliene offshore în numere
și în dezvoltarea de tehnologie; până la jumătatea anului 2013 total off-shore
capacitatea eoliană a fost de 6 GW în 58 de fermele eoliene offshore în 10 țări,
arată că industria a devenit stabilit.
>
~
TWIDELL PAGINARE.indb 309
01/12/2014 11:37

310
Tehnologia energiei eoliene
Beneficiile de fermele eoliene offshore în comparație cu onshore, prin urmare,
să includă următoarele:

mai puternic și mai puțin turbulent vânt, prin urmare, mai multă putere;

zone mari disponibile sub un singur guvern legate de dreptul de proprietate;

nu în apropiere de populație care ar putea fi perturbată;

livrarea și instalarea de grele turn secțiuni și lame lungi de
barca (de livrare, de obicei, mai ușor decât onshore livrare de drum);

mari capacitatea totală de producție legată de înaltă tensiune, terenuri de transmisie a
rețelelor;

ieșire de ferme de vânt individuale controlate de la distanță de către operatorii de rețea.
Dezavantajele includ următoarele:

accesul dificil care necesită abilități speciale și siguranței furnizării;

mai scumpe de instalare și de întreținere, inclusiv submarine
fundații;

corozive saline mediu.
Fig. 8.24 prezintă exemple de construirea și exploatarea offshore wind
farms.
Cerințe speciale pentru turbine offshore includ următoarele:

marin evaluare a impactului asupra mediului, inclusiv a păsărilor migratoare,
mare mamifer de comunicare, benzi de transport, pescuit, vedere la mare;

viteza vântului de evaluare, probabil inclusiv sonic si laser
tehnici (a se vedea §7.4);

submarine fundații și anti-cutreiera protecție de la submarine de apă
curenți;

de specialitate marin instalare nava/platforme (vezi de ex. Fig. 8.24(b) și
(c));
Fig. 8.24
Fermele eoliene Offshore.
o
Middelgrunden, portul Copenhaga, la începutul cooperare
ferma eoliană offshore: 20 × 2 MW turbine.
b
Parc eolian offshore din Great Yarmouth
Anglia, la Scroby Sands, 30 × 2 MW turbine.
c
Jack-up vas de Rezoluție
instalarea de turbine eoliene offshore.
(a)
(b)
(c)
TWIDELL PAGINARE.indb 310
01/12/2014 11:37
§8.8 generarea de energie Electrică
311

întreținere accesul cu barca, inclusiv pe vreme rea;

marin transformare de mare putere conexiune la onshore rețelele electrice;

extrem de amănunțită componentă standarde pentru a reduce defectele și
de întreținere;

densitate mare de putere transmis distanțe semnificative pentru a integra cu
înaltă tensiune onshore grile (a se vedea §R1.4). Înaltă tensiune în curent continuu
(HVDC) este metoda cea mai eficientă, dar necesită o mare putere AC/DC
și DC/AC invertoare pentru conexiuni. Prin urmare, costurile pot fi ridicate.
§
8.8.4 aspectele Tehnice ale grid-connected turbine eoliene
Pentru toate grid-connected turbine eoliene, putere de ieșire P
T
<< P
G,
în cazul în care
puterea totală în rețea
P
G
,
este, de obicei, de la relativ foarte mare
(>~500 MW) centrale, stații electrice care controlează frecvența întreaga
rețea.
§R1.6 consideră generatoare de electricitate, mașini mai în detaliu. Cele
mai ieftine și cele mai robuste tip de generator este un generator de inducție
conectat direct la rețea. Turbina trebuie să funcționeze la aproape
frecvență constantă, într-un termen maxim de alunecare, de obicei, mai puțin de 5% față de
rețea legate de frecvență; aceasta este de obicei numit 'fix viteza'. În vânt slab,
există automate cut-out pentru a preveni automobilism. Dezavantajul unui
cuplat direct generator de inducție este faptul că turbina frecvența nu poate
schimba suficient pentru a menține chiar și aproximativ constantă sfat-raportul de viteza.
Cu toate acestea, există mai multe moduri în care sistemul poate fi făcut pentru a
produce energie electrică la destul de frecvență constantă, în timp ce permițându-variație în
turbina de frecvență. Acestea includ: (1) mai multe (de obicei două) combinație
înfășurări într-un generator de inducție pentru a conecta mai multe pole-perechi în vânt slab
pentru mai mici de rotație frecvență; (2) utilizarea unui generator sincron și
rectificarea acestuia de ieșire pentru curent continuu și apoi producerea prescris
de curent alternativ frecvența rețelei de alimentare cu invertor; (3) creșterea
efectiv aluneca pe un generator de inducție de schimbare activă de curent
și de fază la generator rotorului (de exemplu, într-o de două ori de inductie
generator); acest lucru necesită alimentare externă conexiune la înfășurarea rotorului prin
inele colectoare și perii; și (4) unele mici dimensiuni mașini folosesc două
generatoare din aceeași nacela, să zicem, 5 kW si de 22 kW, care sunt automat
conectate la două viteze în lumina și vânturile puternice respectiv.
§8.8.5 energie Eoliană contribuție națională de electricitate
generație
Pentru generarea de energie eoliană este variabilă, adăugarea de capacitate eoliană de, să zicem,
o capacitate de 100 MW la o rețea nu este echivalentă cu adăugarea a 100 de MW
de capacitate de la o sursă termică (cărbune, gaze naturale, nuclear, biomasă). În general, un
factor mediu de capacitate (uneori numit 'load factor) al unui vânt
turbina este de 20 până la 35%, în timp ce pentru o centrala termica este vorba despre
TWIDELL PAGINARE.indb 311
01/12/2014 11:37
312
Tehnologia energiei eoliene
70 la 90%. Totuși, nu toate sursele termice sunt echivalente (de exemplu energia nucleară
este potrivit numai pentru sarcina de bază și se închide la fiecare 18 luni
pentru realimentare, întrucât turbinele cu gaz sunt cele mai bune pentru răspuns rapid la vârf
cereri). Operatorii de rețea descrie contribuția diferitelor
surse de energie în termeni de capacitate de credit, și anume puterea de rating de o
rețea convențională a plantelor care este deplasat prin instalarea de eoliene
de putere sau al altor surse regenerabile de energie (a se vedea Caseta 15.3). Studii teoretice
indică faptul că 1000 MW (nominală) energia eoliană are o capacitate de credit de la 250 la
400 de MW, în funcție de pe termen lung, vânt caracteristici (Milborrow 2001).
Dacă vântul puterea vine de la o diversitate de site-uri, există mai puține șanse de a
- le pe toate și-a redus producția în același timp, și deci a prezis
capacitate de credit este mai mare. (Pentru o analiză mai amplă a acestor probleme
vezi Twidell (2013), 'Evaluarea Backup Cerințe pentru centralele Eoliene',
articol S15.1 în on-line de materiale suplimentare pentru această carte.)
Un utilitar de rețea trebuie să aibă mereu rezerva de capacitate de generare și
de încărcare-reducere facilități disponibile pentru toate formele de generație, mai ales
pentru că oferta din centrale electrice mari poate eșua brusc și
neașteptat la ori. Ca este, de asemenea, discutate în §15.4, aceasta a stabilit rezerva de
capacitate de la un mix de surse s-a dovedit în practică a fi suficiente pentru
energie din surse regenerabile variabile (în special energie eoliană) să contribuie cu până la 20 la 30%
din capacitatea totală. Cutie de 15,4 descrie astfel energie eoliană furnizarea în
comun Norweb sistemul de Norvegia, Suedia și Danemarca și în Irlanda.
Dispoziție specială este, probabil, necesară numai în cazul în care cota de vânt capacitate
depășește 20 la 30% din capacitatea totală. O astfel de dispoziție nu poate
fi neapărat numai în formă de suplimentare termică sau hidro-rezervor de capacitate, ca
în mod tradițional, dar pot include semnificative de gestionare a de sarcină, și/sau o
diversitate de surse regenerabile (a se vedea Caseta 15.5). Pentru a autorilor de cunoștințe, fără
suplimentare 'back-up de putere' a fost încă necesare sau construite oriunde,
numai pentru instalarea în plus capacitatea de energie eoliană.
§8.8.6 scară mai Mică și sisteme de proprietari independent
Deși turbine eoliene piața este dominată de mari grid-connected
turbine eoliene, alte, de obicei, mai mici, turbinele continua să fie
dezvoltate și vândute în număr mare pentru utilizări speciale, inclusiv
proprietarii independenți, microgenerare și off-grid de alimentare cu energie electrică. Foarte
mici mașini de capacitate între 50 W și 1 kW sunt comune
pentru iahturi și, în regiunile cu vânt, pentru vacanță, rulote și case, pentru
low-power de servicii publice (de exemplu, mediul rural statii de autobuz), și pentru mici
meteorologice și alte site-uri de măsurare. De multe ori solare fotovoltaice de putere este
folosit în paralel cu o astfel de energie eoliană, sau în mod independent. Puțin mai mare,
dar încă "mici" sunt de 5 kW până la 100 kW, turbine eoliene instalate pentru gospodărie,
fermă și instituționale de utilizare. Prin termenul de 'independent de proprietari ne referim la
persoane fizice și societăți individuale de operare mediu (>100 kW), la
mare (>1 MW) turbine individual sau într-un grup de două sau trei mașini,
TWIDELL PAGINARE.indb 312
01/12/2014 11:37
§8.8 generarea de energie Electrică
313
CASETA 8.2 MULTIMODE VÂNT SISTEM DE ALIMENTARE CU LOAD-CONTROL DE GESTIUNE
LA FAIR ISLE, SCOȚIA
Fair Isle este un caz izolat Scottish island în partea de Nord a Mării între continent Shetland și Orkney. La
o populație de aproximativ 70 este bine stabilit și progresivă în limitele dure dar frumos
mediului. Anterior, oamenii au depins în întregime de cărbune și petrol pentru căldură, petrol pentru vehicule și
motoare diesel combustibil pentru generarea de energie electrică. Apoi, în 1982 comunitate condusă de energie electrică de
cooperare instalat un
dar nu fermele eoliene). Cost-eficacitate este probabil dacă alte resurse energetice
sunt scumpe sau nu este disponibil, dacă surplusul de putere este vândut și dacă guvernul
stimulente, cum ar fi tarifele feed-in, sunt disponibile.
De multe ori, cele mai bune energie eoliană site-uri sunt în zonelor rurale, insulare sau marine
zone. Cerințele de energie în astfel de locuri sunt distincte, și aproape
cu siguranță nu va solicita intens de energie electrică de mari industriale
complexe. De multe ori utilizarea finală cerințe pentru controlate de energie electrică (de exemplu,
240 V/50 Hz sau 110 V/60 Hz pentru iluminat, mașini și electronice) sunt
susceptibile de a fi doar 5 până la 10% din necesarul total de energie pentru transport,
de gătit și de căldură. Prin urmare, energia eoliană ar putea furniza energie la prețuri accesibile
pentru căldură și transport, în plus față de standard, electrice utilizează. Astfel de
evoluții au avut loc mai întâi în anumite zonă de la distanță sisteme de putere (de exemplu de
Fair Isle sistemul descris în Caseta 8.2) dar acum sunt din ce în ce mai frecvente
și mai sofisticate (a se vedea, de asemenea, Caseta de 15.7 pentru exemplu la Utsira, Norvegia).
Mai mult decât atât, mediul rural sisteme de rețea sunt susceptibile de a fi "slab", deoarece acestea transporta
relativ joasă tensiune de energie electrică (de exemplu, 33 kV) pe distanțe relativ mari
cu complicată inductiv și rezistiv-putere-probleme de pierdere. Interfațare
o turbină de vânt slab grile este acceptabil, cu moderne electronice de putere
de control și a interfețelor; într-adevăr, energia eoliană poate fi folosit pentru a consolida
rețeaua de alimentare, de exemplu, prin controlul puterii reactive și a tensiunii.
Ca cu cele mai multe alte consumabile, conectare la rețea beneficiile energiei eoliene
instalații din punct de vedere tehnic și pot permite excesul de energie electrică de a fi vândute. Într-adevăr,
întreaga rețea beneficiile de astfel de producerea distribuită și la scară mai mică
microgenerare. Dacă conectarea la rețea nu este posibil, atunci turbine eoliene
pentru alimentarea cu energie electrică poate funcționa ca un autonome stand-alone sistem de
menționat în §8.8.2 pentru Clasele a și B; acest lucru necesită o conexiune cu
baterii și/sau controlabile generație ca de la un generator diesel sau, dacă
este posibil, energia hidro. În practică, funcționarea independentă încurajează
smart tehnologie să folosesc energie electrică pentru toate domeniile de utilizare a energiei, inclusiv
de căldură și de transport, și cu stocare a energiei (a se vedea Capitolul 15). Scopul
este de a optimiza variabilă generație având receptiv sarcini care
se potrivesc cererii la ofertă. Aproape sigur acest lucru necesită sarcini care stochează
energie, dintre care cele mai simple sunt capacităților termice pentru apă încălzită,
frigorifice și de încălzire și de răcire. Alte susceptibile de magazine sunt
baterii (de exemplu, pentru iluminat și vehicule electrice). Multi-mode sistem la
Fair Isle (Caseta 8.2) ilustrează ceea ce poate fi realizat prin luarea unui sistem integrat
întregului sistem de abordare, care acoperă atât de furnizare și utilizare a energiei. Astfel de
tehnologii inteligente apare mult mai mare de sisteme, dar este încă mai puțin frecvente.
TWIDELL PAGINARE.indb 313
01/12/2014 11:37
314
Tehnologia energiei eoliene
§8.9 PUTERE MECANICĂ
Din punct de vedere istoric energia mecanică în vânt a fost valorificată
în principal pentru transportul cu nave cu vele, pentru măcinarea cerealelor și pentru
pomparea apei. Aceste utilizări continua și poate crește din nou în
viitor. Această secțiune prezintă pe scurt aceste sisteme, având în vedere faptul că
energia electrică poate fi un intermediar vector de energie pentru o astfel de mecanică folosește.
§8.9.1 transport Maritim
Piața veche-echipate navele care navighează operate de forțele de frecare și au fost
ineficiente. Moderne de curse de iahturi, cu o subterane chila valorificarea forțele de ridicare,
sunt mult mai eficiente și pot naviga mai repede decât vântul (Fig. 8.9(c)).
Cu toate acestea, pur naviga putere pentru transportul comercial de persoane sau de marfă este
acum depășită cu excepția aplicatii de nisa (cum ar fi în țările în curs de dezvoltare
, cu multe insule împrăștiate (Nuttall et al. 2013). Unele evoluții
moderne nave de marfă au folosit fix vele stabilite de acționare mecanică.
§8.9.2 morărit
Moară de vânt tradițională (descris de obicei ca o moară de vânt olandeză) a
fost eclipsat de motor sau electrice acționate de mașini. Este puțin probabil că
natura variabilă a vântului asupra terenurilor vor fi potrivite din nou pentru
comerciale de frezat în direct de sisteme mecanice. Este mai bine ca wind
turbine sunt utilizate pentru a genera energie electrică într-o rețea și energie electrică, apoi
utilizat în motoare.
60 kW de putere nominală capacitate daneză-produsă de turbine eoliene, având un simplu generator de inducție. Acest
operate în persistente vânturi, cu viteza medie la 10 m înălțime de 10 m/s. Sistemul de control (Fig.
8.21(c)) depinde de frecvență sensibil, switch-uri activate și dezactivate sarcini individuale (de obicei,
radiatoare, atât de necesare în iernile reci!) în funcție de domeniul de frecvență, astfel încât controlul de rotație
rata de turbina. La frecvente perioade excesive de energie eoliană, în continuare căldură au devenit disponibile (de exemplu,
pentru produse alimentare în creștere într-o seră). Pentru o perioadă scurtă de timp, în 1980, un vehicul electric a fost acuzat de
sistem, dar această utilizare nu poate fi viabil cu tehnologia de timp. În ciuda puternice
vânturi, capacitatea de producție totală a fost de mic pentru populația deservită, standarde acceptabile fiind
posibil numai pentru că casele sunt bine izolate și atent strategiile energetice sunt menținute. Această
comunitate de energie eoliană a fost unul dintre primele exemple de smart tehnologie de energie.
Sarcina inițială controlate turbina operat cu succes pentru mai mult de 20 de ani, dar o creștere în
rețeaua de cerere necesare, în 1996, un al doilea 100 kW, turbine, care operează în paralel cu recondiționate
prima mașină și două de 30 kW generatoare diesel. Acest aranjament este necesar un control mai complex și
sistemul de alimentare, cu un dublu sistem de tarifare așa cum se arată în Fig. 8.22(b), și o suplimentare central "depozit" de
sarcină,
care a fost necesar pentru a menține sistemul stabil. Turbinele eoliene alimentare cu o medie anuală de 85% de
energie electrică și termică cererii și 97% în timpul iernii, atunci când vântul este mai puternic, în ciuda cererea fiind
mai mare.
Sursa: http://www.fairisle.org.uk/FIECo/ (care include o analiză detaliată inginerie descriere a actualizat sistemul
de operare începând cu anul 1998). Accesat 23/08/2014.
TWIDELL PAGINARE.indb 314
01/12/2014 11:37
§8.9 putere Mecanică
315
Ei bine țeavă
(a)
Pompa de camera
Apă de masă
Accident vascular cerebral în jos:
Până accident vascular cerebral:
Apa fiind tras în
camera pompei și
până conductei bine
Apa fiind forțat de
camera pompei în
conducta de bine
(b)
Fig. 8.25
De pompare a apei de mecanica direct pe link-ul de la un multi-blade turbina.
o
Volumetrică a pompei de apă. Arborele va fi conectat la rotirea arborelui cotit de turbine eoliene.
b
Turbina ('moara') cu ax legate de subterană.
§8.9.3 de pompare a Apei
Pomparea apei poate fi stocat în rezervoare și lacuri de acumulare sau absorbit în
sol. Acest condensator-ca proprietate netezește variabilă a vântului sursă,
și face vânt de putere benefică în cazul în care conectarea la rețea nu este posibilă
sau prea scumpe. Eoliene mecanice pompe de aproximativ 5 m rotor
diametru și până la 10 kW putere sunt comune în multe țări, inclusiv
Argentina, Australia și Statele Unite ale americii.
Apa este folosit mai ales în fermele de bovine, de irigare sau drenare.
Continuitatea aprovizionării este important, atât de mare soliditate multi-blade turbine sunt
utilizate, având mari inițială a cuplului în vânt slab. Low-viteza de rotație este
foarte acceptabil pentru o astfel de acțiune mecanică directă. Tradițional cilindrul
pompei cu un fix de acțiune (Fig. 8.25) este simplu și de încredere, în ciuda
necesită o relativ mare inițială a cuplului pentru a începe. Cu toate acestea, pentru o astfel de deplasare
pompe, livrate de apă pe unitatea de timp este proporțională cu rata de pompare
și, prin urmare, să turbina frecvența de rotație (P'
o
W). Puterea
vântului este proporțională cu viteza vântului la cub, care, la constanta sfat-viteza de
raport este proporțională cu lama sfat-viteza la cub, adică să
W
3
. La constanta coef-
suficiente de performanță C
P
, acest lucru dă P
T
o
W
3
. Prin urmare, vântul-de-apa-
de pompare de eficiență P'/P
T
scade ca 1/
W
2

. Astfel îmbunătățit pompe care


se potrivesc cu turbine eoliene de caracteristicile și de a menține simplitatea de
funcționare sunt importante pentru o mai eficientă direct de pompare a apei, astfel ca cel
mai scump progresivă-cavitate și pompe centrifuge. Deoarece apa
TWIDELL PAGINARE.indb 315
01/12/2014 11:37
316
Tehnologia energiei eoliene
este, de obicei, disponibile la low locații, și a vântului crește cu înălțimea, este
de multe ori sensibil pentru a avea un generatoare de electricitate turbine eoliene plasate pe un
deal de exploatare o pompă electrică plasate în apropiere de alimentare cu apă.
§8.9.4 producerea de Căldură prin frecare
Direct de disipare a energiei mecanice de la o turbină eoliană (de exemplu,
de paddle wheel systems) produce căldură cu eficiență de 100%, dar
de potrivire turbina la un mecanic 'dissipater' este extrem de dificil.
De turbine eoliene generatoare electrice sunt atât de comune și eficiente,
de energie electrică este favorizat ca intermediar vector de energie pentru electric
de încălzire alimentat.
§8.10 SOCIALE, ECONOMICE ȘI DE MEDIU
CONSIDERAȚII
Națiunea beneficiile de utilizare a energiei eoliene pentru energie electrică din
energie eoliană reduce emisiile și costurile de combustibili fosili, și
, prin urmare, scade impactul cauzează schimbările climatice. Există, de asemenea,
ocuparea forței de muncă și securitatea energetică națională beneficii. Proprietari și proprietarii de
turbine beneficia de venituri din exportate de putere, și de multe ori de propriul lor
uz de puterea lor.
Factorul cheie de succes pentru energie eoliană este site-ul de viteza vântului.
În general
u
trebuie să fie >5 m/s la 10 m înălțime, dar cu vânt site-uri sunt foarte
merită, din cauza u
3
dependența de putere. De exemplu,
o creștere în
u
(la 10 m) la 6 m/s la 8,2 m/s poate crește o turbina e
factor de capacitate de ~33% la ~49%, adică o creștere de 50% în ieșire pentru
aceeași intrare cost (a se vedea Problema 8.10). Îmbunătățiri tehnologice și
economii de scară-au concretizat în costul de capital pe unitatea de capacitate de
turbine eoliene în scădere în mod semnificativ începând cu anul 1990 (a se vedea Fig. 17.2(b); a se vedea
, de asemenea, graficul D6 în Anexa D pentru nivelat costuri).
Politicile guvernului de susținere care recunosc beneficiile de vânt
de putere, cum ar fi tarifele feed-in și obligat achiziții, suport
de creștere a instalațiilor și fabricarea, deci, de stabilire a unui viabile
industrie.
Cele mai multe efecte locale de energie eoliană, pot fi rezumate după
cum urmează:

Vizual: turbine trebuie să fie în teren deschis sau la mare, astfel încât acestea sunt în mod clar
vizibile în vedere directă. Rețineți că mai mare diametrul, cu atât mai lent și
mai grațios' rotația și mai mare turn și sfaturi lama.
Turbina de culoare pot fi alese ca fiind cele mai acceptabile, care este de obicei
alb. Pentru privitorul, în apropiere de obstrucție de dealuri, clădiri și copaci,
etc. poate preveni reperare. Turbinele pot fi considerate dăunătoare dacă
sunt observabile de pe site-uri istorice sau în zonele de frumusetea naturala. Simulare
TWIDELL PAGINARE.indb 316
01/12/2014 11:37
§8.10 Sociale, economice și de mediu
317
software-ul este folosit pentru a oferi o dinamică impresie vizuală de fermă eoliană
din toate punctele de vedere înainte de a se acordă permisiunea de a construi.

Sunet: sonoră, zgomotul produs de utilaje, cu lama sfaturi, turn de trecere etc. este
de așteptat să fie <40 dBA la 250 m, care este sleepable; infrasunetele de la
vibratii nu se aude sau adesea detectate, dar controversate. Astfel de efecte
nu sunt detectabile de la fermele eoliene offshore. Mașinile moderne sunt
considerabil mai silențioase decât dezvoltările timpurii ca producătorii caută
să răspundă la comentarii publice și de a îmbunătăți eficiența: zgomotul este adesea
un semn de mai puțin eficientă a energiei de captare. (On-line suplimentare
material eResource pentru această carte include rezumatul nostru de zgomotul produs de
turbine eoliene și de măsurătorile și criteriile necesare pentru
o analiză obiectivă acces §8.1.)

Păsări și lilieci impact: în general, foarte rar (<casă windows); evita
amplasarea în apropiere de garduri vii sau de alte insecte-zonele de hrănire. Specii variază
considerabil în comportamentul lor și deci expertiza este necesară înainte de a
permite construcția.

Agricultură: cabaline și bovine pot fi alarmat la început, dar va
deveni obișnuiți cu zgomotul. În general, utilizarea anterioară a
site-ului pentru animale și culturi continuă afectată în afară de 1 la
2% din suprafața utilizată de către turnul de baze, posturi de transformare și neasfaltate
drumuri.

TV și cuptor cu microunde: evita linie-de-vedere cu transmițătoare locale.

Radar: interferențele posibile pentru operatorii de zbor, care poate utiliza speciale de
'stergere' software pentru ecranele lor.

Soare umbre: poate da "flicker" prin windows; turbine poate
fi închis automat dacă o astfel de flicker este probabil.

Grila limitări: pentru exportabile putere poate necesita grila de modernizare.

Beneficii pentru comunitatea locală ca un întreg pot fi oferite de către
producător (de exemplu, mai ieftin livrările de energie electrică, donații pentru școli).
Energia eoliană impactul necesită luarea în considerare a mai multor discipline, inclusiv
ecologie, estetica, patrimoniul cultural și percepțiile publice; Pasqualetti
et al. în Energie Eoliană în Vedere (2002) consideră că aceste efecte în detaliu și
cartea lor este recomandat de lectură.
Dezvoltatorii de parcuri eoliene trebuie să obțină locale sau naționale de planificare
permisiunea înainte de instalarea unui parc eolian, care poate implica luarea în considerare
a tuturor factorilor de mai sus de la experți independenți. În consecință,
procesul de pregătire o aplicație a devenit cuprinzătoare și
profesionale. Toate aceste proceduri sunt necesare, dar ele sunt
consumatoare de timp și costisitoare. Dacă cererea este respinsă, atunci de apel poate
fi făcut.
Totuși, rezultatul final este că, națională și mondială de energie eoliană capacități
sunt în creștere, carbon și alte emisii sunt atenuează, tehnologia
se îmbunătățește și de cele mai multe perceput efecte adverse sunt în scădere
pe unitatea de ieșire generate.
TWIDELL PAGINARE.indb 317
01/12/2014 11:37
318
Tehnologia energiei eoliene
ÎNTREBĂRI RAPIDE
Notă: Răspunsurile la aceste întrebări sunt în textul de la secțiunea relevantă
din prezentul capitol, sau poate fi ușor dedusă din aceasta.
1
Pentru generarea de energie electrică de energie eoliană într-un utilitar de rețea,
de ce este o viteză a vântului mai mică de 3 m/s neglijabil beneficiu; încă o
viteza a vantului de 6 m/s este benefic?
REZUMAT CAPITOL
Astăzi, turbinele eoliene sunt acceptate ca 'mainstream generarea de energie electrică pentru rețeaua de utilități rețele de
regiuni cu suficientă vântului, în special în Europa, China și statele UNITE ale americii. Atunci când este instalat în locații cu
vânt
(viteza medie a vântului de cel putin ~5 m/s la 10 m înălțime), energia eoliană este cost-competitive cu toate celelalte forme
de producere de energie electrică, mai ales atunci când este dat credit financiar pentru a nu polua. În consecință, lumea
capacitate instalată de energie eoliană a crescut de la
≥20%/an în ultimul deceniu și continuă să o facă, aproape
tot în mai multe turbine eoliene pe uscat și în larg.
Pentru comune viteza vântului distribuții, rata medie anuală de energie de la o turbină eoliană din zona
se apropie
r

P
COu
()
T
P
0
3
, unde u
0
este în amonte viteza vântului,
r este densitatea aerului și C
p

este
de
a [adimensional] coeficientului de putere. Maximă putere nominală capacitate de o turbină eoliană este dat pentru
o anumită viteza vantului nominala, de obicei de aproximativ 12 m/s. La această viteză, producția de energie electrică de
aproximativ 0,3 kW/
m
2
de secțiune transversală-ar fi așteptat cu C
p
De ~40%. Un simplu argument fizice arată că există o
valoarea maximă a C
P
deoarece aerul care trece prin turbina are să-și păstreze suficientă energie cinetică pentru a
continua aval; astfel C
P
(max)
~ 0.59 (Lanchester-Betz-Zhukowsky criteriu).
Aerodinamica de lame de turbine eoliene este foarte asemănătoare cu cea de aripi de avion, cu folie
forma atent alese pentru a maximiza forta de ascensiune și reduce forțele de frecare. De departe cele mai comune turbine
pentru producerea de energie electrică sunt axei orizontale cu două sau trei lame, cu rază variind de la ~5 m la
~60 m. Dacă turbina este extragerea de energie eficient, diferențele dintre lamele nu sunt evidente la
vânt și captarea energiei este maximizată, în timp ce permițându-aer pentru a scăpa de direcția vântului. Pentru a preveni
generatoare
de supraîncălzire dincolo lor puterea nominală de ieșire, turbinele sunt controlate în mod pasiv sau activ la această ieșire în
vânt cu viteze de mai mult de aproximativ 12 m/s. În vânturi puternice, când violente turbulențe s-ar putea deteriora
aparatul, de rotație este, de obicei, oprit pentru utilitate scară mașini de la cut-out punct de ~30 m/s. Pentru a
preveni uzura de nici un beneficiu, de turbine, de asemenea, un cut-in viteza vântului, de obicei de 3 m/s până la 4 m/s.
A coeficientului de putere C
P
depinde foarte mult de sfat-raportul de viteza
λ = RW /u
0
, unde R este lama
raza și
W este viteza unghiulară de rotație (în radiani/s). Pentru o axă orizontală turbina să funcționeze
eficient, rotorul trebuie să se rotească la o viteză unghiulară astfel că
λ ≈ 6 la 7. Acesta este motivul pentru mașini mici
roti rapid și mașini mari încet. Aceasta implică, de asemenea, că, în calitate amonte viteza vântului u
0
modificări, astfel încât
de asemenea ar trebui să-viteza de rotație
W schimbare. Acest criteriu este strâns legată de controlul fluxului unghi
f între direcția de rotație de viteză și vectorul viteză a aerului raport cu lama. Lama
element teorie consideră că modul în care toți acești parametri variază de-a lungul lamă rotativă, și este utilizat pentru exacte
calcule de proiectare. De asemenea, aceasta explică de ce, practic lame de turbine au o poftă de mâncare, cu lama care se
confruntă mai aproape
de vânt de intrare la hub decât la vârful lamei.
Un bine-a reușit grila poate face față cu variația de energie eoliană de ieșire ca viteza vantului variaza, cu condiția
puterea vântului
≤~20% din puterea totală în rețea, mai ales ca acolo sunt mai multe moduri în care o
turbină eoliană poate fi făcut pentru a produce energie electrică la frecvența rețelei. Capacitate de credit este puterea
de rating care operatorii de rețea în considerare disponibile la diferite forme de generație în întreaga rețea;
pentru națională, energia eoliană, energia eoliană evaluat la 1000 MW poate avea o capacitate de credit de 250-400 MW.
Turbine mai mici continuă să fie dezvoltate și vândute pentru utilizări speciale, inclusiv de microgenerare și off-
rețeaua de alimentare cu energie electrică.
TWIDELL PAGINARE.indb 318
01/12/2014 11:37
Probleme
319
2
Ce este un parc eolian? De ce sunt multe noi parcuri eoliene construite offshore?
3
O turbina eoliana are o putere nominală de 100 kW și turația nominală de
12 m/s. Estima puterea de ieșire într-un punct a (a) 9 m/s;
(b) 18 m/s.
4
Numele a două utilizări ale energiei eoliene altele decât producerea de energie electrică.
5
Definirea coeficientului de putere a unei turbine eoliene. Ce (a) teoretic
și (b) de obicei, este valoarea maximă a acestui parametru?
6
Defini sfat-raportul dintre viteza de turbine eoliene. De ce este important?
7
Cel mai comercial turbine eoliene în uz astăzi au o orizontală
și axa de trei lame. Numele altor două tipuri de eoliene comerciale
turbina.
8
Explica, cu utilizarea de schiță diagrame sau un model de hârtie, de ce
aerodinamic de forță pe o lamă trebuie să fie rezolvate de două ori pentru a obține
forța de accelerare pe o turbină eoliană cu rotor.
9
De ce sunt mari palete de turbine eoliene de multe ori răsucite?
10
Care sunt avantajele și dezavantajele (un) viteză fixă
turbine; (b) cu viteză variabilă turbine?
11
Numele a trei factori care te-ai aștepta să fie considerate într-o
aplicație de planificare pentru un nou parc eolian.
PROBLEME
8.1
Din (8.17) fracțiunea de energie extras din vânt este
coeficientului de putere C
P
= 4o(1a)
2
. Prin diferențierea cu privire la
o, arată că valoarea maximă a C
P
este 16/27 când o
= 1/3.
8.2
Calculul coeficientului de putere C
P
prin impulsului
teoria (§8.3.1) se poate proceda în termeni de b
= u
2
/u
0
în loc de în
termeni de o
= (u
0
− u
1
)/u
0.
Arată că (a) C
P
= (1 − b
2
)(1 + b)/2; (b) C
P
este un maxim la 16/27 când b
= 1/3; (c)
o
b
(1
)/2
=−
; și (d)
coeficientul aerodinamic C
F
= (1 − b
2
).
8.3
(a) Prin luarea în considerare a raportului dintre domeniile - O
0
și O
1

din Fig. 8.5,
arată că puterea maximă de extracție (conform liniar
impuls teorie) pe unitatea de suprafață Un
0
este 8/9 incidentului
putere în vânt.
(b) Dovedesc că cuplul produs de o turbină eoliană cu rotor de
raza R poate fi exprimată ca
CRu
( / 2)
/
P
3
0
2
p
r
λ
G=
.
8.4
O mare turbină eoliană are lame de 50 m lungime. În vânturi puternice de
20 m/s, se calculează rata de rotație dacă lama sfaturi au fost la egalitate de
viteza sunetului. Este probabil ca aceasta să se întâmple?
8.5
Consultați schițele în Fig. 8.13. Ia în considerare un vânt, în amonte
de viteză u
0
, care trece prin rotorul unei turbine, cu n lame
fiecare de lungime R de cotitură la viteza unghiulară
W. Presupunem că această
TWIDELL PAGINARE.indb 319
01/12/2014 11:37
320
Tehnologia energiei eoliene
mișcarea tulbură o lungime d fluxului de aer, care trece în
timp de t
w
:
(i) se Calculează timpul t
b
pentru o lama pentru a trece la poziția de
anterior lama.
(ii) Dacă puterea maximă este extras atunci când t
w
≈ t
b
, arată că
sfat-raportul de viteza
λ ≈ (2pR/nd).
(iii) în Cazul în teste în tunelul aerodinamic pe anumite modele de turbine arată că
putere maximă de extracție apare atunci când aproximativ d
≈ R/2,
arată că puterea maximă a coeficientului apare la
λ ≈ 6 pentru
două lame model, și la
λ ≈ 3 pentru un mandat de patru pale model.
(iv) Ce alte deduceri poate face de la
analiza?
8.6
Fluxul de aer în vântul va fi turbulentă dacă numărul Reynolds
R

2000 (vezi §2.5). Calcula viteza maximă a vântului pentru
flux laminar în jurul valorii de un obstacol de dimensiuni 1.0 m. Este laminar
flow realist pentru turbine eoliene?
8.7
O serie de modele de turbine eoliene trece de ieșire a aerului de la
un set de lame imediat pe un al doilea, set identic (de exemplu,
două contrar lame rotative set pe aceeasi axa orizontală). Până
în vedere două acționare discuri în serie, și folosind liniar
impuls teorie, spectacol care a combinat puterea maximă a coeficientului
C
P
egal 0.64.
Notă: aceasta este doar puțin mai mare decât cea maximă de
16/27
= 0.59 pentru o singură trecere a aerului printr-un set de
lame. Astfel, într-un tandem axa orizontală mașină de identice
lama seturi, și într-adevăr într-o axă verticală turbina, mic plus de putere
este câștigat de către fluxul de aer care trece un al doilea set de lame la o astfel de
apropiere.
8.8
(a) Calcula maximă axială pe unitatea de suprafață de
rotor pentru turbină la 20 m/s vânt.
(b) daneză standard pentru presiunea axială de proiectare este de 300 N/m
2

de
rotor zona. Care este minimul posibil, viteza vântului că aceasta
corespunde?
8.9
Din Fig. 8.12 și ecuația (8.13), arată că, de obicei maxim
de putere de extracție are loc la sfat-raportul de viteza
λ ~ 1.5 cotan
f. Prin urmare,
explica într-un limbaj nontehnic ce putere maximă de extracție
în diferite viteza vântului se referă la menținerea
λ constantă.
8.10
O turbină eoliană evaluat la 600 kW are un cut-in viteza de 5 m/s, cu o putere
de viteză de 15 m/s și viteză de 22 m/s. Puterea de ieșire
în funcție de viteza vântului la înălțimea butucului este rezumată în
tabelul de mai jos. Centrul de înălțime este de 45 m.
TWIDELL PAGINARE.indb 320
01/12/2014 11:37
Probleme
321
Viteza / (m/s)
0
2.0
4.0
6.0
8.0
10.0
Putere de ieșire / kW
0
0
0
80
220
360
Viteza / (m/s)
12.0
14.0
16.0
18.0
20.0
22.0
Putere de ieșire / kW
500
550
580
590
600
0
Calcula aproximativ probabil anuale putere de ieșire, și, prin urmare,
capacitatea sa de factor de Z:
(a) o extrem de vânt site-ul în cazul în care vântul urmează o Rayleigh
de distribuție cu o viteză medie de 8,2 m/s, măsurată la o înălțime de
10 m (de exemplu, condiții cum ar fi North Ronaldsay: §7.3.3);
(b) la un potențial atractiv site-ul în cazul în care viteza medie a vântului la
10 m este de 6 m/s.
8.11
În conformitate cu §7.3.2 viteza vântului u
z
la înălțimea z (>10 m) este
aproximativ proporțională cu z
0.14
, întrucât densitatea de putere
în vântului variază ca u
z
3
. Prin trasarea u
z
3

împotriva z arată că pentru
z >100 m variația de densitate de putere cu înălțime este relativ
mic. Rezultă că nu este util să avem foarte mare turnuri
(de exemplu >100 m sau cam asa ceva) pentru turbine eoliene mici. Cum ar putea
argument fi diferite pentru turbine mari de vânt?
8.12
Ia în considerare o turbina care menține constantă sfat-raportul de viteza (și
, prin urmare constantă C
P
pentru putere de ieșire P
T
> puterea nominală P
R
). Dacă
sale cut-out viteza u
co
este mare (>> viteza nominală u
R

), și vântul
urmează o distribuție Rayleigh, arată că puterea de ieșire
poate fi exprimată ca:
r
p
p
()
=
+
− 










P
CO
u
P
u
u
2
6
exp
4
T
P
R
R
1
3
2
A evalua această expresie pentru unele cazuri tipice (de exemplu, u
= 8 m/s,
u
R
= 15 m/s, P
R
= 600 kW, Un = 800 m
2
).
8.13
În idealizat trageți aparatul (Fig. 8.9(c))., puterea necesară
pentru a împinge clapeta într-o linie dreaptă este (forța) x (viteza). Folosind
expresia (8.39) pentru forță, arată că puterea maximă
de ieșire din acest sistem este obținut atunci când v
= u
0
/3, și că
puterea maximă se poate obține este
P
C
Au
(4 / 27) (
)
D
D
(max)
1
2
0
3
r
=
și
prin urmare, puterea maximă a coeficientului este
C
C
(4 / 27)
P
D
(max)
=
.
TWIDELL PAGINARE.indb 321
01/12/2014 11:37
322
Tehnologia energiei eoliene
NOTE
1
O overbar denotă o medie a ceea ce este sub bara.
2
Termenul de vânt sistem de conversie a energiei' (WECS) este folosit de câțiva autori pentru a distinge tot
ansamblul de la real turbina.
3
Datele de la Global Wind Energy Council (2013).
4
Teoria a fost dezvoltat independent de Lanchester (1915 în marea BRITANIE), Betz (1920 în Germania) și
Zhukowsky (1920 în Rusia; de multe ori scris ca Joukowski). Mai multe surse se referă numai la Betz.
5
US National Aeronautica si Spatiu Administration.
BIBLIOGRAFIE
General
Burton, T., Sharpe, D., Jenkins, N. și Bossanyi, E. (2011, 2nd edn) Energie Eoliană Manual, Wiley, Chichester.
Aceasta este turbine eoliene 'biblia', cu avansate fundamentale de teorie și experiență profesională de proiectare,
de fabricație și de punere în aplicare a energiei eoliene.
Gipe, P. (2004) Energie Eoliană, James și James, Londra. Aprofundată și analiza personală a energiei eoliene
de dezvoltare, în special în statele UNITE ale americii; o prejudecată la proprietar independent.
Golding, E. W. (1976) Generarea de energie Electrică de Energie Eoliană, republicat, cu material suplimentar de R. I.
Harris, E. și F. N. Spon, Londra. Textul clasic, care a devenit un ghid de mult de lucru modern.
Hansen, M. O. (2007, 2nd edn) Aerodinamica Turbinelor Eoliene, Routledge, Londra. În mod clar prezentate, dar
avansate de text dintr-un lector cu experiență; se mută de la fundamentale aeronautică să lama element de teorie, cu
fizic explicații suplimentare complexitatea și aplicații.
Manwell, J. F., McGowan, J. G. și Rogers, A. L. (2010, 2nd edn) Energia Eoliană a Explicat, John Wiley & Sons,
Chichester. Un mare manual pentru studiu de specialitate.
Milborrow, D. (2001) 'energia Eoliană revizuire', în J. Gordon (ed.), Energia solară: starea de arta, International
Solar Energy Society și James & James, Londra.
Pasqualetti, M. J., Gipe, P. și Îndreptător, R. W. (2002) Energie Eoliană în Vedere, Academic Press cu Reed Elsevier, San
Diego; Academic Press, Londra. Un editate set de capitole, cea mai mare parte de către experți, altele decât ingineri, privind
vizuale și alte non-inginerie impactul instalațiilor eoliene.. Descoperiri importante în personal
estetica și a patrimoniului cultural.
Van Est, R. (1999) Vântul Schimbării, Cărți Internaționale, Utrecht (în limba engleză). Un studiu comparativ de politică
și instituțională factori de dezvoltare a energiei eoliene în California și Danemarca.
În mod special referire
Bowden, G. J., Barker, P. R., Shestopal, V. O. și Twidell, J. W. (1983) La funcția de distribuție Weibull și vânt
puterea statistică, Inginerie de Vânt, 7, 85-98.
Jeffreys, H. Jeffreys, B. (1966) Metodele Fizicii Matematice, Cambridge University Press, Cambridge.
Cu atenție a prezentat textul de avansate de matematică pentru ingineri, etc.
Justus, C. G., Hargreaves, W. R., Mikherl, A. S. și Morminte, D. (1977) Metodele de estimare a vitezei vântului
frecvența de distribuție', Journal of applied Meteorologie, 17, 673-678.
TWIDELL PAGINARE.indb 322
01/12/2014 11:37
Bibliografie
323
Kaimal, J. C. și Finnigan, J. J. (1994) Strat Limită Atmosferic Curge, Oxford University Press, Oxford.
Analiza fundamentală și explicarea de către experți.
Kragten, A. (2009) 'Morile de vânt utilizează aerodinamic ca forțele de propulsie: o sarcină fără speranță', web publicate
la www.bidnetwork.org/en/plan/302071, Kragten Design, Populierenlaan 51, 5492 SG Sint-Oedenrode, În
Olanda.
Nuttall, P. Newell, A., Prasad, B., Veitayaki, J. și Olanda, E. (2013) O revizuire a durabilă mare de transport
pentru Oceania: furnizarea de context pentru surse regenerabile de energie de transport pentru Pacific', Politica maritimă,
http://dx.doi.
org/10.1016/j.marpol.2013.06.009.
Panofsky, H A. și Dutton, J. A. (1984) Turbulențe Atmosferice, Modele și Metode de Inginerie
Aplicații, Wiley, New York. Utilă o analiză și fundal pentru turbine eoliene de generare.
Petersen, E. L. (1975) Pe Energia Cinetică a Spectrului de Mișcări Atmosferice în Stratul Limită Planetar,
Raport nr. 285 de Vânt site-ul de Testare, Riso, Danemarca.
Rohatgi, J. S. și Nelson, V. (1994) Vânt Caracteristici: O analiză pentru generarea de putere, Burgess
Publishing, Edina, MA, statele UNITE ale americii.
Reviste și site-uri web
Energia Eoliană Lunar. Vrinners Hoved, Knebel, Danemarca. În limba engleză; nivel mondial știri și articole.
Ingineria Vântului. Multi-Science Publishing Co., 5 Wates Fel, Brentwood, marea BRITANIE. Academice și de cercetare
journal.
Energie Eoliană. Wiley. Academice și de cercetare journal.
World Wind Energy Association (www.wwindea.org/home/index.php). Excelent rapoarte și date.
American Wind Energy Association (www.awea.org/).
Daneză a Industriei Eoliene de Asociere; educație (www.windpower.org/en/knowledge/windpower_wiki.html).
Site excelent pentru studenți; explicații clare, cu suport video.
RenewableUK (anterior Britanic Wind Energy Association) (www.renewableuk.com/). Marea BRITANIE date de vânt și
de activități; de asemenea valurilor și a mareelor.
Spaniolă generarea de energie eoliană și proporția național de aprovizionare în timp real și de datele anterioare (www.ree.es/
ingles/operacion/curvas_eolica.asp#). Instructiv și interesant.
Vântul Atlase din Lume (www.windatlas.dk/). Vântul atlase; fundal, metode și disponibilitatea.
Video de pe Youtube, ceea Ce este în interiorul unei turbine eoliene (2010) (www.youtube.com/watch?
v=LNXTm7aHvWc). Instructiv,
mai ales pentru personalul de întreținere.
TWIDELL PAGINARE.indb 323
01/12/2014 11:37

Resursele de biomasă de fotosinteza


CONȚINUTUL
Obiective de studiu
325
§9.1 Introducere
326
§9.2 Fotosinteza: un proces cheie
pentru viața de pe Pământ
327
§9.3 nivel Trofic fotosinteza
328
§9.4 Raport de fotosinteza
la alte procese de plante
331
§9.5 Fotosinteza la celular
și nivel molecular
332
§9.5.1 Reacție de ansamblu
333
§9.5.2 considerente Termodinamice 335
§9.5.3 Photophysics
338
§9.5.4 Numărul de fotoni pe
carbon fix
341
§9.5.5 Eficiența fotosintezei
la nivel foton
341
§9.6 Energie agricultura: biomasa
producția de energie
343
§9.6.1 Energie agricultura
343
§9.6.2 Lemn de resurse
346
§9.6.3 producției și de îmbunătățire a
347
§9.6.4 Cât de mult biomasă este
disponibil pentru energie?
347
§9.7 R&D a 'îmbunătăți fotosinteza
350
§9.7.1 fiziologie a Plantelor și biomasă
350
§9.7.2 Bioengineered fotosinteza 351
§9.7.3 fotosinteza Artificială
351
§9.8 aspectele Sociale și de mediu
351
§9.8.1 Bioenergie în raport cu
agricultură și silvicultură
351
§9.8.2 Alimentare față de combustibil
352
§9.8.3 gaze cu efect de Seră impactul:
bioenergie și a absorbanților de carbon
352
§9.8.4 Bioenergie în raport cu
sistemul energetic
353
§9.8.5 impactul Uman pe net
producția primară (NPP)
353
Rezumat capitol
354
Întrebări rapide
355
Probleme
355
Note
356
Bibliografie
356
Caseta 9.1 Structura de frunze de plante
334
Caseta 9.2 trestie de Zahăr: un exemplu
de energie agricultura
344
Caseta 9.3 Cum este biomasa, resurse
evaluată?
349
CAPITOLUL

9
TWIDELL PAGINARE.indb 324
01/12/2014 11:37

www.shahrsazionline.com
Lista de tabele
325
OBIECTIVE DE STUDIU

Știu cum solare forme de energie din biomasă prin
fotosinteză.

Dau seama că biomasa este stocată energia solară.

Să fie conștienți de fotosintetice ratele de creștere pentru
producția de culturi alimentare și combustibili.

Compara și de contrast fotovoltaică și pho
tosynthesis.

Apreciez context ecologic de bio -
energie.

Se referă nevoia omului pentru alimente la nevoie pentru
energie.

Fiți conștienți de utilizare a terenurilor și probleme de productivitate.

Ia în considerare biologice captarea și se referă
la schimbările climatice.
LISTA DE FIGURI
9.1
Trofice nivel global fotosinteză.
329
9.2
Planta nivel de fotosinteză.
332
9.3
Nivel Molecular fotosinteză.
333
9.4
Structura și scară de frunze de plante.
334
9.5
Electron de excitație (a) de căldură și (b) absorbție de fotoni.
336
9.6
Reducerea nivelului R de compuși de carbon. Entalpia schimba per atom de carbon.
337
9.7
Reducere de apa în oxigen și protoni de la centrul de reacție a fotosistemului 2.
338
9.8
FranckCondon diagrama.
339
9.9
Transferul de energie de molecule de pigment de lightharvesting sistemul special
de reacție centru.
340
9.10 spectrul de Absorbție (curba continuă) și spectrele de acțiune (curba punctată) tipic verde frunze de plante.
342
9.11 trestie de Zahăr agroindustry: diagramă a fluxului de proces.
344
9.12 O fabrică de zahăr înființat pentru a produce zahăr, etanol, iar surplusul de energie electrică.
345
LISTA DE TABELE
9.1
Aproximativ randament fotosintetic pentru o gamă largă de circumstanțe.
331
9.2
Pierderile de energie în fiecare etapă a fotosintezei.
343
9.3
Avantajele și pericolele de energie agricultura.
345
9.4
Practice maxime de biomasă randamentelor.
348
9.5
O estimare a potențialului tehnic de bioenergie disponibile de noi plantații pe terenuri disponibile
și adecvate pentru speciile de plante selectate.
350
TWIDELL PAGINARE.indb 325
01/12/2014 11:37
326
Resursele de biomasă de fotosinteza
§9.1 INTRODUCERE
Materialul de plante și animale, inclusiv deșeuri și reziduuri,
este numit de biomasă. Este organic, carbonbased material care reacționează cu
oxigenul naturale în procesele metabolice și în combustie pentru a elibera caldura
asta, mai ales dacă la temperaturi >400°C, poate fi folosit pentru a genera
muncă și energie electrică. Inițial materialul poate fi transformat prin metode chimice
și procese biologice de a produce biocombustibili, și anume biomasei procesate într -
o formă mai convenabilă, în special combustibili lichizi pentru transport. Exemple
de biocombustibili includ gaz metan, etanol lichid, metil esteri, uleiuri și
solid carbune. Termenul de bioenergie este folosit pentru a descrie atât de biomasă și
biocombustibili.
Bioenergia este, de departe, cele mai utilizate resurse de energie regenerabilă de
energie, o valoare, fiind de aproximativ 10% din totalul global de energie primară de alimentare
dacă necomerciale lemne de foc pentru gătit și utilizarea comercială a deșeurilor
sunt incluse (Edenhofer et al. 2011). În ciuda istoric utilizarea biocombustibililor,
există un mare potențial pentru mai multe energyefficient și utilizarea durabilă în
ambele în curs de dezvoltare și țările dezvoltate. Tehnologii pentru a face acest lucru sunt
descrise în capitolul următor.
Biomasa este format în mod natural de fotosinteză, care este de conducere
funcție de toate formele de viață, inclusiv, desigur, viața umană prin alimente; - suport
procese sunt prezentate în §9.2 §9.5, cu un accent pe fizica
cal principiile implicate. Susținerea ulterioară a proceselor este o cheie
funcția de sisteme ecologice; procesele care au loc în mod natural și
cu succes, fără intervenția omenirii. Suntem înțelepți dacă am
înțelege și de a participa în astfel de procese, fără a distruge
status quo-ul.
Un aspect de fotosinteza este procesul dominant pentru
rapid stocarea energiei solare într-o formă stabilă. Ar trebui să înțeleagă
procesul și să învețe de la ea, cu speranța tehnologice applica
ări (§9.7). Evident, comparația este cu celule fotovoltaice, dar acestea
nu au inerente de stocare a energiei.
Macroscopic resursele de biomasă sunt luate în considerare în §9.6. Aceasta evaluează
energia agricultura (producția de combustibili și energie ca fel principal sau sub
subsidiare produs de agricultură, silvicultură, acvacultură) și de procesare
de deșeuri organice'. Aceasta include energia potențială resursă de
biomasă.
În §9.8, vom evalua amploarea și potențialul de biomasă, ca o energie
de resurse termen umbrela de sustenabilitate. În special, ne uită-te la
implicațiile sale de gaze cu efect de seră, și rețineți că activitățile umane
deja face uz direct de mai mult de 25% din net fotosintetice
de ieșire a tuturor conexe plante de pe Pământ.
TWIDELL PAGINARE.indb 326
01/12/2014 11:37
§9.2 Fotosinteza: un proces esențial pentru viața pe pământ
327
§9.2 FOTOSINTEZA: UN PROCES CHEIE PENTRU VIAȚĂ
PE PĂMÂNT
Fotosinteza este de a face (sinteza) de structuri organice și
chimice magazine de energie prin acțiunea radiației solare. Este de departe cea
mai importantă de energie regenerabilă proces, pentru că organismele vii
sunt realizate din material fixat prin fotosinteză, iar activitățile noastre se bazează
pe fotosintetică a produs oxigen cu care energia solară
este cea mai mare parte stocate. De exemplu, metabolismul uman continuu
comunicate de aproximativ 150 W per persoană din alimente. Astfel, atât materiale
și de energie pentru toate formele de viață sunt puse la dispoziție în gazele circulă în
atmosfera Pământului, și anume dioxidul de carbon și oxigen.
Deși fotosinteza este un fizic induse de procesul și
funcția de conducere de inginerie naturală, subiectul lipsește de
cele mai multe fizică și inginerie texte. De prea multe ori, fotosinteza este con
studiate doar ca un aspect de biochimie, care, deși a consi
reci importanță, este insuficientă. Prin urmare, am dat un physicsbased
descriere a proceselor cheie, atrăgând atenția asupra sa analogii cu
fotovoltaice de generare (Capitolul 5) și radioreceiving antene, cu
mai multe detalii pe site-ul de această carte. www.routledge.com/books/
detalii/9780415584388
Fotosinteza are loc în ambele desfășurate pe uscat și plante marine, astfel
care influențează concentrația de CO
2

în nostru de atmosfera Pământului și con


sequently efectul de seră (§2.9). Cu toate acestea, cererile de bio -
energie implică în principal terestre biomasă, pe care acest capitol se concentrează.
Fotosinteza pe teren magazine de energie la o rată de aproximativ 0,8
× 10
14

W (adică
aproximativ 10 kW pe persoană; vezi Problema 9.1). Ca biomasă se descompune sau com
busturi, energia stocată este eliberată din reacțiile cu oxigenul. Acest lucru este
echivalentul energetic al puterii de ieșire de aproximativ un milion de mare nucleare
centralele electrice și este de aproximativ de patru ori prezent comerciale totale
a consumului de energie al omenirii.
Practic, toate terestre fotosinteza are loc în frunze de vie
plante. Radiația solară determină electronii să fie încântat într-o parte cheie a
acestor frunze (cloroplaste), care printr-o serie complexă de chemi
cal procesele prezentate în §9.4 9.5, duce la producerea de oxigen
și carbonbased material structural. Aceste procese chimice sunt sen
urilorreprezintăunaspect la temperatura de frunze, astfel încât plantele au evoluat pentru a se asigura
că unii radiația solară este reflectată sau transmisă, mai degrabă decât absorbit
(care este de ce frunzele sunt rareori negru). Rolul de transpirație apă în
ambele reacții chimice și temperatura de control este integrat un
aspect al procesului.
Procesele de energie în fotosinteză depinde de fotoni
(energie pachete) de radiații solare, fiecare de energie h
ν, unde h este
Constanta lui Planck și
ν este frecvența radiației. Organic
materialul produs este în principal carbohidrați (de exemplu, celuloză, care este
TWIDELL PAGINARE.indb 327
01/12/2014 11:37
328
Resursele de biomasă de fotosinteza
un longchain polimer de glucoză C
6
H
l2
O
6

). Dacă acest (uscat) material este ars în


oxigen, caldura degajata este de aproximativ 16 MJ/kg (4.8 eV per atom de carbon,
460 kJ pe mol de carbon). Repararea de un atom de carbon din atmos
pheric CO
2

la carbohidrati veniturile printr-o serie de etape în verde


plante, inclusiv alge:
1
Reacții în luminasolară fotoni excita si separat electronii
și protonii din atomi de hidrogen din apă, cu O
2

ca un important de
produs și cu electroni entuziasmat în două etape pentru a produce o puternică
reducere a substanțelor chimice.
2
Reacțiile nu necesită lumină (numit "întunecat" reacții, dar care apar la
orice moment): reducerea substanțelor chimice din (1) reducerea CO
2
pentru a
drates, proteine și grăsimi.
În general ecuații chimice (9.1) și (9.2), atomii de oxigen ini
în mod CO
2
și H
2
O
.
se disting, acesta din urmă fiind prezentat cu un
punct peste O. Astfel, combinând atât de lumină și întuneric reacții și
neglijarea multe etape intermediare:
CO + 2H
2
2
O
.
→

lumina
O
.
2
+ [CH O] + H O
2
2
(9.1)
în cazul în care produsele au aproximativ 4,8 eV per atom de C mai entalpie (energie
potențialul de producție) decât materialul inițial pentru că de absorbție
de cel puțin opt fotoni. Aici [CH
2
O] reprezintă o unitate de bază a
drate, astfel încât reacția de zaharoză de producție este:
12CO + 24H
2
2
O
.
→

lumina
12 O
.
2
+CHO
+ 13H O
12 22 11
2
(9.2)
Există o mare varietate în toate aspectele legate de fotosinteza, de la scara
de plante până la nivel molecular. Nu trebuie presupus că orice
sistem este la fel de simplă cum este descris în acest capitol, care concen
trates în general principii fizice. Cu toate acestea, rezultatul final este că
energia de la Soare este stocat în stabil produse chimice pentru utilizare ulterioară – o prin
cipal scopul de energie din surse regenerabile tehnologia, dar se întâmplă peste tot în jurul nostru.
§9.3 NIVEL TROFIC FOTOSINTEZA
Animale exista prin obținerea de energie și de materiale, direct sau indirect,
din plante. Acest lucru este numit trofice (alimentare) de sistem. Fig. 9.1 este un
extrem de diagramă simplificată a sublinia procesele esențiale ale
ecologia naturala. Ar trebui să ne amintim, totuși, că caseta etichetată
"animale" pot include, de asemenea, fosile umane fuelbased activități de indus
încerca, de transport, de încălzire, etc. Smochine 9.2 și 9.3 da 'closeup punctul de vedere al pho
tosynthesis respectiv la nivelul plantelor și la nivel molecular.
În timpul fotosintezei CO
2
și H
2
O sunt absorbite pentru a forma n
drates, proteine și grăsimi. Generalizate simbol [CH
2
O] este folosit pentru a
indica bloc de construcție de bază pentru aceste produse. CO
2
este eliberat
TWIDELL PAGINARE.indb 328
01/12/2014 11:37
§9.3 nivel Trofic fotosinteza
329
în timpul respirației atât de plante și animale, și de ardere a
materialului biologic. Această explicație simplificată este satisfăcătoare pentru energie
studiile, dar neglijează roluri esențiale de azot, substanțe nutritive și envi
parametrii de mediu în procesele.
Energia absorbită la formarea de biomasă de radiațiile solare
în fotosinteză este egală cu cea emisă de căldură la ardere, deoarece:




←a se


H + CO + 2H O
[CH O] + O + H O
2
2
fotosinteza
de ardere
2
2
2
(9.3)
∆H = 460 KJ pe mol C = 4.8 eV per atom de C
≈ 16 MJ kg
-1
de uscat carbohidrati material
Aici
∆H este entalpia schimbare a procesului de ardere, egală cu
energia absorbită de la fotonii radiației solare în fotosinteză,
mai puțin energia de respirație în timpul creșterii și pierderilor în timpul precursor
reacții (a se vedea §9.4).
∆H poate fi considerat ca fiind căldura de ardere;
valoarea sa exactă depinde dacă sau nu s-a format apa este lichidă sau
de vapori. Rețineți că de ardere necesită temperaturi de ~400°C, în timp ce
respirația încasări de catalitică de enzime reacții la ~20°C. absorbția
de CO
2
de o frunză de plante este o funcție de mai mulți factori, în special tempera
ture, CO
2
concentrația și intensitatea și lungimea de undă distribuții
de lumina.
Fotosinteza poate avea loc prin reducerea CO
2
în reacțiile cu com
de lire sterline, altele decât apa. În general, aceste reacții sunt de forma:

CO + 2H X
[CH O] + X + H O
2
2
2
2
2
(9.4)
Fig. 9.1
Trofice nivel global fotosinteza, necesită, de asemenea, de apă. Fluxuri: energie, 10
14
W; carbon,
10
11
t/an; CO
2

, 4 x 10
11
t/y; oxigen, 3 x 10
11

t/y; apa (ca reactant), 3 x 10


11
t/y. Atmosferice
concentrații: oxigen, 21%; CO
2

, 0.030% din volum preindustrială în 1850 și


creșterea de activitate umană, ajungând la 0.040% în 2014, și încă în creștere, așa cum este indicat
în Fig. 2.19(a) (a se vedea http://co2now.org/ pentru curent de date de pe CO
2
).
Radiația solară
Producători:
net fotosinteza
Consumatori:
net respirație
[CH
2
O]
Plante
Animale
Descompunătoare:
micro-organisme
CO
2
Căldură pentru a
mediu
O
2
TWIDELL PAGINARE.indb 329
01/12/2014 11:37
330
Resursele de biomasă de fotosinteza
De exemplu, X poate fi sulf, S, referitoare la anumite fotosintetice bacte
ria care cresc în absența oxigenului prin astfel de mecanisme, cum a fost
procesul dominant de pe Pământ înainte de a prezenta 'oxygenrich' atmosfera
a fost format. Astfel de reacții utilizați pigmenți, alții decât clorofila, cu
diferite spectre de absorbție.
La eficiența de fotosinteza
η este definit pentru o gamă largă de cir
fiind condițiile. Acesta este raportul de net entalpia câștig de biomasă pe
unitatea de suprafață (H/O) pentru incidentul a energiei solare pe unitatea de suprafață (E/O), în
special biomasă creștere în unele perioade specificate:
η=
HO
E UN
/
/
(9.5)
Aici O pot varia de la suprafața Pământului (inclusiv deserturi)
pentru suprafața de teren de o pădure, în zonă de un câmp de cereale, și expuse sau
total suprafața unei frunze. Perioade variază de la câțiva ani la câteva minute,
și condiții pot fi naturale sau de laborator controlate. Este deosebit de
important cu culturi pentru a determina dacă, citat de creștere se referă la
sezonul de creștere sau un an întreg. Tabelul 9.1 oferă valori de
η pentru dif
ții condiții.
Cantitățile implicate într-un nivel trofic descriere a fotosintezei
poate fi apreciat din următorul exemplu. Sănătos frunze verzi
în lumina soarelui produce aproximativ 3 litri de O
2

per oră per kg de frunze (umed


baza). Acesta este un flux de energie de 16 W, și ar fi obținut de la o
suprafață expusă de frunze de aproximativ 1 m
2
. O persoană metabolizeaza la aproximativ 100 W
(repaus), 200 W (activ). Astfel, fiecare persoană care obține energie metabolică pentru
24 de ore de la reacția cu oxigenul derivat de la 15 la 30 m
2

de
suprafața foliară. Astfel, în regiunile temperate, anual corporale aportul de oxigen de
o persoană este asigurată de aproximativ un copac mare. La tropice
(în cazul în care plantele cresc mai rapid) cu un astfel de copac ar oferi metabolice
energie pentru trei oameni. Industriale, de transport și combustibil casnic
consum necesita mai mult oxigen per persoană (de exemplu, aproximativ 100 de copaci/
persoană în SUA, aproximativ 60 în Europa, și aproximativ 20 în mare parte din
lumea în curs de dezvoltare).
Dominanta elementelor chimice ale biomasei sunt de carbon, oxigen,
hidrogen și azot, toate de care se miște liber în atmosferă ca
elemente stabile de gaze, vapori de apă și nor (CO
2
,O
2
,H
2
O, NU
x

).
Oxigenul este esential pentru naturale și tehnologice a proceselor energetice, și
toate aceste elemente sunt componente esențiale ale vieții structurii. Astfel appre
ciating că în fotosinteză plantele oferă energie și carbonbased
materiale prin ușor de împrăștiat gaze în atmosferă oferă o
perspectivă clară în mecanismele fundamentale de ecologie durabilă.
Pentru aprovizionarea cu energie, de oxigen, care este format în principal la tropice, dis
perses la nivel global, astfel încât să permită animalelor de viață și de ardere pentru a continua chiar și
în regiunile polare.
TWIDELL PAGINARE.indb 330
01/12/2014 11:37
§9.4 Relația dintre fotosinteză și la alte procese de plante
331
§9.4 RELAȚIA DINTRE FOTOSINTEZĂ ȘI LA ALTE
PROCESE DE PLANTE
O parte din energia captată de la soare prin fotosinteză este uti
lized intern în plante pentru procesele metabolice, inclusiv de creștere. În
general procesul este numit respirație, prin care într-o serie complexă de reactia
țiilor de zaharuri și polimeri format prin fotosinteză se combină cu
oxigenul, deci eliberarea de dioxid de carbon, apă și surplusul de energie sub formă de căldură.
Intermediar reacții implică molecule complexe și enzime
(catalizatori). Cu toate acestea, nivelul general de reacție pentru respirație cu enzime
la temperatura ambiantă, este aceeași ca și pentru ardere la ridicate
de temperatură, și reversul pentru fotosinteză (9.3), adică

∆H
[CH O] + O
CO +H O +
2
2
2
2
[respirație]
(9.6)
în cazul în care
∆H este entalpia lansat, fiind efectiv egal cu căldura
lansat în ardere.
Respirația este un proces vital nu numai în plante, ci și în animale. Am
respira oxigen pentru a "arde" alimente conform (9.6), și respira
('respira') dioxid de carbon și apă, așa cum este indicat în Fig. 9.1. Cu toate acestea,
interne detaliate chimie este diferit pentru plante și animale.
O consecință importantă este că nu toată energia captată de pho
tosynthesis este stocată ca biomasă disponibile pentru reacția cu oxigenul pentru
Tabelul 9.1
Aproximativ randament fotosintetic pentru o serie de circumstanțe; datele raportate variază
pe scară largă pentru multe situații diferite
Condiții
Randament fotosintetic
(%): aproximativ ghid
Tot pământul: 1 an mediu (radiații incident sub atmosfera
pe tot pământul și marea)
0.1
Pădure: generală anuală medie
0.5
Pășuni: anual (tropical, medie; temperat, bine gestionate)
1
Întreaga plantă (net fotosinteză)
Recolta de cereale: strâns plantate, agricultura bun, vegetație numai,
temperate sau tropicale culturi
3
Continuarea culturilor: de exemplu, manioc
2
Condiții de laborator: enhanced CO
2
, temperatura și
iluminat optimizat, mare de apă și substanțe nutritive
5
Inițial fotosintetice proces (de exemplu, nu inclusiv respirația plantelor)
Teoretic maxima cu filtrat de lumină, condiții controlate, etc.:
excitonul proces numai
36
cu reacție centre
20
cu carbohidrați formarea
10
TWIDELL PAGINARE.indb 331
01/12/2014 11:37
332
Resursele de biomasă de fotosinteza
bioenergie. Brut producția primară (APE) este rata inițială de la care
plantele captează energia. Mai util pentru biomasă evaluare a resurselor –
și mult mai ușor de măsurat – este producția primară netă (CNE), care este
rata la care plantele stochează energia chimică mai puțin energia utilizată în
propria lor respirație și creștere economică:
CNE
= GPP - energie de respirație (9.7)
De obicei CNE
≈ 0.5 APE, deși raportul variază între plante și
între ecosisteme.
Global terestre CNE pot fi estimate prin combinarea satelit meas
urements de cantitatea de plantă în suspensie (bazat pe spectrale
ale clorofilei), de obicei, la o rezoluție ~0.5
× 0.5 grade,
calibrat cu multe conexe măsurători. Acest randamentele medii
globale CNE
≈ 50 GtC/y (Potter et al. 2012), cu alte estimări variind
de la 35 la 66 GtC/an.
Fig. 9.2 oferă o procedură simplificată schematică prezentare a plantelor fotografie
sinteza (secțiunea superioară etichetate "frunze") și respirația plantelor (mai mică
secțiune intitulată "rădăcini").
§9.5 FOTOSINTEZA LA CELULAR
ȘI NIVEL MOLECULAR
Fig. 9.3(a) și (b) indică cheia procesele moleculare implicate respecta
relativ în lumină și întuneric reacții de fotosinteză, și Caseta 9.1
rezumă componente celulare. Aici vom prezenta principalele caracteristici
prezentate în Fig. 9.3; pentru o descriere mai detaliată a reacțiilor și
frunze de structuri în care au loc, se referă la Twidell și
Weir (2006), cartea asta e eResource S9.1, și să specialitate, manuale
de fotosinteză.
Fig. 9.2
Planta nivel fotosinteza
Schimb chimic
Rădăcinile
Nutrienți
Solar
radiații
Frunze
CO
2
CO
2
O
2
Lumina
reacție
Întuneric
reacție
O
2
H
2
O
H
2
O
TWIDELL PAGINARE.indb 332
01/12/2014 11:37
§9.5 Fotosinteza la nivel celular si molecular
333
§9.5.1 Reacție de ansamblu
În cloroplaste, la lumina reacțiile sunt fizic separate de
întuneric reacții'.
Toate componentele de lumină reacții sunt aranjate în sau pe pro
teins a avut loc în thylakoid membrană. Lightharvesting 'antene' sunt
proteine care conțin pigmenți de clorofilă aranjate pentru a absorbi lumina
și trece energia din apropiere centre de reacție. Plantele au două reactia
tion centre, așa cum este indicat în energylevel diagrama (Fig. 9.3(a)):
fotosistemului 1 (PS1) și fotosistemul 2 (PS2). Mai degrabă confuzie, la
reacțiile inițiale care produc oxigen apar într-PS2, prin care taxa
de separare permite electronii excitați de a trece 'în sus, în nivelul de energie'
Fig. 9.3
Nivel Molecular fotosinteză. Scară verticală indică excitație energie de
electroni. (o) Lumină de reacție, indicând fluxul de energie și de materiale în cele două
interacționează photosystems de plante verzi. (fotosistemului 2, evidențiate în verde) (b) Întuneric
reacție, cu ciclul Calvin folosind agent de reducere produse din lumina reacție
a fotosistemului 1.
(o) LUMINĂ REACȚII
(b) REACȚII de ÎNTUNERIC
Radiația solară
fotonii
Pigment
moleculele
Fotosistemului 1
Donor de electroni D
1
Fotosistemului 2
Acceptor de electroni
O
2
Electronii e

Electronii e

Protonii H
+
NADPH
agent de Reducere
FDAN
NADPH
Agent de reducere
FDAN
ATP
Energie magazin
ADP
e

Fotosistemului 2
Donor de electroni D
2
Mediu
Calvin
ciclul
H
2
O
C
6
H
12
O
6
CO
2
Fixarea dioxidului de Carbon
O
2
Fotosistemului 1
Acceptor de electroni
O
1
Pigment
moleculele
TWIDELL PAGINARE.indb 333
01/12/2014 11:37
334
Resursele de biomasă de fotosinteza
CASETA 9.1 STRUCTURA DE FRUNZE DE PLANTE
În plante și alge, fotosinteza are loc în organite din celulele plantei numite cloroplaste. Un exemplu tipic
de plante celulă conține aproximativ 10 până la 100 cloroplaste (vezi Fig. 9.4). Cloroplastelor este închisă de o membrană.
În membrana este un fluid apos numit stroma. În stroma conține stive de thylakoids,
care sunt site-ul de fotosinteză. La thylakoids sunt aplatizate discuri, delimitată de o membrană. La
site-ul de fotosinteză este thylakoid membrana, care conține integrantă și periferice membrana
de proteine complexe, inclusiv pigmenți care absorb energia luminii, care formează photosystems.
Plantele absorb lumina în primul rând, folosind pigmentul clorofila, care este motivul pentru care cele mai multe plante au o
culoare verde, deși cele mai multe plante, de asemenea, utiliza alți pigmenți-o anumită măsură. Acești pigmenți sunt poziționate
în plante și alge speciale 'antennaproteins la suprafețele de thylakoid membranei (a se vedea Twidell
și Weir (2006) și să coopereze ca o lightharvesting complexe.
Deși toate celulele în părțile verzi ale unei plante au cloroplaste, mare parte din energie este captată în
frunze. Celulele din interiorul țesuturilor de o frunză poate conține între 450.000 de 800.000 cloroplaste
pe mm
2
de frunze.
Fig. 9.4
Structura și scară de frunze de plante.
o
Secțiunea tipic frunze de un broadleafed plante. Fotosintetică activă verde celule sunt prezentate presarata cu
cloroplaste organite. Aproximativ scară numai. Real celule de presă împreună mai strâns decât se arată, de exemplu, nu
au lacune la fel de mare cum este indicat în figura de claritate.
b
Secțiune prin cloroplastului organelle. La thylakoid membrane interne sunt prezentate în lichid stroma. Anumite
regiuni au stivuite thylakoid membrane (grana), care sunt conectate prin unstacked stroma lamele
membranare.
c
Perspectiva stivuite și unstacked thylakoid structura membranei. Stivuite grana sunt legate prin poduri de
stroma lamele, toate în lichid stroma cloroplastelor organelle. Aproximativ scară numai.
Superioară cuticula
Superioară a epidermei
Celulele de palisade
mesophyll
Epiderma inferioară
Mai mici cuticula
Celulele spongioase
mesophyll
CO
2
O
2
garda de celule
Gaze și vapori
schimb prin
stomatele
~0,1 mm
(a)
Structura frunze
= 100 m
µ
Cloroplast organelle
Secțiune laterală
Secțiune largă
10
µ
m
Organite conțin
lamele
(b)
Thylakoid membranele sunt căptușite cu fotosintetică activă suprafete
1 µm
100nm = 0.1
µ
m
Lamele sunt realizate
din thylakoid membrane
(c)
TWIDELL PAGINARE.indb 334
01/12/2014 11:37
§9.5 Fotosinteza la nivel celular si molecular
335
pentru PS1. PS1 este apoi activat de un al doilea foton, iar electronii se
produce sunt leșinat de thylakoid membrană și pe molecule
implicate în întuneric reacții pentru fixarea CO
2
.
Condus de lumină de energie, fotosintetice chimie în thylakoid mem
brana produce ATP (adenozin trifosfat), o moleculare sursă de
energie, împreună cu agentul de reducere NADPH (redus nicotinamid
dinucleotid fosfat). Aceste molecule sunt apoi consumate în
întuneric reacții.
Întuneric de reacții au loc în stroma (numit "întunecat" pentru că ei fac
nu au nevoie de lumină). Aici enzime conduce un ciclic de reacție care convertește CO
2

și un zahăr care conține cinci atomi de carbon în molecule de o threecar


bon de zahăr. O parte din aceste zaharuri este apoi reintrodus în ciclul,
cu restul utilizate ca blocuri pentru a forma glucide, cum ar fi
glucoză, celuloză sau amidon. Enzime (proteine catalizator) responsabil
pentru fixarea carbonului din CO
2

este numit rubisco, care este, probabil, cele mai


abundente de enzime. Întregul ciclu, inclusiv regenerarea rubisco, este
numit ciclul Calvin.
Primul produs de ciclul Calvin este un threecarbon (C
3
) compus din
cele mai multe plante, astfel încât acestea sunt menționate ca C
3

plante. Sigur cea mai mare parte tropicale,


plante (de exemplu, trestie de zahăr, porumb și sorg) au o preliminare chimice
ciclu care implică un C
4
compus înainte de ciclul Calvin. C
4

plantele au
două tipuri diferite de celulele fotosintetice că funcția de cooperare
în plantă. În moderată până la puternică intensitate a luminii (~0,5 kW/m
2
) și
temperaturi ridicate în frunze (~40°C), C reparații, și, prin urmare,
producția de biomasă de C
4
plantele pot fi de două ori că a C
3
plante. C
4

planta miscanthus giganteus (iarba elefantului) este neobișnuită, deoarece este, de asemenea,


crește în zonele cu climă temperată; prin urmare, utilizarea acestuia ca o cultură energetică (de exemplu, în
Europa).
§9.5.2 considerente Termodinamice
Aici avem în vedere fotosinteza ca un aspect al termodinamicii.
Implicațiile sunt importante pentru a ghida strategia pentru energie din surse regenerabile
de cercetare și pentru a da înțelegere de bază.
Un ideal (Carnot) motor termic are o eficiență
η = (T
h
- T
c
)/T
h

(a se vedea Caseta 16.1). Să presupunem alimentare cu energie termică este radiația solară și căldură
chiuveta este la temperatura mediului ambiant 27°C (300 K). Dacă Soarele exterior tem
temperatura ar putea fi folosit ca sursa, maxim randamentul ciclului Carnot
ar fi (5900
- 300)/5900 = 95%, care este semnificativ mai mare decât
la temperaturi în mod regulat dispozitivele de inginerie. Astfel, există mult
interes în încercarea de a link-ul de procese la cea mai mare temperatura disponibile
pentru noi, și anume temperatura Soarelui.
Singura legătură între Pământ și Soare este prin intermediul solare radia
ției, astfel încât o radiație procesului de absorbție este nevoie. Dacă absorbția este pe
negru colector, procesul este temperatura limitat de punctul de topire
TWIDELL PAGINARE.indb 335
01/12/2014 11:37
336
Resursele de biomasă de fotosinteza
de colector material. Cu toate acestea, este posibil pentru a absorbi radia
ției de către un foton proces în stările electronice dintr-un material fără
imediat creșterea cea mai mare parte a temperaturii. Un asemenea proces are loc în
generarea de energie fotovoltaică (Capitolul 5).
Pentru a compara termică și fotonice excitație, Fig. 9.5 reprezintă un
material care poate exista în două electronice state: normale și entuziasmat. La
diferența dintre aceste state este exclusiv electronice diferite con
figurație; nucleul sau 'grilaj' a materialului rămâne neafectată. În
Fig. 9.5(a) stare excitată poate fi atins numai prin încălzirea întregului
material, și proporția de stari excitate N
e
la normal membre N
n
este
calculat ca pentru semiconductori intrinseci:
N
e
/ N
n
= exp(-∆E / kT ) (9.8)
Vom avea în vedere molecule de pigment în cazul în care
∆E ~ 2 eV,
și T < 373 K
= 100°C, deoarece materialul celular este pe baza de apa. Astfel
N
e
/N
n
~10
-27
. Chiar și la temperatura Soarelui de 5900K, N
e
/N
n

= 0.02 numai.
S-a concluzionat că termică excitație nu produce multe entuziasmat
de state!
Cu toate acestea, în Fig. 9.5(b) excitate electronic de stat este format prin elec
tromagnetic absorbției unui foton de energie h
ν ≥ ∆E. Acest proces nu face
nu se adaugă imediat energia din jur 'grilaj', care rămâne
la aceeași temperatură. Populația din starea excitată depinde de
rata de absorbție de fotoni și de cuplare a entuziasmat electronice
membre la 'grilaj'. Populația limita este N
e
= N
n

când radiația
este transformarea număr egal de state și înapoi și elec
tronic temperatura este practic infinit. Această limită nu este destul de atins în
practică, dar teoria explică cum 10
10
mai multe electronic excitat membre
pot fi formate prin electronicstate solar photon absorbție decât de către solare
termice excitație.
Analiza termodinamică nu este completă până când energia s-a
efectuat o funcție. În fotosinteză energia solară este prima trans
format în stări excitate prin absorbție de fotoni și apoi stocate
în produse chimice. Nu există nici o producție de "muncă" în normal
Fig. 9.5
Electron de excitație (a) de căldură și (b) absorbție de fotoni. Scara verticală indică
excitație energie a electronului.
Conducție stare excitată
(a)
(b)
Conducție stare excitată
Normal stat
D
E
kT
D
E
kT
e

e

Normal stat
+
hn
>>
>>
TWIDELL PAGINARE.indb 336
01/12/2014 11:37
§9.5 Fotosinteza la nivel celular si molecular
337
inginerie mecanică sens, dar absorbit foton de energie permite
producerea de material organic și structuri de magazine chimice de
energie.
Modificările chimice care au loc în fotosinteză sunt, în unele moduri
similare de stat de energie modificări în fizica semiconductorilor. În chimie
apar modificări de reducere și de oxidare. La nivel de reducere (R) este
numărul de molecule de oxigen per atom de carbon necesare pentru a transforma
materialul pentru CO
2
și H
2
O. Pentru compusi de carbon de forma generală
C
c
H
h
O
o
reducerea de nivel este:
R
= (c + 0.25h - 0.5o)/c (9.9)
Energia pentru a forma acestor compuși din CO
2
și H
2
O per unitate de reduceri
tion nivelul R este de aproximativ 460 kJ/(mol de carbon).
Relația de reducere a nivelului de energie și nivelul de energie
membre implicate în fotosinteză este prezentată în Fig. 9.6. Fotosinteza este
, în esență, reducerea de CO
2
în prezența H
2
O să carbohidrati
și oxigen. În procesul de:
H
4
2
O
.

hv
2H
2
O
.
+O
.
2
+ 4H
+
+ 4e

(9.10)
patru electroni trebuie să fie eliminate din patru molecule de apă
(Fig. 9.7).
Fig. 9.6
Reducerea nivelului R de compuși de carbon. Entalpia schimba per atom de carbon,
∆H, de
chimice cuplurilor menționate la CO
2
/H
2

O.
Plus de
electronii
2
1
0
R

H
CH
4
–Metan
–Grăsimi
–Proteine
[CH
2
O] –Carbohidrati
CO
2
–Dioxid de Carbon
Entuziasmat
de electroni membre
~10 eV
4.8 eV
0 eV
H
2
O/CO
2
[CH
2
O]/O
2
Patru electroni
transferați
pe C atom
Reducere:
plus de
hidrogen
Oxidare:
plus de
oxigen
Scădere
de electroni
TWIDELL PAGINARE.indb 337
01/12/2014 11:37
338
Resursele de biomasă de fotosinteza
Fig. 9.7
Reducere de apa în oxigen și protoni de la centrul de reacție a fotosistemului 2 ca patru
electroni sunt eliminate (4H
2

O
.

ν
h
2H
2

O
.
+O
.
2

+ 4H
+
+ 4e
-

). Notă: H
+
este un proton.
H
H
H
H
H
H
ee
ee
ee
ee
ee
ee
ee
ee
e
e
e
e
e
e
ee
e
e
e
e
H
H
H
H
H
+
H
Patru protoni format
Patru electroni eliminat
H
+
H
+
H
+
OH

OH

OH

H
2
O
H
2
O
H
2
O
O
2
H
2
O
H
2
O
H
2
O
OH

OH
ee
ee
e

e

e

e

ee
ee
ee
ee
ee
ee
ee
ee
ee
ee
e
e
e
e
e
e
e
e
e
e
e
e
e
e
e
e
e
e
e
e
ee
§
9.5.3 Photophysics
Fizica de fotosinteza implica absorbtia de fotoni de lumină
de electroni în molecule de pigment. Aceste molecule absorb
energie pentru a forma stari excitate. Când moleculele sunt izolate,
energia este, de obicei, neîntrerupt, fiind ca fluorescente radiații și căldură. Cu toate acestea,
atunci când pigmenții sunt legate în cloroplast structuri, majoritatea dintre
energia este transferată în cooperare pentru a centrilor de reacție chimice
de reduceri, cu exces iese sub formă de căldură, și nu există sau există foarte puțin
de fluorescență.
Izolat proprietăți sunt explicate prin FranckCondon diagrama
(vezi Fig. 9.8). Aceasta descrie pământ și entuziasmat de energie membre ale
moleculei în funcție de poziția relativă dintre atomi. Această relativă
poziție este măsurată prin unele coordonată spațială, cum ar fi distanța x
dintre două vecine atomi. Rețineți că minimele în
energie apar la diferite valori de x din cauza modificări moleculare în mărime
sau poziție după excitație. Un foton de radiații, care călătoresc la 3 x 10
8
m/s,
trece la moleculă, de dimensiune ~10
-9
m, în timp de ~10
-18

s. În acest
timp, interacțiunea electromagnetică cu electronice de stat pot să apară,
și foton de energie de ~2 eV este absorbit (A). Cu toate acestea, de vibrație și
de rotație sunt mișcările care au loc în moleculă, cu energie termică
TWIDELL PAGINARE.indb 338
01/12/2014 11:37
§9.5 Fotosinteza la nivel celular si molecular
339
kT~0.03 eV și perioada de ~10
-13

s. Aceste state sunt indicate de orizont


tal linii pe diagrama ca molecula oscilează despre minimul
de energie poziții. Absorbție (A) are loc prea repede pentru molecula de
structură să se adapteze, și atât de excitat de stat este format distanță de
minim. Dacă excitat de electroni este asociat cu un alt electron (ca va
fi probabilă), excitat de stat va fi de așteptat să fie o stare singlet
(spin =
-=0
1
2
1
2
) cu durata de viata de ~10
-8
s.
În acest timp de 10
-8
s, sunt ~10
5

vibrații moleculare și atât de


excitat de stat relaxeaza la minimul de entuziasmat de energie termică
de schimb pentru mediul înconjurător. După aceasta, unul din cele două procese principale
are loc cu eliberarea sau transferul de restul de energie de excitație.
Fie:
1
Molecula este aproape de alte molecule similare, și absorbite
de energie (numit un excitonul) este trecut pe acestea prin transfer rezonant
în legătură cu termice mișcare în timpul 10
-8
s viață. Aceasta este
procesul dominant de molecule de pigment in vivo.
Sau:
2
Dupa ~ 10
-8

s fluorescente de emisie (F) poate să apară ca molecula


revine la sol de stat. Lungimea de undă de fluorescență este mai mare
decât cea absorbită de lumină, așa cum este descris de către Stokes shift. Alternativ,
electronul se poate schimba orientarea în stare excitată, de către magnetic
interacțiunea cu nucleul, pentru a forma un triplet de stat (spin =
1
1
2
1
2
+=
).
Durata de viață a triplet membre este lung (~10
-3
s) și din nou pierderi de energie
apare, de fosforescență sau prin transfer rezonant.
Rezonant transferul poate avea loc între molecule, atunci când acestea sunt aproape
(~5 x 10
-10

m), și atunci când fluorescență de radiații din transferul


moleculei se suprapune cu banda de absorbție de la vecin. În aceste
condiții, excitate electronic de stat de energie (excitonul) poate transfera
Fig. 9.8
FranckCondon diagrama ilustrează Stokes shift în energie între foton absorbit
- O și fluorescente foton F. spațial de coordonate x indică o schimbare în poziția
sau dimensiunea între entuziasmat de sistem și de stat la sol.
Energie
Singlet excitat
Teren de stat
F
O
~2eV
0
0
1
Coordonată spațială
x
~ 10
-10
m
TWIDELL PAGINARE.indb 339
01/12/2014 11:37
340
Resursele de biomasă de fotosinteza
fără radiații de la alta molecula. Separat nivelul de energie diagrame
de forma din Fig. 9.9(a) poate descrie acest lucru, sau, atunci când moleculele sunt foarte
aproape, de un gradat band gap diagramă ca în Fig. 9.9. (b). În orice descriere
este un spațiale transfer de energie pe un gradient de potențial prin
asamblarea de molecule. Procesul este similar cu banda de conducție elec
tron circulație în gradate decalaj celule fotovoltaice (a se vedea §5.6.2). Cu toate acestea,
în fotosinteză, energia este transferată ca întreg molecule ușor
reglați poziția lor și de a structura timpul electronice de excitație și relaxa
tion, și nu doar de transport a unui electron liber.
Există, cu toate acestea, cea mai semnificativă diferență între electroni
de transport în fotovoltaice cu semiconductori și de transport de energie în pigment
molecule. În fotovoltaice structurale material este fabricat cu
a absolvit dopant proprietăți în întreaga celulă. Fiecare element de material
are un scop precis dopant nivel și trebuie să rămână la locul potrivit. Dacă
celula fotovoltaica este rupt în sus, fiecare piesă își păstrează distinctive
caracteristice. Cu toate acestea, în fotosintetice lightharvesting sistem, aceasta
este structura de cooperare de toate pigmenți care oferă fiecărui pigment
necesare structurii electronice necesare pentru locația sa exactă. Acesta
nu contează în cazul în care o moleculă de pigment se află, întotdeauna va fi
dat proprietățile corectă pentru a se potrivi în lightharvesting matrice, potrivit
Fig. 9.9
Transferul de energie de molecule de pigment de lightharvesting sistemul special
de reacție centru.
o
Poziția spațială a lightharvesting molecule de pigment (m, n, o, p) transferul de energie de
la o reacție centrul R.
b
Clasificate band gap model: electronic continuu structura de lightharvesting pigment
molecule care acționează ca un continuu 'super moleculă'.
2eV
2eV
Reacție
centru
Reacția centru
(a)
h
ν
h
ν
Energie
(b)
Energie
m
n
o
p
R
TWIDELL PAGINARE.indb 340
01/12/2014 11:37
§9.5 Fotosinteza la nivel celular si molecular
341
pentru poziția sa. Deci, atunci când "matrice" este rupt în sus, fiecare pigment revine
sale proprietăți izolate. Aceasta reprezintă diferența dintre
in vivo și in vitro proprietăți de molecule de pigment timpul de absorbție
și de fluorescență.
§
9.5.4 Numărul de fotoni pe carbon fix
Cerința principală de absorbție a luminii este că fotoni individuali pot
fi absorbită și energia stocată pentru o perioadă suficientă pentru a fi utilizate mai târziu în
reacții chimice sau alte foton de excitație. Fiecare fotosistemului este trig
gered de absorbția singur solar photon în molecule de clorofilă.
Apoi cuantificat energia trece ca 'excitons', și anume mobil entuziasmat
electronice membre, și așa trece lateral de-a lungul unui lanț de similare excitabil
molecule, moleculele care formează centrul de reacție. Un minim de patru
operațiuni de PS2 sunt necesare pentru a produce o moleculă de O
2
, adică patru
electronii trebuie să fie ridicat de pe H
2
O (vezi Fig. 9.7). Alte patru fotoni sunt
necesare pentru a produce NADPH pentru CO
2

reducere. Astfel, în plante verzi


cu cuplate PS2 si PS1, cel puțin opt fotonii sunt necesare pentru a stabili un
atom de C , ca de carbohidrați. În practică, se pare că mai fotonii sunt
necesare, fie pentru că un sistem eficient chimice saturație sau pierderea se produce,
sau pentru că în continuare ATP este necesar. Astfel, cele mai multe plante, probabil, funcționează la
aproximativ zece fotoni pe C fixe în condiții optime.
§
9.5.5 Eficiența fotosintezei la foton nivel
Minim foton de energie de intrare în antenă în afara pigment mol
ecules (adică nu la centrul de reacție) poate fi dat ca patru fotoni
de 1.77 eV (PS2 absorbție pentru D
2
la 700 nm) și patru de 1,82 eV (PS1
absorbție pentru D
1
la 680 nm), în valoare totală de 14,4 eV. Real excitații D
2
la Un
2
și D
1
la Un
1
sunt aproximativ 1,1 eV fiecare. Astfel, cele patru operații de fiecare
necesită 8.8 eV. Ieșirile pot fi considerate patru electroni ridicat
de la H
2

O să FDAN peste potențialul redox 1.15 eV (4.60 eV), plus trei


molecule de ATP la 0.34 eV fiecare (1.02 eV), pentru a oferi o putere totală de 5,6 eV.
Rezultatul poate fi, de asemenea, considerat ca O
2
molecula, și un atom de C
fix în carbohidrați, care necesită 4.8 eV.
Un rezonabile teoretică maximă eficiența de absorbție a luminii
la produsul final, astfel, pot fi luate ca 4.8/14.4 = 33%. Cu toate acestea, cele
mai mari proporții de 5.6/14.4 (38%), 5.6/8.8 (63%) și 4.8/8.8 (54%) sunt
uneori considerate. Rețineți că aceste teoretice eficiență nu iau
în considerare distribuirea de radiații în spectrul solar sau de plante
respirația; prin urmare, acestea sunt toate mai mari decât practice, valorile obținute
din aplicarea (9.5) pentru culturi în lumina soarelui.
Rețineți că în discutarea foton interacțiuni, unitatea de einstein este
adesea folosit. Unul einstein este numărul lui Avogadro de fotoni de aceeași
frecvență, adică un mol de identice sau similare fotoni.
TWIDELL PAGINARE.indb 341
01/12/2014 11:37
342
Resursele de biomasă de fotosinteza
Generarea de oxigen dintr-o frunză pot fi măsurate ca o funcție
de lungimea de undă a luminii incidente și portretizat ca un spectru de acțiune
(Fig. 9.10), care indică modul în care un greenleafed planta utilizeaza solare radia
ției pentru fotosinteză în mare parte din spectrul vizibil. Con
regim de linie este optice spectrul de absorbție; scăderea între
0.5
mm si 0,6 mm indică mai puțin de absorbție și deci o mai mare reflectare
de lumina verde. Prin comparație cu frunze spectrul de absorbție, de asemenea,
prezentate în Fig. 9.10, vârfurile în acțiune în spectrul roșu (0.7
mm)
și albastru (~0.4
mm) corespund maximele de absorbție ale clorofilei
a și chlorophyllb respectiv. Dip în acțiunea spectrului este
mai sus de zero pentru că (a) alți pigmenți sunt, de asemenea, prezent, și (b) nu
sunt de cooperare interacțiuni care schimba caracteristicile de absorbție ale
fiecărui pigment in vivo de la ceea ce ar fi în mod izolat (in vitro).
Spectrul solar este format din mai multe fotoni cu energie cuantificată
prea mici pentru a fi fotosintetică activă (
l > 700 nm, hν < 1.8 eV),
și fotoni de energie mai mare decât minimul necesar (h
ν > 1.8
eV), cu exces care apare sub formă de căldură. Prin urmare, doar aproximativ 50% din
lumina solară absorbită este folosit pentru a opera PS2 si PS1. Mai mult decât atât, cele mai multe
frunze nu sunt negru, atât de reflexie și de transmisie reduce maximă
eficiență. Situația este foarte asemănătoare cu celulele fotovoltaice
(a se vedea Caseta 5.2, Fig. 5.14 și Fig. R4.11). Tabelul 9.2 dă aproximativă
de date pentru trecerea energiei solare pe și într-o plantă. Dacă raportul dintre
energia stocată în fotosinteză a energiei incidente pe o frunză este definit
ca "eficienței", Tabelul 9.2 descrie tipic pierderi pentru o frunză în moderată
Fig. 9.10
Spectrul de absorbție (curba continuă) și spectrele de acțiune (curba punctată) tipic verde
frunze de plante. d[O
2
]/d
l este distribuția spectrală a ratei de producție de oxigen pe unitatea de
zona pe unitate de iradiere.
Absorbția
Acțiune
100%
d[O
2
]
d
λ
Albastru
Verde
Roșu
Infraroșu
400
Lungimea de undă / nm
500
600
700
800
0%
TWIDELL PAGINARE.indb 342
01/12/2014 11:37
§9.6 Energie agricultură: producția de biomasă pentru energie
343
iluminare, oferindu-eficiența globală de aici ca ~5%. Cu toate acestea, (a)
frunzele sunt de multe ori complet sau partial umbrite, și (b) radiații de răspuns
este neliniară, astfel încât maximă directă a radiației la ~1000 W/m
2

nu poate fi
pe deplin absorbit pentru că reacțiile sunt saturate. Prin urmare, având în vedere
radiației solare pe terenuri, în general, care include mai multe părți, altele decât
frunze, eficiență ~5% nu sunt atinse în condiții naturale, nici în cel mai bun
agricultură (Tabelul 9.1). Astfel, având în vedere importanța vitală de alimentare
de aprovizionare, securitatea energetică și dezvoltarea durabilă în general, consi
important de cercetare și dezvoltare (R&D) este dedicat îmbunătățirii
eficienței fotosintezei, așa cum este descris mai jos și în §9.7.
§9.6 ENERGIE AGRICULTURĂ: PRODUCȚIA DE BIOMASĂ
PENTRU ENERGIE
§9.6.1 Energie agricultura
Vom folosi termenul de 'energie agricole' în sens larg, pentru a desemna
producția de combustibili sau de energie imediat ca principale sau subsidiare produs din
agricultură (domenii), silvicultura (păduri), acvacultură (proaspete și apă de mare),
și, de asemenea, industriale sau în activități sociale care produc deșeuri organice resi
taxe (de exemplu, prelucrarea produselor alimentare, urban refuza). Tabelul 10.1 oferă câteva exemple
din vasta gamă de posibilități. Scopul poate fi de a produce
numai energie (ca și cu o mulțime de lemn pentru combustibil lemn), dar de obicei este mai bine
pentru a integra producția de energie cu cultură sau alte tipuri de biomasă material
Tabelul 9.2
Pierderile de energie în fiecare etapă a fotosintezei
Procesul
De energie
rămase
după acest
proces
Energie
pierdere în acest
proces
Eficiența
factor
Note
Lumina soarelui incident
pe o frunză
100%
Foton de energie
nepotrivire
53%
47%
0.53
Numai fotoni în intervalul 400-700 nm poate fi
absorbit (Fig. 9.10); acestea sunt de 53% a
energiei în spectrul solar (Fig 2.15 și 5.13)
Incomplete
absorbția
37%
16%
0.70
Fotonii dor de cloroplaste (poate lovi
alte componente)
Foton de energie
degradarea
28%
9%
0.76
Shortwavelength (de energie mai mare) inband
fotonii degradat la nivelul de energie de 700 nm
fotoni ca acestea sunt absorbite
Conversie chimică
pentru a dglucose
9%
19%
0.32
Conversia de la ATP și NADPH pentru
dglucose
Respirație, etc.
5.4%
3.6%
0.60
Planta folosește imediat unele dintre
"capturate" de energie
Pe baza datelor din Hol și Rao (1999).
TWIDELL PAGINARE.indb 343
01/12/2014 11:37
344
Resursele de biomasă de fotosinteza
produse. O remarcabilă și a stabilit exemplu de energie agricultura este
trestie de zahăr industriei (a se vedea Caseta 9.2 și Smochine 9.11 și 9.12).
CASETA 9.2 TRESTIE DE ZAHĂR: UN EXEMPLU DE ENERGIE AGRICULTURA
Fig. 9.11
Trestie de zahăr agroindustry: diagramă a fluxului de proces. Trestie de zahăr este de fibre vegetale reziduuri;
melasa este sugarrich reziduuri.
Trestia de zahar
ferme
Transport
de tăiat trestie de zahăr
Moara cu pasiune
trestie de zahăr
Trestie de zahăr
Cazane
Zahăr
Melasa
Fermentația alcoolică
Hrana vitelor
Rafinarea zahărului
Căldura de proces
De energie electrică
Produse din fibra
Alte produse
Alte produse
Suc
La flowdiagram prezentată în Fig. 9.11 indică modul în care o singură cultură (trestie de zahăr) pot fi prelucrate pentru
ambele aprovizionarea cu energie și o gamă largă de produse fără alte intrări decât cele cultivate pe plan local trestie de zahăr.
La tulpini de trestie de zahăr, aproximativ 3 m lungime cu 5 cm diametru, sunt recoltate și apoi transportate în vrac, fie
prin camioane sau pe o lumină de cale ferată pus pe câmpurile din jur, la o centrală moara. Aici, steampowered
role zdrobi trestia pentru a extrage sucul, ca principal produs inițial. Sucurile trece în principal pentru zahăr
extracție, cu reziduu (melasa): (a) utilizate direct pentru hrana vitelor; (b) fermentat pe site-ul de etanol
pentru sparkignition vehicul de biocombustibili, și (c) utilizate în produsele farmaceutice și alte produse chimice de
specialitate. Bastonul e
reziduuri fibroase de role (trestie de zahăr) este ars în cazane a ridica aburi pentru a genera energie electrică și
furnizarea de energie termică pentru moara proceselor (în special la fierbere sucul pentru a extrage zahăr solid). Surplusul de
trestie de zahăr este
presat cu liant de a face mdf pentru constructii. Cazan de cenușă devine un fosfat
bogat îngrășământ. Modern, cu echipamente eficiente ar trebui să fie excesul de energie electrică pentru vânzarea
la utilitatea rețelei de distribuție (Caseta 10.3).
Energia agricultura are avantaje și dezavantaje (Tabelul 9.3). Un important
dezavantaj este că, culturi energetice pot substitut pentru hrana omului de sarcini
tion. De exemplu, NE cereale ferme cresc aproximativ 40% din producția mondială de porumb
(porumb), vegetale și în mod tradițional de export de aproximativ o treime din acest timp de alimente, astfel
încât
devierea porumb porumb pentru producția de etanol ca NE petroliere aditiv
cu nici un substitut acțiune se reduce cu alimente la nivel mondial a resurselor. Un al doilea mare
dezavantaj este faptul că totalitatea atât de intens alimente și biocombustibil de sarcini
tion în agricultura intensivă poate conduce la sol infertilitate și eroziune. Strategii
pentru a evita astfel de dezavantaje în ceea ce privește culturile energetice includ: (a) utilizarea
mai eficientă a energiei; (b) cresc plante care pot furniza atât umane
TWIDELL PAGINARE.indb 344
01/12/2014 11:37
§9.6 Energie agricultură: producția de biomasă pentru energie
345
Fig. 9.12
O fabrică de zahăr înființat pentru a produce zahăr, etanol, iar surplusul de energie electrică.
Aceasta fotografie prezinta Costa Pinto moara din Brazilia (statul Sao Paulo). În prim-plan este
operația de recepționare a recolta de trestie de zahăr, cu moara imediat adiacente (la stânga).
În dreptul background este asociat de distilare instalației pentru producția de etanol.
Fotografie de Mariordo, reprodus aici, sub Creative Commons AttributionShare Alike 3.0 Unported
License.
Tabelul 9.3
Avantajele și pericolele de energie agricultura
Avantaje
Pericole și dificultăți
Mare potențial de aprovizionare
Poate duce la sol infertilitate și eroziune
Varietate de culturi
Poate concura cu producția de alimente
Varietate de utilizări (inclusiv transport de combustibil și
generarea de energie electrică)
Utilizarea eficientă a produse secundare, reziduuri, deșeuri
Voluminoase biomasă material handicapuri de transport pentru
fabrica de procesare
Link-ul cu înființată agricultură și silvicultură
Poate încuraja inginerie genetică de
necontrolat organisme
Încurajează agricultura integrată practică
Stabilește agroindustry care pot include plin
gama de procedee tehnice, cu nevoia de calificare
și personal instruit
Îmbunătățire a mediului prin utilizarea de deșeuri
Emisiile poluante provenite de la slab controlat
procese
integrate pe Deplin și sisteme eficiente nevoie
au pic de apă și a poluării aerului (de exemplu, sulf
conținut scăzut)
Proiectat prost și incomplet integrat
sistemele pot polua apa si aer
Încurajează dezvoltarea rurală
Scară largă agroindustry poate fi din punct de vedere
social perturbator
Diversifică economia cu privire la produs,
locație și de abilitatea angajatului
mai Mare potențial este în țările tropicale,
frecvent țărilor în curs de dezvoltare
Capitalul străin nu poate fi în simpatie cu locale
sau beneficiul național
TWIDELL PAGINARE.indb 345
01/12/2014 11:37
346
Resursele de biomasă de fotosinteza
alimente (de exemplu, cereale) și energie din deșeuri produse (de exemplu paie); (c) să
nu ardă reziduuri de biomasă în domeniu, și (d) scăderea hrana omului
culturi alimentare pentru animale.
O altă problemă conexă este: dacă scopul de a produce biocombustibili lichizi este
de descreștere a consumului de combustibili fosili, pentru motive de 'energie
de securitate' (§17.2.1) sau de a reduce emisiile de gaze de seră (§17.4), nu
producția de biocombustibil necesita mai mult combustibil fosil decât biocombustibili
ar deplasa? Această problemă este abordată în mod empiric în Cutii 10.3 și
10.4.
§
9.6.2 Lemn de resurse
Lemnul este o resursa energetică sustenabilă numai dacă este crescut la fel de repede ca
acesta este consumat. Mai mult decât atât, există necesităților ecologice pentru
conservarea păduri naturale și a pădurilor. Lumea e din lemn
de resurse se consumă nu doar pentru lemne de foc, dar pentru cherestea,
hârtie și alte utilizări industriale. În plus, multă pădure este
eliminat pentru terenuri agricole și nu "recoltate", cu lemn ars
ca "deșeuri". Statisticile FAO estimează că în lume recolta de lemn
ca un utilizată resursă este de aproximativ 3700 de milioane de m
3

de lemn pe an, din


care aproximativ 45% este necomercial pentru combustibil și alte 6% este
folosit direct ca combustibil comercial (statisticile FAO pentru anul 2012). (Non
comerciale cifră este supusă incertitudine, și este probabil
o subestimare.)
În multe țări, consumul de lemn de foc depășește înlocuire
creștere, astfel încât lemn de foc este o diminuare a resurselor. Lemn de foc de colectare pentru
consumul casnic, de obicei, o sarcină pentru femei și copii, este
de a deveni mai greoaie ca lemn de foc devine tot mai puține. Pro
parte a femeilor din mediul rural afectate de lemn de foc deficitul este de aproximativ 60% în
Africa, 80% în Asia, și 40% în America latină. Mai mult decât atât, colectarea de foc
lemn poate necesita una la cinci ore pe zi. Atenuarea acestor dificultăți
necesită atât intensiv de reîmpădurire și un comutator cu mai eficiente și
alternative metode de gătit (a se vedea §10.3.1 și Fig. 10.8).
Regenerarea poate avea loc în păduri naturale sau provocate de om în planta
țiilor (care, de obicei, cresc mai repede și sunt preferențiale pentru biocombustibili). De
exemplu: (i) din 2009, Brazilia a crescut de 5 milioane de hectare de a susține
capabil de plantații de eucalipt folosit pentru fabricarea de oțel de cărbune
și pentru uscare și prelucrare de soia;
1
(ii) Indian multe gospodării sunt
în creștere private mici plantații pentru propriul lor consum de combustibil.
2

Plantațiile
cultivate în mod special pentru furnizarea de energie au nevoie de diferite management (sil
viculture) tehnici de plantații cultivate în principal pentru lemn (Sims,
2002). Combustibile din lemn nu trebuie să fie în bucăți mari, și, prin urmare, pot
fi recoltate de la trei la cinci ani, mai degrabă decât de ~30 de ani, deci sporirea
ing productivitate. Metoda tradițională de tăierea copacilor (de exemplu, lăsând
rădăcinile în pământ și periodic decuparea numai abovesurface
TWIDELL PAGINARE.indb 346
01/12/2014 11:37
§9.6 Energie agricultură: producția de biomasă pentru energie
347
ramuri) este de succes, cu multe specii de arbori; se reduce de muncă pentru
plantare și plivitul puieți, și, de asemenea, reduce eroziunea solului, comparativ
cu repetate de replantare.
§9.6.3 producției și de îmbunătățire a
Estimarea randamentului culturilor necesită cunoștințe detaliate de meteorologi
cal condiții, tipul de sol, practici agricole, utilizarea de îngrășăminte, irigare, etc.;
mai mult decât atât, de vreme neașteptate condiții de multe ori a contracara astfel de predic
ări. Comparație între diferite culturi și în diferite locuri este difi
cult din cauza diferențelor în perioadele de creștere și metodele de recoltare.
Unele culturi arabile sunt plantate anual (de exemplu, cereale), și pot
fi recoltate de mai multe ori (de exemplu, ierburi). Alții sunt plantate la fiecare
câțiva ani și recoltată anual (de exemplu, trestie de zahăr). Arborilor poate crește de
mai mulți ani și să fie recoltată în totalitate (exploatarea lemnului); alte culturi copac
poate crește de la continuarea rădăcini și de a fi recoltate ca crâng fiecare
câțiva ani (de exemplu, salcie, alun și unele eucalipt). Tabelul 9.4 estimările
maxime de biocombustibil potențial al diferitelor culturi în termeni de căldură de combus
tion și continuarea furnizării de energie. Datele pentru culturi acvatice presupune
abundente substanțe nutritive. Ierburi sunt presupuse a avea frecvente de recoltare în
sezonul de creștere.
Ca energia din biomasă devine mai important, plantele sunt selectate
și dezvoltate pentru a optimiza aprovizionarea cu combustibil, mai degrabă decât doar fructe, cereale
sau similare parte produsul. De exemplu, propagarea de clone de cele mai bune
plante și aplicarea ingineriei genetice a crescut foto
sintetică a eficienței pentru producerea de biomasă.
§9.6.4 Cât de mult biomasă este disponibil pentru energie?
Caseta 9.3 prezintă tehnici pentru a evalua cât de mult biomasă este poten
mod disponibil pentru utilizare de energie. În astfel de evaluări, este esențial
să se concentreze pe cât de mult pot fi luate în mod durabil în fiecare an (de exemplu, cu
regenerarea compensatoare pentru care a folosit) fără a aduce atingere asupra culturilor
și terenurilor necesare pentru hrană și fără a provoca inacceptabil ecologi
daune cal. Deșeuri de biomasă (de exemplu, pădure, articole de pasmanterie, cocos, coji,
resturi de lemn, deșeuri de carton, canalizare, etc.) ar trebui să fie o prioritate
a resurselor pentru ardere de energie. Cu toate acestea, astfel de resurse sunt adesea
dificil de cuantificat și este esențial să plece cantități semnificative de
putrezire biomasă pentru sustenabilitatea ecologică a microflorei și microfaunei,
și pentru structura solului. Prin urmare, multe de estimare a resurselor, inclusiv
Tabelul 9.5, de multe ori includ numai biomasă de noi plantații. Tabelul 9.5
(și rapoartele pe care se bazează) sugerează că cel mai mare potențial
pentru energie, agricultura de biomasă poate apare în țările tropicale, espe
cial cele cu ploi adecvat și starea solului.
TWIDELL PAGINARE.indb 347
01/12/2014 11:37
348
Resursele de biomasă de fotosinteza
Tabelul 9.4
Practice maxime de biomasă randamentelor. Masă totală de plante, nu doar cereale; " R " indică
masa este coppiced, cu rădăcinile rămase în sol. Datele sunt dintr-o varietate de surse și
rezumate de către autori. Precizia de nu mai mult de ±25% este susținut. Majoritatea plantelor și
culturilor randament mult mai puțin decât aceste maxima, cu randamente mult depind de sol, climă, îngrășăminte și
practica agricolă.
Randamentul în biomasa
Cultură
(Presupune o recolta pe an,
daca nu este indicat altfel)
(t ha
-1
y
-1
)
Bază umedă Uscată
Densitatea de energie
(MJ (kg uscat)
-1
)
Energie de uscat randament
(GJ ha
-1
y
-1
)

Naturale
Pășuni
7
3
Pădure, temperat
C3
14
7
18
130
Pădurea tropicală
C3
22
11
18
200
Furaje
Sorg (3 culturi)
R, C4
200
50
17
600
Iarba-de-Sudan (6 culturi)
R, C4
160
40
15
600
Lucerna
C3
40
25
De secara, temperat
C3
30
20
Napier iarba
C4
120
80
Alimente
Cassava (60% tuberculi)
50
25
43
(b)
Porumb (porumb) (35% cereale)
C4
30
25
18
77
(b)
Grâu (35% cereale)
C3
30
22
50
(b)
Orez (60% cereale)
C3
20
Sfeclă de zahăr
C3
45
150
(b)
Trestie de zahăr
R, C4
100
30
18
150
(b)
Boabe de soia
C3
26
20
(c)
Semințe de rapiță (canola)
C3
60
(b)
Plantație
Ulei de palmier
R, C3
50
40
170
(c)
Energia de ardere
Eucalipt
R, C3
55
20
19
380
Sycamore
R, C3
20
10
19
190
Populus
R, C3
18
29
19
380
Salcie (salix)
R, C3
25
15
19
140
(b)
Miscanthus ("iarbă")
R, C4
21
18
18
330
(b)
Zambila de apa
C3
300
36
19
680
Varec (macroalge)
C3
250
54
21
1100
Alge (microalge)
C3
300
45
23
4000
Copac exudate
Bună ieșire
1
1
40
40
Note:
o
C3, C4: fotosinteza de tip (a se vedea §9.5.1). R: recoltate de mai sus rădăcină (coppiced).
b
Ca etanol.
c
Ca biodiesel.
TWIDELL PAGINARE.indb 348
01/12/2014 11:37
§9.6 Energie agricultură: producția de biomasă pentru energie
349
CASETA 9.3 CUM ESTE BIOMASA, RESURSE EVALUATE?
o
De jos în sus
Pentru a evalua resursa pentru un proiect de dezvoltare locală (de exemplu, introducerea de biogaz, §10.7 sau îmbunătățite
sobe de gătit, §10.3.1) în Biomasa de Evaluare a Manualului de RosilleCalle et al. oferă o multitudine de
sfaturi practice și studii de caz, inclusiv cum să se măsoare lemn in situ. Principii-cheie includ
următoarele:

Ia în considerare atât oferta, cât și a consumului, precum și de bază nevoile energetice', care poate fi mai mult sau mai puțin
decât consumul de curent.

Energia din biomasă ar trebui să fie considerat, alături de alte tipuri de biomasă beneficii (de exemplu, produse din lemn cu
deșeuri și resturi de lemn de foc).

Opțiuni de date, utilizatorii sunt mai în măsură să judece ce este bine pentru ei.
Păstrați ipoteze explicite (de exemplu, date medii nu se pot aplica la nivel local).
b
De sus în jos
Estimări ale biomasei, a resurselor la nivel mondial, continental sau la nivel național sunt, de obicei, pe baza existente,
a datelor statistice sau pe teledetecție, sau un amestec al acestora.
Estimarea pe baza datelor statistice
Estimarea se face, în primul rând, pentru fiecare tip de biomasă din diferite surse de date (naționale sau colectate de către ONU
pentru alimentație și Agricultură (FAO)).

Producția agricolă de culturi cheie (t/y) și un multiplu de reziduuri (tulpini, etc.). Există, de asemenea, un
multiplicator pentru a găsi deșeuri de la producția de animale (balegar, seu, etc.) care ar putea fi utilizate pentru bioenergie.

În mod similar pentru producția forestieră.

Deșeurilor urbane statistică (DSM (t/y), dintre care unele sunt combustibile, ape uzate industriale (m
3
/y), unele
de care poate produce biogaz, etc.) sunt colectate de către alte agenții.

Culturile plantate special pentru energie: potențial poate fi estimată din randamentelor medii și a terenurilor
"disponibile". (Cât de mult teren este considerat a fi "disponibile" depinde de cât de mult analistul crede că va
fi necesar, sau în producția de alimente sau a serviciilor ecosistemice: a se vedea §9.8.)
Biomasa potențial disponibile pentru energie este total de toate cele de mai sus; este o "potențialul tehnic"
(Tabelul 9.5 și §1.5.4).
Estimări bazate pe teledetecție
Fundamentul acestei abordări este estimarea biomasei totale și CNE menționate în §9.4. Acest lucru este
de 'potențial teoretic'. Apoi, în fiecare zonă geografică, pentru o mai realist "potențialul tehnic",
estimările sunt obținute în funcție de tipul de resursă și apoi aceste componente sunt rezumate, de exemplu:

Biomasa recoltată (t/y) și, prin urmare, reziduurile disponibile pentru bioenergie.

Biomasa unharvested și neprotejat (adică nu în parcuri naționale, etc.); în principiu, acest lucru este de asemenea
disponibil pentru bioenergie.

Zona de "marginal" de teren (ha) având potrivite climatice și de sol pentru culturi energetice; înmulțit cu o
valoare estimată a producției (t/(ha y)) și a producției de energie (GJ/t) acest lucru vă oferă o a treia componentă de potențial
tehnic pentru
bioenergie.
Surse: RosilloCalle et al. (2007); Long et al. (2013).
TWIDELL PAGINARE.indb 349
01/12/2014 11:37
350
Resursele de biomasă de fotosinteza
Estimările de bioenergie a resurselor disponibile la nivel global în mai
termen (de exemplu, la 2050) variază foarte mult (de la ~50 ~1000 BQ/y), deoarece acestea sunt
foarte dependente de presupuneri despre viitorul populației, cantitatea și
tipul de alimente oamenii vor cere (de exemplu, proporția de carne), îmbunătățirea
productivității în agricultură, t/(ha.y), cererea pentru nonenergy foloseste de
lemn, și de alți factori (Chum et al. 2011).
§9.7 R&D A 'ÎMBUNĂTĂȚI FOTOSINTEZA
Tehnologia continuu avansuri de la studii fundamentale în știință.
Același proces va urmări o eventuală înțelegere deplină de fotografie
sinteza în numeroasele sale variate detalii. Această secțiune examinează unele de energie
legate de aplicații, atât actuale și potențiale.
§9.7.1 fiziologie a Plantelor și biomasă
Ca energia din biomasă devine mai importantă (a se vedea Capitolul 10), plante
sunt selectate și dezvoltate pentru a optimiza aprovizionarea cu combustibil, mai degrabă decât
doar fructe, cereale sau similare parte produsul.
De exemplu, un număr considerabil de cercetări se referă la funcționarea
în enzimei Rubisco, cu scopul de a în cele din urmă 'proiectarea' o formă de
Rubisco, care permite crescut de carboxilare în detrimentul de
reacții secundare care acum apar în mod natural, în special de oxigenare. Când
Tabelul 9.5
O estimare de potențialul tehnic al bioenergiei disponibile de noi plantații pe terenuri
disponibile și adecvate pentru speciile de plante selectate. "Disponibile" teren exclude teren în prezent sub
pădure, în prezent utilizate pentru pășunat sau pentru culturi alimentare, și a zonelor protejate (parcuri naționale, etc.). Culturile
considerate sunt selectate de specii ierboase și lemnoase (miscanthus, iarba bărboasă, canary grass, plop,
salcie, și eucalipt).
Regiune
Total iarbă și
zonă împădurită
(milioane de km
2
)
Potențialul de
bioenergie zona
(milioane de km
2
)
Randamentul mediu
(TJ/km
2
/ y)
Potențialul tehnic
(EJ / y)
America De Nord
6.6
1.1
16.5
19
Europa (inc. Rusia)
9.0
1.2
14
17
Pacific OCDE
5.1
1.0
17.5
17
Africa (subSahara)
10.7
2.7
25
69
Orientul Mijlociu + Africa
1.1
0.01
12.5
0.2
Sud + Est Asia
5.5
0.14
28.5
4
America Latină
7.6
1.6
28
45

Lume (total)
46
7.8
22
171
Adaptat de la Chum et al. (2011), Tabelul 2.3; baza de date de la Fischer et al. (2009).
TWIDELL PAGINARE.indb 350
01/12/2014 11:37
§9.8 aspectele Sociale și de mediu
351
fotosinteza are loc într-o atmosferă cu mărirea concentrației
de CO
2

, "dorit" carbohydrateforming reacție este promovat în


detrimentul "indezirabile" reacție de partea de Rubisco cu oxigen. O
metodă pentru acest lucru are tancuri de alge în solarii prin care gazele de ardere
(care este de ~10% CO
2
si ~ 90% N
2
) trece de la o stație de alimentare.
§9.7.2 Bioengineered fotosinteza
Termenul 'realizat' se referă la sistemele în care unele dintre principalele
componente naturale de fotosinteză sunt în mod artificial asamblate într
'proiectat' sisteme care au ca scop eliminarea caracteristicile care pot limita
biomasă productivitatea (de exemplu, capacitatea plantelor de a se reproduce).
Câteva exemple de bioengineered fotosinteza sistemele sub activ
de investigare sunt revizuite de către Blankenship et al. (2011).
§9.7.3 fotosinteza Artificială
Termenul de fotosinteza artificială este folosit pentru a descrie procesele în care
laboratorul de materiale sunt folosite pentru a capta energie de lumină și de a produce un
magazin chimice de energie. Astfel de procese sunt inspirate de fotografii naturale,
de sinteză, dar, spre deosebire de cele de §9.7.2, nu folosesc aceleași componente
ca natura face. În special, aceasta se referă la producerea de hidrogen
din apa de lightinduced reacții redox Naturale fotosinteza
utilizează clorofila pentru lumina de antenă și hydrogenase enzime pentru
hidrogen reacție, dar curent de R&D se concentrează pe utilizarea de metal
oxid semiconductor și metalbased catalizatori pentru aceste acțiuni
(Jones 2012).
§9.8 ASPECTELE SOCIALE ȘI DE MEDIU
§9.8.1 Bioenergie în ceea ce privește agricultura și silvicultura
Utilizarea și producția de biomasă pentru energie sunt intim conectate cu
mai larg de politici și practici pentru agricultură și silvicultură. Un imperativ
considerare este faptul că o astfel de utilizare și de producție ar trebui să fie din punct de vedere ecologic
durabilă, și anume că resursa fi utilizate într-o manieră regenerabile, cu
(re)de creștere a ține pasul cu utilizarea. Mai mult decât atât, din motive etice, este
vital ca producția de biomasă pentru energie nu este în detrimentul de creștere
suficientă hrană pentru a hrăni oamenii.
Cu toate acestea, în Uniunea Europeană și statele UNITE ale americii, o problemă majoră în
agricultură este supraproducție de alimente, ca și încurajat de către agricole
subvenții financiare. Astfel de subvenții creștere a fiscalității pe salariați
și, în consecință, a surplusurilor de produse agricole denatura comerțul mondial
în dezavantajul țărilor în curs de dezvoltare. Ca un răspuns parțial la
aceste probleme, Uniunea Europeană a introdus stimulente financiare pentru
TWIDELL PAGINARE.indb 351
01/12/2014 11:37
352
Resursele de biomasă de fotosinteza
agricultorii săi pentru a devia de teren din producția de alimente, și fie să-l mențină
neproductiv sau pentru biomasă pentru energie. Astfel de politici și păstreze sociale
beneficiile unui punct de vedere economic activă a populației agricole, în timp ce, de asemenea,
aduce beneficii de mediu, descrise mai jos, de substituire
a biocombustibililor pentru combustibilii fosili.
Utilizarea deșeurilor de biomasă crește productivitatea din agricultură și
silvicultură. Acest lucru este valabil mai ales pentru acceptabile dispoziția de altfel
nedorite ieșiri (de exemplu, biodigestion de gunoi de grajd de la intensiv pigger
e), astfel încât sistemul integrat aduce atât economice, cât și de mediu
beneficii. După cum sa subliniat în §9.6 și §10.1, succes biocombustibil de sarcini
tion utilizeaza deja s-a concentrat fluxurilor de biomasă, cum ar fi resturi
și rumeguș de la tăierea, paiele de culturi, gunoi de grajd de scris
animale și apele uzate din lucrări municipale. Biocombustibil procese care
depind în primul rând transportul și apoi concentrarea difuze biomasă,
resurse sunt mai puțin de dorit.
Evoluțiile de energie utilizand culturi locale și stabilite competențele sunt
cele mai susceptibile de a fi acceptabil social. Astfel formă de biomasă cele mai
susceptibile de a fi viabil ca o sursă de energie va varia de la regiune la regiune.
Mai mult decât atât, ca în orice cultură, agricultură și silvicultură durabile sunt
necesare; de exemplu, extinse monoculturile sunt vulnerabile la boli
și dăunători și neprietenos pentru speciile de faună. Rețineți, de asemenea, că gaze cu efect de seră
beneficiile apar doar atunci când biomasa este utilizat pentru a înlocui utilizarea de combustibili fosili,
deci, lăsând redus de combustibil fosil în subteran.
§9.8.2 Alimentare față de combustibil
Producția de biocombustibili lichizi s-a bazat din punct de vedere istoric pe bază de biomasă de la
cereale, zahăr și ulei de culturi, toate din care sunt esențiale culturi alimentare, în general,
cultivate pe cele mai bune terenuri agricole disponibile. În ciuda producției vegetale sur
plusuri în statele UNITE ale americii și Europa, creșterea cererii la nivel mondial pentru hrană
implică faptul că aceste culturi nu ar trebui să fie deviat semnificativ de la alimente la
energie excepția cazului în care producția vegetală devine suficientă în nevoiași țări.
Prin urmare, producția de biocombustibili ca o contribuție majoră la lume energie
consumabile necesită alte materii prime și pământ decât pentru mâncare și alte obiecte
egies. De exemplu, există o nevoie pentru o mai ieftin, mai eficient energetic
procese pentru producerea de etanol din disponibile pe scară largă lignocellusosic
materiale (de exemplu, coceni de porumb, paie și lemn), mai ales rumeguș și
alte reziduuri lemnoase, mai degrabă decât de foodrelated culturi.
§9.8.3 gaze cu efect de Seră impactul: bioenergie și a absorbanților de carbon
Când o plantă crește, de carbon este extras din aer CO
2
este
absorbit în fotosinteză, astfel încât să devină închis în carbohidrați
material atât deasupra și sub pământ. Cantități semnificative de CO
2

sunt
eliberate în metabolismul plantelor, dar fluxul net de carbon este în plantă.
TWIDELL PAGINARE.indb 352
01/12/2014 11:37
§9.8 aspectele Sociale și de mediu
353
Carbon concentrații în sol poate crește, de asemenea, 'indirect' de
materie organică format din plante detritus în frunzele căzute și ramuri.
Astfel de eliminarea de gaze cu efect de seră CO
2

de atmosferă se numește
o 'carbon'. În consecință, un program dedicat de creștere a plantelor
creștere va compensa temporar o creștere în atmosferică CO
2

din
arderea combustibililor fosili. Cu toate acestea, toate plantele mor și marea majoritate a tuturor
directe și indirecte de carbon în cele din urmă se întoarce în atmosferă, astfel încât
aderarea la un ciclu natural care nici nu epuizeaza nici nu crește atmosferice de
CO
2
concentrații.
Numai dacă materialul vegetal este ars pentru a înlocui (abate) specifice utilizarea de
combustibili fosili va fi un beneficiu pe termen lung de prevenire care fosili
carbon de altfel ajungă în Atmosferă. Rezultă că astfel
s-au redus de carbon fosil ar trebui să rămână mereu sub pământ și nu
fi extrase. În rapoartele naționale elaborate pentru Cadru a organizației națiunilor unite
Convenția privind schimbările Climatice, acest lucru s-au redus de combustibil fosil arată ca o reduceri
t în CO
2
emisiile de combustibili fosili.
§9.8.4 Bioenergie în ceea ce privește sistemul energetic
Biomasa este, în prezent, o mare parte a lumii a sistemului energetic, deși
în principal în formă de ineficient folosite lemne de foc în zonele rurale, în special
în cazul în care este gătit peste un foc deschis. Un sistem energetic mai sustenabil
pentru lumea va trebui neapărat să implice în acest distribuite pe scară largă și
versatil de resurse, dar folosit în mai eficiente și mai moderne modalități,
cum s-a discutat în Capitolul 10. De exemplu, în 160 de energie scenarii
analizate de către SRREN (2011), a celor cu contribuții semnificative la renewa
ble energie, jumătate au bioenergie contribuind cel puțin 125 EJ/an la nivel mondial total de
aprovizionare cu energie primară (TPE), până în 2050, adică de cel puțin 25% din curent TPE
(a se vedea §17.8 pentru o discuție generală a scenariilor energetice). Într-Adevăr, Prietene
et al. (2011) estimează că potențialul tehnic al biomasei pentru producerea de energie
poate fi la fel de mare ca 500 EJ/an până în 2050. Cu toate acestea, o astfel de producție
de bioenergie necesită durabilitate și cadre de politici care să asigure
o bună guvernare de utilizare a terenurilor și îmbunătățiri în silvicultură, agricultură
și creșterea animalelor de management, și în bioenergie asociate tehnologii.
§9.8.5 impactul Uman asupra producția primară netă (NPP)
Cartografiere de la sateliți și de pe teren arată că 35% din cea a Pământului
icefree suprafață de teren este lanuri (~10%) și pășunat, pentru a trăi
stoc (~25%); împreună, aceste face, probabil, cel mai mare ecosistem de pe
planeta, de potrivire de pădure acoperă în măsură. Producția de carne de conturi
pentru ~40% din globale agricole comerciale de ieșire din țară industrializată
încearcă și echivalentul impact în alte țări (Steinfeld et al. 2006).
Înregistrate și gestionate pădurile adauga la impact. Umane alocate
productivitatea primară netă (HANPP) este proporția globală biologice
TWIDELL PAGINARE.indb 353
01/12/2014 11:37
354
Resursele de biomasă de fotosinteza
productivitatea, care este folosit, a reușit, sau cooptat de acțiuni umane; aceasta
este estimată a fi de 20 ± 6% din globală CNE (a se vedea de exemplu Imhoff et al. 2004).
Concluzia este că, prin compensare ecosistemele naturale sau prin intensificarea
ing practici existente lanuri, care pasc pe pășuni și păduri, prezenta
omului landuse activități sunt consumatoare un everlarger social al
planetei productivitatea biologică și modificarea în mod dramatic Pământului eco
sisteme în procesul de. Este important să realizăm că toți oamenii ar trebui să
fie vegetarian și toate de creștere a culturilor de eficient, atunci de utilizare a terenurilor ar fi mult
mai puțin. Acum, cu toate acestea, există mari regiuni ale lumii unde HANPP
este între 60% și 100% din totalul CNE. Oamenii de azi deja recolta de
peste 8 Pg C/y pentru propria lor imediată și mâncare pentru animale. Această biomasă
ridică la aproximativ putere calorifică brută de
≈300 exajouli (EJ)
pe an, din care ~50 BQ/y sunt utilizate pentru furnizarea de servicii energetice.
Totalul este de așteptat să crească în următoarele decenii, cu o suplimentare de
recoltare de 4-7 Pg C/y, care ar fi aproape dublu față de prezent biomasa
de recoltare și de a genera substanțiale presiune suplimentară asupra ecosistemelor
(Haberl et al. 2007).
Având în vedere amploarea acestor efecte, este clar că, la fel ca verde
casa de seră și schimbările climatice, societatea umană cu prezenta dieta,
stilul de viață, economiile și aspirațiile se apropie de o fundamentale envi
mediu limită privind durabilitatea acestuia. Cât de mult mai mult de biosfera
productivitatea putem corespunzătoare înainte de sisteme planetare începe să se rupă
REZUMAT CAPITOL
Toate biologică și economică de viață de pe Pământ depinde de fotosinteză ca procesul prin care trăiesc
plante (o) face propriile lor material structural (biomasa) din principalele intrări de dioxid de carbon și apă,
și (b) se produce oxigen, după cum este necesar pentru viața animală și de ardere, în general. Biomasa și
oxigen împreună deveni chimică magazine de energie solară. Acest lucru implică o serie de complexe fizice
și biochimice reacții, dintre care cele mai multe au loc în frunzele unei plante. Prima etapă (foton
de absorbție, în principal, de pigmenții clorofilieni) are analogii cu celule fotovoltaice, care pot genera
, dar nu stoca energie electrică. Aproximativ jumătate din energia captată de plante de la lumina soarelui este utilizat pentru
plante
propriul metabolism. Aproximativ 3% de expunere la soare pe plante este stocat ca biomasă, chiar și pentru o wellcultivated
culturilor
în timpul sezonului de creștere. Astfel dominante efect imediat de soare este să încălzească Pământul, cu
energie din biomasă în cele din urmă transformându-se în utilizare sau degradare la căldură, de asemenea,. Cu toate acestea, la
nivel mondial primară netă
de producție (NPP, am.e. energia stocată de plante terestre ca biomasă) este de aproximativ trei ori curentul
total comercială a consumului de energie al omenirii.
Aproximativ 20% din globală CNE este folosit, a reușit, sau cooptat de acțiunile umane, deși "doar" 2%
din globală CNE este în prezent folosit pentru energie. Menținerea animalelor pentru producția de carne are un impact major.
Astfel, există puternice de mediu și de constrângerile sociale pe biomasă, resurse disponibile pentru energie
scopuri, inclusiv pentru a acorda prioritate la alimente, furaje și fibre produse ca populația globală crește,
și pentru menținerea mediului natural. Dacă biomasa recrește la o rată cel puțin la fel de rapid ca acesta este folosit,
atunci efectul net pe CO
2
concentrația în atmosferă este zero.
Există un potențial considerabil pentru energia agricultura, în special prin utilizarea de energie din agricultură și
reziduuri forestiere și prin noi plantatii de altfel pe terenuri marginale. Productivitate sporită
TWIDELL PAGINARE.indb 354
01/12/2014 11:37
Probleme
355
jos: 30%? 40%? 50%? Poate că avem deja în necunoștință de trecut
acest prag.
ÎNTREBĂRI RAPIDE
Notă: Răspunsurile la aceste întrebări sunt în textul de la secțiunea relevantă
din prezentul capitol, sau poate fi ușor dedusă din aceasta.
1
Cum este energia radiației solare stocate?
2
Ce aproximativ este căldura de ardere a plantelor de biomasă?
3
De ce este biomasa căldura de combustie mai mică decât cea de, să zicem, gaze naturale
(metan)?
4
Care este numărul minim de solară absorbită de fotoni necesar pentru a
produce o molecula de oxigen?
5
Cum este absorbită energia solară canalizate pentru reacții chimice în
plante?
6
Ce sunt 'photosystems' și ce fac ei?
7
Ce este eficiența fotosinteza plantelor și care sunt impli
cații de fotosinteza plantelor fiind de zece ori mai mare?
8
Defini energia agricultura.
9
Cât de multe produse puteți identifica dintr-o trestie de zahăr 'mill/fabrica?
10
Descrie impactul uman de alimente și consumul de energie pe
suprafața Pământului ecosistem.
11
Cum este un atom de carbon în biomasă diferite, în vigoare de la un carbon
atom din combustibili fosili?
PROBLEME
9.1
Conform (9.3), fotosinteza magazine 460 kJ pe mol C. Utilizați
această pentru a calcula cât de mult de energie este stocat pe an, de la global
terestre producția primară netă. Cât de mult este acest lucru în Wați (J/s)?
9.2
Calcula foarte aproximativ câți copaci sunt necesare pentru a
produce oxigen folosit pentru (i) propriul metabolism, și (ii) să
mențină pe cap de locuitor consumul total de combustibil al țării dumneavoastră.
Compara acest lucru cu numărul aproximativ de copaci pe persoană în
țara dumneavoastră.
prin îmbunătățirea practicilor agricole și de ameliorare a plantelor și de selecție poate adăuga, de asemenea, la potențialul de
bioenergie de resurse.
Fotosinteza naturală a inspirat cercetare în două direcții care ar putea duce la noi din surse regenerabile
de energie tehnologii: (i) 'bioengineered fotosinteză, în care unele dintre principalele componente naturale
de fotosinteză sunt în mod artificial asamblate într 'proiectat' sisteme menite să elimine unele
limite de eficiență de fotosinteza naturală, și (ii) 'fotosinteza artificială', procese în care anorganice
materiale sunt folosite pentru a capta energie de lumină și de a produce un magazin chimice de energie.
TWIDELL PAGINARE.indb 355
01/12/2014 11:37
356
Resursele de biomasă de fotosinteza
NOTE
1
Rechargenews, 7 August 2009.
2
'India lemne de foc criză reexaminat', Resurse pentru Viitor (2006).
BIBLIOGRAFIE
Fotosinteza: Universitare-nivelul de cărți și recenzii
Archer, M. și Barber, J. (eds) (2005) Molecular la nivel Mondial Fotosinteza, Imperial College Press, Londra. Parte
dintr-o serie pe photoconversion de energie solară. A se vedea în special editorilor introducere (nivel similar cu această carte)
și capitolul de A. Holzworth pe de absorbție a Luminii și recoltarea'.
Cogdell, R. (2013) 'Instant Expert #30: Fotosinteza', New Scientist, supliment la problema de 2 februarie.
Edenhofer, O., PichsMadruga R., Sokona, Y., Seyboth, K., Matschoss, P., Kadner, S., Zwickel, T., Eickemeier, P.,
Hansen, G., Schlömer, S. și von Stechow, C. (editori) (2011) IPCC Raport Special privind Sursele de Energie Regenerabile
și Atenuarea efectelor schimbărilor Climatice. Cambridge University Press, Cambridge. Capitolul 2 se ocupă în special cu u. p.
b.
energia; Capitolul 1 este o prezentare generală a surselor regenerabile de energie.
Hall, D. O. și Rao, K. K. (1999, 6 edn) Fotosinteza, Edward Arnold, London. Un scurt și stimularea intro
ducerea cu mai mult fizic prejudecată decât multe altele.
Lawlor, D. W. (2001, 3rd edn) Fotosinteza: Molecular, fiziologice și procesele de mediu, BIOS
Publicații Științifice, Oxford. Text concis pentru biologie studenti.
Monteith, J. și Unsworth, K. (1997, 2nd edn) Principii de Fizica Mediului, Edward Amold, Londra.
Consideră interacțiunea fizică dintre viața plantelor și animalelor cu mediul. Chimice aspecte nu sunt
acoperite. De fundal relevanță pentru fotosinteza.
Morton, O. (2007) Consumul de Soare: Cum de plante de putere de pe planeta, a Patra, Londra. Popularlevel cont,
inclusiv personaj plin de viață, schițe de cheie de oamenii de știință în domeniu.
Wrigglesworth J. (1997) de Energie și de Viață, Taylor & Francis, London. Biochimia metabolismului și fotografie
de sinteză, prezentată în mod clar.
Biomasei, a resurselor
Chum, H., Faaij, A. Moreira, J., Berndes, G., Cea mai, P., Dong, H., Gabrielle, B., Goss Eng, A., Layng, W.,
Mapako, M., Masera Cerutti, O., McIntyre, T., Minowa, T. și Pingoud, K. (2011) 'Bioenergie', Capitolul 2
IPCC Raport Special privind Sursele de Energie Regenerabile și Atenuarea efectelor schimbărilor Climatice, ed. O. Edenhofer,
R. Pichs
Madruga, Y. Sokona, K. Seyboth, P. Matschoss, S. Kadner, T. Zwickel, P. Eickemeier, G. Hansen, S. Schlömer
și C. von Stechow, Cambridge University Press, Cambridge. Recent autoritate revizuire a biomasei,
a resurselor și o gamă completă de bioenergie tehnologii (disponibil online la http://srren.ipccwg3.de/report).
Statisticile FAO (2012) Extras din faostat3.fao.org (statistici site-ul web al națiunilor unite pentru alimentație și Agricultură a
Organizației).
Conține oficial statistici privind producția de culturi, inclusiv randamentele pentru cele mai multe țări și regiuni pentru fiecare an,
începând din
1961; de asemenea, similar pentru statistica forestieră, inclusiv estimările de resurse – toate în mod liber downloadable.
9.3
Căldura de combustie de zaharoză C
12
H
22
O
11

este 5646 kJ/mol.


Calcula folosind constanta lui Avogadro, energia per atom de
carbon în unități de eV.
TWIDELL PAGINARE.indb 356
01/12/2014 11:37
Bibliografie
357
Lung, H., Li, X. Wang, H. și Jia, J. (2013) 'resurse de Biomasă și bioenergie potențial de estimare: o
revizuire', Energia Regenerabilă și Durabilă Recenzii, 26, 344-352. Colecție utilă de intervalul de estimări
pentru diferite regiuni.
RosilloCalle, F., de Groot, P., Hemstock, S. și Pădure, J. (2007) Biomasa de Evaluare a Manualului:
Bioenergie pentru un mediu durabil, Earthscan, Londra. Metode practice pentru evaluarea locale de biomasă
de resurse. Nu include orice scară largă de estimare a resurselor.
Sims, R. E. (2002) Strălucirea de Bioenergie în Afaceri și în Practică, James & James, Londra. Iluminare
text cu accent asupra producției industriale moderne și aplicații; conține numeroase ilustrate cu studii de caz
de sisteme de putere, inclusiv cu biogaz.
Referințe specifice
Blankenship, R. et al. (2011) 'Compararea fotosintetice și fotovoltaice eficiență și recunoașterea poten
țială pentru îmbunătățirea', Știință, 332, 805-809. Include un excelent rezumat al posibilelor direcții pentru îmbunătățirea
natural de fotosinteză și proiectat pentru fotosinteza.
Fischer, G., Hizsnyik, E., Prieler, S., Shah, M. și van Velthuizen, H. (2009) Biocombustibilii și Securitatea Alimentară,
Institutul Internațional pentru Analiza Sistemelor Aplicate, Viena (disponibil la www.iiasa.ac.at).
Haberl, H. et al. (2007) 'Cuantificarea și cartografiere umane credit de producție primară netă în Pământ
ecosistemelor terestre', Proceedings of the National Academy of Science (SUA), 104, 12492-12947.
Imhoff, M., Bounoua, L., Ricketts, T., Loucks, C., Harriss, R. și Lawrence, W. (2004) 'modele Globale de net
producția primară', Natura, 429, 870-874. Estimările pe care oamenii sunt utilizarea ~25% din globală primară netă
de producție.
Jones, N. (2012) 'Noi frunze: promisiunea de fotosinteza artificială', New Scientist, 16 aprilie.
Potter, C., Klooster, S. și Genovese, V. (2012) 'producția primară Netă a ecosistemelor terestre din 2000 până în
2009', schimbările Climatice, 115, 365-378.
Steinfeld, H., Gerber, P., Wassenaar, T., Castel, V., Rosales, M. și de Haan, C. (2006) efectivele de Animale Umbra Lungă a lui:
problemele de Mediu și opțiuni. Efectivele de animale, mediu și dezvoltare, FAO, Roma. Un studiu detaliat al
impactului de animale efectivele de animale asupra mediului, în special informații privind impactul schimbărilor climatice.
Twidell, J. W. și Weir, A. D. (2006, 2nd edn) Resurse Energetice Regenerabile, Taylor & Francis, Oxon. Capitolul 10
a doua ediție conține mult mai multe detalii cu privire la fizica si chimie de fotosinteză la molecu
- lar și plante nivel decât ne-am da aici, în această a treia ediție. Acest capitol este reprodus pe eResource de
publisher-ul pentru această carte, a se vedea S9.1.
Vitousek, P. Ehrlich, P. Ehrlich, A. și Matson, P. (1986) 'Uman credit de produse de photosynthe
sis', Bioscience, 36, 368-373. Pionierat analiză, care a inspirat mult de lucru ulterioare.
Reviste și site-uri web
Fotosinteza este un extrem de domeniu activ de cercetare științifică, de cercetare (>1000 de lucrări pe
an) a raportat în reviste științifice de specialitate, cum ar fi
Fotosinteza Cercetare, evaluare Anuală a Fiziologia Plantelor și Biologie Moleculară, Natura, și
Fotochimie și Photobiology.
TWIDELL PAGINARE.indb 357
01/12/2014 11:37
358
Resursele de biomasă de fotosinteza
Mare parte din acest lucru este apoi distilat în graduatelevel monografii.
Multe dintre cele mai interesant, în general, rezultate de cercetare, inclusiv a celor privind artificiale sau proiectat photosynthe
sis, sunt prezentate pe site-ul (www.sciencedaily.com), care este atât de liber și de căutat.
http://greet.es.anl.gov/main GREET model (gaze cu efect de Seră, Reglementate Emisiile și consumul de Energie în
Transport), dezvoltate și actualizate în permanență de Argonne National Laboratory (SUA), permite cercetarea
ers și analiști pentru a evalua diferite de vehicule de combustibil și combinații pe un plin de combustibil pe ciclu/vehicul baza
ciclului. La
modelul este disponibil gratuit pentru download, și se bazează pe o bază de date extinsă de NOI, agricole și inginerie
practică.
TWIDELL PAGINARE.indb 358
01/12/2014 11:37

Bioenergie tehnologii
CONȚINUTUL
Obiective de studiu
360
§10.1 Introducere
361
§10.2 Biocombustibil clasificare
364
§10.2.1 Fundal
365
§10.2.2 Termochimic de căldură
367
§10.2.3 Biochimice
368
§10.2.4 Agrochimice
369
§10.3 arderea Directă pentru căldură
369
§10.3.1 Interne de gătit
370
§10.3.2 Spațiului și încălzire a apei
373
§10.3.3 uscarea Culturilor
373
§10.3.4 Proces-energie electrică și termică
374
§10.4 Piroliză (distilare distructivă)
374
§10.4.1 Solid carbune (masă randament de 25
la 35% maxim)
377
§10.4.2 Calcinare
377
§10.4.3 Lichide (vapori condensat,
masa randament de ~30% maxim) 377
§10.4.4 Gaze (masă randament de ~80%
maximă în biocombustibili)
377
§10.5 Alte procese termochimice 378
§10.5.1 Hidrogen reducere
378
§10.5.2 Hidrogenare cu CO
și abur
378
§10.5.3 Acid și hidroliză enzimatică
378
§10.5.4 Metanol lichid combustibil
379
§10.5.5 Hidrotermale de lichefiere: HTL 379
§10.6 fermentație Alcoolică
379
§10.6.1 Alcool metode de producție
379
§10.6.2 Etanol combustibil utilizat
382
§10.6.3 producția de Etanol din culturi 384
§10.7 digestie Anaerobă pentru biogaz
387
§Introducerea 10.7.1
387
§10.7.2 procesele de Bază și
energetica
390
§10.7.3 dimensionarea Digestorului
391
§10.7.4 Lucru digestoare
393
§10.8 Deșeuri și reziduuri
395
§10.9 Biodiesel din uleiuri vegetale
și alge
396
§10.9.1 Prime uleiuri vegetale
396
§10.9.2 Biodiesel (esteri)
397
§10.9.3 Microalge ca sursă
de biocombustibil
398
§10.10 aspectele Sociale și de mediu 398
§10.10.1 costuri Interne și externe
de biocombustibili pentru transport
398
§10.10.2 Alte efecte chimice
de biocarburanți și biomasă
ardere
399
§10.10.3 Viitor globale bioenergie
400
Rezumat capitol
401
Întrebări rapide
402
Probleme
402
Note
404
Bibliografie
405
Caseta 10.1 calorifice Brute și nete
366
Caseta 10.2 Etanol din Brazilia
383
Caseta 10.3 Bio/fosili în balanța energetică
de biocombustibili lichizi
385
Caseta 10.4 gaze cu efect de Seră (GES) echilibru
de biocombustibili lichizi
387
CAPITOLUL

10
TWIDELL PAGINARE.indb 359
01/12/2014 11:37

www.shahrsazionline.com
360
Bioenergie tehnologii
OBIECTIVE DE STUDIU

Apreciez principiile generale pentru
sus
durabilă utilizarea biomasei pentru energie
scopuri.

Identifica principalele bioenergie procese și
produse și pentru a înțelege științific
principiile care stau la baza fiecăreia dintre ele.
LISTA DE FIGURI
10.1 Naturale și a reușit sistemele de biomasă.
362
10.2 Creștere în producția mondială de unele moderne de biocombustibili.
364
10.3 procesele de producere a Biocombustibililor.
367
10.4 Îmbunătățirea eficienței sobe de gătit.
372
10.5 Piroliză sisteme: (a) la scară mică unitate de piroliză; (b) tradiționale de cărbune în cuptor.
375
10.6 Etanol procesele de producție.
380
10.7 Gama de raportat emisiile de gaze de seră pe unitate de energie de ieșire de la modern biocombustibili.
387
10.8 Biogaz digestoare.
393
LISTA DE TABELE
10.1 aprovizionarea cu Biomasă și de conversie: câteva exemple.
365
10.2 Piroliză randamentele de lemn uscat.
376
10.3 Aproximativă randamentele de etanol de diverse culturi.
384
10.4 Bio/fosili în balanța energetică a producției de etanol din diferite substraturi de cultură.
386
10.5 Tipic gunoi de grajd de ieșire de la animalele de fermă.
392
TWIDELL PAGINARE.indb 360
01/12/2014 11:37
§10.1 Introducere
361
§10.1 INTRODUCERE
Materialul de plante și animale, inclusiv deșeuri și reziduuri,
este numit de biomasă. Este organic, material pe bază de carbon, care reacționează cu
oxigenul din ardere si naturale procese metabolice de eliberare de căldură.
O astfel de căldură, mai ales dacă la temperaturi >400°C, poate fi folosit pentru a
genera lucru mecanic și energie electrică. Inițial materialul poate fi transformat prin
procese chimice și biologice pentru a produce biocombustibili, și anume biomasa
prelucrate într-o formă mai convenabilă, în special combustibili lichizi pentru transport.
Exemple de biocombustibili includ gaz metan, etanol lichid, metil esteri,
uleiuri și solid carbune. Termenul de bioenergie este uneori folosit pentru a acoperi
biomasă și biocombustibili împreună.
Energia inițială a biomasei sistemul de oxigen este capturat de
radiații solare în fotosinteză, așa cum este descris în Capitolul 9. Atunci când
a lansat în combustie, energie generată de biocarburant este disipată, dar
elemente de material ar trebui să fie disponibile pentru reciclare naturală
sau ecologică procesele agricole, așa cum este descris în Capitolul 1 și
Fig. 10.1. Astfel, utilizarea industriale, biocombustibili, atunci când sunt legate cu grijă pentru a
ecologice naturale cicluri, poate fi non-poluante și durabilă. Astfel de
sisteme sunt numite agro-industrii (§9.6), dintre care cele mai cunoscute
sunt cele de trestie de zahăr și produse forestiere industrii; cu toate acestea, există
creșterea exemple de produse comerciale pentru energie și materiale
realizate din culturi ca un mijloc de diversificare și integrare a
agriculturii.
Substanța uscată masa de material biologic de ciclism în biosferă
este de aproximativ 250 × 10
9
t/y încorporează aproximativ 100 × 10
9
t/an de carbon. La
de energie asociate capturat în fotosinteză este de 2 × 10
21
J/y (= 0.7 ×
10
14

W). Din aceasta, aproximativ 0,5% din greutate este de biomasă ca și culturile umane
alimente. Producția de biomasă variază în funcție de condițiile locale, și este de aproximativ de două ori
la fel de mare pe unitatea de suprafață pe teren decât la mare. Cadrul global de resurse de
biomasă este analizat în Capitolul 9, inclusiv aparentă concurență de
alimente și biocombustibili.
Biomasa este sursa primară de aproximativ 10% (50 BQ/y) a omenirii,
de consumul de energie, care este similar cu utilizarea la nivel mondial de combustibili fosili gaz. Folosește în
proporții aproximative includ (IPCC 2011): (i) ~70% ca cea mai mare parte
necomerciale "tradiționale" de combustibil-lemn pentru uz casnic pentru gătit și încălzire,
predominant în țările în curs de dezvoltare, dar, de asemenea, inclusiv cantități semnificative
în zonele rurale din economiile mature; (ii) ~10% în calitate de combustibil pentru energie electrică
generație, inclusiv combinate de căldură și electricitate – PCCE'; (iii) ~10% pentru
non-procesul intern de căldură, și (iv) ~10% pentru biocombustibili componentă de
combustibil al vehiculului, care este în creștere rapidă atât în valori absolute și procentuale în
termeni. Unele țări sunt notabile pentru lor de utilizare a bioenergiei, inclusiv
Brazilia (31%), Suedia (23%) și Austria (18%).
Dacă biomasa este considerată regenerabilă, creșterea economică trebuie să țină cel puțin
ritmul cu utilizarea. Este dureros pentru ecologiei locale și globale de control al climei
TWIDELL PAGINARE.indb 361
01/12/2014 11:37
362
Bioenergie tehnologii
ca lemn de foc de consum, și mai ales comerciale compensare pădure
cu ardere, sunt, depășind în mod semnificativ creștere copac în creștere zone
ale lumii.
De carbon în biomasa este obținută prin fotosinteză de la CO
2

în
atmosferă. Atunci când biomasa este arsă, digerate sau se descompune în mod natural,
emisiile de CO
2

din biomasă în sine este reciclat în această atmosferă.


În ecosisteme stabile, biomasă crește rata la care
se descompune;
1

în consecință, energia obținută din biomasă este în sine de carbon


neutru. Cu toate acestea, combustibili utilizate astăzi în utilaje agricole și forestiere
și în producția de îngrășăminte sunt în principal combustibili fosili, care
nu sunt ei înșiși "carbon neutru". Prin urmare, bioenergie, dacă obținute
cu sau nesemnificative cantități de combustibili fosili, contrastează cu energie din
combustibili fosili din care extra CO
2

se adaugă la atmosfera Pământului.


Astfel, folosind surse regenerabile de bioenergie în loc de combustibili fosili este o importantă
componentă de mediu pentru politicile pe termen lung pentru reducerea emisiilor de
gaze de seră (IPCC 2011).
De stocare a energiei de energie solară, biomasă și biocombustibili este de
o importanță fundamentală. Toate procesele descrise în acest
capitol au scopul de a produce convenabil și la prețuri accesibile combustibili
pentru o gamă completă de utilizatori finali, inclusiv combustibil lichid pentru transport. La
energia termică disponibilă în arderea biocombustibililor (echivalentul în
practică la entalpie sau net densitate de energie) variază de la aproximativ 8
MJ/kg (neuscate "verde" de lemn) și 15 MJ/kg (lemn uscat), la aproximativ 40
MJ/kg (grăsimi și uleiuri) și 56 MJ/kg pentru metan (vezi Tabelul B. 6,
Anexa B pentru detalii). Tabelul 10.1 liste de exemple de biomasă de aprovizionare și
de conversie.
Succesul de biomasă, de sisteme este reglementată de principiile care sunt
de multe ori nu a apreciat:
Radiația solară
Fotosinteza
Energia din biomasă
magazin
CO
2
nutrienți humus
Naturale
Energia
de presă
Interne și
industriale
biocombustibili
Fig. 10.1
Naturale și a reușit sistemele de biomasă.
TWIDELL PAGINARE.indb 362
01/12/2014 11:37
§10.1 Introducere
363
1
Fiecare biomasă activitate produce o gama larga de produse si
servicii. De exemplu, în cazul în care zahărul este făcut din trestie de zahăr, multe
produse comerciale pot fi obținute de altfel deșeuri de melasă
și fibre. Dacă fibra este ars, atunci orice exces de căldură de proces poate fi
folosit pentru a genera electricitate. Spălări și cenușă pot fi returnate la
sol ca îngrășământ.
2
Unele de mare valoare livrările de energie necesită o cantitate mai mare de
lowvalue de energie pentru producerea lor (de exemplu energie electrică din biomasă termică
putere, etanol din amidon culturi, metan de la animal pastă). Astfel
aparenta ineficiență este justificată, mai ales dacă procesul de energie
este de altfel deșeuri de material (de exemplu, paie, cultură fibre, pădure,
articole de pasmanterie, animal pastă).
3
Întregul beneficiu economic agro-industrii este probabil să fie larg răspândită
și încă dificil de a evalua. Unul dintre mai multe posibile beneficii este o
creștere locală fluxul de numerar de comerț și ocuparea forței de muncă.
4
Producția de biocombustibili este probabil să fie cel mai economic dacă
procesul de producție utilizeaza materiale concentrat deja, probabil, ca un
subprodus, și deci, disponibile la un cost redus sau ca venituri suplimentare pentru
tratarea și eliminarea deșeurilor. Astfel, nu trebuie să fie o sursă de
biomasă deja trece pe langa propuse locul de producție,
doar ca hidro-electrice depinde de un flux natural de apă deja
concentrată de o captare. Exemple sunt deșeuri provenite de la animale
incinte, șpanuri și resturi de la fabricile de cherestea, de canalizare municipal,
pleava și cojile de nuci de cocos, și paie de cereale. Este
extrem de important să se identifice și să cuantifice aceste fluxuri de biomasă
într-un local sau național economiei înainte de a preciza probabil biomasă
evoluții. Dacă concentrat biomasă există deja de
stabilit anterior de sisteme, atunci costul de biomasă de creștere și/sau
de colectare este de multe ori prea mare și prea complex pentru beneficii economice.
Scurt-rotație de culturi pot fi cultivate în principal pentru producerea de energie, ca
parte a agriculturii intensive; cu toate acestea, în răspândită
practica de a subvențiilor agricole este dificil să se evalueze fundamentale
cost-eficacitate.
5
Negative și nejustificate efectele extinse de combustibil din biomasă
producția pe scară largă includ defrișarea pădurilor, eroziunea solului și
deplasarea vital culturi alimentare cu combustibil culturi.
6
Biocombustibilii sunt materiale organice, astfel încât există întotdeauna alternativa de a
utiliza aceste materiale ca materie primă în industria chimică sau materiale structurale.
De exemplu, uleiul de palmier este o componentă importantă de săpunuri; multe
plastice și produse farmaceutice sunt fabricate din produse naturale;
și de mult construirea de bord este fabricat din fibre vegetale construite ca
materiale compozite.
7
Slab controlat de procesare a biomasei sau ardere cu siguranță poate
produce nedorit de poluare, mai ales de la un nivel relativ scăzut de temperatura
de ardere, umed combustibili și lipsa de oxigen pentru ardere
TWIDELL PAGINARE.indb 363
01/12/2014 11:37
364
Bioenergie tehnologii
regiuni. Moderne pe bază de biomasă procese necesită multă îngrijire și
expertiză.
8
Utilizarea durabilă bioenergie și alte surse regenerabile de energie în loc de combustibili fosili
combustibili scădea emisiile de combustibili fosili dioxid de carbon și deci reduce
forțându de schimbările climatice. Recunoscând acest lucru este un aspect cheie al climei
schimba politicile.
Încadrarea sistematică a biocombustibililor urmează în §10.2, și
secțiunile ulterioare luați în considerare tipurile specifice. Secțiunea finală rezumă
sociale, economice și de mediu pentru bioenergie
să contribuie în mod pozitiv și nu negativ la dezvoltarea durabilă.
Creșterea rapidă a producției mondiale moderne de biocombustibili este indicat în
Fig. 10.2.
§10.2 BIOCOMBUSTIBIL CLASIFICARE
Fig. 10.3 este o energie și materiale organigrama care explică
detalii complexe de biocombustibil procese. Începe stânga sus, cu energie solară
și fotosinteza de biomasă culturi și reziduuri, care ne
urmați pe pagina a trei clase principale de energie generată de biocarburant
procese: termochimice, biochimice și agrochimice. Fiecare dintre
aceste clase are nume proceselor secundare și biocombustibil produse
care în cele din urmă reacționează cu oxigenul pentru a elibera căldură la ardere. Notă
că pe măsură ce trece de la stânga la dreapta pe diagramă, inițial amestecat
conținutul de biomasă solidă este procesat în anumite solizi, lichizi și gazoși
combustibili.
0
20
40
60
80
100
120
2000
(a)
2005
2010
2015
An
De producție (GL)
Bioetanol
Biodiesel
0
10
20
30
(b)
2000
2005
2010
2015
An
Pelete din lemn (Mt)
Fig. 10.2
Creșterea în producția mondială de unele moderne de biocombustibili:
o
bioetanol (curba superioară) și biodiesel (inferioară curbă), și
b
pelete din lemn.
Sursă: Datele pentru 2011 de la REN21 (2012).
TWIDELL PAGINARE.indb 364
01/12/2014 11:37
§10.2 Biocombustibil clasificare
365
Tabelul 10.1
Aprovizionarea cu biomasă și de conversie: câteva exemple
Biomasa-sursă sau combustibil
Biocombustibil
produs
Conversie
tehnologie
Aprox.
de conversie
a eficienței
%
Energia necesară
în conversie:
(n) necesar
(o) opțională
Aprox. gama
de energie din
biocombustibili
MJ
De exploatare a pădurilor
fie
Lemn de Ardere
70
Uscare (o)
16-20/ (kg
lemn),
Lemn, din lemn
moara de reziduuri
Lemn de combustibil mult
decuparea
Lemn de Ardere
70
Uscare (o)
16-20/(kg
lemn)
sau
Gaz
Petrol
Char
Piroliza
# 40/(kg gaz)
85
Uscare (o)
40/ (kg ulei)
20/(kg char)
Culturile de cereale
Paie
De ardere
70
Uscare (o)
14-16/(kg uscat
paie)
trestie de Zahăr presat
suc
Etanol
Fermentarea
80
De căldură (n)
3-6 /(kg proaspete
trestie
de zahăr)
trestie de Zahăr presat
reziduuri
Trestie de zahăr
De ardere
65
Uscare (o)
5-8 /(kg proaspete
trestie de zahăr)
Trestie de zahăr total




8-14/ (kg proaspete
trestie
de zahăr)
deșeuri Animale
(tropicale)
Biogaz
Anaerobe
digestia
50

4-8/ (kg uscat
de
intrare)
deșeuri Animale
(temperat)
Biogaz
Anaerobă
digestie
50
De căldură (o)
*2-4 / (kg uscat
de intrare)
Canalizare gaz
Biogaz
Anaerobă
digestie
50

2-4/ (kg uscat
de
intrare)
gaz de fermentare a deșeurilor (de la
MSW
+
)
Biogaz
Anaerobă
digestie
40

2-4/ (kg uscat
compostabile)
Urban refuza (MSW)
+
(De căldură)
De ardere
50

5-16 /(kg uscat
de intrare)
Note
# de Azot eliminat.
* Această valoare este net, având deduse biogaz hrănite înapoi la căldură cazan.
+

Deșeuri municipale solide.


§10.2.1 Fundal
Biomasa este compusă în mare parte din material organic și apă. Cu toate acestea,
cantități semnificative de sol, shell sau alte străine material poate fi
amestecat cu biomasa recoltată, care este evaluată în funcție să fie
umed sau uscat-contează masă, împreună cu conținutul de umiditate.
Dacă m este masa totală a materialului inițial și m
0
este atunci când masa
complet uscate, conținutul de umiditate este:
w
= (m − m
0
)/m
0
[uscată]
(10.1)
w'
= (m − m
0
)/m [- o bază umedă]
TWIDELL PAGINARE.indb 365
01/12/2014 11:37
366
Bioenergie tehnologii
Umezeala este în formă de ambele extracelulare și intracelulare de apă,
care trebuie să fie eliminat în mare parte din recolta inițială și pentru conservarea
prin uscare (vezi §4.3). Atunci când sunt recoltate, umed baza conținutului de umiditate
a plantelor este de obicei de 50%, și poate fi la fel de mare ca 90% din acvatice,
alge, inclusiv alge marine (vareci). Materialul este considerat "uscate", atunci când
se ajunge la echilibru pe termen lung cu mediul, de obicei, la aproximativ
10 până la 15% conținut de apă de masă.
Combustibilii pe bază de Carbon pot fi clasificate prin reducerea lor la nivel (§9.5.2).
Atunci când biomasa este transformată în CO
2
și H
2

O, energia disponibilă
este de aproximativ 460 kJ pe mol de carbon (38 MJ per kg de carbon; ~16 MJ
/ kg biomasă uscată), pe unitatea de reducere a nivelului R. Acest lucru nu este exact
cantitatea din cauza la alte schimbări de energie. Astfel, zaharuri (R = 1) au o căldură
de ardere de aproximativ 450 kJ pe 12 g de conținutul de carbon. Complet redus de
material (de exemplu, metan CH
4
(R = 2)) are o căldură de ardere de aproximativ
890 kJ pe 12 g de carbon (de exemplu, pe 16 g de metan).
CASETA 10.1 CALORIFICE BRUTE ȘI NETE
Puterea calorifică superioară (PCS) este căldura evoluat într-o reacție de tip
CH
2
O+O
2

CO
2
(gaz) + H
2
O (lichid).
(de exemplu, producția este apă lichidă și nu de abur sau vapori de apă, ca într-un cazan cu condensare care atât recuperează
căldura latentă). Chimiști de multe ori se referă la GCV ca numai căldura de combustie. Dacă nu se prevede altfel,
aceasta este măsura utilizată în această carte.
Unii autori citez pe net (sau mai mici) putere calorifică (LCV), care este căldura evoluat dacă finală H
2
Este O
gazoase, ca un abur, astfel încât nu există nici căldură latentă de recuperare (de exemplu, ca într-un motor cu ardere internă).
LCV este de aproximativ 6 la 7% mai puțin decât PCS pentru majoritatea biocombustibililor, și ~8% mai puțin pentru fosili,
petrol și
a motorinei.
În cazul arderii, umiditate în umede și umede biomasă combustibil solid determină
reducerea semnificativă termică utilă de ieșire, deoarece (i)
evaporarea apei necesită 2.3 MJ / kg, care este, în general, nu au fost recuperate;
(ii) temperatura de ardere este redusă; și (iii) poluant
emisii de fum este probabil. În schimb, combustibil uscat este un deliciu. Acest lucru afectează
modul în valoarea termică a combustibilului este măsurată (Caseta 10.1). Cu
cazane în condensație, de mult de astfel de căldură latentă poate fi recuperat prin condensare
de vapori de apă în emisie, astfel încât de intrare apa rece este
preîncălzit.
Densitatea de biomasă, și densitatea în vrac a stivuite fibros
biomasă, sunt importante, mai ales pentru transport și depozitare. În
general, de trei-patru ori volumul (de masă) de uscat biologic
materialul trebuie să fie acumulate pentru a oferi aceeasi energie pe cărbune.
Astfel, potrivit de transport și de manipulare a combustibilului este necesară în cazul în care biomasa nu este
utilizat la sursă.
TWIDELL PAGINARE.indb 366
01/12/2014 11:37
§10.2 Biocombustibil clasificare
367
Pentru utilizare ca un combustibil solid, biomasa solidă este ușor de depozitat și uscat într -
acoperite, deschise față-verso hambare. Cu toate acestea, ca un combustibil pentru motoare și pentru general
de utilizare, biomasa solidă este transformat în lichid și gazos biocombustibili, după cum
este indicat pe partea dreaptă a Fig. 10.3.
§10.2.2 Termochimic de căldură
Există multe clasificări, după cum, de asemenea, detaliate în secțiunile următoare.
(a)
Direct de combustie pentru imediată de căldură (§10.3). Aceasta este cea mai importantă utilizare
de lemne si busteni in ambele în curs de dezvoltare și dezvoltate.
Uscat omogen de intrare este mult mai de preferat. Cele mai bune rezultate au
(uscat) de lemn ars in soba, cuptor sau cazan, cu controlul
de intrare a aerului, astfel încât nu este plin, dar nu excesiv de ardere. Aer
Agrochimice
Bio-
chimice
Termo-
chimice
Oxigen
Densificarea
Solar
energie
Direct
de ardere
Piroliza
Gazeificare
Continuare
procese
Alcoolice
fermentarea
Anaerobe
digestia
Biophotolysis
Extragerea combustibililor
Lichide
Uleiuri
Liquifaction
Hidrogen
Groapa de gunoi
gaz
Biogaz/
gaz metan
Anaerobe
digestia
Căldură
Etanol
Gaze
Lichide
Gaze
Lichide
Uleiuri
Cărbune
Ardere
de combustibili pentru:
Procesul termic
de Transport
de Putere
de Aprovizionare etc.
Fotosinteza
Reziduuri
Organic
humus
și nutrienți
reciclate
Biomasa
producție
Fig. 10.3
Procesele de producere a biocombustibililor.
TWIDELL PAGINARE.indb 367
01/12/2014 11:37
368
Bioenergie tehnologii
intrarea este nevoie în jurul valorii de combustibil pentru perioada inițială de ardere și
de evacuare fierbinte pentru secundar de ardere. Unele biomasă, cum ar fi
fabrica de cherestea a deșeurilor sau intenționat produse rumeguș, estedensificat' de numai
compresie în pelete (~15 mm × ~5 mm) sau brichete (~100 mm
× ~40 mm). Acest proces face ca biomasa mai ușor și mai ieftin de a
transporta și livra la magazine, mai ușor să se hrănească direct din magazin
în camerele de ardere cu melc șuruburi, și este mai ușor de a
controla arderea injectat cu aer; în consecință, există acum
substanțial comerțul regional și internațional în pelete din lemn. Municipale de
deșeuri solide (MSW: §10.8) și uscate de canalizare pot fi prelucrate prin
densificare a produce solid cu ardere de combustibili,
(b)
Piroliza (§10.4). Biomasa este încălzită, fie în lipsa de aer,
sau de ardere parțială a unora din biomasa într-un restricționat
aer sau oxigen. Produsele sunt extrem de variate,
formate din gaze, vapori, lichide și uleiuri, și solid char și cenușă. La
ieșire depinde de temperatura, tip de material de intrare și tratamentul
proces. În unele procese de prezența apei este necesară și
, prin urmare, materialul nu trebuie să fie uscat. În cazul în care producția de gaze combustibile
este principalul produs, proces numit de gazeificare. Tradiționale
fabricarea mangalului și moderne calcinare la temperaturi moderate
de ~200°C la ~300°C produce solid char ca produsul dorit.
(c)
Alte procese termochimice (§10.5). O gamă largă de
pretratare și procesul de operații sunt posibile. In mod normal acestea
implica chimice sofisticate de control și la scară industrială de
fabricare; producția de metanol este un astfel de proces (de exemplu pentru combustibil lichid).
De o deosebită importanță sunt procese care descompun celuloza
și amidonul în zaharuri, pentru fermentație ulterioară.
§10.2.3 Biochimice
(a)
Aerobic digestie. În prezența aerului, microbiene aerobe
metabolismul de biomasă generează căldură cu emisiile de CO
2

, dar
nu de metan. Acest proces este de mare importanță pentru biologică
ciclu de carbon (de exemplu, decăderea din gunoi de pădure), și pentru prelucrare canalizare,
dar nu este utilizat în mod semnificativ comerciale bioenergie.
(b)
Digestia anaerobă (§10.7). În absența oxigenului liber, anumite
micro-organisme pot obține propria lor aprovizionare cu energie prin reacția
cu compuși de carbon de mediu nivel reducere (a se vedea §10.4) pentru a
produce atât CO
2
și pe deplin redus de carbon, metanul, CH
4

. La
proces (cea mai veche biologic 'dezintegrarea' mecanism) poate fi, de asemenea, numit
'fermentație, dar este de obicei numit 'digestie' din cauza similare
proces care are loc în tractul digestiv de animale rumegătoare.
A evoluat amestec de CO
2
, CH
4

și gazele se numește biogaz ca
un termen general, dar poate fi numit de canalizare-gaz sau depozitele de gaze ca
este cazul.
TWIDELL PAGINARE.indb 368
01/12/2014 11:37
§10.3 arderea Directă pentru căldură
369
(c)
Fermentația alcoolică (§10.6). Etanolul este un lichid volatil de combustibil care poate
fi folosit în loc de benzină rafinat (benzina). Acesta este fabricat de
acțiune de micro-organisme și este, prin urmare, un proces de fermentare.
Convenționale ("prima generație") etanol are zahăr ca materie primă,
care poate au fost produse din amidon (de exemplu, porumb, grâu,
orz) de alte micro-organisme în preliminară în procesul de malțificare.
(d)
Biophotolysis. Fotoliza este divizarea de apă în hidrogen și
oxigen prin acțiunea luminii. Recombinarea are loc atunci când
hidrogenul este ars sau a explodat ca un combustibil în aer. Anumite biologice,
organisme produc, sau pot fi făcute pentru a produce hidrogen în biophotolysis.
Rezultate similare pot fi obținute din punct de vedere chimic, fără a organismelor vii,
în condiții de laborator. Randamentele sunt mici, astfel încât R&D continuă pentru
exploatarea comercială (a se vedea §9.7).
§10.2.4 Agrochimice
(a)
Extragerea combustibililor. Ocazional, lichizi sau combustibili solizi pot fi obținute
direct din living sau proaspăt tăiat plante. Materialele sunt numite
exudate și sunt obținute prin tăiere în (atingerea) tulpini sau trunchiuri de
plante vii sau de strivire proaspăt recoltat material. Un bine-cunoscut
proces similar este producția de cauciuc natural latex. Legate de
plante pentru a planta de cauciuc Herea, cum ar fi specii de Euphorbia,
produce hidrocarburi de mai puțin cu greutate moleculară decât de cauciuc, care
poate fi folosit ca petroliere înlocuitori și terebentină. Unele
soiuri de algele , de asemenea, produce hidrocarburi direct la mare randament pe
unitatea de suprafață; în curs de desfășurare R&D cauta cost-eficiente de biocombustibil.
(b)
Biodiesel și esterificare (§10.9). Concentrat de uleiuri vegetale din
plante pot fi utilizate direct ca combustibil în motoarele diesel; într-adevăr, Rudolf
Diesel a conceput original 1892 motorul pentru a rula pe o varietate de combustibili,
inclusiv uleiuri naturale vegetale. Cu toate acestea, apar dificultăți cu utilizarea directă
de ulei de plante din cauza vâscozitate mare și arderea depozitelor
comparativ cu standardul diesel combustibil-ulei mineral, mai ales la ambiantă scăzută
temperatură de 5°C. Ambele dificultăți sunt depășite prin convertirea
ulei vegetal corespunzătoare ester, care este, fără îndoială, un combustibil
mai potrivit pentru motoarele diesel decât convenționale (pe baza de petrol)
motorină.
§10.3 ARDEREA DIRECTĂ PENTRU CĂLDURĂ
Biomasa este ars pentru a furniza căldură pentru gătit, confort termic (căldură a spațiului),
uscarea culturilor, fabrica de procese, și creșterea de aburi pentru energie electrică
de producție și de transport. Utilizarea tradițională a biomasei de ardere include: (a)
de gătit, cu lemne, cu acesta din urmă, probabil, furnizarea de aproximativ 15%
din consumul global de energie (o proporție extrem de dificil să se evalueze); și
(b) utilizări comerciale și industriale de căldură și energie (de exemplu, pentru zahăr
<
~
TWIDELL PAGINARE.indb 369
01/12/2014 11:37
370
Bioenergie tehnologii
trestie de frezat, de ceai sau de ulei de copra uscare, ulei de palmier prelucrarea și fabricarea hârtiei).
Eficiență și minim de poluare sunt ajutate prin utilizarea de combustibil uscat și
controlate, temperatura inalta de ardere. Tabelul B. 6 dă căldura de
ardere pentru o serie de culturi energetice, reziduuri, combustibili derivate și organice
produse, presupunând material uscat. Aceste date sunt importante pentru
utilizarea industrială a combustibilului din biomasă.
§10.3.1 Interne de gătit
O proporție semnificativă din populația lumii depinde de combustibil-lemn
sau alte tipuri de biomasă pentru gătit, încălzire și alte aplicații domestice. Mediu
zilnic, consumul de combustibil este de aproximativ 0,5 la 1 kg de biomasă uscată pe persoană,
adică 10-20 MJ d
-1
≈ 150 W. Înmulțită cu, să zicem, 2 × 10
9

oameni, acest lucru


reprezintă consumul de energie cel substanțial rata de 300 GW. Cel mai
intern de combustibil-lemn utilizat, dar cu siguranță nu toate, este în țările în curs de dezvoltare,
cu majoritate nu sunt incluse în comerciale statisticile în domeniul energiei. Aici vom
presupune combustibil s-a uscat bine, deoarece acesta este un prim pas esențial
pentru arderea biomasei (vezi §4.3 și §10.3.3); folosind umede sau ude de combustibil
ar trebui să fie evitate.
Un consum mediu de 150 W "continuu", exclusiv pentru gătit,
poate părea surprinzător de mare. Astfel un consum mare rezultă din
utilizarea pe scară largă de ineficiente metode de gatit, cele mai frecvente dintre
care este un foc deschis. Astfel de metode pot avea o eficiență termică de
încălzire de alimentare de doar aproximativ 5%, deși de trei-piatră' șemineu
permite lemnului să fie împins în pentru ardere controlată și îmbunătățită
eficiența. 'S-a pierdut energia include arderea incompletă a
lemnului, vântul dispersează căldura departe de foc, și prin radiație și
convective pierderile din nepotrivire de foc și de mărimea potului. Considerabile
de energie este, de asemenea, pierdut în evaporarea de descoperit vase (ca și în
bucătăriile din întreaga lume) și de la umed combustibil. Fum (de exemplu, nearse de carbon și tars)
dintr-un foc, este o dovadă de combustie incompletă, și nu pot fi puțin
control asupra rata la care lemnul este ars. În plus, fumul este un
pericol pentru sănătate dacă nu există un sistem eficient de extragere fum. Cu toate acestea,
un motiv pentru care să permită interne de fum poate fi de a descuraja insecte și dăunători
de pe acoperiș, și de a vindeca ("fum") mâncare uscată. Eficient ars, uscat
lemn, în care inițial a produs gaze nearse și tars arde într-o
reacție secundară, emite numai CO
2
și H
2
O cu complet ars, cenușă.
Gătit eficiență și facilități pot fi îmbunătățite prin:
1
Folosind combustibil uscat.
2
Introducerea de alimente alternative și metode de gătit (de exemplu, abur,
mașini de gătit).
3
Reducerea pierderilor de căldură folosind închis arzătoare sau sobe, și
wellfitting oale cu capace.
4
Facilitarea secundar de ardere de nearsă gazele de ardere.
TWIDELL PAGINARE.indb 370
01/12/2014 11:37
§10.3 arderea Directă pentru căldură
371
5
Introducerea aragaz controale care sunt robuste și ușor de utilizat.
6
Explicație, de formare și de management.
Cu aceste îmbunătățiri, cele mai bune de gătit sobe care utilizează combustibil-lemn și
circulație naturală a aerului pot plasa mai mult de 20% din energia de ardere
în vase de gătit. Modele folosind forțat și în mod activ controlat
de ventilație, să zicem, cu un ventilator electric, poate fi mai mult de 80% eficient, dar,
de gătit poate fi lent. Există multe științific pe bază de programe
pentru a îmbunătăți sobe de gătit, dar plin de piață de acceptabilitate nu este întotdeauna de
ajuns, mai ales dacă culturale și de gen factori nu sunt considerate
în mod adecvat. De departe cel mai mare astfel de program a fost în China, cu peste 170 de
milioane de noi sobe în utilizare, mai ales în zonele rurale. În Rwanda, mai mult de
jumătate din gospodării au acum astfel de sobe, cu proporția
în creștere. Banca Mondială (2010) a revizuit lecțiile învățate din mai multe
astfel de programe. Programe de succes oferă o gamă largă de eficient
aragaz modele adaptate la cerințele utilizatorilor și vândute în comerț; în
sobe și-au dovedit eficiența, capacitatea de a reduce poluarea aerului din interior,
durabilitate bună și sunt în siguranță.
Arderea de lemne de foc este un complex și variabile de proces. Mult
depinde de tipul de lemn și de conținutul de umiditate. Inițială
de ardere emisiile de CO, care în sine ar trebui să ardă în surplus de aer. La temperaturi
mai mari decât 370°C, oxalat de calciu în lemnul se descompune cu
eliberare de oxigen, astfel încât îmbunătățirea combustiei și reducerea
particulelor combustibile și a emisiilor. Un design bun se asigură că (i) de înaltă
temperatură de combustie este limitată la un alb-fierbinte volum mic de
regia, poate forțat, intrare aer; și (ii) că pirolitică gaze sunt
ei înșiși ars într-o combustie secundar regiune unde mai intra aer.
Dacă spațiul de încălzire este necesar, apoi, aparent pierdut de căldură de la
gătit devine util (§10.3.2).
O paralelă metodă pentru reducerea interne de combustibil-lemn cererii este de a
încuraja regenerabile alternative de energie, cum ar fi de biogaz
(metan, cu CO
2

) (a se vedea §10.7); combustibil de cultură deșeuri; și la scară mică


hidro-electrice (§6.6). Necesitatea acestor îmbunătățiri este copleșitoare
atunci când pădurile se împuținează și deserturi în creștere.
Fig. 10.4 prezintă două tipuri de sobe cu lemne, conceput pentru a face
o mai bună utilizare a lemnului ca combustibil de gătit. Ambele modele sunt suficient de ieftine pentru a
permite utilizarea pe scară largă în țările în curs de dezvoltare. Mai scump sobe
(adesea numite intervale) atât pentru gătit și încălzire a apei sunt obiecte de lux
în multe bucătării din europa de Nord și Nord-Americane, case,
unde unele modele permit arderea lemnului.
În sobă prezentată în Fig. 10.4(a), focul este complet închis în
focar pe stânga. Fier (de culoare închisă) ușa este eliminat numai atunci când
combustibilul este introdus. Aerul intră printr-o gaură de dimensiunea reglabil pe sub
ușă (complet închis în fotografie). Astfel, rata de ardere poate fi atent
controlate pentru a se potrivi tipului de gătit se face. Gazele fierbinți din
TWIDELL PAGINARE.indb 371
01/12/2014 11:37
372
Bioenergie tehnologii
Ciment
Ash
280
120
60
110
Ceramica grătar
(b)
(a)
(c)
Fig. 10.4
Îmbunătățirea eficienței sobe de gătit. (a) O mare de aragaz proiectat de către Fiji Ministerul
Energiei. Aceasta este o modificare de Indian (Hyderabad) chula, și este construit în principal
din beton muluri. Funcționarea sa este descrisă în text. (b) 'Thai găleată aragaz
(schiță). (c) secțiune Verticală prin aceeași (unitate: mm).
focul sunt conduse printr-un canal îngust de sub oale de gătit,
care sunt de dimensiuni pentru a se potrivi îndeaproape în găurile de pe partea de sus. În această etapă de aer
poate
intra prin alte canale secundare de combustie. Complet ars
gaze și vapori trece la mediul exterior prin horn
la sfârșitul departe de aragaz; acest lucru previne poluarea în zona de gătit și
încurajează fluxul de aer.
Ciment
Ash
280
120
60
110
Ceramica grătar
(b)
(a)
(c)
TWIDELL PAGINARE.indb 372
01/12/2014 11:37
§10.3 arderea Directă pentru căldură
373
Aragaz prezentată în Fig. 10.4(b) este mai simplu și mai ieftin, dar are mai puțin
control și mai puțină flexibilitate. Cu toate acestea, își masă mică înseamnă că este
transportabil și puțină căldură este folosit în instalațiile de încălzire aragaz distincte din
oală, care este un avantaj pentru gătire rapidă. Aer ajunge la combustibil de
mai jos, printr-un grătar. Deoarece focul este conținută, iar caldura este
canalizată spre oala, eficiența este mare. Acest aragaz este foarte potrivit pentru
utilizarea cu cărbune drept combustibil, deoarece cărbune arde curat, fara fum.
§10.3.2 Spațiului și încălzire a apei
Pentru confort (spatiu) de căldură în clădiri, ca și cu sobe de gătit, este
important pentru soba sau centrala-cazan de încălzire pentru a avea un foc controlat, cu
bun secundare de combustie. Eficiența este îmbunătățită în cazul în care aerul de
ardere este introdus direct în camera de ardere din exteriorul
clădirii, care scade interne proiecte și pierderea de căldură. Sofisticat
și eficient lemn-arzatoare pentru incalzire sunt folosite pe scară largă, mai ales
în unele din lemn bogat industrializate țări (de exemplu, Norvegia, Canada și
Noua Zeelandă). Dacă termică utilă este căldura livrată benefic, apoi
închise sobe si cazane cu controlate primare și secundare
de ardere poate fi de la 80 la 90% eficienta.
Unele țări (de exemplu, în Europa de Nord) încurajarea piețelor (i)
lemn de foc chips-uri (mașină de tăiat palmier de dimensiuni lemn); și (ii) sub formă de pelete ( comprimat
rumeguș din lemn de metri). Deși principala piață pentru aceste produse
pot fi de co-ardere cu cărbune în centrale electrice (§10.3.4), acestea sunt utilizate
pentru spațiu, apă caldă și căldură în clădiri individuale. Pentru aceasta din urmă,
sunt special concepute sofisticate sobe cu intrare automată de combustibil,
care sunt ușor de utilizat, au o eficiență energetică sporită și sunt curate, cu
minim de poluare.
§10.3.3 uscarea Culturilor
Uscarea culturilor (de exemplu, fructe, copra, cacao, cafea, ceai), pentru depozitarea și
vânzarea ulterioară, este de obicei realizat prin arderea de lemn și
resturi vegetale, sau folosind căldura reziduală din producerea energiei electrice. De
material pentru a fi uscate pot fi introduse direct în coșul de evacuare a gazelor, dar
există un pericol de foc și de contaminare a produselor alimentare. Mai
frecvent, aerul este încălzit într-un gaz/aer schimbător de căldură înainte de a trece prin
cultură. Uscare teorie este discutat în §4.3.
Arderea de reziduuri de recoltă pentru uscarea culturilor este o utilizare rațională a
biocombustibilului, deoarece combustibilul este aproape de locul unde este nevoie. Ardere într-o
eficientă cuptor produce un flux de cald, curat gazelor de eșapament (CO
2
+H
2

O+
exces de aer) la aproximativ 1000°C, care poate fi diluat cu aer rece la
temperatura necesară. Dacă cantitatea de biomasă reziduuri depășește pe cea
necesară pentru uscarea culturilor, excesul poate fi folosit pentru alte scopuri,
cum ar fi producerea de abur industrial.
TWIDELL PAGINARE.indb 373
01/12/2014 11:37
374
Bioenergie tehnologii
§10.3.4 Proces-energie electrică și termică
Abur de proces-căldură este de obicei obținut pentru fabrici de lemne
sau alte tipuri de biomasă reziduuri în cazane, poate de operare fluidizat cu
paturi. Fizic, este rațional de a utiliza abur în primul rând pentru a genera
energie electrică înainte de căldură se degradează de la o scădere de temperatură utilă. La
eficiența de generare a electricității din biomasă poate fi doar aproximativ 20
la 25% datorită temperaturii scăzute de ardere, astfel încât 75 la 80% din energia
rămâne la fel de proces-căldură și un util finale temperatura este menținută.
Frecvent la funcționarea optimă a unor astfel de procese tratează energie electrică
ca produs secundar din procesul-generarea de căldură, cu excesul de energie electrică de a fi
vândute local de energie electrică agenția de aprovizionare, ca și în moderne de trestie de zahăr mills
(Fig 9.11 și 9.12).
O modalitate relativ simplă de a utiliza culturile energetice și de biomasă reziduuri este
cofiring în cărbune de ardere de la centralele electrice. Metoda prin combustie este adaptat
pentru cunoscută amestec de cărbune și biomasă. Având o uniformă de combustibil
oferă cele mai fiabile de funcționare, astfel încât densificat produse cum ar fi lemn
pelete sunt favorizate. Calcinare (controlate de temperatură scăzută de piroliză
pentru a produce char) de amestec de biomasă și înainte de ardere îmbunătățește
final de ardere în cazane. În ultimii ani, un internațională substanțială
a comerțului în pelete din lemn (>10 PJ/y) a apărut, în special din Rusia și
Canada în Europa de Vest. O astfel de substituție (de reducere) de cărbune este o
politică realistă pentru biomasă pentru a reduce emisiile de gaze de seră, în
termen scurt, în ciuda intrinsecă eficienței tuturor acestor centrale
fără combinată de căldură și energie fiind doar aproximativ 35%.
§10.4 PIROLIZĂ (DISTILARE DISTRUCTIVĂ)
Piroliza este un termen general pentru toate procesele prin care materialul organic
este încălzit sau parțial arse cu minim de aer pentru a produce secundar
combustibili și produse chimice. Intrare poate fi din lemn, biomasă reziduuri,
deșeuri municipale, sau, într-adevăr, cărbune. Produsele sunt gaze, condensat de
vapori și lichide, gudroane și uleiuri, și reziduu solid char (cărbune) și
cenușă. Tradiționale de cărbune a face este de piroliză la temperaturi relativ joase,
cu vapori și gaze nu colectate; echivalentul modern este
calcinare, dar cu efluentul gazele arse pentru încălzirea proces.
Gazeificarea este piroliză adaptat pentru a produce o cantitate maximă de
combustibil secundar gaze.
Diverse piroliză unități sunt prezentate în Fig. 10.5. Vertical top-încărcare
dispozitive sunt, de obicei, considerat a fi cel mai bun. Combustibilul produse sunt
mult mai convenabil, curat si usor de transportat decât originalul biomasă.
Produsele chimice sunt importante ca materie primă în industria chimică pentru alte
procese sau în mod direct bunurile tranzacționabile. Ardere parțială dispozitive,
care sunt concepute pentru a maximiza cantitatea de gaz combustibil, mai degrabă
decât char sau volatile, sunt de obicei numite biocombustibili. Procesul este
, în esență, piroliză, dar nu pot fi descrise ca atare.
TWIDELL PAGINARE.indb 374
01/12/2014 11:37
§10.4 Piroliză (distilare distructivă)
375
Eficiența este măsurată ca căldura de combustie secundar
combustibili produse, împărțite de căldura de combustie de intrare biomasă
fi folosit. Mare eficiență de 80 până la 90% poate fi atins. De exemplu,
biocombustibili din lemn poate produce 80% din energia inițială în formă de
gaze combustibile (în principal H
2

și CO – producător de gaze), potrivit pentru


funcționarea în convertite petroliere alimentat de motoare. În acest fel generală
a eficienței producției de energie electrică (spun, 80% × 30% = 24%) ar putea fi
mai mare decât cea obținută cu un cazan de abur. Astfel de biocombustibili sunt
potențial utile pentru micii generarea de energie (<150 kW).
Procesele chimice din piroliză sunt strâns legate de similare
distilare a cărbunelui pentru a produce sintetic gaze, gudroane, uleiuri și coca-cola. De
exemplu, utilizarea pe scară largă de conducte de gaze orașului (H
2
+ CO) în Europa,
înainte de a schimba fosili "natural" de gaze (în principal CH
4
), a fost posibil de
reacția apei pe cărbune încălzite cu aport redus de aer:
H
2
O+C

H
2
+ CO
(10.2)
C+O
2

CO
2
; CO
2
+C

2CO
Următoarele este dat ca un rezumat al gamă largă de condiții și
produse de piroliză. Materialul de intrare trebuie să fie clasificate pentru a elimina
Filtru de particule
pot fi introduse
aici
Alte arzătoare
în paralel
(a)
(b)
Uscate de intrare
Char
Gauri de aer
cu capace
Condensat
vaporii
Aer
Aer
Perie din lemn
Stivă de lemn
Pământ
Materie primă uscată
Titularul de gaze
cu sigiliu de apă
Mecanice
de lucru;
transport
de energie Electrică
de producție
Aer
Gazul de lemn
titular
Gătit
căldură
Motor
cu gaz de intrare
Fig. 10.5
Piroliza sisteme: (a) la scară mică unitate de piroliză; (b) tradiționale de cărbune în cuptor.
TWIDELL PAGINARE.indb 375
01/12/2014 11:37
376
Bioenergie tehnologii
excesiv material incombustibil (ex. sol, metal), uscate, dacă este necesar
(de obicei complet uscat materialul este evitat cu biocombustibili, spre deosebire de cazane),
tocate marunt sau tocate, și apoi stocate pentru utilizare. Raportul aer/combustibil și timpul
de ardere este un parametru critic care afectează atât temperatura și
tipul de produs. Piroliza unități sunt cel mai ușor de operat la temperaturi
mai puțin de 600°C. temperaturi Crescute de la 600 la 1000°C nevoie de mai mult
rafinament, dar mai mult de hidrogen va fi produs în gaze. La mai puțin
de 600°C există, în general, patru etape în procesul de distilare:
1
~100 ~120 °C: intrare materialul se usuca cu umezeala trece
prin pat.
2
~275°C: producția de gaze sunt, în principal, N
2
, CO și CO
2
; acid acetic și
metanol distila off.
3
~280 la ~350 °C: reactii Exoterme au loc, de conducere off complexe
amestecuri de substanțe chimice (cetone, aldehide, fenoli, esteri), CO
2
,
CO, CH
4
,C
2
H
6
și H
2
. Unor catalizatori (de exemplu, ZnCl
2
) permite acestor a s-
ții să apară la temperaturi mai scăzute.
4
> 350 °C: Toate volatile sunt respinși, o proporție mai mare de H
2
este format
cu CO, și de carbon rămâne ca tăciunele, cu reziduuri de cenușă.
Cu temperaturi variind de la 350°C-550°C, condensate
lichide, numit tars și pyroligneous acid, pot fi separate și tratate
pentru a oferi identificare a produselor chimice (de exemplu, metanol, CH
3
OH, un lichid
de combustibil). Tabelul 10.2 oferă exemple și detalii suplimentare.
Secundar combustibili de piroliză au mai puțin totală de energie de
de ardere decât originalul biomasă, dar sunt mult mai convenabil de a
folosi. Unele dintre produse au în mod semnificativ mai mare densitate de energie
decât media de intrare. Confort include: o mai ușoară manevrare și
transport, livrare prin conducte de gaz, control mai bun al arderii, o mai mare
varietate de end-utilizați dispozitive, și mai puțin de poluare a aerului la punctul de utilizare. Pe
următoarele sectiuni ia în considerare solide, lichide și gazoase produse
respectiv.
Tabelul 10.2
Piroliza randamentele din lemn uscat
(aproximativă randamentele la 1000 kg (tone) lemn uscat (pentru 350°C <
~ T
<
~ 550°C))
Cărbune
~300 kg
De gaz (ardere 10.4 MJ m
-3
)
~140 m
3
(NTP)
Alcool metilic
~14 litri
Acid Acetic
~53 litri
Esterii
~8 litri
Acetona
~3 litri
Ulei de lemn și lumină tar
~76 litri
Ulei de creozot
~12 litri
Teren
~30 kg
TWIDELL PAGINARE.indb 376
01/12/2014 11:37
§10.4 Piroliză (distilare distructivă)
377
§10.4.1 Solid carbune (masă randament de 25 la 35% maxim)
Moderne de cărbune replici de operare la aproximativ 600°C produce 25 la 35% din
materia uscată a biomasei ca de cărbune. Tradiționale de lut cuptoare de obicei, dau
randamente mai aproape de 10%, deoarece există mai puțin control. Cărbune este de 75 la 85%
carbon, cu excepția cazului mare grijă este luată pentru a îmbunătăți calitatea (ca pentru industria chimică
clasa cărbune), și căldura de ardere este de aproximativ 30 MJ/kg. Astfel, dacă
cărbune singur este produs din lemn, între 15 și 50% din
original energie chimică de ardere rămâne. Cărbune este util ca un curat
controlabile de combustibil. Chimice, cărbune de clasa are multe utilizări în laborator
și procese chimice industriale. Cărbune este superioară față de produsele de cărbune
pentru a face din oțel de înaltă calitate.
§10.4.2 Calcinare
Aceasta este o formă de piroliză la temperaturi reduse, ~200°C la ~320°C,
cu efluentul gazele utilizate pentru încălzire. Este un produs uscat,
non-putrezire solid char, numit uneori 'bio-coal', care, ulterior, poate
fi ars în mod eficient, cu un minim de poluare. 'Bio-coal' este foarte potrivit pentru
co-ardere de combustibili fosili cărbunele în cazane, etc., deoarece reduce proporțional
cu emisii nedorite de la fosili, cărbune, inclusiv fosili dioxid de carbon.
De obicei, produsul își păstrează 80% din masa și 90% de încălzire
valoarea inițială de biomasă. Caracteristicile sale pot fi îmbunătățite în continuare
prin densificare. Controlate de calcinare este un proces relativ nou,
care este în creștere în utilizarea pe scară largă.
§10.4.3 Lichide (vapori condensat, masă randament de ~30%
maxim)
Aceste împărțiți între (1) un lipicios fenolici tar (creozot), și (2)
lichide apoase, pyroligneous acid, în principal acid acetic, metanol
(maxim 2%) și acetonă. Lichidele pot fi separate sau folosite
împreună ca o brută, potențial poluante și cancerigene, combustibil cu o
căldură de ardere de aproximativ 22 MJ/kg. Randamentul maxim corespunde
la aproximativ 400 de litri de lichid combustibil pe tona de biomasă uscată. De
lichide sunt mai bine folosit ca o sursă de substanțe chimice, dar acest lucru necesită
relativ scară largă și sofisticată operațiune.
§10.4.4 Gaze (masă randament de ~80% maxim în biocombustibili)
Mixt gaz ieșire cu azot este cunoscut sub numele de gaz de lemn, de sinteză,
gaz, gaz de producători sau de gaz de apă, și are o căldură de ardere în aer de
la 5 la 10 MJ/kg (4 până la 8 MJ/m
3

la STP). Acesta poate fi folosit direct în motoare diesel


ciclu sau motoarele cu aprindere prin scânteie, cu ajustare de la injector sau
carburator, dar extreme de îngrijire trebuie să fie luate pentru a evita consumul de cenușă și
vapori condensabili. Gazul este în principal de N
2
,H
2
și CO, cu, probabil,
TWIDELL PAGINARE.indb 377
01/12/2014 11:37
378
Bioenergie tehnologii
cantități mici de CH
4
și CO
2

. Gaz poate fi stocat în rezervoarele de gaze


în apropiere de presiune atmosferică, dar nu este convenabil comprimat. O mult
mai curat și mai uniformă a gazului poate fi obținut prin gazeificarea umed
cărbune, mai degrabă decât lemnul, deoarece majoritatea din tars originale din
lemn au fost deja eliminate.
Fischer-Tropsch este un termen general pentru o mare varietate de
metode care conversia CO și H
2
,
principalele gaze de gaz de sinteză
(producător de gaze), în uleiul potrivit pentru vehiculul de combustibil. De multe ori cărbune a fost
starter material utilizat pentru a genera inițială producător de gaze, dar biomasa poate
fi, de asemenea, starter material. Au fost multe pe scară largă industrială
unitățile care folosesc variante de proces în mai multe țări în
ultimii 100 de ani, dar nu au pe scară largă internaționale de replicare.
§10.5 ALTE PROCESE TERMOCHIMICE
În secțiunile anterioare, biomasa a fost folosită direct preliminare după
sortare și de tăiere pentru ardere sau piroliză. Cu toate acestea, biomasa
poate fi tratat din punct de vedere chimic: (1) pentru a produce un material potrivit pentru alcoolică
fermentație (§10.6); sau, (2) pentru a produce secundar sau îmbunătățite combustibili.
Luați în considerare următoarele câteva exemple importante de numărul mare
de posibilități.
§10.5.1 Hidrogen reducere
Dispersate, tocat sau digerate de biomasă (de exemplu, gunoi de grajd) este încălzit în
hidrogen la aproximativ 600°C sub presiune de aproximativ 50 de atmosfere.
Gaze combustibile, cea mai mare parte metan și etan, sunt produse care pot
fi arse pentru a da aproximativ 6 MJ / kg inițială de material uscat.
§10.5.2 Hidrogenare cu CO și abur
Procesul este ca mai sus, dar de încălzire este într-o incintă cu CO și
abur la aproximativ 400°C și 50 de atmosfere. Un ulei sintetic este extras
din care rezultă produse care pot fi folosite drept combustibil. Un catalizator este
necesar pentru a produce reacții de forma următoare:
CO + H
2
O
→ CO
2
+H
2
(10.3)
C
n
(H
2
O)
n
+ (n + 1)H
2
→ nH
2
O + H(CH
2
)
n
H
în cazul în care acesta din urmă reacție implică o conversie de carbohidrati material
pentru uleiuri hidrocarbonate. Energie eficienta de conversie este de aproximativ 65%.
§10.5.3 Acid și hidroliză enzimatică
Celuloza este constituentul major (30-50%) de plantă uscată a biomasei
și este foarte rezistent la hidroliză, și, prin urmare fermentare de
TWIDELL PAGINARE.indb 378
01/12/2014 11:37
§10.6 fermentație Alcoolică
379
micro - organisme (§10.6). Conversie la zaharuri, care pot fi fermentate,
este posibil de încălzire în acid sulfuric sau prin acțiunea enzimelor
(celulaze) de anumite microorganisme (§10.6). Produsele pot fi, de asemenea,
folosite ca hrană vitelor.
§10.5.4 Metanol lichid combustibil
Metanol, un lichid toxic, este facut din reacție catalitică a H
2
și CO
2
la 330°C și la 150 de presiunea atmosferică:
2H
2
+ CO
→ CH
3
OH
(10.4)
Intrare gaze sunt componente ale gazului de sinteză (§10.4.4), și poate
fi obținută din gazeificarea biomasei. Metanolul poate fi folosit ca un
combustibil lichid în petrol motoarele cu aprindere prin scânteie, cu o densitate de energie de
23 MJ/kg. Este, de asemenea, folosit ca un anti-knock aditiv de combustibil pentru a îmbunătăți
cifra octanică, și este un potențial important de combustibil pentru celulele de combustibil (§15.8).
§10.5.5 Hidrotermale de lichefiere: HTL
HTL este un proces termochimic care urmărește să imite, cel mult
a crescut viteza, procese care s-a transformat biomasei în combustibili fosili
peste perioadele geologice în scoarța Pământului. Procesele au nevoie
de încălzire pe biomasă, cum ar fi gunoiul de grajd solid, canalizare și culturi, cu apă
și, eventual, cataliză la temperaturi de ~300°C și presiuni de ~20 la
~50 MPa. În aceste condiții, proprietățile chimice ale apei favoarea
biomasei de rupere în jos, în uleiuri și reziduuri. Chimia este variat
și complexe, precum și viabilitatea comercială pentru produse petroliere pentru a concura
în general, cu biocombustibili convenționali deja în piață nu a
avut loc. Vezi Zhang (2010) pentru mai multe detalii.
§10.6 FERMENTAȚIE ALCOOLICĂ
§10.6.1 Alcool metode de producție
Etanol, C
2
H
5

OH, este produsă în mod natural de anumite micro-organisme


de zaharuri în condiții acide, pH de la 4 la 5. Acest alcoolice
procesul de fermentare este folosit în întreaga lume pentru a produce băuturi alcoolice. Cele mai
frecvente microorganisme, drojdie Saccharomyces cerevisiae, este
otrăvit de C
2
H
5

O concentrație mai mare de 10%, și așa mai puternic


concentrații de până la 95% sunt produse de distilare și de fracționare
(Fig. 10.6). Când distilată, constanta rămase punct de fierbere amestec
este etanol 95%, 5% apă. Anhidru etanolul este produs
comercial cu azeotrop de îndepărtare din apă de un plus de proces, cum ar fi
co-distilare cu solvenți cum ar fi benzen sau (mai recent) de
utilizare a site moleculare' (Mousdale 2010). Doar aproximativ 0,5% din
TWIDELL PAGINARE.indb 379
01/12/2014 11:37
380
Bioenergie tehnologii
potențialul de energie din zaharuri este pierdut în timpul fermentației, dar semnificative
cantități de căldură de proces sunt necesare pentru concentrare și
procese de separare (a se vedea Tabelul 10.4). Acest proces de căldură pot fi furnizate de
ardere sau instalații de gazeificare de altfel deșeuri de biomasă și de
recuperarea căldurii reziduale.
Zaharurile pot fi obținute prin următoarele rute, enumerate în ordine de
dificultate în creștere:
1
Direct din trestie de zahăr. În cele mai multe caen-țări producătoare,
comerciale zaharoză este eliminat din trestie de sucuri, iar
restul de melasă, utilizate pentru alcool procesul de producție (Fig 9.11 și
9.12). Aceste melasă înșiși au aproximativ 55% conținut de zahăr.
Dar dacă melasă au pic de valoare comercială, apoi etanol
de producție din melasă are favorabile posibilitățile comerciale, mai ales
dacă reziduuri de trestie de zahăr (trestie de zahăr) este disponibil pentru a oferi căldură de proces. În
Brazilia, în cazul în care politica și condiții agricole ambele favorizează
producția de etanol combustibil (vezi Caseta 10.2), cel mai nou mills sunt concepute pentru a fi
capabil de a procesa, pentru sucul de trestie direct la etanolului ca produs principal
, atunci când aceasta este din punct de vedere financiar favorabil. Marile reacție este transformarea
zaharozei pentru etanol:
C
12
H
22
O
11
+H
2
O
drojdie
4C
2
H
5
OH + 4CO
2
(10.5)
În practică, randamentul este limitată de alte reacții și creșterea în
masă de drojdie. Comerciale randamentele sunt aproximativ 80% din cele estimate
Căldura de proces,
de-produse
Zaharuri naturale:
trestie de zahăr
, sfeclă
de fructe
Amidon:
cereale (de exemplu la orz)
rădăcini (de exemplu, manioc)
Celuloza
lemn sau
tulpini de plante
Crush
Reziduuri
Hidroliza
(ușor)
Zahăr
Termochimice
pretratare
Hidroliza
(dificil)
Drojdie
fermentarea
la 10%
etanol
Distilare
la 95%
etanol
General
de ardere
a combustibilului
Petroliere
aditiv sau
substitut
Purificare
pentru a anhidru
alcool
Fig. 10.6
Etanol procesele de producție.
TWIDELL PAGINARE.indb 380
01/12/2014 11:37
§10.6 fermentație Alcoolică
381
de (10.5). Fermentarea reacții pentru alte categorii de glucide (de exemplu, glucoză,
C
6
H
12
O
6
) sunt foarte asemănătoare.
2
Sfecla de zahăr este un mid-latitudine recoltat pentru obținerea majore livrările de
zahăr. Zahărul poate fi fermentat, dar procedeu de obținere de căldură de
la reziduuri de culturi este, în practică, nu atât de simplu pe cât cu trestie de
zahăr, astfel încât producția de etanol este mai scump.
3
Amidon culturi (de exemplu, cereale și cassava) poate fi hidrolizat la zaharuri.
Amidonul este principalul stocare a energiei carbohidrati de plante, și este
compus din două mari greutate moleculară componente: amiloză și
amilopectină. Aceste relativ mare de molecule sunt, în esență, liniară,
dar au ramificat lanțuri de molecule de glucoză legate prin distinctiv
obligațiuni de carbon. Aceste legături pot fi rupte de către enzimele din malț
asociate cu culturi specifice (de exemplu, orz sau porumb), sau de către enzimele
de la anumite mucegaiuri (fungi). Astfel de metode sunt comune în whisky
distilleries, sirop de porumb fabricarea și producția de etanol din manioc
rădăcini. Link-uri pot fi, de asemenea, rupt de tratament cu acid, la pH 1,5-2
atmosfere presiune, dar randamentele sunt mici și procesul mai
scump decât enzima alternative. Un important produs al
enzimei de proces este reziduul folosit pentru hrana vitelor sau sol condiționat.
Toate procesele de mai sus sunt bazate pe secole de tehnologie
și de a folosi materie primă care ar putea fi, de asemenea, alimente; produsul lor este adesea numit
"prima generație" de bioetanol.
4
Celuloza reprezintă aproximativ 40% din biomasa uscată, inclusiv
culturi reziduuri rămase după cereale, sucuri și fructe au fost
eliminate. Anterior am menționat sale important de a folosi ca o ardere
de combustibil, dar, de asemenea, are potențialul de a fi un important material pentru etanol
de producție. O astfel de utilizare ar evita combustibil versus alimente problemele discutate
în §10.10 care limitează beneficiile de procese (1) la (3) de mai sus. Etanol
din celuloză, prin urmare, este adesea numit "a doua generație"
de bioetanol. Celuloză (greutate moleculară ~500.000 de) are o structură de polimer de
glucoză legate de molecule și formele principale mecanice-structură
componentă a părților lemnoase ale plantelor. Aceste link-uri sunt mult
mai rezistente pentru a rupe în jos în zaharuri sub hidroliza decât
echivalentul link-uri in amidon. În plante, celuloza se găsește în strânsă
asociere cu 15 la 25% din masa de lignină, un polimer care este chiar mai greu
pentru a rupe în jos decât celuloza – astfel, aceste materii prime lemnoase
(inclusiv ierburi și tulpini) sunt denumite colectiv 'ligno-celulozice'.
Mousdale (2010) oferă o analiză cuprinzătoare a stării de arta de
acest traseu pentru etanol. Hidroliza acidă este posibil ca cu amidon, dar
procesul este costisitoare și consumatoare de energie. Hidroliza este mai puțin costisitoare,
și mai puțin energia de intrare este necesară în cazul în enzime din lemn natural-putrezire ciuperci
sunt folosite, dar procesul este uneconomically lent. Cu toate acestea,
biotehnologic optimizat enzime da rezultate mai rapide. Pentru woody
material, fizică inițială defalcare este dificilă și costisitoare etapă,
TWIDELL PAGINARE.indb 381
01/12/2014 11:37
382
Bioenergie tehnologii
necesită mai multă energie pentru rulare și percuție mașini.
În consecință, un prototip comercial procese au folosit ca
intrare: (i) pastă de lemn sau din ziare vechi; (ii) coceni (reziduuri de tulpini
și frunze ale porumbului) și diverse ierburi, care sunt mult mai ușor de
tocat și adunat decât lemnul. Pentru toate substraturile, termochimice
pretratare a lignoceluloză crește ratele de prelucrare (de exemplu, acide
sau alcaline abur la ~200°C pentru 10 la 60 de minute), care slăbește
structura fizică, astfel încât creșterea suprafeței disponibile la
enzime.
Substanțiale R&D în statele UNITE ale americii și Scandinavia din 1990 încoace
a dus la procese cu randament îmbunătățit și potențial mai ieftin
de producție, principalele caracteristici ale care sunt acide catalizată de hidroliza
hemicelulozelor, mai eficiente enzime pentru a descompune celuloza și genetic
proiectat bacterii care fermentează toate biomasă zaharuri (inclusiv 5-carbon,
zaharuri, care rezista standard drojdii) la etanol cu randamente ridicate. Acolo
sunt câteva prototip plante care produc etanol din lignoceluloză, dar
, în general, mai multe fonduri pentru dezvoltare este nevoie pentru a progresa la
scară largă operațiune.
§10.6.2 Etanol combustibil utilizat
Combustibilii lichizi sunt de o mare importanță, deoarece de ușurința de manipulare
și controlabile de ardere în motoare. Azeotrop de etanol (de exemplu, la
constanta punct de fierbere amestec cu 4.4% apă) este un lichid între -114°C
și +78°C, cu o temperatură de aprindere de 9°C și o auto-aprindere (autoaprindere)
temperatura de 423°C; prin urmare, ea are caracteristici pentru o reclamă
combustibil lichid, fiind folosit ca un substitut direct sau aditiv pentru benzină
(benzină). Acesta este utilizat în trei moduri:
1
ca azeotrop etanol, utilizate direct în modificata si în scop-a construit
motoare cu aprindere prin scânteie;
2
amestecat ca o soluție cu fosili, petrol pentru a produce gasohol; folosit
la mic ~5% concentrațiile în nemodificat motoarele cu aprindere prin scânteie,
iar la concentrații mai mari, în 'flexi-mașină și special acordat de motoare;
3
ca o emulsie cu combustibil diesel pentru motoare diesel motoarele de compresie (acest lucru
poate fi numit diesohol, dar nu este comun).
Carburant cu conținut de bioetanol
2

are proporția de etanol indicat ca


EX, unde X este procentul de etanol (de exemplu, E10 a 10% etanol
și 90% fosili, petrol). Gasohol pentru motoarele nemodificate este, de obicei,
între E10 și E15; proporții mai mari de etanol necesită
moderată de modificarea motorului ca 'flexi-masini'. (Rețineți că apa nu se amesteca
cu benzina, si deci, apa este de multe ori prezent ca un nedizolvate de nămol în
partea de jos petroliere rezervoare de combustibil de automobile, fără a provoca dificultate;
dacă gasohol este adăugat la un astfel de rezervor, apa se dizolvă în etanol
fracțiune și combustibil poate deveni improprii pentru un motor nemodificat.)
TWIDELL PAGINARE.indb 382
01/12/2014 11:37
§10.6 fermentație Alcoolică
383
Gasohol, cu etanol cea mai mare parte din trestie de zahăr, este acum un standard în Brazilia
(a se vedea Caseta 10.2) și în țări din Africa de Sud. Este mandatat, inițial
ca E5, în Europa și, de asemenea, în statele UNITE ale americii în cazul în care etanolul este predominant
de porumb (porumb), cereale.
Etanolul aditivul anti-knock proprietăți și este de preferat să
mai frecvente tetraetil de plumb, care produce poluarea gravă a aerului. Cele
excelent de ardere proprietăți de etanol permite un motor modificat pentru a
produce până la 20% mai multă putere cu etanol decât anterior cu
petrol. Densitatea de masă și valoarea calorică a etanolului sunt ambele la mai puțin de
cele de petrol, astfel încât energia pe unitatea de volum de etanol (24 GJ/m
3
)
este cu 40% mai puțin decât pentru petrol (39 GJ/m
3

) (a se vedea Tabelul B. 6). Cu toate acestea,


la mai bine de ardere proprietăți de etanol aproape compensa când
măsurată ca volum pe unitate de distanță (de ex. litri/100 km). De combustibil
consumul de volum în mașini similare folosind benzină, gasohol sau etanolul pur este în
raport de 1: 1: 1.2, de exemplu etanolul pur este de numai 20% inferior de aceste criterii.
Observăm, totuși, că obiceiul de măsurare combustibil lichid
de consum pe unitate de volum este înșelătoare, deoarece măsurare pe unitatea de masă
se referă mai bine să entalpia combustibilului.
Costurile de producție de etanol combustibili locale depind foarte mult de
condițiile și cererea se referă la prețurile plătite pentru produse alternative.
Politica guvernului și ratele de impozitare sunt extrem de importante în
stabilirea prețului de vânzare cu amănuntul și, prin urmare, scara de producție (a se vedea §10.10 și
Caseta 10.2).
CASETA 10.2 ETANOL DIN BRAZILIA
De etanol Brazilian program este cel mai celebru exemplu de suport pe scară largă și pentru producția
de biocombustibili. A fost înființată în anii 1970 pentru a reduce dependența țării de importurile de petrol și de a
ajuta la stabilizarea producției de zahăr, și, prin urmare, ocuparea forței de muncă, în contextul instabil prețurile mondiale pentru
ambele
zahăr și petrol. Programul ambele creșterea ocupării forței de muncă în industria zahărului și a generat
sute de mii de noi locuri de muncă în prelucrare și de fabricație. Aceasta a condus la economii de scară și
dezvoltarea tehnologică, care a redus costul de producție de etanol din trestia de zahăr, în măsura în
care, în 2013, nesubvenționat costul de producție de azeotrop de bioetanol în Brazilia a fost de ~25 USc/L.
în Consecință, chiar și etanol anhidru fost mai ieftin decât fosili benzina pentru țiței prețurile mai mult decât
~NE-45 dolari/bbl. În 2013, petrolul brut vândut de ~NE-110 dolari/bbl!
Programul a evoluat de-a lungul timpului, ca răspuns la condițiile în schimbare de pe piețele internaționale de
zahăr și de petrol, în special cele mai mici preturi pentru combustibili fosili în 1990. L-a folosit atât stimulente fiscale
(de exemplu, a redus taxele pentru unele forme de combustibil) și regulamentul (de exemplu, necesită rafinării de a lua și de
piață,
întreaga producție de bioetanol, fie ca amestecuri (de obicei E20 să E25) sau ca azeotrop etanol), și
încurajează cu tărie utilizarea de flexi-fuel' vehicule, capabile să funcționeze pe combustibili variind de la E0 la
E85, și pe azeotrop etanol. Succesul programului a fost ajutat de mai mulți factori locali:
(1) coexistența de zahăr agro-industrie și naționale, industria de automobile, ambele având capacitatea
de tehnologia de echilibru de dezvoltare; (2) internă de automobile de piață suficient de mare pentru a susține noul
motor de lege, și (3) voința politică de a continua programul și forța de masini importate pentru a fi
'flexi-fuel'.
TWIDELL PAGINARE.indb 383
01/12/2014 11:37
384
Bioenergie tehnologii
§10.6.3 producția de Etanol din culturi
Tabelul 10.3 dă contur date de etanol de producție și randamentul culturilor.
Producția mondială de etanol pentru combustibil depășit 80 de miliarde de litri în 2010:
dublu în 2003. De acest lucru, statele UNITE ale americii a produs 60% și Brazilia 30%
(REN21 2012).
Caseta 10.3 evaluează măsura în care această producție a face o
contribuție pozitivă la reducerea utilizării combustibililor fosili și reducerea
emisiilor de gaze cu efect de seră.
Frecvent, lichid, biocombustibilii sunt produși din culturi alimentare (de exemplu, prin
fabricarea de combustibil etanol din porumb, utilizate anterior în întregime ca hrană pentru
oameni și animale). În efect, alimente ferme sunt transformate în energie
ferme (§9.6). În plus, terenul nu este deja în uz comercial ar putea fi
folosite pentru a creste culturi pentru consumul de energie. Aceste metode ridica două importante
probleme socio-economice:
1
Va exista o alimentație adecvată, la un preț accesibil pentru a hrăni prezentul
și viitorul populației umane (a se vedea §9.8)?
2
Două dintre cele mai de multe ori-a declarat motive pentru producerea de biocombustibili lichizi
într-o țară sunt: (a) de descreștere a consumului de combustibili fosili
pentru motive de securitate energetică națională' (articolul 17.2); și (b) pentru a reduce
naționale de gaze cu efect de seră (articolul 17.2). Dar nu o țară proprie
producția de biocombustibili utilizați mai mult combustibil fosil decât biocombustibili ar
Consecința a fost o expansiune majoră de zahăr/etanol industrie (~400% din 1980), cu
multe noi morile moderne, îmbunătățirea productivității agricole și operațiunile de fabrică și semnificative
co-generarea de energie electrică de la mills (Caseta 10.3). Producția de bioetanol în Brazilia acum depășește 30 GL/y,
dintre care unele sunt exportate, în special în Europa.
Sursa: Goldemberg (2007); Alonso-Pippo et al. (2013).
Tabelul 10.3
Aproximativ randamentele de etanol din diferite culturi, pe baza randamentelor medii în
Brazilia (cu excepția porumb, care se bazează pe NOI producții). Două recolte pe an sunt posibile în unele zone.
Randamentele reale depind foarte mult de practica agricolă, sol și vreme.
De litri de etanol
pe tonă de cultură
De litri de etanol
pe hectar și an
Trestie de zahăr
86
6200
Cassava
180
2160
Sorg dulce
86
3010
Cartofi dulci
125
1875
Porumb
(porumb boabe, ploaie)
370
2300
(irigat)
370
4600
Lemn
160
3200
TWIDELL PAGINARE.indb 384
01/12/2014 11:37
§10.6 fermentație Alcoolică
385
deplasa? Și nu-l reduce, în fapt națiunii emisiilor de GES?
Cutii 10.3 și 10.4 ia în considerare aceste întrebări empiric.
3

Tabelul 10.4 subliniază importanța crucială pentru bioenergie sisteme de


folosind low-cost biomasă reziduuri de proces, de energie electrică și termică
producția. Noi procese intră uz comercial, care produce
bioetanol din celulozice intrări, cum ar fi coceni de porumb ('stover'), special cultivate
plante (de exemplu, miscanthus), și reziduuri forestiere (a se vedea §10.6.1). Deoarece aceste
produse, toate folosesc biomasă reziduuri de proces, de energie, astfel cum sunt definite în
Caseta 10.3, lor energia din combustibili fosili, rapoarte de Cercetare și de combustibili fosili net de energie
câștiguri
G va fi mult mai mari decât cele de porumb etanol produs cu utilizarea
de combustibili fosili. Dacă de produse și utilizarea de biocombustibili sunt incluse
(de exemplu, deplasarea pe bază de cărbune, energie electrică), apoi pentru etanol din porumb stover
sau de miscanthus, R devine extrem de mare și G2 depășește
entalpia de etanol (Wang et al. 2011). O lecție de învățat din astfel de
analize pe produse comerciale este că întregul sistem trebuie să fie
atent definite și controlate pentru a evalua impactul lor asupra mediului,
amprenta de carbon și de durabilitate, etc.
CASETA 10.3 BIO/FOSILI ÎN BALANȚA ENERGETICĂ A BIOCOMBUSTIBILI LICHIZI
Intrări luate în considerare sunt tranzacționate de energie utilizate în mașini agricole, uscare, prelucrare, transport,
fabricarea de echipamente și îngrășăminte, etc. Ca disciplină stabilită de Analiză Energetică (care
definește termenii de altfel), nu considerăm energia solară ca o intrare. Această analiză este limitată la
biocombustibilii produși în întregime într-o anumită țară și folosit în întregime pentru a înlocui (abate) combustibili fosili –
care este mai aproape de realitate pentru ambele Brazilian trestie de zahăr etanol și pentru etanolul din porumb în statele UNITE
ale americii. Vom folosi doi
parametri și indicatori pentru o anumită națiune:

La nivel național bio/energie fosilă raportul R (= conținutul de energie (entalpia) de combustibil de ieșire împărțit de fosile de
combustibil utilizată pentru producerea e).

La nivel național bio/fosili castigul net de energie G (= entalpia combustibilului ieșire minus entalpia de
combustibili fosili de intrare folosit pentru a produce).
Dacă nu combustibililor fosili este folosit, atunci R este egal cu infinitul și G este egal cu entalpia (luate să fie căldura de
ardere) de biocombustibil. Scopul pentru sustenabilitatea este că ambele R și G trebuie să fie cât mai mare posibil.
Dacă G este negativ, deci R <1, contribuția de biocombustibil ca un înlocuitor pentru combustibilii fosili în care țara
este negativ. Două variante de R și G apar în literatura de specialitate, desemnate în Tabelul 10.4 ca R1 și G1 și ca
R2 și G2. R1 consideră numai entalpia de biocombustibil lichid ca ieșire, în timp ce R2 include, de asemenea,
entalpia în unele co-produse ca o ieșire.
Pentru calcularea R presupune energia utilizată în prealabil procese (de exemplu, fabricarea de îngrășăminte), energie,
echilibru calcule se referă la analiza ciclului de viață (§17.4). Ca un exemplu, Tabelul 10.4 rezumă publicat
calculele pentru producția de combustibil etanol din trestie de zahăr în Brazilia și din porumb (porumb) în statele UNITE ale
americii.
Deși ambele indica un rezultat pozitiv net câștigul de energie, pentru trestie de zahăr este mult mai mare. Principalul motiv
este că trestie de zahăr de frezat folosește zero combustibili fosili (rd (6)), deoarece procesul de căldură și de energie electrică
sunt din
arderea de reziduuri tulpini de trestie de zahăr (trestie de zahăr) (vezi fig 9.11 și 9.12). În moderne fabrici de zahăr, la fel ca în
Brazilia,
procesul de cogenerare produce nu doar procesul de căldură și energie electrică pentru moara de la sine, dar, de asemenea, un
TWIDELL PAGINARE.indb 385
01/12/2014 11:37
386
Bioenergie tehnologii
Tabelul 10.4
Bio/fosili în balanța energetică a producției de etanol din diferite substraturi de cultură.
Datele se referă la combustibili fosili (FF) utilizate în producerea culturilor și apoi în prelucrare la etanol (EtOH):
unitate MJ per L de etanol anhidru produs. Calcule bazate pe încălzire mai mici valori (etanol 21.1
MJ/L; petro-diesel 36.4 MJ/L). A se vedea Caseta 10.3 pentru explicații suplimentare.
BRAZILIA TRESTIE DE ZAHĂR STATELE UNITE ALE AMERICII PORUMB (PORUMB)
MJ/(L EtOH)
MJ/(L EtOH)
INTRĂRI de combustibili fosili pe litru de etanol produs
(1) operații de teren și de transport la moara
1.5
7.0 [o]
(2) îngrășăminte
0.8
2.1
(3) mașini agricole
0.1
n/a [b]
------
--------
(4) subtotal (agric ops)
2.3
9.1
(5) moara cu utilaje (încorporat)
0.3
n/a
(6) direct FF folosi la moara
0.0
6.3
------
---------
(7) subtotal (de prelucrare)
0.3
6.3
(A) TOTAL FF INTRARE
2.5
15.4
IEȘIRI pe litru de etanol produs
(B) Etanol (LHV)
21.1
21.1
(8) surplus de biomasă
1.9
0.0
(9) excedentul de energie electrică
0.9
0.0
------
------
(C) TOTAL IEȘIRI pe litru
de etanol produs
23.9
21.1
R1 Bio/energie fosilă raport
R1= (B)/(A)
8.4
1.4
R2 Bio/energie fosilă raport
R2= (C)/(A)
9.5
1.4
G1 Bio/fosili castigul net de energie G1= (B)-(A)
18.6
6.6
G2 Bio/fosili castigul net de energie G2= (C)-(A)
21.4
6.6
Note
o
Include 4 MJ/L pentru transportul la moara, pe baza Persson et al. (2009), calculate astfel G1 este mai mică decât cea de Wang et al.
(2011).
b
n/a = nu este disponibil; acești termeni sunt, probabil, cel puțin la fel de mare ca cea corespunzătoare pentru Brazilia.
Surse de date: Brazilia: Macedo et al. (2008) ; statele UNITE ale americii: Wang et al. (2011); autor calcule.
vandabil surplus de energie electrică și de trestie de zahăr (rânduri (8) și (9)). (În anii 1980, când fabricile au fost mai puțin
eficiente
și randamentele de trestie de zahăr per ha au fost mai mici, R1 și R2 pentru o fabrică de zahăr au fost semnificativ mai mică, de
obicei ~4.)
În schimb, etanolul din porumb proces în statele UNITE ale americii utilizează substanțiale de combustibil fosil (rd (6));
reziduul de porumb,
tulpini, în general, rămân neutilizate la fermă. În 1970 acest lucru utilizarea combustibililor fosili a fost atât de mare încât G de
porumb pentru
etanol a fost negativ. De atunci, procesul de eficiența consumului de combustibil s-a îmbunătățit cu un factor de ~4 și utilizarea
de îngrășăminte
a scăzut cu un factor de ~2, deci G este acum în mod clar pozitiv.
TWIDELL PAGINARE.indb 386
01/12/2014 11:37
§10.7 digestie Anaerobă pentru biogaz
387
CASETA 10.4 GAZE CU EFECT DE SERĂ (GES) BALANȚA DE BIOCOMBUSTIBILI LICHIZI
Prin enumerarea CO
2

emisiile asociate cu utilizarea energiei în fiecare etapă a unui proces, un


echilibru energetic poate deveni un echilibru asociate emisiilor de gaze cu efect de seră. Astfel de calcule
se facă distincția între (a) biomasă reziduuri de proces energie (zero plus CO
2

emisiilor, deoarece
reziduurile-ar fi descompus în mod natural, oricum), și (b) de combustibili fosili intrările de energie (a se vedea Keshgi et
al. 2000).
Rezultate pentru biocombustibili lichizi sunt prezentate în Fig. 10.7. Intervalele pentru fiecare combustibil indică: (a) diversitatea
de
materii prime și site-ul de productivitate, în special cantitatea de combustibili fosili consumați în producerea acestora;
(b) schimbarea ipoteze despre tehnologie randamentele din cauza tehnologii în curs de dezvoltare rapid; (c)
utilizarea de co-produse; (d) importanța N
2

O a emisiilor (de multe ori legate de utilizarea îngrășămintelor); și (e)


sistem de
limite. Unele sisteme sunt prezentate în Fig. 10.7 ca având negative a emisiilor de GES, de exemplu gaze cu efect de Seră s-au
redus
de bioetanol din trestie de zahăr depășește gaze cu efect de Seră emise în sale de producție. Cu toate acestea marea majoritate a
biocombustibililor duce la o reducere a emisiilor de GES, atunci când înlocuirea combustibililor fosili. Nu sunt incluse în
Fig. 10.7 sunt de GES în serios
handicap emisiile generate de schimbarea utilizării terenurilor (de exemplu, drenarea turbăriilor pentru a planta ulei de palmier),
care pot
provoca emisii de metan, un puternic GES. Astfel de efecte evident handicap potențial emisiilor de GES
reducere folosind astfel de ulei de palmier ca biodiesel. Tehnic, politic și etic, leadership-ul este vital, dacă suntem pentru a
obține cele mai bune beneficiile economice și de mediu de biocombustibili.
Ciclul de viață al emisiilor de GES
(gCO
2
-eq/ MJ)
Transformarea lignocelulozei FTD
Petrol și diesel
Trestie de zahăr
Sfeclă de zahăr
Porumb și grâu
Transformarea lignocelulozei
Petroliere benzină
Ulei de plante BD
Alge BD
Uleiuri vegetale RD
200
100
-100
0
Fig. 10.7
Gama de raportat emisiile de gaze de seră pe unitate de energie de ieșire de la modern biocombustibili. Bioetanol din diverse
substraturi la stânga; biomotorină din diverse substraturi la dreapta; petrol benzină și motorină din petrol prezentate pentru
comparație. De utilizare a terenurilor legate de modificări nete a stocurilor de carbon și de gestionare a terenurilor efectele sunt excluse.
Sursa: adaptat de la IPCC (2011, Fig. 2.10).
§10.7 DIGESTIE ANAEROBĂ PENTRU BIOGAZ
§Introducerea 10.7.1
Descompunere biomasă și deșeuri animale sunt defalcate în mod natural să
elementare substanțe nutritive și humus sol prin descompunere microorganisme, fungi și
bacterii. Procesele sunt favorizate de umed, cald și în condiții de întuneric.
Etapele finale sunt realizate prin mai multe specii diferite de bacterii
clasificate fie ca aerobe sau anaerobe.
TWIDELL PAGINARE.indb 387
01/12/2014 11:37
388
Bioenergie tehnologii
Aerobe bacteriile sunt favorizate în prezența oxigenului cu
biomasa de carbon fiind complet oxidat la CO
2

. Acest proces de compostare


eliberează un foc lent și la nivel local, dar nu este un proces util pentru
furnizarea de energie. Pentru a fi aerobic, aerul trebuie să pătrundă, așa că o pierde 'heap' de
biomasă este esențial. Interne de compostare este ajutat foarte mult, inclusiv prin
straturi de șifonată de ziar si carton, care permite aerului buzunare
și introduce benefice carbon din carbohidrati material. Astfel
aerobic digestie a minim a emisiilor de metan, CH
4

, care, per
suplimentare moleculă, este de aproximativ opt ori mai puternic ca un efect de seră
de gaze decât CO
2
(a se vedea §2.9).
În închis condiții, fără oxigen disponibil din mediu,
anaerobe bacterii exista prin descompunerea glucidelor material. De
carbon poate fi în cele din urmă împărțit între complet oxidat CO
2
și pe deplin
redus CH
4

(a se vedea Fig. 9.6). Substante nutritive, precum azotul solubil


compuși rămân disponibile în soluție, astfel încât furnizarea de îngrășământ excelent și
humus. Fiind realizat de catre micro-organisme, reacțiile sunt toate
clasificate ca tipuri de fermentație, dar în condiții anaerobe termenul
"digestia" este de preferat.
Ar trebui subliniat faptul că, atât aerobe și anaerobe
de descompunere sunt fundamentale procese naturale, ecologie, care afectează toate
biomasă, indiferent de implicarea umană. Ca și cu toate celelalte forme
de energie regenerabilă, suntem capabili de a interfata cu procesul natural
și canal, energie și resurse pentru economia noastră. Descompus
de deșeuri ar trebui să fie apoi eliberat pentru proceselor ecologice naturale pentru a
continua.
Biogazul este CH
4
/CO
2

gazos se amestecă evoluat de la digestoarele anaerobe,


inclusiv a deșeurilor și a apelor uzate gropi; să utilizeze acest gaz, digestoare sunt
construite și controlate pentru a favoriza producția de metan și extragerea
de lichid, suspensii (Fig. 10.8). Energia disponibilă din arderea
biogazului este între 60% și 90% din substanță uscată de căldură de
de combustie din material de intrare. Cu toate acestea, gazul este obținut din
reziduuri de până la 95% apă, astfel încât, în practică, biogaz energie este de multe ori
disponibil în cazul în care nici unul, altfel, ar fi fost obținute. Un alt
beneficiu este faptul că digerat efluentului forme în mod semnificativ mai puțin de o sănătate
de pericol decât material de intrare. Cu toate acestea, nu toate paraziți și agenți patogeni
sunt distruși în digestie. Pentru intensiv incintele de
'agricultura industrială', posibilitatea de a trata deșeurile fecale și efluenților
pentru a le face acceptabile pentru mediu și necesitatea de a evita
sancțiuni pentru poluarea sunt principalele stimulente pentru a încorpora digestoarele anaerobe
și de a utiliza biogazul.
Economia și beneficiile generale de biogaz sunt întotdeauna cele mai
favorabile atunci când digestor este plasat într-un flux de deșeuri deja
prezent. Exemple sunt sisteme de canalizare, cocină de porci spălări, bovine vărsat
reziduuri, deșeuri de abator, alimentară, de prelucrare a reziduurilor, apelor uzate și
municipale refuza depozitele de gunoi. Beneficiile economice sunt ca material de intrare
TWIDELL PAGINARE.indb 388
01/12/2014 11:37
§10.7 digestie Anaerobă pentru biogaz
389
nu trebuie să fie colectate special, supravegherea administrativă este
deja prezent, de eliminare a deșeurilor este îmbunătățită, și folosește sunt susceptibile de a fi
disponibil pentru biogaz și bogat în substanțe nutritive efluent. Cu toate acestea, în mare și
latitudini medii, rezervor digestoarele trebuie să fie încălzită pentru rapid digestia
(mai ales în timpul iernii); de obicei, astfel de căldură ar veni de la ardere
la ieșire gaz, prin urmare, reducerea randament net semnificativ. Digestie lenta
nu are nevoie de astfel de încălzire. Evident obținere a biogazului din
depozitele de deșeuri urbane deșeuri este o altă sarcină de inginerie decât la bovine
reziduuri. Cu toate acestea, biochimie este similar. Cele mai multe dintre următoarele
se referă la rezervorul de digestoare, dar aceleași principii se aplică la alte tipuri de biogaz
sisteme.
Generare de biogaz este potrivit pentru mici - pentru o operațiune pe scară largă.
Până în 2013, mai multe milioane de uz casnic-dimensiuni sisteme au fost instalate
în țările în curs de dezvoltare, în special în China (>40 milioane) și India
(>4 milioane de euro), cu gaze utilizate pentru gătit și iluminat. Rurale sisteme în
India folosesc cea mai mare parte balega de vaca ca intrare, dar ~1 milion de sisteme sunt "urban",
hrănite în principal cu bucatarie deșeurilor (a se vedea de exemplu, §10.7.4). Succes pe termen lung
operație necesită: (a) antrenat de întreținere și reparații tehnicieni; (b)
utilizatorii să perceapă beneficiile; (c) combustibili alternativi (de exemplu, kerosen și o sticlă
de gaz), să nu fie subvenționate, și (d) o sursă durabilă de organice de intrare
și de apă. Din păcate, multe sisteme cu biogaz au eșuat, deoarece
unul sau mai mulți dintre acești factori a fost lipsă.
Cele mai multe sisteme cu biogaz în țările industrializate funcționeze la intensiv
ferme de animale (Fig. 10.8(d)), la mari fabrici de bere și similare de culturi folosind
industrii, de la urban stații de epurare și ca parte de depozite municipale
('gunoi sfaturi'). Biogaz de la aceste site-uri pot fi injectat pentru vânzare în
gaze naturale-rețea de distribuție rețele (fie direct, ca amestecat CH
4
/ CO
2
sau ca
numai CH
4
, având în eliminat CO
2

, de exemplu, prin barbotare prin apă).


Mai mult, de obicei, cu toate acestea, biogazul devine combustibil pentru cu aprindere prin scânteie
motoare generatoare de energie electrică atât pe site-ul utilizat și pentru export la un
utilitar de rețea. În Europa mai ales, astfel de sisteme pot funcționa
ca combinată de căldură și energie (CHP), mai ales dacă în apropiere de orașele
unde căldura poate fi folosit pentru termoficare (§15.3.3). Germania are
semnificativ capacitatea de generare de biogaz (>5000 de sisteme cu un număr total
de energie electrică-capacitate de generare ~2300 MW),
Sisteme cu biogaz în ferme sunt un pas spre 'integrat',
care a emula un plin ciclu ecologic pe o singură fermă. Cele mai bune
exemple sunt unde de plante și animale deșeuri sunt anaerob digerat pentru
biogaz și digerate efluentul trece de aerobic digestie deschide în
rezervoare înainte de dispersie. Biogazul poate fi (a) utilizate direct pentru piața internă
și căldura de proces, pentru export la un utilitar de gaz, și, eventual, pentru
iluminat, sau (b) în calitate de combustibil pentru motoare și generatoare electrice, cu, probabil,
de export la o grila de utilitate. Algele pot fi cultivate în aer liber, tancuri și
eliminat pentru hrana vitelor. De la aerobic digestie, efluentului tratat
trece prin stuf, atunci poate să acvarii și iazuri de rață
TWIDELL PAGINARE.indb 389
01/12/2014 11:37
390
Bioenergie tehnologii
înainte de final, fiind trecut la câmp ca îngrășământ. Succesul unor astfel de
sisteme depinde de un favorabile site-ul, design integrat, bun standardele
de construcție, și de entuziasmul și angajamentul operatorului,
nu în ultimul rând pentru întreținere regulată necesară.
§10.7.2 procesele de Bază și energetică
Ecuația generală pentru digestie anaerobă în intrare de suspensie este:
C
x
H
y
O
z
+ (x
− y/4 − z/2)H
2
O
(10.6)
→ (x/2 − y/8 + z/4)CO
2
+ (x/2
+ y/8 − z/4)CH
4
Pentru celuloză, aceasta devine:
(C
6
H
10
O
5
)
n
+ nH
2
O
→ 3nCO
2
+ 3nCH
4
(10.7)
Unele materiale organice (de exemplu, lignină) și toate incluziuni anorganice nu sunt
digerate în proces. Acestea se adaugă la cea mai mare parte a materialului, forma un
gunoi și cu ușurință pot bloca sistemul. În general, 95% din masa de
material este apa.
Reacțiile sunt ușor exotermă, cu tipic căldurilor de reacție
fiind de aproximativ 1,5 MJ / kg uscat de digerat material, egală cu aproximativ 250 kJ
pe mol de C
6
H
10
O
5

. Acest lucru nu este suficient pentru a afecta în mod semnificativ


temperatura materialului în vrac, dar nu indică faptul că cele mai entalpia de
reacție este trecut la gazul produs.
Dacă materialul de intrare de suspensie au fost uscate și arse, căldura
de ardere ar fi fost de aproximativ 16 MJ/kg. În finaliza anaerobă
digestie doar aproximativ 10% din potențialul de căldura de combustie este pierdut în
procesul de digestie, astfel oferindu-90% eficiență de conversie de biogaz.
Mai mult decât atât, foarte umed de intrare este procesat pentru a da acest convenabil și
controlabile combustibil gazos, întrucât uscarea suspensie apoasă ar fi
necesară multă energie (aproximativ 40 MJ/kg de substanțe solide utilizate). În practică,
digestie rareori se duce la finalizare pentru că de mult timp implicat, deci
60% de conversie este comună. Gaz randamentul este de aproximativ 0,2-0,4 m
3

per kg de
uscat digerat de intrare la STP, cu debit de aproximativ 5 kg uscat de digerat
solid pe m
3
de lichid.
În general, se consideră că trei intervale de temperatură favorizează
anumite tipuri de bacterii. Digestia la temperaturi mai mari, încasări mai
rapid decât la o temperatură mai scăzută, cu gaz ratele de randament dublare la
fiecare 5°C de creștere. Intervalele de temperatură sunt: (a) psicrophilic, aproximativ
20°C; (b) mezofile, aproximativ 35°C, și (c) termofile, aproximativ 55°C. În
țările tropicale, neîncălzite digestoare sunt susceptibile de a fi la mediu sol
temperatura între 20 și 30°C. prin Urmare, digestia este
psicrophilic, cu timpi de retenție fiind de cel puțin 14 zile. În climatele mai reci, la
psicrophilic procesul este mult mai lent, astfel încât acesta poate fi decis la căldură
digestatului, probabil, utilizând o parte din producția de biogaz; o temperatura
TWIDELL PAGINARE.indb 390
01/12/2014 11:37
§10.7 digestie Anaerobă pentru biogaz
391
de aproximativ 35°C este probabil să fie ales pentru mezofile digestie. Câteva
reactoare funcționează la 55°C, cu excepția cazului în care scopul este de a digera material, mai degrabă decât
produce excesul de biogaz. În general, cea mai mare temperatura, cu
atât mai repede procesul de timp.
În procesele biochimice au loc în trei etape, fiecare facilitată de
seturi distincte de bacterii anaerobe:
1
Insolubil materiale biodegradabile (de exemplu, celuloză, polizaharide și
grăsimi) sunt descompuse în glucide solubile și acizi grași (
hydrogenesis). Acest lucru se întâmplă în aproximativ o zi la 25°C într-un activ de fermentare.
2
Formează Acid bacterii produc în principal acid acetic și acid propionic (
acidogenesis). Această etapă, de asemenea, nevoie de aproximativ o zi la 25°C.
3
Gaz metan-bacteriilor care formează lent, în aproximativ 14 zile la 25°C, complet,
digestia la un maxim de ~70% CH
4
și minim ~30% CO
2
cu urme de Sec
2
și, probabil, H
2
S (metanogeneza). H
2

poate
juca un rol esențial, și într-adevăr, unele bacterii (de exemplu Clostridium) sunt
distinctivă în producerea de H
2
ca produs final.
Metan-formarea de bacterii sunt sensibile la pH-ul, și condiții
ar trebui să fie ușor acidă (pH 6.6-7.0), dar nu mai acid decât pH 6.2.
Azot ar trebui să fie prezente la 10% din masa uscată de intrare, și
fosfor la 2%. O regula de aur pentru succes digestor operațiunea este de a menține
condiții constante de temperatură și potrivit material de intrare;
în consecință, potrivit populației de bacterii devine stabilite pentru a se potrivi
aceste condiții.
§10.7.3 dimensionarea Digestorului
Energia disponibilă de la o instalație de biogaz de fermentare este dat de:
E
=
ηH
b
V
b
(10.8)
în cazul în care
η este randamentul arderii de arzatoare, cazane, etc. (~60%).
H
b
este căldura de combustie pe unitatea de volum de biogaz (20 MJm
-3
la 10 cm
ecartament de apă sub presiune, 0.01 atmosferă) și V
b

este volumul de biogaz.


Rețineți că o parte din căldura de combustie a metanului se duce la
încălzirea CO
2
prezent în biogaz, și, prin urmare, este indisponibil pentru
alte scopuri, astfel încât scăderea eficienței.
O analiză alternativă este:
E =
ηH
m
f
m
V
b
(10.9)
unde H
m
este căldura de ardere a metanului (56 MJ/kg, 28 MJ/m
3
la
STP) și f
m
este fracțiunea de metan în biogaz. Pentru biogaz direct
din digestor, f
m
ar trebui să fie între 0,5 și 0,7, dar nu este greu
pentru a trece de gaz printr-un contracurent de apă pentru a dizolva CO
2
și
crește f
m
la aproape 1.0.
TWIDELL PAGINARE.indb 391
01/12/2014 11:37
392
Bioenergie tehnologii
Volumul de biogaz este dat de:
V
b
= cm
0
(10.10)
unde c este randamentul de biogaz pe unitatea de masă uscată de intrare întreg (0,2 -
0,4 m
3
/ kg) și m
0

este masa uscată de intrare.


Volumul de lichid în digestor este dat de:
V
f
= m
0
/
r
m
(10.11)
în cazul în care
r
m
este densitatea de materie uscată într-un lichid (~ 50 kg/m
3

).
Volumul digestorului este dat de:
V
d
= V
.
f
t
r
(10.12)
în cazul în care V
.
f
este debitul de fermentare lichid și t
r
este timpul de retenție
în digestor (~8 la 20 de zile).
Parametrii tipice pentru deșeuri de origine animală sunt prezentate în Tabelul 10.5.
Tabelul 10.5
Tipic gunoi de grajd de ieșire de la animalele de fermă; notă mare
proporție de lichid din gunoi de grajd care favorizează producția de biogaz, mai degrabă
decât de uscare și ardere
Animale
Total ude
gunoi de grajd per
animal per
zi/kg
Din care,
total solide
/kg
Umiditatea
masei
de conținut /umed
masa
Vacă de lapte (~500kg)
35
4.5
87%
Carne de vită pilota (~300 kg)
25
3.2
87%
Îngrășare porc (~60 kg)
3.3
0.3
91%
Găină ouătoare
0.12
0.03
75%
EXEMPLUL 10.1
Calcula (1) volumul de biogaz de fermentare potrivit pentru producția de 6000 de porci; și (2) puterea
disponibilă de la digestor, presupunând un timp de retenție de 20 de zile și un arzător de eficiență de 0.6.
Soluție
Masa de solide (pe zi) în deșeuri este de aproximativ:
m
0
= (0,3 kg d
-1
)(6000)
= 1800 kg d
-1
(10.13)
Din (10.11) volumul de fluid (pe zi) este:
=
=



V
.
(1800kg d )
(50 m )
36m d
f
1
3
3
1
(10.14)
În (10.12), reactor de volum este:

V = (36 m d )(20 d) = 720 m
d
3
1
3
(10.15)
TWIDELL PAGINARE.indb 392
01/12/2014 11:37
§10.7 digestie Anaerobă pentru biogaz
393
Din (10.10), volumul de biogaz este:
V = (0.24 m kg )(1800 kg d ) = 430 milioane de d
b
3
1
1
3
1



(10.16)
Deci, din (10.8), energia de ieșire este:




E = (0.6)(20 MJ m )(430 m d )
= 5200 MJ d = 1400 kWh d
= 60 kW (continuu, termice)
3
3
1
1
1
(10.17)
Dacă în mod continuu convertite în energie electrică, acest lucru ar genera aproximativ 20 kW
e
de
energie electrică din biogaz-a tras generator stabilit la 25% eficiența globală.
§10.7.4 Lucru digestoare
Fig. 10.8 prezinta o serie de biogaz digestoare de la elementar la cel
sofisticat, care permite principii pentru a fi explicat.
o
Uz casnic din mediul Rural de fermentare (Fig 10.8(a) și 10.8(b)). Acesta este un
design recomandat în Republica China pentru consumatorii casnici și satul
comune, în cazul în care mai multe milioane de euro au fost instalate; modele similare
sunt acum comune în India și Nepal. Principala intrare este de obicei de porc
bălegar în China și bălegar de vacă în India. Principala caracteristică de design
este beton capac care permite gaz sub presiune pentru a fi obținute,
deși o parte din capacul este detasabil pentru întreținere. Acest top este
mult mai ieftin decât la metale grele plutitoare gasholder de vârstă Indian
sisteme. Fluxul se mișcă încet prin îngropat caramida rezervor în
aproximativ 14 la 30 de zile de la priza, de la care este bogat în nutrienți îngrășământ este
obținut. Ca gazul evoluează, volumul său înlocuiește digestor fluid și
presiunea crește. Frecvente (~zi cu zi) de inspecție a conductelor, etc. și
Material
de admisie
(a)
Gaz
țeavă capac Detașabil
Priza
Separarea
perete
Anaerobe
digestia
Rezervorul de stocare a gazului
(b)
Fig. 10.8
Biogaz digestoare.
o
Chinez 'dom' pentru utilizarea la scară mică. Bălegar diluat curge în subteran pentru digestor, care deține, de asemenea, biogaz la
presiune moderată (adaptat de la Van Buren (1979)).
b
Fotografie de similare în uz în mediul rural din India; 'dom' este vizibilă în spatele flori la dreptul de admisie (autor foto).
TWIDELL PAGINARE.indb 393
01/12/2014 11:37

394
Bioenergie tehnologii
Rezervor
(c)
Agitator
Ieșire
Schimbător de căldură
Arzător cu gaz
Digestor
Gaz decolare
Domenii
Setarea
(preaplin a rezervorului)
Apa sigilate titularul de gaze
(d)
Fig. 10.8
(cont.)
c
Ritm accelerat ferma de fermentare cu încălzire, pentru utilizarea în mijlocul latitudini (adaptat de la El (1976)).
d
Un sistem mare la Hotelul din Devon, marea BRITANIE, care prelucrează ~80.000 de m
3
/an de material organic. La partea din spate sunt două dintre
trei 4000m
3
rezervoare de fermentare; la dreapta este un 2500m
3

pierde-vară rezervorul tampon. Modificat motoare diesel în prim-plan


genera aproximativ 20 GWh/an de energie electrică (foto: AnDigestion Ltd.).
întreținerea regulată este esențială pentru a evita înfundarea de non-digerabile
material.
b
Urban gospodar (nu se afișează). Compact, de uz casnic de fermentare
Corespunzătoare Rurale Institutul de Tehnologie (ARTI) în India este urbane
utilizare, folosind amidon și zahăr deșeuri de culturi și produse alimentare de intrare (de exemplu,
răsfățat de cereale, coapte sau diform fructe, non-semințe comestibile, bucătărie
deșeuri, resturi alimentare, etc.). Acesta este construit din cut-jos
highdensity polietilenă rezervoare de apă, care sunt disponibile. Pentru că
aproape toate de intrare este rapid digerat, productivitatea este bună (500 g
de metan de la ~2 kg de intrare) și timpul de retenție este extrem de
scurt: ~3 zile pentru utilizarea completă a deșeurilor alimentare. Există atât de puțin
(d)
TWIDELL PAGINARE.indb 394
01/12/2014 11:37
§10.8 Deșeuri și reziduuri
395
reziduuri care ARTI recomanda pur și simplu amestecarea cu următorul lot de
intrare pentru a menține cultura de bacterii anaerobe.
c
Design Industrial (Fig 10.8(c)). Diagrama prezinta un design pentru
exploatarea comercială în mijlocul latitudini pentru accelerarea digestiei complet sub
condiții controlate. Digestor rezervor este, de obicei, încălzit la cel puțin
35°C. scopul principal al unui astfel de sistem este de multe ori tratamentul
de altfel inacceptabil deșeurilor, cu biogaz fiind un
beneficiu suplimentar.
d
Studiu de caz (Fig 10.8(d)). Fotografie prezinta centralizat de
digestie anaerobă stației electrice de la Hotelul din Devon, marea BRITANIE. Trei
4000m
3
digestoare (din care doar două sunt prezentate în fotografie) poate
procesul de 80.000 m
3

pe an de material organic. Primite de deșeuri


stream este încălzită prin schimb de căldură cu ieșire reziduu și
cu "deșeuri" de căldură de motoare utilizate pentru generarea de energie electrică.
Planta are 3.9 MW capacități de producție. Cantitatea de
energie electrică generată la un moment dat depinde de cantitatea
și natura materiilor prime fiind furnizat de plante. De obicei,
planta produce ~1700 MWh de energie electrică pe lună, din care 10% este
folosit în exploatare a instalației și 90% este vândut la rețeaua locală. De
alimentarestocurile de plante provin din diverse surse locale, inclusiv produse lactate
ferme industriale, brutării și procesatorii de alimente, abatoare, pește,
procesatori, producători de brânză, producătorii de biomotorină și consilii. După
pasteurizare și digestia, nămolul este returnat la fermele locale ca un
bio-îngrășământ, pentru utilizarea pe ambele arabil și pășuni.
§10.8 DEȘEURI ȘI REZIDUURI
Deșeuri și reziduuri provenite din activitatea umană și economică de producție
sunt o formă de "indirecte" din surse regenerabile de energie, deoarece acestea sunt de neoprit
fluxurilor de energie potențială în mediul nostru. Deșeurile și reziduurile care rezulta
din: (a) primare activitate economică (de exemplu, silvicultura, lemn mills, recoltate
culturi, abatoare, prelucrare a produselor alimentare); și (b) urbane, municipale și de interne
refuza, inclusiv canalizare. La potentialul de generare a energiei din astfel de
deșeuri este în primul rând de conținutul de biomasă. Cu toate acestea, există, de obicei,
o proporție semnificativă de deșeuri combustibile de la surse de minerale (de ex.
cele mai multe materiale plastice); cu toate acestea, o astfel de ardere necesită regulament pentru reducerea
inacceptabilă a emisiilor. Un factor-cheie cu privire la deșeuri și să refuze este că
acestea sunt de obicei disponibile în punctele de concentrare, unde au ușor
deveni un pericol pentru mediu. De-a face cu aceasta "problema" devine
o necesitate. Deșeurile operator, prin urmare, va fi plătit pentru materiale
și așa mai fi subvenționate în mai târziu, producția de energie.
Majore de deșeuri sunt: (a) deșeurile municipale solide (MSW); (b) depozite de deșeuri; (c)
canalizare. MSW este deșeurile eliminate de către autoritățile municipale de la
interne și surse industriale; de obicei, conține cantități semnificative
de metal, sticlă și plastic (de exemplu, non-biomasă) material. Reciclare de cele mai multe
TWIDELL PAGINARE.indb 395
01/12/2014 11:37
396
Bioenergie tehnologii
materiale plastice, metal, sticlă și alte materiale trebuie să aibă loc înainte de deșeuri
sau de ardere. Cu toate acestea, non-biomasă materiale rămân de obicei în
cantități semnificative. MSW este slăbit, material solid de variabile
de compoziție, disponibile direct pentru ardere sau piroliză. Dacă compoziția este
acceptabil, acesta poate fi sub presiune și extrudat ca 'refuza derivate combustibil,
RDF', de obicei disponibil ca pelete uscate de aproximativ 5 cm dimensiune pentru
ardere internă scară cazane.
'Deșeuri' este a deșeurilor, de obicei, deșeurile municipale solide (MSW), depozitate în
gropi mari. O mare parte din deșeurile municipale solide (MSW) este biologic
material care, odată închis în groapa de gunoi, se degradează anaerob pentru biogaz,
emisă ca un amestec de CH
4
și CO
2
, de multe ori contaminate cu aer (O
2
și
mai ales N
2

), de obicei numit 'gazul de depozit'. Procesul este mai lent decât în


cele mai biogaz digestoare, pentru că a redus temperatura la sol, dar
atunci când stabilizat după mai multe luni de zile, compoziția de gaz este similar (a se vedea
§10.7). Dacă nu sunt colectate, scurgeri de gaz încet în Atmosferă, împreună
cu diverse smellier gaze, cum ar fi H
2

E, deci, provocând neplăcut


de poluare a mediului și de a fi potențial explozive. Reglementări în mai multe
țări au nevoie de captura de cel puțin 40% din metan de la depozitele de deșeuri,
în scopul de a reduce emisiile de gaze de seră și de pericol. Prin urmare,
depozitele de deșeuri site-ul este construit cu atenție pentru a preveni contaminare a solului și
după umplere este limitat (de exemplu, cu lut), astfel încât gaz pot fi colectate
(de exemplu, printr-o serie de țevi perforate fixate orizontal ca groapa de gunoi este
finalizat sau forate vertical în îngropate refuza de un site existent). În afară
de inutil evazată în partea de gaz pentru a evita accidentele, există trei moduri
de a utiliza gazul de depozit: (a) într-o turbină cu gaz sau modificate motor cu aprindere prin scânteie
pentru a genera pe site-ul de energie electrică și vinde în exces; (b) vândute direct la o
apropiere instalații industriale pentru ardere directa in cazane, pentru căldura de proces
și/sau în motoare pentru producerea de energie electrică; (c) injectat și-a vândut într-un
utilitar de alimentare cu gaz, probabil după barbotare prin apă pentru a elimina CO
2
.
Cu limitată a terenurilor pentru depozitele de deșeuri și creșterea de sortare și reciclare a
deșeurilor, multe municipalități motiva și de a ajuta gospodăriile să-și locul
de deșeuri alimentare și de grădină de plante deșeurilor în containere etichetate, care sunt colectate
de cotitură în grădină și horticole compost de tocare și apoi
aerobic digestie (Fig.10.2).
Producția de energie la nivel global de incinerare a deșeurilor și gaz de fermentare a deșeurilor
depășește 1,5 EJ/an la nivel global și este de multe ori o proporție semnificativă de național
comerciale din surse regenerabile de energie.
§10.9 BIODIESEL DIN ULEIURI VEGETALE SI ALGE
§10.9.1 Prime uleiuri vegetale
Uleiurile vegetale sunt extrase din biomasa pe o scară considerabilă pentru utilizare
în săpun de luare, alte procese chimice și, în formă mai rafinată, pentru
gătit.
TWIDELL PAGINARE.indb 396
01/12/2014 11:37
§10.9 Biodiesel din uleiuri vegetale si alge
397
Categorii de materiale adecvate sunt după cum urmează:
1
Semințele (de exemplu, floarea-soarelui, de rapiță, de jatropha, boabe de soia).
2
Nuci (de exemplu, ulei de palmier, de cocos, copra); ~50% din masa uscată de ulei (de exemplu, în
Filipine anuală de producție de ulei de cocos este de ~1.5 milioane t/y).
3
Fructe (de exemplu producția de măsline ~3 Mt/an).
4
Frunze (de exemplu, eucalipt).
5
Exploatat exudate (de exemplu, latex de cauciuc, de jojoba (Simmondsia chinensis)
ulei de arbore).
6
De-produse din biomasa recoltată, de exemplu, uleiuri și solvenți pentru a
15% din masa uscată a plantelor (de exemplu terebentină, oleorășini de arbori de pin,
ulei de Euphorbia).
§10.9.2 Biodiesel (esteri)
Concentrat de uleiuri vegetale pot fi folosite direct ca combustibil în motoarele diesel,
dar apar dificultăți de viscozitate ridicat și din arderea
depozitelor, comparativ cu convenționali (fosili) pe baza de petrol diesel
ulei, mai ales la temperatură ambiantă joasă (
≤ 5°C).
Ambele dificultăți sunt considerabil diminuat prin reacția extras
ulei vegetal cu metanol sau etanol pentru a forma echivalentă ester. Astfel de
esteri, numit biodiesel, au caracteristici tehnice ca și combustibili, care sunt
mai bine adaptate la motoarele diesel decat pe baza de petrol motorină. De
reacție dă echivalent ester și glicerină (de asemenea, numit glicerol). La
proces folosește, de obicei, KOH, ca un catalizator. Glicerina este, de asemenea, un instrument util și
produs vandabil.
Esterificarea procesul este simplu pentru cei cu bază de
cunoștințe chimice, și, având în siguranță, poate fi realizată ca un
mic proces discontinuu. Continuă comercială de producție, evident, are nevoie de
mai mult rafinament, și folosește orice ulei este cel mai ușor și ieftin
disponibil în țara în cauză (de exemplu, ulei de rapiță în Europa (numit
'rapita' în alte țări) și ulei de soia în statele UNITE ale americii). Biodiesel, de asemenea, pot
fi făcute din deșeuri (folosit) ulei de gătit și de grăsimi animale (seu). Astfel
Argentina, cu mare industrie a animalelor, a devenit o substanțială
exportator de biomotorină (>1 GL/y). Utilizarea deșeurilor de ulei de gătit ca prime
materialul este atractiv în atât de mediu, cât și punctul de vedere al costurilor, mai ales pe
o scară mică; costul de colectare este o problemă pe o scară mai mare.
Atunci când unele guverne eliminate barierele instituționale la
producția și vânzarea de biodiesel, lumea comerț a crescut dramatic de la
aproape zero în 1995 la peste 20 GL/y, până în 2011, două treimi din care a fost
produsă în UE, care a RE ținte și o susținere instituțională
cadru. Combustibilul este vândut, fie ca 100% biodiesel sau amestecate cu
petrol pe bază de motorină. Deși costul de producție de biodiesel
depășește substanțial de diesel convenționale de combustibili fosili, cum ar
guvernele justificat politica în ceea ce privește "extern" beneficiile pentru
mediu (de exemplu.g. absența emisiile de sulf, atenuează carbon fosil).
TWIDELL PAGINARE.indb 397
01/12/2014 11:37
398
Bioenergie tehnologii
Considerații similare se aplică la multe alte biocombustibili, în special bioetanol
(a se vedea §10.6 și §10.10).
Densitatea de energie de biomotorină ca un ester variază cu compoziția
și este de obicei de aproximativ 38 MJ/kg, care este mai mare decât pentru ulei brut și
aproape de petrol pe bază de motorină la aproximativ 46 MJ/kg. Cu toate acestea,
în practică, consumul de combustibil pe unitatea de volum de un motor diesel de vehicule
care rulează pe biodiesel este puțin diferită de cea pe diesel fosili. De calitate
au fost stabilite standarde privind compatibilitatea de biomotorină cu
cele mai multe vehicule. Un minor beneficia de utilizarea biodiesel este că evacuare
miros amintește de gătit (de exemplu, de popcorn).
Echilibrul energetic calcule pentru biodieselul obținut din ulei de soia și
metanol, în general, economia SUA indică faptul că producția de
1 MJ de combustibil poate utiliza aproximativ 0,3 MJ de combustibili fosili de intrare. La
producția de metanol din (fosili) de gaze naturale reprezintă aproape jumătate de
0.3 MJ, astfel de analiză ar fi chiar mai favorabil în cazul în care metanol (sau
etanol) a venit din biomasă, a se vedea Tabelul 10.4 și Caseta 10.4.
§10.9.3 Microalge ca sursă de biocombustibil
Cultivarea culturilor energetice nu ar trebui să reducă necesare culturi alimentare,
mai ales pe o scară globală. O strategie este de a utiliza microalge cultivate în
apă; singur celulare, fotosinteza, plantele de ~10
-6
m la ~10
-4

m
în diametru. Ele cresc rapid, în general, dublarea numere în termen de 24 de
ore, iar unele specii conțin uleiuri, la fel ca plantele mai mari. Conținutul de ulei
variază de la 15 la 75% (greutate uscată); prin urmare, potențialul anual de ulei
producția de bogate în ulei de microalge este de 50.000 la 150.000 L/ha, care este de
~10 ori mai mult pe unitatea de suprafață decât din surse vegetale. Comerciale
de creștere este, de obicei, în iazuri deschise la 20°C la 30°C, cu intrare de lumina soarelui,
CO
2
și substanțe nutritive (N, P, și minerale). Biodiesel din alge este un-
ori menționată ca o a doua generație de biocombustibili.
Producerea de biodiesel din microalge este un proces dovedit, dar
scumpe, fiind de ~10 ori mai mult ulei de palmier brut (probabil cel mai ieftin
ulei vegetal) sau fosili-petro-diesel (Cheng și Timilsina 2011). În deschide
iazuri, oleaginoase microalge devin contaminate de către autoritățile locale alge
și bacterii, atât de transparent incintele sunt necesare. Aceste și alte
provocări conduce considerabile R&D pentru un produs comercial.
§10.10 ASPECTELE SOCIALE ȘI DE MEDIU
§10.10.1 costuri Interne și externe de biocombustibili pentru transport
Costul producerii de bioetanol și biodiesel este în general mai
scumpe decât extragerea și rafinarea combustibililor fosili. Cu toate acestea,
auto petroliere, combustibili, de obicei, sunt impozitate, cu, probabil 70% din
prețul cu ridicata fiind fiscale. O astfel de impozitare ridică venituri și descurajează
inutile de conducere pentru a reduce poluarea, congestionarea traficului rutier și, de obicei,
TWIDELL PAGINARE.indb 398
01/12/2014 11:37
§10.10 aspectele Sociale și de mediu
399
importurile costa de schimb valutar. Guvernele pot, prin urmare,
să încurajeze includerea de biocombustibili ca un procent de combustibil pentru vehicule: (a) cu
un impozit mai mic pe biocombustibili decât pe fosili, petrol, și/sau (b) mandatul
care toate mijloacele de transport de combustibil trebuie să conțină un anumit procent de biocombustibili (a se
vedea
Caseta 10.2 pe Brazilia etanol program). Desigur, astfel de măsuri sunt
posibile numai dacă producătorii de motoare sunt necesare pentru a produce
biofuelcompatible vehicule, care este punct de vedere tehnic nu este dificil. Subvenții acordate
pentru producătorii agricoli de biocombustibili, ca la politica Comună a UE
, Politica Agricolă, sunt un alt instrument de politică.
Ecologic, înlocuind biocombustibil pentru fosili, petrol reduce
emisiile de gaze de seră, cu condiția ca biocombustibil provine dintr-o potrivit
procesul (a se vedea Caseta 10.3). Biocombustibili de ardere sub controlate în mod corespunzător
condițiile de obicei, este mai completă decât pentru fosili, petrol, astfel încât
nesănătoase emisiile de particule sunt mai mici. Mai mult decât atât, (a) toți biocombustibilii
au un procent mai mare de oxigen și o proporție mai mică de sulf
impurităților din compoziția lor chimică decât fosili, petrol
hidrocarburi, atât
2

poluarea este neglijabil; și (b) pe bază de biocombustibil de o planta


specii tinde să aibă identice compoziția chimică, întrucât fosili
petrolul este un amestec complex de substanțe chimice diferite, astfel încât biocombustibil
procesul de ardere poate fi reglat mai eficient.
§10.10.2 Alte chimică efecte de biocombustibili și biomasă
de ardere
Fiecare țară are reglementări privind permis și
interzis de emisii de gaze, vapori, lichide și solide. Aceasta este o mare și
complexă, în studii de mediu.
Cel mai important aspect pentru arderea optima de combustibil este de a
controla temperatura de intrare și de oxigen, de obicei, ca de aer. Scopul cu
biomasă și biocombustibili de ardere, ca cu toate tipurile de combustibili, este de a avea emisii
cu minimum de particule (nears și parțial ars materiale), cu
complet de carbon oxidat la CO
2
și nu CO sau CH
4,

și cu minim de
concentrația de oxizi de azot (de obicei rezultă din aer excesiv
de temperatură). Prin urmare, în practică, de ardere ar trebui să fie limitată la un
spațiu relativ mic, la aproape alb-cald; acest volum trebuie
să fie alimentat cu aer și combustibil proaspăt. În plus, numai pe deplin ars cenușă ar trebui să
rămână (cel mai bun la o pulbere fină care se mișcă aproape ca un lichid). Termică utilă
este extras prin radiație de ardere și prin conducție de la
gazele de ardere printr-un schimbător de căldură, de obicei, la apă. De ardere
de biocombustibili în motoare, inclusiv turbine, are aceleași cerințe de bază,
dar apare cu mult mai mare rafinament. Astfel de combustie este
posibil în funcție de diferite circumstanțe, de exemplu:

Cu lemne de foc: poziția de lemn, astfel încât focul este conținută în termen de
două sau trei suprafețe de ardere (de exemplu, la sfaturi de trei busteni (clasic
TWIDELL PAGINARE.indb 399
01/12/2014 11:37
400
Bioenergie tehnologii
'trei-foc de piatră'), sau, în cel longitudinal spațiu între trei paralel
busteni.

Cu aschii de lemn sau pelete: hrana combustibil cu banda transportoare sau panta de la un
buncăr de la un relativ mic zonei de ardere, pe care a comprimat
aerul este suflat și de la care ash cade.

Cu generalul lemn și forestiere deșeuri: alimentare combustibil ca mai sus, dar
probabil cu o mișcare sau agitare grătar.

Cu lichid și gazos biocombustibili, arderea trebuie să fie controlate
în cazane și la motoare cu lichizi și gazoși combustibilii fosili, dar cu
diferite fluxul de aer și de combustibil/aer-amestec cerințe.
Arderea contaminate de biomasă (de exemplu, atunci când este amestecat cu materiale plastice,
etc., în deșeurile municipale solide) sau în mai puțin condiții controlate (mai
ales de gătit peste un foc deschis într-un spațiu închis) are considerabil
impact negativ asupra mediului, cu excepția cazului mare grijă este luată.
Peste un milion de decese pe an, de femei și copii în
țările în curs de dezvoltare au fost atribuite bucătărie fumeze, criminalul în
bucătărie'. Îmbunătățirea internă a calității aerului este o motivație majoră pentru
îmbunătățirea sobe de gătit descrise în §10.3.1.
La scară industrială, particulele pot fi eliminate printr-o mai bună
ardere, filtre, cicloane și evacuare condens, care, de asemenea, recuperează
căldura latentă de condens și crește eficiența. Oxizi de azot,
NR.
x

formarea poate fi atenuat prin controlul temperaturii de ardere.


Paie de cereale poate conține relativ concentrații mari de
potasiu și clor, care pot provoca coroziune în cazan grătare; aceasta
poate fi redusă prin instalarea grătarele rotative pentru a preveni o masă solidă de
cenușă de formare. Cu toate acestea, cenușa de la arderea completă de orice
biomasa este întotdeauna un valoros îngrășământ, în special pentru conținutul de fosfați.
Deși ciclul de carbon natural de creștere a plantelor pe deplin reînnoiește
de carbon într-o cultură sau plantație, poate exista o pierdere netă de azot și
, eventual, alte elemente nutritive atunci când biomasa este arsă sau altfel
prelucrate. Care este, de azot nu este returnat în mod suficient la sol
'automat' și trebuie să fie pus înapoi ca un produs chimic de intrare, eventual în formă de
gunoi de grajd sau de rotație cu culturi fixatoare de azot, cum ar fi fasole, trifoi
sau leucaena.
În cele din urmă, vom preamări beneficiile compostarea deșeurilor de biomasă: (a)
substante nutritive si produse pentru ameliorarea solului se întoarcă la sol; (b) de carbon se adaugă la
sol, mai degrabă decât să fie imediat emise în calitate de CO
2

(compostul este un carbon


chiuveta'); sau (c) îngrășăminte artificiale devin inutile.
§10.10.3 Viitor globale bioenergie
Biomasa este o parte importantă a lumii sistemului energetic, în special în
zonele rurale pentru gătit și încălzire. Această dependență va crește pentru
o mai globală durabilă a sistemului energetic, care implică distribuite pe scară largă
TWIDELL PAGINARE.indb 400
01/12/2014 11:37
§10.10 aspectele Sociale și de mediu
401
și versatil resurse, dar folosite mai eficient și în mai multe
moduri moderne. Deja ~35% din ~53 EJ/an de bioenergie folosit
la nivel global pentru energie moderne utilizează (REN21 2012). Până în 2050, biomasa la nivel mondial
consumul de energie poate fi de 500 BQ/y (Chum et al. în IPCC 2011). Cu toate acestea, după cum
sa discutat în §9.8, acest lucru necesită nu numai tehnologiile descrise în
acest capitol, dar, de asemenea, durabilitatea și cadre de politici care să asigure
o bună guvernare de utilizare a terenurilor, îmbunătățiri în silvicultură, agricultură și
gestionarea efectivelor de animale, și mai presus de toate, aprovizionarea cu alimente suficiente.
REZUMAT CAPITOL
Biomasa este un material vegetal, inclusiv a animalelor de deșeuri și reziduuri. Biomasă oferă acum aproximativ 13% din
omenire este consumul de energie, dintre care aproximativ două-treimi este utilizarea de lemn de combustibil în țările în curs de
dezvoltare
pentru gătit și iluminat. Biocombustibilii sunt biomasă prelucrate într-o formă mai convenabilă, în special
combustibili lichizi pentru transport. Termenul de bioenergie acoperă atât biomasă și biocombustibili, referitoare la Capitolul 9
(fotosinteza și biomasă potențial de resurse) și Capitolul 10 (tehnologii pentru producerea și utilizarea
biocombustibililor).
Important principiile generale de bioenergie includ următoarele:

Utilizarea de co-produse și reziduuri de energie sau în alte scopuri (de exemplu, îngrășăminte, compostare).

Producția de biocombustibili este cel mai probabil să fie economică dacă procesul de producție se utilizează materiale deja
concentrată, probabil ca un produs și deci, disponibile la un cost redus sau ca venituri suplimentare pentru
tratarea și eliminarea deșeurilor.

Biocombustibilii sunt materiale organice, astfel încât există întotdeauna alternativa de a folosi aceste materiale ca chimică a
materiilor prime sau materialelor structurale.

Utilizarea sustenabilă a bioenergiei în loc de combustibili fosili scădea emisiile de combustibili fosili dioxid de carbon
și, astfel, reduce forța de schimbările climatice.

Principalele pericole de extinse de combustibil din biomasă utilizare sunt defrișarea pădurilor, eroziunea solului și deplasarea
de culturi alimentare cu combustibil culturi.

Slab controlat de procesare a biomasei sau de ardere pot produce cu siguranță nedorit de poluare
(de exemplu, de a deschide focul pentru gătit, 'criminal în bucătărie').
Principalele bioenergie procese și produse sunt după cum urmează:

Arderea directă de căldură (și de multe ori pentru cogenerarea de energie electrică).

Piroliza (încălzire într-o limitat sau nul de alimentare cu aer) mai ales pentru a produce utile gaze și char.

Digestia anaerobă a deșeurilor biodegradabile construite în digestoare și depozitele de deșeuri pentru producerea de biogaz (un
amestec de CH
4
și CO
2
), utilizate pentru gătit și căldură, căldură de proces, generarea de energie electrică și de export de
gaz de utilități rețeaua de alimentare.

Fermentare de către microorganisme pentru a produce bioetanol (lichid) de combustibil al vehiculelor de zaharuri sau amidon
(prima
generatie, acum comerciale) sau din materiale ligno-celulozice ("a doua generație" din tulpini de plante, etc.)
material.

Biodiesel (combustibil pentru transport) a făcut prin esterificarea uleiurilor vegetale.
Cele mai multe dintre aceste "moderne" cererile au continuat să crească producția de rapid în ultimii 20 de ani.
Astfel, până în 2013, producția de bioetanol a fost >80 GL/y și de biomotorină >20 GL/y; acolo au fost mai mult de
170 de milioane de "îmbunătățit" de lemn sobe de gătit, de 40 de milioane de uz casnic, sisteme cu biogaz, și 6000
industrialscale biogaz sisteme din întreaga lume.
TWIDELL PAGINARE.indb 401
01/12/2014 11:37
402
Bioenergie tehnologii
ÎNTREBĂRI RAPIDE
Notă: Răspunsurile la aceste întrebări sunt în textul de la secțiunea relevantă
din prezentul capitol, sau poate fi ușor dedusă din aceasta.
1
Da o explicație chimică termenului "biomasă".
2
Explica două diferențe între carbon în CO
2
de la arderea cărbunelui
și de la arderea biomasei.
3
Compara căldura de combustie (MJ/kg) de lemn uscat și de
petrol.
4
Pentru un eșantion dat de biomasă, care este mai mare: uscat-baza sau
umed-baza conținutului de umiditate?
5
Ce este un 'peleți din lemn și cât de mare este probabil să fie?
6
Care biocombustibil este cel mai sigur pentru un polițist să bea și de ce?
7
Pentru gătit, care sunt avantajele și dezavantajele de a folosi un
aragaz comparativ cu foc deschis?
8
Cum s-ar putea obține hidrogen din lemn?
9
Listă cu cât mai multe vandabile produse dintr-o trestie de zahăr moara ca tine poate.
10
Ce este "a doua generație" de bioetanol?
11
Care este principalul avantaj al unui Brazilian 'flexi-mașină?
12
Ce este naționale de energie din combustibili fosili raport și de ce este important?
13
Care biomasă culturi energetice și produsele sunt (a), cel mai probabil, și
(b) cel puțin susceptibile de a afecta rezervele de hrană?
14
Ce beneficii poate apărea dacă un digestor anaerob este instalat la o
fermă de vaci?
15
Numele și cuantifica digestia anaerobă intervale de temperatură.
16
Ce se poate întâmpla cu depozitele de gaz?
17
Ce este de biodiesel și în ce fel diferă de bioetanol?
18
Identifica două sociale avantaje și două dezavantaje de utilizarea
biocombustibililor.
PROBLEME
Notă: *indică o "problemă", care este deosebit de potrivit pentru clasa de
discuție sau grupul de tutoriale.
10.1
Un fermier cu 50 de porci a propus utilizarea biogazului generat de
deșeurile lor la putere ferma e motor de masina.
(a) Discuta despre fezabilitatea de a face acest lucru. Tu ar trebui să calculeze cât
conținutul de energie al gazului și a energiei utilizate în
comprimarea gazelor la un volum utilizabil, și comparați-le cu
energia necesară pentru a rula masina.
(b) un Scurt comentariu asupra a ceea ce alte beneficii (dacă există) ar putea fi
dobândită prin instalarea unui digestor.
TWIDELL PAGINARE.indb 402
01/12/2014 11:37
Probleme
403
S-ar putea presupune că o 100 de kg de porc excretă aproximativ 0,5 kg de
solide volatile (VS) pe zi (plus 6 kg de apă), și că 1 kg de VS randamentele
0,4 m3 de biogaz de la STP.
10.2
Studiile arată că marile consumul de energie în Fijiană sate
este lemnul care este folosit pentru gătit la foc. Tipic
consumul de lemn este de 1 kg de persoana
-1
zi
-1
.
(a) Estimarea energie termică necesară pentru a fierbe 2 litri vas plin de
apă. Presupunând că acest lucru să fie gătit cerință de fiecare
persoană, compara acest lucru cu căldură conținut de lemn, și
, astfel, să estimeze eficiența termică a deschide focul.
(b) Cât de mult lemn trebuie să fie tăiați în fiecare an pentru a găti pentru o
sat de 200 de persoane?
Presupunând sistematică de replantare, ce zona de cultură trebuie
, prin urmare, sat deoparte pentru utilizarea combustibilului, dacă aceasta nu este de a face un net
defrișărilor? Hint: consultați Tabelul 10.4.
(c) Comentariu pe realismul ipotezelor, și să revizuiască
estimările dvs. în mod corespunzător.
10.3
(a) Un cauciuc butilic geanta din volumul total 3.0 m
3
este folosit ca o instalație de biogaz
digestor. Fiecare zi este alimentat de o intrare de 0.20 m
3

de suspensie, de
care 4.0 kg este volatile solide, și un volum corespunzător de
digerat suspensie este eliminat. (Această intrare corespunde aproximativ la
deșeurile provenite de la 20 de porci.)
Presupunând că o reacție normală, în procesul de digestie este bacterii:
C
12
H
22
O
11
+H
2
O
→ 6CH
4
+ 6CO
2

și că reacția are șapte zile pentru a finaliza, se calculează:


(i) volumul de gaze; (ii) de căldură obținut prin arderea de
gaz pentru fiecare zi de funcționare a digestor; și (iii) cât de
mult kerosen ar avea aceeași valoare calorică ca o zi de
biogaz.
(b) rata De reacție în digestorul poate fi aproape dublat prin
creșterea temperaturii de suspensie de la 28°C (ambient) pentru
35°C. (i) care ar fi avantajul de a face acest lucru? (ii) Cât de
mult de căldură pe zi ar fi necesare pentru a realiza acest lucru? (iii) Ce
proporția de acest lucru ar putea fi contribuit de căldură a evoluat în
digestia reacție?
10.4
(a) Scrie un echilibrat ecuație chimică pentru conversia
zaharoză (C
l2
H
22
O
11
) pentru etanol (C
2
H
5

OH). Utilizați această pentru a calcula


cât de mult etanolul poate fi produs în teorie, dintr-o tonă
de zahăr. Ce crezi că ar fi un realist randament?
TWIDELL PAGINARE.indb 403
01/12/2014 11:37
404
Bioenergie tehnologii
(b) Fiji este o țară mică, în Pacificul de Sud, a cărui principală de export
cultura este zahăr. Fiji produce 300.000 de tone/an de zahăr, iar importurile
de 300.000 t de fosile de combustibil de petrol. Dacă toate acest zahăr au fost
convertite în etanol, ce procent din importurile de petrol ar putea
înlocui?
10.5
Ia în considerare o gramada de green wood chips-uri la 60% conținut de umiditate
(stare umedă) și o greutate de 1 tonă. Ceea ce este la cuptor-masa uscată a
biomasei din gramada?
Biomasa are o căldură de ardere de 16 MJ pe cuptor-uscate kg.
Aceasta este puterea calorifică superioară', corespunzătoare puterii termice
într-o reacție de tipul:
[CH
2
O] +O
2
→ CO
2
(gaz) + H
2
O (liq)
Valoarea calorifică netă (sau 'calorifică inferioară') este căldura evoluat
când ultima apă este gazos; în practică, acest lucru este de maximă
energie termică disponibile pentru utilizare atunci când biomasa este arsă.
(i) În care gramada este lăsat să se usuce la 50% conținut de umiditate (umedă),
atunci când se pare mult la fel, dar are mai puțină apă în ea.
(ii) În care gramada este lăsat să se usuce timp de câteva săptămâni, și ajunge la 20%
m.c. (w.b.), la care etapă a scăzut un pic în volum și
foarte mult în masă.
Pentru fiecare situație calculați masa totală a gramada, net de căldură,
de energie disponibile la arderea gramada, și puterea calorifică netă
per umed kg.
Următoarele întrebări sunt deosebit de potrivite pentru clasa de
discuție:
10.6*
'Alimentate cu biocombustibili' este numele unui program de televiziune arată
familia ta de viață de grup în întregime pe resursele biologice;
descrie cum o astfel de familie ar putea trăi.
10.7*
Lista cu cele mai importante cinci motive pentru ce " și "de ce nu'
comerciale de energie din biomasă ar trebui sau nu ar trebui să crească. Discuta
aceste motive.
NOTE
1
Acest "biologic" carbon are o proporție mai mare de izotopi
14
C decât carbonul din combustibilii fosili; acest lucru permite iso-
subiect de analiză de aer pentru a clarifica proporția atmosferice de CO
2
asta este din combustibili fosili (a se vedea Caseta 2.3).
2
Aici, termenul de 'bioetanolul reprezintă etanol făcute de către oricare dintre rute (1) - (4) din §10.6.1, dar unii autori
folosesc termenul în sensuri diferite. Rețineți că de mult alcool industrial este realizat din fosile de petrol.
3
De 'echilibrul energetic calculele prezentate aici pot apărea similare cu cele ale energiei analizei de Leach
(1976), Kumar și Twidell (1981) și Twidell și Pinney (1985). Cu toate acestea, aici restrângem analiza
TWIDELL PAGINARE.indb 404
01/12/2014 11:37
Bibliografie
405
la numai combustibili fosili intrări, cu biomasa de creștere și producția de biocombustibili toate într-o anumită țară.
Prin urmare, analiza este orientativă, dar încă foarte relevant pentru politica.
BIBLIOGRAFIE
Sinteze
Chum, H. L. și Overend, R. P. (2003) 'de Biomasă și bioenergie, în Statele Unite ale americii', Progresele în domeniul Energiei
Solare,
15, 83-148. Revizuire cuprinzătoare comerciale și aproape tehnologii comerciale, și sprijinirea politicilor
și R&D, cu accent pe mari și mijlocii.
AIE Bioenergie (2009) Bioenergie – O Durabilă și de Încredere Sursă de Energie: O revizuire a statutului și perspective,
Organizația Internațională pentru Energie, Paris (disponibil online la http://www.ieabioenergy.com/. Cuprinzătoare
de revizuire care vizează factorii de decizie politică', deci nu foarte detaliate din punct de vedere tehnic.
IPCC (2011) (H. Chum, A. Faaij, J. Moreira, G. Berndes, P. Cea, H. Dong, B. Gabrielle, A. Goss Eng, W. Layng,
M. Mapako, O. Masera Cerutti, T. McIntyre, T. Minowa, K. Pingoud), 'Bioenergie', ch. 2 din O. Edenhofer, R.
PichsMadruga, Y. Sokona, K. Seyboth, P. Matschoss, S. Kadner, T. Zwickel, P. Eickemeier, G. Hansen, S. Schlömer
și C. von Stechow (eds), IPCC Raport Special privind Sursele de Energie Regenerabile și Atenuarea efectelor schimbărilor
Climatice
Cambridge University Press, Cambridge. Recent autoritate revizuire a biomasei, a resurselor și o gamă completă
de bioenergie tehnologii (disponibil online la http://srren.ipcc-wg3.de/report).
Klass, D. L. (1998) Biomasă pentru Energia Regenerabilă, Căldură și substanțe Chimice, Academic Press, London. O extrem de
cuprinzătoare și de încredere text. Bazat pe principii chimice, dar conștienți de toate disciplinele corespunzătoare, inclusiv
în economie. Folosește S. I. de unități.
Sims, R. E. (2002) Strălucirea de Bioenergie în Afaceri și în Practică, James & James, Londra. Iluminare
text cu accent asupra producției industriale moderne și aplicații; include numeroase ilustrate cu studii de caz
de sisteme de putere, inclusiv cu biogaz.
Estimările de resurse
A se vedea bibliografia de la Capitolul 9.
Arderea directă, în special lemn de combustibil
De Lepeleire, G., Prasad, K. K., Verhaart, P. și Visser, P. (1981) O Soba Compendiu, Eindhoven
University, Olanda. Oferă principiile de lemn ars, și descrieri tehnice de mai multe sobe conceput pentru
aparate de gătit în țările în curs de dezvoltare.
Kammen, D. M. (1995) 'Cookstoves pentru lumea în curs de dezvoltare', Scientific American, 273, 64–67.
Wahlund, B., Yan, J. și Westermark, M. (2004) Creșterea biomasei, utilizarea în sistemele de energie: un
studiu comparativ al CO
2

și reducere a costurilor pentru diferite bioenergie opțiuni de procesare', Biomasă și Bioenergie, 26,


531–544. Se concentrează pe lemn în Suedia; concluzionează că pelletization pentru cărbune de substituție este mai bună opțiune
în acest
caz.
Banca mondială (2010) Îmbunătățirea Cookstoves și o Sănătate Mai bună în Bangladesh: Lecții de energie pentru uz casnic și
salubritate programe (raport final). Banca Mondială, Washington. Recenzii multe alte programe în afară de cei din
Bangladesh.
TWIDELL PAGINARE.indb 405
01/12/2014 11:37
406
Bioenergie tehnologii
Biocombustibili
Cheng, J. J. și Timilsina, G. R. (2011) Statutul și bariere de biocombustibil avansat tehnologii: o revizuire', din surse
Regenerabile
de Energie, 36, 3541-3549. Review util, mai ales pe lignocellusosic etanol și alge.
Goldemberg, J. (2007) 'Etanol pentru un viitor energetic durabil', Știință, 315: 808–810.
Keshgi H. S., Prințul, r. C. și Marland, G. (2000) 'potențialul de biomasă combustibili în contextul climatice globale
schimba: se concentreze pe combustibili de transport', Revizuirea Anuală de Energie și Mediu, 25, 199-244. Larga
de fundal, plus energia analiza de bioetanol din SUA și Brazilia.
Marc Londo
,

et al. (2010) 'REALIMENTARE foaie de parcurs a UE pentru biocombustibili în transporturi: Aplicarea


proiectului instrumente pentru
unele pe termen scurt probleme de politică', Biomasă și Bioenergie, 34, 244–250.
Mousdale, D. (2010) Introducerea de Biocombustibili, CRC Press, Londra. Deosebit de bun în biochimie implicate,
și probabil costurile de rute diferite.
Wyman, C. E. (1999) 'Biomasei în etanol: progresul tehnic, oportunități și provocări comerciale', Anual de
Revizuire a Energiei și Mediului, 24, 189–226. Subliniază potențialul noilor tehnologii pentru a produce
etanol din celuloză, (de exemplu, "deșeuri" din culturi alimentare).
Zhang, Y. (2010) 'Hidrotermale de lichefiere pentru conversia biomasei în țiței', ch. 10 în Biocombustibili din Agricultură
Deșeuri și produse Secundare, H. P. Blaschek, T. C. Ezeji și J. Scheffran (eds), Blackwell Publishing, Oxford.
Zuubier, P. și van der Vooren, J. (eds) (2008) Trestie de zahăr Etanol: Contribuții la atenuarea schimbărilor climatice și
mediul, Wageningen Academic Publishers, Olanda. Multi-autor de muncă, concentrându-se pe Brazilian
experiență.
Biogaz
Chynoweth, D. P., Owens, J. M. și Legrand, R. (2001) 'Regenerabile de metan de la fermentarea anaerobă a
biomasei', Energie Regenerabilă, 22, 1-8. Susține digestia anaerobă ca principala cale pentru utilizarea de
culturi energetice.
Meynell, P. J. (1976) Metan – Planificare-un Digestor, Prism de Presă, Dorchester, marea BRITANIE. Un vechi, dar încă util
scurt
și practice carte cu tehnică de bază și biochimice explicații.
Muller, C. (2007) Digestia Anaerobă a Biodegradabilă a Deșeurilor Solide în mici și cu venituri medii Țări,
EAWAG acvatice cercetare, Elveția (disponibil la www.eawag.ch). Recenzii sisteme pentru uz casnic și
deșeurile municipale.
Van Buren, A. (1979) Un Chinez De Biogaz Manual, Tehnologie Intermediară Publicații, Londra. O stimularea
și manual util, bazat pe o experiență considerabilă de mici dimensiuni digestoare în China rurală. Retipărită
de mai multe ori.
Wellinger, A., Murphy, J. și Baxter, D. (eds) (2013) Biogazul Ghid: Știință, producție și aplicații,
Woodhead Publishing, Cambridge. Un grav inginerie monografie.
Referințe specifice
Alonso-Pippo, W., Luengo, C. A., Alberteris, L. A. M., del Pino, G. C. și Duvoirsin, S. (2013) Practic
punerea în aplicare a biocombustibili lichizi: transferabilitatea Brazilian experiențe, Politici Energetice, 60, 70–80.
TWIDELL PAGINARE.indb 406
01/12/2014 11:37
Bibliografie
407
Kumar, M. și Twidell, J. W. (1981) 'analiza Energetică a unor trestia de zahăr, ferme în Fiji: având în vedere muncii și
utilizare masina', a Energiei, 6, 139–152.
Leach, G. (1976) de Energie și Producția de Alimente., IPS Știință și Tehnologie de Presă de la Institutul Internațional
pentru Mediu și Dezvoltare, Guildford. Seminale devreme text pe analiza de energie'.
Macedo, I. C., Seabra, J. E. și Silva, J. E., (2008) 'emisiile de gaze de Seră în producția și utilizarea de etanol
din trestie de zahăr în Brazilia: 2005/2006 medii și o predicție pentru anul 2020', Biomasă și Bioenergie, 32,
582-595.
Meynell, P. J. (1976) Metan – Planificare-un Digestor, Prism de Presă, Dorchester, marea BRITANIE. Un vechi, dar încă util
scurt
și practice carte cu tehnică de bază și biochimice explicații.
Persson, T., Garcia, A., Paz, J., Jones, J. și Hoogenboom, G. (2009) 'Net de energie valoarea de bioetanol din porumb,
ca răspuns la diferite condiții climatice și de sol din sud-estul SUA, Biomasă și Bioenergie, 33,
1055-1064.
REN21 (2012) surse Regenerabile de energie 2012: Global status report, Energie Regenerabilă, Politica de Rețea pentru secolul
21,
de la Paris. Raportul este actualizat anual; ultima este la www.ren21.net.
Twidell, J. W. și Pinney, A. A. (1985) 'calitatea și exergetică a sistemelor energetice, folosind convenționale și
resurse regenerabile', în L. F. Jesch (ed.), Soare-la-locul de Muncă în marea Britanie, marea BRITANIE-ISES. Comentarii
privind analiza de energie'.
Van Buren, A. (1979) Un Chinez De Biogaz Manual, Tehnologie Intermediară Publicații, Londra. O stimularea
și manual util, bazat pe o experiență considerabilă de mici dimensiuni digestoare în China rurală. Retipărită
de mai multe ori.
Wang, M. Q., Han, J., Haq, Z., Tyner W. E., Wu M. și Elgowainy, A. (2011) de Energie și gaze cu efect de seră
emisii efecte de porumb și etanol celulozic cu tehnologie de îmbunătățiri și schimbări în utilizarea terenurilor', Biomasă și
Bioenergie, 35, 1885-1896.
Reviste și site-uri web
Biomasă și Bioenergie, lunar, Elsevier. Acoperă o gamă largă de știință de bază și aplicații.
http://journeytoforever.org. Un ghid pentru do-it-yourself - biodiesel.
www.biodiesel.org. Elementele de bază de biodiesel tehnologie, și o mulțime de link-uri începând de știri și evoluții;
accent pe statele UNITE ale americii.
www.ieabioenergy.com. Rapoarte de colaborare internațională de cercetare în tehnologie și politică.
www.hedon.info. De uz casnic de Energie în Rețea, cu prim accent pe țările în curs de dezvoltare, reunește
informații pe biogaz, lemn, sobe de gătit, și subiecte conexe.
www.Ren21.net. Anual rapoarte de date de piață și a politicii cu privire la toate formele de energie din surse regenerabile,
inclusiv cu bioenergie.
www.iea/org/statistics/. Statisticile în domeniul energiei liber accesibile on-line, pentru toate țările industrializate și cele mai mari
țări în curs de dezvoltare, din 1990 până în cel mai recent an disponibil.
http://greet.es.anl.gov/main. GREET model (gaze cu efect de Seră, Reglementate Emisiile și consumul de Energie în
Transport), dezvoltat și actualizat în permanență de Argonne National Laboratory (SUA) permite cercetătorilor și
analiștilor pentru a evalua diferite vehicule și de combustibil combinații pe un plin de combustibil pe ciclu/vehicul baza
ciclului. Modelul este liber,
disponibil pentru descărcare și se bazează pe o bază de date extinsă de NOI, agricole și inginerie practică.
TWIDELL PAGINARE.indb 407
01/12/2014 11:37

Val de putere
CONȚINUTUL
Obiective de studiu
409
§11.1 Introducere
410
§11.2 Val de mișcare
413
§11.3 Val de energie și putere
417
§11.3.1 Derivare: de Energie în
valul la o anumită locație 417
§11.3.2 Formule pentru putere de extracție
din valuri
418
§11.4 Real (neregulate) valurile mării:
modele și putere
421
§11.5 extragerea de Energie din dispozitive
427
§11.5.1 Clasificarea dispozitivelor
427
§11.5.2 Captura lățime și reglate
captarea energiei
428
§11.6 Val de dispozitive de putere
430
§11.6.1 Pe malul terminator,
Tapchan, creativitate val
de captare
430
§11.6.2 Wave Dragon: plutitor
creativitate terminator
431
§11.6.3 Oscilant coloana de apă
(OWC) terminator: prima
generație de la țărm
și în apropiere de țărm
432
§11.6.4 Pelamis atenuator, offshore,
a doua și a treia generație
436
§11.6.5 Rezumat al operaționale
dispozitive
437
§11.7 Sociale, economice și
aspecte de mediu
437
Rezumat capitol
439
Întrebări rapide
440
Probleme
440
Note
442
Bibliografie
443
Confirmare
444
Caseta 11.1 prin Satelit de măsurare
de înălțime a valurilor, etc.
423
Caseta 11.2 energiei Valurilor în marea BRITANIE
430
Cutie 11.3 teoria de Bază a unui dispozitiv OWC
434
CAPITOLUL

11
TWIDELL PAGINARE.indb 408
01/12/2014 11:37

www.shahrsazionline.com
Lista de figuri
409
OBIECTIVE DE STUDIU

Apreciez mari de energie fluxurilor și
formidabil condiții de valurile mării.

Analiza și evalua propagarea undelor în
termeni de lungime de undă, val înălțimea, frecvența
și perioada.

Să fie conștienți de caracteristicile hidrodinamice
de valuri și valurile de extracție.

Cunoască principalele clase de val de putere
dispozitive.

Apreciez modul de succes dispozitive absorbi
energia valurilor la o mai mare distanță decât
propria lățime, prin urmare, definind termenul de captare
lățime.

Aprecia dezvoltarea
comercială de energie electrică din prototip
dispozitive.
LISTA DE FIGURI
11.1 (a) numărul Maxim de înălțime a valurilor de la nivel mondial; (b) Media anuală a energiei valurilor.
411
11.2 Particule în mișcare valuri de apă.
413
11.3 Apă de suprafață perpendicular pe rezultanta gravitațională și forța centrifugă ce acționează
pe un element de apă, de masă m. 414
11.4 Val caracteristici.
414
11.5 Rezultanta forțelor de pe suprafața particulelor.
415
11.6 Accelerații și viteze de apă de suprafață a particulelor.
415
11.7 Elementar mișcare de apă într-o apă adâncă de val.
417
11.8 Local fluctuațiile de presiune în val.
419
11.9 Simulat val-înălțimea record.
422
11.10 (a) Val-putere harta pentru marea de vest a Norvegiei din satelit derivate de date.
(b) Conturul medie val de energie pe Nord-vestul Europei.
424
11.11 de Distribuție de energie pe frecvență interval într-un tipic Atlantic de apă adâncă model de val.
425
11.12 Ilustrative diagramă de dispersie de valuri cu o înălțime semnificativă H
s
împotriva trecerea prin zero perioada T
z
. 426
11.13 Val dispozitiv de clasificare prin metoda de captare a energiei valurilor.
427
11.14 diagrama Schematică a Tapchan val de energie a plantelor.
431
11.15 Wave Dragon plutind val dispozitiv de putere.
433
11.16 diagrama Schematică de un val mal sistem de alimentare folosind un oscilant coloana de apă.
434
11.17 Simplist model de oscilant coloana de apă, val dispozitiv de putere.
435
11.18 Schiță curba arată puterea extras dintr-un oscilant coloana val dispozitiv de putere
ca o funcție de amortizare în aer turbina.
435
11.19 Pelamis atenuator val dispozitiv de putere.
436
11.20 turbinei Wells.
441
11.21 (a) O clapeta cu balamale oscilează ca valurile afecteze din stânga; (b) un mod mai eficient dispozitiv
(Salter e "rață").
442
TWIDELL PAGINARE.indb 409
01/12/2014 11:37
410
Val de putere
§11.1 INTRODUCERE
Foarte mari de energie fluxurile pot apărea în apă adâncă a mării valuri. Este arătat
în §11.3 de mai jos că puterea pe unitatea de lățime în astfel de valuri este dat de:
r
p
'=

P
gHT
HT
32
22
2
(11.1)
Prin urmare valuri cu perioadă lungă de timp T (
~10 s) și mari crest-să-jgheab înălțime
H (
~4 m) au energie fluxurilor de obicei o medie între 50 și
70 kW pe metru de lățime de sens opus val, care le face de
un interes considerabil pentru generarea de energie. Fig. 11.1 indică val de energie
de distribuție în oceane și continental al coastelor cu substanțiale
val de resurse energetice.
Posibilitatea de a genera putere utilă de valuri a fost
realizat de mai mulți ani, și există nenumărate idei pentru masini pentru a extrage
puterea, cu, probabil, cea mai veche brevet în 1799 și o mai devreme electrice
dispozitiv de putere în 1909 în California pentru port de iluminat. Interesul moderne
a fost spasmodic din anii 1970, mai ales în Japonia, marea BRITANIE, Scandinavia și
India, dar încet-încet, începând cu anul 2000, un număr tot mai mare de dispozitive,
fiind dezvoltat pentru utilizarea în scopuri comerciale a devenit conectat la utilitățile,
mai ales în marea BRITANIE și în alte țări Europene cu coastele mării și
cu favorabile tarifele feed-in pentru a curăța și durabilă a energiei regenerabile.
Foarte mici dimensiuni sisteme autonome sunt fabricate in mod curent pentru
marin lumini de avertizare pe geamanduri dar mult mai mari dispozitive de rețea de alimentare
generație necesită inițial guvernul R&D de finanțare.
Mediul marin este intensă, atât de mic (kW scară)
waveenergy dispozitive pentru generarea rețelei de energie electrică nu sunt avute în vedere
(spre deosebire de majoritatea celorlalte surse de energie regenerabile); prezent "comerciale" dispozitive
genera
la circa 100 kW la 1 MW de dispozitive modulare, fiecare captarea de energie
de la 5 la 75 m de val frontal. Inițial dispozitive funcționează la
țărm sau plutesc aproape de țărm pentru un acces mai ușor și mai puțin violente mărilor.
R&D este facilitat foarte mult la wave hub-uri' au mal pe bază de facilități
și un off-shore plutitoare hub de conexiune electrică. Până în 2013 nu
au fost de cel puțin 20 de concurente comerciale tehnologii în dezvoltare
funcțiune și în utilizare comercială la nivel mondial, cea mai mare parte de semnificativ
diferite de design; va fi de cel puțin un deceniu înainte de 'favoriți' devenit
stabilit.
Este important pentru a aprecia numeroasele provocări cu care se confruntă val de putere
evoluții. Acestea vor fi analizate în capitolele următoare, dar sunt
rezumate aici:
1
Val modele sunt neregulate în amplitudine, fază și direcție. Este
o provocare de a proiecta dispozitive pentru a extrage energia în mod eficient de-a lungul
gamă largă de variabile.
2
Valuri de apă sunt analizate de către hidrodinamică (sensul literal apa
circulație'), care este o subdisciplină de mecanica fluidelor, susținută
TWIDELL PAGINARE.indb 410
01/12/2014 11:37
§11.1 Introducere
411
Fig. 11.1
o
Maxim de înălțime a valurilor de nivel mondial pe o perioadă de 20 de ani, indicând regiuni
semnificative a valurilor de resurse (prin satelit altimetrie date a fuzionat prin Ifremer și
cartografiate de CLS pentru a învăța.eo proiect).
b
Medie anuală val de energie (MWh.m
-1
) pentru unele regiuni de coastă.
Sursa: Adaptat de la NEL (1976).
(a)
(b)
535
300
455
370
405
465
415
340
335
420
420
de specialitate experimente de laborator și de modelare computațională.
(Notă: aceste analize sunt prea specializate și sofisticate pentru a fi
incluse în Revizuire 2.) Pentru mare, analiza este complicată de
distribuție de frecvențe de undă în locație și de timp. Aceste efecte
TWIDELL PAGINARE.indb 411
01/12/2014 11:37

412
Val de putere
sunt modelate pentru a obține forțele dinamice pe val-dispozitive de energie
și structuri care trebuie să supraviețuiască timp de cel puțin 25 de ani. O parte cheie a
hidrodinamice analiza este capacitatea de a succes un dispozitiv pentru a trage
puterea în sine de la o regiune mai mare mare decât propria sa amprentă; o
măsură de această abilitate este dispozitivul de captare lățime (§11.5).
3
Există întotdeauna o probabilitate extrem de furtuni sau uragane care produc
valuri de ciudat intensitate. Val-dispozitive de putere trebuie să fie capabil de a
supraviețui în astfel de condiții. De obicei, la 50 de ani de vârf de val este de 10
ori înălțimea medie val. Astfel de structuri trebuie să
supraviețuiască în mările cu
De ~100 de ori puterea de intensitatea cu care acestea sunt
în mod normal potrivite. Pentru a permite acest lucru necesită un design sofisticat și
de testare, și adaugă foarte mult la costul inițial de val-sisteme de alimentare.
4
Puterea de vârf este, în general, disponibile în apă adâncă de valurile din larg
se umfla produs din timp aducerile predominantă a vântului (de exemplu, dincolo de
Insulele Vestice a Scoției în una dintre cele mai furtunos zone din
oceanul Atlantic de Nord și în regiunile din Oceanul Pacific). Dificultățile
de a construi dispozitive de putere pentru aceste tipuri de val regimuri, de
menținere și fixare sau ancorare-le în poziție, și de a transmite
puterea de a terenurilor, sunt formidabile. Prin urmare, mai protejate și
zonele accesibile, în apropiere de țărm sunt folosite pentru dezvoltarea de prototipuri și
inițial de comercializare.
5
Val de perioade sunt, de obicei,
~5 la 10 s (frecvență de ~0.1 Hz). Este
o provocare pentru acest cuplu neregulate slow motion pentru generarea de energie electrică
la
~500 de ori mai mare frecvență.
6
Mai multe tipuri de dispozitive au fost sugerate pentru val-putere
de extracție și astfel sarcina de a selecta și de a dezvolta o anumită metodă
a fost oarecum arbitrară. Devotamentul și capacitatea de pionier
ingineri și finanțatorii sunt vitale pentru succes.
7
Dezvoltarea și aplicarea de val de putere au avut loc cu
spasmodic și schimbarea guvernului interes, în mare măsură, fără a
beneficia de stimulente de piață. Val de putere are nevoie de același învățare
curba de extinderea în mod constant cereri de la începuturi mici, care a
avut loc cu energie eoliană.
La avantajele distinctive ale val de putere sunt mari de energie fluxuri
disponibile și previzibilitatea de val condiții pe perioade de zile
înainte. Valurile sunt create de vânt, și în mod eficient a stoca energie pentru
transport pe distanțe mari. De exemplu, valuri mari care apar
pe Europa va fi fost inițiat în vreme de furtună în mijlocul Atlanticului
sau cât mai departe în Caraibe.
Următoarele secțiuni scopul de a oferi o bază generală pentru înțelegerea
val de dispozitive de energie. În primul rând, vom prezenta teoria de adâncime-valuri de apă
și se calculează energia fluxurilor disponibile într-o singură frecvență. Apoi
vom revizui modele de valuri care au loc, de fapt. În cele din urmă vom
descrie val-dispozitive de putere si dezvoltarea comerciala a acestora.
TWIDELL PAGINARE.indb 412
01/12/2014 11:37
§11.2 Val de mișcare
413
§11.2 VAL DE MIȘCARE
Cele mai multe-val energie dispozitive sunt proiectate pentru a extrage energie de la
deepwater valuri. Aceasta este cea mai comună formă de val, a fost găsit atunci când
adâncimea medie a mării D este mai mult decât jumătate din lungimea de undă
(
l). De exemplu, o medie val de mare pentru generarea de energie poate fi
de așteptat să aibă o lungime de undă de
~100 m și amplitudinea ~1 m sau
mai mult, și să se comporte ca un adânc val de apă la adâncimi de fundul mării mai mare
decât
~30 m. Chiar și în ușor adâncimi mai mici, în cazul în care mai multe tipuri de
dispozitivul funcționează acum, teoria este o bună aproximare.
Fig. 11.2(a) ilustrează mișcarea particulelor de apă într-o apă adâncă de
val. Circular de particule de mișcare are o amplitudine care scade
exponențial cu adâncimea și devine neglijabil pentru D >
l / 2. În mai puțin adâncă
apă (Fig. 11.2 (b)), mișcarea devine eliptică și de circulație a apei
are loc pe fundul mării, producătoare de disipare a energiei.
Proprietățile de valuri de apă adâncă sunt distincte, și pot fi
rezumate după cum urmează:
1
Valuri de suprafață sunt seturi de neîntreruptă sine valuri neregulate
lungime de undă, fază și direcție.
2
Moțiunea de orice particule de apă este circulară. Întrucât suprafața
formă de val arată o certă progresie, particulele de apă
nu au net progresie.
3
Apa de pe suprafața rămâne pe suprafață.
4
Amplitudinile de apă de particule propuneri descrește exponențial
cu adâncimea. La o adâncime de
l/2p sub medie suprafață poziție,
amplitudinea este redusă la
l/e de suprafața de amplitudine (e = 2.72, de bază
de logaritmilor naturali). La adâncimi de
l/2 propunerea este neglijabil, fiind
mai puțin decât 5% din suprafața de mișcare.
5
Amplitudinea - o de suprafața de undă depinde în principal pe istoria
de vânt regimuri deasupra suprafeței, dar este puțin dependentă de
Fig. 11.2
Particule în mișcare valuri de apă:
o
apă adâncă, mișcare circulară de particule de apă;
b
apă de adâncime mică, mișcare eliptică de particule de apă.
D
(a)
2p
am
D
(b)
TWIDELL PAGINARE.indb 413
01/12/2014 11:37
414
Val de putere
lungimea de undă
l, viteza c și perioada T. Este rar pentru amplitudinea
pentru a depăși o zecime din lungimea de undă, cu toate acestea.
6
Un val se va rupe în alb de apă atunci când panta de suprafață este
de aproximativ 1 la 7, și, prin urmare disipa energia potențială.
DERIVAREA UNOR FORMULE CHEIE PENTRU APĂ ADÂNCĂ' VALURI
Analiza formală de valuri de apă este dificil, dar cunoscute, a se vedea Coulson și Jeffrey (1977) pentru standard
teorie. Pentru deep-valuri de apă (de asemenea, numit 'undele de suprafata'), frecare, tensiunea superficială și forțele de inerție
sunt mici în comparație cu cele două dominante forțele de greutate și mișcare circulară. Ca urmare, apa
de suprafață are întotdeauna o formă astfel încât tangenta se află perpendicular pe rezultanta acestor două forțe
(Fig. 11.3).
Este de cea mai mare importanță pentru a realiza că nu există nici un net de mișcare a apei în adânc-valuri de apă.
Obiecte suspendate în apă arată propuneri ilustrat în Fig. 11.2(a) în apă adâncă și (b) în
ape mai puțin adânci.
O particulă de apă în suprafață are o mișcare circulară de rază de un egală cu amplitudinea undei
(Fig. 11.4). Val înălțimea H de la partea de sus de o creasta la partea de jos a unui jgheab este de două ori amplitudinea:
H
= 2o. Viteza unghiulară de particule de apă este w (radian pe secundă). Valul de suprafață are o formă
care progreseaza ca un val în mișcare, deși apa în sine nu progres. De-a lungul direcției de
mișcare val în mișcare formă rezultate din diferentele de faza în mișcare succesive de particule
de apă. Ca o particulă în creasta picături într-o poziție inferioară, o altă particulă într-o poziție înainte
cercurile de sus pentru a continua creasta forma și transmite mișcare de val.
Rezultanta forțelor F pe o suprafață de apă de particule de masa m sunt indicate în Fig. 11.5. Apa
de suprafață ocupă poziția produs de această rezultantă, astfel încât tangenta la suprafață este
perpendiculară F. O particulă în partea de sus de o creastă, poziția P1, este aruncat în sus de forța centrifugă
ma
w
2
. Un moment mai târziu, particula este în scădere, și poziția în creasta este luată de către un vecin
Fig. 11.3
De apă de suprafață perpendicular pe rezultanta gravitațională și
forța centrifugă care acționează pe un element de apă, de masă m.
Direcția val
Val
de suprafață
F
Val
suprafața
tangentă
Rezultanta F
dl.
ω
2
mg
Fig. 11.4
Caracteristici Wave
Val de mișcare
h
x
o
H = 2o
ω
λ
TWIDELL PAGINARE.indb 414
01/12/2014 11:37
§11.2 Val de mișcare
415
particule de rotație cu o întârziere de fază. La P2 o particulă este la nivelul mediu de apă, iar suprafața
se orientează perpendicular pe forță rezultantă F. La jgheab, P3, forță descendentă este maximă.
La P4 particula are aproape finalizat un ciclu complet de mișcare sale.
Accelerațiile de o suprafață de particule sunt desenate în Fig. 11.6. (b). Inițial t
= 0, particula este la
nivelul mediu de apă, și ulterior:
φp
w
=
− t
( / 2)
și
(11.2)
w
φ
w
φ
w
φ
=
+

s
o
go
o
g
tan
păcat
pentru că
păcat
2
2
2
(11.3)
întrucât, în practică, g >> un
w
2
pentru non-rupere valuri (de exemplu, un
= 2 m, T (perioada) = 8 s, o
w
2
= 1.2 ms
-2
și
g
= 9.8 ms
-2
). Lasă - h fi înălțimea suprafeței de mai sus de nivelul mediu. Panta tangentei la
suprafața este dat de:
=
h
x
s
d
d
tan
(11.4)
Din (11.2), (11.3) și (11.4),
w
φ
w

w
w
=
=





=
h
x
o
g
o
g
o
g
t
d
d
păcat
pentru că
2
pentru că
2
2
2
(11.5)
Din Fig. 11.5(c), vertical viteza particulelor este:
w
φ
w
w
=−
=−
h
t
o
o
t
d
d
păcat
pentru că
(11.6)
Fig. 11.5
Rezultanta forțelor de pe suprafața particulelor.
Direcția de propagare a undei
Circular de particule
mișcare
P1
P3
P2
P4
F
F
F
F
ma
ω
2
ma
ω
2
ma
ω
2
ma
ω
2
mg
mg
mg
mg
Fig. 11.6
Accelerații și viteze de apă de suprafață a particulelor:
o
de apă de suprafață;
b
accelerarea particulelor; general derivare;
c
viteza particulelor.
Înălțimea deasupra
nivelul mediu, sec
(a)
(b)
(c)
Val
direcția x
Rezultanta
accelerare
păcat
φ
g
g
s
s
o
ω
2
wt
wt
o
ω
2
o
ω
o
ω
2
φ
φ
φ
TWIDELL PAGINARE.indb 415
01/12/2014 11:37
416
Val de putere
Soluția din (11.5) și (11.6) este:
ho
g
t
păcat
x
2
w
w
=





−
(11.7)
Comparând acest lucru cu generalul val de călătorie ecuația de undă
am și viteza c, vom obține:
π
λ
π
ω
ω
=

=
λ





=

ho
x ct
o
x
t
o
kx
t
păcat
2
(
)
păcat
2
sin(
)
(11.8)
unde k
= 2p/
l se numește număr de undă.
Suprafața de mișcare, prin urmare, pare a fi o undă progresivă, cu:
p
w
l=
g
2
2
(11.9)
Această ecuație este important; se dă relația între frecvența și lungimea de undă de
adâncime valuri de suprafață.
Perioada mișcării este T
= 2p/w = 2p/(2pg/l)
1/2
. Deci:
g
p
=
am




T
2
1
2
(11.10)
Viteza unei particule la creasta de val este:
w
p
=
=
am




vo
o
g
2
1
2
(11.11)
Valul de suprafață viteza in x direcție, de la (11.8), este:
w
pw
p
=
am
=
=√
am




c
g
g
g
2
2
am
p
p
=



=
c
g
gT
de exemplu
2
2
1
2
(11.12)
Viteza c este numită fază viteza de deplasare a undei făcut de suprafața de mișcare. Rețineți că
faza viteza c nu depinde de amplitudine - o, și nu este legată în mod evident de particule
viteza v.
EXEMPLUL 11.1
Ce este perioada și faza viteza de o apă adâncă val de 100 m lungime de undă?
Soluție
Din (11.9),
g
2
(2 )(10ms )
100m
,
0.8 s
2
2
1
w
p
am
p
w
=
=
=


w = 0.8 s
-1
și așa T
=2
p /w = 8.0 s.
Din (11.12),
p
=√



=


c
(10ms )(100m)
2
13ms
2
1
Deci:
l=
=
=

T
c
100m,
8s,
13ms
1
(11.13)
TWIDELL PAGINARE.indb 416
01/12/2014 11:37
§11.3 Val de energie și putere
417
§11.3 VAL DE ENERGIE ȘI PUTERE
§11.3.1 Derivare: de Energie în valul la o anumită locație
În teoria elementară de apă adâncă valuri începe prin luarea în considerare a unui singur regulat val. Particulele de
apă aproape de suprafață se va muta în orbite circulare, în diferite faze, în direcția de propagare x. Într -
o coloană verticală amplitudinea un egal cu jumătate din creasta la jgheab înălțimea H de la suprafață, și scade
exponențial cu adâncimea (Fig. 11.7(a)).
Particula de mișcare rămâne circulară dacă fundul mării adâncime D > 0.5
eu, când amplitudinea devine
neglijabil la fundul mării. Pentru aceste condiții (Fig. 11.7(a)) este prezentată în texte standard pe care o apă
de particule a căror medie de poziție sub suprafața este z se mișcă într-un cerc de rază dată de:
=
roe
kz
(11.14)
Aici k este numărul de undă, 2
p/l, iar z este adâncimea medie mai jos de suprafață (o cantitate negativă, deoarece
vom lua z ca fiind pozitiv în sus, ca în Fig. 11.7).
Considerăm elementar 'benzi' de apă pe unitatea de lățime de undă față de înălțime dz și "lungime" dx la
poziția (x, z) (Fig. 11.7(b)). Volumul pe unitatea de lățime de undă față de această bandă de densitate r este:
=
dV dx dz
(11.15)
iar masa este:
r
r
=
=
dm
dV
dxdz
(11.16)
Să E
K
fi energia cinetică totală de mișcare a valurilor pe fundul mării, pe unitatea de lungime de-a lungul x
direcție, pe unitatea de lățime a undei față. Energia cinetică totală de lungime dx de val este E
K
dx. Fiecare
elementul de apă de înălțime dz, lungime dx și unitatea de lățime este în mișcare circulară de la constanta vitezei unghiulare
w,
raza de orbită circulară rși viteza v
= rw (Fig. 11.7(b)). Contribuția de acest element pentru a cinetice
energie într-o coloană verticală de pe fundul mării la suprafață este
dE
K
dx, unde:
d
r
w
=
=
E dx
mv
dz dx r
(
)
K
1
2
2
1
2
2
2
(11.17)
Prin urmare:
d
rw
=
E
r
dz
K
1
2
2
2
(11.18)
Este mai ușor să ia în considerare un moment în timp, atunci când elementul este în poziția medie, și toate celelalte
elemente în coloana a sunt în mișcare pe verticală în același fază în direcția z (Fig. 11.7(c)).
Fig. 11.7
Elementar mișcare de apă într-o apă adâncă de val, trase pentru a arăta exponențială scădere de amplitudine cu adâncimea.
Spun
la nivel de suprafață
Adâncime
(a)
(b)
(c)
z = 0
H = 2o
z
dz
dx
r
dx
dz
ω
x
o
TWIDELL PAGINARE.indb 417
01/12/2014 11:37
418
Val de putere
Din (11.14) raza de orbite circulare este dat de:
=
r ae
kz
(11.19)
unde z este negativ sub suprafață.
Prin urmare, din (11.18),
d
r
w
=
E
un e
dz
(
)
kz
K
1
2
22
2
(11.20)
și energia cinetică totală în coloană este:

rw
rw
=
=
=−∞
=
E dx
o
e
dz dx
o
k
dx
2
1
4
kz
z
z
K
22
2
22
0
(11.21)
Deoarece k
= 2p/l, iar din (11.9) w
2
= 2pg/l, energia cinetica pe unitatea de lățime de val frontal pe unitate de lungime
de val este:
r
p
p
r
=
am
am
=
E
o
g
og
1
4
2
2
1
4
K
2
2
(11.22)
În Problema 11.1 se arată că energia potențială pe unitatea de lățime de val pe unitatea de lungime este:
E
og
1
4
P
2
r
=
(11.23
)
Astfel, cum ar fi de așteptat pentru mișcări armonice, media cinetice și potențiale contribuții
sunt egale. Total de energie pe unitatea de lățime pe unitate de lungime de undă față (adică energia totală pe unitatea de suprafață
de
suprafață) este:
total
= cinetică + potențial
r
=
+
=
EEEE
E
og
K
P
1
2
2
(11.24)
Rețineți că rădăcina medie pătrată amplitudine este
√(o
2
/2), astfel:
r
=
E
g(root mean square amplitudine)
2
(11.25)
Energia pe unitatea de lungime de undă în direcția de val, pe unitatea de lățime de val de față, este:
am
r
=
=
am
am
E
E
og
1
2
2
(11.26)
Din (11.9),
l = 2pg/w
2
, astfel:
pr
w
=
am
E
un g /
22
2
(11.27)
Sau, din T
= 2
p/w:
p
r
=
am
E
ogT
1
4
222
(11.28)
Este util pentru a arăta cinetică, potențială și totală energiile în aceste diverse forme, deoarece toate sunt diferit
folosit în literatura de specialitate.
TWIDELL PAGINARE.indb 418
01/12/2014 11:37
§11.3 Val de energie și putere
419
§11.3.2 Formule pentru putere de extracție din valuri
Până acum, am calculat totalul exces de energie (cinetică plus potențiale) într-o dinamică mare din cauza
undă continuă mișcare în apă adâncă. Energia este asociat cu apa care rămâne în aceeași
locație, atunci când a fost în medie de-a lungul timpului. Cu toate acestea, aceste calcule ne-au spus nimic despre transportul
de energie (puterea) pe secțiuni verticale de apă.
Texte Standard (de exemplu, Coulson și Jeffrey 1977) calcula această putere de la primele principii, prin luarea în considerare
presiunile în apă și rezultă deplasările. De matematică aplicată necesar este dincolo de
domeniul de aplicare al acestei cărți. Aici am extras esența analiză completă, care este simplificat pentru
deepwater valuri.
Ia în considerare un element sau de particule de apa de sub suprafața medie de nivel (Fig. 11.8). Pentru o suprafață val de
amplitudine - o și numărul de undă k, raza de particule de mișcare sub suprafața este:
=
roe
kz
(11.29)
În Fig. 11.8(b), deplasarea verticală din poziția medie este:
w
w
Δ=
=
zr
toe
t
păcat
păcat
kz
(11.30)
Componenta orizontală a vitezei u
x
este dat de:
w
w
w
w
=
=
u
r
t
un e
t
păcat
păcat
x
kz
(11.31)
Prin urmare, din Fig. 11.8(a), puterea efectuate în val la x, pe unitatea de lățime de undă față în orice
clipă, este dat de:
'

=

=−∞
=
P
p
pudz
(
)
x
z
z
1
2
0
(11.32)
În cazul în care p
1
și p
2
sunt presiuni locale, cu experiență de peste elementul de înălțime dz și unitatea de lățime
peste valul din fata. Astfel (p
1
− p
2

) este diferența de presiune cu experiență de elementul de lățime


Δy (= 1 m) într-o direcție orizontală. Singura contribuție la fluxul de energie care nu media de
la zero la o anumită adâncime medie în apă este asociat cu o schimbare în energia potențială
a particulelor de rotație în căile circulare (vezi Coulson și Jeffrey 1977). Prin urmare, de conservare
a energiei:
r

=
Δ
p
p
gz
1
2
(11.33)
Substituind pentru
Δz din (ora 11.30),
r
w

=
p
p
ga e
t
păcat
kz
1
2
(11.34)
Fig. 11.8
Local fluctuațiile de presiune în unda:
o
presiunile în val;
b
deplasarea locală de apă de particule.
Poziția deplasată
de rotație a particulelor
z
dx
dz
r
ω
wt
u
x
p
1
p
2
Poziția medie
(b)
(a)
z

r
TWIDELL PAGINARE.indb 419
01/12/2014 11:37
420
Val de putere
În (11.32), și cu (11.31) și (11.34),
'


r
ww
w
r
w
w
=
=
= −∞
=
= −∞
=
P
ga e
t
un e
tz
ga
e
tz
(
sin )(
sin )d
păcat
d
kz
z
z
kz
kz
z
z
0
2
2
0
2
(11.35)
Timpul mediu de-a lungul multor perioade de păcat
2
wt este egal cu 1/2, deci:
'

r
w
r
w
=
=
= −∞
=
P
ga
e dz
ga
k
2
2
1
2
kz
z
z
2
2
0
2
(11.36)
Viteza undei (strict faza viteza) ca este vizibil cu ochiul liber este, de (11.8),
w
=
=
am
c
k
T
(11.37)
Deci puterea reportate în val pe unitatea de lățime peste val fața devine în termeni de val
amplitudine - o, de perioadă T și lungimea de undă
l:
'
r
r
=
=
am
P
ga c
ga
T
22
4
2
2
(11.38)
Din (11.24) și (11.38) puterea P ' este egal cu totalul de energie (cinetică plus potențiale) E în val pe
unitatea de suprafață de suprafață, ori c/2. c/2 se numește viteză de grup (u) din adânc-val de apă, adică viteza
la care nevăzute de energie în grup a undelor este reportată,
=
u c / 2
(11.39)
deci, din (11.38),
'
r
r
=
=
=
P
ga c
ga
u Ue
22
2
.
2
2
(11.40)
în cazul în care de la (11.24), E
=
rga
2
/2. Dar din (11.9),
pl
w
=
=
k
g
2/
/
2
, astfel încât viteza de fază c și de grup
viteza u sunt:
w
w
p
p
=
=
=
=
c
k
g
g
T
gT
(2 / )
2
(11.41)
p
==
u
c
gT
2
4
(11.42)
Această diferență între viteza de grup și viteza de fază este comună pentru toate undele unde
viteza depinde de lungimea de undă. Aceste valuri se numește dispersie valuri și sunt bine descrise în
literatura de specialitate. Substituind pentru c din (11.12) în (11.38) vă oferă:
puterea efectuate în val pe un plan vertical, pe unitatea de lățime de val fața ca:
r
p
'=
P
goT
8
22
(11.43)
Care este, puterea în val crește direct ca pătratul amplitudinii undei și direct ca
perioadă. Atragerea de lungă durată, de mare amplitudine valuri de val-ingineri de putere este evidentă.
Am act de faptul că lungă perioadă de valuri poate la fel de bine fi caracterizat ca lungime de undă lungă valuri, cu (11.43)
scrisă în termeni de lungime de undă și amplitudinea folosind (11.10):
r
p
pl
'=




P
go
g
8
2
22
1
2
(11.44)
TWIDELL PAGINARE.indb 420
01/12/2014 11:37
§11.4 Real (neregulate) valurile mării: modele și putere
421
EXEMPLUL 11.2
Ce este puterea într-o apă adâncă de undă cu lungimea de undă de 100 m și amplitudinea de 1.5 m?
Soluție
Din (11.44):
'=
=




P
(1025kg.m ).(9,8 m.s ).(1,5 m)
8
.
2 .100m
9,8 m.s
72 kWm
3
2
2
2
1/2
1
π
π
Alternativ, de Exemplu Lucrat 11.1, c
= 13 m/s. Cu (11.40),
u = c/2 = 6.5 ms
-1
unde u este viteza de grup a energiei și c este viteza de fază.
Marea valuri de apă au o amplitudine o
= 1,5 m (H = 3 m), nu nerealiste pentru Atlantic valuri, astfel încât în
(11.38):
'=
=




P
(1025 kgm )(9.8 ms )(1,5 m) (6.5 ms ) 72 kWm
1
2
3
2
2
1
1

De Exemplu Lucrat 11.2, putem aprecia că pot fi


extrem de mari densități de putere disponibile în apă adâncă valuri de
ocean realist umfla.
§11.4 REAL (NEREGULATE) VALURILE MĂRII: MODELE ȘI
PUTERE
Val de sisteme nu sunt, în practică, singură undă sinusoidală modele de mișcare
într-o direcție ca idealizat în secțiunile anterioare. Foarte
ocazional naturale sau inventate val de difracție modele, sau bagati valuri,
abordare această condiție, dar în mod normal o mare va fi un model neregulat de
valuri de diferite perioadă, direcția și amplitudinea. Sub stimul de
o predominantă a vântului val de trenuri poate arăta o direcție preferată (de exemplu,
la sud-vest spre nord-est, direcția de Atlantic valuri pe British
Isles), și produce o semnificativă perioadă lungă de timp mare 'umfla'. Vânturile care sunt
mai neregulate produce neregulate apa mișcare tipic pentru perioade mai scurte,
numit "mare". La fundul mării adâncuri
~30 m sau mai puțin semnificative, concentrându-se
și direcționale adverse pot apărea, eventual, produc mai regulate sau
putere sporită de valuri la site-uri locale. Val-dispozitive de putere trebuie
, prin urmare, se potrivesc o bandă largă de condiții naturale, și să fie proiectate pentru a extrage
maximum de puterea medie pe o perioadă considerabilă de timp pentru fiecare
special poziție de implementare. În proiectarea acestor dispozitive, acesta va fi mai întâi
necesar pentru a înțelege val de modele de site-ul special, care pot apărea
pe o perioadă de 50 de ani; acest lucru necesită statistice și alte modelarea dacă astfel de
măsurători pe termen lung nu sunt disponibile.
Înălțimea valurilor la o poziție în mod tradițional a fost monitorizată pe o
val-înălțimea analog recorder. Separat măsurători și analize
TWIDELL PAGINARE.indb 421
01/12/2014 11:37
422
Val de putere
sunt necesare pentru a obține direcția de valuri. Fig. 11.9 oferă o
simulat o urmă de astfel de un recorder. O creasta apare ori de câte ori vertical
mișcare schimbări de sus în jos pentru a, și vice-versa pentru un
jgheab. Recordere moderne folosesc metode digitale pentru calculator bazate pe
analiza unor cantități mari de date. Informații esențiale despre valuri de peste
oceane este obținut din satelit măsurători folosind radar
1
(Caseta 11.1).
Diverși parametri sunt utilizate pentru a cuantifica marea membre, astfel cum sunt definite mai jos,
unde H este diferența de înălțime între o creastă și reușita
jgheab (Fig. 11.9):
1
N
c
, numărul de vreme; în Fig. 11.9 există 10 de vreme.
2
H
1/3

, la o a treia' înălțimea semnificativă a valurilor. Aceasta este înălțimea medie


a cea mai mare o treime din valuri măsurată între o creastă
și, ulterior, prin. Astfel, H
1/3
este media N
c
/3 mai mare
valori ale Sec.
3
H
s
, 'true' înălțimea semnificativă a valurilor. H
s
este definit ca:

=
=










=
H
o
H
n
4
4
/
am
n
s
rms
2
1
1/2
(11.45)
în cazul în care un
rms

este rădăcina pătrată medie de deplasare a apei de suprafață


din poziție, așa cum este calculat de n măsurători la egal
intervale de timp. De îngrijire trebuie să fie luate pentru a evita erorile de eșantionare, prin
înregistrarea la o frecvență de cel puțin două ori de la cel mai mare val
de frecvență prezent.
4
H
max
este măsurată sau cele mai probabile înălțime maximă de un val.
Peste 50 de ani H
max
poate egale de 50 de ori H
s
și așa acest lucru necesită
considerabile asupradesign pentru structuri în mare.
5
T
z
, care înseamnă trecerea prin zero perioada este durata de înregistrare împărțit
de n, unde (n
+ 1) este numărul de ascendent punctele de trecere a vrut
nivelul apei. În Fig. 11.9 n
+ 1 = 3 deci
τ
=
T
/2
z
; în practică, n este foarte
mare, astfel încât reducerea de eroare în T
z
.
6
T
c
, media creasta perioadă, este durata de înregistrare împărțit de N
în cazul în care (N
+ 1) este numărul de vreme. În Fig. 11.9, N + 1 = 10,
τ
=
T
/9
c

;
în practică N este foarte mare și atât de mici erori de numărare nu sunt
semnificative,
Fig. 11.9
Simulate val-înălțimea record de la o poziție (cu un exagerat set de vreme pentru a
explica terminologia).
Suprafață
H
T
TWIDELL PAGINARE.indb 422
01/12/2014 11:37
§11.4 Real (neregulate) valurile mării: modele și putere
423
7
Spectral lățime parametru
e dă o măsură a variației în
model de val:
e = − T T
1(/)
c
z
2
2
(11.46)
Pentru o uniformă singură frecvență de mișcare, T
c
= T
z
, deci
e = 0. În exemplul nostru
e = [1 − (0.3)
2
]
1/2

= 0.9, ceea ce implică un amestec de multe frecvențe. Plin


de informații este afișat prin transformarea Fourier a unui spectru de frecvență
(de exemplu, Fig. 11.11).
De la putere pe unitatea de lățime a undei față, într-un pur sinusoidală o
adânc val de apă este:
'
r
p
r
p
=
=
P
goT
gHT
8
32
22
22
(11.47)
în cazul în care jgheabul de creasta înălțimea este H
= 2o. Rădăcina medie pătrată (rms)
val de deplasare pentru o pură val sinusoidal este un
max
= un / √2, deci în (11.47):
'
r
p
=
P
goT
4
2
rms
2
(11.48)
CASETA 11.1 PRIN SATELIT DE MĂSURARE DE ÎNĂLȚIME A VALURILOR, ETC.
Teledetecție de la sateliți este singura metodă de măsurare a Pământului toată suprafața oceanului; acolo
sunt două tipuri de măsurare: (i) altimetru radar; (ii) radar cu apertura sintetica (SAR). Ambele metode sunt
folosite cu Orbită Joasă (LEO) sateliți care orbitează la înălțimi de
~1000 km și a perioadelor de ~15 orbite/
zi peste rotație a Pământului de mai jos. Datele sunt disponibile gratuit acces (de exemplu, din www.globwave.org), cu
explicații și de precizie al metodelor utilizate.
Radarul de date: Mare-suprafata instituție de învățământ superiorght (cu privire la satelit distanța de la Pământ). La suprafața
mării
înălțime este derivat din timpul de zbor' de radar impulsuri emise vertical în jos de instrumente
(altimetre) și se reflectă de pe ocean și interioare de suprafață mare. Sofisticat, cu o medie de data aceasta vă permite
mediu de undă de măsurare înălțime de peste mare de patch-uri la 5 km în diametru, de-a lungul calea prin satelit
în jurul Pământului. Precizie de ~1 cm este suficientă pentru a măsura medii de înălțime a valurilor de valuri fi
definite în acest capitol.
Sintetice aperture radar este emisă în mare de impulsuri de putere în fascicule înguste în unghiuri drepte la satelit
cale și la controlabile unghiuri de verticală. Raza reflectată se întoarce cu frecvența sa schimbat
de efectul Doppler în cazul în care reflectarea este de la o suprafață în mișcare (de exemplu, apă de rotație de valuri).
Sofisticate de analiză permite valuri cu o înălțime să fie măsurată cu o precizie de ~1 mm; o astfel de precizie este mai mult
decât adecvat pentru mare val de analiză, și suficiente pentru a măsura amplitudinea mică capilar valuri
referitoare la viteza vântului.
Utilizarea de date val. Prin satelit-date derivate dintr-o varietate de tipuri de instrumente sunt importante pentru multe
scopuri, inclusiv servicii meteorologice, maritime și terestre, de temperatură, de transport maritim și de pescuit, geologice
de prospecțiune offshore construcții, formațiuni de gheață, val de evaluare a energiei din surse regenerabile de energie, vânt
viteză de evaluare pentru parcurile eoliene offshore și de agrement care navighează. Fig. 11.10 ilustrează valoarea unor astfel de
date pentru val de energie de evaluare a resurselor.
TWIDELL PAGINARE.indb 423
01/12/2014 11:37
424
Val de putere
Fig. 11.10
o
Val-putere harta pentru marea de vest a Norvegiei din satelit derivate de date. Umbrită pete se referă la puterea medie
în valuri (kW/m) în cele trei luni septembrie-noiembrie. Datele pe parcursul mai multor ani au fost calculate
de Fugro OCEANOR e WorldWaves SCWM baza de date pe care este derivat din ECMWF operaționale val modelul
de arhivă, calibrat și corectată (de OCEANOR) împotriva Topex prin satelit altimetru date.
b
Conturul medie val de energie pe Nord-vestul Europei. Numerele indica anual de energie în MWh, și puterea
de intensitate (paranteze) în kWm
-1

.
(a)
66
°
N
100
m
63
°
N
60
°
N
2
°
E
(b)
8
°
E
14
°
E
SEP − NOV
Înseamnă val de putere
kW/m
44 la 58
la 40 la 44
< 40
45°
55°
50°
620(70)
440(50)
210(24)
(24)
(30)
30(3)
80(9)
210(24)
350(40)
440
440(50)
620(70)
440(50)
700(80)
700(80)
TWIDELL PAGINARE.indb 424
01/12/2014 11:37
§11.4 Real (neregulate) valurile mării: modele și putere
425
Fig. 11.11
Distribuția de energie pe frecvență interval într-un tipic Atlantic de apă adâncă a undelor
(Shaw 1982). Netezit spectru este folosit pentru a găsi T
e
energia perioadă.
Frecvență/Hz
Densitate spectrală de putere funcția/m
2
H
z
-1
3
2
1
0
0.1
0.2
0.3
În practică, valurile mării sunt cu siguranță nu se continuă cu un singur frecvența
undelor sinusoidale, deci puterea pe unitatea de lățime în val este dat în termeni
de valuri cu o înălțime semnificativă H
s
(11.45) și energia perioada T
e
. Astfel, în
forma (11.47),
'
r
p
=
P
gHT
64
2
s
2
e
(11.49)
Aici T
e

, l 'energie perioadă', este perioada de la vârf de putere


spectrală a densității de distribuție cum ar fi ca în Fig. 11.11. Acest lucru poate fi comparat cu
T
z
anii 'înseamnă trecerea prin zero perioadă' sunt definite la punctul 5 de precedent
lista de variabile de bază. Pentru mai multe mări,

≤≤
T
kT
k
cu1.1
1.3
e
z
(11.50)
Până evoluțiile moderne în val de putere doar o valoare aproximativă
de P
'ar putea fi obținute de la analog de înregistrare val de metri, astfel încât:
'
r
p





P
gHT
H kT
64
(490 Wm m s )
e
e
2
1/3
2
1
2
1
1/3
2
(11.51)
Cu toate acestea, cu echipamente moderne și analiza de calculator, mai
sofisticate metode pot fi utilizate pentru a calcula H
s
și T
e
. Luând k
≈ 1.2 și
r = 1025 kg m
-3
(pentru apa de mare), (11.51) se obține:
'
=






P
HT
HT
(490Wm m s )
(590Wm s )
1
2
1
s
2
e
3
1
s
2
z
(11.52)
TWIDELL PAGINARE.indb 425
01/12/2014 11:37
426
Val de putere
Fig. 11.12
Ilustrativ diagramă de dispersie de valuri cu o înălțime semnificativă H
s

împotriva trecerea prin zero perioadă de T


z
.
Numerele de pe grafic denotă numărul mediu de apariții ale fiecărui H
s
, T
z

în
fiecare 1000 de măsurători făcute peste un an. Cele mai frecvente cazuri sunt de la H
s

~3 m, T
z
~9 s, dar rețineți că maxim probabil de putere are loc pe perioade mai lungi.
- - - - - Aceste valuri au aceeași pantă maximă sau panta. (1/n) gradientul maxim
de astfel de valuri (de exemplu, 1 din 20).
_____ Linii de val constant de putere, kW m
-1

. Date de 58°N 19°W în mijlocul Atlanticului.


1/20
1/40
14
13
12
11
10
9
8
7
6
5
4
3
2
1
0
7.0
8.0
9.0
10.0
La trecerea prin Zero perioadă de T
z
/s
H
s
/m
11.0
12.0
13.0
14.0 15.0
1600
1200
800
400
200
100
50
20
10
5
1
1
11
1
11
1
1
1
3
2
2
1
2
1
11
21
1
1
1
1
1
1
2
3
1
4
6
7
14
4
2
1
1
1
1
1
1
1
11
1
11
1
1
1
1
2471
34331
15541
2659561
1
2
3
2
4
5
9
15
14
19
8
8
2
1 4 4 13 9 10
10
10 8 4
1
1
34
4
3
11
2
2 4 13 18 24 22 16 8 5 3
1
1
1
1
11
2 12 21 26 39 24 12 8 3 1 1
5 13 39 23 42 20 13 5 2
7 24 39 24 24 12 6 1 1
3 12 17 16 12 5 1
2
12
1
11
Din moment ce un model de val nu este de obicei compus din valuri toate progresează
în aceeași direcție, puterea primită de către un dispozitiv de direcționare va fi
redus în mod semnificativ.
Val-model de date sunt înregistrate și prezentate în detaliu la standard
meteorologice mare de stații. Poate cel mai important grafic pentru orice
site-ul este pe val diagramă de dispersie peste un an (de exemplu, Fig. 11.12). Acesta înregistrează
numărul de apariții ale val de măsurători ale intervale special
de valuri cu o înălțime semnificativă și la trecerea prin zero perioadă. Presupunând că perioada
este legată de lungimea de undă de (11.10) în care este posibil, de asemenea, se trasează pe
diagrama de linii de constantă de înălțime a valurilor de lungime de undă.
TWIDELL PAGINARE.indb 426
01/12/2014 11:37
§11.5 extragerea de Energie din dispozitive
427
Fig. 11.13
Val dispozitiv de clasificare prin metoda de captare a energiei valurilor; liniile punctate indica
val de energie fiind absorbită în dispozitiv.
Sursa: După J. Falnes (NTNU).
Wave front
Val
direcția
Punct de amortizoare
Atenuator
Terminator
De valul de date, este posibil să se calculeze valoarea maximă, medie,
minimă, etc. de putere în valuri, care pot fi apoi reprezentate pe
hărți pe termen lung anuale sau medii lunare.
Vezi fig 11.1 și 11.10 pentru mediu anual de energie intensități în întreaga
lume și Nord-vestul Europei.
§11.5 EXTRAGEREA DE ENERGIE DIN DISPOZITIVE
§11.5.1 Clasificarea dispozitivelor
Ca un val trece pe o poziție staționară: (a) suprafața modificări înălțime,
(b) volume mici de apă roti aproape de suprafață, și (c) de apă
sub presiune pe suprafața modificări. O mare varietate de dispozitive au
fost sugerate pentru extragerea de energie, folosind una sau mai multe dintre aceste
variante ca intrare pentru dispozitiv, astfel încât clasificarea este utilă,
2
așa cum se arată în
Fig. 11.13, în cazul în care suntem în căutarea în jos pe valuri și dispozitive:
3
1
Punct de amortizoare au atât de lungime și lățime <<
eu, la lungimea de undă
de valurile mării. Astfel de dispozitive au mare captura lățime pentru a 'trage'
de energie din valuri.
2
Atenuatoare (linie amortizoare) sunt de mai multe lungimi de undă lungă (de exemplu, lungimea
>~50 m) și înguste (lățimea<<
l). Înălțimea valurilor reduce ca
val progreseaza si ca puterea este absorbită de-a lungul lungimii. Pentru
atenuatoare pentru a avea succes, ei trebuie să aibă o mare de captare lățime
comparativ cu lor efectivă lățime.
3
Terminatori minciună pe sens opus val de față cu lățimea >
am si
lungime scurtă. Amplitudinea undei scade rapid la
interacțiunea ca puterea este absorbită; valul trece cu mult
scăzut flux de energie.
TWIDELL PAGINARE.indb 427
01/12/2014 11:37
428
Val de putere
Dispozitivele sunt clasificate în continuare de locația lor, ca pe uscat (de exemplu, pe
teren), mal (de exemplu, în adâncime relativ mică de apă, adâncimea ~
l ), sau offshore
(de exemplu, în apă adâncă, adâncime >>
l ).
O altă clasificare se referă la dezvoltarea de val de putere
dispozitive:
1
Prima generație: acestea sunt oscilante coloană de apă (OWC) dispozitive, după cum
este descris în §11.6.3, au fost cele mai simple dispozitive
pentru a construi onshore și lângă țărm. Aer de deasupra coloanei de apă este
forțat printr-o turbină de generator de energie electrică.
2
A doua generație: dezvoltare dispozitive plutitoare preluarea puterii de
valuri pentru a pompei interne de fluide prin intermediul unui generator de turbină.
3
A treia generație: foarte singur mare dispozitive sau la fel de mari tablouri de mici
dispozitive de operare din punct de vedere comercial, atunci când anumite tehnologii sunt
considerate dovedite și finanțate. Foarte puține dispozitive au ajuns la acest
stadiu până în 2013.
§11.5.2 Captura lățime și reglate captarea energiei
Val de energie dispozitive de extras o putere medie
P
D
(unitate de măsură W) din
din sens opus fluxului de energie de la undele P
'(unitate de măsură W/m); P ' se calculează din
(11.43) sau (11.52). Raportul dintre acești doi parametri are dimensiuni de
lungime (m) și este captura lățime C
w
:
=
'
C
PP
/
w
D
(11.53)
Dacă C
w
este împărțit de către dispozitiv, lățime w, avem non-dimensional sau
relativă lățimea de captare:
=
=
'
C
C
w
PP
w
/
(
/)/
wr
w
D
,
(11.54)
Captura lățime (uneori numit absorbția lățime) este utilizat
ca o măsură a eficacității' de un dispozitiv și depinde de: (a)
condițiile de undă, mai ales de frecvența undelor, înălțime și direcție;
(b) limitele de model de val (de exemplu largul mării sau val rezervor),
și (c) dispozitiv fix sau cu poziție variabilă. La o medie de criterii
trebuie să fie definite mai atent (de exemplu, pe val perioada de "regulat"
singură frecvență, care sunt puțin probabil să apară la mare, sau pe
perioada de repetate val de energie fluxuri în "ilegali" valuri de repetarea
model, aproximativ repeta la fiecare cinci până la zece valuri, sau pe an,
pentru o anumită locație). Succes dispozitive au
C
w r,
>
~ 3 deoarece
dispozitivul absoarbe putere de un val frontal mai mare decât aparatul în sine; pentru
acest motiv, nu este util să se ia în considerare C
w
,
r
o eficiență în normal
sens.
Putem, cu toate acestea, pentru a defini eficiența dispozitivului ca raportul dintre
putere utilă de ieșire din aparat pentru puterea incident imediat
TWIDELL PAGINARE.indb 428
01/12/2014 11:37
§11.5 extragerea de Energie din dispozitive
429
de pe dispozitiv. În practică, această eficiență depinde în mare măsură de
lungimea de undă și amplitudinea de valuri și caracteristicile
dispozitivului.
Un principiu important pentru proiectarea unui plutitoare val de energie
absorbant a fost mai întâi exprimată de Falnes (2002); aceasta poate fi declarată caun
succes val de absorbție a energiei este un succes val de energie filtru'.
Pentru a înțelege acest lucru, vom imagina val dispozitiv forțat să oscileze într-o
mare liniștită; se va face valuri de model A. În funcțiune, valuri de model
B sunt incidentul de pe dispozitiv, care apoi oscilează. Dacă modelul B este în
anti-fază cu Un model, apoi apa de pe dispozitivul devine
calm, și anume dispozitivul a absorbit 100% din jur primite
val de putere. Acest concept este vital pentru proiectarea unui acordabile val
powerabsorbing dispozitiv.
Cruz (2008) se aplică Falnes principiul atât un punct de absorbție și o linie
atenuator/amortizor.

Un punct de absorbant (constrâns să se mute în heave numai, adică pe verticală)
în apă generează Un model la fel de circular valuri în toate direcțiile.
Cu toate acestea, sosite de valuri cu model B cu lungimea de undă
eu sunt
dintr-o singură direcție, deci, Un model nu poate anula model B.
teoretică maximă de captare lățime pot fi afișate pentru a fi
l/2p (adică
aproximativ 16 m de mare și 100 m de lungime de undă valuri indiferent de dimensiunea
de "punct").

Cu toate acestea, o linie atenuator cu cuplate secțiuni în mișcare de
faza cu altele (ca nu Pelamis wave putere dispozitiv de
§11.6.4) pot fi controlate pentru a produce Un model ca o unidirecțional
val de tren se propagă în linie cu sine. Acest lucru este pentru că modele
de la fiecare out-of-faza secțiunea anulează reciproc în toate celelalte
direcții. Cruz (2008) subliniază că acest lucru este același val mecanism
folosit pentru a produce regizat 'fazată' radar grinzi cu nici o mișcare
antene. De potrivit scalare și de proiectare a cuplat secțiuni
și de 'tuning' hidraulic de acționare, model pot fi facute pentru a fi
anti-fază echivalent de intrare val de mare, pentru care
teoretic de captare lățime este
l/2 (de exemplu, 50 m de mare și 100 m de lungime de undă
valuri).
La Falnes principiu este foarte important în a explica de ce și cum
optimă val de putere dispozitive trebuie să fie dinamic acordabile structuri,
și cu siguranță nu sunt fixe structuri statice. Totuși, acest criteriu este departe de a fi ușor
pentru a obține pentru dispozitive operaționale de durată pentru mulți ani în largul oceanelor,
chiar dacă tuning este ajustată în funcție de oră sau de zi cu zi, mai degrabă decât în stare proaspătă pentru
fiecare
val de intrare. În practică, acordabile eficiența este mai puțin importante decât
supraviețuirea; speranța este de a avea pe amândouă.
TWIDELL PAGINARE.indb 429
01/12/2014 11:37
430
Val de putere
CASETA 11.2 ENERGIEI VALURILOR ÎN MAREA BRITANIE
Situat în partea de est a Atlanticului de Nord, Regatul Unit (Anglia, Scoția, țara Galilor și
Irlanda de Nord) este un stat insular cu mari oportunități de a dezvolta și de a folosi puterea valurilor. (Valurile
se apropie de coasta de vest a Irlandei și Scoția sunt chiar mai puternici decât cei care se apropie de
coasta de vest a Norvegiei (Fig. 11.10); conturul de 70 kW/m mediu anual numai minciuni
~50 de km de pe
coasta de vest a Irlandei.) Prezenta politica este pentru un guvern independent stabilit de agenție,
Carbon Trust, pentru a gestiona 'Marin Provocare pentru promovarea atât a valurilor și a mareelor. Studii
și dispozitive fabricate sunt pe deplin sau în cooperare finanțate cu scopul de a stabili un important marea BRITANIE
de alimentare cu energie la costuri realiste. Rezultatele de până în prezent includ următoarele:

Total val de energie care intră marea BRITANIE apele de coastă este de aproximativ 350 TWh/an, egale din punct de vedere
energetic în marea BRITANIE
anual de furnizare de energie electrică.

Cele mai evoluțiile economice vor fi plasate în Oceanul Atlantic ~100 km în largul coastei de vest
a Scoției.

Din această locație numai, livrate de putere ar fi de aproximativ 35 TWh/an la aproximativ 10 GW de instalat val
capacitatea de putere, adică la 35% factor de capacitate.

Acest lucru ar fi de aproximativ 10% din marea BRITANIE anual total de energie electrică.

Cu dispozitive performante în 2012, generația costul ar fi de aproximativ 23 p/kWh (aproximativ 36 c/kWh).

Dezvoltarea și experiența sunt de așteptat pentru a reduce în mod semnificativ costurile de a fi competitiv
carbonfree de alimentare.

Offshore wave hub-uri pentru proiecte de Cercetare și dezvoltare există în apropiere de țărm pe Orkney insulei principale (nord
Scoția)
și pe sud-vestul Angliei.

În Mai 2013, guvernul Scoțian a aprobat un 40 MW val ferma de pe Insula Lewis (coasta de vest
a Scoției) cu Acvamarin 'Scoica' ~1 MW dispozitive.
Surse: Carbon Trust (2006); Cruz (2008).
§11.6 VAL DE DISPOZITIVE DE PUTERE
În această secțiune vom examina o gamă selectată de dispozitive pentru a ilustra
criteriile de clasificare de mai sus.
§11.6.1 Pe malul terminator, Tapchan, creativitate
val de captare
Principiul de val de captare este simplu, cum s-a observat în multe țărmului
lagune și bazine de înot. Valurile ridica o bagati mare de perete și
de overspilling apa este reținută într-un rezervor deasupra mării
nivel. Controlate de evacuare a apei revine la mare printr-un convențional
low-cap hidroelectrice turbine generator.
Fig. 11.14 este o diagramă schematică de 350 kW Tapchan sistem de
demonstrat în Norvegia în 1985 și de replicat la câteva site-uri
din întreaga lume. Valurile de intrare de pâlnie, o îngustare (conic) canal,
al cărui ziduri de beton a ajuns la 2 la 3 m deasupra nivelului mării. Pe
vârfuri de aceste valuri de creștere în înălțime deasupra jgheaburi ca valurile
progresul de-a lungul conice canal, astfel încât apa se scurge în
rezervor ca valul ajunge la perete. Valuri mai mari pot întrece în înălțime perete
TWIDELL PAGINARE.indb 430
01/12/2014 11:37
§11.6 Val de dispozitive de putere
431
Fig. 11.14
Diagrama schematică a Tapchan val de plante energetice (a se vedea textul). Valuri de creștere în crest
înălțime ca acestea trec de-a lungul îngustarea (conic) canal, deci scurse în
rezervor. Apa iese din rezervor printr-un low-cap turbină de generator de energie electrică
(a se vedea §6.5).
Cliff fata
Rezervor
P
Q
Turbina casa
direct. Site-ul a fost ales cu grijă pentru a incorpora formațiuni naturale
cu peretele și bazin. Site-uri pentru val de sisteme de captare a beneficia de
următoarele caracteristici:

Persistente valuri cu medie mare val de energie.

Apă adâncă de aproape de țărm, astfel încât puterea de valurile nu este
în mod nejustificat s-a disipat.

Un mic mareelor (<1m).

Caracteristici naturale care beneficiază de val canal și rezervor.

Construcție robustă din canal și pereți împotriva furtunilor violente,
deoarece furtuna valuri sunt, de asemenea, canalizate și îmbunătățită în înălțime față de
valorile crescute ale bazinului.
De criterii de (11.53), capturarea lungime de un Tapchan dispozitiv poate
fi definită ca raport de medii de ieșire a turbinei de putere medie
putere pe unitatea de lățime de valurile care intră capătul inferior al canalului.
Original Tapchan în Norvegia a fost distrus de un mod neașteptat
furtună violentă.
4

Lecțiile de asta sunt importante, inclusiv vârsta-vechi


sfaturi de transport maritim să nu fie prins în port într-o furtună violentă atunci când
este sigur pentru a fi pe mare. Astfel plutitoare val de energie a dispozitivelor care permit
furtuna valuri să curgă peste ele ar putea fi mai robust decât rigid
nearshore construcții și dispozitive.
§11.6.2 Wave Dragon: plutitoare și creativitate terminator
Acest dispozitiv a fost dezvoltat în Danemarca, care are o linie de coastă la
Marea Nordului, care are o mai puțin intensive val regim decât Oceanul Atlantic.
Se folosește și creativitate metoda de mai devreme Tapchan, dar cu tot
TWIDELL PAGINARE.indb 431
01/12/2014 11:37
432
Val de putere
Fig. 11.15
Wave Dragon plutind val dispozitiv de putere:
o
schematic;
b
fotografie de prototip, pe Danemarca. Valurile de intrare (dreapta jos) domină în
rezervor (centru-stânga). În fundal (la dreapta) este unul dintre cele concentrare "wings",
care crește în zona de captare.
Surse: (a) Retragerea de la http://amsacta.unibo.it/3062/1/overtopping_devicex.pdf.,
(b) Fotografie Wave Dragon Aps, Danemarca, folosit cu permisiune.
Rezervor
(a)
Turbina
Creativitate
Priza
(b)
structura plutitoare pe mare. Creativitate valuri metamorfoza într-un rezervor din
care apa curge prin turbină generatoare (Fig. 11.15(a)).
Lowhead turbine Kaplan au fost folosite pentru prototipuri (a se vedea §6.5). Undele
sunt reflectate și concentrate în creativitate regiunea de
concaveshaped "aripi", care sunt toate parte din structura plutitoare (Fig. 11.15(b));
aceste oferi, de asemenea, structura mai mare stabilitate, ceea ce este important în furtuni.
După succes prototip de dezvoltare pe Danemarca, 7 MW
capacitate dispozitiv urmează să opereze de pe coasta de vest a țării Galilor. De 'deschidere' de
reflexie aripi este de 300 m pentru această scară comercială mașină.
TWIDELL PAGINARE.indb 432
01/12/2014 11:38
§11.6 Val de dispozitive de putere
433
Dacă lățimea activă dispozitiv este considerat a fi suma
turbina afluxul diametre, vom calcula relativ captura lățime raport
C
w,r

să fie aproximativ 2,4 într-un 36 kW/m val regim (date de Bevilacqua


și Zanuttigh 2011). Acest lucru arată beneficia de reflexie aripi în
captarea de energie.
Înălțimea de la partea de sus a rezervorului este conceput pentru a prinde
și creativitate de mediu valuri, astfel încât să se optimizeze generație prin
an. Extins structura îmbunătățește stabilitatea în marea agitată, și
limitează adâncimea de structură plutitoare și limitează înălțimea rezervorului
permite energiei de mare unde furtuna să treacă pe dedesubt și pe
structură.
§11.6.3 Oscilant coloana de apă (OWC) terminator:
prima generație de la țărm și în apropiere de țărm
Valurile trec pe un parțial scufundate cavitatea deschide sub apă
(Fig. 11.16), astfel că o coloană de apă oscilează în sus și în jos în cavitate.
Aceasta induce o mișcare oscilatorie în aer deasupra coloanei, care
poate fi conectat la atmosfera printr-o turbină conectată
la un generator de energie electrică. Turbina este de obicei folosit un turbinei Wells,
care a început o dată continuă să rândul său, în aceeași direcție, în funcție de
direcția fluxului de aer. Prin urmare generație continuă, fără o pauză,
dar în diferite grade de putere amplitudine (a se vedea Problema 11.3 și Fig. 11.16).
O dezvoltare dispozitiv de acest fel conectat la rețeaua de energie electrică
operat pe insula Scoțiană Islay în 1990, timp de mai mulți ani,
fără daune, dar la mai puțin decât era de așteptat puterea de ieșire. Bazat pe
experiență, o mai mare și mai eficient dispozitiv a fost proiectat și numit
'Melcul' (Terenuri Instalat Marine Energie Emițător) după crustacee
de acest nume de renume pentru firma lor atașament față de stânci. Sugarul e
generale de proiectare vă permite pentru două Fântâni turbina 250 kW generatoare în paralel,
dar pe Islay site-ul potrivit doar unul din aceste sisteme la 250 kW
capacitate, care a continuat să exporte energie electrică în energie electrică de distribuție
rețea începând cu anul 2000. Exploatarea comercială a beneficiat de export au crescut
prețul pentru energia electrică sub Feed-in Tariff legislației Scoțiene de
Administrare (a se vedea §17.5).
Un avantaj al folosind un oscilant coloana de apă pentru putere
de extracție este că viteza aerului este crescută prin reducerea buna în
aria secțiunii transversale a canalului se apropie de turbina. Această cupluri
lentă mișcare de valuri la rapid de rotație a turbinei fără
mecanice de actionare. Un alt avantaj este că generatorul electric
este deplasat semnificativ de la coloana de apă salină. Structurale
forma și dimensiunea determina frecvența de răspuns, cu fiecare formă și
dimensiune a cavității răspunde cel mai bine la valuri de o anumită frecvență. Este
esențial faptul că caracteristicile generator cu turbină sunt potrivite pentru
valul de circulație.
TWIDELL PAGINARE.indb 433
01/12/2014 11:38
434
Val de putere
CUTIE 11.3 TEORIA DE BAZĂ A UNUI DISPOZITIV OWC
Cu toate ca dispozitivul teorie, vom începe prin simplificarea problemei. Deci, vom lua în considerare dispozitiv distinct
volumul de apă, de masă M, secțiune transversală zona A, oscilează în sus și în jos în interiorul structură tubulară
(Fig. 11.17). La momentul t, centrul de greutate din această masă este la înălțimea z deasupra nivelului mării.
Mișcarea de astfel de 'antrenat' de volum creează componente de circulație în marea adiacentă
apă, așa că M include o contribuție la un astfel de "în plus" de volum. Volumul este centrul de greutate oscilează
între înălțime
+z
max
și adâncime
−z
max

din poziție staționară pe o mare calma. Propunerea din acest volum


împinge aerul înainte și înapoi prin Puțuri cu turbină cu aer, provocând o forță de amortizare pe volumul de apa
proporțională cu viteza de coloana dz/dt. Prin urmare, depinde de momentul forței F(t) cu experiență de
volumul de apa este dat de:
=
+
+
Ft
M
dz
dt
D
dz
dt
Bz
()
2
2
(11.55)
în cazul în care:
(i) M este masa indicat oscilant volum de apă și se adaugă volum, astfel încât M(d
2
z/dt
2
) este
Newton, forța reactivă de accelerare d
2
z/dt
2
.
(ii) D dz/dt este forța de amortizare care decurg din cele trei componente ale factorului de amortizare D: (1) D
1
din
aer turbina ca acesta rezista la fluxul de aer și extracte utile de energie din acesta; (2) D
2
, de la secundar
ieșire valuri create de oscilante de volum; (3) D
3
de nedorite de frecare.
(iii) B = O
rg e gravitațională forță de revenire la poziția z (volumul dislocat V = Az de apă de mare, densitate
r, accelerația gravitațională g).
Forma generală de ordinul al doilea ecuație diferențială de (11.55) este frecvent utilizat pentru analiza
în multe sistemelor mecanice și electrice. Puterea medie P
D
extras de turbine de
oscilație de apă poate fi calculată din (11.55), potrivit Mei (1989) și Cruz (2008), ca funcție:
Fig. 11.16
Diagrama schematică a unui val mal sistem de alimentare folosind un oscilant coloana de apă.
Bazat pe SUGARUL dispozitivul de pe insula Islay, la Vest de Scoția, pentru
gridconnected generarea de energie electrică.
Wells
turbina
Sunet
șicane
Aer
Apa
Val de mișcare
Rock
Cel mai prima generație de dispozitive au fost pe mal și near shore
(mal) OWC dispozitive, în linii mari, similare cu Sugarul. În practică,
cel mai bun captura lățime raport C
w,r
de astfel de dispozitive
5
este de ~3.
TWIDELL PAGINARE.indb 434
01/12/2014 11:38
§11.6 Val de dispozitive de putere
435
w
w
w
=

+
+
+
P
DF
BM
D
D
D
/
(
)
(
)
D
1
2
1
mod
2
22
1
2
3
2
2
(11.56)
Această funcție, ca și testate în val de tancuri, este schițat de un val de frecvență
w și constantă empirică
F
mod

în Fig. 11.18 cum este indicat de către Cruz (2008). Rețineți că putere medie de ieșire este foarte dependentă de
amortizare cauzate de turbină cu aer, care, în practică, trebuie să fie reglabil pentru a optimiza generat
de putere; pe de amortizare este mai sigură strategie decât sub-amortizare. Insuficienta de amortizare poate duce la rezonanță
oscilație de creștere a amplitudinii care determină în cele din urmă nedorite deteriorări mecanice; ca și în multe
sisteme mecanice, cum ar rezonanță trebuie să fie evitate.
Fig. 11.18
Schiță curba arată puterea extras (P
D

) de la un oscilant coloana val de putere


dispozitiv ca o funcție de amortizare D
1

în aer turbina.
Sursa de analiză: Cruz, J. (2008).
Turbină de aer de amortizare factor D
1
Putere extras
P
D

Fig. 11.17
Simplist model de oscilant coloana de apă, val dispozitiv de putere, pentru analiza Cutie 11.3. Energia electrică este generată
de Puțuri de aer turbina.
Wells
turbină de aer
Mare
nivelul
z<0
z=0
z
Wells
turbină de aer
Wells
turbină de aer
O
O
O
Aer
în
Aer
afară
TWIDELL PAGINARE.indb 435
01/12/2014 11:38
436
Val de putere
§11.6.4 Pelamis atenuator, offshore, doua
și a treia generație
Plutitoare val de energie atenuatoare plutesc pe sau aproape de suprafața mării și
răspunde la forma de valurile incidente. Ele sunt ancorate de
fundul mării, astfel încât acestea să se alinieze la media val de flux de energie. Cel
din sens opus val de energie este absorbită treptat de dispozitiv și astfel
diminuează (atenuează), ca valul trece. Astfel de dispozitive au o lungime
comparabilă cu marea lungimi de undă (de exemplu,
~150m), și un relativ mic
lățime.
La Pelamis mașini (Fig. 11.19) au mai multe tubular
semisubmerged module conectate prin racorduri capabil să se deplaseze cu amortizare
în heave (pe verticală) și sway (orizontal) (adică într-un plan
perpendicular pe sens opus val de model). La acordabile de două-dimensional
de cuplare permite ancorat dispozitivul pentru a avea o mare de captare lățime C
w

, după cum
explică prin Falnes principiul de §11.5.2. În efect, ca val de energie este
absorbit in aparat, suplimentare val de putere este aspirat din
regiunile adiacente, astfel încât lățimea de captare (11.53) este semnificativ mai mare decât
dispozitivul cap-pe lățime. În dezvoltarea prototipului capturarea lățime
raport C
w,r
a fost > 6.
Val de mișcare induse de cuplaje este reglată de acordabile
hidraulice, pistoane, care a pompei de înaltă presiune ulei prin motoare hidraulice
prin netezire acumulatori. Aceste motoare hidraulice conduce vehicule electrice,
generatoare, la fiecare cuplare, deci producerea de energie electrică transmisă prin
cablu submarin la mal. Mai multe dispozitive pot fi aranjate într-o matrice sau 'ferma',
fiecare cu racord electric la un " hub " de care puterea este
transmisă la mal printr-un cablu submarin. Un 750 kW capacitate de prototip, la 150 m
de mult în patru module, iar fiecare 3.5 m în diametru, a fost instalat în anul 2004
offshore de principala insula Orkney, Scoția de nord ca lumea e
primul off-shore val de putere generator conectat la reteaua de utilitati. Acest pro-
Fig. 11.19
Pelamis atenuator val dispozitiv de putere. Diagrama unui ancorat dispozitiv, după cum se vede din
lateral și de sus. Mișcare la cuplaje (flexibil în ambele heave (vertical) și
girație (orizontală)) produce hidraulice de putere alimentat la generatoare electrice, care la rândul lor hrănesc
putere la mal printr-un cablu submarin.
Val
direcție
TWIDELL PAGINARE.indb 436
01/12/2014 11:38
§11.7 Sociale, economice și de mediu
437
totype masina a fost dezvoltat într-un de dimensiuni similare și capacitatea model
P2 pentru experiență comercială și de utilitate generație. În utilizarea moderată în
condiții de mare de "normal" (adică neregulate) valuri, electrice putere de ieșire
ajunge la 400 kW în rafale și o medie de aproximativ 270 kW.
6
Pelamis
dezvoltarea progresează la val de putere ferme de ~10 MW capacitate de
tablouri de aproximativ 15 P2-scară mașini.
Energia electrică generată "imediat" de val dispozitive de putere (respectiv
fără vreo formă de medie sau de depozitare în creativitate dispozitive)
variază în amplitudine cu timpul. Astfel variabilitatea este similar, și, probabil,
mai mare decât variabilitatea de turbine eoliene individuale. În ambele cazuri,
cu o medie de ieșire de mai multe mașini în matrice scade considerabil
variabilitatea ieșire combinată.
§11.6.5 Rezumat al operaționale dispozitive
Sunt multe de dezvoltare val de putere dispozitive, multe dintre care
sunt în comerciale de implementare de rețea compatibil generarea de energie electrică.
Tabelul §11.1 în suplimentare on-line eResource pentru această carte
rezumă situația ca în 2013.
§11.7 SOCIALE, ECONOMICE ȘI DE MEDIU
ASPECTE
Cu toate ca dezvoltare, atentă și cuprinzătoare
a impactului asupra mediului de control este esențială. Energia valurilor este o energie regenerabilă
resursă și acțiuni generale caracteristicile de durabilitate,
securitate energetică, minim poluarea chimică, ocuparea forței de muncă locale și
variabilitatea naturală – caracteristici care cea mai mare parte contrast cu cele ale
combustibililor fosili și energie nucleară. Există caracteristici distinctive de
val de sisteme de putere, cel principal fiind esențiale marin
circumstanță și relevanță doar în țări cu malurilor și offshore
drepturi. În mod clar, siguranța personalului la mare este de o importanță capitală,
mai ales ca la aparate și de a lucra pe ele în funcțiune sunt
individual nou și distinctiv.
Politicile naționale pot favoriza val de putere din cauza pozitiv
beneficiile:

Atenuarea emisiilor de gaze de seră prin înlocuirea fosili
combustibili utilizați (cu toate ca din surse regenerabile de energie).

Crescând securitatea energetică națională locale de generare de energie electrică.

Creșterea ocupării forței de muncă și de investiții, în special în mediul marin legate de
industriile de construcții și reparații.

Cooperarea și integrarea cu fermele eoliene offshore și alte
resurse marine.
TWIDELL PAGINARE.indb 437
01/12/2014 11:38
438
Val de putere
Cu doar câteva val de sisteme de alimentare cu mulți ani de
funcționare, experiența este limitată, dar impactul negativ potențial de val de putere
dispozitive includ următoarele:

Aer turbine de operare cu val periodicities poate fi acustic
zgomotos. Cu toate acestea, vânt și valuri care se sparg sunt susceptibile de a masca astfel de
zgomot. Cu toate acestea, de reducere a zgomotului la sursă este întotdeauna necesar.

Nivelului de zgomot sub apă, eventual confuz pește și, mai ales, marine,
mamifere.

Pe-shore structurale și de daune vizual pentru a coastelor la punctele de
contact (on-shore structuri, cablu submarin conexiuni la rețeaua
de linii, depozite de întreținere, etc.).

Scurgeri de uleiuri hidraulice anti-depunerile de substanțe chimice pot deteriora marin
viață.

Obstrucție la pescuit.

Distragere a atenției de lumini pe timp de noapte pentru păsări, inclusiv cele migratoare.

Pericol ori pentru bărci și de transport maritim, mai ales de la
halfsubmerged sau rupt structuri plutitoare cu vizibilitate redusă și radar
de profil.

Pericol de dispozitive plutitoare de rupere a lor de acostare și de a deveni un
necunoscut pericol pentru transport maritim.

La o scară foarte mare de implementare, modificări la curenți marini
și energie fluxuri ar putea fi în detrimentul ecologiei marine.
Impactul asupra pește este, de obicei, neutru și poate fi pozitiv, deoarece structurile
oferi zonele de reproducere și de protecție de pescuit comercial. Cele mai multe
modele de val centrala nu face rău individuale de pește. În același
mod, păsări marine ar putea găsi de bine structurile de bun venit. Posibile
efecte negative la câmpuri electrice în jurul valorii de cabluri submarine au fost
sugerat, dar până în prezent nu s-a obținut dovada.
Ca cu toate efectele, recunoașterea în faza de proiectare vă permite pentru a planificat
reducerea la minimum a efectelor negative și a crescut beneficiul pozitiv
impact.
Naționale și internaționale marine și de drept maritim are o istorie lungă,
fiind atât de complex și cuprinzător. Near-shore și off-shore
val de putere dispozitive sunt incluse în acest lucru, deoarece sunt bărci și nave.
Exemple sunt necesitatea de a include lumini de avertizare și a dispozitivelor de alte
mări, și nevoia de siguranță a personalului. Val de putere
dezvoltatorii se așteaptă, și, de obicei, bine ai venit, expres includerea val de
dispozitive de putere într-o astfel de legislație, astfel încât să poată planifica în consecință.
Normele internaționale în astfel de chestiuni sunt importante, deoarece
producătorii se așteaptă la piață dispozitive în întreaga lume. Tendința clară este mult
crescut 'pentru construcții, activitate la mare (de exemplu petroliere și gaziere offshore de
explorare și extracție, eoliene, a valurilor ferme, flux-curent de alimentare
tablouri, maree-gama de putere bariere). Cuprinzătoare de planificare este esențială
TWIDELL PAGINARE.indb 438
01/12/2014 11:38
§11.7 Sociale, economice și de mediu
439
REZUMAT CAPITOL
Valurile oceanului conține considerabile de energie mecanică, care poate fi valorificat mai ales în acele
locații în care resursa este mare și relativ aproape de mal pentru perioade lungi de timp (de exemplu, Nord
Atlantic coaste ale Americii, Canada și Europa, și coastelor de nord-est Asia de sud și
Australia). În astfel de locații, resursa este frecvent
~30 la 50 kW pe metru de lățime de undă față.
Dispozitivele pot fi clasificate ca punct de amortizoare, atenuatoare și terminatori. Printre provocările
de accesarea și utilizarea energiei valurilor sunt posibilitatea de deteriorare a dispozitivelor de extrem de
puternice valuri și dificultățile de a aduce puterea electrică de la mal; provocări care se adaugă la
complexitatea și costurile. Cu toate acestea, succesul este posibil, mai ales ca designerii și autoritățile de reglementare beneficiază
de
experiență cu alte stabilit structuri offshore offshore de petrol și gaze extracția și offshore
wind farms.
Teoria matematică de valuri de apă este bine dezvoltat; aceasta arată că puterea disponibilă în
adânc-valuri de apă este proporțională cu perioada de val și să-piața de înălțimea valurilor.
Acum sateliții măsurarea acestor parametri la nivel mondial pentru valurile oceanului, care beneficiază, de asemenea, de transport
maritim și
meteorologice înțelegere.
Există mai multe mecanisme prin care energia mecanică a valurilor este extras și transformat
de utile (electric) putere. Starea lor de dezvoltare variază de la studiile de laborator la creșterea
desfășurarea la scară comercială a produselor, dar nici unul nu sunt încă stabilite în utilizare la nivel mondial. Offshore,
desfășurarea de mai multe dispozitive în "centrele" face puterea de extracție mai ușoară și reduce costurile.
dacă toate aceste activități sunt de a exista alături de practicile stabilite de
transport maritim și de pescuit.
În termeni de costuri și stadiul de dezvoltare, val de putere de astăzi este la
aproximativ stadiul la care energia eoliană a fost acum 30 de ani. Din
experiența inițială plante, costul estimat al val de putere generat de
energie electrică încurajează optimismul. De exemplu, chiar înainte de 2006,
Sugarul și Pelamis instalații ambele acceptate contractele de furnizare
de energie electrică timp de 15 ani, la mai puțin de 7 p/kWh (
≈ NE-0,15 usd/kWh). Este
rezonabil să proiect cu o mai mare desfășurare, care se răspândește
costurile de dezvoltare pe mai multe unități, și cu incrementale inginerie
îmbunătățiri de pilot plante, aceste costuri pot înjumătăți în termen de zeci
de ani (un exemplu de curbe de învățare' discutate în §17.8).
Fiabilitate si costuri mici de exploatare sunt factorii cei mai critici în
realizarea redus costurile medii per kWh pentru sistemele care sunt capital
intensive (a se vedea Capitolul 17). Acest lucru este valabil în special pentru val de sisteme de putere,
care în mod necesar opera viguros în condiții de mare. Dacă un sistem este
distrus de o furtună în primii câțiva ani de funcționare, acesta nu va plăti
drumul său, și furnizorii de energie electrică nu va dori să investească în continuare, similare
dispozitive. Din fericire, inginerii acum poate să se bazeze pe experiența
offshore de petrol și windpower industrii pentru a 'ruggedize lor modele și
permite mai încrezători instalare și operare.
TWIDELL PAGINARE.indb 439
01/12/2014 11:38
440
Val de putere
ÎNTREBĂRI RAPIDE
Notă: Răspunsurile la aceste întrebări sunt în textul de la secțiunea relevantă
din prezentul capitol, sau poate fi ușor dedusă din aceasta.
1
Schiță mișcarea de mici elementar volume de apă la o
adâncime de val; apoi, de la schițe explica cum totalitatea unor astfel de
mișcări produce un val de mișcare.
2
Care este relația dintre grup și de fază (val) viteza de
o apă adâncă de val?
3
(i) Ce este o relație matematică între frecvența
și lungimea de undă de o apă adâncă de val? (ii) există un model matematic de
relația dintre lungimea de undă și amplitudinea de apă adâncă
valuri, și dacă nu, de ce nu?
4
Energia reportate într-o apă adâncă val de călătorie la
aceeași viteză ca val?
5
Cum puterea transmisă înainte într-o apă adâncă val
se referă la amplitudinea și lungimea de undă a undei?
6
Valurile mării sunt neregulate în amplitudine. Cum este înălțimea semnificativă a valurilor'
definit?
7
Cum de putere pe unitatea de val fața profundă-valuri de apă se referă
lor valuri cu o înălțime semnificativă?
8
Numele de trei clase principale de dispozitive pentru captarea energiei valurilor și principalele trei
locații.
9
Într-un Tapchan val de energie dispozitiv, nu apa de mare intra un rezervor
în principal din cauza de intrare canal modificări în lățime sau în profunzime?
10
Listă de trei sau juridice de planificare probleme care sunt importante pentru
desfășurarea de val dispozitive de putere.
PROBLEME
Notă: *indică o "problemă", care este deosebit de potrivit pentru clasa de
discuție sau grupul de tutoriale.
11.1
Prin luarea în considerare elementele de apă ridicate de la adâncimea z sub
nivelul mării la o înălțime z deasupra acestui nivel într-o creasta, arată că
energia potențială pe unitate de lungime pe unitatea de lățime de val fața în
direcția de val este:
r
=
E
og
P
1
4
2
11.2
Cum Fig. 11.11 (distribuție de val de mare putere cu privire
la frecvența) și ecuația (11.9) (relația dintre lungimea de undă la
frecvența) se referă la proiectarea unui Pelamis dispozitiv?
11.3
Fig. 11.20(a) arată o perspectivă schiță a unei Sonde de turbină;
Fig. 11.20(b) arată (schematic) o secțiune transversală a acestuia
simetric lama și mișcarea, așa cum este văzută de un observator fix.
TWIDELL PAGINARE.indb 440
01/12/2014 11:38
Probleme
441
Fig. 11.20
Turbinei Wells:
o
schiță,
b
mișcare de turbine blade (cum este văzută de un observator fix).
Alternativ
fluxul de aer
(b)
(a)
Simetric
folie
Unidirecțional
de rotație
Lama de
viteza
Debit de aer
viteză
Direcția
de rotație
Generator
Prin desen și analiza o lamă diagrama similar cu Fig. 8.12 în
cadrul rotative, cu turbină arată că este posibil ca
fluxul de aer pentru a genera un net înainte vigoare pe lama dacă
liftand-forțele de frecare sunt de potrivite magnitudine. (Indiciu: Face lama
setare unghi
γ
zero și trage F
roti
pentru fiecare direcție de
u
0
.
*11.4
Fig. 11.21(a) prezinta un dispozitiv pentru extragerea de energie din
mișcarea orizontală a apei în valuri. Un vane plane articulate despre o
axă orizontală de la Un (despre
l /8 sub medie la nivel de suprafață) oscilează
ca a indicat ca valurile afecteze pe ea. Experiment indică faptul că
un astfel de dispozitiv poate extrage aproximativ 40% din energie în sosite de
valuri; aproximativ 25% din energie este transmisă încoace (adică de la
apă în aval de vane) și aproximativ 20% este reflectată.
Salter (1974) a conceput "rață" se arată în Fig. 11.21(b), cu o
vedere la minimizarea pierderilor de o clapeta cu balamale. De "rață" se rotește
despre axa centrală la O. Sale stern este o jumătate de cilindru (cu o rază de o)
centrat în O (mai mică cu linie punctată continuă circular locus), dar
din punctul de jos forma se schimbă într-o suprafață care este
un alt cilindru centrat în O', mai presus de O. Această formă continuă
până când se ajunge la un unghi
θ vertical, moment în care se dezvoltă
într-o dreaptă tangentă care continuă deasupra suprafeței. Pentru
cazul prezentat, OO' = 0.5- o și
θ = 15°.
(a) Prin luarea în considerare mișcarea particulelor de apă care ar
apar în val în absența dispozitivului și privind
această formă de dispozitiv, explica cum pentru lungimi de undă
de ~4o la ~12un dispozitiv poate absorbi ~70% din
energia de intrare.
TWIDELL PAGINARE.indb 441
01/12/2014 11:38
442
Val de putere
(b) Până în 2004, aparatul a fost supus de laborator extinse și
de dezvoltare teoretică. Fig. 11.21(c) indică un
fullscale (o ~ 8m) sistemul ar putea uita în secțiune transversală. La exterior
corpul se mișcă (oscilează) în raport cu cilindrul interior. Sugerez să -
și justifica (i) un mod în care cilindrul interior ar putea fi făcute
într-un mod suficient de stabil punct de referință, și (ii) un mod în care
neregulate mișcare oscilatorie ar putea fi valorificate în utilizabile
energie pentru distribuție la mal.
NOTE
1
Vezi www.globwave.org, din care recunoaștem multe dintre informațiile din această secțiune.
2
Noi recunoaștem Gareth Thomas capitolul "teorie" în spatele conversie a energiei oceanice – un comentariu', în
Cruz (2008) pentru această clasificare.
3
Vezi http://people.bath.ac.uk/sb515/ pentru diagrame dinamice dintre aceste clasificări.
4
Vezi video de pe YouTube la clip www.youtube.com/watch?v=vG6R_R2YyAo.
5
Estimarea noastră este bazată pe date în Wang et al. (2002, fig. 3), disponibil la www.sciencedirect.com/science/
articol/pii/S0029801801000580, (vizualizate iunie 6, 2013).
6
Vezi www.pelamiswave.com/our-projects/project/1/E.ON-at-EMEC (ca la data de 5 iunie 2013) pentru astfel de detalii și
operaționale clipuri video.
Fig. 11.21
o
O clapeta cu balamale oscilează ca valurile afecteze din stânga;
b
un mod mai eficient dispozitiv (Salter e "rață") proiectat pentru a extrage mai multă energie din
valuri;
c
un sistem pentru extragerea de energie de la un full-scară de rață (~10m diametru).
(a)
(b)
Flotabilitate
tancuri
Balast

Waterfilled
rulment
Oțel
coloanei vertebrale
Putere
canistra
Val
direcție
(c)
o' +
o+
O
θ
TWIDELL PAGINARE.indb 442
01/12/2014 11:38
Bibliografie
443
BIBLIOGRAFIE
General
Brooke, J. (ed.) pentru Inginerie Comitetului Oceanic Resurse (2003) Val de Conversie a Energiei, Elsevier,
Oxford. Excelent studiu de atunci, de ultimă oră, cu susținerea teoriei și descrieri ale instalațiilor și
prototipuri.
Carbon Trust (2006) Ocean de Energie și a Energiei Valurilor de Proiectare Dispozitiv; și (2012) marea BRITANIE Val de
Resurse Energetice.
Acestea și numeroase alte rapoarte privind energia valurilor și a mareelor, cu referire în special în marea BRITANIE, sunt
disponibile la http://www.carbontrust.com/resources/reports/technology/marine-energy/.
Cruz, J. (2008) Val Ocean de Energie: starea Actuală și perspectivele de viitor, Springer-Verlag, Berlin. În limba engleză: o
serie de capitole editate de către experți în prezent de lucru pe dispozitive operaționale; include atât teoria, cât și practice
detaliu.
Enferad, E. și Nazarpour, D. (2013) Studiu de Caz Valuri, ch. 12 în Oceanul de Energie din surse Regenerabile și Revizuirea
Tehnologiilor, pp. 273-300, Intech Open Source la: http://cdn.intechopen.com/pdfs/42182/, vizualizat iunie 6,
2013. Excelent review conturarea teoriei și descrie influente aplicații.
Falnes, J. (2002) Valurile Oceanului și Sisteme Oscilante, Cambridge University Press, Cambridge. Minuțioase
analize fizice de valuri și extracția de val de putere; nivelul de cercetare.
McCormick, M. (1971) Val Ocean de Conversie a Energiei, Wiley, Chichester (1981) (republicat în 2007 de către Dover
Cărți). Inginerie ghid, cu fizice de bază de analiză.
Mei, C. C. (1989, revizuit edn 2005) Aplicate Dinamica Ocean Valuri de Suprafață, Lumea Științifică Publishing
Co. Pte. Ltd., Singapore. Teorie riguroasă.
Stevens, C., popescu, M. și Gorman, R. (2005) 'Ocean de recompense: energie din valuri și maree'., Apă Și
Atmosferă, 13(4).
Interes istoric
Energie Tehnologie Unitate de Sprijin (1992) Energia Valurilor Revizuire, ETSU, AEA Harwell, marea BRITANIE.
NEL (1976) Dezvoltarea de Puterea Valurilor – Un tehnico-economice Studiu, de Leishman, J. M. și Scobie, G. de
National Engineering Laboratory, East Kilbride, Glasgow, Raportul EAU M25.
Ross, D. (1995) Putere de Valuri, Oxford University Press, Oxford. Jurnalistice cont de istoria
val de energie, în special, inclusiv mașinațiunile de val de energie politică în marea BRITANIE. Încorporează și se extinde
mai veche a autorului, cartea de Energie din Valuri (1979), Pergamon, Oxford.
Referințe specifice
Bevilacqua, G. și Zanuttigh, B. (2011) 'Creativitate val convertoare de energie: aspecte generale și stadiul de
dezvoltare', AMS Acta ISSN: 2038-7954 Contributi di ricerca dell ' Alma Mater Studiorum – Università di
Bologna; a se vedea http://amsacta.unibo.it/3062/1/overtopping_devicex.pdf (vizualizate iunie 6, 2013).
Coulson, C. A. și Jeffrey, A. (1977) Valuri, Longman, London. Didactice teoretice text, parțial, având în apă
valuri; frumos scrise și clare. Disponibil ca download gratuit pe internet în 2013.
Glendenning, I. (1977) 'Energie de la mare', Chimie și Industrie, 592-599.
TWIDELL PAGINARE.indb 443
01/12/2014 11:38
444
Val de putere
Salter, S. H. (1974) 'Val de putere', Natura, 249, 720-724. Acum văzut ca un clasic de hârtie pentru val de putere. Mai târziu,
documentele de acord cu 'Salter duck' evoluții.
Shaw, R. (1982) Energia Valurilor – O Provocare de Design, Ellis Horwood, Chichester, și Halstead Press, New York.
Wang, D. J., Katory, M. și Li, Y. S. (2002) de Analiză și investigații experimentale privind hidrodinamica
performanță de uscat val-dispozitive de putere', Ocean Inginerie, 29, 871-885.
Reviste și site-uri web
Val de putere dezvoltare este publicat într-o gamă de inginerie și științe marine reviste. În plus, cele mai multe
analize este raportat la conferințe și seminarii de specialitate. Deosebit de utile sunt bienalei Europene de Val
și Energia Mareelor Conferințe (EWTEC, www.ewtec.org). Activitatea comercială este încurajat fiind în
sfera de RenewableUK (anterior Britanic Wind Energy Association (BWEA) (www.bwea.com).
Site-urile de dispozitiv dezvoltatorii sunt adesea informativ (de exemplu, Ocean Livrare de Energie (re Pelamis) la www.
oceanpd.com și Wavegen (re Sugarul) la www.wavegen.co.uk.
Alte site-uri utile includ: wikipedia (http://en.wikipedia.org/wiki/Wave_power).
Falnes curs pe mecanica de valuri și de extracție de putere este la
http://folk.ntnu.no/falnes/teach/wave/JF_introduction2010-06-28.pdf.
CONFIRMARE
Autorii mulțumesc Profesorului Falnes din Norvegia pentru comentarii utile la acest capitol.
TWIDELL PAGINARE.indb 444
01/12/2014 11:38

Maree-curent și maree-gama de putere


CONȚINUTUL
Obiective de studiu
445
§12.1 Introducere
447
§12.2 cauza mareelor
450
§12.2.1 lunar induse de reflux
450
§12.2.2 Perioada de maree lunare
453
§12.2.3 solar induse de reflux
și efectele combinate
454
§12.3 Accesoriu de maree
456
§12.4 Maree curent/flux de putere
459
§Teoria 12.4.1
459
§12.4.2 Dispozitive
461
§12.4.3 Blocaj efecte în
restricționat fluxul
463
§12.5 Maree-gama de putere
465
§12.5.1 Teoria De Bază
465
§12.5.2 Cerere
466
§12.6 Lume a mareelor site-uri
467
§12.7 Sociale și de mediu
aspecte
469
§12.7.1 Maree-gama de putere
469
§12.7.2 Maree-curent de alimentare
470
Rezumat capitol
471
Întrebări rapide
472
Probleme
472
Note
474
Bibliografie
474
Caseta 12.1 Tsunami
457
Caseta 12. 2 Blocarea efectelor pe turbina
de ieșire în canale înguste
464
CAPITOLUL

12
OBIECTIVE DE STUDIU

Apreciez de ce există două maree pe zi și în
ce interval de aceste valuri poate fi
considerabil îmbunătățită în anumite estuare și golfuri.

Explica cum curenții de maree transporta energia într-un
mod similar cu vânt, astfel încât un analog
teoria se aplică la extracția de această
sursă de energie regenerabilă.

Descrie unele dispozitive pentru extragerea de energie
din curenții de maree.

Explica teoria de a extrage energie din
ridicarea și căderea (gama) de maree, și de ce această
sursă de energie regenerabilă nu a fost foarte
larg exploatate până în prezent.
TWIDELL PAGINARE.indb 445
01/12/2014 11:38

www.shahrsazionline.com
446
Maree-curent și maree-gama de putere
LISTA DE FIGURI
12.1 Regiuni de mare a mareelor.
449
12.2 Mișcarea Lunii și Pământului.
451
12.3 fizice de Bază explicație de semi-diurne și maree diurne.
452
12.4 Compararea a trei tipuri diferite de "zile" care pot fi observate de pe Pământ:
(a) sideral și solare zi;
(b) sideral și lunar.
453
12.5 (a) variație Sinusoidală a mareelor.
(b) Tidal gama de variație pentru o lună de semi-diurne de maree. (c)
Poziții de Soare,
Luna și Pământul.
455
12.6 Mișcare de apă într-un val de maree.
457
12.7 Rezonanță accesoriu de un val într-un estuar.
458
12.8 Un reprezentant dispozitive pentru valorificarea curent de maree putere.
462
12.9 Ilustrează efectul asupra înseamnă fluxul de blocaj de o turbină într-un canal.
464
12.10 generarea de Energie din mareelor.
465
LISTA DE TABELE
12.1
Majore ale lumii energia mareelor site-uri și posturi de
467
TWIDELL PAGINARE.indb 446
01/12/2014 11:38
§12.1 Introducere
447
§12.1 INTRODUCERE
Nivelul de apă în oceane se ridică și cade în mod previzibil ca mareele din cauza
pozițiile relative ale Soarelui, Pământul și Luna. Deoarece astronomice
periodicities sunt cunoscute cu exactitate și efectele special
coastelor rămân constante, de predicție mareelor ritmuri și amplitudini este
matematic exact. Principalele perioade
t de maree sunt diurne la aproximativ
24 de ore și semi-diurne la aproximativ 12 ore 25 de minute. Schimbarea
în înălțime succesive între maree înaltă și joasă este cea a mareelor, R.
Aceasta variază între aproximativ 0,6 m in mijlocul oceanului, la aproximativ 10 m de la câteva
locații de continental mase de teren. Mișcarea apei
produce periodic curenții de maree, care poate ajunge la viteze maxime de ~5 m/s în
zonele de coastă și inter-insulare canale. Creșterea maree și a mareelor
la locații specifice permite două tipuri distincte de tehnologii pentru energie electrică
generație, și anume (a) mareelor-curent de alimentare (de asemenea, numit tidal stream-putere),
și (b) de maree-gama de putere. Considerăm ambele tehnologii în acest capitol,
în ciuda lor diferențe considerabile.
curenții de Maree poate fi valorificat cu unele dispozitive într-un mod similar,
pentru vant, deși, spre deosebire vântului, curenții de maree sunt previzibile în amplitudine
și frecvență. Astfel, după cum este arătat în §12.4.1, pentru vârf debitul u
max
mare
densitatea apei
r și presupunând că 40% de conversie a energiei electrice, de medie
puterea generată pe unitatea de suprafață de captare este:
r

q
u
0.1
max
3
(12.1)
De exemplu, pentru u
max
= 3 m/s,
q
~14 kW/m
2

. Generarea de energie este doar


atractiv, în cazul în care curenții de maree sunt relativ rapidă pentru că de (a) relativ
mare a mareelor, și/sau (b) cresterea vitezei de circulație a apei în strâmtori
în apropiere de insule, sau de estuar sau de lagună prize. Astfel mareelor actuala putere este
foarte specifice site-ului.
Maree-curent generatoare de plante pot fi construite offsite ca un standard de
module. Acest lucru poate fi poziționat pe site-ul fără semnificativă civilă
funcționează pentru a funcționa în mod individual sau ca un grup de peste o maree. Diverse
maree-sistemele actuale sunt în curs de dezvoltate cu sprijin financiar de la
guverne și de capital de risc, prezentată în §12.4.2. Multe proiecte
au fost sprijinite de Uniunea Europeană și de către autoritățile BRITANICE.
Maree-tehnologice actuale de dezvoltare de astăzi poate fi comparată cu
cea a vântului de tehnologie de la sfârșitul anilor 1970 și începutul anilor 1980, atunci când mai multe
forme diferite de turbine eoliene au fost studiate și înainte standard
modele comerciale evoluat. Ca push pentru dezvoltarea durabilă, cu emisii
generarea de energie electrică continuă, în următorii 20 de ani va clarifica
alegerea tehnologiilor și a vedea creșterea cererii.
Amplitudinea mareelor este valorificată pentru generarea de energie electrica prin captarea apei
în spatele barajului (de obicei numit un baraj) la maree înaltă într-un estuar bazinul
de zonă - O în spatele unui baraj sau barieră. Dacă apa de densitate
r expirarea
TWIDELL PAGINARE.indb 447
01/12/2014 11:38
448
Maree-curent și maree-gama de putere
prin turbine la reflux de perioada
t, puterea medie produsă
(§12.5.1) este:
P
= rAR
2
g/(2
t)
(12.2)
De exemplu, dacă Un
= 10 km
2
, R
= 4 m, t = 12 h 25 min, apoi
P
= 17 MW.
Evident, site-uri de mare gamă da cel mai mare potențial pentru energia mareelor,
dar alți factori vitali sunt oportunități de a integra puterea în cadrul
unei rețele, precum și costuri și beneficii secundare de construcție.
Astfel, dezvoltarea tidal gama de putere este, de asemenea, foarte specifice site-ului.
Maree-gama de putere a fost folosit istoric pentru mici puterii mecanice
dispozitive (de exemplu, în Anglia medievală și în China), dar interesul moderne
se concentrează pe scară largă producția de energie electrică. Cel mai cunoscut sistem este
de 240 MW
e

'La Rance' sistemul de la un estuar în Golful St. Malo,


Bretania, Franța, care a operat în mod fiabil din 1967, așa dovedind
fezabilitatea tehnică a acestei tehnologii pe scară largă. De obicei
baraj se extinde complet peste maree de admisie, dar poate fi folosit ca un
transport rutier sau feroviar de trecere, ca la 'La Rance'. În cazul în care navele trebuie să treacă, un sistem de
blocare este construit
în baraj. Prin urmare, costurile directe, mai ales pentru cele civile,
inginerie, sunt mari și, de obicei, necesita fonduri de la guvern, dar
costurile operaționale sunt mici pentru o durată de viață de cel puțin 100 de ani. Efectul de
un baraj este de natură să aibă impact considerabil asupra mediului ca
estuare cu mare mareelor tind la apă mică să aibă o vastă mlaștini
și zone umede cu o distinctiv de floră și faună, mai ales trecere prin vad păsări.
În ciuda studiilor de fezabilitate în ultimii 100 de ani concluzionând că
substanțială de energie electrică este posibilă din relativ putinele site-uri
cu mare mareelor (de exemplu, Estuarul Severn din marea Britanie ar putea produce
10% din naționale de energie electrică: a se vedea Fig. 12.1), implicațiile de cost de capital
și impactul asupra mediului au însemnat că foarte puțini tidal gama de sisteme
au fost puse în aplicare la o scară semnificativă.
Gama, debit și periodice comportament de maree în cele mai multe regiuni de coastă
sunt bine documentate și analizate, din cauza cerințelor de
navigare și oceanografie. Comportamentul poate fi prezis cu exactitate, în termen de
o incertitudine mai mică de ±4%, și deci energia mareelor este un foarte fiabile și
durabile sursă de energie curată, care este un avantaj major în comparație
cu alte surse de energie.
Provocările majore pentru toate formele de energia mareelor sunt după cum urmează:
1
Doar câteva site-uri sunt potrivite, și acestea pot fi îndepărtate de pe
cererea de energie.
2
Nepotrivire de directorul lunar condus perioade de 12 ore
25 de minute și 24 de ore 50 de minute cu om (solar) perioada de
24 de ore, astfel încât optim energia mareelor generație nu este în fază cu
cererea.
După cum sa menționat mai sus, maree-gama de putere (dar nu mareelor-curent de alimentare), de asemenea,
suferă de următoarele:
TWIDELL PAGINARE.indb 448
01/12/2014 11:38
§12.1 Introducere
449
3
Cerința de apă de mare debit la înălțimi mici,
necesitând multe special construite turbine set în paralel.
4
Foarte mari costurile de capital de cel mai mare potențial de instalații.
5
Potențial ecologic rău și de perturbare a extins estuare sau
regiuni marine.
Optimă pentru producerea energiei electrice din maree, turbinele ar trebui să
fie exploatate într-un mod regulat și în mod repetabil. Modul de funcționare
va depinde de scara de centrala, iar cererea și
disponibilitatea altor surse. Foarte multe variante sunt posibile, dar anumite
generalizări se aplică:
o
Dacă mareelor energia electrică produsă este pentru uz local, apoi alte asigurat
surse de alimentare trebuie să existe atunci când energia mareelor este indisponibil.
b
Dacă electricitatea produsă poate hrăni într-o grilă mare și deci formează un
proporțional minore sursă într-un sistem național, apoi
previzibil energia mareelor variante poate fi scufundat în national
a cererii.
-90
180
90
Latitudine ø/grade
Latitudine
λ
/grade
0
90
180
-60
-30
0
30
60
9m
57 GW
11m
29 GW
10m
2 GW
8m
11 GW
7m
16 GW
6m
6 GW
5m
5m
7m
6m
5m
7m
90
Fig. 12.1
Regiunile de mare a mareelor (în verde închis). Pentru unele regiuni, de asemenea, indicat este valoarea medie
a mareelor și înseamnă potențialul tehnic pentru energia mareelor în golfuri și estuare (puncte negre)
de-a lungul coastei. Regiunile de mare a mareelor sunt în mod necesar, de asemenea, regiunile de înaltă maree
curent, dar unele site-uri specifice (de exemplu, între insule, ca în Indonezia) au puternice maree
curenți chiar și fără mare a mareelor.
Sursa: Adaptat de la OpenHydro.com și Sørensen (2011).
TWIDELL PAGINARE.indb 449
01/12/2014 11:38
450
Maree-curent și maree-gama de putere
c
Dacă cererea imediată nu este fixat umane (solar) perioada de
24 de ore, apoi energia mareelor poate fi folosit ori de câte ori este disponibil. De
exemplu, dacă puterea electrică produce un combustibil (de exemplu, hidrogen) sau
oferă de desalinizare a apei (de exemplu, prin osmoza inversa), atunci o astfel de
decuplare de furnizare și utilizare pot apărea.
Următoarele secțiuni evidențiază fizice înțelegere a mareelor și
a mareelor. Cititorii interesați doar de putere generatoare de instalații
ar trebui să rândul său, direct la §12.4 și §12.5. Aspectele sociale și de mediu
ale tehnologiilor sunt prezentate în §12.7.
§12.2 CAUZA MAREELOR
1
Analiza fluxului de comportament a fost dezvoltat de numeroși
matematicieni și aplicate fizicieni, inclusiv Newton, Aerisit,
Laplace, George Darwin (fiul lui Charles Darwin) și Kelvin. Vom folosi
Newton fizică teorie pentru a explica fenomenele de maree. Cu toate acestea,
ziua de analiză și predicție depinde de metodă matematică
de analiză armonică dezvoltat de Lordul Kelvin în Glasgow. Un complet de
fizică înțelegere a mareelor dinamica nu a fost încă atins,
datorită topologice complexitatea bazine oceanice. Această secțiune
oferă doar un cont de bază.
Mările sunt lichide avut loc pe o suprafață solidă de rotație a Pământului de către
gravitație. Atracția gravitațională a Pământului cu Luna și
Soarele tulbură aceste forțe și mișcări, astfel că mareele sunt produse. Tidal
power este derivat de la turbine set în acest lichid, astfel încât valorificarea cinetică în
energie de rotație a Pământului. Chiar dacă toată lumea majore energia mareelor
site-uri au fost utilizate, acest lucru ar duce la un plus de incetinire a Pământului
de rotație cu nu mai mult de o zi în 2000 de ani; acest lucru nu este un semnificativ
plus de efect.
§12.2.1 lunar induse de reflux
Luna și Pământul se învârte în unul despre celălalt în spațiu (Fig. 12.2), dar
din masa Pământului este de aproape 100 de ori mai mare decât Luna
masă, miscarea Lunii este mai mult aparentă. Centrul de revoluție este
la O, astfel încât:
=''
ML M L
'=
'+
Am
MD M
M
/(
)
(12.3)
'=
Am
4670 km.
Pământul este raza medie este de 6371 km, deci ideea de revoluție este în interiorul
suprafața Pământului.
TWIDELL PAGINARE.indb 450
01/12/2014 11:38
§12.2 cauza mareelor
451
Un echilibru de atracție gravitațională și forța centrifugă menține
Pământ–Lună de separare. Dacă constanta gravitațională este G,
w
w
'
=
=''
GMM
D
ML
M-AM
2
2
2
(12.4)
Dacă masa Pământului ar putea fi amplasat în centrul Pământului
E, atunci fiecare element de masă ar fi în poziția de echilibru cu
privire la Luna. Cu toate acestea, masa de Pământ nu este toate la un
moment dat, și deci nu este totul în acest echilibru. Materialul cel mai departe de
Luna la Y (Fig. 12.2) experimentează o creștere spre exterior forța centrifugă
cu distanța de rotație (r
+ L') și o scădere a forței gravitaționale din
Luna. Material de cea mai apropiată Lună de la X-a crescut
forța gravitațională spre Lună, plus forța centrifugă, de asemenea, față de
Luna, dar a redus, din cauza reducerii rotație distanța (r -L').
Materialul solid de Pământ experiențe aceste schimbarea forțele ca
Luna se învârte, dar este loc doar cu mici deformări structurale
forțele de stare solidă. Lichid pe suprafața este însă liber să se miște,
și este această mișcare relativă față de suprafața Pământului, care provoacă
mareele. Dacă Luna este în planul ecuatorial al Pământului, apa din
largul mării încearcă să grămadă împreună pentru a forma vârfuri în punctele X și Y,
la cel mai apropiat și cel mai îndepărtat de Lună. Solidă a Pământului s-ar roti cu
o perioadă de o zi sub aceste două vârfuri (Fig. 12.3(a)). Astfel, cu
nici un alt efect care apar, fiecare mare-poziție acoperită de Pământul va
experimenta două se ridică și două căderi de nivel de apă ca pe Pământ se transformă
Fig. 12.2
Mișcarea Lunii și Pământului.
Luna:
Luna
X
Z
O
E
Y Masă M'
M
M
M'
Masa
Pământ (văzută de-a lungul
axei de rotație)
M
=
7.35
×
10
22
kg
M'
=
598
×
10
22
kg
D
=
Am
+
L'
=
384
×
10
6
m
r
=
6.38
×
10
6
m
Pământ:
L'
=
4670 km
De rotație a pământului și a lunii privire la O frecvență
ω
.
Am
D
r
L'
ω
TWIDELL PAGINARE.indb 451
01/12/2014 11:38
452
Maree-curent și maree-gama de putere
prin cele două vârfuri. Aceasta este semi-diurne (jumătate de zi) maree. Rețineți că
rotația zilnică a Pământului în jurul axei sale, are în primul rând scopul de efect, ca
atare, în producerea mareelor.
Analiza in dinamica a centrifuge și forțele gravitaționale (a se vedea Problema
12.1) arată că net " dus " forța care acționează pe o masă m de la X în
Fig. 12.2 este:
w
=
+
'




F
dl.
Am
D
1
2
X
2
(12.5)
și că F
Y
, similar cu " dus " forța care acționează pe partea opusă a
Pământ la Y, este numeric egală cu F
X
.
În general, pentru mari oceane, doi lunar mareelor variază au loc în fiecare zi de
aproximativ egală amplitudine. La reflux pe acest echilibru valul model
lunar legate de forța este domnul
w
2
și atât de valul de sensibilizare vigoare în termen de (12.5)
este dl.
w
2
2L'/D. Se poate demonstra (vezi Problema 12.2) că aceasta produce o
maxim de echilibru a mareelor 0,36 m.
Există trei motive principale pentru real mareelor comportamentul este diferit
din acest simplist 'echilibru maree' explicație:
1
În practică, vârfuri de apă nu se pot deplasa suficient de repede (~1600 km/h)
de a rămâne la jumătatea Lunii (vezi Problema 12.5).
2
Luna nu este, de obicei, în planul ecuatorial al Pământului (Fig. 12.3(b)),
și deci o componentă diurne de valul apare.
3
Rezonanțe de circulație apă apar de-a lungul oceanelor și, mai ales,
în apropiere de platoul continental și în estuare, care produc distincte
Fig. 12.3
Fizice de bază explicație de semi-diurne și maree diurne.
o
Simplu teoria echilibrului valul cu Luna în planul ecuatorului Pământului, P,
experiențe două egale mareele fiecare zi (semi-diurne maree).
b
În mod normal, Luna nu este în planului ecuatorial, și deci, de exemplu, la P
nu poate fi doar unul vizibil valul fiecare zi (maree diurne).
Luna
(a)
(b)
X
X
Y
Y
P
P
Luna
Pământ axa
Pământul e
planul ecuatorial
Pământul e
planul ecuatorial
TWIDELL PAGINARE.indb 452
01/12/2014 11:38
§12.2 cauza mareelor
453
îmbunătățiri ale mareelor. Vom arăta în §12.3 care rezonanță
îmbunătățiri în anumite estuare sunt de o importanță vitală pentru
tidalrange instalații de putere.
În plus, funneling de apă curenți între insule și în apropierea
coastelor poate crește maree-curent de viteze, astfel încât să beneficieze de maree-curent
de alimentare de plante.
§12.2.2 Perioada de maree lunare
Pentru a calcula perioadele de maree, trebuie să se definească cu atenție o " zi " (a se vedea Fig. 12.4).
La un punct de pe Pământ, o zi solară este intervalul dintre momentul în care
Soarele traversează meridionali avion de la-O pe o anumită zi și atunci când se
face în ziua următoare. Această perioadă variază de fapt prin
an din cauza neregulilor de pe orbita Pământului, și atât de comună în
unitatea de timp, adică solare zi t
S
, este definit ca fiind intervalul medie
peste un an întreg. Valoarea sa este definit ca fiind la exact 24 de ore, adică
×
×
t = 24.0000 h
60 min
h
60 s
min
= 86400s
S
(12.6)
Fig. 12.4
Compararea a trei tipuri diferite de "zile" care pot fi observate de pe Pământ:
o
sideral și solare zi;
b
sideral și lunar.
Solar zi este de 24 de ore, mai exact, prin definiție, sideral este puțin mai scurt și
lunar ceva mai lung. Diagramele nu sunt la scară.
A se vedea, de asemenea, Fig. 2.4 descrierea meridionali avion.

'fix
stele

'fix
stele
M
(a)
(b)
O'
θ
2
θ
1
θ
1
E
O
S
O
E
E'
O"
O'
M'
TWIDELL PAGINARE.indb 453
01/12/2014 11:38
454
Maree-curent și maree-gama de putere
Pe zi sideral t* este definit ca fiind intervalul mediu între
succesive tranzitul de o stea fixă, adică unul atât de îndepărtat că aparentă
mișcare relativă a Pământului este neglijabilă. Sideral zi, prin urmare, este
'adevărat' perioada de rotație a Pământului, așa cum este văzută de un observator distant.
În mod similar, adică ziua lunar t
M

este definit ca intervalul mediu dintre


aliniamente succesive de E, O și Luna centru. Fig. 12.4(b) prezinta
la "fictivă" înseamnă Luna M se deplasează uniform pe o orbită circulară în jurul
Pământului. Într-un timp t
M
, Luna se mișcă printr-un unghi
q
2
de la M
să-M', în timp ce pe Pământ se rotește prin intermediul a 2
p+q
2
. Astfel, văzut de un
observator distant,
q
p

t
T*
T
t*
t*
2
=
=
2
M
M
(12.7)
unde T*
= 27.32t
S

este sideral lună, adică "adevărat" lunar. T*


este perioada de revoluție a Lunii despre Pământul este văzut
de către un observator distant. Acest lucru este mai scurt decât lună înregistrată de
un observator de pe Pământ (T
M
= 29.53 zi) din cauza Pământului se deplasează în jurul
Soarele. Ecuația (12.7) implică faptul că:

t
t*
(t* / T*)
=
1
M
(12.8)
= 89428s = 24h 50min 28s
Acesta este principalul motiv pentru care valul de mare într-un anumit loc este, de obicei,
aproximativ 50 de minute mai târziu decât a fost în ziua precedentă. Astfel de
perioadă este numit 'diurn', deoarece este aproape de 24 de ore.
Cu un argument similar la care conduce la ecuația (12.8), se poate arăta
care:
=
+
t
t
tT
*
1(/)
S
S
S
(12.9)

= 86164s = 23h 56min 4s

(a se Vedea Problema 12.3).


§12.2.3 solar induse de maree și efectele combinate
O suplimentare de două ori pe zi valul solare este indusă printr-o perioadă de o jumătate de 24 de ore
solare zi. Cele două efecte pot fi comparate din cauza mareelor este
proporțională cu diferența de forța gravitațională din Luna
sau Soarele pe diametrul d al Pământului. Dacă M
M
și M
S
sunt
masele de Luna și Soarele la o distanță de Pământ de D
M
și
D
S
atunci, fie pentru sistem:
forța gravitațională

M/D
2



=−
F
D
d
Md D
diferența în vigoare
2
/
3
(12.10)
TWIDELL PAGINARE.indb 454
01/12/2014 11:38
§12.2 cauza mareelor
455
(a)
(b)
(c)
S
S
S
sau
M
Maree de cvadratură
M
M
S
Sizigii
E
E
E
E
M
sau
1
0
1
2
3
4
5
3 5 7 9 11 13 15 17 19 21 23 25 27 29 31
Zi din luna
∼14 zile
Nivelul mării
Adică maree înaltă
(R
s
− R
n
)/4
R
n
/2
4
R
s
+
R
n
m
Gama
2
R
s
Fig. 12.5
o
Variație sinusoidală a mareelor.
b
Tidal gama de variație pentru o lună de semi-diurne de maree. Mare gama de la
sizigii, pe distanțe mici la maree de cvadratură.
c
Poziții ale Soarelui (S), Luna (M) și Pământ (E) care produc primăvara și maree de cvadratură
de două ori pe lună.
TWIDELL PAGINARE.indb 455
01/12/2014 11:38
456
Maree-curent și maree-gama de putere
Raportul lunar de rază R
M
și gama solar R
S
este prin urmare:
=




R
R
(M / D )
(M / D )
=
D
D
M
M
M
S
M
M
3
S
S
3
S
M
3
M
S
(12.11)
×
×




×
×




=
=
1.50 10 m
3.84 10 m
7.35 10 kg
1.99 10 kg
2.2
11
8
3
22
30

de exemplu gama de valul solare este de 2,2 ori mai puțin decât intervalul lunar
de maree, care, prin urmare, predomină.
Solar valul se mută în și în fază cu fluxul lunar. Atunci când
Soarele, Pământul și Luna sunt aliniate, în colaborare, lunar și solar mareele
sunt în fază, deci producătoare de maree de rază maximă. Acestea sunt nume
'sizigii' de rază maximă care apar de două ori pe lunar (synodic)
lună în momente atât de plină și lună nouă (Fig. 12.5).
Când Soarele/Luna și Pământ/Pământ direcții sunt perpendiculare (în
cuadratură) intervalele de mareele sunt cel mai puțin. Acestea sunt numite 'pnapm
maree' că din nou au loc de două ori pe synodic lună. Dacă valul de primăvară este
considerat a rezultat din suma lunar și solar mareele, și cvadratură
valul de diferență, atunci raportul de primăvară a pnapm variază ar putea fi
de așteptat să fie:
=
+

=
R
R
(primăvara)
(pnapm)
1 (1/ 2.2)
1 (1/ 2.2)
2.6
s
n
(12.12)
În practică, dinamice și efecte locale modifica această naiv model, și
raportul de primăvară a pnapm gama este mult mai frecvent despre 2.0. Sizigii
la Luna perigeu avea o raza mai mare decât sizigii la apogeu,
și o combinație de efecte, inclusiv vânt, pot să apară de a provoca
valuri neobișnuit de înalte.
§12.3 ACCESORIU DE MAREE
In mijlocul oceanului, a mareelor este de numai aproximativ 0,6 m și curenții de maree sunt
neglijabile, astfel încât generarea de energie este total nerealist. Cu toate acestea, în apropiere de numeroase
estuare și alte caracteristici naturale, îmbunătățirea tidal
gama și curenții de maree se poate produce prin: (1) funneling de maree (ca cu
soundwaves într-un vechi de modă în formă de trompetă auditiv); (2)fluxul de
perturbare lângă insule și țărmuri neregulate, și (3) de rezonanță
de cuplare la frecvențe naturale de circulație a apei în zonele de coastă contururi și
estuare. Acest local accesoriu este esențială pentru energia mareelor potențial;
subliniem acest moment cel mai puternic.
Mareelor obișnuite induse de circulație a apei din mare, are forma unui
anumit tip de perturbare a numit un 'val'. Întreaga coloană de
apă de suprafață de la mare pat poate propaga la unison (Fig. 12.6). La
TWIDELL PAGINARE.indb 456
01/12/2014 11:38
§12.3 Accesoriu de maree
457
maree viteza c se referă la accelerația gravitațională g și mare
adâncime h ca:
c
= √(gh)
(12.13)
(Derivarea (12.13) este dat pe site-ul pentru această carte.) Astfel
c ~750 km/h peste marile oceane, care au o adâncime de ~4000 m. Aceasta
viteza este mult mai mică decât viteza aparentă a Lunii (1670 km/h la
ecuator), astfel încât nu există nici o cuplare în marile oceane pentru armat
de maree mișcare.
Fig. 12.6
Mișcare de apă într-un val de maree; elementar secțiune de mare grosime a
D
x

, adâncime h
și lățime b (de-a lungul y axa).
Adâncime
Viteza u

x
z
=−
h
z
z
x
z
=
0
CASETA 12.1 TSUNAMI
Vulcanice subacvatice sau cutremur activitate poate induce un mod liber de înmulțire 'seismică mare val' în adâncul
oceanelor corect numit un tsunami, dar, uneori, incorect numit un 'val' în ciuda faptului că există
nici o relație de cauzalitate cu mareele. Un tsunami este inițiat de către un relativ localizate, dar extreme, bruscă
schimbare în înălțime de pe fundul mării, care injecteaza un imens impuls de energie pentru o perioadă scurtă de relativ
orizontală distanța pe fundul mării. Rezultat șoc ' creează un puls (val) de circulație, care
cuprinde întreaga adâncime. Din punct de vedere matematic, este echivalentul a o 'mică adâncime adâncime de undă (cu
l >> adâncime), în cazul în care "mici" trebuie să fie interpretată în comparație cu oceanul adâncime de ~4000 km.
Val se răspândește rapid la viteza c
= √(gh) și lungimea de undă ~150 km. Când tsunami-ul a ajunge la
scăderea adâncimii mării lângă țărm, de frecare la mare pat incetineste val și așa scurtează
lungimea de undă, cu consecința crescut rapid suprafață amplitudinea sa poate 30 m. Această
amplitudine va fi evidente la coasta ca poate o excepționale fluxul de apă de mare, urmat rapid
de mare și deteriorarea rupere valuri.
Având în vedere solare și lunare forțelor implicate în normal maree, nici nu este
în formă de un impuls, deci nu 'tsunami-ca' comportament se produce (cf. Caseta 12.1).
Singura posibilitate de mișcare este îmbunătățită de natural de maree mișcare pentru a
fi în rezonanță cu solar și lunar forțe. Dar, cum se vede de pe Pământ,
Soarele se mișcă deasupra capului la ~2000 km/h, iar Luna de la ~60 km/h.
Prin urmare, maree forțându-propuneri pentru lunar și solar induse de mareele
nu, în general, coincide cu cerințele pentru o săditor liber
TWIDELL PAGINARE.indb 457
01/12/2014 11:38
458
Maree-curent și maree-gama de putere
val în ocean adânc și atât de rezonant de îmbunătățire a
forțat mișcare nu are loc în oceane.
În anumite estuare și golfuri, de rezonanță pot să apară, însă, și foarte
vizibile schimbări în mareelor mișcare, prin urmare apar. Rezonant
accesoriu de maree în estuare și golfuri are loc în același mod ca și
rezonanța sunetului-valuri în deschise și închise de conducte (de exemplu, așa cum se arată
în Fig. 12.7). Rezonanță cu mare mareea se produce atunci când:
am
=
Lj
j
/ 4, un ciudat întreg
(12.14)
Frecvența naturală de rezonanță f
r
și perioada T
r
este dat de:
am
=
=
f
T
c
1
r
r
(12.15)
Deci:
am
=
=
=

T
c
Am
jc
Am
j gh
4
4
()
r
(12.16)
Rezonanța apare atunci când această perioadă natural este egal forțată perioada de
maree în largul mării T
f
, caz în care:
=


=

T
Am
j gh
Am
h
j
Tg
4
()
;
4
f
f
(12.17)
Semi-diurne de maree perioadă este de aproximativ 12 ore, 25 minute (45.000 de s), deci
rezonanță pentru j
= 1 are loc atunci când:

=

=

Am
h
45000s
4
(9.8 ms ) 36000m
2
1/2
(12.18)
De obicei, dacă apare la toate, astfel de accesoriu apare în râul estuare și
ocean golfuri, la fel ca în Estuarul Severn (Exemplul 12.1). Cu toate acestea,
există un mic accesoriu generală pentru întregul Ocean Atlantic.
Deschide mare
Teren
Am
λ
/4
Fig. 12.7
Rezonant accesoriu de un val într-un estuar, vedere în plan. Idealizat golful constantă
adâncimea h. Amplitudinea mareelor este indicat pentru un sfert de lungime de undă de rezonanță.
TWIDELL PAGINARE.indb 458
01/12/2014 11:38
§12.4 Maree curent/flux de putere
459
A LUCRAT EXEMPLU 12.1 REZONANȚĂ ÎN ESTUARUL SEVERN
Râul Severn estuar dintre țara Galilor și Anglia are o lungime de
aproximativ ~200 km și o adâncime de aproximativ 30 m, astfel:


×


Am
m
0
h
200 10
(3 m)
36400m
3
1/2

(12.19)
Ca urmare, nu este aproape de potrivire a estuarului frecvența de rezonanță
cu frecvența normală a mareei dat de (12.19), și atât de mare amplitudine
a mareei propuneri de la 10 la 14 m gama apară.
În practică, estuare și golfuri nu au uniformă dimensiunile implicite
în calculele noastre, și de analiză este extrem de complicat. Acesta devine
necesar pentru a modela condiții: (1) în laborator val de tancuri folosind
atent tehnici de scalare, și (2) de analiză teoretică. Una dominantă
considerare pentru energia mareelor instalații este de a descoperi cât de obstacole și
baraje va afecta rezonanță accesoriu. Pentru estuarul Severn,
unele studii au ajuns la concluzia că barierele de o anumită configurație
ar reduce amplitudinea mareelor și, prin urmare, puterea disponibile; cu toate acestea, alte
studii din alte configurații au ajuns la concluzia că intervalul ar fi
crescut. Construcția de maree-gama scheme de alimentare este prea
scump pentru a permite greșeli să apară în înțelegerea acestor efecte. În
schimb, modularitate de maree-dispozitive de curent permite domeniul de aplicare pentru
"învățare prin practică".
§12.4 MAREE CURENT/FLUX DE PUTERE
În apropierea coastelor și între insule, mareele pot produce apa puternic
curenți care pot fi luate în considerare pentru generarea de energie. Acest lucru poate fi
numit tidal-curent, maree-stream sau mareelor-fluxul de putere. Puterea totală
produsă nu poate fi foarte mare la nivel național, dar generatie la cele mai competitive
prețuri de export către o grila de utilitate sau pentru consumul local este posibil la
unele site-uri, mai ales de la tablouri de dispozitive similare pentru a parcurilor eoliene.
Prin urmare rafală de dispozitiv de dezvoltare descrise mai jos.
§Teoria 12.4.1
Teoria mareelor-actuala putere este similar cu energie eoliană (a se vedea Capitolul 8),
de la bază dinamica fluidelor este același atât pentru apă și aer în 'open
flow, respectiv a fluxului nu este constrânsă într-o conductă și pentru hidroenergie. La
avantaje sunt: (a) previzibil vitezele de lichid și, prin urmare,
utilizat de generare de energie, și (b) densitatea apei aproape de 1000 de ori mai mare
decât aerul și, prin urmare scară mai mică turbine. Principalele dezavantaje sunt: (a)
mici viteza fluidului; și (b) în mod intrinsec dificil mediului marin.
TWIDELL PAGINARE.indb 459
01/12/2014 11:38
460
Maree-curent și maree-gama de putere
Densitatea de putere în curent de apă este, din (8.3),
r
=
q
u / 2
3
(12.20)
De exemplu, pentru o maree sau curentul de viteza de 3 m/s,
q
= (1025 kg m
-3
)(27 m
3
s
-3
)/2
= 13.8 kW m
-2

Doar o fracțiune Cp a puterii în curent de apă pot fi transferate


la putere utilă, în cazul în care (ca și pentru energie eoliană) C
p
este coeficientul de putere
definită în (8.6) prin:
r
=
P
COu
/
T
p
1
2
0
3
(12.21)
în cazul în care u
0
este fluxul continuu de viteză, Un este evident zonă de
turbina (în plan perpendicular pe u
0
) și P
T

este puterea mecanică


de ieșire a turbinei. Pentru un singur izolat turbina, Betz analiză
a §8.3 arată că

C
0.59
p
; în practică, turbine comerciale au
C
p
~0.40.
Curent de maree vitezele variază cu timpul aproximativ ca:
pt
=
uu
t
sin(2 / )
max
(12.22)
în cazul în care
t este perioada de natural de maree, 12 ore, 25 minute pentru o semi-
diurne maree, și u
max
, este viteza maximă de periodice curente.
Generarea de energie electrică pe unitatea de secțiune transversală, prin urmare, poate fi
în medie:


hr
pt
h
r
t
p
t

=
t
t
=
=
=

q
u
t
t
t
u
2
sin (2 / )d
d
( / 2)
( / 3 )(4 / )
t
t
t
max
3
3
0
/4
0
/4
max
3

(12.23)
hr
=
u
(
)/4
max
3

Presupunând o eficiență
h = 40% pentru conversie de flux de maree puterea de a
de energie electrică, atunci:
r

q
u
0.1
max
3
(12.24)
Pentru un dispozitiv care ar putea genera energie in reflux (spre exterior) și
de curgere (spre interior) curenții de maree, și cu un curent maxim de 3 m/s,
q
~2.8 kW/m
2
. Cu un curent maxim de 5 m/s, care apare într-o foarte
câteva inter-insulare canale,
q
~14 kW/m
2
; în cazul în care a interceptat zona este un cerc
din zona de 100 m
2

(de exemplu, raza de 5,6 m), atunci numărul total de putere medie
generație ar fi de 1,4 MW. (Putem nota că pe cele mai multe site-uri, pentru a obține o
medie similare de producție de energie de la o turbină eoliană ar avea nevoie de unul
cu o capacitate nominală ~4 MW de capacitate și, astfel, o lama raza de ~60 m (vezi
Tabelul 8.1).)
Periodice natura generarea de energie ar duce la atâ-
ări, dar am act de faptul că maree putere gal-uri despre
p/2 spate gama
putere dintr-un singur bazin, astfel încât cele două sisteme ar putea fi complementare.
TWIDELL PAGINARE.indb 460
01/12/2014 11:38
§12.4 Maree curent/flux de putere
461
§12.4.2 Dispozitive
La momentul actual, curent de maree puterea nu este un dovedit, în general,
de tehnologie comerciale. Cele mai multe tipuri de dispozitive sunt de dezvoltare; cu toate acestea,
unele prototipuri cu capacități de până la ~2 MW au fost generatoare într -
o grilă în mod obișnuit, cu planuri de a intensifica în matrice similare la parcurile eoliene
(a se vedea Tabelul 12.1 în §12.6). Exemple sunt date mai jos de fiecare dintre mai multe
clase de dispozitive.
Multe dintre curent de apă sisteme de conversie a energiei seamănă cu eoliene
turbine eoliene generatoare. Cu toate acestea, marine turbine trebuie să fie proiectate pentru
inversarea fluxurilor, cavitație și dure marin subacvatic condiții (de exemplu,
sare de coroziune in apa, resturi, reziduuri, etc.). Axial-flow turbine trebuie să fie
în măsură să răspundă la inversarea direcții de curgere, în timp ce cross-flow turbine într -
un reglabil cabina continua să funcționeze indiferent de fluxul de curent
direcție. Axial-flow turbine fie invers nacela direcția de aproximativ 180°
cu alternative maree sau, alternativ, nacela are o poziție fixă cu
palele rotorului schimbarea pas să accepte inversarea fluxului. Un important
aspect de design este de a permite întreținerea (de exemplu, având la activ
o parte a sistemului de naștere din apă pe suporturile sale).
(a) Clasa 1: axa Orizontală
Majoritatea maree-curent dispozitivele în funcțiune sunt de acest tip, cu
câteva dintre cele mai promițătoare start-up companii acum preluat de către
principalii furnizori de electricitate-echipamente de generare, a căror ingineri
și finanțatori folosi experiența lor anterioară de vânt și hidro turbine.
Primul din lume comercial maree-curent turbina exportul
de energie electrică de la rețea la rețea a operat în gura mare de Strangford
Lough în Irlanda de Nord, din 2008. În esență există două
axe orizontale twin pale de turbine generatoare a avut loc pe brațul orizontal, care poate
fi ridicat din apă pentru instalare și întreținere (Fig. 12.8(a)).
Teren de lame este ajustat pentru a se potrivi fie refluxul sau fluxul de
condiții ale ciclului mareelor. Lichid-dinamica acestei turbine de apă este similară
cu cea a axei orizontale de turbine eoliene (a se vedea Capitolul 8). De exemplu,
vârful-speed ratio (raport de lama-indicație de viteză în apă cu viteza de
apa, curent) trebuie să fie optimizat și rămân constante ca apa
schimbări de viteză.
Rotor capace (de asemenea, cunoscut sub capota metalică sau conductele) de a spori
hidrodinamice de performanță prin creșterea vitezei de curgere prin rotor
și reducerea sfat pierderi (pentru motivele prezentate în Caseta 12.2). Mai multe
promițătoare dispozitive (de exemplu, de OpenHydro (Fig. 12.8(b)) să includă astfel de
conducte. Pentru a fi benefic punct de vedere economic, energie suplimentare de captare trebuie să
compenseze costul de giulgiul pe durata de viață a dispozitivului.
TWIDELL PAGINARE.indb 461
01/12/2014 11:38
462
Maree-curent și maree-gama de putere
(a)
(b)
Fig. 12.8
Un reprezentant dispozitive pentru valorificarea curent de maree putere.
o
SeaGen, ax orizontal tip (Siemens Marine current Turbines, Bristol, Anglia).
b
OpenHydro, ax orizontal tip.
c
Kobold, axa verticală tip.
Vezi textul pentru detalii suplimentare ale acestor dispozitive.
(c)
TWIDELL PAGINARE.indb 462
01/12/2014 11:38
§12.4 Maree curent/flux de putere
463
(b) Clasa 2: cu ax Vertical
Acolo au fost mult mai puține propuneri pentru verticală-axa mareelor dispozitive decât
axa orizontală (ca cu energie eoliană). La Kobold turbină (Fig.12.8(c))
dezvoltat în Italia este un astfel de dispozitiv, cu prototipuri de operare din
2009 în Italia (Strâmtoarea Messina), China (Jintang Strâmtoarea) și Filipine
(Cebu, cu un curent maxim de 4 m/s). Acesta dispune de un special concepute
aripi portante, cu lama unghiuri controlat de o serie de pârghii pentru a
menține un unghi optim de atac.
2
(c) Clasa 3: cu Piston lama
Principiul este că forțele de apă o farfurie sau "lama" în sus și în jos, sau
lateral, într-un curent. Aceasta acționează o roată sau pompă hidraulică pentru a
trece de alimentare de la un generator.
În Puls Mareelor dispozitiv, fiecare dintre cele două lame este orizontală și
se deplasează pe verticală în flux, conectate prin intermediul unei cutii de viteze de la un
generator. Avantajul este susținut pentru a fi faptul că sistemul va funcționa
cu lame mari în adâncime relativ mică de apă pentru a produce putere semnificativă
(de exemplu, 1.2 MW la 18 m adâncime, 5 MW în 35 m adâncime). În timpul funcționării,
sistemul se așează pe fundul mării și este complet scufundat chiar în mică adâncime
de apă. Cu toate acestea, pentru întreținere, sistemul poate veni la suprafață
fără a fi nevoie de macarale și complicat nave offshore – efectuarea
lucrărilor de întreținere simplă. O demonstrație dispozitiv de 100 kW
capacitate a operat în estuarul Humber, Anglia de est, începând din
2007. Desfășurarea de un full-scară' 1.2 MW mașină este de așteptat.
12.4.3 Blocaj efecte în restricționat fluxul
Deoarece curenții de maree au tendința de a fi mai puternic în canale înguste, o serie
de turbine în canal poate ocupa o fracție semnificativă de
canal secțiune transversală. Turbinele pot constitui astfel o importantă
blocaj la fluxul continuu, pentru o măsură mult mai mare decât apare
pentru parcurile eoliene. Acest restricționat fluxul poate duce la o turbina (sau matrice de
turbine) produce mai multă energie decât cele indicate de Betz analiza
§8.3, adică C
p

>0.6. În efect, fluxul este "împins" puternic din amonte


și doar o mică parte poate devia în jurul turbinei, astfel încât
restul este obligat prin turbina la o mai rapidă viteză decât ar fi
cazul pentru un izolat turbina, așa cum este ilustrat în Fig. 12.9. (Acesta este motivul
de ce hidroelectrice turbine, închise de conducte, sunt, de asemenea, nu sunt supuse
la Betz "la limită".)
Caseta 12.2 prezintă unele rezultate de la unul din multe laborator sau
simulări numerice pe care urmăresc să cuantifice aceste efecte. Nu este surprinzator,
efectul crește proporția de flux care este "blocat" de
turbine, deci acest "accesoriu" este probabil să fie mai semnificativă în inguste
canale (
<
~100 m
) decât în estuare largi, cum ar fi Golful Fundy.
TWIDELL PAGINARE.indb 463
01/12/2014 11:38
464
Maree-curent și maree-gama de putere
CASETA 12.2 BLOCAJ EFECTE PE TURBINA DE IEȘIRE ÎN CANALE ÎNGUSTE
Fig. 12.9 prezinta un exemplu de un sistem de blocaj care efectele au fost simulate numeric.
Spulberate de cerc (de rază R) reprezintă un cross-flow turbine cu axa perpendiculară pe diagrama.
Blocajul este măsurată prin 'blocaj raport' b, raportul dintre suprafața prezentat fluxul de turbină
cu aria secțiunii transversale de curgere. În Fig. 12.9(a), canalul limitele sunt departe de turbină,
astfel încât, pentru debitul de aproape de turbina, condițiile sunt aproape echivalente cu fluxul liber asumat în
Betz analiză; calculat C
p

= 0.52, chiar sub limita Betz. Cu toate acestea, în Fig. 12.9(b), canalul


limitele sunt mult mai aproape de turbina, astfel încât turbina (care prezintă o zonă 2R la intrare
flux) are un blocaj raport b
= 50%. Notă cât de ieșire flux de linii pentru b = 0.50 sunt mult mai aproape împreună
decât în (a), respectiv de ieșire fluxul este mai rapid, cu calculat C
p
= 1.25, aproximativ dublu Betz "limita" pentru a deschide fluxul.
A se vedea, de asemenea, alte lucrări de cercetare pe acest subiect enumerate în bibliografie pentru acest capitol (rețineți că
Kim et al.
(2012) examinează o conducte turbina în acest context).
+2R
–2R
+2R
–2R
–2R
+2R
0
b = 0.125
(a)
(b)
b = 0.50
+ 8R
– 8R
1.1 R
1.1 R
2.1 R
2.9 R
0
0
Fig. 12.9
Ilustrând efectul asupra înseamnă fluxul de blocaj de o turbină într-un canal (vazut de sus). Adică fluxul este de la stânga
la dreapta. Spulberate de cerc de rază R reprezintă un trei-lama 'Darrieus' turbina, cu palele turbinei ocupând
doar 12,5% din circumferința cercului. Turbina se rotește în sens invers acelor de ceasornic. Subțire linii solide sunt simplifică a
vrut să curgă. În (b), canalul limitele sunt de la y =
+2R și y = −2R, astfel încât turbina (care prezintă o lățime 2R) are o
blocaj raportul b = 50%. În (a), canalul limitele sunt de la y =
+8R și y = −8R, deci b = 12.5%.
Sursa: După Consul et al. (2013, Fig. 8.)
TWIDELL PAGINARE.indb 464
01/12/2014 11:38
§12.5 Maree-gama de putere
465
§12.5 MAREE-GAMA DE PUTERE
§12.5.1 teoria de Bază
Teoria de bază a mareelor-gama de putere, spre deosebire de valurile
ei, este destul de simplu. Ia în considerare apă prinse la maree înaltă într-un bazin, și
- a permis să fugă printr-o turbina la reflux (Fig. 12.10). Bazinul
are o constantă de suprafață O că rămâne acoperită cu apă la reflux.
Prins de apă, având o masă
rAR la un centru de gravitate R/2 de mai sus
reflux nivel, este presupus a alerga afară la reflux. Potențialul
maxim de energie disponibile pe valul dacă toată apa cade prin R/2 este
, prin urmare, (a neglija mici schimbări în densitatea apei de mare valoare,
de obicei
r = 1025 kg/m
3
):
r
(
)
= AR g R
energie pe valul
( / 2)
(12.25)
Dacă această energie este, în medie pe perioada mareelor
t, potențialul de mediu
putere pentru o singură maree perioadă devine:
rA
t
=
P
Rg
2
2
(12.26)
Gama variază de prin luna la un maxim de R
s
pentru primăvară
mareele, de la un minim de R
n
pentru pnapm maree. Plic de această variație
este sinusoidală, conform Fig. 12.5, cu o perioadă de jumătate de lună.
Suprafață O
Maree înaltă la nivel
Reflux nivel
Bariera cu turbina
Gama R
Fig. 12.10
Generarea de energie din mareelor.
DERIVAREA 12.1 ÎNSEAMNĂ A MAREELOR PUTERE
La orice timp t după o maree înaltă termen lunar din perioada T (
= 29.53 zile), intervalul este dat de:








p
=
+
+

R
R
R
R
R
tT
2
4
4
sin(4
/)
s
n
s
n
(12.27)
Dacă
o
=
R
R
n
s
(12.28)
atunci intervalul este dat de:
o
o
p
=
+
+−
R
R
tT
2
[(1
) (1
)sin(4
/ )]
s
(12.29)
TWIDELL PAGINARE.indb 465
01/12/2014 11:38
466
Maree-curent și maree-gama de putere
Deoarece
o ~0.5, (12.32) diferă puțin de cele două aproximări de multe ori
utilizate în literatura de specialitate (ca Exemplu Lucrat 12.2 arată), și anume:
(i)
r
2t

P
Ag
R
()
2
(12.33)
în cazul în care
R
este intervalul de toate mareele, și
(ii)
r
2t

+
P
Ag R
R
(
)
2
max
2
min
2
(12.34)
în cazul în care R
max
și R
min
sunt maxime și minime variază.
Puterea este obținută din media pătratului gama:


o
o
p
=
+
+−
=
=
R
R
tT
t
t
4
[(1
) (1
)sin(4
/ )] d
d
t
T
t
T
2
s
2
2
0
0
(12.30)
Prin urmare:
o
o
=
+
+
R
R
8
(3 2
3)
2
s
2
2
(12.31)
Puterea medie produsă în decurs de o lună de la (12.26) este:
r
2t
o
o
=
+
+
P
Ag R
8
(3 2
3)
luna
s
2
2
(12.32)
în cazul în care R
n
=
oR
s
și
t este intertidale perioadă.
EXEMPLUL 12.2 VALORI TIPICE DE MEDIE A MAREELOR PUTERE
Dacă R
s
= 5 m, R
n
= 2,5 m, o = 0.5,
R
= 3,7 m, R
max
= 5 m, R
min
= 2,5 m, O = 10 km
2
,r
= 1.0
3
× 103 kg/m
3
și
t = 12 h 25 min = 4.47 × 10
4
s,
apoi
(12.32) randamentele
P
= 16.6 MW
(12.33) randamentele
P
= 15.4 MW
(12.35)
și
(12.34) randamentele
P
= 16.1 MW
Z factor de Capacitate este definită în §1.5.4(b) ca (electric) energie
de fapt a generat peste o perioadă de timp, împărțit la (electric)
energie care ar fi fost generate la capacitate maximă de-a lungul
aceleiași perioade. Din cauza mareelor ritmuri sunt corect previzibil, capacitatea de
factori pot fi, de asemenea, a prezis cu exactitate dacă caracteristicile sistemului sunt
cunoscute și rămân constante. Maree-gama de putere de plante este considerat a
fi Z în intervalul de 20%, în general, la, probabil, 30% în cele mai bune
condiții, întrucât maree-curent de alimentare de plante se așteaptă ca Z de aproximativ 35% (Ernst
și Young 2010). (A se vedea Tabelul D. 4 din Anexa D pentru factorii de capacitate de toate
tehnologiile de energie regenerabilă.)
§12.5.2 Cerere
Potențialul maxim de putere de a mareelor sistem nu pot fi obținute
în practică, deși de înaltă eficiență sunt posibile. Complicațiile sunt
după cum urmează:
TWIDELL PAGINARE.indb 466
01/12/2014 11:38
§12.6 Lume a mareelor site-uri
467
1
Generarea de energie electrică nu poate fi menținută aproape de reflux condiții
și deci un potențial de energie nu este valorificat.
2
Turbinele trebuie să funcționeze la înălțimi mici cu debite mari, o condiție
că este mai puțin frecvente în hidroenergetice convenționale practică, dar similare cu
'run-of-the-river' hidroelectrice. Francezii au cea mai mare experiență de
astfel de turbine, au dezvoltat low-cap, mare-flow turbine bulb pentru
generația de la râuri și La Rance mareelor sistem. Turbinele
sunt mai puțin eficiente, la cele mai mici cap.
3
Energia electrică ar putea fi necesare la o apropiere rată constantă, și deci
nu există o constrângere de a genera la ori mai puțin decât valoarea maximă cap.
Eficiența poate fi îmbunătățită dacă turbinele funcționează ca pompe la mare
valul să crească cap. Ia în considerare un sistem în cazul în care intervalul este de 5 m.
Apă ridicat 1 m la maree înaltă pot fi lăsați afară pentru o generație la reflux
, când capul devine 6 m. Chiar dacă pompe și generatoare sunt 50%
eficient, nu va fi un câștig net de energie de ~200% (a se vedea Problema 12.6).
În Fig. 12.10, rețineți că puterea poate fi produs ca apa curge atât cu
cea de intrare ("flux") și de ieșire (ceb) maree. Astfel, o atenție optimizat
energia mareelor sistem care utilizează reversibile turbinele pentru a genera la ambele ebb
și flow, și în cazul în care turbina poate funcționa ca pompele să crească
cap, poate produce energie de 90% din potențialul dat de (12.32).
§12.6 LUME A MAREELOR SITE-URI
Cea mai mare experiență de departe de maree-gama de putere este de La Rance
240 MW capacitate stație în Bretania, Franța, care a funcționat ca
planificată din 1966. Tabelul 12.1 prezinta alte instalații de lucru semnificativ de
capacitate mai mică, și, de asemenea, recenta (2010) de plante de mare de 254 MW de capacitate
la Siwha în Coreea de Sud.
Total de disipare de energie de apă mareele pe Pământ este estimat
a fi de 3000 de GW, din care nu mai mult de aproximativ 1000 GW apare la mică adâncime
a mării zone accesibile pentru mari lucrări de inginerie civilă. Site-uri de
cel mai mare potențial de resurse în întreaga lume sunt indicate în Fig. 12.1;
au o tehnică combinată de potențial de aproximativ 120 GW, care este
de aproximativ 10% din totalul mondial hidroenergetice (râu) potențial. Acesta
este un semnificativ potențial de energie și de un potențial mare importanță pentru
anumite țări (de exemplu, în marea BRITANIE, unde, în principiu, aproximativ 25% din totalul anual
de energie electrică ar putea fi generate de energia mareelor de cunoscut estuare cu
îmbunătățită a mareelor).
Mai multe detalii despre unele dintre cele mai promițătoare site-uri sunt prezentate în
Tabelul 12.1. Foarte mare (GW) resurse în unele locații-a tentat
susținătorii să elaboreze propuneri pentru gigantice gama de stații de alimentare, nici unul
dintre care au fost de fapt construite, în mare parte datorită mari de capital
cost în comparație cu mici pe termen scurt câștiguri financiare și sociale și de
factorii de mediu discutate mai jos. De exemplu, estuarul Severn
TWIDELL PAGINARE.indb 467
01/12/2014 11:38
468
Maree-curent și maree-gama de putere
Tabelul 12.1
Majore ale lumii energia mareelor site-uri și posturi (umbrită rânduri indice curent de maree putere)
Locație
Adică
gama
Înseamnă potențial
de putere
Capacitatea instalată
și de tipul
o

(R) a mareelor
(C) curent de maree
în 2012
Consimțit
și proiecte
tip
b
în 2013
o

Data comandat/
observații
b
Canada
Golful Fundy
(Annapolis)
6.4 m
765 MW
20 MW (R)
1985
Golful Fundy
5.5 MW(C)
Golful Fundy
(Minas-Cobequid)
10.7 m 20.000 de MW
În planificarea
Coreea
Sihwa
5.6 m
254 MW (R)
2011
Incheon
1320 MW(R)
În așteptare?
alte
2000 MW (R)
Potențialul
Uldolmok
1,5 MW (C)
2009
Franța
La Rance
8.4 m
349 MW (R)
240 MW (R)
1966
Norvegia
Andritz Hydro
Hammerfest
1 MW (C)
2013 studii în
Orkney, Scoția
Regatul Unit
Strangford Lough,
Irlanda De Nord
3.6 m
1.2 MW (C)
2008
Atlantis AR-1000
3 MW (C)
2012 finalizat complet
scară de studii
de mare
Maree Generație Ltd/
Alstrom, Orkney,
Scoția
1 MW (C)
2012 500 kW proces,
1 MW din 2013
Alte (proiecte)
MW scară (C)
Numeroase (C)
Vezi cele mai recente
informatii la www.
renewableuk.com
Severn
9.8 m
1680 MW (R)
China
Jiangxia
7.1 m
3.2 MW (R)
1980
Numeroase mici
instalatii
DE 0,7 MW (R)
1961-1978
Curent de maree
0.1 MW (C)
3.7 (C)
în planificarea
Rusia
Kislaya
2.4 m
2 MW (R)
1966
TWIDELL PAGINARE.indb 468
01/12/2014 11:38
§12.7 aspectele Sociale și de mediu
469
în marea Britanie a realizat un nou studiu de fezabilitate aproximativ la fiecare deceniu
, începând din 1880! În schimb, aproape toate propunerile acum sub activ
de dezvoltare sunt pentru mai multe modulare de curent de maree putere, mai degrabă decât gama
de putere.
§12.7 ASPECTELE SOCIALE ȘI DE MEDIU
§12.7.1 Maree-gama de putere
Site-uri pentru a mareelor-gama de putere sunt alese pentru lor mare a mareelor; o
caracteristic, care este asociat cu estuare având suprafețe mari de
noroi apartamente expuse la mai mici maree. Maree-gama de putere depinde de
plasarea de o barieră pentru o diferență de înălțime în apă la nivel
turbinele. În funcțiune: (i) nivelul apei în bazin este întotdeauna mai sus
neperturbat reflux, și întotdeauna sub neperturbat de înaltă
maree; (ii) ratele de flux atât de intrare și de ieșire mareele sunt
reduse în bazin, și (iii) valurile mării sunt oprit la barieră.
Acești factori mecanici sunt funcții de conducere susceptibile de a provoca
următoarele efecte:
1
Zonele expuse apartamente noroi sunt reduse, astfel reducând semnificativ
de produse alimentare disponibile pentru păsări; de obicei, inclusiv a păsărilor migratoare în mod obișnuit
trece printr-o astfel de habitate speciale. Schimbarea în debitul, adâncimea și
valuri de mare poate fi de așteptat să se schimbe multe alte ecologic
caracteristici, dintre care multe pot fi unice pentru site-uri special.
2
Fluxul de râu poate fi controlat pentru a reduce inundații.
Locație
Adică
gama
Înseamnă potențial
de putere
Capacitatea instalată
și de tipul
o

(R) a mareelor
(C) curent de maree
în 2012
Consimțit
și proiecte
tip
b
în 2013
o

Data comandat/
observații
b
Tugurskaya (Okhotsk
Mare)
3640 MW
Potențialul
Mezenskaya (Alb
Mare)
~8000 MW
Potențialul
Australia
Kimberley
6.4 m
630 MW
40 (R)
propus 2012
Argentina
San Jose
5.9 m
5870 MW
Note
o
Tip: R = raza de putere (baraj), C = curent de putere, (umbrită rânduri).
b
Am f nu funcțiune data indicată numai atunci, au fost realizate studii la site-ul, dar nici o instalare.
Surse: AIE-OES (2012) , RenewableUK (2012), Wikipedia (2010), Twidell și Weir (2006), și diverse altele, inclusiv
clasic totalizarea voturilor de către Hubbert (1971).
TWIDELL PAGINARE.indb 469
01/12/2014 11:38
470
Maree-curent și maree-gama de putere
3
Acces pentru ambarcațiuni de porturi din bazinul este posibil dacă adecvate de blocare(s)
sunt incluse în baraj; într-adevăr, restricționat mareelor în
bazinul poate fi avantajoasă.
4
Controlate de adâncime și de curgere a bazinului permite pentru activități de agrement
, cum ar fi navigatie.
5
Impactul vizual este schimbat, dar cu o barieră singura necesar
de construcție.
6
Bariera poate fi folosit ca un viaduct pentru transport și de punere alte
construcții (de exemplu, turbine eoliene).
Maree barierele sunt mari și scumpe structuri care ar putea necesita ani pentru a
construi. Nici o putere nu pot fi produse, și, prin urmare, nici un venit generat,
până în ultima secțiune a barierei este completă. Dificultăți în finanțare poate
duce la lipsa de grija fata de mediu. Deși instalarea la la Rance
acum are o înfloritoare ecosistem natural, este vizibil diferită
de cea care a fost acolo înainte de baraj, și-a luat câțiva ani pentru a
restabili de la sine. Prin urmare, s-a observat că La Rance poate nu
au fost construite dacă ar fi trebuit să se confrunte astăzi a impactului asupra mediului
proceduri.
Un dezvoltator este principalul criteriu pentru succesul unei maree centrala este
costul per kWh de energie produsă. Ca și cu alte mari consumatoare de capital
tehnologii energetice, economice costul per kWh generat poate fi
redus (a) dacă alte avantaje pot fi costat ca beneficiu pentru proiect,
inclusiv fosili-carbon de reducere a emisiilor; (b) dacă ratele dobânzilor de bani
împrumutate pentru finanțarea costului ridicat de capital sunt mici, și (c) în cazul în care producția
de energie pot fi utilizate pentru a reduce consumul de scump combustibili cum ar fi
ulei. (A se vedea Capitolul 17 pentru o discuție generală a acestor probleme.) De
exemplu, numai la scară mare (>50 MW) mareelor-gama de putere de plante
comandat din 1970, Sihwa sistem în Coreea, a fost construit într-un baraj
construit mai devreme pentru atenuarea viiturilor și în scopuri agricole.
12.7.2 Maree-curent de alimentare
Aspecte sociale și de mediu sunt foarte diferite de maree-actuala
putere. Pentru mareelor-sistemele actuale, spre deosebire de maree-gama de sisteme, este
necesar pentru a bloca un întreg maree, astfel încât obstrucție la
trecerea de pește și bărci este mult mai puțin. Pentru același motiv,
construcția poate continua într-o manieră modulară, cu doar câteva turbine
fiind pus în locul inițial, și altele adăugate mai târziu. Ca cu energie eoliană
sisteme, acest lucru simplifică foarte mult costul de capital cerință, mai ales
la fel de utile putere pot fi generate și venitul obținut pas-cu-pas ca
porțiuni din costul de capital s-au consumat. Modularitatea permite, de asemenea,
dezvoltarea tehnologiei rapid prin "învățare prin practică".
O preocupare deosebită în ceea ce privește instalarea precoce la Strangford Lough
(Fig. 12.8) a fost impactul potențial asupra pești, foci, păsări de mare și bărci.
TWIDELL PAGINARE.indb 470
01/12/2014 11:38
§12.7 aspectele Sociale și de mediu
471
REZUMAT CAPITOL
Schimbarea în înălțime între succesive mareele înalte și joase (intervalul) variază de la linii de coastă între
aproximativ 0,5 m, în general, și de aproximativ 10 m, în special site-urile favorabile (de exemplu, anumite râu estuare). De
circulație a apei produce curenții de maree, care poate fi valorificat într-un mod similar cu vânt
de putere. În practică, curenții de maree sunt susceptibile de a fi atractiv pentru generarea de energie numai în cazul în care
acestea sunt
în mod natural puternic (>~3 m/s), din cauza mare a mareelor, și/sau consolidate în viteza de circulație a apei în
strâmtori înguste între insule și continent sau între insule.
La maree înaltă într-un estuar bazinul poate fi prins în spatele unui baraj sau barieră pentru a produce maree-gama de putere,
folosind low-cap 'hidroelectrice' turbine. 240 MW
e

'La Rance' sistemul în Franța a operat în mod fiabil


din 1967, dovedind astfel fezabilitatea tehnică a acestei tehnologii la scară largă. Din păcate, pentru a fi
eficient pentru acest scop, barajul are să se extindă aproape sau complet peste tot estuar mareelor.
Acest lucru presupune nu numai foarte mare inginerie civilă costurile, dar este probabil pentru a bloca, de asemenea, de transport
maritim, și pentru a produce
mari efecte asupra mediului, în special zonelor umede maree. Numeroase studii de fezabilitate în ultimii 100 de
ani sugerează că substanțială gama de putere este, în principiu, disponibile la cele câteva site-uri cu mare mareelor
(e.g. estuarul Severn din marea Britanie și Golful Fundy, în SUA/Canada border); acești factori au
însemnat că foarte puțini tidal gama de sisteme au fost implementate pentru generarea de energie în sine pe orice
scară semnificativă. Este demn de remarcat faptul că 254 MW maree-gama de putere de plante la Siwha, Coreea de Sud, utilizate
de
un pre-existente de captare a apei, barajul de la "insert" sale turbine; subliniind astfel importanța
multifuncțional instalații.
Cel mai maree-curent de alimentare de dispozitive sunt similare, în principiu, să orizontală-axa turbine eoliene care
funcționează în
extinsă de curgere a fluidului. Un aspect important este faptul că acestea nu bloca întregul maree, având astfel în mod
semnificativ
mai puțin impact decât tidal gama de plante. De asemenea, mari "sisteme" poate fi construit într-o manieră modulară, astfel încât
produce
puterea utilă treptat. În consecință, economice, sociale și de mediu ale
tidalcurrent putere sunt în multe privințe mult mai favorabile decât cele pentru tidal gama de sisteme. În consecință, o
varietate de prototip maree-sistemele actuale sunt explorate viguros, cu sprijin financiar din
partea guvernelor și capital de risc.
Gama, debit și periodice comportament de maree în cele mai multe regiuni de coastă sunt bine documentate și
analizate, din cauza cerințelor de navigare și oceanografie. Variabilitatea rezultă din neconcordanța
dintre principalele lunar-condus de maree perioade de 12 ore 25 de minute și 24 de ore 50 de minute cu om
(solar) perioada de 24 de ore, astfel încât optim energia mareelor generație nu este în fază cu cererea. Această
variație handicapuri utilizarea energia mareelor, dar predictibilitatea, să
+/- 4%, permite pre-planificate de integrare
în mare rețea electrică rețele, poate, de asemenea, cu un spatiu de depozitare generos. Astfel, energia mareelor, mai ales
maree-gama de putere care pot fi combinate cu alte active de capital, prezintă o foarte siguri sursă
semnificativă de energie durabilă, care este, probabil, avantaj major în comparație cu alte tipuri de energie
surse. Cele mai importante dezavantaje pentru toate formele de energia mareelor sunt: (a) numai câteva site-uri sunt potrivite, și
acestea
pot fi îndepărtate de pe cererea de energie; și (b) variabilitatea de generare a energiei electrice.
Prin urmare subacvatice camere video înregistrate mișcarea de pește
și alte animale trecut lame rotative și a arătat că nici un rău nu
a fost cauzat. Nici alte efecte adverse de importanță au fost
înregistrate, publicul acceptă impact vizual și, ca cu cele mai multe
proiecte de energie regenerabilă, aparatul se adaugă la atractie turistica – toate din care este
încurajatoare pentru proiecte de viitor.
TWIDELL PAGINARE.indb 471
01/12/2014 11:38
472
Maree-curent și maree-gama de putere
ÎNTREBĂRI RAPIDE
Notă: Răspunsurile la aceste întrebări sunt în textul de la secțiunea relevantă
din prezentul capitol, sau poate fi ușor dedusă din aceasta.
1
Descrie cum au loc mareele ca daca esti explica-le la o
10 ani a copilului.
2
De ce mareele nu se propagă ca tsunami?
3
Explicați diferența între primăvară și de cvadratură maree.
4
Mid-ocean mareelor este de aproximativ 0,35 m, deci, de ce este mareelor poate
de 10 m în unele locații?
5
Care sunt diferențele de bază între maree-gama de putere de plante și
de maree-curent de alimentare de plante?
6
Mareele sunt foarte previzibile, deci, de ce sunt factorii de capacitate de maree
centrala nu 100%?
7
Dacă ambele maree-curent și maree-gama de putere de plante sunt conectate într-o
utilitate rețeaua de energie electrică, este comună de putere mai mult sau mai puțin variabilă? De ce?
8
Explica de ce anumite locații poate da putere sporită de ieșire dintr -
o maree de curent dispozitiv de putere.
9
Cum de operare tidal gama de turbine uneori ca pompe permite
enhanced generarea de energie electrică?
10
Lista pozitive și negative asupra mediului al mareelor-gama de putere
de posturi.
PROBLEME
12.1
Calcula la prima comanda fluxul lunar-crearea de forțe F
X
la X și
F
y

la Y pe o masă mică de apă de mare m în Fig. 12.2. Rețineți că


mările noastre se rotească în jurul punctului O, nu despre centrul Pământului
la E. procedura este de a calcula pentru fiecare poziție X și Y,
forța rezultată din: (i) forța centrifugă pe m despre O la
frecvența lunar
w, și (ii) forța de atracție între m și
Luna, de masă M. Sugestie: să nu uităm că din (12.4) [GM/D
2
]
= L'w
2
] și
asta deoarece r<<D,
Dr
D
r
D
Dr
D
r
D
1
(
)
1
1
2
și
1
(
)
1
1
2
2
2
2
2
+
=
+




+
=





12.2
(a) În Fig. 12.2, suntem în căutarea de-a lungul axei de rotație a
Luna despre Pământ și având în vedere lunar de rotație de
frecvență
w. Ia în considerare lunar legate de forța F
Z

pe o masă
m de mid-ocean mare de apă de-a lungul raza Pământului EZ. Deoarece
D >>r, arată că F
Z
= dw
2
.
(b) valul-creșterea forței F
t

este diferența în
lunar-rotationrelated vigoare la această masă între refluxului (pozitia Z) și
maree înaltă (pozițiile X și Y) în mijlocul oceanului. Arată că:
TWIDELL PAGINARE.indb 472
01/12/2014 11:38
Probleme
473
F
t
= F
X
− F
Z
= 2MmGr/D
3
(c) de Masă m este în echilibru între valul de sensibilizare vigoare și

diferența de Pământ de atracție gravitațională pe m la

joasă și înaltă maree. Prin urmare, arată că a mareelor R în

midocean este dat de


=
=
R
Dl.
MD
'
0.36 m
4
3
12.3
Arată că lungimea zi siderală este dat de:
=
+
t
t
tT
*
1(/)
S
S
S
= 86164s = 23h 56min 4s
Indiciu: luați în considerare Fig. 12.4.
12.4
Sideral luna T* este definită după (12.7). Synodic lună
T
M
este definit ca perioada medie dintre doua Luni noi ca
văzută de un observator de pe Pământ. T
M

este mai mare decât T* din cauza


mișcării Pământului și a Lunii, împreună cu Soarele, care
în mod eficient cu întârziere la aspectul de Lună nouă. Care este
relația dintre T* și T
M
?
12.5
Un ocean tipic de pe suprafața Pământului are o adâncime de 4400 m.
(a) Arată că viteza de un mod natural de înmulțire val în
oceanul este de aproximativ 200 m/s (750 km/h).
(b) Compara acest lucru cu viteza aparentă de viteză lunar de maree-
creșterea forței ca Pamantul se roteste.
(c) Cât timp ar lua pentru astfel de un val de a călători pe o distanță de
egală cu circumferința Pământului la Ecuator?
(d) Dacă o astfel de val este inițiat de influența Lunii,
poate propunerea să fie întărită continuu ca Pamantul se roteste,
(i), în principiu, și (ii) în practică?
12.6
Apa este pompată rapid de la ocean la maree înaltă pentru a oferi o
creștere a nivelului apei într-o mare putere în bazinul de 1,0 m. Dacă tidal
gama este de 5,0 m și dacă pompa/generator de sistem este de numai 50%
eficiente, arată că surplusul de energie acumulată poate fi aproape de două ori
energia necesară pentru pompare.
12.7
În Fig. 12.9, distanța dintre cele simplifică este invers
proporțională cu viteza de curgere. (Aceasta este de obicei convenția pentru
astfel de diagrame.) Dacă în amonte viteza fluxului este u
0
= 3 m/s, cal-
culate viteza de curgere u
2
în aval de turbina. Compara
raportul u
2
/u
0

pentru că, pentru idealizat (Betz) sistemul de §8.3, și


comentarii cu privire la diferența pentru fiecare dintre cazurile (a) b = 0.125,
(b) b = 0.50.
TWIDELL PAGINARE.indb 473
01/12/2014 11:38
474
Maree-curent și maree-gama de putere
NOTE
1
O discuție mai detaliată a cauzelor de maree este dat în Capitolul 13 din cea de-a doua ediție a acestei cărți,
care este disponibil prin intermediul editorilor site-ul (www.routledge.com/books/details/9780415584388).
2
Sursa: http://energiesdelamer.blogspot.com/2011/01/enermar-un-projet-hydrolien-italien.html (raport din
ianuarie 13, 2011).
BIBLIOGRAFIE
Bahaj, A. S. (2013) Marin curent de conversie a energiei: zorii unei noi ere în producția de energie electrică', Filosofice
Tranzacțiile Societății Regale O, vol. 371 (parte de o ediție specială pe acest subiect). Se concentreze pe UK evoluții.
Charlier, r. C. (2003) Durabilă co-generare de maree: o revizuire', Energia Regenerabilă și Durabilă
Recenzii, 7, 187-213. Revizuire cuprinzătoare gama de putere, inclusiv de muncă înainte de anul 1980 și până în 2002;
contraste vise și realitate.
Clare, R. (ed.) (1992) energia Mareelor: Tendințe și evoluții, Thomas Telford, Londra. Lucrări de conferințe,
cea mai mare parte studii de site-uri potențial și instalații în marea BRITANIE, dar, de asemenea, inclusiv M. Rodier, 'Rance tidal
power
station: la un sfert de secol în funcțiune'. Indică faptul că acolo nu a fost mult progresele înregistrate între 1970 și
1990.
Stetler, J. (2009) O Analiză de Flux de Maree Putere, Wiley-Blackwell, Oxford. Aceasta consideră că: (i) lichid dinamice
teoria mareelor, lichide și putere turbine susținute cu informații istorice, cu aplicație pentru dispozitive recente;
(ii) sfaturi practice și analize economice pentru proiecte operaționale și pentru amplasarea în întreaga lume; și (iii) să sprijine
material pe site-ul său include un model pentru estimarea potențialului de putere la un site.
Hubbert, M. K. (1971) Scientific American, Septembrie, 60-87. Clasic estimări globale mareelor potențialul de putere.
Lewis, A., Estefen, S., Huckerby, J., Musial, W., Pontes, T. și Torres Martinez, J. (2011) 'Ocean de energie'. În
O. Edenhofer, R. Pichs Madruga, Y. Sokona, K. Seyboth, P. Matschoss, S. Kadner, T. Zwickel, P. Eickemeier, G.
Hansen, S. Schlömer și C. von Stechow (eds), IPCC Raport Special privind Sursele Regenerabile de Energie Climatice și
de Atenuare a schimbărilor climatice, Cambridge University Press, Cambridge. Recenzii actual și potențial viitor
de toate formele de ocean de energie, inclusiv de maree.
Referințe specifice
Consul, C. A., Willden, R. H. și McIntosh, S. C. (2013), Blocarea efectelor pe hidrodinamice de performanță de
marin cross-flow turbine', Filosofice Tranzacțiile Societății Regale O, vol. 371 (parte de o ediție specială pe
acest subiect).
Ernst și Young LLP (2010) Costul și Susținere a Valurilor, a Mareelor-stream și Tidal gama Generație în marea BRITANIE,
un Raport (marea BRITANIE guvernul) Departamentul pentru Energie și schimbările Climatice, și guvernul Scoțian.
Kim, K-P., Ahmed, M. R. și Lee, Y-H. (2012) 'de îmbunătățire a Eficienței de un curent de maree utilizand o turbina mai mare
zona de canal', Energie Regenerabilă, 48, 557-564.
Sørensen, B. (2011, 4 edn) surse Regenerabile de Energie, Academic Press, London. Include un util, dar scurt rezumat
al mareelor potențialul de putere.
Twidell, J. și Weir, A. (2006, 2nd edn) Resurse Energetice Regenerabile; ch. 13, 'energia Mareelor este pe site
pentru curent (a treia) a ediție.
TWIDELL PAGINARE.indb 474
01/12/2014 11:38
Bibliografie
475
Vennell, R. (2012) 'Realizarea potențialului de curenții de maree și eficiența turbinei ferme într-un canal',
Regenerabile De Energie, 47, 95-102.
Wikipedia (2010) Lista de maree centrale' (actualizat noiembrie 2010).
Site-uri
AIE Ocean Sisteme de Energie. Colaborarea internațională cu util a rapoartelor de progres și a politicilor; vezi esp.
lor de "Rapoarte Anuale" (www.ocean-energy-systems.org).
http://energiesdelamer.blogspot.com/2011/01/enermar-un-projet - hydrolien-italien.html. Un newsletter pe marine
de energie, în principal în limba franceză.
http://social.tidaltoday.com. Un newsletter special pe energia mareelor.
leeds-faculty.colorado.edu/lawrence/.../Lectures/Oceanic%20Energy.ppt. Prelegeri pe ocean de energie.
TWIDELL PAGINARE.indb 475
01/12/2014 11:38

Ocean gradient de energie: OTEC și osmotic


putere
CONȚINUTUL
Obiective de studiu
476
§13.1 introducere Generală
478
§13.2 conversia energiei termice
(OTEC): introducere
478
§13.3 OTEC principii
479
§13.4 considerente Practice despre OTEC 483
§13.4.1 schimbătoare de Căldură
483
§13.4.2 depunerile organice și coroziune
483
§13.4.3 cerințele de Pompare
484
§13.4.4 Teren pe bază de plante și
platforme plutitoare
484
§13.4.5 Construcția de frig
conducta de apă
484
§13.4.6 conexiunile electrice
485
§13.4.7 turbina generator
485
§13.4.8 Rezumat al avantajelor și
dezavantajele OTEC
485
§13.5 OTEC Dispozitive
486
§13.6 tehnologii Conexe
487
§13.7 Sociale, economice și de mediu
aspecte ale OTEC
488
§13.8 Osmotic putere de salinitate
degradeuri
489
Rezumat capitol
491
Întrebări rapide
492
Probleme
492
Notă
493
Bibliografie
493
Caseta 13.1 ciclu Rankine motor
482
OBIECTIVE DE STUDIU

Înțelege principiile de două
diferite ocean de energie tehnologii de extracție.
În primul rând, conversia energiei termice
(OTEC), depinde de temperatura
gradient de sub suprafața oceanelor tropicale. În
al doilea rând, osmotic putere, depinde de
gradienți de concentrație de sare de mare și
apă dulce.

Înțelege principiile de bază și
limitele de fiecare dintre aceste tehnologii.

Revizui progresele înregistrate de aplicații.
CAPITOLUL

13
TWIDELL PAGINARE.indb 476
01/12/2014 11:38

www.shahrsazionline.com
Lista de tabele
477
LISTA DE FIGURI
13.1 schema de principiu a unui OTEC sistem.
480
13.2 mediu Sezonier de diferența de temperatură
DT între suprafața mării și o adâncime de 1000 m.
481
13.3 profilul de Temperatură cu adâncimea de tipic mările tropicale.
482
13.4 Presiune-volum graficul de ciclul Rankine.
482
13.5 Coajă și tub schimbător de căldură.
483
13.6 Unele sisteme (dispozitive) destinate să demonstreze OTEC.
487
13.7 Ilustrarea presiunii osmotice.
490
13.8 diagrama Schematică a unui osmotic sistem de putere, folosind presiune retardat osmoza.
491
LISTA DE TABELE
13.1 Rezumat al OTEC Demonstrație Plante (bazat pe Ravindran (1999), Nihous (2008) și R&D
rapoarte de Universitatea din Delft (Olanda), Institutul Național de Ocean Technology (India), a
Laboratorului de Energie Naturala din Hawaii Autoritate (SUA), etc.)
486
TWIDELL PAGINARE.indb 477
01/12/2014 11:38
478
Ocean gradient de energie:
§13.1 INTRODUCERE GENERALĂ
În acest capitol vom considera două semnificativ diferite tehnologii,
de care nici până în prezent au progresat dincolo de cercetare și
dezvoltare a activității comerciale profitabile în aplicare ca aprovizionarea cu energie.
Cu toate acestea, ar trebui să o astfel de R&D a fi de succes, atunci potențialul este
răspândită relativ instalații de mari dimensiuni. Link-ul comun este variația
în ocean, unul dintre temperatura de suprafață și altul de salinitate. Ambele
depind de bine stabilit de știința, dar ambele au considerabilă
de inginerie provocări pentru a depăși înainte de a deveni stabilite industrii.
Cele mai multe din acest capitol (§13.2 §13.6) se ocupă cu oceanul de energie termică
de conversie (OTEC) ca acest lucru a fost pe larg studiate; §13.7 prezintă
principiile de putere osmotic.
§13.2 CONVERSIA ENERGIEI TERMICE
(OTEC): INTRODUCERE
Oceanul este cel mai mare colector solar. În mările tropicale,
diferențele de temperatură de aproximativ 20°C apar intre cald, aproape de suprafața
apei și rece "profunde" apa de la 500 la 1000 m adâncime. Motoare termice
pot funcționa între sursele termice și chiuvete cu astfel relativ
mică diferență de temperatură, dar lor intrinsecă eficiența este mică din cauza
legilor și practice ale termodinamicii. Ocean de energie termică
de conversie (OTEC) este extragerea și conversia acest termică
energie în lucru util pentru generarea de energie electrică. Dat scară suficient
de eficiente echipamente, producerea energiei electrice ar putea fi susținut
de zi și de noapte, la ~200 kW
e
/km
2
în zonele de mare tropicală. O astfel de putere
este egal cu aproximativ 0,07% din solară absorbită de iradiere de intrare pentru acea zonă.
Primele OTEC demonstrație planta a fost în 1930. R&D efort fost
finanțate din Franța pre-1970 și apoi din statele UNITE ale americii, Japonia și
Taiwan în anii 1980 și continuând foarte activitate moderată de
atunci; a se vedea §13.5. Avery și Wu (1994) oferă un istoric detaliat, actualizat
prin Nihous (2008, 2013). Demonstrația plante descrise în §13.5
confirmat că pentru a obține costul pe unitatea de putere competitive
cu celelalte surse de energie regenerabile necesită mari dimensiuni (>
~100MW)
și îmbunătățirea eficienței energetice. Rezultă că, finanțată privat,
dezvoltarea si comercializarea sunt puțin probabil fără a continua guvernamentale
de finanțare. De asemenea, este clar că economia s-ar fi îmbunătățit dacă beneficii
în plus față de generarea de energie electrică sunt incluse (de exemplu, de desalinizare a apei,
construirea de răcire, plus nutrienti pentru piscicultură), după cum este indicat în §13.6.
Atractivitatea OTEC de succes planta este eficient
nelimitate de energie dintre cele mai fierbinți de apă de suprafață în raport cu rece
apă adâncă și potențialul său pentru constanta, cerere scăzută de energie electrică;
adică planta are potențialul de capacitate mare factor se apropie de 100%.
Cu toate acestea, OTEC se confruntă cu trei limitări fundamentale:
TWIDELL PAGINARE.indb 478
01/12/2014 11:38
§13.3 Otec principii
479
1
De pompare. Motoare termice depind de energia care trece jos un
gradient de temperatură de la cald sursă pentru o mai reci chiuveta (de exemplu în abur de la
~150°C la temperatura mediului ambiant-25°C). Pentru OTEC sursa fierbinte este
tropical de apă de suprafață de la ~25°C și rece chiuveta este apa din
ocean adânc de la ~5°C. Această salină rece watert trebuie să fie pompat până la
suprafața de nivel pentru a deveni un mai rece termică chiuveta pentru motor termic, pentru
care considerabilă putere de pompare este necesară. În practică, de pompare
este la o rată de aproximativ 6 m
3
/s de apă pe MW
e

de energie electrică produsă,


care poate necesita până la 50% din puterea generata. Astfel de sisteme
necesită mari pompe, conducte cu diametru mare și mare schimbătoare de căldură,
toate din care sunt scumpe.
2
Eficiență mică. În practică, diferența de temperatură disponibile pentru a
opera motor termic este mic (<20°C) și deci eficiența chiar și
un "perfect" motorul este mic la > ~ 5%.
3
Locație la distanță. Site-uri cu OTEC potențiale sunt fie la tropicale
de coastă sau marine, folosind mari instalații plutitoare. Aceste site-uri sunt
, de obicei, departe de locuințele având capacitatea de a utiliza OTEC de ieșire.
Pentru a aborda limitări tehnice, OTEC designerii folosesc metode de
stabilit industrii de recuperare a energiei (de exemplu, fluxuri mari de încălzit
de descărcare de gestiune din metal rafinării, centrale electrice și industriei alimentare). În
plus, OTEC poate combina cu alte aplicații folosind apă adâncă fi
explicate în §13.6; termenul general pentru o astfel de dezvoltare este adânc ocean
de apă aplicații (DOWA). Este probabil că doar în comun OTEC/DOWA
sisteme sunt mai probabil să fie de succes comercial.
§13.3 OTEC PRINCIPII
Fig. 13.1 descrie un sistem pentru OTEC; și cu un motor termic funcționează un
închis-ciclul Rankine proces (a se vedea, de asemenea, Caseta 13.1). Fluidul de lucru (de exemplu,
amoniac) fierbe în evaporator' la ~25°C ~30°C temperatura
de la suprafața apei, așa că a conduce o turbină generator pentru alimentarea cu energie electrică.
Pe partea de ieșire din turbină, aburul se condensează la un lichid la
~5°C temperatura de pompat apă adâncă.
1

Alternative open-ciclu
sistemele de apă de mare ca intrare fluid de lucru, care se evaporă la
presiune redusă înainte de a trece printr-o turbina. Condensul este
'apă distilată, care poate fi folosit atat potabilă și apă pentru irigații.
Esențiale termodinamice, principiile și limitele de ciclu deschis
și închis ciclu sunt la fel.
Într-un sistem idealizat perfect cu schimbătoare de căldură, debitul Q de
apa calda trece în sistem, la temperatura T
h
și pleacă la T
c

(temperatura apei reci din adâncimi mai mici). Puterea dat de


apă caldă într-un sistem ideal este:
ρD
P
cQ T
=
0
(13.1)
TWIDELL PAGINARE.indb 479
01/12/2014 11:38
480
Ocean gradient de energie:
în cazul în care
DT = T
h
- T
c
(13.2)
Cea de-a doua lege a termodinamicii dictează că maximă de ieșire
de putere P
1
pot fi asigurate de puterea de intrare P
0
este:
η
P
P
=
1
Carnot
0
(13.3)
în cazul în care
η
DT T
=
/
h
Carnot
(13.4)
este eficiența unui ideal Carnot motor de operare de la un infinit de lent
rata între T
h
și T
c
= T
h

DT. Deși Carnot teoria neglijează
dependența de timp și practice ale schimbătoarelor de căldură, pe scară largă este
luat ca un criteriu pentru a judeca eficiența (a se vedea Caseta 16.1). Pentru OTEC
având
DT doar ~20°C (= 20K), chiar ideal randamentul ciclului Carnot este foarte
mic: ~7%. În practică, temperatura scade de ~5°C apar pe fiecare
schimbător de căldură și o parte din puterea de ieșire este folosit pentru pompare, deci
net eficiența unui sistem real este în mod substanțial mai puțin la aproximativ 2 până la 3%.
Cu toate acestea, analiza de bază ilustrează atât promisiunea și
limitări de OTEC.
Din (13.1) – (13.4) ideal brut mecanic putere de ieșire este:
ρ
=
D
P
cQ T
T
(
/)(
)
h
1
2
(13.5)
Astfel, creșterea
DT de 1°C (~5%) crește P cu aproximativ 10%. Partea
teoretică dependența de producția brută de energie pe piața de
diferența de temperatură este un rezultat important aplică, de asemenea, să practice
Evaporator
Fluidul de lucru
Suprafața oceanului ~27°C
Ocean adâncimi ~5°C
Turbina
Condensator
Generator
Apă rece
de admisie
Cald
apă
de admisie
Fig. 13.1
Diagrama schematică a unui OTEC sistem. Motorul termic funcționează între cald
de apă de la suprafața oceanului și apă rece din adâncurile oceanului, la aproximativ
500 m la 1000 m sub suprafața.
TWIDELL PAGINARE.indb 480
01/12/2014 11:38
§13.3 Otec principii
481
EXEMPLUL 13.1 DEBITULUI NECESAR
Pentru
DT = 20°C debitul necesar pentru randament 1.0 MW de la un motor termic ideal și ideal de schimbător de căldură este,
din (13.5):
=
×
=
=
-
-
-
-
Q
(10 J s )(300K)
(10 kg m )(4.2 10 J kg K )(20 K)
0.18 m /s
650 t/h
1
6
1
3
3
3
1
1
2
3
N
40°
20°
20°
22°
24°
22°
22°
20°
18°
16°
40°
20°
20°
40°
S
Eq.
N
20°
24°
20°
18°
16°
22°
20°
40°
16°
18°
20°
22°
22°
16°
18°
20°
22°
22°
20° 18°
16°
20°
20°
18°
16°
S
Eq.
Fig. 13.2
Mediu sezonier de diferența de temperatură
DT între suprafața mării și o adâncime de
1000 m. Zonele cu
DT ≥ 20°C sunt cele mai potrivite pentru OTEC. Aceste zone de toate se află în
tropice.
Sursa: Departamentul de Energie al SUA.
motoare termice, inclusiv a celor care utilizează ciclul Rankine descrise în
Caseta 13.1.
Exemplul 13.1 arată că un flux substanțial de adânci și reci
de apă este necesară pentru a da un rezultat semnificativ. Un astfel de sistem necesită
mari, și, prin urmare, scumpe, mașini.
Din P
1
este proporțională cu (
DT)
2
în practică, numai site-uri cu
D≥
T 20 ° C
o

de-a lungul anului ar putea fi, eventual, economice. Fig. 13.2 arată că astfel de


site-uri sunt la tropice, și Fig. 13.3 indică faptul că apa rece are, de obicei,
să fie pompată de la adâncimi >
~100m pentru maxim disponibil tempera-
adevărat diferența.
TWIDELL PAGINARE.indb 481
01/12/2014 11:38
482
Ocean gradient de energie:
CASETA 13.1 CICLU RANKINE MOTOR
Toate motoare termice se ia în energie la o temperatură mai mare și respinge pierdere de căldură la o temperatură mai mică.
Ciclul Rankine se aseamănă cu ciclul Carnot, dar foloseste o presiune constantă (isobaric) modificări de stat
în loc de temperatură constantă (izoterme) modificări (a se vedea Fig. 13.4 și manuale de inginerie
termodinamicii). Prin urmare, ciclul Rankine se aseamănă cu operațiunea de real motoare mult
mai realist decât ciclul Carnot, care este în mare parte "folosit" ca un vitale teoretice dispozitiv în pură
termodinamicii. Fluidul de marea majoritate a ciclul Rankine motoare cu abur, așa cum este folosit
în cărbuni și centralele nucleare. Cu alte fluide decât aburul, ciclul este adesea numit
Organic Rankine cycle (ORC), deși principiile sunt aceleași. Astfel de motoare sunt utilizate pentru generarea
de energie din deseuri industriale de căldură, energie geotermală (§14.4) și energia solară concentrată (punctul 4.8). Cu
mici diferențe de temperatură și apropiat-condiții ambientale pentru OTEC duce la amoniac fiind comune
fluid de lucru.
Presiune
Volumul
Lichid + vapori
Munca
de exemplu turbina
Lichid
numai
Vapori
numai
Constantă
de presiune
evaporator
Constantă
de presiune
condensator
Fierbere
Căldură
în
Căldură
în
Căldura
Condensare
Fig. 13.4
Presiune-volum graficul de ciclul Rankine.
0
0
5
10
15
Thermocline
20
25
30
500
1000
1500
2000
Adâncime (m)
Temperatura (°C)
Fig. 13.3
Profilul de temperatură cu adâncimea de tipic mările tropicale. Anii 'thermocline' este regiunea
unde schimbările de temperatură mai rapid cu adâncimea.
TWIDELL PAGINARE.indb 482
01/12/2014 11:38
§13.4 considerente Practice despre otec
483
§13.4 CONSIDERENTE PRACTICE DESPRE OTEC
Nu sunt fundamentale de termodinamică motive prevenirea OTEC
sisteme de lucru cu succes, dar sunt anumite tehnice
provocări, cele mai importante dintre care sunt prezentate mai jos. (Pentru mai multe detalii și
un calculele relevante, vezi online materiale suplimentare pentru
acest capitol.)
§13.4.1 schimbătoare de Căldură
Fig. 13.5 arată conturul design de coajă și tub schimbător de căldură
potrivit pentru circuit închis OTEC, dar, pentru 1 MW putere termică la mici
diferențe de temperatură, acest lucru ar necesita mai multe mii de interne
tuburi cu o suprafață totală de >>2000 m
2

. Astfel OTEC de căldură


schimbătoare trebuie să fie relativ mare pentru a oferi o suprafață suficient de mare pentru transferul de căldură
la temperatură scăzută diferență, și, prin urmare, sunt scumpe (probabil
50% din costurile totale). Mai mult decât atât, calculul ideal putere de ieșire
P
1

în asumă perfect de transfer de căldură între exterior ocean de apă


și interne a fluidului de lucru; acest lucru este nerealist, mai ales din cauza
depunerile organice în afara și în interiorul țevilor. Prin urmare, dezvoltarea de OTEC
include îmbunătățiri existente în schimbătoare de căldură pentru a reduce
rezistența termică între apa și fluid de lucru. Scopul este
mai eficientă, și, prin urmare, mai mici, schimbătoare de căldură, care cu
mai puțin de metal poate da o reducere semnificativă a costurilor.
§13.4.2 depunerile organice și coroziune
Interior deosebit de conducte deveni incrustate de marin
organisme, care crește rezistența termică, reducând astfel eficiența. Astfel
depunerile organice este una dintre problemele majore în OTEC design, deoarece creșterea
Fluidul de lucru (fierbinte)
Fluidul de lucru (frig)
Apa
(rece)
Apa
(fierbinte)
Fig. 13.5
Shell-and-tube heat exchanger (cut-away vedere).
TWIDELL PAGINARE.indb 483
01/12/2014 11:38
484
Ocean gradient de energie:
suprafața disponibilă pentru transferul de căldură crește, de asemenea, posibilitatea
de organisme de a se atașa. Printre metodele încercat să păstreze
acest depunerile de sub control sunt mecanice de curățare de circulație continuă
de prim-montaj bile și de curățare chimică de aditivi pentru apa. În
plus, serioase de coroziune pot apărea cu structuri metalice, inclusiv
interioară, transfer de căldură tuburi de schimbătoare de căldură.
§13.4.3 cerințele de Pompare
Lucru este necesar pentru a muta cantități mari de apă caldă și rece în jurul valorii de
sistemul împotriva frecării; acest lucru este alimentat de la putere brută
de ieșire, astfel încât reducerea ideal putere de ieșire P
1

de (13.5). Deși rece
conducta de apa pot fi construite suficient de mare pentru a evita semnificative de frecare (pentru că
șeful pierderi variază în funcție de diametrul D
-5

: a se vedea R2.6 și Problema 6.7), frecare


pierdere poate deveni semnificativ în legături și în conducte mai mici
în interiorul schimbătoarelor de căldură. Depunerile organice în schimbatorul de caldura
creste frecarea cu rugozitate și scade diametrul tubului,
ceea ce face situația mai rău. Prin urmare, peste 50% dintre pompare
de putere pot fi pierdute din cauza frecare fluidă.
§13.4.4 Teren pe bază de plante și platforme plutitoare
Teren pe bază de sisteme sunt posibile numai la anumite favorabile tropicale
locații, în cazul în care fundul mării pante brusc în jos. Principalul lor
avantaj este costul redus, deoarece link-urile de la mal, de asamblare și de întreținere
sunt mult simplificate. Conducta de apă rece nu este, în mod nejustificat, a subliniat, deoarece
se bazează pe fundul mării; cu toate acestea, este încă vulnerabil la daune furtună
de undă a mișcării la o adâncime de aproximativ 20 m.
Foarte mare concepute în acest scop plutitoare, platforme offshore pentru OTEC
ar putea genera energie electrică la ~500 MW
e

. O astfel de putere ar


fi adus la teren prin cabluri sau ar putea fi folosite la bord (de exemplu, pentru
producerea de hidrogen drept combustibil, §15.9.1).
§13.4.5 Construcția de conducta de apă rece
Suspendat conducta de apă rece este supusă mai multor forțe, inclusiv
cele datorate drag de curenți, vortex vărsare, mișcare de geamanduri și
platforme, și mort greutatea conductei în sine. În plus, există
dificultăți substanțiale implicate în asamblarea și amplasarea conductei. Unii
ingineri favoarea aduce un prefabricate țeavă și încet se scufundă-l
în loc; cu toate acestea, de a transporta un obiect de mai multe de metri în diametru
și, probabil, pe un kilometru este dificil. Defectarea prematură a rece
conducta de apa (de ex. din daunelor provocate de furtuni) a cauzat eșecul mai multor
proiecte demonstrative (vezi Tabelul 13.1).
TWIDELL PAGINARE.indb 484
01/12/2014 11:38
§13.4 considerente Practice despre otec
485
§13.4.6 conexiunile electrice
De înaltă tensiune, mare putere cabluri submarine sunt componente standard
de energie electrică sisteme de transmisie. Cablurile de la 500 km în lungime
sunt practicabile, cu pierderea de putere de aproximativ 0.05% pe km pentru AC și de 0,01%
pe km pentru DC. Acolo este acum o experiență considerabilă pentru subacvatic
conexiuni în power-grid și rețele de energie eoliană offshore. Mare
OTEC plante situate departe de cererea de energie ar putea, în principiu, folosesc
energie electrică pentru a produce un magazin chimice de energie (de exemplu, H
2
: §15.6).
§13.4.7 turbina generator
Turbina trebuie să funcționeze între mici diferențe de temperatură la
apropierea de temperatura mediului ambiant. Prin urmare, fluidul de lucru trebuie să introduceți
turbina, ca încălzite cu gaz sau s-a evaporat de vapori și apoi să fie răcit sau
condensat la ieșire. Caseta 13.1 descrie ciclul Rankine al potrivite
turbine și arată layout pentru un sistem închis fluid de lucru. Pentru
a OTEC condiții, există mai multe comune fluide cu un
caz punctul de fierbere (de exemplu, amoniac, freon și propan). Din păcate,
multe astfel de fluide nu sunt acceptabile pentru siguranță sau din motive de mediu;
amoniac, prin urmare, este o alegere comună.
Prin aplicarea unui vid parțial (de exemplu, reducerea presiunii), cu punct de fierbere
punctul de apă poate fi redusă la temperatura apei calde
de admisie, astfel încât să permită apei să fie fluid de lucru. Aceasta este baza de
deschide ciclul de sistem, în care apa de mare caldă în sine este folosită ca
fluid de lucru. Un astfel de sistem oferă nu numai puterea, dar, de asemenea, importante
cantități de apă distilată de la turbina de ieșire.
§13.4.8 Rezumat al avantajelor și dezavantajelor
de OTEC
Avantaje: (i) constantă de ieșire; (ii) utilizări convenționale de inginerie
hardware de turbine, conducte și schimbătoare de căldură; (iii) limitat doar de dimensiunea
a echipamentelor; (iv) liniște; (v) aparent mic impact asupra mediului; (vi)
pot fi asociate cu asociate ocean adânc de apă aplicații (DOWAs).
Dezavantaje: (i) extrem de mic eficiență termodinamică;
(ii) prin urmare, instalații de mari dimensiuni necesare pentru semnificativă putere de ieșire; (iii)
prin urmare, scumpe; (iv) suprafață sau aproape de suprafață echipamente expuse la
cicloane și furtuna valuri; (v) depunerile organice în conducte limitează fluxurile,
crește presiunea de pompare și reduce eficiența schimbătorului de căldură,
scăzând astfel eficiența globală; (vi) depunerile organice pot fi depășite
cu rafale de chimice, erbicide, dar nedorit de mediu
impact; (vii) submarin și mai mare țevile sunt dificil de a izola la mare
scară, prin urmare nedorite de căldură de intrare și pierderea de putere potențial; (viii)
cooperarea internațională este limitată la doar câteva țări interesate.
TWIDELL PAGINARE.indb 485
01/12/2014 11:38
486
Ocean gradient de energie:
§13.5 DISPOZITIVE
Fundamentale limitări ale §13.2 și considerațiile practice de
§13.4-au combinat pentru a limita OTEC sisteme de la data la câteva
demonstrații de unități, construit pentru R&D în scopuri mai degrabă decât pentru operațiuni comerciale,
cum este indicat în Tabelul 13.1 și Fig.13.6. Rețineți că foarte puține dintre aceste OTEC
sisteme au produs netă pozitivă de ieșire pentru mai mult de șase luni;
Pierderea de conducta de apă rece (uneori chiar înainte de orice operațiune) a fost
cea mai comună tehnică cauza eșecului.
Tabelul 13.1
Rezumat al OTEC Demonstrație Plante (bazat pe Ravindran (1999), Nihous (2008) și
R&D de rapoarte de Universitatea din Delft (Olanda), Institutul Național de Ocean Technology (India), a
Laboratorului de Energie Naturala din Hawaii Autoritate (SUA), etc.)
An
Locație
Tip
Ciclul
Agenția
(țara)
Putere:
brut
Putere: net
de pompare
Note (CWP
= apă rece
țeavă)
1930 Matanza
Bay,
Cuba
plutitoare
deschide
Claude (a)/
Franța
22 kW –
Principiul de dovedit, dar
CWP-a rupt în termen de
săptămâni
1935 Pe Brazilia
coasta
plutitoare
deschide
Claude/
Franța
nil
Scalate-up versiune a
1930 sistem; CWP
probleme.
1979 Hawaii,
Statele UNITE ale americii
plutitoare
închis
Rankine
NELHA/
Lockheed
Mini
OTEC
(statele UNITE ale americii)
53 kW 18 kW
1980 Hawaii,
Statele UNITE ale americii
plutitoare
închis
Rankine
Lockheed
OTEC 1
(STATELE UNITE ALE AMERICII)
1 MW –
Vezi Fig. 13.6(b)
1982 Nauru,
South
Pacific
mal
închis
Rankine
Toshiba-
TEPEC
(Japonia)
120 kW 32 kW
Vezi foto si detalii
în on-line suplimentar de
material. CWP rupt
în termen de șase luni.
1993 Hawaii,
Statele UNITE ale americii
mal
deschide
NELHA
(Statele UNITE ale americii)
50 kW –
1993 Hawaii,
Statele UNITE ale americii
mal
deschide
NELHA
(Statele UNITE ale americii)
210 kW 60 kW
Cinci ani de funcționare
(Fig.13.6(a))
1996 Hawaii,
Statele UNITE ale americii
plutitoare
închis
Rankine
NELHA
(Statele UNITE ale americii)
50 kW –
2000 60 km
Tuticorin
plutitoare
închis
Rankine
NIOT (India)
1 MW –
Instalat pe barja. CWP
probleme -> nici o ieșire
2013 de Sud
China
plutitoare
închis
Rankine
(amoniac)
Lockheed
(Statele UNITE ale americii și
China)
10 MW –
Construcții propuse
2013 private resort (b)
Note
o
Claude a fost un milionar francez, care a făcut o avere din alt proces pentru a produce aer lichid.
b
Vezi http://spectrum.ieee.org/green-tech/geothermal-and-tidal/lockheed-martin-pioneers-ocean-energy-in-china (iulie 2013).
TWIDELL PAGINARE.indb 486
01/12/2014 11:38
13.6 tehnologii Conexe
487
§13.6 TEHNOLOGII CONEXE
OTEC este unul dintre mai multe posibile ocean adânc de apă aplicații (DOWAs)
asociate cu de pompare apă de mare de la adâncimi de cel puțin 100 m. Alții
sunt enumerate mai jos. Ca OTEC în sine, toate au dimensional scalare
factori încurajarea echipamente mari, spre deosebire de funcționarea modulare și
la scară mai mică, de multe opțiunile de energie regenerabilă. Cu toate acestea, dacă OTEC,
sau tehnologii similare, sunt vreodată să fie acceptată din punct de vedere comercial, se
pare inevitabil ca un set integrat de operațiuni va fi folosit pentru o
combinație de mai multe beneficii.
1
Marin agricultura. Apa de mare de la adâncimi de aproximativ 500 m, este
bogat în substanțe nutritive, și acestea pot fi pompate la suprafață, ca de la
un OTEC plante. Acest lucru încurajează creșterea de alge (fitoplancton),
care se hrănesc alte creaturi marine mai sus în lanțul trofic și deci
de a oferi o bază pentru pește comerciale agricole.
2
Răcire. Adânc, rece apă pompată la suprafață poate fi folosit pentru a răci
clădirile, tropicale horticole 'sere' sau de plante de inginerie ca
într-chimice, rafinării.
3
Apă proaspătă/desalinizare. Flash evaporarea superioare-suprafață mare
de apă pe condensatoare răcite cu apă adâncă produce 'distilat'
"proaspete" apă pentru băut, horticultură, etc. Acest proces poate fi
integrat cu solar de distilare. Pentru OTEC un open-ciclul Rankine motor
intrări de vapori de apă și ieșiri-o ceață de parțial condensat apă; această
ieșire este în vigoare apă distilată și pot fi utilizate pentru apa potabilă și
apă pentru agricultură.
(a)
Amestecat
apa
scurgere
Cald
apa
aportul
Geamandura
Apa rece
țeavă
Fundul mării
(1000 m)
Ancora
picior
(b)
Fig. 13.6
Unele dintre sisteme (dispozitive) destinate să demonstreze OTEC: (a) de pe malul sistem la
Hawaii (big island), în 1990: conducta de apă rece din oceanul este în partea de sus a fotografiei; (b)
barjă sistem (schematică, bazat pe NIOT c. 2003).
TWIDELL PAGINARE.indb 487
01/12/2014 11:38

488
Ocean gradient de energie:
4
CO
2
injecție. Scopul este de a absorbi CO
2
emise de
pe scară largă
fosile
de combustibil de ardere prin absorbție în apă de mare și apoi de pompare această
adâncime. Acesta este un tip de captare și stocare a carbonului (CCS). Este
aproape invers de tehnologie pentru OTEC conducta de apă rece,
și, potențial, ar fi pe o scară foarte mare. Dacă este combinată cu OTEC,
vaporizator de ieșire ar absorbi CO
2

și apoi să fie pompat


în jos pentru descărcarea de gestiune de la adâncime. Cu toate acestea, există diferențe semnificative
probleme nerezolvate cu astfel de sugestii, inclusiv de mediu
impactul asupra biotei la adâncime, costul și stabilitatea pe termen lung a
captura.
5
Plutitoare, complexe industriale. Concepte există pentru a se potrivi pe scară largă
de OTEC și DOWA cu industria, pe foarte mari, plutitoare, plute de km
scară (de exemplu, pentru producția de hidrogen pentru transport terestru piețe
ca de stocare a energiei). Talk este ieftin!
§13.7 SOCIALE, ECONOMICE ȘI DE MEDIU
ASPECTE ALE OTEC
Așa cum este ilustrat în Fig.13.2, centrul de resurse pentru OTEC este efectiv limitată la
zonele de coastă sau insule de la tropice. Cu toate acestea, cele mai multe astfel de locuri sunt în cele mai
sărace
țări care dispun de fonduri pentru a suporta riscul și sarcina asociată
cu capital roman-tehnologii intensive, cu câteva excepții notabile
, cum ar fi Hawaii, Florida și Brazilia. În astfel de locuri, sociale, impactul unei
OTEC plantă ar putea fi similare cu operare, o platformă de petrol offshore sau onshore
stație de putere (de exemplu, furnizarea de locuri de muncă și în apropiere de industrie, inclusiv
marine activități de servicii).
La economie de OTEC sunt dominate de mare de capital estimate
costurile care decurg din dimensiunea mare a OTEC componentele și cerințele
impuse de offshore medii, pe echipamente de supraviețuire și puterea
de producție logistică. Acest lucru, precum și relativ mici iesiri de putere,
rezultat în analizele bazate pe costul nivelat al energiei electrice generație
(§17.6) și găsească în mod constant OTEC proiecte au prea mic cost/
beneficiu să fie valoros din punct de vedere economic. Chiar dacă
rentabilitatea decalaj între OTEC și cel mai scump combustibil fosil
putere generație de tehnologii (de exemplu.g., ulei) a scăzut în mod constant, începând cu
anul 2000, OTEC de penetrare pe piață nu a reușit încă. Din cauza
lipsei de experimentale și date operaționale în stare de funcționare OTEC sistemele
cu toate acestea, profitând de această pretinsă economia de scară prezintă
un mare financiar și inginerie de risc, cu capital cheltuieli la fel de mare ca
NE-300 de milioane de dolari pentru puteri de ordinul a 10 MW. Prin urmare,
rămâne probabil că o demonstrație de scalabil OTEC
sisteme va fi realizat numai cu un angajament puternic publice
fonduri.
Principalele impacturi asupra mediului de OTEC și DOWA tehnologii
se referă la următoarele:
TWIDELL PAGINARE.indb 488
01/12/2014 11:38
§13.8 Osmotic putere de salinitate degradeuri
489

scurgere, și probabil de poluare, de inginerie de plante, mai ales din
fluidele de lucru și antivegetative chimice;

volume mari de pompat apă marină;

amestecul de adânc, bogat în substanțe nutritive (nitrați, fosfați și silicați) cu apă
, cu partea superioară, solar iradiate, apă;

operațiune de inginerie de plante, de obicei, în marin curat locații.
O dominantă dăunătoare amenințare este locală pe uscat, lângă-shore și off-shore
poluarea cauzată de scurgeri de fluide de lucru. Amestecul de bogat în substanțe nutritive adânc
de apă cu apă de suprafață are impact ecologic, care poate fi
benefic pentru pescuit, dar nu altfel. Termice de amestec de apă nu este
considerat nociv de dezvoltare sau izolate OTEC de plante;
chiar și ipotetic locație de aproximativ 1000 de posturi de 200 MW
e

fiecare
din Golful Mexic a fost calculat pentru a reduce suprafața mare
de temperatură de doar 0,3°C, care nu este considerată semnificativă. Mare desfășurare
de OTEC plante, să zicem, 100 de stații, la 10 km de separare, ar provoca
mișcări de nitrat la o concentrare a găsit în mod natural de pe Peru, unde
populațiile de pești sunt mult mai crescut. Perspectiva de îmbogățire a pescuitului
cu apă adâncă de substanțe nutritive în general, este favorizat.
Dacă adânci și reci de apă este deversată în ocean suprafață, o parte
de altfel sale stabile dizolvat CO
2
trece în atmosferă. Dacă
50% din excesul de CO
2
este emisă, rata ar fi de aproximativ 0,1 kg/kW
e
,
comparativ cu aproximativ 0,8 kg/kW
e

de la generarea de energie electrică de fosile de


combustibil. Acest lucru nu impact nesemnificativ duce la discuție dacă OTEC este
într-adevăr o manieră durabilă sursă de putere (în sensul
discutat în Capitolul 1).
§13.8 OSMOTIC PUTERE DE SALINITATE DEGRADEURI
Putere Osmotic este extragerea de energie utilă din diferența de
concentrație de sare între ocean și în apropiere o sursă de apă proaspătă
(de exemplu, un râu). Tehnica utilizează presiune osmotică , care este evident
atunci când două volume de solvent (de exemplu, apă) având diferite
concentrații de substanță dizolvată (de exemplu, sare) sunt separate printr-o membrană semi-permeabilă,
așa cum se arată în Fig. 13.7. La microscop moleculele de solvent sunt
capabile să difuzeze și înapoi prin membrana, dar moleculele
de solut nu poate face acest lucru. În consecință, mai concentrată solu-
tion devine mai puțin concentrată pentru mai multe solventul trece-un fel
decât celelalte. Acest lucru determină o macroscopice diferența de presiune pe
membrană. În cele din urmă a atins starea de echilibru, pentru care statice
diferența de presiune de-a lungul membranei este numit osmotică
presiunea. Presiunea osmotică este foarte mare (de exemplu, 30 de atmosfere între
apă dulce și apă de mare). (Pentru detalii suplimentare a se vedea manualele de fizică,
chimie.) Presiunea osmotică diferențe și mișcarea de solvenți
este un proces esențial în viața sisteme (de exemplu, funcția renală și apă
de circulație prin semi-permeabile peretii celulelor).
TWIDELL PAGINARE.indb 489
01/12/2014 11:38
490
Ocean gradient de energie:
Pentru osmoză inversă, externă, forțele de presiune a apei din sărat/
salmastră partea proaspete parte de o membrană semi-permeabilă împotriva
presiunii osmotice. Tehnica este utilizată pentru desalinizarea de sărată
apă și apă de mare, de exemplu, pe o scară largă în unele părți din Orientul
Mijlociu, de obicei, alimentat de combustibili fosili, dar pe o scară mică în zonele rurale,
de obicei, alimentat de solare fotovoltaice sau eoliene.
Tehnici de a valorifica osmotic procese pentru util de energie sunt
considerate regenerabile, astfel cum sunt definite în §1.1, deoarece acestea depind numai de
hidrologic natural ciclu, dar numai relativ recent a fost
considerat comerciale de aprovizionare cu energie. Primul sistem pilot a fost construită în
2009 de către Statkraft, o hidro-generatoare de energie electrică de utilitate Norvegiei; acest lucru
a condus la inițierea construcției de 2 MW putere osmotic planta în
Sunndalsora fjord în 2013.
(a) Presiune retardat osmoza (PRO) pentru putere
Fig. 13.8 arată o diagramă a fluxului de PRO power sistem pilotat de
Statkraft (Norvegia). Apă proaspătă dintr-un hidro hidrografice sau în apropiere,
râul este alimentat în instalație și se filtrează înainte de a intra în membrana
module. Fiecare membrana modul conține spirala rana sau fibre tubulare
membranele, peste care 80 la 90% din apa dulce transferuri de
osmoza. Acest presurizează mare conductelor de apa (verde închis) și creșterea
debitului volumetric la înaltă presiune. La 'presiune schimbător' este în
vigoare o pompa pentru afluxul de apă de mare.
De apă sărată din membrana modul împarte în două fluxuri.
Aproximativ o treime din apa asta merge la turbina pentru a genera putere, și
două treimi trece la presiune schimbător de a face presiuni asupra hrana din
apa de mare. Pentru a optimiza puterea plantelor, tipic presiune de funcționare este
în intervalul de la 11 la 15 atmosfere. Acest lucru este echivalent cu un cap de apă
de la 100 la 145 de metri într-o hidrocentrală, generând aproximativ 1 MW
pentru fiecare m
3
s
-1
de apă proaspătă (a se vedea §6.2).
Unele pre-tratament atât pentru apă dulce și apă de mare este
necesar. Experiența norvegiană de epurare a apelor arată că
filtrare mecanică până la 50 µm în combinație cu un standard de
curățenie și întreținere ciclu este suficient pentru a sustine membrana
performanta de la 7 la 10 ani.
Ca cu OTEC, conceptul de salinitate gradient de putere este simplu, dovedite
la nivel de instalație pilot și are un potențial de resurse ori de câte ori o apă dulce,
râul curge în ocean. Prin urmare sale globale de resurse de nivel este foarte mare.
Dificultatea actuală este că valoarea de putere netă rămasă
după pompare este mic în raport cu mari costuri de capital, în special al
membranelor. Cu toate acestea, sistemele de membrane sunt îmbunătățite și
de a deveni mai ieftin ca R&D răspunde la cererea de desalinizare prin
osmoza inversa.
Soluție
Semipermeabila
membrana
Pur solvent
h

Fig. 13.7
Ilustrarea presiunii osmotice:
echilibru osmotic peste o
membrană semi-permeabilă.
Presiunea osmotică din
'solvent pur' parte (la dreapta) de
membrana este echilibrată de
greutatea ('gravity presiune')
de suplimentare înălțimea h a lichidului pe
solutului bogate în lateral (în partea stângă).
TWIDELL PAGINARE.indb 490
01/12/2014 11:38
§13.8 Osmotic putere de salinitate degradeuri
491
(b) Alte posibile mecanisme de putere
Mecanismele sunt cercetate pentru obținerea puterii de salinitate
degradeuri, altele decât cele descrise mai sus. Exemple sunt după cum urmează:
1
Folosind nitrura de bor nanotuburi în loc de standard de membrane.
2
Inversă, electrodializă, prin care osmotic energie de amestecare proaspete și
apă sărată este capturat de către conducerea soluție printr-o alternativ
serie de pozitiv și negativ perceput membrane de schimb. În
rezultat chimice diferența de potențial creează o tensiune pe fiecare
membrană și duce la producerea directă a energiei electrice.
Apă sărată
Apa de mare
Apă proaspătă
W
ater filtru
W
ater filtru
Membrana module
Putere
Turbina
Apă sărată
Apă proaspătă sângera
Presiune
schimbător de
Fig. 13.8
Diagrama schematică a unui osmotic sistem de putere, folosind presiune retardat osmoza (după
Aalberg (2003)). Intrare apă de mare (lumina verde) este diluat cu apă proaspătă de trecere
de membrană semi-permeabilă pentru a deveni 'apă sărată' (verde inchis). Negru
linia punctată indică membrana.
REZUMAT CAPITOL
Conversia energiei termice (OTEC) se referă la conversia în energie electrică de unele de
mare energie termică diferența dintre suprafața caldă apele oceanului tropical și rece
de apă la adâncimi ~1000 m. Din păcate, randamentul unui motor termic pentru aceasta este neapărat mici
(~3%) pentru această diferență de temperatură este de numai ~20°C. prin Urmare, pentru a obține o putere de ieșire semnificativă
necesită volume foarte mari de apă de mare rece pentru a fi pompat, care necesită (i) mari și scumpe foc
schimbătoare și țevi, și (ii) mari pompe alimentate de la generator cu turbină de care reduce semnificativ
net exportate puterea de a o grilă. Diverse practice de inginerie dificultăți, cauzate de daunele provocate de furtuni,
coroziunea și depunerile organice, avea la data de limitat OTEC la câteva relativ mic pilot de plante; cu toate acestea,
mai mare multi-megawatt proiecte sunt acum luate în considerare. Cost-eficiență poate fi îmbunătățită prin comune de
cooperare cu alte ocean adânc de apă aplicații (DOWAs), cum ar fi clădiri de răcire și utilizare a
nutrienților în evacuate de apă pentru pescuit.
Putere Osmotic sisteme utilizează presiune osmotică între apă dulce și apă de mare, separate
printr-o membrană semi-permeabilă. În principiu, metoda promite o foarte mare net de resurse energetice,
dar din punct de vedere comercial sponsorizat de R&D este mai recentă, cu inițială mică-pilot la scară largă plante acum
avansarea la
TWIDELL PAGINARE.indb 491
01/12/2014 11:38
492
Ocean gradient de energie:
ÎNTREBĂRI RAPIDE
Notă: Răspunsurile la aceste întrebări sunt în textul de la secțiunea relevantă
din prezentul capitol, sau poate fi ușor dedusă din aceasta.
1
Ce face abrevieri OTEC și DOWA stand pentru?
2
De ce este OTEC nu este posibil în afara tropice?
3
Descrie OTEC termodinamic limita pe eficiență.
4
Cum OTEC potențialul de putere variază în funcție de diferența de temperatură
dintre suprafață și de adâncime?
5
Ce este depunerile organice și de ce este o provocare pentru OTEC sisteme?
6
De ce sunt mari, rezistente la coroziune schimbătoare de căldură necesară pentru
OTEC?
7
Ce sunt două motive principale de ce OTEC a fost încă să progreseze dincolo de
câteva instalații pilot?
8
Ce este presiunea osmotică?
9
Este osmotic de putere limitează la tropice și de ce?
10
De ce este OTEC favorizat offshore și osmotic putere favorizat
onshore?
11
De ce este important să se diferențieze net de putere "de la" brut de energie în
ambele OTEC și osmotic putere, dar nu atât de important pentru cele mai multe alte
tehnologii generatoare?
PROBLEME
13.1
Dacă P
∝ DT
2
/ T
h
(13.5) se calculează rata de schimbare a eficienței cu
privire la diferența de temperatură
DT. Care este procentul
îmbunătățirea în producția de energie electrică dacă
DT crește de la 20°C la
21°C?
13.2
Ia în considerare definiția termenului de "energie regenerabilă" în §1.1,
și a discuta despre modul în care atât OTEC și putere osmotic se potrivesc sau nu se potrivesc
aceste definiții.
Notă: în Continuare probleme la Capitolul 14 în Twidell și Weir (2006)
Resursele de Energie regenerabile, 2nd edn, sunt disponibile pe site-ul
asta de-a treia ediție. Acestea se referă la a extins analiza cantitativă a unor
aspecte inginerești ale OTEC.
MW aplicarea pe scară. Costurile pot scădea substanțial în viitor, ca membrana tehnologie îmbunătățește în
debitul și fiabilitatea, determinată în mare măsură de creșterea cererii pentru desalinizare prin osmoză inversă
folosind similare membrane.
TWIDELL PAGINARE.indb 492
01/12/2014 11:38
Bibliografie
493
NOTĂ
1
Apă adâncă curenții oceanici de apă rece să circule la nivel global, condus la scufundarea apă de mare rece de la Poli.
BIBLIOGRAFIE
OTEC
Avery, W. H. și Wu, C. (1994) surse Regenerabile de Energie din Ocean – Un ghid pentru OTEC, Oxford University Press,
Oxford (Universitatea Johns Hopkins serie). O substanțială și studiu de autoritate de știință, inginerie și
istoria OTEC.
Curzon, F. L. și Ahlborn, B. (1975) Eficiența de un Carnot motor la puterea maximă de ieșire', Jurnalul American
de Fizica, de 43 de ani, 22-24.
d ' Arsonval, J. (1881) Revue Scientifique, 17, 370-372. Poate cel mai vechi publicat de referință la potențialul
de OTEC.
Gauthier, M., Golman, L. și Lennard, D. (2000) 'Conversia Energiei Termice (OTEC) și Apă Adâncă
Aplicații (DOWA) – oportunități de piață pentru Industria Europeană', în cadrul Procedurilor de Conferința Europeană,
Tehnologii noi si Regenerabile pentru o Dezvoltare Durabilă, Madeira, iunie. Excelent comentariu de lucru
de plante din 1930 până în 2000, cu viitorul industriale potențial de piață.
Johnson, F. A. (1992) s-a încheiat ciclul de conversie a energiei termice', în R. J. Seymour (ed.), Ocean de Recuperare a
Energiei:
starea de arta, Societatea Americană a Inginerilor civili. Rezumat util al termodinamicii, economie și
istorie.
Masutani, S. M. și Takahashi, P. K. (1999) 'Conversia Energiei Termice', în J. G. Webster (ed.),
Enciclopedie de Inginerie Electrică și Electronică, 18, 93-103, Wiley, New York. Autoritate rezumat.
McGowan, J. G. (1976) 'conversia energiei termice – o semnificativă solare resurse', a Energiei Solare, 18, 81-92.
Revizuite NE filosofie de design la un moment important din punct de vedere istoric.
Meyer, M., Cooper, D. și Varley, R. (2011) 'suntem încă acolo? Un producător de parcurs pentru a OTEC
comercializarea', Hawaii Naționale de Energie din surse Regenerabile Marine Center OTEC Referințe, Lockheed Martin
Misiunea Sisteme
si Senzori, Manassas, VIRGINIA, statele UNITE ale americii.
Nihous, G. (2008) Conversia Energiei Termice (OTEC) și Tehnologii Derivate: Starea și perspectivele,
raport disponibil la http://www.ocean-energy-systems.org/ocean_energy/ocean_ thermal_energy/.
Nihous, G. (2013) 'Conversia Energiei Termice', în G. M. Crawley (ed.), Manualul de Energie, World
Scientific Publishing Co. Pte. Ltd., Singapore.
Ravidran, M. (1999) 'Indian de 1 MW plante plutitoare: o privire de ansamblu', în cadrul Procedurilor de IOA '99 Conferințe,
IMARI,
Japonia.
Twidell, J. W. și Weir, T. (2006) Resurse Energetice Regenerabile, 2nd edn, ch. 14, 'Ocean de Energie Termică
de Conversie' (schimbătoare de căldură, pp. 461-463), Earthscan, Londra. Acest capitol, care este disponibil pe site-ul web
pentru actuala ediție a acestei cărți, are mai multe detalii de schimbătoare de căldură și depunerile organice de mult în acest (a
treia) a ediție,
inclusiv unele calcule orientative.
ONU (1984) - UN Ghid pentru Conversia Energiei Termice pentru Țările în curs de Dezvoltare, organizația Națiunilor Unite
Publicații,
New York.
TWIDELL PAGINARE.indb 493
01/12/2014 11:38
494
Ocean gradient de energie:
Putere Osmotic
Acest subiect este atât de nou că cele mai multe din literatura de specialitate pe aceasta este doar în formă de revista stil de
articole tehnice și
rapoarte, dintre care cele mai multe apar doar pe internet. Se vedea, de exemplu:
http://www.statkraft.com/energy-sources/osmotic-power/ (Statkraft este o mare utilitate în Norvegia, și este activ
în curs de dezvoltare osmotic putere).
www.yuvaenegineers.com (un site compilate de către Indian studenți de inginerie; vezi mai ales un articol din 2010
pe osmotic putere de Rohini și Ahmed Bere).
Aaberg, R. J. (2003) 'Osmotic de putere: o nouă și puternică sursă de energie regenerabilă?', Se reorienteze, 4, 48-50.
Siria, A. et al. (2013) 'Gigant osmotic de conversie a energiei măsurate într-un singur transmembranar de nitrură de bor
nanotuburi', Natura, 494, 455. DOI: 10.1038/nature11876.
Site-uri
La buletine de știri și site-uri de pe ocean de energie, citată în Capitolele 11 și 12, raporta, de asemenea, pe OTEC, deși există
nu de multe povești pe OTEC, comparativ cu mai active domenii de val de putere și curenții de maree. A se vedea în special:
AIE Ocean Sisteme de Energie. Colaborarea internațională cu util a rapoartelor de progres și a politicilor – a se vedea
în special "Rapoarte Anuale" (www.ocean-energy-systems.org).
http://energiesdelamer.blogspot.com/. (Un buletin de energie marină , în principal în limba franceză).
TWIDELL PAGINARE.indb 494
01/12/2014 11:38

Energia geotermală
CONȚINUTUL
Obiective de studiu
495
§14.1 Introducere
497
§14.2 Geofizică
500
§14.3 rocă Uscată și fierbinte acvifer analiza
503
§14.3.1 Uscat rock: algebra pentru a calcula
potențialul de ieșire de căldură
503
§14.3.2 Fierbinte acvifere: algebra
calculați potențialul rata
de extracție la căldură
505
§14.4 Valorificarea resurselor geotermale
507
§14.4.1 Potrivire de aprovizionare și
cerere
507
§14.4.2 tehnici de Extracție:
hidrotermală
508
§14.4.3 tehnici de Extracție:
'geotermale avansate
sisteme' (EGS)
509
§14.4.4 generatoare de Electricitate
sisteme de
510
§14.4.5 Directă folosește de energia geotermală
căldură
510
§14.5 Sol-sursa de pompe de căldură
512
§14.6 aspectele Sociale și de mediu
514
Rezumat capitol
516
Întrebări rapide
517
Probleme
518
Bibliografie
519
OBIECTIVE DE STUDIU

Identifica sursa de energie geotermală și
aprecia probleme legate de sustenabilitatea acestuia.

Pentru a identifica cerințele pentru energia geotermală
de potențial util pentru energie electrică
generație și de a înțelege de ce locații adecvate
sunt restricționate geografic.

Apreciez potențialul punct de vedere geografic mai
larg răspândită utilizarea energiei geotermale pentru
aplicatii termice.

Înțelege principiile de operare a
groundsource pompe de căldură.
CAPITOLUL

14
LISTA DE FIGURI
14.1 Creștere în lume instalații geotermale.
4
97
14.2 Cheie numit regiuni valorificarea energiei geotermale pentru producerea de căldură și/sau energie electrică. 498
14.3 Geotermale și structura Pământului.
500
TWIDELL PAGINARE.indb 495
01/12/2014 11:38

www.shahrsazionline.com
496
Energia geotermală
14.4 Geologie de un acvifer într-un hidrotermale regiune.
502
14.5 (un) Profil de fierbinte acvifer sistem de calcul conținutul de căldură; (b) un gheizer, o priveliște comună în
multe hidrotermale regiuni.
504
14.6. (a) schema de principiu, nu este la scară, de hidrotermale centrale într-o hipertermie regiune
(de exemplu, Gheizere domeniul geotermal, California). (b) Geologie de un acvifer într-un hidrotermale
regiune și o regiune de uscat și cald, rock.
508
14.7 schema de căldură de extracție la cald, uscat rock sistem.
509
14.8 diagrame Schematice de două aplicații majore de căldură geotermale.
511
14.9 'Geotermale' pompe de căldură.
513
14.10 La Wairakei geotermale stație de putere în Noua Zeelandă.
516
LISTA DE TABELE
14.1 Țări cu un consum semnificativ de energie geotermală.
499
14.2 aplicații Directe de căldură geotermale, 2010.
512
TWIDELL PAGINARE.indb 496
01/12/2014 11:38
§14.1 Introducere
497
§14.1 INTRODUCERE
Miezul interior al Pamantului ajunge la o temperatură maximă de aproximativ
4000ºC, cu exterior debitul termic întreținute predominant de
natural radioactive degradare a anumitor elemente dispersate (de exemplu, uraniu,
toriu și a anumitor izotopi de potasiu). Căldura trece prin
solide de submarine și de suprafață de teren cea mai mare parte prin conducție – geotermale
de căldură și uneori de activ curenților de convecție de magmă topită
sau apă încălzită. Media de căldură geotermale fluxul de pe Pamant la
suprafata este doar 0,06 W/m
2

, cu medii de gradient de temperatura de la 25 la


30 ° C/km. Această căldură continuă actuală este trivial în comparație cu alte
surse regenerabile de provizii în cele de mai sus suprafața mediului, care în total, în
medie, aproximativ 500 W/m
2
(a se vedea Fig. 1.2).
Cu toate acestea, în anumite
locații crescut gradienti de temperatura apărea, indicând semnificativă
a resurselor geotermale. Regiunile de geotermală de potențial au, în general,
permeabile rock din zona
~10 km pătrați și o adâncime de ~5 km prin care
apa poate circula. Prin urmare, acestea pot fi valorificate la fluxurilor de
10 până la 20 W/m
2
pentru a produce
~100 MW (termică) pe km
2

în comerciale
provizii pentru cel puțin 20 de ani de funcționare. Regiunile de uscat și cald, rock'
au să fi fracturat în mod artificial pentru a deveni permeabile, astfel încât apa poate
fi circulat prin fracturi de a extrage căldura.
Există trei principale folosește de energia geotermală, astfel cum sunt enumerate mai jos în
ordinea descrescătoare a termodinamice de calitate, care se întâmplă, de asemenea, să fie
ordinea lor crescând disponibilitatea geografică.
1
Generarea de energie electrică. La câteva locații de caldura geotermale este disponibil
la temperaturi de peste 150ºC, ca un flux natural de
highpressure apă și/sau abur, având astfel potențialul de electrice
producția de energie electrică de la turbine. Mai multe geotermale de energie electrică
0
20
40
60
80
0
4
8
12
16
20
(a)
1970
1980
1990
2000
2010
2020
Capacitate / GW
e
An
Lumea geotermale de energie electrică
Pr
od
n
/
TW
/
h
/
y
0
10
20
30
40
1990
1995
2000
2005
2010
2015
An
De căldură geotermale instalații de capacitate / GW
th
teren
adânc
(b)
Capacitate / GW
th
Fig. 14.1
Creștere în lume instalații geotermale.
o
De căldură a energiei electrice; electrice capacitatea de producere (GW
e
) (axa din stânga) și anual de energie electrică (TWh) (axa din dreapta);
capacitate în 2015 este estimată la a anunțat planurile.
b
Căldură utilizați numai: capacitate instalată (GW
th
) desen pe căldură profundă' (curba continuă) și pe de caldura sol' (curba punctată).
Sursa: date de la WGC(2010).
TWIDELL PAGINARE.indb 497
01/12/2014 11:38
498
Energia geotermală
complexe au funcționat timp de mai mulți ani, în special în Italia, Islanda,
Noua Zeelandă și statele UNITE ale americii (vezi Fig. 14.2). Numărul de similare
instalații a crescut în mod constant începând cu anii 1970 (Fig 14.1(a)). Ca și pentru
hidroenergie, hidrotermală putere tehnologia este matură și de lungă durată
când adaptate la anumite site-uri. Puterea poate fi folosit în mod constant pentru
sarcina de bază, la un preț ieftin pe unitatea de cost. Noile evoluții au crescut
rapid în relativ neexploatat geotermală activă regiuni din
Filipine, Indonezia și SUA de vest (vezi Tabelul 14.1).
2
De alimentare cu apă caldă. În mult mai multe locații, de caldura geotermale este
disponibil la
~50 la 70°C; de exemplu, pentru 'medicinal' băi în
Imperiul Roman, și astăzi pentru încălzire cu efect de seră pentru culturi de legume
și fructe moi, pentru uscarea culturilor, pentru acvacultură de pește și alge, pentru
termoficare deservirea clădirilor și pentru proces industrial de căldură (de exemplu,
pentru fabricat pastă de lemn de prelucrare a lemnului, iar pentru scurgerea substanțelor chimice).
Mai mult de 60 de țări din lista de astfel de utilizări, dintre care multe nu produc
energie electrică geotermală (a se vedea Tabelul 14.1 și Fig. 14.1(b)).
3
Pompe de căldură. Căldură la temperatura mediului ambiant din apropierea suprafeței solului
(la adâncimi de obicei, aproximativ 3 m), sau din râuri și lacuri, este de intrare
pentru alimentare cu energie electrică a pompelor de căldură, care furnizează căldură la clădirile
la creșterea temperaturii. Sistemele sunt adesea numite
'geotermale, deși intrare de căldură rezultă din sol încălzită de soare și de
aerul înconjurător. Rețineți că, sol, la adâncimi de peste 2 m a
Gheizere
Matsukawa
Tiwi
Cerro
Prieto
Hawaii
Islanda
Paris
Larderello
Wairakei
Fig. 14.2
Cheie numit regiuni valorificarea energiei geotermale pentru producerea de căldură și/sau energie electrică
generație. Liniile punctate indică limitele placă. Linii colorate indică zone suplimentare de
tulpina.
TWIDELL PAGINARE.indb 498
01/12/2014 11:38
§14.1 Introducere
499
În Capitolul 1, energia din surse regenerabile a fost definit ca 'energia obținută de la
natural repetitive și persistente fluxurilor de energie care au loc în local
mediu'. Prin această definiție, mai livrărilor de energie geotermală
poate fi clasificat ca fiind din surse regenerabile, pentru că energia care altfel ar fi
disipată în mod continuu în mediul local (de exemplu, de la izvoare termale sau
gheizere). În alte geotermale de alimentare, curentul de căldură este crescută
artificial (de exemplu, prin fracturarea și în mod activ de răcire fierbinte roci, care rămân în
loc, dar nu reîncălzire cu excepția pe termen foarte lung, astfel de resurse
, în practică, are o viață finită). Astfel de sisteme geotermale avansate
(EGS) cu siguranta au potențialul de a furniza energie fără minerit și
extracția de materiale, atât de fierbinte pietre' tehnologie este cercetat și
dezvoltat ca un mijloc de alternative de energie (§14.4.3).
Tabelul 14.1
Țările cu un consum semnificativ de energie geotermală. Tabelul arată capacitatea instalată pentru
generarea de energie electrică (MW
e
), factor de capacitate Z pentru energie electrică geotermală, și capacitatea instalată direct
de utilizare a căldurii (exclusiv 'de suprafață' - sol și aer-surse de căldură pompe) (MW
th
). Toate datele sunt pentru 2010.
Tara
De energie electrică
capacitate
MW
e
% din lume
geotermale
total
De energie electrică
factor de capacitate Z
(%)
Încălzire directă
capacitate
(cu excepția
pompelor de căldură)
(MW
th
)
% din lume
total de direct
încălzire
Statele UNITE ale americii
3093
29
61
611
4
Filipine
1904
18
62
3
Indonezia
1197
11
92
2
Mexic
958
9
84
155
1
Italia
843
8
75
636
4
Noua Zeelandă
628
6
74
386
3
Islanda
575
5
91
1822
12
Japonia
536
5
65
2093
14
El Salvador
204
2
79
2
Kenya
167
2
98
16
Costa Rica
166
2
78
1
Nicaragua
88
1
40
0
Turcia
82
1
68
1548
10
Rusia
82
1
61
307
2
China
24
71
3690
24
altele
168
2
4075
27
---------
--------
---------
---------
LUME TOTAL
10715
100%
15347
100%
Sursa: Bertani (2010), Lund et al. (2010).
aproape temperatură constantă pe tot parcursul anului. În mod invers
extragerea de căldură de la clădiri, la fel pompe de căldură poate fi utilizată pentru răcire,
adică ele funcționează ca frigidere. Această tehnologie este disponibilă
în întreaga lume și este de departe cel mai rapid în creștere 'geotermale de aplicare (Fig.
14.1(b)). Tehnologia în cauză este prezentată în §14.5.
TWIDELL PAGINARE.indb 499
01/12/2014 11:38
500
Energia geotermală
§14.2 GEOFIZICĂ
Secțiuni prin Pământ sunt prezentate în Fig. 14.3. Transferul de căldură de la
semi-lichid manta menține o diferență de temperatură de peste relativ
crustă subțire de 1000ºC, și un gradient de temperatură de
~30 ° C/km.
Crusta solidă material are o densitate
~2700 kg/m
3
, căldura specifică
capacitate
~1000 J kg
-1
K
-1
și conductivitate termică
~2 W m
-1
K
-1
. Prin urmare,
la medie în sus fluxului geotermic este
~0.06 W/m
2
, cu căldura stocată
în crustă la nivel global la temperaturi mai mari decât temperatura suprafeței
fiind
~10
20
J/km
2
. Dacă doar 0,1% din această căldură a fost să fie 'extras' peste 30 de
ani, energie termică disponibilă va fi de 100 MW/km
2

. Astfel căldură
de extracție de roci ar putea fi completată în termen foarte lung, eoni după
artificiale extragerea caldurii s-a oprit. Aceste calcule dea ordinul de
mărime de cantitățile implicate și arată că surse geotermale
sunt un mare potențial de aprovizionare cu energie.
Căldura trece exterior din crusta de (1) de răcire naturală și frecare
de bază; (2) dezintegrare radioactivă de elemente; și (3) chimice
reacții. La constante de timp de astfel de procese de-a lungul întregului Pământ sunt
atât de mult timp că nu este posibil să se știe dacă temperatura Pământului
este în prezent în creștere sau în scădere. Elementele radioactive sunt
concentrate în crustă de fracționare recristalizare din material topit,
și sunt deosebit de pronunțate în granit. Cu toate acestea, producția de
căldură prin radioactivitate sau acțiune chimică este semnificativă numai peste multe
milioane de ani (a se vedea Problema 14.2). Prin urmare, de căldură geotermale
provizii de inginerie de extracție (distinct de izvoare termale) se bazează
pe eliminarea stocate căldură în capacitatea termică de material solid și
apă în crustă, mai degrabă decât pe reaprovizionare. Dacă conducție prin
uniformă de material au fost singurul mecanism de transfer de căldură geotermale,
gradientul de temperatură prin întreaga crustă ar fi constantă.
Crusta
STRATURI
Cel mai MIC
ADÂNCIME
Manta
~ 1000°C
Exterior
core
Miezul interior
~ 4000°C
6370 km
5180 m
2900 km
30 km
Fig. 14.3
Geotermale structura Pământului, arătând medie adâncimi mai mici de nume straturi. La
crusta are variații semnificative în compoziție și grosime pe o scară locală de mai mulți
kilometri.
TWIDELL PAGINARE.indb 500
01/12/2014 11:38
§14.2 Geofizică
501
Cu toate acestea, dacă convecție are loc 'local', ca de circulație a apei, sau, dacă
locală radioactive sau chimice exoterme surse de căldură apar, sunt
anormale gradienții de temperatură în Scoarța terestră.
Pe-o perspectivă globală, Scoarța Pământului este format din plăci mari
(Fig. 14.2). La placa de limite nu este activ de convecție termică
de contact cu Mantaua, evidențiată de activitate seismică, vulcani,
gheizere, gheizere și izvoare termale – așa-numitul 'cerc de foc'. De
energia geotermală de potențial dintre aceste regiuni este foarte mare, din cauza
a crescut anormal gradienții de temperatură (pentru a
~100 ° C/km) și active
cu eliberare de apă, ca abur sau lichid supraîncălzit, de multe ori considerabile
de presiune atunci cand este atinsa de foraj. Prin urmare, nu este nici o coincidență faptul că fiecare
dintre cele opt mari producători de energie electrică geotermală-au confruntat
la nivel local un cutremur major și/sau erupție vulcanică în ultimii 100 de ani
(de exemplu, "acum" în termeni geologici).
Creșteri Moderate în gradient de temperatură a
~50 ° C/km apar în
regiuni localizate departe de granițele placă, din cauza anomalii in crusta de
compoziție și structură. Căldura poate fi eliberat din aceste regiuni
în mod natural de adânc pătrunderea apei în acvifere și, ulterior, prin convecție
de curgere a apei. Rezultate izvoare termale, cu concentrații crescute de
substanțe chimice dizolvate, sunt de multe ori de celebru ca și băi. "Profunde" acvifere sunt
astăzi exploatat de foraj la adâncimi de
~5 m sau mai puțin, astfel încât furnizarea de surse
de căldură la temperaturi de la
~50 ~200ºC. Dacă anomalia este asociat
cu material de conductivitate termica mic (de exemplu uscat rock), apoi o 'mai mare decât
de obicei' gradient de temperatură apare cu o creștere legată stocate de căldură.
Geotermale informațiile au fost obținute din minerit, de explorare de petrol
și studii geologice; prin urmare, unele geotermale informații este
disponibil pentru cele mai multe țări. Cel mai important parametru este temperatura
gradient; măsurători precise depind lăsând gaura de foraj
netulburată timp de mai multe săptămâni, astfel că temperatura de echilibru este restabilit
după foraj. Adâncime forate sondaj wells frecvent ajunge la adâncimi de 6 m,
iar tehnologia este disponibil pentru a detalia la 15 km sau mai mult. Costurilor mari
din aceste sondaj wells este, parțial, de ce suspectat de grad înalt
potențial geotermal din multe țări în curs de dezvoltare nu a fost încă suficient
explorat; grad inferior de căldură nu are nevoie de astfel de evaluare detaliată
înainte de a putea fi exploatat. Principalele componente ale unui geotermale
de energie plante sunt foraje, deci extragerea caldurii din adâncurile la 15 km
pot fi inventate în cele din urmă.
Există trei clase de nivel mondial geotermale regiuni:
1
Hipertermie: gradient de Temperatură
≥80ºC/km. Aceste regiuni sunt
, de obicei, pe faliilor tectonice. Prima asemenea regiune a fi exploatat
pentru generarea de energie electrică a fost în 1904 la Larderello în Toscana, Italia.
Aproape toate centralele electrice geotermale sunt în astfel de zone.
2
Semithermal: gradient de Temperatură
~40 ° C/km la 80ºC/km. Astfel
regiunile sunt asociate în general cu anomalii departe de farfurie
TWIDELL PAGINARE.indb 501
01/12/2014 11:38
502
Energia geotermală
limite. Extragerea caldurii este din valorificarea acvifere naturale sau
fracturarea uscat rock. Un exemplu bine cunoscut este geotermale district
sistem de încălzire pentru case din Paris.
3
Normal: gradient de Temperatură
<40 ° C/km. Aceste regiuni rămase
sunt asociate cu medie geotermale conductoare fluxului de căldură la
~0.06 W/m
2

. Este puțin probabil ca aceste zone poate furniza geotermale


de căldură la prețuri competitive pentru a prezenta (finite) sau viitor (alte
surse regenerabile) livrările de energie.
În fiecare clasă este, în principiu, este posibil pentru căldura care urmează să fie obținute prin
următoarele:
1
Naturale hidrotermale de circulație, în care apa pătrunde adânc
acvifere pentru a fi încălzită cu abur uscat, vapori/lichid amestecuri sau în cada cu
apă. Emisiile de fiecare tip pot fi observate în natură. Dacă
presiunea crește cu aburi formarea la niveluri profunde, gheizere spectaculoase
pot să apară, ca la gheizere lângă Sacramento, în California și în
Wairakei zona Rotorua din Noua Zeelandă (vezi Fig. 14.5(b)). Rețineți,
totuși, că apa lichidă este evacuată, și nu de abur.
2
Fierbinte sisteme magmatice asociate cu căldură de semi-magmă topită
care se solidifică la lava. Prima centrală electrică ce utilizează această sursă a fost
de 3 MW
e
stație în Hawaii, finalizat in 1982.
3
Uscat rock fracturare. Efectuarea slab uscat rock (de ex. granit) magazine
de căldură de-a lungul a milioane de ani, cu o creștere ulterioară a temperaturii.
Artificiale de fracturare din puțuri permite apei să fie pompat
prin stâncă, astfel încât (în principiu) de căldură pot fi extrase.
Cu toate acestea, există multe dificultăți de ordin practic cu aceasta, cum s-a discutat
în §14.4.3.
În practică, energia geotermală plante în hipertermie regiuni sunt
asociate cu naturale sisteme hidrotermale; în semithermal regiuni ambele
hidrotermală și (probabil) hot rock extracție pot fi dezvoltate;
normal domenii au prea mici pentru un gradient de temperatură pentru comerciale
de interes, cu excepția aproape de suprafata pompe de căldură.
Temperatura de suprafata T
0
Suprapunerea de material
Zona O
z
2
z
1
δ
z
Cald uscat rock
T
1
minim de temperatură utilă
T
2
, temperatura la adâncimea maximă
Adâncime
Fig. 14.4
Profil de cald uscat rock sistem de calcul conținutul de căldură al resurselor (a se vedea §14.3.1).
TWIDELL PAGINARE.indb 502
01/12/2014 11:38
§14.3 rocă Uscată și fierbinte acvifer analiza
503
§14.3 ROCĂ USCATĂ ȘI FIERBINTE ACVIFER ANALIZA
§14.3.1 Uscat rock: algebra pentru a calcula potențialul de ieșire de căldură
Considerăm o masă mare de material uscat se extinde de lângă suprafața Pământului de la adânc în interiorul scoarței
(Fig. 14.4). Piatra are densitatea
r
r
, capacitate de căldură specifică c
r
și secțiune transversală O. Temperatura la suprafață este de T
0

Cu uniformă de material și nu convectie, G este rata de creștere liniară de temperatură T cu adâncimea z. Dacă z


crește în jos de la suprafață la z
= 0,
TT
T
z
zT
Gz
d
d
0
0
=
+
=
+
(14.1)
Dacă minimul de temperatură utilă este T
1
la adâncimea z
1
atunci
T
T
Gz
și z
TT
G
;
(
)/
1
0
1
1
1
0
=
+
=
-
(14.2)
Utile conținutul de căldură
dE, la temperatura T (> T
1
), într-un element de grosime
dz la adâncimea z este:
E
Un z c T T
Un z c G z z
(
)(
)(
)
(
)
r
r
1
r
r
1
d
rd
rd
=
-
=
-
(14.3)
Total util de căldură conținut de rock la adâncimea z
2
devine:
E
Ac G z z dz
Ac G
z
zz
Ac G
z
zz
z
z
Ac G
z
zz
z
Ac G
z
z
(
)
2
2
2
2
(
2
)
2
(
)
zz
z
z
z
r
r
2
1
1
2
r
r
2
2
12
1
2
1
2
r
r
2
2
12
1
2
r
r
2
1
2
0
r
r
1
1
2


-
r
r
r
r
r
=
-
=




=
-




-
-










=
-
+
=
-
=
(14.4)
Alternativ, să-media disponibile temperatură mai mare decât minimul T
1
fi
q:
2
q=
-
=
-
TT2Gzz
(
)/
(
)
2
1
2
1
(14.5)
apoi:
E
C
CGz
z
(
)
2
r
t
0
2
1
q=
=
-
(14.6)
în cazul în care C
r
este total capacitatea termică a rocii între z
1
și z
2
,
C
Ac z
z
(
)
r
r
r
2
1
r
=
-
(14.7)
deci înlocuind pe C
r
în (14.6),
E
Ac G z
z
(
)
2
0
r
r
2
1
2
r
=
-
(14.8)
și în (14.4).
Presupunem căldura este extrasă din stâncă în mod uniform în funcție de temperatura exces peste T
1
de un
fluxul de apă cu debitul volumic
V
.
, densitate
r
w
, capacitate de căldură specifică c
w
. Apa este încălzită de la T
0
printr-o diferență de temperatură
q. Presupunând o perfectă schimbător de căldură, apoi piatra de capacitatea termică
C
r
va misto de un egal schimbările de temperatură, și anume
V
c
t
C
d
d
ww
r
=
r
q-
q
(14.9)
c
d
C
d
d
ww
r
V
t=t
q
q
r
t
=-
-
(14.10)
deci
t
exp(
/)
0
qq
t
=
-
(14.11)
TWIDELL PAGINARE.indb 503
01/12/2014 11:38
504
Energia geotermală
în cazul în care piatra se raceste cu o constantă de timp
t dat de
C
V
c
r
ww
t
r
=
(14.12)
Înlocuind pe C
t
din
Ac z
z
V
c
(
)
r
r
ww
2
1
t
r
r
=
-
(14.13)
Căldura utilă conținutul E
= C
r
q, deci
E
t
e
exp( / )
0
t/
0
ΕΕ
t
=

-
t
-
(14.14)
și rata de extracție la căldură în mod constant scade ca
E
t
E
t
d
d
exp (
/)
0
t
t
=
-
(14.15)
Temperatura de suprafata T
0
Material
deasupra acviferului
Adâncime
Apă caldă
acvifer la T
2
Zona O
T
2
z
2
h
(a)
(b)
Fig. 14.5
o
Profil de fierbinte acvifer sistem de calcul conținutul de căldură;
b
un gheizer, o priveliște comună în multe hidrotermale regiuni.
A LUCRAT EXEMPLU 14.1
(După Garnitura (1976))
1
Calculați căldura utilă conținut pe kilometru pătrat de uscat piatră de granit la o adâncime de 7 km. La
geotermale gradient de temperatură G este constant la 40 ° C/km. Minim de temperatură utilă pentru
generarea de energie electrică este de 140 K peste temperatura de la suprafata T
0
.
r
r
= 2700 kg/m
3
,
C
r
= 820 J kg
-1
K
-1
.
2
Ce este constanta de timp de căldură utilă de extracție folosind un debit de apă de 1,0 m
3
s
-1
m
-2
?
3
Ceea ce este util de căldură rata de extracție inițial și după 10 ani?
TWIDELL PAGINARE.indb 504
01/12/2014 11:38
§14.3 rocă Uscată și fierbinte acvifer analiza
505
Soluție
La adâncimea de 7 m temperatura T
2
este la 7 km
× 40 K/m = 280 K mai mult de T
0
. Minim utile
temperatura este de 140 K mai mult de T
0
, care apare la adâncime 140/40 km
= 3,5 km. Astfel, numai între rock
adâncimi de 3,5 m și 7 m este utilizabil.
Deci de (14.7),
1
E
O
cGz
z
/
(
)/2
(2.7 10 kg m )(0.82 10 J kg K )(40K km)(7.0 m 3.5 m) /2
(2.7 0.82 40 3.5 3.5)(10 )m J. km .m /2
(543 10 J. m ) (10 m) = 543x10 J/m
5.4 10 J/m
0
rr
2
1
2
3
3
3
1
1
2
6
3
1
2
6
3
9
15
2
17
2

r
=
-
=
×
×
-
=
×
×
×
×
=
×
×
=
×
-
-
-
-
-
-

(14.16)
2
Înlocuind în (14.12):
Ac z
z
V
c
VO
c
c
z
z
x
(
)
.
1
(
.
/)
(
)
1
1m s m
2700
1000
820
4200
(3,5 km)
1.84 (
m
ms
) 1.84 10 s = 58y
r
r
ww
r
w
r
w
2
1
2
1
3
1
2
3
3
1
9
t
r
r
r
r
=
-
=
×
×
×
-
=












=
×
=
-
-
-
(14.17)
3
De (14.15),
E
t
d
d
=
5.4 10 J m
1.84 10 s
290 MW km
t = 0
17
2
9
2





×
×
=
-
-
(14.18)
E
t
d
d
= 290MW.m exp ( 10 / 58) = 250MW km
t = 20y
2
2





-
-
-
(14.19)
§14.3.2 Fierbinte acvifere: algebra pentru a calcula potențialul rata de extracție la căldură
Într-o fierbinte acvifer, căldura de resurse constă într-un strat de apă adânc sub suprafața solului
(Fig. 14.5(a)). Vom presupune că grosimea acviferului (h) este mult mai mică decât adâncimea (z
2
) de mai jos
la nivelul solului, și care, în consecință, apa este la temperatura T
2
. Porozitatea, p, este parte din
acvifer conțin apă, presupunând că spațiul rămas a fi piatra de densitate
r
r
. Minim utile
temperatura este T
1
. Caracteristicile de resurse se calculează în mod similar cu cele pentru uscat rock în
§14.3.1.
T
T
T
z
zT
Gz
d
d
2
0
0
=
+
=
+
(14.20)
E
O
CT
T
(
)
0
2
1
=
-
(14.21)
în cazul în care C
o
este eficient termic capacitate de acvifer volumul considerat; compara:
C
o
= [p'
r
w
c
w
+ (1p')
r
r
c
r
]Ah (14.22)
Ca cu (14.9) mai departe, vom calcula îndepărtarea căldurii de către un volum de apă debit de
V
la
q de mai sus T
1
:
V
c
C
t
d
d
ww
o
r
q
q
=-
(14.23)
TWIDELL PAGINARE.indb 505
01/12/2014 11:38
506
Energia geotermală
Deci
EEEE
t
exp(
/)
0
o
t
=
-
(14.24)
E
t
E
t
d
d
( / )exp (
/)
0
o
o
t
t
=-
-
(14.25)
și
C
V
c
p
c
p
ch
V
c
[
(1
)
]
o
o
ww
ww
rr
ww
t
r
r
r
r
=
=
'
+-'
(14.26)
A LUCRAT EXEMPLU 14.2
(După Garnitura (1976))
1
Calcula temperatura inițială, și conținut de căldură pe kilometru pătrat de mai sus 40ºC, de un acvifer de
grosime 0,5 m, adâncimea de 3 m, porozitate 5%, sub sedimentele de densitate 2700 kg/m
3
, căldura specifică
capacitate 840 J kg
-1
K
-1
, gradient de temperatură 30 ° C/km. Sugerează o utilizare pentru căldură în cazul în care suprafața medie
temperatura este de 10ºC.
2
Ce este constanta de timp de căldură utilă de extracție cu un pompat de extracție a apei de 0,1 m
3
s
-1
m
-2
?
3
Ce este puterea termică extrasă inițial și după 10 ani?
Soluție
1
Temperatura inițială:
T
10 C + (30 3)K= 100 C
2
=
×
(14.27)
Din (14.22),
C
[(0.05)(1000)(4200) (0.95)(2700)(840)](kg m J kg K )(0.5 km)
1.18 10 J K km
o
3
1
1
15
1
2
=
+
=
×
-
-
-
-
-

(14.28)
Cu (14.21),
E
(1.18 10 J K m )(100 40) C
0.71 10 J m
0
15
1
2
17
2
=
×
-
=
×
-
-
-
(14.29)
Calitatea energiei (a se vedea §14.4.2) este potrivit pentru fabrica de procese sau de uz casnic de termoficare.
2
În (14.26),
(1.2 10 J K km )
(0.1 m s m )(1000kg m )(4200J kg K )
2.8 10 s 90 y
o
15
1
2
3
1
2
3
1
1
9
t=
×
=
×
=
-
-
-
-
-
-
-
(14.30)
3
Din (14.25),
E
t
d
d
(0.71 10 J m )
(2.8 10 s)
25 MW km
t 0
17
2
9
2





=
×
×
=
=
-
-
(14.31)
Verifica:
E
t
V
cT
T
d
d
(
)
(0.1 m s m )(1000 kg m )(4200 J kg K )(60 K)
25 MW km
t 0
ww2
1
3
1
2
3
1
1
2
r




=

=
=
=
-
-
-
-
-
-
TWIDELL PAGINARE.indb 506
01/12/2014 11:38
§14.4 Valorificarea resurselor geotermale
507
§14.4 VALORIFICAREA RESURSELOR GEOTERMALE
Energie geotermală provine din surse de căldură având o gamă mare de
temperaturi și de particularitățile locale. În general, disponibile temperaturile
sunt mult mai mici decât la cuptoare; prin urmare, deși de multă energie este
accesibil, termodinamică este de slabă calitate. Sursele împărtășesc multe
asemănări cu căldura reziduală industrială procese și ocean termice
de conversie a energiei (Capitolul 13). În această secțiune vom revizui
strategia pentru utilizarea energiei geotermale.
§14.4.1 corelarea cererii cu oferta
Căldura din surse geotermale tinde să fie disponibile în mod semnificativ la
temperaturi mai mici decât căldură din combustibili; prin urmare, eficiența
producției de energie electrică este mai mic. Cu toate acestea, exportul de energie prin
rețelele de energie electrică este convenabil și de multe ori se întâlnește nevoile naționale. Dacă
căldura reziduală de la o generație pot fi utilizate, cu atât mai bine.
Generarea de energie electrică, probabil, va fi atractiv dacă sursa
de temperatură este
>300ºC, și neatractive dacă <150ºC. Cu toate acestea, energia
cererea de căldură la
<100 ° C este de obicei mai mare decât cea pentru energie electrică,
și, astfel, utilizarea energiei geotermale de căldură este important, chiar și atunci când
a resurselor geotermale nu este "destul de bine" pentru generarea de energie electrică
(a se vedea §14.4.5).
Mai mulți factori repara scara de energie geotermală de utilizare. Cea
dominantă costurile sunt costuri de capital, în special pentru foraje, ale căror costuri
crește exponențial cu adâncimea. Când temperatura crește cu
adâncimea, și valoarea energiei crește cu temperatura, cele mai multe
sisteme se așeze pe optimă a sonda adâncurile
~5 km. În consecință,
scara de alimentare cu energie de ieșire este de obicei
≥100 MW (energie electrică și
căldură pentru temperaturi ridicate, caldura numai pentru temperaturi scăzute), așa cum se arată în
Exemplele 14.1 și 14.2.
Cantitatea totală de căldură care poate fi extras dintr-o sursă geotermală poate
fi crescută prin re-injectarea parțial răcit cu apă de la
suprafață schimbător de căldură înapoi în rezervor, dar la un cost semnificativ.
Acest lucru are avantajul suplimentar de eliminare a efluentului, care poate
fi de aproximativ 25 kg/m
3
de substanță dizolvată și de a fi un substanțiale poluante (de exemplu, improprii
pentru irigare) (a se vedea §14.6).
Din (14.25),
E
t
d
d
25 MWkm exp( 10 / 90)
22 MW km
t 10y
2
2





=
-
=
=
-
-
(14.32)
TWIDELL PAGINARE.indb 507
01/12/2014 11:38
508
Energia geotermală
§14.4.2 tehnici de Extracție: hidrotermale
Cele mai de succes proiecte geotermale foraje au scufundat în
apă naturale canale în hipertermie regiuni (Fig. 14.6). Aceasta este
metoda utilizată la Wairakei, Noua Zeelandă (Fig. 14.10), și la gheizere
în California. Metode similare sunt utilizate pentru extracție la cald acvifere în
semithermal regiuni, unde convecție naturală poate fi stabilit de
foraj, fără suplimentare de pompare.
Naturale
gheizer
Putere
planta
Centrala
Hot rock
~ 30 km
Manta magna
Adânc plictisesc
~ 5 km
De abur/apă rezervor
~ 280° C
Gheizer
(a)
Fierbinte roci de granit
Fierbinte roci de granit
Reîncărcare zona
Primăvară fierbinte
sau
aburul
Fierbinte
fluide
Fierbinte
lichid
Rece
fluid
Geotermale
bine
Impermeabil cap rock
(termice
conducție)
Impermeabile de rocă de
(conducție termică)
Rezervor
(termice
convecție)
(b)
Fig. 14.6
o
Diagrama schematică, nu este la scară, de hidrotermale centrale într-o hipertermie regiune (de exemplu, Gheizere geotermale
domeniu, California).
b
Geologie de un acvifer într-un hidrotermale regiune (stânga diagramei) și o regiune de uscat și cald, rock (dreptul de diagrama) (nu
la scară). Diagrama indică, de asemenea, unele dintre fluxurile de căldură ("larg" săgeți) și apă ("de linie" săgeți) relevante pentru
energie geotermală.
TWIDELL PAGINARE.indb 508
01/12/2014 11:38
§14.4 Valorificarea resurselor geotermale
509
Centrala
Suprapunerea
rock
Districtul
încălzire
Apă
de injecție
plictisesc
Fluxul
Artificial
fracturat regiune
Pierderile
Cald uscat rock:
de exemplu, granit
~300°C
~ 5-7 km
Fig. 14.7
Diagrama schematică de căldură de extracție la cald, uscat rock sistem. Săgețile negre indică
direcția dorită de curgere a apei; verde (punctată) săgețile indică apă pierdută prin
'nedorite' căi prin zona fracturată.
§14.4.3 tehnici de Extracție: 'geotermale îmbunătățite
sisteme' (EGS)
Surse de cald, uscat rock' (HDR) sunt mult mai abundente decât
hidrotermale regiuni: temperaturi de 200°C sunt accesibile sub un
procent semnificativ de lumea terestră. Acesta și-a motivat costisitoare
de cercetare și dezvoltare în statele UNITE ale americii și Europa de tehnici pentru a
valorifica această căldură pentru producerea energiei electrice. Un rezultat a fost
recunoașterea faptului că puțini subsol, rocile sunt complet uscate, dar acolo
sunt multe regiuni în care utilizarea lor de căldură geotermale necesită
'sisteme geotermale avansate', în care re-injectare este necesar pentru a
menține producția comercială.
În 1980, grupul de cercetare de la Los Alamos Laborator Științific,
statele UNITE ale americii pionier metode de fracturare rock cu presiune de apă rece
în jurul valorii de sfârșitul injecției de foraj (Fig. 14.7). După inițială de fracturare,
apa era pompat în jos injecție plictisesc să se infiltreze în sus prin
hot rock la adâncimi de
~5 m și la temperaturi de ~250°C înainte de a
se întoarce prin superficială conductele de retur. Folosind astfel de 'geotermale avansate
sisteme' (EGS), complexe de injecție și a reveni foraje s-ar putea,
în principiu, permite gigawatt livrările de căldură să fie obținute. Cu toate acestea, ea
s-a dovedit dificil de a constrânge fracturare, astfel încât o destul de mare
parte din injectat apa iese din conductele de evacuare; injectat
scurgeri de apă în alte fracturi și este pierdut, așa cum este indicat în Fig. 14.7.
TWIDELL PAGINARE.indb 509
01/12/2014 11:38

510
Energia geotermală
Aceste dificultăți tehnice și costurile mari de EGS-au limitat
dezvoltarea la doar câteva instalații pilot, în special în Europa și SUA.
Cu toate acestea, 1.7 MW
e

'Desertul de Vârf 2' sistem în Nevada, statele UNITE ale americii


exploatate comercial în 2013. Pentru EGS pentru a deveni o lume
de aplicare cu suficientă putere, tehnologia va trebui să fie redus
în etape, de la instalații pilot de
~1 MW
e
pentru intervalul de la 50 la 200 MW
e

.
Pentru a realiza acest lucru până în 2025, ca avute în vedere de către AIE Geotermale Parcurs,
va necesita o politică puternică și sprijin financiar.
§14.4.4 generatoare de Electricitate, sisteme de
Alegerea de schimb de căldură și turbină de sistem pentru o anumită
sursă geotermală este complex, necesitând o experiență specializată. Fig. 14.8
schițe unele configurații de sistem în uz comun. Aproape
întotdeauna emergente plictisesc apă după utilizare este re-injectat în
rezervor. Cele mai simple sisteme de trecere "uscat" aburi de la sol, direct într -
o turbină cu abur (Fig. 14.8(o)), așa cum este folosit în primul energie geotermală
planta în Italia în 1904, și, ulterior, în alte locuri (de exemplu, Wairakei,
Noua Zeelandă: Fig. 14.10). Geotermale rezervor conține supraîncălzit
apă la temperaturi
>180°C și la presiune mare. Ca apa curge
la suprafață, presiunea scade și unele furuncule ("clipește") în
abur, care este injectat în turbine cu abur putere generatoare
(Fig. 14.8(b)). În alte situații, apa la temperaturi mai mici (110°C până la
180°C) se incalzeste alte fluide de lucru, de obicei, compuși organici, într-o căldură
schimbător; în general, acestea se fierbe la aproximativ 80°C, astfel încât furnizarea
presiune de vapori la o turbină (Fig. 14.8(c)). Turbinele funcționează cu un Rankine
ciclul, ca pentru OTEC solare și iazuri (a se vedea Caseta 13.1). În căldură
schimbătoare, contra-curge lichide sunt separate, dar cu toate acestea, dificultățile
apar din cauza depunerilor și coroziunii de substanțe chimice în răcire
sondă de apă. Probleme similare apar pentru ocean de energie termică
de conversie (Capitolul 13).
§14.4.5 Directă folosește de căldură geotermale
În ciuda folosind țevi izolate, de căldură nu pot fi distribuite în mod eficient
pe distanțe mai mari decât
~30 km, astfel încât utilizarea geotermală trebuie să fie
aproape de aprovizionare. În zonele cu climă rece, de uz casnic și de afaceri
districtheating scheme s-au dovedit viabile, dacă densitatea populației este de
≥350 de persoane/km
2
(
>100 spațiilor/km
2
). Astfel, o 100 MW
th
geotermale
planta poate servi o zonă urbană
~20 m × 20 m la ~ 2 kW
th

per incinta.
Alte încălzire sarcini sunt pentru sere, încălzire, piscicultura, uscarea produselor alimentare,
fabrica de procese, etc.
Tabelul 14.2 enumeră unele dintre principalele directă folosește de căldură geotermale
și țările care au cea mai mare utilizare. Doar Islanda și Japonia sunt
TWIDELL PAGINARE.indb 510
01/12/2014 11:38
§14.4 Valorificarea resurselor geotermale
511
Sarcina
Geotermale rezervor de apă
fluid de Lucru
Turbina
Turbina
Generator
Geotermale rezervor
Schimbător de căldură
cu fluidul de lucru
De producție
bine
Pentru injectare
bine
Căldură
schimbător de
Ajungând
unitate de rezervă
Utilizator apartament
(a) Uscat-centrale de abur
(c) ciclu Binar centrala
(d) sistemul de termoficare
(b) Fulgeră-centrale de abur
Generator
Sarcina
Flash
rezervor
Turbina
Separat
apa
Generator
Sarcina
Producție
bine
Producție
bine
Producție
bine
Injecție
bine
Injecție
bine
Injecție
bine
Straturi de rocă
Straturi de rocă
Straturi de rocă
Fig. 14.8
Diagrame schematice de aplicații de căldură geotermale:
o
pentru a
c
trei tipuri de generatoare de energie electrică a sistemului,
d
sistemul de termoficare. Pentru mai multe detalii, a se vedea textul.
Sursa: După EERE (2004).
de asemenea, mari producători de energie electrică geotermală (Tabelul 14.1); mai mult decât atât, în
Islanda, energia geotermală este o sursă principală de energie electrică și
de încălzire. Pentru încălzire directă aplicații (spre deosebire de energie electrică) se poate folosi
surse geotermale la temperaturi
<100°C, mult mai multe țări folosesc
surse geotermale de căldură decât pentru energie electrică. Tabelul 14.1 indică, de asemenea,
că multe dintre țările cu cea mai înaltă calitate din surse geotermale
sunt la Tropice, și deci au nevoie de puțin pentru încălzirea spațiului.
TWIDELL PAGINARE.indb 511
01/12/2014 11:38
512
Energia geotermală
§14.5 SOL-SURSA DE POMPE DE CĂLDURĂ (GHP)
Pompe de căldură acționate de o sursă de alimentare de încălzire și/sau răcire,
și sunt adesea descrise ca o formă de energie regenerabilă (§14.1). Căldura
trece într-un spațiu construit (încălzire) sau spațiu (de răcire), fie
(i) au fost extrase din sol sau aer din exterior (de încălzire); sau
(ii) care trece în pământ sau aer (răcire). Atunci când cele mai multe dintre energia
de schimb este cu locale de teren sau de apă, tehnologia este numit '
Groundsource pompe de căldură (GHP)'. Sistemele de schimb de căldură cu aproape
temperatură constantă, T
g
, sub pământ, la adâncimi de la 2 la 50 m, pro-
viding căldură în timpul iernii și răcire în timpul verii. T
g

la 2 m adâncime frecvent
este egală cu temperatura medie anuală de deasupra solului (a se vedea Problema 14.3
pentru a aprecia ce T
g

rămâne aproape constant la această valoare). Deși acest


schimb de energie nu este legată de adâncime geofizice fenomene
prezentate în §14.2, vom include tehnologia în același capitol, pentru că
de populare, dar inexacte, descriere, ca o pompă de căldură geotermală',
ceea ce implică faptul că este o formă de energie geotermală. Când schimbul
este cu aer, este numit 'Aer-sursa de pompe de căldură'.
O pompă de căldură este, în esență, un frigider lucru invers'. Un motor,
de obicei electric, care funcționează la puterea P
m
permite dispozitivului pentru a extrage căldură
la o rată P
g
din aer sau de la sol de mediul exterior, și să livreze
Tabelul 14.2
Aplicații directe de căldură geotermale, 2010.
Cerere
Instalat
capacitate
GW
th

Nr. de
țări
de raportare
Mai mari utilizatori de națiune [o]
Medie
capacitate
factor de Z
Observații
Încălzirea spațiului
5.4
24
Islanda, China, Franța, Turcia,
Rusia
0.47
În principal de termoficare
Baie și
înot
6.7
67
China, Japonia, Turcia, Brazilia,
Mexic
0.52
Estimările [b]
Cu efect de seră
încălzire
1.5
34
Turcia, Ungaria, Rusia, China,
Italia
0.48
Acvacultură
0.6
22
China, SUA, Italia, Islanda, Israel
0.56
Uscarea culturilor
0.1
14
0.42
Utilizări industriale
0.5
14
Islanda
0.70
Alte utilizări
0.4

------Subtotal (cu excepția


GHP)
15.3
Geotermale
(aproape de suprafață)
pompe de căldură
35.2
43
Statele UNITE ale americii, China, Elveția,
Norvegia, Germania
0.19
În statele UNITE ale americii, cea mai mare parte pentru
de răcire în timpul verii
Note
o
Islanda este cel mai mare utilizator pe cap de locuitor în fiecare categorie, cu excepția GHP.
b
În multe geotermal încălzite/piscine, apa calda curge continuu dacă piscina este în uz sau nu.
Sursa: Date din sondajul de Lund et al. (2010).
TWIDELL PAGINARE.indb 512
01/12/2014 11:38
§14.5 Sol-sursa de pompe de căldură (ghp)
513
fluxul de caldura P
afară
pentru un scop. Setarea P
afară
= C
polițist
P
m
definește coeficientul de
de performanță (COP); aici cu simbolul C
polițist

. Analiza termodinamica
tratează o pompă de căldură ca un motor termic în sens invers (Fig. 14.9(b)). În sistemele de încălzire
modul de caldura P
g
este luat din pământ folosind motor de putere P
m
; așa căldură
P
afară
= P
g
+ P
m
este livrat. În modul de încălzire, POLIȚISTUL este P
afară
/P
m
= 1 + (P
g
/ P
m
);
în modul de răcire COP este în vigoare P
g
/P
m
.
Pentru o reclamă la sol de origine pompă de căldură, C
polițist

este de aproximativ 3 la 5,
în funcție de temperaturile de intrare și de ieșire. Astfel, utilizatorul primește
de la 3 la 5 mai mult de căldură cu o pompă de căldură decât de disipare a energiei electrice
direct ca energie termică. (Pentru un aer-sursa de pompe de căldură, C
polițist

, este, în general, mai puțin la


aproximativ 2.) Temporar răcit mediu este restaurat din surse regenerabile de
energie care intră din mediul mai larg. Toate aparatele de aer conditionat sunt
pompe de căldură, și de multe pot comuta între încălzire 'ca o pompă de căldură sau
răcire în frigider'.
Pentru o buclă închisă 'sol-pompe de căldură care utilizează' (GSHP), P
g

este obținut
dintr-un fluid de transfer (probabil apa) care circulă în interiorul țevilor de o
îngropat schimbător de căldură. Acest lucru poate fi construită atât timp țevi dispuse
orizontal sub, de obicei, o grădină sau parc auto, sau ca țevi verticale în
relativ adânci foraje (Fig. 14.9(a)). Pentru acesta din urmă, structurale
piloți fundație comerciale-clădiri de scară pot fi folosite în dublu scop.
O instalare tipică poate extrage (P
g
)
≈ 50 kWh/(m
2
an) de la sol
în jurul schimbătorului de căldură poate 25% din timp (
~2000 h/an), în
timpul iernii pentru a incalzi o reglare a temperaturii spațiului încălzit. Aceasta permite original,
la temperatura de la sol în jurul schimbătorului de căldură pentru a fi restaurat
(a)
(b)
Pompa de caldura
P
m
P
g
P
afară
=
P
g
+
P
m
HP
Fig. 14.9
'Geotermale' pompe de căldură:
o
diagrama schematică de unul popular de configurare, cu buclă închisă fluid de lucru în apropierea suprafeței solului;
b
fluxurile de energie descrise în text.
TWIDELL PAGINARE.indb 513
01/12/2014 11:38
514
Energia geotermală
prin conducție termică prin sol și, eventual, de către apele subterane
mișcare. Sursa cea mai de restaurată de căldură este, de obicei, de
soare și de aerul înconjurător deasupra solului, mai degrabă decât de la orice semnificativă a
fluxului termic în sus din surse geotermale; media
fluxului geotermic este
<0,4 W/m
2

, care este nesemnificativă. În practică, pentru incalzirea unei


constructii izolate, zonă necesare pentru captarea căldurii prin
orizontală schimbătoare de căldură este de aproximativ 1,5 ori peretele exterior zonă a
clădirii.
Când operare pentru răcire, pompă de căldură inversează fluxul de a acționa ca un
frigider, astfel încât adăugarea de căldură subterane împrejurimi. Pentru adânc,
vertical orientat schimbătoare de căldură (de exemplu, dacă este combinată cu structurală
piloni), subteran împrejurimile căldură în timpul verii și rece în timpul iernii,
astfel încât să devină o căldură magazin cu șase luni de ciclu reversibil.
Factorii de capacitate pentru GHPs sunt mici în comparație cu alte directă
folosește de căldură geotermale (Tabelul 14.2) pentru GHPs sunt rar folosite
pe tot parcursul anului, și sunt de multe ori supradimensionate pentru vârf de vară și/sau de
iarnă.
Optim performanța teoretică este ca un ciclu Carnot de operare
între intrare temperatura absolută T
g
și de ieșire temperatura T
afară
, astfel:
C
polițist
(carnot)
= P
afară
/P
m
= T
afară
/(T
afară
–T
g
).
(14.33)
Cu T
afară
= 298 K (25°C) și T
g
= 278 K (5°C), C
polițist
(carnot)

= 15. Cu toate acestea,


acest "ideal" este mult mai mare decât valorile de la 3 la 6 obținute în practica de
teren-surse de pompe de căldură, deoarece Carnot analiza presupune infinit de
lent, procese reversibile.
§14.6 ASPECTELE SOCIALE ȘI DE MEDIU
Energia geotermală de la hidrotermale regiuni are un record dovedit
de a oferi, în general, în condiții de siguranță și de încredere producerea energiei electrice la
cost relativ scăzut. Prin urmare, utilizarea sa a crescut constant în
ultimele decenii (vezi Fig. 14.1). Costurile de Capital ale noilor sisteme
sunt despre NOI 2500 dolari pe kilowatt instalat (electric) capacitate, care sunt
similare cu cele ale nucleară și hidro a centralelor electrice. Puterea este generată
continuu de la evaluare completă, cu reduceri pentru întreținere și reparații,
atât de mediu factorii de capacitate sunt
~70% (Tabelul 14.1) și similar cu cărbune și
centrala nucleară, adică producția anuală este de
~70% din plin de rating de 8760 de ore pe
an. Astfel, în site-urile favorabile, costul nivelat al producției de energie electrică
este competitiv cu convenționale (maro) de surse, în special dacă
costurile externe sunt incluse (a se vedea Tabelul D. 4 din Anexa D). Odată utilizate,
de căldură - extragerea fluidelor sunt fie evacuate la nivel de suprafață sau
re-injectat. Suprafața de descărcare de gestiune necesită o atentă monitorizare a mediului
și poate fi dăunător. Re-injectarea de fluide sub presiune
în rezervor, în general, îmbunătățește producția de energie, dar poate
provoca micro-cutremure dacă forțat în formațiuni profunde. Hidrogen sul-
TWIDELL PAGINARE.indb 514
01/12/2014 11:38
§14.6 aspectele Sociale și de mediu
515
phide gaz pot fi emise cu fluide, fiind neplăcut să miros
încă nu, în general, la concentrații periculoase. Calitatea apei
monitorizarea în apropiere este esențial să se monitorizeze dizolvate substanțe chimice.
Resursele de cald-uscat-rock' (HDR) sunt mult mai abundente decât
hidrotermale resurse: temperaturi de 200°C sunt accesibile sub
o mare parte a planetei. Goldstein et al. (2011)
indică faptul că dacă 'sisteme geotermale avansate' (EGS) au fost pentru a deveni
de succes la scară comercială, atunci de energie electrică-capacitate de generare
din surse geotermale ar putea fi comparabile la nivel global de energie primară
de alimentare (de exemplu,
>20 EJ/an). Din păcate, chiar și după mai multe decenii de tech-
tehnică de dezvoltare, EGS sunt în continuare doar la o instalație pilot' scena.
Pentru energie geotermală, dimensiunea de resurse este neconfirmate până
foraj are loc, ca și cu ulei sau proiecte miniere. După o astfel de
prospectare, de succes proiecte geotermale dura cel puțin cinci până la șapte ani
să se dezvolte de la descoperirea resurselor pentru dezvoltarea comercială. De mult
ori de dezvoltare și în avans pentru risc financiar de costurile de
explorare a face dezvoltare de resurse deosebit de dificilă în curs de dezvoltare
țări cu vizibile activitate geotermala de la o margine a plăcii, dar limitată
a cererii de energie, cum ar fi Insulele Solomon, insulele mici din
Indonezia sau părți din Africa de Est.
Sol-sursa de pompe de căldură sunt total diferite tehnologii de
energie geotermală de extracție. Ele sunt de obicei de mici dimensiuni furnizarea
de spațiu pentru clădiri de căldură și apă caldă, și poate fi inversat pentru răcire.
Ele sunt un matur și de încredere tehnologie, cu milioane de unități
de operare la nivel mondial.
Vom ilustra impactul asupra mediului de energie geotermală prin
exemplu de 140 MW
e

Wairakei stație de putere în Noua Zeelandă


(Fig. 14.10). Stația a fost construită în anii 1950 în una dintre cele mai
din punct de vedere geologic active zone din lume. Wells (stânga sus al fotografiei)
robinet într-un amestec de apă și abur; apa caldă este separat cu
înaltă presiune abur fiind dirijată prin conducte la putere
stație la dreapta jos. La Wairakei există o considerabilă suprapresiune
în foraje. Nori de abur de la partea de sus stânga veni la cald cu
apa clocotita ca presiunea pe care este eliberat.
Îndepărtarea apei calde de la sol prin stația de alimentare
a dus la tasarea afectează unele clădiri locale. În consecință,
unele de ieșire debitul de apă a fost re-injectat în zona, atenuarea
dificultate. Acolo a fost o diminuare în intensitate de unele dintre
gheizere naturale din zonă ca urmare a centralelor, deși cele mai multe rămân
în mod substanțial afectată. Rețineți că astfel de impact negativ asupra naturale
geotermale fenomene inhibă utilizarea pe scară mai largă a energiei geotermale în Japonia.
În partea de jos a fotografia din Fig.14.10 este Râul Waikato,
care ambele oferă apă de răcire și primește abur condensat
și alte emisii la externare. Inerente de emisii de Sec
2
S este tratat
înainte de descărcare de gestiune. La Waikato este una dintre cele mai mari râuri din țară,
TWIDELL PAGINARE.indb 515
01/12/2014 11:38
516
Energia geotermală
REZUMAT CAPITOL
Există două principalele utilizări ale energiei geotermale, adică de căldură provenind din nucleul fierbinte al Pământului, accesat
la adâncimi de la 1 la 5 km.
1
La câteva locații, căldură geotermală este disponibil la temperaturi >150ºC, cuplat cu un flux natural de
înaltă presiune de apă/abur, astfel încât să permită energie electrică generație de turbine. Mai multe importante
geotermale de energie electrică complexe sunt pe deplin stabilite, în special în Italia, Islanda, Noua Zeelandă
și statele UNITE ale americii. La nivel mondial numărul de geotermală energie electrică plante la o astfel de 'hidrotermale'
Fig. 14.10
La Wairakei geotermale stație de putere în Noua Zeelandă. Bine-capete sunt la partea de sus a fotografiei; abur condensat este
deversată în Râul Waikato în partea de jos.
deci evacuate de căldură și produse chimice rămase sunt rapid diluat. Un
studiu de mediu, în 2001, a constatat că din aval concentrațiile de
elemente chimice, Ca, B și Hg, și de-a dizolvat amoniacul, au fost
mult mai puțin decât în limitele permise de apă cu pești nativi.
Sisteme geotermale, de asemenea, emit gaze cu efect de seră CO
2
. Wairakei e
emisii de 0,03 kgCO
2
/kW
e
h este mai mică decât concentrația medie
pentru energie geotermală stație de emisie de
~0.1 kg CO
2
/ kW
e
h produs,
care este mult mai mică decât valoarea tipică de 1.0 kg CO
2
/ kW
e

h de la o
centrală pe cărbune. În raportul beneficii/costuri de sisteme geotermale
este îmbunătățită prin utilizarea de grad scăzut de căldură lăsând putere
stație. La Wairakei, un crevete ferma de beneficii din acest lucru; acesta este vizibil la
zone dreptunghiulare la stânga fotografiei.
TWIDELL PAGINARE.indb 516
01/12/2014 11:38
Întrebări rapide
517
locatii a crescut în mod constant la o capacitate de ~15 GW(elec) până în 2011, cu creșterea în continuare așteptat.
Tehnologia este matură și de lungă durată, dar trebuie să fie adaptate la fiecare site-ul; acesta poate fi folosit pentru
o bază de grilă de alimentare la o per unitate de cost nivelat al energiei electrice printre cele mai ieftine disponibile.
2
Într-un mult mai larg set de locații, de caldura geotermale este disponibil, dar numai la
~50 la 70°C. este folosit în
mai mult de 60 de țări pentru aplicatii termice, inclusiv apă fierbinte, spa-uri, cartier-de încălzire a spațiului și
de proces industrial de căldură.
Resursele de cald, uscat rock' (HDR) sunt mult mai abundente decât de resurse hidrotermale, la temperaturi de
200°C fiind potențial accesibil sub o mare parte a planetei. Geotermale avansate
sisteme (EGS) exploata această resursă prin care circulă apă la HDR și apoi atingând în încălzit
apă de ieșire. Dacă EGS se dezvolta la scară comercială, de energie electrică-capacitate de generare de energie geotermală
surse ar putea fi
>20 BQ/y, adică comparabile la nivel mondial aprovizionare cu energie primară în anul 2008. Cu toate
acestea, după mai multe
decenii de dezvoltare tehnică, tehnologia este încă la instalație pilot' scena.
Sol-sursa de pompe de căldură (GHPs) – de multe ori eronat numit 'pompe de căldură geotermale' – atingeți în
aproape constantă temperatura la sol, T
g
la adâncimi de la 2 la 50 m, fie pentru încălzire (în timpul iernii, atunci când
temperatura aerului este semnificativ mai mică decât T
g
) sau de răcire (vara, atunci când temperatura aerului este
semnificativ mai mult decât T
g
). Această cerere este disponibil în întreaga lume și în creștere rapidă, deși nu
geotermale în sensul sensurilor (1) și (2).
ÎNTREBĂRI RAPIDE
Notă: Răspunsurile la aceste întrebări sunt în textul de la secțiunea relevantă
din prezentul capitol, sau poate fi ușor dedusă din aceasta.
1
Lordul Kelvin a calculat varsta Pamantului presupunând că este un corp fierbinte
de răcire în vid la o masă topită. El a fost corectă în
ipoteză, și dacă nu, de ce nu?
2
În cazul în care ați uita-te pe Pământ pentru majoritatea activitate geotermala?
3
Temperatura crește cu adâncimea de foraj; cum rata
de crește cu adâncimea indica tipul de geotermală regiune?
4
Descrie cel puțin două mecanisme de căldură pentru a lăsa un geotermale
de resurse.
5
Descrie circumstanțele de apă geotermală pentru a "flash" în abur.
6
Ce tipuri de motoare pot funcționa de la energia geotermală? Explica
modul în care acestea funcționează.
7
Dați două motive pentru re-injectare de efluent lichide înapoi în
rezervoare geotermale și două motive pentru a nu face așa.
8
Din punctul de vedere al unui operator de rețea, compara energie electrică
producției de energie din surse geotermale cu asta din surse fotovoltaice.
9
Căldură reiese dintr-un sol-sursa de pompe de căldură; în cazul în care nu provin de
la?
10
O pompă de căldură este inversată pentru a deveni un răcitor de aer; este coeficientul de
performanță neschimbată? Explicați răspunsul.
TWIDELL PAGINARE.indb 517
01/12/2014 11:38
518
Energia geotermală
PROBLEME
14.1
(a) Un cub de rocă fierbinte' din partea sec are suprafața sa superioară la o adâncime d
sub suprafața Pământului. Piatra are o densitate
r și
capacitate de căldură specifică c. Materialul de deasupra cubului are termică
conductivitate k. Dacă roca este tratată ca o izotermă în masă la
temperatura T deasupra suprafeței Pământului cu interne de căldură
sursă, arată că constanta de timp pentru răcire este dat de:
t
r
=
hcd
k
(b) să Calculeze
t pentru o cubi de masă de granit (cu latura de 10 km, densitatea
2.7
× 10
3
kg/m
3
, capacitate de căldură specifică 0.82
× 10
3
J kg
-1
K
-1

),
situat la 10 km sub pământ, sub un strat uniform de material
de conductivitate termică 0.40 m J
-1
s
-1
K
-1
.
(c) Compara naturale conductoare pierderea de căldură din granit
cu extracția în scop comercial la 100 MW din întreaga masă.
14.2
(a) se Calculează puterea termică produsă din radioactive
decăderea
238
U la 5 km
3
de granit. (
238

U este de 99% de uraniu din


granit, și este prezentă în medie la o concentrație de
4
× 10
-3
%. Căldura produsă de pură
238
U este de 3000 J kg
-1
y
-1
.)
(b)
238

U radioactivitatea reprezintă aproximativ 40% din totalul radioactive


sursă de căldură în granit. Este total radioactive căldură o importantă
sursă continuă de energie geotermală provizii?
14.3
(a) Prin luarea în considerare echilibrul termic între timp t și t
+ dt de
o lespede de unitatea de suprafață la adâncimea z și grosimea dz, arată că
schimbările de temperatură, la o rata de date de
κ


=


T
t
T
z
2
2
(14.34)
(b) să Presupunem că temperatura la sol–aer interfață variază
cu timpul ca
ω
=
+
Tt
T
o
t
(0, )
păcat
0

și că fluxul de căldură și în afară


de solul este numai prin conducție. Arată că temperatura la
adâncimea z este
ω
=
+
-
Tzt
T
oz
tzD
(,)
( )sin(
/)
0
(14.35)
cu
κω
=
D 2( / )
1/2
și
=
-
oz
o
zD
()
(0)exp(
/)
(Indiciu: se Diferențieze stânga (14.35) în raport cu t și
partea dreaptă de două ori cu privire la z.
(c) Pentru un sol tipic
κ=
×
-
-
0.3 10 m s
6
2
1

. Pe o perioadă de un an,


se calculează D și, prin urmare, a găsi vârf–vârf de variație a
temperaturii la adâncimi de 1 m, 3 m și 5 m pentru cazul T
0
=15°C,
o(0)
= 20°C.
TWIDELL PAGINARE.indb 518
01/12/2014 11:38
Bibliografie
519
BIBLIOGRAFIE
General
Dickson, M. și Fanelli, M. (coord) (2005) Energie Geotermală: Utilizarea și tehnologie, UNESCO și Routledge,
Abingdon. Manual de nivel; are un capitol referitor la sistemele de alimentare și patru de capitole privind cererile de incalzire.
DiPippio, R. (2012, 3rd edn) Centralele electrice Geotermale: Principii, aplicații, studii de caz și de mediu
impact, Elsevier, New York. Excelent și până-la-data de nivel profesional, cu text, cu acoperire rezumate de
titlu.
Goldstein, B., Hiriart, G., Bertani, R., Bromley, C., Gutiérrez ‐Negrín, L., Huenges, E., Muraoka, H., Ragnarsson,
A., Tester, J. și Zui, V. (2011) "energie Geotermală", în O. Edenhofer, R. Pichs Madruga, Y. Sokona, K. Seyboth,
P. Matschoss, S. Kadner, T. Zwickel, P. Eickemeier, G. Hansen, S. Schlömer și C. von Stechow (eds), IPCC
Raport Special privind Sursele de Energie Regenerabile și Atenuarea efectelor schimbărilor Climatice, Cambridge University
Press,
Cambridge. O autoritate recentă revizuire a stat de artă, și perspectivele de viitor pentru, energie geotermală.
Agenția internațională a Energiei (2011) foaia de Parcurs pentru Energie: căldură Geotermală și energie, Paris (gratuit pentru
descărcare de pe
www. iea.org). {Starea actuală și perspectivele pentru
~2050; constată o creștere majoră necesită o dezvoltare semnificativă
a
EGS.
Tester, J. W., Drake, E. M., Driscoll, M. J., Golay, M. W. și Peters, W. A. (2012, 2nd edn) Energia Durabilă:
Alegerea între opțiuni, MIT Press, Boston, MA. Capitolul 11 pe energie geotermală se bazează pe co-autor Tester
multe publicații pe acest subiect.
Pompe de căldură
Băncile, D. (2008) Introducere în Thermogeology: Teren de sursa de încălzire și de răcire, Oxford University
Press, Oxford.
Ochsner, K. (2008) Pompe De Căldură Geotermale, Earthscan, Londra. Rezumat excelent de principii și
aplicații. Prezentare foarte clar.
Referințe specifice
Bertani, R. (2010) 'Geotermale generarea de energie din lume: 2005-2010 update', în cadrul Procedurilor de Lume
Geotermale Congres. Disponibil online la www.geothermal-energy.org/pdf/IGAstandard/WGC/2010/0008.pdf
(accesat iunie 4, 2013).
Chandrasekharam, D. și Bundschuh, J. (2008) de Joasă entalpie Resurse Geotermale pentru producerea de Energie:
Explorare și economie, Taylor & Francis, Abingdon. Deosebit de bune pe site-uri mici în țările în curs de dezvoltare.
EERE (2004) Departamentul de Energie al SUA, Raportul DOE/DU-te-102004, 'Îngropat comoara: ecologică, economică
și ocuparea forței de muncă beneficiile energiei geotermale'. Disponibil online la www.nrel.gov/docs/fy05osti/35939.pdf
(iunie 2013).
Mediu Waikato (2004) de Mediu Acordul de Auz la Contactul Energie – Wairakei geotermale
operațiuni. Disponibil online la: www.ew.govt.nz/resourceconsents/hearingsdecisions/contact.htm#Bookmark_tech
apendicele>. A se vedea în special anexe tehnice care dau detalii despre istoria și prezentul funcționare.
Garnitura, J. D. (1976) Energie Geotermală: cazul pentru cercetare în marea BRITANIE, Departamentul pentru Energie hârtie
nr. 9,
HMSO, Londra. Succintă evaluare cu analiza de bază.
TWIDELL PAGINARE.indb 519
01/12/2014 11:38
520
Energia geotermală
Lund, J. W., Freeston, D. H. și Boyd, T. L. (2010) 'utilizarea Directă a energiei geotermale: 2010 în întreaga lume
revizuire'. Disponibil online la www.geothermal-energy.org/pdf/IGAstandard/WGC/2010/007.pdf (accesat iunie
4, 2013).
Site-uri și publicații periodice
Energia geotermală de Asociere (www.geo-energy.org), cu sediul în SUA, dar la fiecare câțiva ani, publică un util
Internaționale Raport de Piață.
Internaționale Geotermale de Asociere (www.geothermal-energy.org), c/o ENEL DP-PDG, Prin intermediul A. Pisano 120,
56122 Pisa, Italia.
Internațional Sursă de Căldură Sol Pompa de Asociere
(www.ceat.okstate.edu/international-ground-sourceheat-pump-association).
Departamentul american de Energie Geotermală de Tehnologie de Birou (www1.eere.energy.gov/geothermal). Are multe
publicații utile și actualizări.
Lumea Geotermale Congrese: Conferința a avut loc în fiecare
~5ani (de exemplu, 2010 Congresul de la Bali, Indonezia);
procedurile disponibile de la Internațional Geotermale de Asociere.
Geothermics – un specialist de cercetare jurnal acoperind atât de căldură și aplicații de putere.
TWIDELL PAGINARE.indb 520
01/12/2014 11:38

Sisteme de energie
De integrare, de distribuție și de depozitare
CONȚINUTUL
Obiective de studiu
522
§15.1 Introducere
523
§15.2 sisteme de Energie
523
§15.2.1 Terminologie
523
§15.2.2 problemele Tehnologice de
integrarea RE în energie
sisteme
525
§15.3 tehnologii de Distribuție
526
§15.3.1 Conducte
528
§15.3.2 Lot de transport
529
§15.3.3 de distribuție a Căldurii
529
§15.4 alimentare cu energie Electrică și rețele
530
§15.4.1 rețelele de energie Electrică (rețele)
530
§15.4.2 Echilibrarea ofertei și
cererea într-o grilă
533
§15.4.3 rețelele Inteligente și virtual
depozitare
537
§15.5 Comparație de tehnologii
pentru stocarea energiei
538
§15.6 stocare a Energiei pentru rețeaua de energie electrică
541
§15.6.1 Pompat hidro
541
§15.6.2 Volante
541
§15.6.3 aer Comprimat
542
§15.6.4 de Mare putere electrică de stocare 544
§15.7 Baterii
544
§15.7.1 plumb-acid baterie
545
§15.7.2 de Litiu pe baza de baterii
550
§15.7.3 Alte tehnologii de baterii
551
§15.8 celule de Combustibil
552
§15.9 substanțe Chimice ca magazine de energie
553
§15.9.1 Hidrogen
553
§15.9.2 Amoniac
555
§15.10 Depozitare pentru încălzire și răcire
sisteme de
555
§15.11 sisteme de Transport
558
§15.12 aspectele Sociale și de mediu
de aprovizionare cu energie și de stocare
559
Rezumat capitol
560
Întrebări rapide
560
Probleme
561
Notă
563
Bibliografie
563
Caseta 15.1 este un mit faptul că energia
depozitarea este o provocare
doar pentru energia din surse regenerabile
532
Caseta 15.2 Auto-suficiente sisteme de energie
532
Cutie 15.3 Capacitate de credit, dispatchability
și predictablity
535
Cutie de 15,4 Grila de stabilitate, cu vânt mare
de penetrare: vest Danemarca
și Irlanda
536
Cutie 15.5 care Combină mai multe tipuri de
variabila RE permite mare
de penetrare: două modelat cazuri 537
Cutie de 15.6 Scalare baterii: fluxul de celule
550
Cutie 15.7 O mică insulă autonomă
vânt-de hidrogen sistemului energetic
554
CAPITOLUL

15
TWIDELL PAGINARE.indb 521
01/12/2014 11:38

www.shahrsazionline.com
522
De integrare, de distribuție și de depozitare
OBIECTIVE DE STUDIU

Apreciem că aplicarea pe scară largă a surselor regenerabile de
energie necesită mecanisme de a integra
variabile consumabile în sistemele energetice.

Înțeleg de ce rețelele de energie electrică se poate
ușor realiza o astfel de integrare până la ~20%
din totalul ofertei (și, în unele cazuri, mai mare %).

Să înțeleagă importanța de stocare a energiei
ca o componentă de o astfel de integrare și
principii fizice de stocare de bază
tehnologii.
LISTA DE FIGURI
15.1 generală a sistemului energetic.
524
15.2 energie Eoliană, cererea de energie electrică, și instantanee de penetrare niveluri în rețelele electrice.
534
15.3 Energie pe unitatea de cost versus energie pe unitatea de volum de metode de depozitare.
538
15.4 aer Comprimat de stocare a energiei și de recuperare a sistemului.
543
15.5 schema de plumb-acid de celule.
545
15.6 caracteristicile de Operare ale tipică plumb-acid baterie.
549
15.7 Fluxul de baterie (schematică).
551
15.8 diagrama Schematică a unei celule de combustibil.
552
LISTA DE TABELE
15.1
Rezumat de mare înseamnă și a fluxurilor de distribuție a energiei
527
15.2 dispozitive de Stocare și performanța lor
539
TWIDELL PAGINARE.indb 522
01/12/2014 11:38
§15.2 sisteme de Energie
523
§15.1 INTRODUCERE
Cererea noastra de energie în forme utilizabile este un factor major în societatea umană
și a economiilor, care necesită soluții tehnologice (în acest capitol),
de gestionare a cererii (Capitolul 16), și la prețuri accesibile și reglementate în mod corespunzător
de alimentare (Capitolul 17).
Luând energie pentru a în cazul în care este de dorit este numit de distribuție sau
transport; păstrarea la dispoziție până când acesta este dorit este numit de stocare. De
exemplu, în cadrul naturale, ecologie, biomasa este o energie magazin pentru animale,
cu fructe și semințe o formă de distribuție. În cadrul societății umane, locale
de distribuție, pe distanțe lungi de transport și de depozitare se stabilesc
de servicii energetice de o serie de tehnologii, inclusiv de transport și
conducte pentru combustibili, energie electrică, rețelele, baterii, hidro-pompat
de stocare și construirea de masă pentru căldură. Combustibili nucleari și fosili sunt în mod eficient
ab initio energie de lungă durată magazine cu mare densitate de energie; utilizarea lor
depinde de minerit, prelucrarea minereurilor și combustibililor, distribuție, de
transport și conducte, și, după generarea de energie electrică, transport
și distribuție de cabluri de înaltă tensiune. În schimb, energia din surse regenerabile
este ab initio o aprovizionare continuă de mediul natural care necesită
potrivit cererii și, pentru utilizarea abundentă, depozitare (reamintim Capitolul 1).
Acest capitol începe cu o prezentare generală a problemelor tehnice în
integrarea energiei regenerabile (RE) în prezent și în curs de dezvoltare sisteme de energie
(articolul 15.2), și apoi recenzii mecanismelor pentru distribuirea de energie, fie ca
energie electrică (§15.4) sau în alte forme (§15.3). §15.5 oferă o imagine de ansamblu de
tehnologii de stocare a energiei, cu mai târziu secțiuni elaborarea pe
anumite tehnologii de stocare și principii fizice, și §15.12
prezintă unele asociate aspectelor sociale și de mediu.
§15.2 SISTEME DE ENERGIE
§15.2.1 Terminologie
Energia este util numai dacă este disponibil în forma necesară, atunci când și
acolo unde a vrut. La "formulare" pot fi clasificate ca căldură, combustibil și
energie electrică. Energia este livrată de procese interconectate de resurse
la final, ca influențată de mai mulți factori (vezi Fig. 15.1). Întregul
sistem la nivel național și internațional este surprinzător de complicată,
indicat în figura de:
(A)
Resursă primară; surse regenerabile de energie, combustibili fosili sau de combustibil nuclear.
(B)
De conversie a gestionat formă (de exemplu, combustibil lichid, electricitate, gaz).
(C)
De stocare pentru o utilizare ulterioară (de exemplu, rezervoare, baterii).
(D)
De distribuție (de exemplu, transport maritim, transport rutier și feroviar, energie electrică,
rețele și rețele electrice, conducte).
(E)
Sector de utilizare finală de clasificare (de exemplu, transportul de combustibil, industriale și
domestice de alimentare cu căldură, electricitate).
(F)
De consum (de exemplu, gospodar, magazin, fabrica).
TWIDELL PAGINARE.indb 523
01/12/2014 11:38
524
De integrare, de distribuție și de depozitare
În plus, există mulți factori care influențează sistemul energetic,
după cum este indicat de următoarele:
(G)
Eficiența energetică: tehnice și îmbunătățiri ale sistemului (principalul
subiect de Capitolul 16; vezi, de asemenea, §15.4.3).
(H)
Rolul naționalizate și private utilități, care sunt mari
organizații reglementate de către guvernele naționale la scară livrările de energie
(a se vedea §17.5).
(I)
O gamă largă și variată de factori instituționali, care sunt principalul
subiect de la Capitolul 17. Acestea includ obiceiurile moștenite (de exemplu, construirea
de proiectare, de preferinta alimente), sănătate (de exemplu, de control al poluării), de securitate (de exemplu,
utilizarea resurselor locale, naționale capacitate de stocare), suport pentru
inovare tehnologică (de exemplu, R&D de finanțare, subvenții și tarife), și
grijă pentru mediu și durabilitate (de exemplu, atenuarea schimbărilor climatice).
Rețineți că sistemele de vedere al Fig. 15.1, cu mici modificări, pot fi
aplicate în mod util la subnaționale scale, inclusiv satele și gospodăriile.
Fig. 15.1
Generală a sistemului energetic, arată de intrare, de ieșire, de distribuție a energiei subsistemului
(inclusiv purtători de energie și de stocare a energiei) și sectoarele de consum final. Vezi textul pentru
titlurile de coloană (A) etc.
Sursa: Adaptat de la SRREN (2011).
Regenerabile
Energie
Resurse
DISTRIBUȚIE

Transport

(solid și lichid

combustibili)

conducte de Gaz

energie Electrică rețele

(grile)
de Încălzire și răcire

rețele

de stocare a Energiei
UTILIZARE FINALĂ
SECTOARE
De Transport și
vehicule
Clădiri și
gospodării
Industria
CONVERSIE
Tehnologii
Fosili și
combustibili nucleari
INTRARE
(A)
(B)
(G) măsurile de eficiență Energetică
(H) Utilitati si furnizorii de servicii energetice
(I) Societal, instituțional și a factorilor de mediu și a legislației
(C)
CONSUMATORII
IEȘIRE
DEPOZITARE
(D)
(E)
(F)
DEPOZITARE
(C')
TWIDELL PAGINARE.indb 524
01/12/2014 11:38
§15.2 sisteme de Energie
525
§15.2.2 problemele Tehnologice de integrare a RE în energie
sisteme de
Tehnologii de marcat (C) și (D) în Fig. 15.1 permite distribuirea și
stocarea de RE, pentru care tehnologiile sunt în mare parte stabilite și
disponibile. Cele mai multe din acest capitol se analizează aceste tehnologii și
principii fizice de bază.
Deoarece utilizarea de RE consumabile necesită o diversiune de o continuare
naturală a fluxului de energie, există provocări în furnizarea de potrivire și a
cererii în domeniul timp, respectiv în potrivire rata la care energia
este utilizată. Aceasta variază cu timpul pe scale de luni (de exemplu, casa de încălzire
în frig și zonele cu clima temperata), zile (de exemplu, iluminat artificial), ore (de exemplu,
de gătit) și secunde (de exemplu, pornind de motoare). În contrast cu combustibilii fosili,
inițial intrări primare de furnizare de energie regenerabilă sunt în afara
controlului. Astfel, așa cum am discutat în Capitolul 1, trebuie să se adapteze (meci) a
cererii (de încărcare) pentru furnizarea de energie regenerabilă și/sau stoca o parte din energie
pentru următoarele beneficii ca biomasă și biocombustibili, forma chimică (de exemplu,
baterii), căldură (de exemplu, masă termică), energia potențială (de exemplu, pompat hidro),
energia cinetică (de exemplu, volante) sau ca potențial electric (de exemplu, condensatori).
Utilizează de RE depind în mod semnificativ de tipul lor și de scara. Unele
surse regenerabile de energie electrică poate fi de scară relativ mare, cu deplin
controlabile de ieșire (de exemplu, mare hidroelectrică, biomasă, de căldură și/sau de putere
de plante), și așa mai utilizate într-o manieră similară cu combustibili fosili plante. Cu toate acestea, în
general, cum s-a discutat în Capitolul 1, livrările de energie electrică din surse regenerabile au
cerințe diferite pentru depozitarea și distribuția decât tradiționale central
de alimentare. De exemplu, unele mari de sursele regenerabile de energie au diferite de timp
dependența (de exemplu, a mareelor, parcurile eoliene, solare concentrate
de plante) și deci necesită integrarea cu alte generarea de energie electrică într-un
sistem comun, care poate benefic includerea de stocare a energiei.
Cu toate acestea, toate formele de energie din surse regenerabile la prețuri relativ mici și moderate
de intensitate pot fi integrate în sistemele în vigoare (de exemplu,
micro-generație în energie electrică, rețelele de distribuție și de biogaz în gaz distribuție
conducte). De intensitate scăzută și răspândită locație de cele mai multe regenerabile
surse de serviciu descentralizat generație și de a folosi.
Toate țările au sistemelor energetice naționale care au fost stabilite
din punct de vedere istoric, potrivit nevoilor și resurselor; cele mai multe, dar nu toate, depinde
de combustibilii fosili și centralizat de furnizare. Astfel, pentru următoarele câteva decenii
, cel puțin, credem că problema este de a modifica acest sistem, pentru a permite
buna integrare a unui procent tot mai mare de surse regenerabile de energie, cu
obiectivul pe termen lung de mișcare întregul sistem de la sursele regenerabile de energie.
Cu privire la fezabilitatea de a face acest lucru, suntem de acord cu concluzia de
autoritate de revizuire de către IPCC (Sims et al. 2011, adesea menționată ca
SRREN):
Costurile și provocările integrării creștere a cotelor de RE
într-un existent de sistem de alimentare cu energie depinde de sistemul
TWIDELL PAGINARE.indb 525
01/12/2014 11:38
526
De integrare, de distribuție și de depozitare
caracteristici, ponderea actuală a RE, RE resursele disponibile
și modul în care sistemul evoluează și se dezvoltă în viitor. Dacă
pentru energie electrică, de încălzire, de răcire, combustibili gazoși sau lichizi combustibili, RE -
integrarea contextuală, site-ul specific și complex. [...]
Infrastructurii energetice existente, piețe și alte instituționale
aranjamente ar putea avea nevoie de adaptare, dar există puține, dacă este cazul,
limitele tehnice planificate de integrare a RE tehnologii
întreaga gamă foarte largă de prezenta sistemelor de alimentare cu energie
la nivel mondial, desi alte bariere (de exemplu, economică și instituțională)
ar putea exista.
Integrarea RE în rețelele de energie electrică (§15.4 și Revizuire 1) este
punct de vedere tehnic simplu, cu natura variabilă a unor surse regenerabile de energie, cum ar
fi energia eoliană și solară, cu atât mai puțin de o problemă decât scepticii au susținut.
Această integrare poate fi îmbunătățită de stocare a energiei, inclusiv prin pompat
de stocare a apei (§ 6.7), volanta de stocare de energie cinetică (§15.6.2),
aer comprimat de stocare (§15.6.3) și de baterii (§15.7). Dacă vehiculele electrice sunt
utilizate, bateriile lor poate deveni o componentă de tehnologii inteligente pentru a
optimiza rețeaua de distribuție și de gestionare a variabilei RE alimentare.
Integrarea RE în termoficare și răcire rețele de gaze
rețele de distribuție și combustibil lichid sisteme este, în general, de asemenea, simplă
dată de compatibilitate și standarde tehnologice au fost îndeplinite. Aceasta
include de pompare de hidrogen în conducte rețelele de distribuție, cu condiția ca
standardele de siguranță sunt îndeplinite (§15.3, §15.9.1). Stocarea de energie sub formă de căldură este
frecvent practicată astăzi (§15.10), și este o opțiune de încălzire și răcire
rețele care să includă variabile RE surse. Diverse RE tehnologii
pot fi, de asemenea, utilizate în mod direct în toate sectoarele de utilizare finală (de exemplu, combustibil-
lemn,
constructii-integrat încălzitoare de apă solare și fotovoltaice, și la scară mai mică,
energia eoliană).
§15.3 TEHNOLOGII DE DISTRIBUȚIE
Această secțiune explică Tabelul 15.1, care rezumă și compară
diferitele moduri de distribuire a energiei. Subiect vital de energie electrică
de distribuție este discutat separat în §15.4.
Distanța și amploarea cântare de RE distribuție, evident,
depinde de capacitatea de alimentare de la sursă și localizarea
cererii. În general, cea mai mare capacitate de aprovizionare, mai mult
rețeaua de distribuție, ca cu cele mai multe de energie hidro și eoliană offshore
putere. Cu toate acestea, un avantaj al energiei regenerabile locale de alimentare poate
fi de multe ori adaptate cererii locale, mai ales atunci când sursele sunt
larg răspândite și, probabil, de capacitate relativ mică. Exemple
sunt scurt-curier transportul de biomasă, și distribuția de căldură
și în clădiri. De furnizare de energie regenerabilă care sunt
mecanice în origine (de exemplu, hidro, val și vânt) sunt, de obicei, cel mai bine distribuit de
TWIDELL PAGINARE.indb 526
01/12/2014 11:38
Tabelul 1
5.1
Rezumat de mare înseamnă și a fluxurilor de distribuție a energiei
Distanta
Fluxul
Distanță medie
Fluxul
Pe distanțe scurte
Fluxul
(>1000 m)
---------
--------
(1-1000 km)
--------
--------
(10m–1 km)
--------
--------
MW
MJ
MW
MJ
MW
MJ
pe unitate
utilizator
-1
zi
-1
pe unitate
utilizator
-1
zi
-1
pe unitate
utilizator
-1
zi
-1
continuă
conducta de petrol
15000
60
conducta de petrol
10000
60
conducta de gaz
(de înaltă presiune)
500
20
conducte de gaz (mare
presiune)
500
20
conducta de gaz
(de joasă presiune)
7
de energie electrică
(de înaltă tensiune)
100
20
de energie electrică
(joasă tensiune)
10
de căldură de la gaze,
vapori sau lichid
lot
petrolier
1200
ulei (sau substitut,
de exemplu etanol)
în vehicul ca
cargo
200
în vehicul în calitate de combustibil
28
cărbune în nave
cărbune în trenuri
biomasa pe camion
15
biomasa pe camion
15
lemn de mână
0.03
15
TWIDELL PAGINARE.indb 527
01/12/2014 11:38
528
De integrare, de distribuție și de depozitare
de energie electrică în rețelele regionale și naționale, în cazul în care, prin natura de
energie electrică, de energie este utilizat la cel mai apropiat locații disponibile. Rețineți că
energia electrică este un vector (vector) de energie, și nu neapărat cel mai important
end-cerința de utilizare. Mișcarea de gaz, poate pe scară largă
de conductele de gaze naturale de astăzi, va fi necesar dacă hidrogen devine o
comună de aprovizionare și a stoca energie.
§15.3.1 Conducte
În conducte care transportă gaze combustibile, curgerea este turbulentă, dar nu supersonic.
Prin urmare, conducta de frecare ecuații de §R2.6 se aplică, deși
interpretarea este afectat de compresibilitate a gazului, după cum urmează.
Ecuația (R2.11) implică faptul că gradient de presiune de-a lungul o lungime mică
a conductei de diametru D este:
r
=−
p
x
f
u
D
d
d
2
2
(15.1)
unde f este factorul de frecare conductă, și
r și u sunt respectiv densitatea
și viteza medie a fluidului. Într-un flux constant de gaz, ambele
r și u varia
de-a lungul lungime de o țeavă lungă, dar debitul masic
r
=
m
uA
.
(15.2)
este constantă; O
= pD
2
/4 este aria secțiunii transversale. În plus, densitatea
r variază cu presiunea p:
r
r
=




p
RT
M
K
0
(15.3)
cu K aproximativ constantă pentru un anumit gaz. R
0
este universală a gazelor
constantă, T este temperatura absolută, iar M este gram masă moleculară/1000,
(deci având unități de kg/mol). Dacă numărul Reynolds (
R
= uD/
ν) (R2.10)
este mare, f nu va varia în mod semnificativ de-a lungul conductei, și putem integra
(15.3) între stațiile x
1
și x
2
pentru a obține:
p

=

p
p
fR Tm x
x
MD
64
.
(
)
1
2
2
2
0
2
2
1
2
5
(15.4)
Astfel, presiunea scade rapid de-a lungul lungimea conductei, și
frecvente de pompare (recompresie) posturi sunt necesare pentru a menține
fluxul. Ca un exemplu numeric, o țeavă de diametru 30 cm, care transportă
gaz metan la o presiune de aproximativ 40 de ori atmosferice, deține un
flux de energie de aproximativ 500 MW, care este foarte importantă (vezi Problema
15.7). Conform (15.4), conducte mari (mai mare D) va avea nevoie de mult mai puțin
de pompare. Cel mai economic echilibru între dimensiunea conductei (cost de capital)
și a pompei de separare (de funcționare cost) depinde în mare măsură de accesibilitate
a conductei.
TWIDELL PAGINARE.indb 528
01/12/2014 11:38
§15.3 tehnologii de Distribuție
529
Compresibilitate a gazului oferă un alt beneficiu. Conducta de sine
poate fi folosit ca un magazin de capacitate reglabila prin simpla pompare de gaz în
mai repede decât este luat de afara, deci gazul comprimat se acumulează în
țeavă. Pentru conducta considerat mai sus, energia "stocate" în 100 de km
lungime ar putea fi:
(32 kg/m
3
) (50 MJ/kg) (10
5
m)
p (0,15 m)
2
= 11 x 10
6
MJ
Astfel "virtuale" de stocare este foarte importantă.
Rata de fluxul de energie (de exemplu, puterea) poate fi, de asemenea, foarte mare în
combustibil lichid conducte (a se vedea Tabelul 15.1 și Problema 15.7).
§15.3.2 Lot de transport
Biomasa poate fi transportat în vehicule adecvate de transport rutier, feroviar, fluvial sau
maritim. Cu toate acestea, densitate mică și voluminoase natura de cele mai multe biomasei ca
recoltate înseamnă că este rar economică fără subvenții pentru a distribui
pe distanțe lungi (>~500 km). Chiar și pe distanțe medii (100 -
500 km), este puțin probabil să fie economic pentru a distribui astfel de biomasă pentru
energie valoarea singur. Principiul de bază economice și ecologice
de utilizare a biomasei este de a interacționa cu un "flux" de biomasa recoltată care
are loc deja pentru un alt scop. Un exemplu excelent este
extracția zahărului din trestie de zahăr, astfel lăsând reziduuri de trestie de zahăr
(trestie de zahăr), pentru a alimenta fabrica, așa cum este descris în Caseta 9.2. În acest caz,
transportul de combustibil pot fi considerate ca fiind "gratuit", sau aproape așa. Biocombustibili poate,
totuși, să fie transportate peste mediu pe distanțe lungi, după ciobire sau
granulare sau după conversie de la brut biomasă (de exemplu, prin piroliză (§10.4),
sau ca biodiesel (§10.9)). În toate țările, lemn de foc este de obicei folosit de aproape de
sursă (<100 km).
§15.3.3 de distribuție a Căldurii
Mișcarea de căldură dintr-o clădire, fie prin aer cald "conductă" sau
ușile deschise, și prin apă fierbinte sau abur conducte este un important mijloc
de distribuire a energiei pe distanțe scurte. Acest lucru este valabil mai ales în
zonele cu climă rece, în cazul în care spațiul de încălzire domină consumul de energie (Fig. 16.3(b)).
Distribuție a căldurii de aburi este, de asemenea, utilizate în multe procese industriale.
Evident eficientă prin conducte sau conducte de distribuție a căldurii nevoile corespunzătoare
de izolare.
A LUCRAT EXEMPLU 15.1 PIERDERILE DE CĂLDURĂ DINTR-O ȚEAVĂ DE ABUR
O țeavă de 6 cm în diametru este de a livra căldură pe o distanță de 100 m. Este izolat cu vata de sticla de
grosime
Dx = 1.0 cm. Estimarea pierderilor de căldură de-a lungul calea. (Ia mediului ambiant T
o
= 10°C.)
TWIDELL PAGINARE.indb 529
01/12/2014 11:38
530
De integrare, de distribuție și de depozitare
Pierderea calculată în Exemplul 15.1 este independentă de fluxul
rata în țeavă. În mod evident, foarte mari fluxurile de căldură (~10 MW) în izolate
conducte sunt necesare în cazul în care pierderile sunt să fie proporțional mai mic. Districtul
de încălzire de acest tip funcționează cu succes în multe orașe.
The heat-pipe oferă un alt mod de a muta cantități relativ mari
de căldură pe distanțe foarte scurte, la fel ca în solare-tuburi vidate colectoare
(§3.6) și Fig. 3.12. Acesta este un tub care conține vapori cu condens
reciclate de un fitil, care are un efectiv de conductivitate mult mai mare
decât cea de cupru (Fig. R3.15).
§15.4 ALIMENTARE CU ENERGIE ELECTRICĂ ȘI REȚELE
§15.4.1 rețelele de energie Electrică (rețele)
Furnizare de energie regenerabilă care sunt mecanice în origine (de exemplu, hidro,
val și vânt) sunt, de obicei, cel mai bine distribuită de energie electrică. În acest fel,
energia electrică este un operator de transport sau vector de energie, și nu neapărat cel mai important
end-cerința de utilizare. Energia electrică este un convenabil și adaptabil formă de
energie atât pentru consumatori, cât și furnizorii, de exemplu, proporția din totalul mondial
consumul de energie s-a dublat de la 11% în 1973 la 22% în 2011 (AIE statistică).
Generatoarele de energie electrică, de obicei, link-uri pentru a încărca cererea de
comun regională sau rețea națională, adesea numit "grila". La
generatoarele pot fi stații de alimentare centralizată sau distribuită mai mică
capacitate încorporat generație, cum ar fi turbinele cu gaz, eoliene și
de uz casnic micro-generare. Grila permite schimbul de generație
și de consum, și astfel oferă o încredere și cele mai cost-eficiente
general mijloace de aprovizionare.
Elementele de bază de generare a energiei electrice și de transmisie sunt prezentate în
Revizuire 1. Majoritatea energiei electrice este generat ca curent alternativ (AC), care
este ușor și eficient transformat de joasă tensiune la o tensiune ridicată de
transmitere a redus pierderile și, ulterior, la tensiune mai mică (de obicei
110 V sau 240 V) pentru utilizare finală'.
În cazul în care energia este furnizată de o rețea de fire, este sensibil pentru o
regiune de a avea un singur (monopol) operatorul, mai degrabă decât concurenții; acest
Soluție
Ca o primă aproximare, să presupunem abur este la 100°C de-a lungul întregii conducte. (Abur la presiune mai mare
va fi de fapt cel mai mare temperatura: a se vedea cele mai multe cărți pe inginerie termodinamicii.) Conductivitatea
de vata minerala este k
= 0.04 W m

1
K

1

(similare de la alte izolatoare, folosind prins aer). Marile


rezistența la pierderile de căldură se face prin conducție prin izolație, astfel încât din (R3.9), unde semnul negativ
indică matematic fluxului de căldură de la cald la rece.
P
kA T x
=
/
= (0.04 W m K )(100 m) (0,06 m)(10
100) °C/(0,01 m)
= 6.8 kW
pierderea
1
1
p

DD




TWIDELL PAGINARE.indb 530
01/12/2014 11:38
§15.4 alimentare cu energie Electrică și rețele
531
este, de asemenea, valabil și pentru conducta de gaze rețele. Cu toate acestea, în multe țări, deși
operațiunea de rețea fizică poate fi un monopol privat, companiile
sunt autorizate de către guvernele să cumpere și să vândă energie electrică competitive,
așa cum este subliniat în Capitolul 17.
Generarea și trimiterea de energie electrică de la o sarcină constantă (cererea) sunt
simple, dar pentru rețelele de utilități cererea se schimbă tot timpul,
astfel încât controlul de generație, de tensiune și de frecvență este nevoie pentru a schimba
la o generație la meci instantanee de curent alternativ cererii.
Adevarate strategii de control și metode s-au dezvoltat, astfel încât
cererea de variație este prevăzută și de aprovizionare nu este întreruptă. Cheia
pentru înțelegerea integrării surselor variabile generație în
rețea este de a realiza că o creștere în variabila generație apare
pentru controlerul de rețea ca fiind similar cu o scădere variabilă a cererii și
invers; deci, aceleași metode de control, în general, pentru a satisface atât pentru
variabila cererii și variabile generație. Bineînțeles, există și alți factori
care trebuie să fie asigurat, mai ales că dispersate sursele regenerabile de energie
opri automat generatoare sau deconectați dacă rețeaua locală conexiunea eșuează.
Cu toate acestea, nici unul dintre acești factori prezintă importante dificultăți. În general,
nu există pic de dificultate și avantaj semnificativ în integrarea într-o
rețea de până la aproximativ 20% din generarea de noi dispersate sursele regenerabile de energie;
sistemele de control instituite pentru că deja a satisface probabil 50% schimbare
în cerere sunt în măsură să controleze dispersate și variabile de intrare.
Apariția de consum redus de energie și de încredere electronice digitale permite
smart tehnologie prin care multe mici sarcini electrice pot fi activate
de la distanță și local pentru a opri sau porni potrivit generație
disponibile, tariful la consumatorul alege, de nevoile de aparat (de exemplu,
temperatura din frigider) și de alți factori. Impactul total de mai multe
mii de astfel de dispozitive într-o "rețea inteligentă" poate avea cele mai semnificative
și impactul pozitiv pe grila de rețele și produce costuri de energie reduse
pentru consumatori.
Generarea de energie electrică din surse regenerabile de energie la scară mică (să zicem de la 1 la
100 kW) pentru gospodarii, ferme, întreprinderi, etc. este posibil, în condiții de siguranță și
cost-eficiente, în special cu panouri fotovoltaice (solare), la scară mică,
turbine eoliene și run-of-the-râul hidro turbine; astfel de tehnologie este
de calitate comercială și de obicei este licențiat pentru a fi conectat la sursa de
grile, precum și pentru stand-alone de putere. Astfel de sisteme sunt denumite
micro-generare. Este posibil pentru a mareelor-curent și maree-gama de putere,
val de putere și alte surse regenerabile de energie generație pentru a funcționa pe astfel de o mică
scară, dar oportunitățile sunt rare și echipament comercial este puțin probabil să
fie disponibil.
Pentru a rula o încredere mare de alimentare cu energie electrică rețeaua de multe diferite
unități este o sarcină dificilă, cu sau fără RE în amestec (Caseta 15.1)
deoarece cererea variază continuu și generarea întotdeauna trebuie să
corespundă cererii. Acest lucru necesită o abordare de portofoliu, inclusiv
sofisticate de distribuție și sisteme de control, control de tensiune și frecvență,
TWIDELL PAGINARE.indb 531
01/12/2014 11:38
532
De integrare, de distribuție și de depozitare
rețelelor inteligente, stocarea energiei, generarea de "rezerve" (dintre care unele trebuie să
răspundă în termen de minute, iar altele să fie disponibile pentru întreruperile planificate și
de întreținere), și o atenție deosebită la modificările cererii pe intervale de timp
variind de la câteva minute la câteva luni. Integrarea unui dispersate geografic
mix de surse de energie regenerabile diferite de timp variabilitatea ambele pot
complica și ușurința de gestionare a rețelei (Cutie 15.5). Mai mult decât atât,
caracteristici inerente unor sisteme pot contribui în mod pozitiv la
stabilizarea rețelei, astfel cum sunt enumerate în §15.4.2.
CASETA 15.1 E UN MIT CARE DE STOCARE A ENERGIEI ESTE O PROVOCARE DOAR PENTRU SURSELE
REGENERABILE DE
ENERGIE
De stocare a energiei este adesea descris ca o provocare deosebită pentru sursele regenerabile de energie, pentru două motive
principale:

Cele mai furnizare de energie regenerabilă sunt variabile de la sursă (soare, vânt, culturi sezoniere, etc.) și nu
în sincronism cu nevoile schimbătoare.

Mai multe surse regenerabile de energie sunt folosite pentru energie electrică, în cazul în care oferta trebuie să fie echilibrat
instantaneu de încărcare pentru un
sistem stabil.
Cu toate acestea, provocări similare pentru energie nucleară (sau mare pentru centralele pe bază de cărbune) nu sunt atât de des
recunoscut:

Producerea de energie nucleară ar trebui să rămână constantă și continuă și deci nu în sincronism cu
noastre schimbătoare are nevoie.

Energia nucleară este utilizată cu precădere pentru energie electrică, în cazul în care oferta trebuie să fie echilibrat instantaneu
de încărcare pentru un sistem stabil.
Atunci când o centrală nucleară 'cade' (de obicei datorită unui defect electric), 1000 MW de putere
generație dispare din rețea în câteva secunde. Acest lucru se întâmplă la întâmplare, la intervale de aproximativ 18
luni pentru fiecare stație, încă grila ajustează și naționale de alimentare este menținută. Deoarece operatorii de rețea a face față cu
întreruperi, în mod normal, neschimbătoare nucleare de aprovizionare, putem fi siguri că ei vor, de asemenea, face față cu o mare
cantitate de energie din surse regenerabile de energie de diferite tehnologii.
Mecanismele de menținere a stabilității rețelei sunt discutate în continuare în §15.4.2 și Revizuire 1.
CASETA 15.2 AUTO-SUFICIENTE SISTEME DE ENERGIE
Din cauza constrângerilor de aprovizionare, impactul asupra mediului și costurile de combustibili fosili, mai ales la locații de la
distanță,
există o tendință de creștere în toate țările față de utilizarea locală RE resurse. Auto-suficiente sisteme de energie
(de asemenea, numit autonome de energie de alimentare) sunt conectate la un utilitar de rețea electrică sau de gaz de alimentare
rețea și nu folosiți combustibili din import, cu excepția cazului în calitate de așteptare. Astfel de sisteme sunt de obicei de
dimensiuni mici și
sunt adesea situate în zone îndepărtate, insule mici, sau clădiri individuale în cazul în care furnizarea de servicii comerciale
de energie nu este ușor disponibile prin rețelele magistrale și rețele. Pe unitate de energie produsă, astfel de sisteme
pot fi costisitoare pentru a stabili, dar de obicei sunt ieftine în exploatare.. Pentru alimentarea cu energie electrică de la 100%
energie din surse regenerabile variabile generație (de exemplu, energia eoliană și solară), de echilibrare a ofertei cu cererea este o
semnificativă
provocare, de obicei, necesită electrice de stocare (de exemplu, baterii) și/sau controlabile cererii. Utilizarea eficientă
a energiei electrice (de exemplu, lumini cu LED-uri) este, în practică, foarte benefic. Electrice de echilibrare poate utiliza bateria
TWIDELL PAGINARE.indb 532
01/12/2014 11:38
§15.4 alimentare cu energie Electrică și rețele
533
depozitare și/sau (bio)diesel generatoare și încărcați-control de gestiune (de exemplu, cu rezistive de încălzire a apei)
pentru a absorbi surplusul de energie. Două exemple pe insule mici sunt Fair Isle în Scoția (Caseta 8.2) și Utsira în
Norvegia (Cutie 15.7). Utilizarea locală lemn si alte deseuri de biomasa pentru căldură fotovoltaice solare de energie electrică
generație și hidro run-of-the-râu (dacă este disponibil) este deosebit de simplu.
Viabilitatea financiară autonome RE sisteme depinde de local, RE resursele disponibile, de capital
și costurile de instalare, costurile de funcționare și granturi; în contrast cu alimentare din instalațiile centrale. În
plus, angajamentul și principala motivație a operatorilor și proprietarilor au valoare.
§15.4.2 Echilibrarea cererii și ofertei într-o grilă
Cererea de energie electrică variază în funcție de nevoile utilizatorului, de obicei, la un
minim pe timp de noapte și în creștere de la un vârf în timpul orelor de lucru (vezi
Fig. 15.2). În plus, există, în mod normal, diferențele între lucrează
zile și week-end/sărbătorile și, de asemenea, între anotimpuri, cele mai multe sisteme
, de asemenea, arată o creștere anuală a consumului de la an la an. Prin urmare,
generarea de pe un sistem trebuie să fie programată (expediate) pentru a se potrivi aceste
variații de-a lungul anului și de rețea corespunzătoare
de infrastructură pentru a transfera puterea de a fi disponibil. Natural pasiv corective
mecanism este că reducerea frecvenței și a tensiunii reduce sarcina,
deci acomodarea intervale scurte de insuficientă generație. Cu toate acestea, aceasta
nu este o bună practică ca tensiune și frecvență variază atât de activ de echilibrare
se realizează de către operatorul de sistem.
De potrivire a cererii și ofertei (de echilibrare) pe un minut-cu-minut
a fost în mod tradițional făcut în principal prin controlul de generație. Acest lucru este cunoscut
ca regulamentul/load urma cu mic spre mediu variații în producția
din centralele electrice. Acesta este, de obicei, controlate automat sau de către o centrală
de energie electrică, operatorul de sistem, care este responsabil pentru monitorizare și
operare a echipamentelor din sistemul de transport și în power-generatoare de
posturi. Dacă este deja de operare de mai jos maximă de ieșire (așa-numitele
'rezerva de filare'), producția de un adecvat de combustibili fosili și biomasă
termică a unității generatoare poate fi crescut sau scăzut fără probleme într-o
câteva minute, deși pornire la rece poate dura ore. Gaze (fosili sau
biogaz) turbinele sunt mult mai flexibile și pot fi rapid a început într-o
câteva secunde. Hydro power systems, cu lor de stocare a energiei inerente
în rezervor, sunt la fel de flexibile; nu există alte sisteme de stocare în prezent
în uz în rețelele electrice au o astfel de capacitate mare, deși R&D este
de a trece (§15.5). Moderne de control electronice și de telecomunicații care
permite feed-forward control (§1.5.3) prin gestionarea cererii, chiar și
pe o scară largă folosind rețelele inteligente', care facilitează integrarea
variabilă RE resurse.
Peste puțin mai lungi perioade de timp (de exemplu, 30 de minute la 24 de ore),
specificate stații de putere activa/dezactiva sau sol/rampă de ieșire
pentru a asigura echilibrul. Unele unități de producere a rula la capacitate maximă toată
ziua (furnizarea baseload); centrale nucleare sunt inflexibile și poate doar
TWIDELL PAGINARE.indb 533
01/12/2014 11:38
534
De integrare, de distribuție și de depozitare
Fig. 15.2
Energia eoliană, cererea de energie electrică, și instantanee de penetrare niveluri în energie electrică
rețelele de
o
vest Danemarca pentru o săptămână, în ianuarie 2005, și
b
insula Irlanda pentru două zile în luna aprilie 2010.
Aceste cazuri ilustrează (i) ciclul de zi cu zi a cererii: în mod semnificativ mai puțin pe timp de noapte decât în
timpul zilei, și (ii) o în mod corespunzător structurată rețea electrică de operare stabil, cu mai mult de
40% din puterea sa vine de la o variabilă sursă regenerabilă.
Sursa: Sims et al. (2011, Fig. 7.16).
Cererea și vânt [MW]
16/01
15/01
14/01
13/01
12/01
11/01
10/01
101%
98%
100
90
80
70
60
50
40
30
20
10
0
Vântul penetrare [%]
5,000
(a)
4,000
3,000
2,000
1,000
0
Cerere
% Vânt
Vântul
Cererea și vânt [MW]
05-Apr-10
04-Apr-10
42%
50
40
30
20
10
0
Vântul penetrare [%]
5,000
(b)
4,000
3,000
2,000
1,000
0
Cerere
% Vânt
Vântul
contribuie la sarcina de bază. Alte unități (de exemplu, turbinele cu gaz și pompat hidro)
au timp de răspuns rapid și pot fi utilizate în principal în perioadele de vârf ale
cererii (maxim de unități). Potrivit tarifelor stimuleze consumatorii pentru a reduce
cererea la orele de vârf (de exemplu, pentru sisteme de încălzire sau răcire care includ
temporară de stocare a energiei).
Generarea de întreruperi sunt sigur de a avea loc (de exemplu, pierderea de 1000 MW de
centrale generatoare de aprovizionare sau pierderea de o conexiune de rețea), ceea ce necesită
reechilibrare rapidă de 20% din putere în sistem. Prin urmare,
rețelele sunt concepute pentru a rezista la pierderea de orice singur element critic,
TWIDELL PAGINARE.indb 534
01/12/2014 11:38
§15.4 alimentare cu energie Electrică și rețele
535
deci, nici un alt element este supraîncărcat și satisfăcătoare de alimentare continuă.
Din toate marile sisteme de rețea poate satisface rapid cererea de fluctuațiile de
~20%, este de asemenea posibil pentru aceleași sisteme pentru a satisface aprovizionare rapidă
fluctuațiile de ~20%. Astfel, includerea de variabile consumabile, cum ar fi
energia eoliană, este posibil. (A se vedea on-line de materiale suplimentare pentru acest
capitol pentru mai multe discutii de cât de mult de back-up nu-rețea integrată
de energie eoliană nevoie?)
Mai mult, unele sisteme pot contribui în mod pozitiv la stabilizarea
rețelei, de exemplu:

Mici și mijlocii solare FOTOVOLTAICE este de obicei instalat lângă cererea
și conectat la nivelul distribuției. La mici străpungeri în
distribuție alimentatoare (PV capacitate <100% orele de vârf de sarcină pe feeder), PV poate
compensa nevoie de distribuție upgrade-uri (în cazul în care cererea de vârf de pe
feeder apare la lumina zilei) și de a reduce pierderile.

Conectat la rețea a sistemelor FOTOVOLTAICE folosesc invertoare de rețea interfață,
care permite în principiu control de caracteristicile electrice relevante pentru
integrarea în rețea.

Solare bazate pe sisteme electrice de putere (PV sau CSP) ambele oferă putere de vârf
de energie atunci când cererea de vârf apare la cald, locurile insorite (de exemplu, California)
în cazul în care cererea de vârf este determinată de aer condiționat și alte răcire
sisteme.

Pentru rezervor pe bază de hidroenergie, atunci când este disponibilă apă, electrice
producția de plante este foarte controlabile și pot oferi semnificativ
de flexibilitate pentru sistemul de operare. Rezervor de capacitate poate varia de la
termen scurt la sezon la multi-sezon. De stocare a energiei în
rezervor permite hidrocentrale să funcționeze în bază modul sau ca sarcina
următoarele plante.

Modern centralele eoliene sunt conectate la sistemul de alimentare prin
convertoare electronice de putere, și pot fi echipate pentru a oferi o grilă
servicii, cum ar fi de putere activă, putere reactivă și tensiune de control,
frecvența de răspuns (de tip inerțial de răspuns) și sistemul de putere sprijin
în timpul defecte de rețea.
CUTIE 15.3 CAPACITATE DE CREDIT, DISPATCHABILITY ȘI PREDICTIBILITATE
Pentru funcționarea rețelei, o unitate generatoare de' este un individ mare generator electric sau un cuplate grup de
generatoarele de mici dimensiuni. Dispecerizabile unități sunt cele în care producția poate fi ușor variat în mod direct sau
indirect de către operator între un minim și un nivel maxim. Ieșirea din unele unități (de exemplu, cele mai multe
turbine eoliene) nu este, de obicei, controlate și depinde de variabile locale resurse energetice. Cu toate acestea,
control de la distanță este posibil pentru unități mai mari (de exemplu, fermele eoliene offshore) printr-o reducere a producției sau
printr-o creștere anterior de-capacitatea nominală. Astfel de unități sunt parțial dispecerizabile'.
Generație de centrale hidroelectrice de putere mare putere (cu baraj), geotermale și biomasă este pe deplin dispecerizabile,
întrucât
mare de energie eoliană, FOTOVOLTAICĂ, și a valurilor sau a mareelor cu control de la distanță sunt doar parțial
dispecerizabile.
Concentrarea energiei solare (CSP) generatoare poate să includă mai multe ore de stocare termică, care
TWIDELL PAGINARE.indb 535
01/12/2014 11:38
536
De integrare, de distribuție și de depozitare
le face capabile de a întâlni vârfuri de zi cu zi, adică acestea pot fi considerate ca dispecerizabile. Un concept legat este
predictibilitate, adică precizia cu care planta puterea de ieșire poate fi prezis cel relevante scale de timp
pentru a ajuta puterea sistemului de operare. Pentru o zi înainte, previzibilitatea poate fi evaluat "ridicat" pentru bioenergie,
CSP cu acumulare de căldură, geotermală, hidroelectrică și energia mareelor, dar numai "moderate" pentru PV, val
și vânt de putere (a se vedea Tabelul D. 4 din Anexa D). Meteorologice și aliate tehnici de prognoză sunt
îmbunătățirea continuă (a se vedea §7.4) și, în consecință, previzibilitatea de variabile surse regenerabile. În
plus, unele resurse regenerabile de energie sunt mai previzibile atunci când agregate pe o suprafață mare,
mai degrabă decât incluși în eșantion la un anumit site. Aceasta este diversitatea geografică potențial indicate în Tabelul D. 4.
Nu generator poate fi invocată pentru complet asigurat de aprovizionare din cauza timpii de întreținere,
neprevăzute de defecte, și, pentru cele mai multe surse regenerabile de energie, de mediu și meteorologice variație. Capacitate de
credit
este o măsură statistică pentru operatorii de sistem de contribuția pe care un generator poate fi presupus în
orice moment viitor, să contribuie la asigurat de aprovizionare. Aceasta se referă la disponibilitatea de care a ofertei. De
exemplu,în
special operatorii de sistem poate rata de 1 GW de la centrala nucleară ca disponibilitate 80% și deci capacitate de credit de 0,8
GW, și 1 GW de la nivel național dispersate de vânt de putere ca disponibilitatea 25% și deci capacitate de credit 0.25
GW. Acesta
este incorect să spunem că variabila regenerabile generație are zero capacitate de credit pe o rețea națională și
deci are nevoie de 100% "backup" generație. Pentru discuții suplimentare de capacitate de credit de energie eoliană, vezi
online materiale suplimentare pentru acest capitol.
Cutie de 15,4 descrie două cazuri în care rețeaua a rămas echilibrat
chiar și cu penetrare mare de energie din surse regenerabile variabile. În ambele cazuri,
rețeaua a rămas stabilă în ciuda instantanee fracțiune de energie eoliană în
sistemul de peste 40% (vezi Fig. 15.2).
Diversitatea de caracteristici ale diferitelor RE resurse și
tehnologii pot ajuta, de asemenea, pentru a stabiliza o rețea electrică. Cu alte cuvinte, este mai ușor
de furnizare a energiei electrice cu un procent mare de surse regenerabile de energie să urmeze
cererea de energie electrică în cazul în care include o serie de tehnologii, și nu doar un singur
tip. Cutie de 15,4 prezintă două astfel de cazuri.
CUTIE DE 15,4 GRILA DE STABILITATE, CU VÂNT MARE DE PENETRARE: VEST DANEMARCA ȘI IRLANDA
Danemarca are cea mai mare energie eoliană penetrare din orice țară din lume (vânt de alimentare cu energie
s-a cifrat la 28% din totalul anual 2012 cererea de energie electrică). Total putere a capacității eoliene instalate până la sfârșitul
anului 2012 s-a cifrat 4 GW, în timp ce cererea de vârf a fost de 6,5 GW. Cele mai multe dintre capacitatea de energie eoliană (3
GW)
este situat în vestul Danemarcei, care rezultă în instantanee a vântului putere de ieșire care depășește cererea totală
în vestul Danemarcei în unele cazuri (a se vedea Fig. 15.2). Daneză exemplu demonstrează beneficiile
de interconectare cu țările vecine (Germania, Suedia și Norvegia) și de sistemul internațional
operatorii (NORWEB) pentru a integra energia eoliană cu energia de echilibrare din complet dispecerizabile de energie hidro.
Insula Irlanda (Republica Irlanda și Irlanda de Nord) are un singur AC rețeaua de energie electrică
cu două înaltă tensiune DC submarin interconexiuni (fiecare de 500 de MW de capacitate) la insula din marea Britanie,
care la rândul său are un HVDC 2 MW capacitate submarin conexiune cu restul Europei. Unul dintre scopurile legate
de rețele este pentru exces de energie eoliană să fie exportate din Irlanda în marea Britanie. Toate acestea conectivitatea este parte
din
embrionare Europene Supergid. Până în 2013, Irlanda, capacitatea instalată de energie eoliană de 2 GW a fost capabil de
furnizarea aproximativ 15% din Irlanda anuală a cererii de energie electrică. Irlandezii grila operatorilor de sistem cu succes,
a reușit vântul putere de penetrare, care uneori furnizate 40% a cererii în combinație cu
TWIDELL PAGINARE.indb 536
01/12/2014 11:38
§15.4 alimentare cu energie Electrică și rețele
537
§15.4.3 rețele Inteligente și de stocare virtuale
Dezvoltarea de ieftin și eficient de comunicare,
de monitorizare și de mici dimensiuni controlul transformă structurile de putere electrică
sisteme. Termenul de "rețea inteligentă" este adesea folosit pentru a se referi la acest amestec
de tehnologii noi, care să ofere o mai de încredere rețea de alimentare prin:
(a) comutarea de la distanță a sarcinilor și generație după convenit tarifelor;
(b) în mod automat identificarea și rezolvarea problemelor, și (c), prin urmare,
îmbunătățirea calității ofertei. Comunicarea poate fi prin înaltă frecvență semnale
suprapuse pe linia de alimentare sau de telecomunicații. Prin reducerea
cererii, în special la orele de vârf, producătorii de energie electrică pot reduce
capacitatea de generare și consumatorii pot beneficia de mai ieftine tarife.
Mannheim, în Germania funcționează un bun exemplu de astfel de "rețea inteligentă".
O "rețea inteligentă" este un exemplu de conceptul mai general de virtual
de stocare, în care puterea cererii și ofertei de nepotriviri sunt depășite
de dinamic remodelarea cererii de energie pentru a se potrivi o variabilă de energie
generație de turbine cu gaz (ca un exemplu, a se vedea Fig. 15.2). Irlanda are doar moderat de dealuri și deci nu este
semnificativ naționale la scară hidro de putere; prin urmare, la scară mare de stocare a energiei cu flow-baterii
(Cutie 15.6) este în curs de dezvoltare, în principal pentru a echilibra energia eoliană.
CUTIE 15.5 CARE COMBINĂ MAI MULTE TIPURI DE VARIABILE RE PERMITE MARE DE PENETRARE:
DOUĂ CAZURI MODELAT
Rețelele de energie electrică bazată pe combustibili fosili și nucleari intrări necesită întotdeauna grila pentru a avea substanțiale
excesul de
capacitate (Caseta 15.1) de poate 30% pentru a acoperi vârfurile de cerere și a plantelor întreruperi. Ce s-ar întâmpla
dacă un mix de energie din surse regenerabile variabile capacităților în comun a devenit dominantă de aprovizionare și nu există
nici o singură
dominantă dispecerizabile de aprovizionare, cum ar fi hidro de putere, pentru a "ancora" grila? În astfel de cazuri, un amestec de
diferite
energii regenerabile generație este mult mai probabil de a furniza semnificativă a ofertei naționale decât un singur regenerabile
de aprovizionare. De exemplu, energia eoliană este de multe ori mai mare în timpul iernii și pe cer acoperit de zile, și solar este
mai mare în
timpul verii și în zilele senine; astfel, energia eoliană și solară se completează reciproc. Pentru a testa și a planului de astfel de
amestecuri
necesită o atentă modelare în avans de construcție.
Un astfel de studiu considerat o combinație de solare (FOTOVOLTAICE și CSP), eoliană, geotermală și hidro în
California (SUA), cu turbine cu gaz preluarea mic dezechilibru [1]. Un alt considerat o combinație de
onshore și offshore, eoliene, FOTOVOLTAICE și electrochimice de stocare într-o mare rețea interconectată în partea de nord-est
a SUA, care acoperă aproximativ o cincime din totalul cererii de energie electrică din statele UNITE ale americii, dar nu are
'baseload RE potențial de hidro sau geotermale [2]. Ambele studii au produs mai puțin costisitoare soluții în concordanță
cu sarcina și de constrângerile legate de resurse, și a constatat că acestea necesită o capacitate totală instalată a surselor
regenerabile de energie
considerabil mai mare decât în orele de vârf de sarcină. Cu toate acestea, reducerea anticipate în costul de eoliene și
FOTOVOLTAICE de generare a sugerat că excesul de capacitate de aceste surse regenerabile de energie ar fi mai ieftin decât
adăugând suficientă
capacitate de non-hidro de stocare a energiei.
Surse: [1] E. K. Hart și M. Z. Jacobson (2011) 'Monte Carlo de abordare a generator de portofoliu de planificare [...] de
sisteme cu mare penetrari variabilă a energiei din surse regenerabile, Energie Regenerabilă, 36, 2278-2286. [2] C. Budischak et al.
(2013) 'Cost-minimizarea combinații de energie eoliană, energie solară și electrochimice de stocare, alimentarea grila de până la
99,9% din timp, Jurnalul de Surse de Energie, 225, 60-74.
TWIDELL PAGINARE.indb 537
01/12/2014 11:38
538
De integrare, de distribuție și de depozitare
de aprovizionare. Său caracter dinamic face virtual storage o extensie a
mai static conceptul de gestionare a cererii'. Dinamic
de răspuns poate fi realizat prin crearea 'inteligente distribuite de energie
eficiență , precum și folosind construirea de structuri și sisteme pentru a modifica
consumul de energie (a se vedea §16.4 pentru exemple). După cum sa subliniat în Capitolul 16,
modificarea cererii de eficiență îmbunătățită a consumului final de energie este aproape
întotdeauna o mai cost-eficientă și un risc mai mic soluție decât adăugarea de alimentare
, sau folosind greu tehnologii de stocare.
§15.5 COMPARAȚIE DE TEHNOLOGII
PENTRU STOCAREA ENERGIEI
Fig.
15.3 rezumă performanța de stocare diverse meca-
aajutorului. Performanța poate fi măsurată în unități, cum ar fi MJ $
-1
, MJ m
-3
și MJ kg
-1

. Dintre acestea, prima unitate (cost-eficacitate) este, de obicei,


factorul decisiv pentru comerț, dar este mai greu de estimat (a se vedea
Capitolul 17); rețineți că "cost" aici este costul de gros înainte de taxe și că
impozitarea, în special de transport, combustibili, variază foarte mult între țări.
Cea de-a doua unitate este important atunci când spațiul este la o primă (de exemplu, în
clădiri de dimensiuni fixe). Cea de-a treia unitate se consideră atunci când greutatea este esențială (de exemplu
în avion). În acest capitol vom arata cum aceste cifre de performanță
sunt estimate.
Tabelul 15.2 rezumă principalele caracteristici ale tuturor stocare a energiei
mecanisme examinate în acest capitol mai în detaliu decât Fig. 15.3;
Fig. 15.3
De energie pe unitate de costuri versus energie pe unitatea de volum de depozitare metode (orientativ prețurile
în dolari SUA în 2012). NB: scale logaritmice sunt utilizate. Pentru mai multe detalii a se vedea Tabelul 15.2. Notă
superioritatea de "ulei", care include petroliere și cele mai biocombustibili lichizi.
0.01
10
-4
0.01
0.1
1
10
100
0.1
1
10
Fierbinte
apa
Ulei
Hidrogen
Chimice
Comp
aer
Avansat
baterii
LA
baterii
Energie pe unitatea de cost/(MJ
$
-1
)
De energie pe unitatea de volum/(MJL
-1
)
100
Pompat
hidro
TWIDELL PAGINARE.indb 538
01/12/2014 11:38
Tabelul 1
5.2
Dispozitive de stocare și performanța lor
[o]
Magazin
Densitatea de energie
De operare
Probabil comerciale
Valoarea sa de utilizare
Conversie
Eficiența
temperatura
timpul de dezvoltare
tip
--------
-------
-------
----------------
-----------
--------
MJ kg
-1
MJ L
-1
°C
ani
MJ $NE
-1
%
Combustibilii convenționali
Motorina
45
39
ambient
în uz
100 [b]
chem–>munca
30
Cărbune
29
45
ambient
în uz
500 [b]
chem–>munca
30
Lemn
154
7
ambient
în uz
200 –

chem –>caldura
60
Alte substanțe chimice
Gaz de hidrogen
140
1.7 [d]
(-253) la (-30)
10
0.2–20 [b]
elec–>chimie
60
Amoniac (la N
2
+H
2
)
2.9
0.3 [d]
0-700
10
~1 [b]
căldură –>chimie
70
FeTiH
1.7
1.8
20
100
10
1 [b]
chem –>chimie
90
Căldură sensibilă
Apa
0.2
0.2
20-100
în uz
3-100 [c]
căldură –>caldura
50-100 [e]
Fonta
0.05
0.4
20-400
în uz
0.1 [c]
căldură –>caldura
50-90 [e]
Căldură (schimbare de fază}
Steam
2.2
0.02 [f]
100-300
în uz
10
căldură –>caldura
70 [d]
Na
2
DECI
4
.10H
2
O
0.25
0.29
32
în uz
2 [c]
căldură –>caldura
80
Electrice
Condensatori

10
-2
puțin probabil [g]
0.005
elec–>elec
Magneți superconductori

10
-3
puțin probabil [g]
(continuare pe pagina)
TWIDELL PAGINARE.indb 539
01/12/2014 11:38
Magazin
Densitatea de energie
De operare
Probabil comerciale
Valoarea sa de utilizare
Conversie
Eficiența
temperatura
timpul de dezvoltare
tip
--------
-------
-------
----------------
-----------
--------
MJ kg
-1
MJ L
-1
°C
ani
MJ $NE
-1
%
Baterii (în practică)
[c]
plumb–acid
0.15
0.29
ambient
în uz
0.02
elec –>elec
80
NiCd
0.2
ambient
în uz
0.01
elec –>elec
75
ZnBr
0.3
ambient
5
0.02
elec –>elec
70
Li–ion
0.5
0.8
ambient
în uz
0.04
elec –>elec
75
Fluxul de baterii
0.1
0.1
ambient
În uz
0.02
elec–>elec
80
Mecanice
Pompat hidro
0.001
0.001
ambient
în uz
0.2–20
elec –>elec
80
Volanta (disc de oțel)
0.05
0.4
ambient
în uz
0.04 [c]
elec –>elec
80
Volanta (compozit)
0.05
0.15
ambient
în uz
0.02 [c]
elec –>elec
80
Aer comprimat
0.2–2
5 [f]
20-1000
în uz
1 [c] [h]
elec –>elec
50
Note
[o] Aceste cifre sunt pentru "tipic" operațiune și numai prin ordin de mărime aproximări pentru o anumită aplicație.
Acest lucru este valabil mai ales pentru cele legate de aplicații comerciale și a costurilor. Aici costurile sunt aproximative prețuri en-gros
înainte de impozitare.
Tabelul adaptate și actualizate pentru această ediție de Jensen și Sorensen (1984).
[b] Energie tranzitată pe unitatea de cost.
[c] capacitate de Energie pe unitatea de cost. Dar, rețineți că (de exemplu) dacă un acumulator cicluri de 10 ori la 50% de descărcare de
gestiune, sa
transfer
pe unitate de cost ar fi (0.02 MJ/$NE)(1000)(50%)
= 10 MJ/$ deoarece acesta poate fi utilizat în mod repetat.
[d] La 150 de atmosfere presiune.
[e] Depinde de timp și a pierderilor de căldură.
[f] La 50 de atmosfere presiune.
[g] puțin Probabil pentru
pe scară largă
de stocare a energiei, deși în uz, în 2012, pentru evitarea rapidă de tensiune de-a schimba tranzitorii privind alimentarea cu energie
electrică, și anume îmbogățirea'.
[h] în cazul în Care potrivit scară largă cavități naturale există, de exemplu, minele abandonate.
TWIDELL PAGINARE.indb 540
01/12/2014 11:38
§15.6 stocare a Energiei pentru rețeaua de energie electrică
541
masa este destinat în principal pentru a indica contrastante ordine de
mărime de densitatea de energie în diverse magazine de energie, mai degrabă decât de precis
datele.
§15.6 STOCARE A ENERGIEI PENTRU REȚEAUA DE ENERGIE ELECTRICĂ
Considerăm de stocare a energiei opțiuni de aici, care sunt predominant pentru
integrare cu retelele de transport. Baterii și celule de combustibil sunt relevante, dar
sunt în principal pentru uz local, astfel încât sunt considerate separat în §15.7 și §15.8
respectiv.
§15.6.1 Pompat hidro
Un pompat hidro sistemul utilizează două rezervoare, un sus și un jos.
Atunci când suficient de energie electrică este disponibil și nu este necesară de altfel,
apa este pompată în sus. Atunci când cererea se produce, apa este lăsată
să cadă din nou, de conducere o turbină hidroelectrică de la partea de jos și, astfel,
generatoare de putere (a se vedea punctul 6.7). Energia potențială înmagazinată într-un baraj la
100 m cap are o densitate de energie W
v
= 1.0 MJ/m
3

(a se vedea Problema 15.1).


Deși aceasta este o relativ mică densitate de energie, energia totală stocată
într-un baraj hidro poate fi încă foarte mare.
Unele foarte mari sisteme de acest tip neted fluctuația cererii
pe centralelor convenționale, permițându-le pentru a rula la sarcină constantă și
o mai mare eficiență globală. Centralele nucleare mai ales au nevoie de
un astfel de sprijin. Din moment ce aproximativ 15% din puterea de intrare păstrează turbine/
pompe de filare pentru a permite un răspuns rapid și de o suplimentare de 15% este pierdut
în frecare și de distribuție, se poate argumenta că mari de capital, costul de
astfel de scheme ar fi fost mai bine cheltuite pe controlul cererii (a se vedea
§1.5.3 și Capitolul 16).
§15.6.2 Volante
Energia cinetică a unui obiect de rotație este:
w
=
E
Am
1
2
2
(15.5)
unde I este momentul de inerție al obiectului în jurul axei sale, și
w este
viteza unghiulară (rad/s). În cel mai simplu caz, de masă m este concentrată
într-o margine de rază o, așa că am
= d
2
. Cu toate acestea, pentru o uniformă disc de aceeași
masă, nu este mai puțin (ma
2
/2) deoarece masa mai aproape de axul contribuie
mai puțin de inerție decât la rim.
Prin urmare, din (15.5) densitatea de energie de o uniformă de disc devine:
w
=
=
W
Em
o
/
1
4
m
22
(15.6)
Pentru un volant de a fi util un depozit de energie și nu doar o netezire
dispozitiv, rezultă din (15.6) că trebuie să se rotească cât mai repede posibil.
TWIDELL PAGINARE.indb 541
01/12/2014 11:38
542
De integrare, de distribuție și de depozitare
Cu toate acestea, viteza unghiulară este limitată de rezistența materialelor
rezistente la forțele centrifuge care tind să-l arunce afară. Pentru o uniformă
roata de densitate
r, maximă de tracțiune este:
s
rw
=
o
max
22
(15.7)
În general, am
= Kma
2
/2 pentru o anumită formă solidă, unde K este o constantă ~1.
Deci:
w
=
W
Ka
/2
m
22
(15.8)
și:
s
r
=
W
K
2
m
max
max
(15.9)
Materialele convenționale, cum ar fi oțel, au relativ mic de energie
densități.
A LUCRAT EXEMPLU 15.2 MAXIMĂ DENSITATE DE ENERGIE DE ROTAȚIE DISC DE OȚEL
Pentru o destul de puternic, din oțel, (15.9) vă oferă, cu K
= 1,
=
×
=



W
(1000 10 Nm )
(2)(7800 kg.m )
0.06 MJm
m
max
6
2
3
3
Mult mai mare densitatea de energie pot fi obținute prin utilizarea ușor
de fibre materiale compozite, precum fibra de sticla în rășină epoxidică, care au o
mai mare rezistență la tracțiune
s
max
și mai mică densitate
r. Pentru a face cea mai bună utilizare
a acestor materiale, volante ar trebui să fie făcut în forme neconvenționale
cu puternic fibrele aliniate în direcția de stres maxim. Astfel de
sisteme pot fi densitatea de energie de 0,5 MJ/kg (mai mult de plumb-acid
baterii) sau chiar mai mare (Problema 15.3).
Pentru utilizarea în netezirea cererii în mari rețele de energie electrică, volante
au avantajul peste pompat hidro sisteme care pot fi
instalate oriunde și ia puțin suprafață de teren. Unități cu o 100 de tone
volanta ar avea o capacitate de stocare de aproximativ 10 MWh. De stocare mai mare
cere, probabil, ar fi cel mai bine îndeplinite de către cascadă multe astfel de modulare
"mici" de unități. Volante oferă, de asemenea, un teoretice, dar nu din punct de vedere comercial
utilizat, alternativă la depozitarea baterii pentru utilizare în electric alimentat
de vehicule, mai ales că energia într-un volant poate fi alimentată
mai repede decât într-o baterie (vezi Problema 15.2).
§15.6.3 aer Comprimat
Aerul poate fi rapid comprimat și încet-încet s-a extins, iar acest lucru oferă
netezire pentru mari fluctuații de presiune în sistemele hidraulice. La
TWIDELL PAGINARE.indb 542
01/12/2014 11:38
§15.6 stocare a Energiei pentru rețeaua de energie electrică
543
densitatea de energie disponibile sunt moderat de mari. O mică scară exemplu
este berbec hidraulic pompa menționate în Bibliografia de la Capitolul 6.
Dacă potrivită mare cavitate pentru stocarea aerului comprimat este disponibil,
de aer comprimat poate fi utilizat pentru a stoca energie pe o scară util pentru
a utilităților electrice (Fig. 15.4). De exemplu, un 110 MW sistem în Alabama
(SUA) folosește o singură peșteră de sare de 560.000 de m
3

, proiectat pentru a funcționa


între 45 și 74 de bar, și poate furniza acestuia o putere nominală de 26 de ore.
Energia este recuperată prin utilizarea aerului comprimat într-o modificat de gaz
cu turbină, care are o eficiență mai mare decât în mod normal pentru că alimentarea cu aer
este pre-comprimat.
Fig. 15.4
Aer comprimat de stocare a energiei și sistem de recuperare: schematic pe o scală de utilitate de sistem.
Sursa: © Robert Rohatensky (2007), reprodus sub un Design Science License din http://www.
energytower.org/cawegs.html.
Vânt puternic de aer
, compresoare
Naturale sau biogaz
Aer comprimat de stocare
Aer
Aer
Căldură
de recuperare
Generator
Combinate
ciclu de
gaz
cu turbină
Aer comprimat și
combustibil sunt combinate
și pre-încălzit
EXEMPLUL 15.3 DENSITATE DE ENERGIE DE AER COMPRIMAT
Ia în considerare compresie lent de V
1
= 50 m
3
de aer, la o presiune p
1
= 1.0 atmos = 1.0 x 10
5
Nm
-2
, pentru a p
2
=
50 atmos, la temperatură constantă. Pentru n moli de aer, considerat ca un gaz perfect:
=
pV
nR de T
0
(15.10)
din care rezultă că V
2
= V
1
(p
2
/p
1
)
= 1.0 m
3
și activitatea desfășurată (energie stocată) este:
E
pV
nR de T
V
V
pV
VV
d
d
log ( / )
19 MJ
V
V
V
V
e
1
2
0
1
2
11
1
2



=−
=−
=
=
(15.11)
Prin urmare, în stare comprimată, W
v
= E/V
2
= 19 MJ/m
3
.
TWIDELL PAGINARE.indb 543
01/12/2014 11:38
544
De integrare, de distribuție și de depozitare
Pentru sisteme de operare în mai puțin idealizat condiții energia stocată
pe unitatea de volum, W
v
va fi mai puțin, dar de o magnitudine similară. Un important
dificultatea este de a reduce pierderea de energie la producerea de energie termică în timpul
de compresie. Kreith și Kreider (2011), da mai detaliată
descriere a inginerie aspecte de astfel de sisteme.
§15.6.4 de Mare putere electrică de stocare
Un supraconductoare electromagnetice de stocare a energiei (IMM-uri) sistemul este un
dispozitiv pentru stocarea și foarte repede evacuează cantități mari de
energie electrică (de exemplu, 10 MW în ~1 s). Acesta stochează energie în câmpul magnetic
creat de fluxul de curent continuu într-o bobină supraconductoare material care a
fost răcite criogenic la ~4 K. La aceste temperaturi foarte scăzute,
anumite materiale au, în esență, rezistenta la zero la curent electric și
poate menține un curent continuu de ani de zile, fără o pierdere semnificativă. IMM-uri
sistemele au fost în folosință timp de câțiva ani pentru a îmbunătăți puterea industrială
calitate și pentru a oferi o calitate premium de servicii pentru cei energie electrică
utilizatorii care sunt deosebit de vulnerabile la fluctuațiile de tensiune. Un IMM
reîncarcă în câteva minute și se poate repeta ciclu de încărcare/descărcare
de mii de ori fără nici o degradare a magnetului. Deși
au existat propuneri de a utiliza mai multe IMM-uri în general pentru stocarea de
cantități mari de energie electrică, costul pare a fi prohibitive
(a se vedea Tabelul 15.2).
Alte mari sisteme cu răspuns rapid sunt dezvoltate pentru similare
putere condiționat foloseste. În special, condensatori electrochimici (ECs)
a stoca energie electrică în cele două serii condensatori care există în electric
cu dublu strat (EDL) la interfața de fiecare electrod și electrolit
soluție. Distanța pe care se produce o separare este la doar
câțiva angstromi. Capacitatea și densitatea de energie a acestor dispozitive
sunt de mii de ori mai mare decât pentru "standard" condensatori electrolitici.
În comparație cu bateriile plumb-acid, ECs au mai puțin de densitate de energie, dar ele
pot fi pedalat zeci de mii de ori și sunt mult mai funcțional
decât bateriile (rapid de încărcare și descărcare capacitate). În timp ce mici
condensatori electrochimici sunt destul de mature tehnologie, produse cu
mai mare densitatea de energie sunt încă în curs de dezvoltare.
§15.7 BATERII
Energia electrică este o înaltă calitate, formă de energie, și, prin urmare, un mare efort a
fost făcut pentru a găsi un ieftine și eficiente mijloace pentru stocarea acestuia. Un dispozitiv
care are energie electrică, atât ca intrare și de ieșire se numește o (electrice) de stocare
a bateriei sau – ocazional – electrice 'acumulator. Bateriile sunt o
componentă esențială de cele mai multe sisteme de alimentare autonome (mai ales
cu fotovoltaice și turbine eoliene mici generație), în modul de așteptare și
sisteme de alimentare de urgență, și de vehicule electrice.
TWIDELL PAGINARE.indb 544
01/12/2014 11:38
§15.7 Baterii
545
§15.7.1 plumb-acid baterie
Deși multe reacții electrochimice sunt reversibile în teorie, puțini
sunt potrivite pentru un spațiu de depozitare practic baterie, care vor fi necesare pentru ciclul
de sute de ori între încărcarea și descărcarea curenților de la 1 la
100 sau mai mult. Cele mai utilizate pe scară largă baterie de stocare este de plumb-acid
baterie, inventat de Planté în 1860 și s-a dezvoltat continuu de atunci.
Astfel, bateria este construit din celule, dintre care unul este prezentat
schematic în Fig. 15.5. La fel ca în toate celulele electrochimice, există două electrod
'farfurii' cufundat într-o efectuarea de soluție (electrolit). În acest caz
electrozii sunt în formă de grile care deține paste de plumb și
dioxid respectiv; paste sunt realizate din pulberi de a crește
suprafața în 'spongios formular. Electrozi în formă de tuburi da un plus de
rezistență mecanică și de a rezista 'vărsare' (a se vedea mai târziu), și deci sunt potrivite
pentru a descărcării profunde. Electrolitul este acid sulfuric, care ionizează după
cum urmează:
H
H DECI
HSO
2
4
4

+
+

(15.12)
În timpul de descărcare de gestiune, reacția la electrodul negativ este:



Pb + HSO
PbSO + H + 2e
4
4
+
(15.13)
Spongios plumb (Pb) este oxidat la Pb
2+
, care este depus ca PbSO
4

cristale. Cea mai mică densitate de sulfat ia locul Pb inserați codul în


farfurie și, având în mare formă moleculară, cauzele mecanice de expansiune.
Fig. 15.5
Diagrama schematică de plumb-acid de celule. Purtatorii de sarcina se deplaseze în direcția indicată
în timpul de descărcare de gestiune reacții de (15.13) și (15.14). Reacțiile și purtător de
mișcări sunt inversate în timpul încărcării (deschide întrerupătorul S1 și S2 închis).
Taxa
e

e

De descărcare de gestiune
Electrolit
Pb
+

PbO
2
S
2
S
1
H
2
O
H
2
DECI
4
HSO
4

H
+
Am
Am
TWIDELL PAGINARE.indb 545
01/12/2014 11:38
546
De integrare, de distribuție și de depozitare
A LUCRAT EXEMPLU 15.4 TEORETICE DE DENSITATE DE ENERGIE DE PLUMB-ACID BATERIE
Reacțiile (15.12) și (15.13) arată că, pentru a transfera 2 mol de electroni necesită:
1 mol Pb
= 207 g
1 mol PbO
2
= 239 g
2 mol H
2
DECI
4

= 196 g
Total active materiale 642 g
Dar 2 mol de electroni reprezintă o taxă (unitatea de Coulomb):
(2 mol)(-1.60 x 10
-19
C)(6.02 x 10
23
mol
-1
)
= –(2) (9.6 x 10
4
)C
= -1.93 x 10
5
Coulomb
Potențialul de electrod, în condiții standard de concentrare, pentru (Pb/PbSO
4
) este de 0,30 V și pentru
(PbSO
4
/Pb
4
+
) este -1.62 V. Deci teoretic mobil-CEM în condiții standard pentru (Pb/PbSO
4
/H
2
DECI
4
/PbSO
4
/
PbO
2
) este
x
celula
= +1.92 V, cu PbO
2
placa pozitivă, conform IUPAC semneze convenția.
Efective de celule EMF depinde de concentrația de reactivi, și poate fi calculat de standard
metode electrochimice. În general, cu circuit deschis de tensiune a unei celule diferă cu numai câteva procente din
teoretică celule tensiune (Fig. 15.6). În special, baterii plumb-acid produce un circuit deschis potențial
diferența de 2.0 V per celula. Dacă rezistența internă a celulei este mult mai mică decât cea din sarcină externă
(ca poate fi de așteptat cu un nou sau " bun " de celule), atunci diferența de potențial la bornele va fi
aproape de valoarea circuitului deschis.
Prin urmare, activitatea desfășurată în mișcare 2 mol de electroni:
(1.93 x 10
5
C) (2.0 V)
= 0.386 x 10
6
J
Astfel, energia stocată într-1.0 kg de ingrediente active este, în teorie,
W
m
(0)
= (0.386 x 10
6
J)/(0.642 kg)
= 0.60 MJ/kg.
Electronii eliberat de călătorie prin circuitul exterior la
electrod pozitiv, în cazul în care acestea contribuie la reacția:



PbO + HSO + 3H + 2 e
PbSO + 2H O
2
4
+
4
2
(15.14)
Astfel PbSO
4
înlocuiește PbO
2

în farfurie, cu similar, dar mai puțin


perturbator, efecte mecanice decât în placa negativă. Electrice
de curent prin soluția este efectuată de către Sec
+
și HSO
4

ionii de
acid sulfuric electrolit, care se ia parte la placa de
reacții. Transportabile 'gel celule au acest electrolit imobilizat în
pirogene siliciu, cu fibros mat de sticlă separator oferindu-gaze deschis
căi pentru eliberarea de hidrogen și oxigen în suprapreț. Deși
acest lucru le face relativ scumpe, sunt mai sigur de a utiliza și
de transport, deoarece nu există nici un pericol de vărsarea extrem de coroziv acid sulfuric,
și acestea sunt de 'intretinere'.
Știind reacțiile implicate și standard corespunzătoare
potențiale de electrod (având în chimice, tabele), teoretice de densitate de energie
de orice propus bateria poate fi calculată.
TWIDELL PAGINARE.indb 546
01/12/2014 11:38
§15.7 Baterii
547
Din păcate, densitatea de energie W
m
de orice practică bateriei este întotdeauna
mult mai puțin decât valoarea teoretică W
m
(0)

bazat pe active totale


de masă, conform explicațiilor de mai jos pentru baterii plumb-acid. Prin urmare, cel mai
comercial baterii W
m
~0.15 W
m
(0)

deși mai atent (și mai


scumpe!) modele pot fi în mod rezonabil de așteptat pentru a obține energie
densități de până la 25% din valorile teoretice.
În cazul specific al plumb-acid baterie, principalele motive pentru
'slabe' sunt după cum urmează:
1
O baterie de lucru conține în mod obligatoriu și non-active materiale (de exemplu,
cazul), separatoare (care împiedică electrozi de
shortcircuiting) și apa în care acidul este dizolvat. Mai mult decât atât,
concentrația de acid nu trebuie să fie prea mare, deoarece bateria ar apoi
de descărcare de gestiune în sine. Deoarece masa reală a bateriei conținutul depășește
masa de ingrediente active, densitatea de energie pe baza
de masă a bateriei este mai mică decât valoarea teoretică bazată pe
active masa singur. Cu toate acestea, acest factor nu este de mare importanță
pentru baterii stationare.
2
Reacțiile nu poate fi permis să meargă la finalizare. Dacă toate cele de plumb
au fost consumate de reacție (15.13) nu ar fi nici electrod stânga
pentru reacția inversă să funcționeze, adică bateria nu a putut fi pedalat.
În mod similar, în cazul în care concentrația de H
2
DECI
4

este permis de a reduce prea mult,


electrolit încetează să mai fie un nivel adecvat de dirijor. În practică, multe
tipuri de baterii nu ar trebui să fie permis să se descarce mai mult de
50% din totalul potențiale stocate de energie, sau ele pot fi distruse. Cu toate acestea,
special concepute baterii permite descărcare de gestiune profundă' dincolo de 50%.
O limitare suplimentară de real baterii este faptul că acestea nu vor dura pentru totdeauna. Solid
Pb este aproape de două ori la fel de dens ca PbSO
4
se găsește în reacție de descărcare de gestiune
(15.13). Prin urmare, este dificil pentru a se potrivi PbSO
4

cristale în spațiul
ocupat inițial de Pb inserați codul în electrodul negativ. După mai multe
cicluri de încărcare/descărcare, repetate de expansiune și contracție cauza
material placă și unele PbSO
4

să cadă în partea de jos a celulei. Acest lucru


constituie o pierdere ireversibilă de material activ. Această pierdere este mai gravă dacă bateria
este permis să-și îndeplinească pe deplin; într-adevăr, poate deveni rapid imposibil
să reîncărcați bateria. În plus, 'magazie pentru material poate oferi un
conductor electric legătură între plăci, astfel încât creșterea de auto-descărcare de gestiune'.
Depozitare baterii ar trebui să aibă un spatiu generos sub plăci, astfel încât
resturile pot acumula fără scurtcircuitarea electrozilor.
Alte principalul factor care limitează viața chiar și un bine întreținute bateria
este auto-descărcare de gestiune de electrodul pozitiv. Acest lucru este deosebit de acută în
vehicul SLI (de pornire, iluminat și aprindere) baterii în care rețeaua nu este
pur Pb dar, de obicei, un plumb-antimoniu-calciu aliaj. Electrod plăci cu
antimoniu sunt fizic mai puternici și mai capabili să stea mecanice
subliniază în timpul mișcării. Din păcate antimoniu promovează reacție:

5PbO + 2Sb + 6H DECI
(SbO ) DECI + 5PbSO + 6H O
2
2
4
22
4
4
2
(15.15)
TWIDELL PAGINARE.indb 547
01/12/2014 11:38
548
De integrare, de distribuție și de depozitare
care, de asemenea, încet, dar ireversibil, elimină material activ de la
baterie. Astfel de baterii sunt proiectate pentru utilizarea cu autovehicule nu
, de obicei, funcționează bine în fotovoltaice și sisteme eoliene.
Baterii pentru aplicații staționare (de ex. sisteme fotovoltaice de iluminat)
se poate folosi Sb-gratuit plăci, și au o viață mai lungă (de obicei de cel puțin 8 ani și
poate la fel de mult ca 20 de ani), dar numai dacă perceput într-o manieră controlată
și dacă nu în mod excesiv și frecvent evacuate.
Performanța a unei baterii depinde de curent la care este
încărcat și descărcat, iar adâncimea la care este evacuate cu regularitate.
Fig. 15.6(a) prezinta caracteristici de descărcare de tipică plumb-acid mașină de
baterie. Capacitatea sa nominală este Q
20

= 100 Ah, care este taxa care


poate fi extras dacă este descărcat de la un curent constant de peste 20 de ore
(de obicei etichetate am
20
). Tensiunea pe celulă de o baterie nouă în această
descărcare de gestiune ar trebui să scadă doar ușor, de la 2.07 V la 1.97 V, după primele 60%
din Q
20
este descărcată. Această descărcare de gestiune elimină dens HSO
4

ionii de
soluție de electrolit, și le stochează ca solid PbSO
4

în electrozi de
către reacțiile (15.13) și (15.14), reducându-se astfel densitatea de
soluție de electrolit așa cum se arată în Fig. 15.6(c). Astfel densitatea de
'acid de baterie', măsurată cu un hidrometru, poate fi folosit ca o măsură
de starea de încărcare a bateriei. Dacă același bateria este descărcată
între aceleași tensiuni peste aproximativ o oră, tensiunea scade mult
mai brusc, iar suma totală care poate fi îndepărtată de la ea poate fi
doar aproximativ 0,5 Q
20

. Aceasta pentru că viteza de reacție dintre electrozi


este limitata de rata la care reactanții pot difuza în contact cu
fiecare alte. O acumulare rapidă de produse de reacție (PbSO
4
în special) poate
bloc de acest contact. Mai mult decât atât, rezistența internă peste acest PbSO
4
strat reduce tensiunea din celulă.
Un set de încărcare caracteristici pentru aceeași baterie este prezentată în
Fig. 15.6. (b). Pentru a începe încărcarea, un EMF de cel puțin 2.1 V pe celulă
este necesar. Tensiunea necesară inițial crește încet, dar crește
rapid la aproximativ 2,6 V per celula ca bateria se apropie de încărcare completă (dacă este constantă,
curentul de încărcare este menținută). Acest lucru este pentru că apa în celulă
începe să electrolyze.
Când celula este supraîncărcat, H
2

gazul va fi eliberat. Astfel 'bule'


pot beneficia baterie de amestecare electrolit și deci diminuarea baterie
de stratificare; într-adevăr, sofisticat controlere de incarcare aranja pentru ca acest lucru să
se întâmple periodic. Cu toate acestea, gaze în exces de presă de la
electroliza necesită electrolitului să fie completat cu apă distilată, și
emise H
2

poate produce un amestec exploziv cu aerul și așa trebuie să


fie canalizat departe. Sigilat baterii – uneori vândute ca
'maintenancefree' – au un catalizator în partea de sus a bateriei de peste care electroliza
hidrogen poate combina cu oxigenul pentru a reforma apă în baterie
carcasa, astfel încât 'completarea' electrolitului cu apa distilata nu este
necesar. Extreme supraîncărcarea poate provoca daune mecanice în
celulă și poate crește concentrația de acid la punctul în care ionii
TWIDELL PAGINARE.indb 548
01/12/2014 11:38
§15.7 Baterii
549
Fig. 15.6
Caracteristicile de funcționare ale tipică plumb-acid baterie (SLI tip de aproximativ 100 Ah nominal
capacitate).
o
De descărcare de gestiune. Curbele sunt etichetate de descărcare de gestiune curent (presupus constant) și de
timpul necesar pentru a 'complet' de descărcare de gestiune la curent.
b
Taxa. Curba este pentru încărcare la o constantă de curent redus.
c
Densitatea electrolitului în funcție de starea de încărcare.
Sursa: După Crompton (2000).
1240
1200
1160
1120
0
20
11.5 V
12.0 V
12.5 V
40
Starea de încărcare/%
Densitate/(k
g/m
3
)
60
80
100
(c)
0
1.6
Circuit deschis v
oltage pe mobil 2.0
2.4
(a)
20
0.15 sec
5O
15 O
50 O
Am
= 200 O
Am
=7O
1h
5 sec
20 sec
40
Nominală energie stocată folosit/%
Energia stocată/%
60
80
100
0
1.6
2.0
2.4
(b)
20
40
60
80
100
nu sunt suficient de mobile pentru a permite bateriei să lucreze. Multe cicluri de ușoară
de încărcare și descărcare (de exemplu, ca și în mic sistemelor fotovoltaice)
provoca mari PbSO
4

cristale pentru a dezvolta în plăci și în mod eficient


a elimina active materiale, precum și provocând daune mecanice. În astfel de
condiții, ocazional adânc-descărcare poate reactiva a bateriei.
General lecție este că de încărcare/descărcare de gestiune de control este esențială pentru
viata lunga a bateriei; cel puțin încărcare la tensiune constantă și în cel mai bun
, având un sofisticat controller permite ocazionale de-stratificare
barbotare, controlat de încărcare și descărcare a curenților, tensiunii cut-off-uri
și, poate, ocazional adânc-descărcare. O baterie buna are extrem de
mică impedanță internă (<0.1
Ω) și este capabil de a oferi mari cur-
chiriile la înaltă frecvență. Anii 'farad capacitate' este foarte mic, în ciuda
TWIDELL PAGINARE.indb 549
01/12/2014 11:38
550
De integrare, de distribuție și de depozitare
'capacitatea de încărcare fiind de mare, astfel încât să nu fie induși în eroare de către distincte două
sensuri ale cuvântului "capacitate".
Dezvoltarea îmbunătățit baterii plumb-acid continuă,
produce o varietate de modele cu performanțe optimizate pentru diferite
aplicații, în termeni de fiabilitate, de viață lungă, costuri, raport putere/greutate, etc.
Evoluții-cheie de-a lungul ultimelor decenii includ: polipropilenă
pentru inert, anti-scurgere incinte; 'absorbent glass mat' tehnologie pentru
placa separatoare; supapa reglementate baterii plumb-acid (sigilat pentru a preveni
pătrunderea aerului, dar care permite eliminarea excesului de gaz și având interne
reforinformații de suprapreț electroliza hidrogen și oxigen); - o gama larga
de retete cu concentrație mică de aditivi pentru plăci de specialitate și
separatoare; și controlat electronic de încărcare.
Pentru aplicații care necesită o foarte mare capacitate (de exemplu, ca parte dintr-o
rețea de energie electrică), baterii plumb-acid pot fi scalate în sus în formă de celule de debit
(Cutie 15.6).
CUTIE DE 15.6 SCALARE BATERII: FLUXUL DE CELULE
Fluxul de baterii folosi o geometrie diferită de bateriile convenționale pentru a permite scară-up utility
nivel. Fluxul de celule baterie de stocare utilizează două soluții chimice pentru a stoca energie electrică (Fig. 15.7) substanțe
chimice
au loc în tancurile adiacente și apoi pompat atunci când este necesar, printr-o electrolitice "baterie" de celule. Astfel,
volumul de electrolit, și, prin urmare, energia stocată, sunt determinate separat de construcția celulară
caracteristici. Energia electrică este produsă și cu un convențional tip de baterie. Acumulatorul este încărcat într-un
proces invers.
Fluxul de celule din punct de vedere tehnic sunt foarte potrivite pentru instalații statice stocarea de cantități semnificative de
electrice
de energie cu răspuns rapid, ca și în rețelele naționale de electricitate.
De exemplu, Notrees matrice de celule de debit în Texas (SUA), instalat în 2012, se poate livra 36 megawatt
de putere de la rețea pe o perioadă de 15 minute. Acesta cuprinde autobuz de dimensiuni, de plumb-acid baterie module cu
suprafață mare de electrozi și mai multe terminale, astfel încât fluxurile de energie electrică în și rapid, și este
folosit pentru a netezi furnizarea de energie electrică din 153 MW Notrees eolian în apropiere. Ea face, de asemenea,
întreaga rețea mai rezistente la piroane în cerere, deoarece bateria matrice poate răspunde aproape instantaneu,
întrucât gaze naturale a centralelor dura aproximativ 15 minute pentru a stimula producția lor.
Electrolit alte combinații decât plumb-acid sunt subiectul activ fluxul de celule de dezvoltare, în special cu
Vanadiu redox flow celule și zinc, brom fluxul de celule.
Sursa: H. Hodson, 'Texas mega-baterie are ca scop verde grila', New Scientist, februarie 1, 2013.
15.7.2 de Litiu pe baza de baterii
Dezvoltarea altor sisteme electrochimice de stocare a bateriilor este
de asemenea activ. Factorul major care contribuie la relativ mic de energie
densitatea de plumb-acid baterie este greutatea atomică mare de plumb (207). Acest lucru
a condus activ dezvoltarea bateriilor pe baza de elemente mai ușoare,
în special litiu (Li, atomic greutate 7), în special pentru aplicații în cazul în care
greutatea este mai mare constrângere decât costul, în special pentru vehiculele electrice
(a se vedea §16.5.5(b)).
Moderne pe bază de litiu-bateriile nu utilizați litiu metal ca atare, deoarece
se aprinde ușor și reacționează violent cu apa. În schimb, litiu este
TWIDELL PAGINARE.indb 550
01/12/2014 11:38
§15.7 Baterii
551
combinat cu alte elemente în mai benigne compuși care
nu reacționează cu apa. Tipic litiu-ion cu celule folosește de carbon pentru anod
și un compus, cum ar fi litiu, cobalt sau dioxid de litiu fosfat de fier
ca și catod. Ionii de litiu sunt intercalate în electrodul pozitiv în
evacuate de stat și în electrodul negativ în stare încărcată,
și pentru a muta de la unul la altul prin electrolit. Electrolitul este
de obicei bazat pe o sare de litiu într-un solvent organic.
Volumul de producție s-a redus prețul, astfel încât reincarcabile cu litiu
baterii domina portabil pentru echipamente electronice de larg consum (de exemplu,
telefoane mobile și computere portabile). Până în 2012, aceste progrese în
densitate de energie și a prețului a condus la aproape toate vehiculele electrice (Ve) folosind
baterii Li-ion. Cu toate acestea, dezvoltarea activă de variante continuă ca
prețul de baterii trebuie să scadă în continuare, și lor utile
de viață crește, înainte de Sev concura pentru distanțe de călătorie cu dimensiuni similare
vehiculele convenționale.
Cu toate acestea, Li este un destul de rare element, și bateriile chiar și pentru un singur
EV, evident, nevoie de mult mai mult decât Li fac cei de sute de laptop
calculatoare. Prin urmare, există pericolul ca utilizarea pe scară largă a Sev folosind
Li pe baza de baterii pot fi resurse și, prin urmare, prețul de limitat. Acest
factor este un stimulent important pentru dezvoltarea altor roman baterie
tipuri.
§15.7.3 Alte tehnologii de baterii
Numeroase alte tipuri de acumulatoare, au fost și sunt în curs de
dezvoltate pentru aplicatii speciale; unele sunt enumerate în Tabelul 15.2 a
§15.5. Pentru mai multe detalii, a se vedea, de exemplu www.electropaedia.com. Cercetare
Fig. 15.7
Fluxul de baterie (schematică).
Sursa: http://www.mpoweruk.com/flow.htm (care link-uri către www.electropaedia.com).
Schimb de ioni
membrana
Putere
Electrod
Electrod
Electrolit
fluxul
Electrolit Tancuri
+

TWIDELL PAGINARE.indb 551
01/12/2014 11:38
552
De integrare, de distribuție și de depozitare
continuă, de asemenea, pe fotochimice celule (c.f. §9.7), parțial, în speranța
că acestea ar putea fi capabil de a stoca utile cantitățile de energie electrică generată
în mod direct de energia solară.
§15.8 CELULE DE COMBUSTIBIL
O celulă de combustibil care transformă energia chimică a unui combustibil în energie electrică direct,
fără intermediar ciclu de combustie. Deoarece nu există nici un
intermediar căldura de conversie, eficiența de celule de combustibil nu este limitată
de cea de-a doua lege a termodinamicii, spre deosebire de convenționale de combustibil

căldură
→locul de muncă→energie electrică a sistemelor. Eficiența de conversie din
energie chimică în energie electrică de către o celulă de combustibil poate fi, teoretic, 100%.
Deși nu strict "stocare" dispozitive, celule de combustibil sunt tratate în acest
capitol din cauza lor multe similitudini cu baterii, și posibila lor
utilizare cu H
2

magazine (§15.9). Într-o economie de hidrogen', celule de combustibil sunt utilizate


atât pentru staționare de energie electrică și pentru alimentarea electrică
a vehiculelor (§16.5.5(b)). Prin urmare, vom discuta numai celule de combustibil, folosind H
2
, deși
alte tipuri există.
Ca o baterie, o celula de combustibil este format din doi electrozi separați de un
electrolit, care transmite ioni dar nu electronii. În celule de combustibil
de hidrogen (sau un alt agent de reducere) este furnizat la electrodul negativ
și oxigen (sau aer) la electrodul pozitiv (Fig. 15.8). Un catalizator pe
poroase anod determină moleculele de hidrogen de a disocia în
ioni de hidrogen și de electroni. H
+

ionii migrează prin electrolit,


de obicei, un acid, la catod, unde reacționează cu electroni, furnizate
prin circuitul exterior, și de oxigen pentru a forma apa.
Fig. 15.8
Diagrama schematică a unei celule de combustibil. Hidrogenul și oxigenul sunt combinate pentru a da de apă și
energie electrică. Poros electrozi permite ioni de hidrogen pentru a trece.
Electrolit
2H
+
2H
+
2e

2e

Sarcina
Am
(–)
(+)
H
2
O
H
2
O
2
O
2
+
+
+
2
1
TWIDELL PAGINARE.indb 552
01/12/2014 11:38
§15.9 substanțe Chimice ca magazine de energie
553
Eficiența practică celule de combustibil, dacă hidrogen/oxigen sau
alte gaze "cuplu", sunt mult mai puțin decât teoretic de 100%,
pentru aceleași motive ca și pentru baterii. În practică, eficiența
este, probabil, 40% pentru conversia energiei chimice în energie electrică, dar
asta nu depinde de faptul dacă este sau nu celule este de lucru la întregul său nominală
putere. Acest lucru contrastează cu cele mai multe motoare diesel, turbine cu gaz și alte
motoare.
Deoarece eficiența de un ansamblu de celule de combustibil este aproape egală cu
cea a unei singure celule, există câteva economii de scară largă. Prin urmare,
localizate mici plante de la 1 la 100 kW capacitate sunt promițătoare, o
propunere. Folosind celule de combustibil ca un combinată de energie termică și de sursa de energie, o singură
clădire ar putea fi alimentate cu energie electrică și căldură (de pierdere
de căldură de celule), pentru aceeași cantitate de combustibil este necesara în mod normal pentru
energie termică singur. Principalul motiv pentru care celulele de combustibil nu sunt în larg
de utilizare pentru astfel de aplicații este capitalul lor costa ~$700/ kW.
§15.9 SUBSTANȚE CHIMICE CA MAGAZINE DE ENERGIE
$15.9.1 Hidrogen
Hidrogenul poate fi făcut din apă prin electroliză, folosind orice sursă de
curent continuu de electricitate. Gaz poate fi stocat, distribuit și arse pentru a elibera
căldură. Singurul produs de ardere este apă, așa că la final nu
a poluării rezultate. Entalpia schimbare este
DH = -242 kJ/mol; adică 242 kJ sunt
lansat pentru fiecare alunita (18 g) H
2
O formează.
Hidrogen (cu CO în formă de oraș gaze' a făcut din cărbune) a fost
folosit de mai mulți ani ca un magazin cu energie și de aprovizionare, și nu există nici
imperative tehnice motiv de hidrogen pe bază de sisteme nu ar putea veni
în utilizarea pe scară largă din nou. Rețineți, totuși, că cele mai multe hidrogen este format acum
din combustibili fosili.
Electroliza este un bine-stabilit comerciale procesului productiv pur
de hidrogen, dar, în general, eficiență au fost doar
De ~60%. Unele dintre aceste
pierderi se datorează rezistenței electrice în circuit, în special în jurul
electrozilor unde evoluează bule de gaz a blocului curent
care transportă ioni în apă. Electrozi cu bule-eliminarea mecanisme
ar trebui să fie avantajoasă. Cel mai bun electrozi au porozitate mare, astfel încât
să dea o mai mare zona de eficient și, astfel, permite o mai mare densitate de curent,
care implică a avea mai puține celule și cost redus pentru un anumit gaz de ieșire.
Eficiență
~80% au fost astfel obținute, și poate fi crescută în continuare
prin utilizarea – de obicei scumpe – catalizatori.
O dificultate tehnică în electroliza apei de mare este că clor
poate fi, de asemenea, a evoluat de la 'oxigen' electrod. Aproximativ chimice
calculele sugerează că O
2

poate fi păstrat curat dacă tensiunea aplicată


pe mobil este la mai puțin de 1.8 V, dar din păcate aceste limite, densitatea de curent,
deci electrozi de suprafață mare ar fi necesare.
TWIDELL PAGINARE.indb 553
01/12/2014 11:38
554
De integrare, de distribuție și de depozitare
Temperaturi ridicate, de asemenea, promovează descompunerea chimică a apei.
Modificarea energiei libere Gibbs asociat cu un reversibile
reacție electrochimică la temperatura absolută T este:
x
D=
=D−D
G nF
HTS
(15.16)
în cazul în care
x este potențialul electric, DH este entalpia schimba și DS este
entropia schimba, F
= 96,500 Coulomb mol
-1
este a lui Faraday constanta,
și n este numărul de moli de reactant.
Reacția de descompunere
H
2
O
→H
2
+
1
2
O
2
(15.17)
a
DG, D,H, DS . toate pozitive. Prin urmare, din (15.16), creșterea T
scade potențialul electric
x necesar pentru descompunere. Problema
15.10 arată că
x = 0 pentru T >~2000 K, astfel încât este imposibil să
se descompună apa numai prin simplă încălzire. O strategie mai promițătoare
este de a reduce de intrare a energiei electrice necesare pentru electroliza prin încălzire
la o sursa mai ieftina. Căldură la T ~1000 K de la concentratoare solare poate
fi mai ieftin decât utilizarea de energie electrică, iar acest lucru poate fi cea mai ieftină cale
de a produce hidrogen.
Mai multe alte metode de producere a hidrogenului, fără a utiliza combustibili fosili
combustibili au fost încercate în laborator, inclusiv alge, care
'fotosinteză' H
2
(a se vedea §9.7.3); dar nici unul nu a demonstrat încă util
eficiență.
CUTIE 15.7 O MICĂ INSULĂ AUTONOMĂ VÂNTUL-ENERGIE PE BAZĂ DE HIDROGEN SISTEM
Un autonome de vânt/de hidrogen energie demonstrație sistemul situat pe insula Utsira, Norvegia,
a fost lansat oficial de către Norsk Hydro (acum Statoil) și Enercon (un German producător de turbine eoliene)
în iulie 2004. Principalele componente ale sistemului sunt 600 kW
e
evaluată de turbine eoliene, o apă electrolizor
pentru a produce aproximativ 10 Nm
3
/h de hidrogen, cu aproximativ 2400 Nm
3
de stocare a hidrogenului (la 20.000 kPa), un
hidrogen motor cu ardere internă de conducere 55 kW
e
generator, și la 10 kW
e

proton exchange
membrane (PEM) de celule de combustibil. Acest lucru inovatoare demonstrație sistem consumabile 10 gospodării de pe insula,
oferind două-trei zile de deplină autonomie energetică (Ulleberg et al. 2010).
Experiență operațională și de datele colectate de la plante peste patru-cinci ani au arătat că
eficiența globală a vântului la AC-energie electrică pe bază de hidrogen pentru a AC-sistem de energie electrică, presupunând că
nici depozitare
pierderile, este de numai aproximativ 10%. Dacă hidrogen motorul a fost să fie înlocuit de un 50 kW
e

PEM fuel cell, la


eficiența globală ar crește de la 16 la 18%. Înlocuirea prezent electrolizor cu o mai eficientă
unitate (cum ar fi un PEM sau mai avansat alcaline design) ar crește eficiența generală a sistemului la
aproximativ 20% (Ulleberg et al. 2010).
Relativ scăzut de eficiență a sistemului ilustrează provocare pentru comerciale de hidrogen
evoluții. Mai compact hidrogen sisteme de stocare și mai robust și mai puțin costisitoare celule de combustibil trebuie
să fie elaborate înainte autonome vânt/sistemelor pe bază de hidrogen poate deveni din punct de vedere tehnic și economic
viabil.
Surse: SRREN (2011, §8.2.5.5); O. Ulleberg, T. Nakken și A. Ete (2010) "vântul/hidrogen demonstrație sistem
la Utsira în Norvegia: evaluarea performanței sistemului", Revista Internațională de Energie pe bază de Hidrogen, 35, 1841-1852.
TWIDELL PAGINARE.indb 554
01/12/2014 11:38
§15.10 Stocare pentru sistemele de încălzire și răcire
555
Pentru a stoca hidrogen în cantități mari nu este trivial. Cele mai promițătoare
este utilizarea de caverne subterane, cum ar fi cele de la care naturale
de gaze este acum extrase, dar de depozitare de gaze naturale, chiar dacă comprimat, este voluminos.
Hidrogenul poate fi lichefiat, dar din punctul său de fierbere este de 20 K (adică -253°C),
aceste depozite frigorifice sunt costisitoare pentru a construi și a opera, care necesită
a continuat refrigerare. Depozitarea substanțelor chimice ca hidrurile metalice, din care
hidrogenul poate fi eliberat de încălzire, este mai ușor de gestionat și vă permite
volume mari de H
2
pentru a fi stocate (a se vedea Tabelul 15.2). De exemplu,
FeTiH
1.7
T
~50°C
+
FeTiH
0.8 Sec
0.1
2
(15.18)
Această reacție este reversibilă, astfel încât un portabil hidrură de magazin poate fi
alimentată cu hidrogen la o central 'statie'. Căldura eliberată în
acest proces pot fi utilizate pentru termoficare, și portabil hidrură de
magazin poate fi folosit ca rezervor de combustibil' de un vehicul. Principala dificultate este
greutatea și costul de metale utilizate (a se vedea Tabelul 15.2). Hidrogenul poate
fi, de asemenea, distribuite prin intermediul extinse rețelele de conducte deja folosit
livra gaze naturale în mai multe țări, deși hidrogen transportă mai puțin
de energie pe unitatea de volum decât metanul.
Unii scriitori în vedere o economie de hidrogen în care hidrogenul
devine principalul mijloc de depozitare și distribuție a energiei. Dar
beneficiile de acest lucru sunt dubioase cu excepția cazului în care H
2

în sine este produs din RE, așa


că costul de noi infrastructuri majore necesare ar părea să fie
nejustificată.
§15.9.2 Amoniac
Spre deosebire de apă, amoniac poate fi disociată de la realizabilă temperaturi:
+

←a se
N
H
NH
3
2
2
2
3
(15.19)
În combinație cu un motor termic, aceste reacții constituie baza de
sisteme care poate fi cel mai eficient mod de a genera continuu de
energie electrică din energie solară termică (a se vedea §4.8.3).
Sisteme similare au fost de asemenea propuse bazează pe reacția

←a se
CO + 2H O
CH + 2O
2
2
4
2
(15.20)
utilizarea de energie electrică pentru a influența reacția la dreapta (de exemplu, atunci când perioadele de
generare a energiei eoliene depășește cererea). Acest lucru este uneori menționată
ca 'puterea de a gas' (P2G) de stocare.
§15.10 DEPOZITARE PENTRU ÎNCĂLZIRE ȘI RĂCIRE
SISTEME de
O fracțiune substanțială din lume, consumul de energie este scăzut, temperatura de căldură.
De exemplu, Fig. 16.3(b) prezintă cererea în marea Britanie pentru total de energie
pentru încălzirea spațiului. Deși detaliile schimba de la an la an, acest
TWIDELL PAGINARE.indb 555
01/12/2014 11:38
556
De integrare, de distribuție și de depozitare
sugerează că în timpul iernii, peste jumătate din consumul național de energie este
pentru spațiu de încălzire în clădiri, la temperaturi de aproximativ 18 ± 3°C. este
de obicei sensibil pentru a satisface această cerere de căldură de la cel mai bun
termodinamic calitatea aprovizionării cu energie (§1.4.2), deoarece acestea ar trebui să fie salvat
pentru generarea de energie electrică, motoare și unități cu motor. Astfel, de exemplu,
este mai bine pentru a capta caldura solara câștiguri, și apoi să păstreze clădirile în termen de
temperaturi confortabile, folosind o medie de căldură și de stocare a
caracteristicilor de masa clădirii (a se vedea §16.4). De stocare de căldură, de asemenea, oferă un
fel de fructuos folosind "deșeuri" de energie utilizată sau recuperate de la alte
procese (de ex. prin dispozitive de încărcare de control: §1.5.3).
La latitudini mai mari, caldura solara de aprovizionare este semnificativ mai mare în
timpul verii decât în timpul iernii (vezi fig 2.7 și 2.10), dar cererea de căldură
este mai mare în timpul iernii. Prin urmare, beneficiul maxim de căldură solară
necesită stocare de căldură pentru cel puțin trei luni, să zicem, în apă fierbinte într-un
subteran cabina. Să ia în considerare această posibilitate, vom estima
timpul, t
pierderea
, pentru o astfel de căldură magazin pentru a avea 50% din conținutul său retras
în timp ce menține o temperatură uniformă T
s.
Presupunând că pre-
a mâncat mediu (de exemplu, temperatura solului) are temperatura constantă T
o
,
echilibrul termic ecuația este:
=−

mc
dT
dt
T
T
R
s
s
o
(15.21)
în cazul în care mc este capacității sale de căldură, iar R este rezistența termică între
magazin și împrejurimi. Soluția de (15.21) este:
T
T
T
T
t
mcR
(0)
exp
s
o
s
o


=





(15.22)
din care rezultă că 'constanta de timp'
=
t
mcR
1.3
pierderea
(15.23)
Dacă magazinul este o sferă de rază o, rezistența termică este R
= r/4p- o
2
,
unde r este rezistivitatea termică de unitatea de suprafață, și m
= 4pa
3
r/3, deci pentru o
sfera,
r
=
t
cra
0.43
pierderea
(15.24)
A LUCRAT EXEMPLU 15.5 DIMENSIUNE ȘI IZOLARE DE CĂLDURĂ INTERN MAGAZIN
Presupune un bine izolate, casa în timpul iernii necesită o medie interne de alimentare cu căldură de 1.0 kW. Împreună cu
gratuit câștiguri de iluminat, etc., acest lucru va menține o temperatură internă de 20°C. este decis să construiască o cadă cu
apă se păstrează într-un rezervor dreptunghiular al cărui vârf forme etaj al casei, de zona de 200 m
2

. Sistemul de
încălzire trebuie să
fie adecvate pentru 100 de zile, ca toate pierderile de căldură din rezervor trece prin conducție prin podea, și ca
apa se raceste la o inițială de 60°C, la o finală de 40°C.
TWIDELL PAGINARE.indb 556
01/12/2014 11:38
§15.10 Stocare pentru sistemele de încălzire și răcire
557
A lucrat Exemplu 15.5 arată că trei luni de stocare de căldură este realist
dacă acestea fac parte din proiectarea inițială criterii, și dacă alte aspecte ale
construcției sunt luate în considerare. Acestea includ cele mai bune standarde de termică
de izolație cu umed-dovada bariere, ventilație controlată (cel mai bine cu
reciclare de căldură), și includerea gratuit câștiguri de iluminat, gătit și
metabolismul. Există exemple de astfel de înaltă tehnologie case, iar
cele mai bune au, de asemenea, imaginativ caracteristicile arhitecturale, astfel încât acestea sunt plăcut
de a trăi în (a se vedea §16.4). Multe astfel de clădiri să utilizeze rock pat de stocare, mai degrabă
decât sistemul de apă de exemplu. Termoficare cu sezon
de stocare este de asemenea posibil (de exemplu, sistemul de la Neckarsulm, în Germania,
începând cu 2001, care colectează energia solară prin umplut cu apă
colectoare și magazine, ca și căldură în sol).
Rezultă de Exemplu Lucrat 15.5 că pe termen scurt de stocare de căldură de
aproximativ patru zile este cu ușurință posibil, cu material de constructii folosit ca
1
Calculați volumul de rezervor.
2
Calcula rezistivitatea termică a căldurii calea de la rezervor la podea.
3
Sugerează cum rezervorul ar trebui să fie închise din punct de vedere termic.
4
Ce este densitatea de energie de stocare?
Soluție
1
Căldura necesară
= (1 kW)(100 de zile)(24 h/zi)(3.6 MJ/ kWh) = 8640 MJ
Volumul de apă
=



(8640MJ)
(1000 kgm )(4200J kg K )(20 K)
3
1
1
= 103 m
3
Adâncimea de rezervor
= (103 m
3
) / (200 m
2
)
= 0,5 m.
2
Presupunem căldură numai frunze prin partea de sus a rezervorului.
Din (15.23),
=
=





R
(100 de zile) (86400s zi )
(1.3) (cele 103 mn ) (1000 kgm ) (4200 J kg K )
0.0154 KW
1
3
3
1
1
1
Din (R3.5) rezistivitatea termică r
= R × (zona)
= (0.0154 KW
-1
)(200 m
2
)
= 3.1 m
2
KW
-1
3
Material izolant (de exemplu uscat polistiren expandat) are o conductivitate termică k
~ 0.04 W m
-1
K
-1

. Un
satisfăcătoare strat pe partea de sus a rezervorului, protejate împotriva excesului de presiune, ar fi o adâncime
d
= (3.1 m
2
KW
-1
)(0.04 W m
-1
K
-1
)
= 12 cm
, Pentru a evita nedorite pierderile de caldura, baza și părțile laterale ar trebui să fie izolate de la echivalentul a 50 de cm de uscat
polistiren expandat.
4
Densitatea de energie a folosit de stocare mai mare de 40°C
= (8640 MJ)/(103 m
3
)
= 84 MJ m
-3
.
Densitate de energie mai sus ambient casa de temperatura la 20 °C
= 168 MJ/m
3
.
Notă: un activ metoda de extragere a căldurii prin convecție forțată printr-un schimbător de căldură ar permite un mai bun
control, o mai mică temperatură inițială, și/sau un rezervor mai mic.
TWIDELL PAGINARE.indb 557
01/12/2014 11:38
558
De integrare, de distribuție și de depozitare
magazin. În mod similar, capacitatea termică și depozitare la rece poate avea importante
implicații pentru proiectarea clădirilor în condiții de vreme.
Materiale care modifica faza oferă o mult mai mare capacitate de căldură, peste
un interval limitat de temperaturi, decât sistemele care utilizează căldură sensibilă. De
exemplu, Glauber lui (Na
2
DECI
4
. 10H
2

O) a fost folosit ca un magazin


pentru camera de încălzire. Se descompune la 32°C la o soluție saturată de
Na
2
DECI
4
plus o anhidru reziduuri de Na
2
DECI
4
. Această reacție este reversibilă
și evoluează 250 kJ/kg ~650 MJ/m
3

. Deoarece o mare parte din costul de un magazin


pentru casa încălzire este asociat cu construcția, astfel de magazine ar putea
fi mai ieftin per total decât simple rezervoare de apă mai mică densitate de energie pe
unitatea de volum. Cu toate acestea, această metodă aparent simplă necesită
practică dificultăți să fie depășite. În special, solide și lichide fazele
de multe ori în cele din urmă separate spațial, astfel încât recombinare este împiedicat;
în consecință, fără a se amesteca, sistemul devine ineficient după
mai multe cicluri.
§15.11 SISTEME DE TRANSPORT
În multe țări, transport reprezintă aproximativ 30% din utilizarea la nivel național
a energiei comercial, cu dominantă de energie primară de intrare pentru
transport fiind de combustibili fosili, în special de ulei (vezi fig 16.3, 16.9 și
§16.5). Electrice de cale ferată, sisteme cu energie primară de energie hidro
sunt o mică excepție. La trenurile cu abur care au fost dominantă în secolului al 19 -
lea sunt acum rare; de asemenea, comercial sail-bărci, cu excepția cazului în nișă
aplicații, cum ar fi în țările în curs de dezvoltare, cu multe insule împrăștiate.
Prin urmare transportul este cel mai dificil sector de RE pentru a înlocui fosile
de combustibil utilizat.
În §16.5 avem în vedere reducerea utilizării combustibililor fosili pentru transporturi, prin
îmbunătățirea sistemului de transport ca un întreg, și în special uita la
modalități mai eficiente de gestionare a cererii (de exemplu, prin urbane
de planificare, care nu este bazat pe privat cu motor de mașină) și de întâlnire a
cererii (de exemplu, prin mai atractive sisteme de tranzit în masă). Pe termen scurt,
strategiile discutate în §16.5-au resemnat cu ceva ca actualul
model de vehicul utilizat, și să se concentreze pe îmbunătățirea eficienței consumului de combustibil al
vehiculelor ei înșiși și/sau pe combustibili alternativi sau motoare. Alternative de
combustibili care pot fi furnizate din surse regenerabile, mai degrabă decât de surse fosile
includ biocombustibili lichizi (Capitolul 10) sau a hidrogenului (§15.9.1 de mai jos). La
alternative motoare sub cele mai active de dezvoltare sunt vehicule electrice,
cheia care sunt îmbunătățite de baterii și poate celule de combustibil.
Considerabile R&D de finanțare a plecat în dezvoltarea de vehicule
care folosesc hidrogen (H
2
) ca și combustibil (de exemplu, ca o energie magazin, a se vedea §15.9.1.
Acest lucru este pentru că arderea H
2
sau de reacție într-o celulă de combustibil pentru elec-
tricity (Fig. 15.8) produce doar H
2

O, evitându-se astfel 'coada conducta de' poluare.


Cu toate acestea, poluarea este mutat în altă parte, deoarece cele mai multe hidrogen astăzi este
produs ca un produs de rafinare a petrolului. Poluarea poate fi doar
TWIDELL PAGINARE.indb 558
01/12/2014 11:38
§15.12 aspectele Sociale și de mediu de aprovizionare cu energie și de stocare
559
eliminate prin utilizarea de hidrogen din electroliza apei cu energie electrică
de la non-termice din surse regenerabile de energie.
§15.12 ASPECTELE SOCIALE ȘI DE MEDIU
DE APROVIZIONARE CU ENERGIE ȘI DE STOCARE
Energie de livrare și de depozitare sunt importante. Economia mondială depinde
pe distribuția de energie pe o scară foarte mare. Comerțul internațional în
combustibili fosili (cărbune, petrol și gaze) de la relativ puține țările care exportă
în cantități mari a depășit 11% din comerțul mondial în 2012.
1

Cele
concentrate liniile de aprovizionare sunt vulnerabile la întreruperi, astfel încât mai multe războaie sunt
atribuite consumatoare de petrol județe încercarea de a asigura aprovizionarea lor (a se vedea
Yergin 1992). Faptul că petrolul și cărbunele sunt ieftine magazine de mari
cantități de ușor accesibil de energie (vezi Fig. 15.3) a permis creșterea rapidă
a orașelor. Nu a fost puțin inițial atenția asupra mediului
consecințe (McNeill 2000) și a eficienței globale cu care energia
a fost folosit (Capitolul 16). Ocazional eșecuri ale sistemelor de distribuție
au consecințe grave asupra mediului, în special pe scară largă scurgeri de ulei.
Guvernele naționale să accepte responsabilitatea de a supraveghea și asigura
aprovizionarea cu energie la toate nivelurile societății (articolul 17.2). De exemplu, energie
rutele de distribuție a primi prioritate de planificare permisiuni și grave
perturbări ale personalului militar sunt folosite pentru a menține aprovizionarea.
Cum s-a discutat în Capitolul 1, mai puțin concentrate și dispersate natura
surselor regenerabile de energie permite o schimbare majoră departe de internațional
și centralizat de furnizare a energiei și vulnerabile de distribuție. Prin urmare,
este general recunoscut faptul că furnizare de energie regenerabilă au un
impact favorabil, mai ales în ceea ce privește securitatea aprovizionării. Cu toate acestea,
considerabil interese în status quo handicap din toată inima
dezvoltarea și utilizarea de surse regenerabile de energie.
Metode de stocare a energiei sunt importante pentru a sprijini continuitatea de
alimentare (și cu pompat hidro pentru energie electrică și naționale de rezervă de ulei
de magazine) și vitală pentru putere autonomă (ca în baterii pentru vehicule
de plecare și de iluminat, și pentru stand-alone și puterea de urgență).
Sunt relativ minore de mediu pericole la unele de stocare
mecanisme descrise în acest capitol. În special, baterii de tot felul
sunt umplute cu substanțe chimice nocive, astfel încât eliminarea în condiții de siguranță este necesar.
Bateriile plumb-acid sunt atât de răspândite pentru vehiculele care există o înfloritoare
afacere de reciclare în cele mai multe țări, cu plumb, mai ales reciclate de la
"mort" bateriile. Operaționale pericole sunt întotdeauna important pentru a proteja
împotriva, și pericolele de defecțiuni mecanice, de incendiu și de explozie, scurgeri
și șoc electric trebuie să fie recunoscute și să ne păzim.
Contrar populare impresie, gazul de hidrogen este mai periculoase
cu privire la incendiu și de explozie mult mai familiar de gaze naturale (metan).
Desigur, două greșeli nu fac un drept, deci este nevoie de grijă prin utilizarea
stabilit criteriile de siguranță. Astfel, aspectele sociale și de siguranță, fără a nega o
TWIDELL PAGINARE.indb 559
01/12/2014 11:38
560
De integrare, de distribuție și de depozitare
'economia de hidrogen, care este mult mai limitat de economie,
infrastructură și necesitatea de a adapta cele mai multe end-utilizarea de dispozitive.
Masă termică în clădiri este o formă de stocare a energiei. 'Grele
de construcții, cu corespunzătoare de izolare externă, permite solare pasive și de
alte variabile (poate fi variabil, de preț, de asemenea,) câștigurile de căldură să fie depozitate
în mod intrinsec, cel puțin de la zi la noapte și de la o zi la alta. Alternativ, la
'racoare' de o clădire de la pierderea de căldură în noapte pot fi "stocate"
prin intermediul a doua zi. Astfel simplu de stocare a energiei are implicații majore pentru
confort și mai eficientă a aprovizionării cu energie în clădiri, care
, în general, să utilizeze cel puțin 30% din național de aprovizionare cu energie, după cum se consideră în
§16.4. Pe scară largă reintroducerea unor astfel de "grele" și în mod corespunzător
izolate clădiri are implicații considerabile pentru eficiență energetică,
regulamentele de planificare și de construcție a resurselor.
REZUMAT CAPITOL
Eficiente de distribuție și stocare de energie sunt legate de teme pentru toate livrările de energie, inclusiv combustibil și
energie electrică. Natura variabilă de cele mai multe surse regenerabile necesită atât integrarea cu alte bunuri
și de stocare a energiei. Cu toate acestea, cele mai multe dintre tehnologii și metode pentru acest lucru sunt deja în uz și
necesare pentru convențională provizii. Acest capitol demonstrează că nu există motive tehnice pentru a preveni
integrarea de o creștere semnificativă a surselor regenerabile de energie (RE) în aprovizionarea cu combustibili și energie
electrică.
Stocarea de energie sub formă de căldură este frecvent practicată în prezent în zonele cu climă rece. Hidrogen și metan produs
de la RE sunt destul de simplu de a se integra în rețelele de distribuție a gazelor. La scară mică de stocare de energie electrică
în baterii este larg răspândită. Unele RE tehnologii pot fi, de asemenea, utilizate în mod direct în sectoarele de utilizare finală
(cum ar fi
prima generație de biocombustibili și clădire integrat încălzitoare solare de apă).
Proporția de energie livrată în energie electrică este în creștere în majoritatea țărilor, și poate crește
în continuare ca electricitatea este folosită pentru transport și distribuite de generare să fie împărtășită în rândul consumatorilor.
Integrarea RE în rețelele de energie electrică, chiar și cu variabile surse regenerabile cum ar fi energia eoliană și solară
putere, este acum o practică standard. Această integrare poate fi îmbunătățită de stocare a energiei, inclusiv pompat
de stocare a apei, volanta de stocare de energie cinetică și aer comprimat de stocare. Moderne de microelectronică
și de comunicații, utilizate în rețele inteligente și în micro-generare, poate îmbunătăți, de asemenea, de integrare, inclusiv
prin gestionarea dinamică a cererii de energie electrică. Substituirea surselor regenerabile de energie pentru combustibilii fosili
stocate
de energie în transport este o provocare majoră.
ÎNTREBĂRI RAPIDE
Notă: Răspunsurile la aceste întrebări sunt în textul de la secțiunea relevantă
din prezentul capitol, sau poate fi ușor dedusă din aceasta.
1
Numele a trei sisteme principale utilizate pentru distribuirea de energie pentru consumatori.
2
Ce este o rețea de energie electrică, și de ce sunt astfel de grile atât de utilizat pe scară largă?
3
Explica de ce potrivire ofertei la cerere este o provocare pentru
furnizorii de energie electrică (a) în general; (b) atunci când energia eoliană este o proporție majoră
de aprovizionare se amestecă.
4
Ce este micro-generație?
TWIDELL PAGINARE.indb 560
01/12/2014 11:38
Probleme
561
5
În ce condiții se poate de stocare a energiei a fi benefice?
6
Nume șase tehnologii utilizate în comerț pentru stocarea de energie.
7
Ce este o "rețea inteligentă" și cum i-ar permite reducerea costurilor atât pentru
producătorii și consumatorii de energie electrică?
8
Sublinia ceea ce se înțelege printr-o economie bazată pe hidrogen și explica cum s -
ar putea să se refere la energia din surse regenerabile.
9
Ce factori limită (a) de viață; și (b) masa de plumb-acid
baterii?
10
Sublinia avantajele și dezavantajele de vehicule electrice pentru
(a) utilizatorului; (b) publice; și (c) rolul energiei din surse regenerabile.
PROBLEME
15.1
Estima densitatea de energie (MJ/m
3

) de o pompat hidro magazin


, care este de 100 m deasupra acesteia stație de putere. (Sugestie: ia în considerare schimbările în
energie potențială gravitațională.)
15.2
Un autobuz de pasageri utilizate în Elveția derivat motiv putere
de energia stocată într-o mare volant. Volanta a fost
pus la curent, atunci când autobuzul s-a oprit la o stație, de către un
motor electric, care poate fi atașat la liniile de alimentare electrică.
Volanta a fost un solid cilindru de oțel de masa a 1000 kg, cu diametrul de
180 cm, și s-ar transforma până la 3000 rot min
-1
.
(a) La viteza de top, ceea ce a fost energia cinetică a volantului?
(b) Dacă puterea medie necesară în exploatare autobuz a fost de 20 kW,
ceea ce a fost timp mediu între opriri?
15.3
O volanta de trei uniformă baruri, rotirea lor centrală puncte
ca spițele unei roți, este facut din fibre de " E " de sticlă cu densitatea
r = 2200 kg/m
3
și rezistența la tracțiune 3500 MN/m
2
. Fibrele sunt
aliniate de-a lungul baruri și deținute împreună cu o cantitate minimă
(10%) din rășină de neglijabilă rezistența la rupere și densitate similare.
Calcula valoarea maximă densitate de energie obținute. Dacă o
= 1.0 m,
ce este corespunzătoare vitezei unghiulare?
15.4
Estimări ale cererii și ofertei de energie pentru Marea Britanie.
(a) Totalul consumului final de energie al Marii Britanii a fost de aproximativ 150 de milioane de euro
tone echivalent petrol (TEP), în 2008, și atunci când populațiile
din marea Britanie și din lume au fost de aproximativ 60 de milioane de euro și de 6,6 miliarde de euro
, respectiv. Compara respectiv consumul de energie
per persoană. (Indiciu: vezi Fig. 16.3.)
(b) Cum non-necesarul de căldură variază cu sezonul? Ce
tipuri de produse industriale și de uz casnic face acest lucru corespunde
la? (Sugestie: verifica împotriva Capitolul 16, mai ales Smochine 16.3 și
16.12.)
TWIDELL PAGINARE.indb 561
01/12/2014 11:38
562
De integrare, de distribuție și de depozitare
(c) Utilizarea datelor în Capitolul 2 (mai ales Fig. 2.18) pentru a estima
solar de căldură de intrare pe 1 m
2
de suprafață orizontală, și pe 1 m
2

de
(orientat spre sud) suprafață verticală în fiecare sezon. (Latitudine de
marea Britanie este de aproximativ 50°N.) Ce este un tipic eficiența unei instalații solare
de încălzire? Ce captatorului ar fi necesare pentru a furniza
puterea necesară pentru încălzire indicat în Fig. 16.12? Cât de multe
m
2

fiecare casă reprezintă? Este rezonabil? Ar


energie solară pasivă, tehnici, combinate cu termica
de izolare, de ventilare, și utilizarea gratuită câștiguri, fie de
semnificație?
(d) Aproximativ ce este energia electrică obținută din
1m
2

de măturat zona într-o medie a vântului de 8 m/s (vezi §8.1). De


pământ și apele de mică adâncime din marea Britanie pot fi tratate foarte
aproximativ după două dreptunghiuri 1000 m x 200 m, cu
mai laturi vântul. Considerăm mare de 100 m
cu diametrul de turbine eoliene cu viteza medie a vântului de 8 m/s la hub
înălțime. Cât de multe turbine eoliene vor fi necesare pentru a produce
o putere medie de 15 GW pentru întreaga țară? Ce
ar fi distanța medie între ele în cazul în care jumătate au fost pe
teren și jumătate de la mare?
(e) Utilizarea val de putere harta din Fig. 11.10 pentru a estima lungimea
de un baraj pentru a genera o putere medie de 15 GW de pe
coasta de nord-vest a marii Britanii. Cum face acest lucru lungime compara cu
lungimea de coasta?
15.5
Cel mai mare câmp magnetic, care pot fi menținute în mod obișnuit printr-un
convențional electromagnet este B
0
~1 Wb/m
2
. Densitatea de energie
într-un câmp magnetic este W
v
= ½B
2
/
m
0
. Calcula W
v
pentru B
= B
0
.
15.6
Calculați fluxurile de energie în următoarele cazuri:
(a) Aproximativ 30 de milioane de barili de petrol pe zi fiind expediate din
Persic zonă (1 baril
= 160 de litri).
(b) ROBINET de țiței prin conducte din Irak la marea Mediterană
transporta aproximativ 10 milioane de tone de petrol pe an.
(c) O familie de patru persoane într-o gospodărie bucătari, folosind o butelie de
GPL (gaz) (13 kg) pe lună.
(d) aceeași familie conduce o masina care acoperă 8000 km/an, cu o
benzină consum de 7 litri la 100 km (= 31 km pe NOI
gal = 40 km / gal marea BRITANIE).
(e) Un sătean din Papua Noua Guinee nevoie de două ore pentru a aduce un
sarcina de 20 kg de lemn de la bush, care transportă-l pe spate.
(f) O 3 t camion transportă combustibil-lemn în oraș la o viteză de 30 km/h.
(g) O 40 de litri de carburant auto, rezervorul fiind plin de gol în două minute.
TWIDELL PAGINARE.indb 562
01/12/2014 11:38
Bibliografie
563
15.7
Un otel de conducte de diametru 30 cm transportă gaz metan (CH
4

).
Recompresie stații sunt amplasate la 100 km intervale de-a lungul
conductei. Presiunea gazului este amplificat de la 3 la 6 MN/m
2
la fiecare
station. (Acestea sunt tipice condiții comerciale.) Calcula
(a) fluxul de masă; și (b) fluxul de energie. (c) Ce volum pe zi
de gaze de la STP ar corespunde? (Indiciu: se referă la (15.4) și
Fig. R2.5 (în curs de Revizuire 2), apoi face o prima estimare a f, presupunând
R
este 'destul de mare'. Apoi găsi m
. și verificarea coerenței. Repeta
dacă este necesar. Vâscozitate de metan de la aceste presiuni este:
m=
×


10 10 N s m .
6
2
15.8
Un electrice de transmisie linie de link-uri de 200 MW de centrale hidroelectrice
de instalare a unui oraș B 200 km, de la 220 kV. Cablurile
sunt concepute pentru a disipa 1% din puterea de realizat. Calcula
dimensiunile de sârmă necesară, și să explice de ce pierderi de 1% poate
fi din punct de vedere economic, de preferat la pierderi de 10% sau 0,1%.
15.9
Având în vedere șase pole-pereche generator de inducție (§R1.6), dacă s
=
-0.1 în generație într-o 50 Hz grilă, determina indus rotor
de curent de frecvență f
2
și f
s
.
15.10
Modificările în entalpia, energia liberă și entropia în formarea
de apă
H
2
+ ½O
2
→H
2
O
(gaz)

sunt, respectiv,
DH = -242 kJ/mol
DG = -228 kJ/mol
DS = -47 J K
-1
mol
-1
Estimarea temperatura peste care H
2
O este termodinamic
instabil. (Sugestie: ia în considerare (15.16)).
NOTĂ
1
ONU pentru Comerț Internațional Statistică (www.comtrade.un.org).
BIBLIOGRAFIE
General
Institutul de inginerie Mecanică (2000) surse Regenerabile de Stocare a Energiei: rolul Său în surse regenerabile de energie și
viitor
piețele de energie electrică, Inginerie Profesionist Publicații, Bury St. Edmunds. Set de lucrări ale conferințelor,
inclusiv articole scurte pe regenerare celule de combustibil', volante și câmpurile magnetice de stocare a energiei.
Jensen, J. și Sorensen, B. (1984) de Stocare a Energiei, Wiley, Chichester. Încă una dintre puținele cărți în mod special pe
acest subiect. Bună acoperire la aproximativ același nivel ca în această carte.
TWIDELL PAGINARE.indb 563
01/12/2014 11:38
564
De integrare, de distribuție și de depozitare
Kreith, F. și Kreider, J. F. (2011) Principii de Energie Durabilă, CRC Press, Londra. Capitolul 10 se referă la energia
de stocare, în special cu detalii pe sisteme de aer comprimat.
Sims, R., Mercado, P., Krewitt, W., Bhuyan, G., Flynn, D., Holttinen, H., Jannuzzi, G., Khennas, S., Liu, Y.,
O 'Malley, M., Nilsson, L. J.,Ogden, J., Ogimoto, K., Outhred, H., Ulleberg, O. și van Hulle, F. (2011) Integrarea
surselor regenerabile de energie în prezent și viitor sisteme de energie', în O. Edenhofer, R. Pichs-Madruga, Y. Sokona, K.
Seyboth, P. Matschoss, S. Kadner, T. Zwickel, P. Eickemeier, G. Hansen, S. Schlömer și C. von Stechow (eds),
IPCC Raport Special privind Sursele de Energie Regenerabile și Atenuarea efectelor schimbărilor Climatice, Cambridge
University Press,
Cambridge. Recenzii de stat de artă și perspectivele de viitor, mai ales de la o energie a sistemelor de perspectivă.
NOI de Stocare a energiei electrice de Asociere. O asociație de comerț cu mult mai utile informații tehnice pe site-ul său
(www.electricitystorage.org/technology/storage_technologies/technology_comparison).
Cele mai multe cărți în special surse de energie regenerabile (menționate în capitolele corespunzătoare) includ unele
discuții din mass-media de stocare aplicabile la sursă (de exemplu, de căldură magazine solare, baterii pentru vânt).
Depozitarea substanțelor chimice
Carden, P. O. (1977) de Energie corradiation folosind reversibile amoniac reacția', Energie Solară, 19, 365-378. Stabilește
principalele caracteristici ale unui solar/amoniac sistem folosind distribuite colectorilor. Multe mai târziu documentele elaborate
pe
detalii și sisteme similare
Dunn, R., Lovegrove, K. și Burgess, G. (2012) O revizuire pe bază de amoniac termochimic de stocare a energiei pentru
concentrarea energiei solare', IEEE Journal, 100, 391-400).
Goel, N., Miraball, S., Ingley, H. A. și Goswami, D. Y. (2003) 'producția de Hidrogen', Progresele în domeniul Energiei Solare,
15, 405-451. Accent pe producția de energie din surse regenerabile; include estimări de costuri.
Consiliul național de Cercetare [SUA] (2004) Din Economia de Hidrogen: Oportunități, barierele și costurile de R&D are
nevoie,
Academiile Naționale de Presă, New York. Disponibil online la nap.edu.
Wald, M. L., (2004) Întrebări despre o economie de hidrogen', Scientific American, 290, 42-48. S-a uitat la puțuri pentru
roți' analiza de energie.
Oricare dintre mai multe manuale de chimie fizică vă va oferi o analiza termodinamica a eliberarea de căldură în
reacții chimice (de exemplu, Atkins, P. W. și de Paul, J. (2002) Atkins' Chimie Fizică, Oxford University Press,
Oxford).
De stocare de căldură
Duffie, J. A. și Beckman, W. A. (2006, 3rd edn) Solar de Inginerie a Proceselor Termice, Wiley, New York.
Capitolul 9 este preocupat în special cu stocare de căldură.
Ryle, M. (1977) 'Economie de surse alternative de energie', Natura, 267, 111-116. Clasic, de hârtie, de marca, afirma
argumentând
că stocare pentru aproximativ șapte zile permite vânt/val/solar pentru a se potrivi cele mai multe fluctuații în marea BRITANIE a
cererii.
Electrice de stocare
Oricare dintre mai multe manuale de chimie fizică vă va oferi o introducere elementară electrochimie
folosit în acest capitol (de exemplu, Atkins, P. W. și de Paul, J. (2002) Atkins' Chimie Fizică, Oxford University Press,
Oxford).
TWIDELL PAGINARE.indb 564
01/12/2014 11:38
Bibliografie
565
Crompton, T. R. (2000. 3rd edn) Baterie Carte de Referință, Newnes, Oxford.
Electropaedia (www.mpoweruk.com/index.htm). O sursă utilă de informații tehnice de bază despre bateriile
de toate tipurile, compilate de către Woodbank de Comunicații.
ITDG (2004) Baterii, ITDG tehnice scurtă. Disponibil online la www.itdg.org. Foarte jos-la-pamant ghid cu privire la modul
de a utiliza și să se uite după bateriile.
Lindsay, T. J. (1999) Secretele de Baterii Plumb-Acid, Lindsay Publications Inc., Il 60915, statele UNITE ale americii. Patruzeci
și opt de pagini de
explicații practice și de orientare, care este greu de găsit în altă parte.
Rand, D. A. J., Pădure, R. și Dell, R. M. (1998) Baterii pentru Vehicule Electrice, Societatea Inginerilor de Automobile,
Pennsylvania. Acoperă toate tipurile de depozitare baterii.
De yuani, X., Liu, H. și Zhang, J. (eds) (2012) Baterii Litiu-ion: materiale și tehnologii Avansate, CRC
Press, Londra. Multi-autor compilație cu o analiză detaliată de materiale și provocări tehnice pentru anozi,
catozi, electroliti, etc.
Celulele de combustibil
Alleau, T. (2003) 'O stare de arta de hidrogen și celule de combustibil tehnologii: difuzie perspective și bariere', în
A. Avadikyan, P. Cohendet și J-A. Heraud (eds), Dinamica Economică a Tehnologiilor pilelor, Springer,
Berlin.
Hoogers, G. (ed.) (2003) Tehnologia de Celule de Combustibil Handbook, CRC Press, Londra. Comentariu de tehnologie și
aplicații în sistemele de putere și vehicule.
Srinivasan, S. (2006) Celule de Combustibil: De la concepte fundamentale la aplicații, Springer, Berlin. Manual cuprinzător
de >600pages.
Volante
Genta, G. (1985) Energia Cinetică de Stocare: Teorie și practică avansată volanta sisteme, Butterworths,
London. Carte-lungime detaliu.
De distribuție și de transport și aspecte sociale
Boyle, G. (ed.) (2007) surse Regenerabile de Energie și Grila, Earthscan, Londra.
BP statistical Review of World Energy (anual). Date și hărți care arată producția, consumul și comerțul,
în special în combustibilii fosili.
El-Sharkawi, M. A. (2012, 3rd edn) Energie Electrică: O introducere, CRC Press, Londra. Inginerie manual
acoperă ambele mașini electrice și sisteme de distribuție a energiei.
Freris, L. și Teren agricol, D. (2008) surse Regenerabile de Energie în Sistemele de Putere, Wiley, Chichester. De obicei
sistemele de putere
mecanica, dar cu un accent special pe RE.
Galan, E. M. et al. (2012) Rurală din surse regenerabile de energie', Capitolul 5 în REN21 (2012) Global Status Report 2012.
Disponibil online la www.ren21.org.
McNeill, J. R. (2000) Ceva Nou Sub Soare: Un mediu de istoria secolului al xx-lea, Penguin,
London. Creșterea de combustibili fosili-a tras orașe și impactul acestora asupra apei, aerului și biosfera.
TWIDELL PAGINARE.indb 565
01/12/2014 11:38
566
De integrare, de distribuție și de depozitare
Nuttall, P. et al. (2013) O revizuire a durabilă mare de transport pentru Oceania: furnizarea de context pentru surse regenerabile
de energie
de transport pentru Pacific', Politica maritimă, http://dx.doi.org/10.1016/j.marpol.2013.06.009i.
Yergin, D. (1992) Premiul: epic quest pentru petrol, bani și putere, Simon & Schuster, New York. Cartea a câștigat
Premiul Pulitzer pentru non - ficțiune în 1992 pentru sale de autoritate de raportare și comentarii pe industria de petrol.
TWIDELL PAGINARE.indb 566
01/12/2014 11:38

Utilizarea eficientă a energiei


CAPITOLUL

16
CONȚINUTUL
Obiective de studiu
568
§16.1 Introducere
569
§16.2 servicii Energetice
571
§16.3 consumului final de Energie pe sectoare
574
§16.4 consum redus de Energie (solar) clădiri
576
§16.4.1 concepte Generale
576
§16.4.2 Spațiu de încălzire: principii
576
§16.4.3 solare Pasive clădiri
578
§16.4.4 solare Active de constructii
sisteme de
581
§16.4.5 zonele cu climă Rece
582
§16.4.6 climă Temperată
584
§16.4.7 climat Cald, uscat
587
§16.4.8 Cald, tropical umed
clima
589
§16.4.9 Compozit climate
591
§16.5 Transport
591
§16.5.1 Fundal
591
§16.5.2 Vehicule
591
§16.5.3 durabil de transport
sistem?
593
§16.5.4 de Transport și forma urbană
593
§16.5.5 Îmbunătățit vehicule
596
§16.5.6 de transport de Marfă
598
§16.5.7 Aviației
599
§16.6 industria Prelucrătoare
599
§16.7 Interne de utilizare a energiei
601
§16.8 aspectele Sociale și de mediu
602
§16.8.1 Negawatts sunt mai ieftine decât
megawați!
602
§16.8.2 Impact privind energia din surse regenerabile 603
§16.8.3 Căi economice
dezvoltarea
603
§16.8.4 Clădiri
604
§16.8.5 implicațiile asupra Mediului de
eficiența energetică
604
Rezumat capitol
605
Întrebări rapide
606
Probleme
606
Note
608
Bibliografie
608
Caseta 16.1 eficiență Maximă a
motoare termice
573
Caseta 16.2 impactul tehnologiei
schimbare în iluminat în Anglia,
1500-2000
573
Cutie 16.3 Rezumat al RE aplicații în
selectate sectoare de utilizare finală
575
Cutie de 16,4 coduri de Construcție
578
Cutie De 16,5 Solar Decathlon
586
Cutie de 16,6 Electrochromic windows
589
Cutie 16.7 Curitiba: un studiu de caz din mediul urban
design pentru durabilitate și
reducerea cererii de energie
595
Cutie de 16,8 dimensionarea Corespunzătoare a țevilor și
pompe economisește energie
600
Cutie 16.9 consumul de Energie în China
604
TWIDELL PAGINARE.indb 567
01/12/2014 11:38

www.shahrsazionline.com
568
Utilizarea eficientă a energiei
OBIECTIVE DE STUDIU

Apreciez că sistemele energetice includ ambele
utilizări finale (cerere) și-a generație (de aprovizionare).

Dau seama că consumatorii doresc servicii de energie,
nu energie ca atare.

Să definească și să cuantifice end-eficiența utilizării și
ansamblu (sistem) de eficiență.

Dau seama că utilizarea eficientă a energiei îmbunătățește
satisfacția consumatorilor.

Înțeleg de ce eficiența energetică și
energia regenerabilă sunt complementare.

Apreciaza că emisiile de gaze de seră
sunt astfel reduse.

Examina exemple de modul în care energia solară,
izolare, materiale avansate și alte aspecte legate de
proiectarea clădirilor dau eficiente energetic și
de viață confortabil în ambele calde și reci climate.

Ia în considerare modul în care economia beneficii de
eficiență energetică în sectoarele de transport,
de aprovizionare cu energie și clădiri.

Ia în considerare aspectele sociale și de mediu
pentru punerea în aplicare a eficienței energetice.
LISTA DE FIGURI
16.1 fluxul de Energie de la sursă la final de servicii.
570
16.2 Progresie de servicii de iluminat în Anglia în timpul perioadei de la 1500 la 2000, indicând
dominantă tehnologie de iluminare.
574
16.3 (a) Lumea consumului de energie primară de sector de utilizare finală. (b) cererea de Energie de serviciu.
575
16.4 Directă a obține pasive de încălzire solară.
579
16.5 Clădiri potrivite pentru un climat rece.
583
16.6 Patru clădiri potrivite pentru un climat temperat.
585
16.7 climat Cald, uscat zona.
588
16.8 Cald, umed zona.
590
16.9 combustibil Lichid utilizat de către sectorul 2008 până în 2035 (milioane de barili de petrol/zi).
592
16.10 Transport consumul de energie pe cap de locuitor într-o serie de orașe.
594
16.11 Una dintre stațiile de autobuz din sistem integrat de transport utilizate de către 85% din populația
Orașul Brazilian Curitiba.
596
16.12 economii de Energie, în marea BRITANIE residences.
601
16.13 pierderea de Căldură printr-o fereastră.
607
LISTA DE TABELE
16.1 performanță Posibilă a unui viitor "avansat" autoturisme
597
TWIDELL PAGINARE.indb 568
01/12/2014 11:38
§16.1 Introducere
569
§16.1 INTRODUCERE
Motivația pentru acest capitol într-o carte despre energia regenerabilă este că
îmbunătățirea eficienței de utilizare a energiei și, astfel, mai puțin cererea de energie crește
oportunitățile pentru sisteme de energie regenerabilă. Acest lucru prezintă oportunități
pentru a reduce utilizarea combustibililor fosili și trecerea la curat sisteme durabile –
o dublă oportunitate de reducere a gazelor de seră.
Alimentarea cu energie este costisitoare, necesită infrastructură (de exemplu, transport,
rețelele de electricitate, întreținere) și echipamente pentru a converti primare
de energie de la o formă utilizabilă. Prin urmare, reducerea cererii de energie, fără
pierdere de beneficiu reduce în curs de desfășurare costuri pentru consumator. De exemplu,
capacitatea, și, prin urmare, costul, de panouri fotovoltaice și auxiliare,
echipamente pentru timpul zilei interne de iluminat depinde de controlul
necesar de iluminare și eficiența lumini; un control mai bun și
de eficiență necesită o capacitate mai redusă și deci mai puțin PV costul de capital. Pentru
furnizorul de energie, o cerere mai puțin înseamnă mai puține cheltuieli de capital asupra plantelor
capacitatea și posibilitatea de a integra surse de energie regenerabile.
Cu toate acestea, furnizorul de venituri din facturile consumatorilor reduce cu excepția cazului per
unitate taxele au crescut, așa că reglementarea guvernamentală este necesară pentru o
corectă și eficientă energetic de sistem.
Un principiu general, este că nu avem nevoie de energie ca atare, dar
servicii energetice furnizate, cum ar fi de iluminat, de încălzire, de comunicare și de
transport. Aceste servicii sunt, de asemenea, numit end-utilizări.
În furnizarea de energie pentru un serviciu, de energie primară este, de exemplu,
energia chimică conținut de combustibil sau energie de la radiația solară
pe o fotovoltaice dispozitiv. Sunt multe de conversie și de alimentare cu pași
de-a lungul drum, cum este indicat în Fig. 16.1. Întregul proces și fiecare
intermediar de conversie etape în Fig. 16.1 au o eficiență energetică
definit în general prin:
η=
producția de energie
energia de intrare
(16.1)
Există întotdeauna unele pierderi în orice conversie a energiei, deci
η ≤ 1. De
ieșire de o etapă este de intrare la alta, rezultă că
eficiența în a merge la, de exemplu, etapa 1 la etapa 3 este:
η
13
=
η
12
η
23
(16.2)
și așa mai departe. În special, putem calcula un sistem global de energie
de eficiență:
η
pu
= E
u
/ E
p
(16.3)
unde E
p
este energetice primare utilizate în furnizarea de servicii și E
u
este
energia livrata de utilizare finală pentru un serviciu.
În practică, eficiența de toate aceste etape pot fi îmbunătățite (de exemplu,
îmbunătățit turbine, vehicule care sunt mai eficient de combustibil, mai bine
TWIDELL PAGINARE.indb 569
01/12/2014 11:38
570
Utilizarea eficientă a energiei
de izolare termică, cu LED-uri (light emitting diode) și fluorescente compacte
(CFL) lumini în loc de becuri incandescente). Îmbunătățirea eficienței energetice
de fiecare parte a sistemului reduce cererea de energie primară. Dacă
eficiența globală crește, atunci impactul scădere per unitate de
energie livrată. De exemplu, cu combustibili fosili, mai eficiente
de conversie utilizarea finală și procese de reducere pe unitate de gaze cu efect de seră asociate
Fig. 16.1
Indicativ diagrama fluxurilor de energie de la sursă la final de servicii. Notă modul în care diferite
rute depind de tehnologiile în cauză. De exemplu, de vânt pentru iluminat în
diagrama poate necesita două etape, încă de combustibil nuclear pentru iluminat necesită patru etape. La
fiecare pas există o pierdere de energie, deoarece nici o transformare nu este 100% eficient.
Sursă primară
de exemplu, combustibil fosil,
combustibil nuclear,
soare, vânt, etc.
Convertirea la altă formă
, de exemplu solid la combustibil lichid,
dezintegrare radioactivă la căldură,
fotosinteza pentru a biomasei,
debitul de apă de rotație a turbinei
De conversie a căldurii
(inclusiv motoare termice)
, de exemplu apă de căldură de la abur pentru
turbine arde combustibil într-un gaz
cu turbină, exploda combustibil într-un
motor cu piston
Generație (de energie electrică)
de exemplu, turbine eoliene, turbine cu abur
se transformă un generator de energie electrică
De distribuție a energiei
exemplu, energie electrică, cabluri și linii,
petrolier de mare și pe uscat,
combustibil lichid si gazos conducte
Sfârșitul utilizarea serviciului
de exemplu, de iluminat, de telecomunicații,
de calcul, de transport,
de refrigerare, de încălzire a spațiului
Sfârșitul utilizați dispozitivul
de exemplu, soba pe lemne, solare de apă
incalzitor, lumina electrica,
vehicul cu motor.
TWIDELL PAGINARE.indb 570
01/12/2014 11:38
§16.2 servicii Energetice
571
emisiile și, astfel, treptat riscul de schimbările climatice globale (a se vedea
Caseta 17.1). Cu turbine eoliene (Capitolul 8) per unitate de energie electrică livrată,
cu atât mai mare turbină și a sistemului de eficiență, mai puțin numărul de
turbine, impactul acestora și costurile de capital și operaționale (a se vedea §17.6).
Fiecare aprovizionare cu energie o folosim are impact asupra mediului și costurile de
bani. Prin urmare, utilizarea mai eficientă a energiei necesită mai puțin pentru
aceeași prestație, astfel încât reducerea impactului și de economisire pe recurente și de capital
costul serviciului. Mai mult, folosind mai puțină energie pentru același beneficiu
crește securitatea energetică pentru persoane fizice și pentru țara lor (a se vedea §17.2).
Este esențial, prin urmare, că ne-am gestioneze utilizarea eficientă a energiei. Această
necesitate se aplică la toate formele de aprovizionare cu energie; cu toate acestea, modul de
aplicare poate diferi în funcție de tipul de alimentare; de exemplu:.

Reactoare nucleare în practică a produce o aprovizionare constantă de căldură și
, prin urmare, de energie electrică, astfel încât integrarea cu alte controlabile de provizii și
de stocare a energiei este esențială.

Furnizare de energie regenerabilă varia semnificativ în funcție de
sursa de mediu; prin urmare, producția lor necesită indeplinesc sarcini sau
integrarea cu alte bunuri și de stocare a energiei (Capitolul 15).

Combustibilii fosili produc atmosferică și poluarea apei; utilizarea mai puțin mai
eficient, prin urmare, este vital.
În practică, majoritatea utilizatorilor nu știu sursa primară de energie, astfel
lăsând de gestionare a cererii lor de reglementare de stat
și furnizori (de exemplu, prețul de combustibil, prețurile tarifare pentru energie electrică,
obligațiile furnizorilor de a oferi un mix de materiale, eficiența energetică
campanii, codurile și standardele, interzicerea produse ineficiente (de exemplu,
lămpi electrice cu incandescență), taxe reduse și/sau subvenții pentru energie
eficienta a produselor (de exemplu, izolarea termică, încălzitoare solare de apă; a se vedea §17.5).
Cu toate acestea, consumatorii individuali pot și ar trebui să încă să își ia în serios
responsabilitatea pentru consumul de energie (a se vedea §17.2.3).
Această carte este în primul rând despre energia din surse regenerabile, astfel încât în acest capitol vom
acorda o atenție deosebită utilizării energiei în clădiri (§16.4) în cazul în care economiile
din achiziționat energie poate veni de la solară pasivă a structurilor și
microgenerare. Secțiunile ulterioare sublinia potențialul de economii de energie
în transport (§16.5) și industrie (§16.6), deoarece aceste sectoare reprezintă
mai mult de jumătate din consumul total de energie la nivel mondial. §16.7 se concentrează pe
sectorul rezidential (de exemplu, energie utilizate în case), în special electrice,
aparate, ca subiecte mai deschise la acțiune de către persoane fizice. Capitolul
se încheie cu o scurtă examinare a problemelor sociale, beneficiile și
costurile de eficiență energetică (§16.8).
§16.2 SERVICII ENERGETICE
Acesta este serviciul funcție de energia care contează, nu de energie în sine.
Avem nevoie de încălzire, de iluminat, de livrare a mărfurilor, libera circulație a oamenilor,
TWIDELL PAGINARE.indb 571
01/12/2014 11:38
572
Utilizarea eficientă a energiei
comunicarea de informații și multe alte astfel de beneficii. Fig. 16.1
oferă exemple de multi-pas proces ca sursă primară de energie este transformată
într-un serviciu, de obicei, prin intermediar de conversie (de exemplu, alimentarea cu energie electrică și
puterea la arborele motor) și de energia de vectori (de exemplu, prin conducte de încălzire,
energie electrică, rețele de distribuție, a conductelor de gaz). Figura prezintă
importante terminologie, cum ar fi "conversie", "generație" și "distribuție".
În termodinamică limitări de conversie termică de motoare termice
sunt deosebit de importante, ca și cu vehicule cu motoare cu ardere și cu
stație de putere turbine pentru producerea de energie electrică. Carnot teorie explică
faptul că numai o fracțiune de intrare de căldură de energie poate fi transformată în
lucru mecanic de ieșire, cu restul emise pentru mediu
temperatură mai mică de căldură (a se vedea Caseta 16.1). Consecința practică este
că ori de câte ori căldura este transformată în lucru, ca și în motoare și turbine,
mecanic de eficiență este cel mai bun la circa 45% (de înaltă temperatură gaze
turbina), de multe ori aproximativ 35% (cărbune, centrale nucleare) și de obicei
mai puțin (ardere biomasa și energia geotermală de posturi, cele mai multe vehicule
motoare). Cea mai mare energie de ieșire este, prin urmare, nu o muncă utilă (de exemplu, ca
puterea la arborele de producătorii de energie electrică), dar a căldurii reziduale din sistemul de evacuare
sau turn de răcire! Desigur, în cazul în care producția de căldură în sine poate fi folosit
benefic (de exemplu, pentru locale de termoficare), este mai pierde și în general
eficiența ar trebui să fie îmbunătățit considerabil.
Direct conversii de energie la solar FOTOVOLTAIC, hidro, ocean, și eoliană
energie pentru a lucra și de energie electrică nu au termodinamic limitări
de motoare termice. Cu toate acestea, experiența altor conversie
ineficiențe în extragerea energiei din fluxurile naturale de energie, cum este explicat în
subiect relevant capitole.
Rețineți că cel mai specific de energie pot fi furnizate servicii alternative
moduri, de exemplu, de iluminare , de la lumina directă prin ferestre și prin soare-conductele
prin acoperisuri de camere, de la lumânări și lămpi cu ulei, și, mai ales, de
lumini electrice de mai multe tipuri; transport de mers pe jos, cu bicicleta, și de electric
și un motor cu combustie vehicule. Eficiențele mai multe energie,
conversii și dispozitive individuale variază foarte mult; de asemenea, impactul acestora.
Iluminatul este un exemplu excelent de o progresie constantă tehnologice în
îmbunătățirea și lărgirea cererii. Caseta 16.2 oferă un exemplu.
Economii de energie rezulta din schimbarea activități care necesită energie
intrări, de exemplu, de cotitură pe luminile atunci când nu este necesar, mersul pe jos în loc
de a folosi o mașină, schimbarea de control pentru încălzire sau de aer condiționat pentru a
evita excesiv de încălzire sau de răcire, sau eliminarea unui anumit aparat
și efectuează o sarcină în mai puțin consumatoare de energie modul. Economiile de energie
pot fi realizate de către tehnice, organizatorice, instituționale, structurale și
schimbări de comportament.
Creșterea eficienței energetice a serviciilor poate reduce primare
de energie necesară la toate formele de aprovizionare cu energie. Cu toate acestea, acest lucru poate fi
deosebit de important pentru energia din surse regenerabile în cazul ofertei disponibile este limitat
(de exemplu, de uz casnic-micro generație de module solare fotovoltaice
TWIDELL PAGINARE.indb 572
01/12/2014 11:38
§16.2 servicii Energetice
573
CASETA 16.2 IMPACTUL TEHNOLOGIEI SCHIMBARE ÎN ILUMINAT ÎN ANGLIA, 1500-2000
Într-o lucrare de istorie economică, Fouquet (2008) au analizat tranziții între tehnologiile energetice.
El a urmărit servicii energetice în Anglia peste 500 de ani de la 1500 până în 2000, și a examinat principalele
tehnologii și sursele primare de energie utilizate. El a analizat prețurile (în termeni reali, adică ajustat pentru inflație),
eficiență și cum schimbările socio-economice afectate de utilizarea unor astfel de servicii. El a făcut acest lucru separat pentru
încălzire, iluminat, putere mecanică și transport; vom rezuma aici concluziile sale pentru iluminat.
Lumânări făcute din seu (grăsimi animale) au fost dominante tehnologie pentru iluminat pe timp de noapte și a rămas
în esență neschimbată de secole (Fig. 16.2). Un impozit pe lumânări în 1820 solicită lumânare factorii de decizie să utilizați
îmbunătățirile tehnologice (de exemplu, fitile împletite) pentru o eficiență îmbunătățită, astfel încât utilizatorii au mai multă
lumină de la
cel mai scump produs. Iluminat de la producator-gaze (monoxid de carbon și hidrogen, cea mai mare parte din
cărbune, dar, de asemenea, biomasă) a fost introdus la acel moment, dar a fost scump și nu a venit în
CASETA 16.1 EFICIENȚĂ MAXIMĂ DE MOTOARE TERMICE
Un motor termic este un dispozitiv care transformă căldura în lucru printr-un proces ciclic. Eficiența motorului este
dat de (16.1), cu intrare de energie fiind conținutul energetic al combustibilului utilizat, și producția de energie
mecanică puterea de ieșire a motorului.
Teoretic eficiență maximă a unui motor termic este prezentat în manualele de termodinamicii să fie
că de o Carnot motor, care este:
T
T
T
(
)/
h
h
Carnot
c
η
=

(16.4)
în cazul în care T
h
și T
c
sunt, respectiv, temperaturi minime și maxime ale fluidului de lucru în timpul
ciclu. (Rețineți că fluidul de lucru într-un motor pe benzina sau diesel este amestecul aer/combustibil în cilindru.)
Complet termodinamic argumente care să justifice o astfel de teorie sunt prezentate în manuale și acceptabil
site-uri de referință la sfârșitul acestui capitol. Carnot teorie este de o conceptuale valoare de căldură
motoare (de exemplu, căldura nu poate fi transformată în totalitate să lucreze și mai fierbinte intrare, mai eficient
motor), dar acest "simplu" teorie a intrinseci limitări cantitative. De exemplu, teoria necesită
rever bil schimbări cu zero frecare și de timp infinit; ambele sunt complet nerealiste în practică. Maxim
de putere teoreme a aborda astfel de probleme, dar sunt dincolo de sfera de aplicare a acestei cărți. Vezi Chen et al. (1999) pentru
mai multe detalii.
În practică, randamentul real motoare este întotdeauna mai mică decât jumătate din
η
Carnot

, adică între aproximativ


35% și 50% eficient de combustibil la arbore putere pentru motoarele cu ardere internă, cu restul de
energie fiind emise de căldură de nici o valoare într-un vehicul, altul decât pentru confort de încălzire în condiții de frig.
Diesel motoarele de compresie funcționează la temperaturi mai mari decât motoarele cu aprindere prin scânteie; prin urmare, lor
intrinsecă eficiența este mai mare. Cu toate acestea, cele mai recente motoare cu aprindere prin scânteie cu injecție de
combustibil, etc. poate
fi la fel de eficient de combustibil ca motoarele diesel.
beneficia de controlat de comutare electrice end-utilizarea de dispozitive). De asemenea,
de aprovizionare cu biocombustibili este inadecvat acum la putere toate zilele vehicule, dar
dacă toate vehiculele avut eficiență a consumului de combustibil de cea mai bună demonstrație pe modele,
o mult mai mare fracțiune ar putea rula pe biocombustibili. De distribuție a energiei electrice și
de management este simplificat și sistemul de echilibrare costurile sunt mai mici dacă
cererile de energie devin mai mici (Synapse 2008). Importanța
end-eficiența utilizării în clădiri pentru regenerabile tehnologia de optimizare este
considerat în §16.4.
TWIDELL PAGINARE.indb 573
01/12/2014 11:38
574
Utilizarea eficientă a energiei
utilizarea pe scară mai largă până pe la 1850. Deși gaz de iluminat-a dat mult mai multă lumină (~0.07 lumen / W)
1

pentru o
anumită cantitate de energie primară decât lumânări (~0.03 lumen / W), iluminare cu gaz a fost inițial mult mai
costisitoare și cea mai mare parte limitată la conducte consumabile în orașe și orașe. În consecință, primii utilizatori au fost
bogate gospodării și fabrici mari de Revoluția Industrială, de asemenea, folosind cărbune motoare cu abur
pentru puterea mecanică. Ca prin conducte de distribuție și de infrastructură îmbunătățită, mai mulți clienți au fost
conectate și prețul unitar de aprovizionare cu energie a scăzut ca urmare a economiilor de scară. Deși
prețul de gaz combustibil (energie primară) a scăzut între 1850 și 1900 de numai un factor de 2, (a)
prețul pe unitatea de lumina a scăzut cu un factor de 10, ca urmare a perfecționării tehnice în mantii, etc. care
amplificat de performanță la ~0,5 lumeni / W; și (b) de utilizare a crescut cu un factor de 50 (ajutat de
companii de închiriere de echipamente de la mai sărace clienții care nu-și putea permite să-l cumpere definitiv). Un similare
povestea se aplică pentru iluminat electric introdus în jurul anului 1880; îmbunătățiri continue în tehnologia și
infrastructura de aprovizionare a avut loc înainte de corpuri de iluminat cu energie electrică a devenit dominant în jurul anului
1920. Mai mult decât atât,
îmbunătățiri continue la fel de solid-state diode emițătoare de lumină (Led-uri, cu ~100 lumeni/W) înlocuiți
vacuumtube incandescente (~10 lumeni / W) și de iluminat fluorescent (~40 lumeni / W).
Din acest exemplu, aflăm că:

Pe scară largă acceptare a schimbărilor tehnologice dureaza ~20 la 30 de ani.

Ca tehnică crește eficiența per unitate de serviciu (de exemplu lumina), astfel încât nu există o creștere în numărul de
utilizatori; astfel, deși un anumit consum poate reduce, național total energie consumată poate
nu scade, ci crește.

Tehnologie și de fabricație (de exemplu, lumini) devin tot mai eficiente și sofisticate cu timpul ca
piețe de creștere.
Fig. 16.2
Progresia de servicii de iluminat în Anglia în timpul perioadei de la 1500 la 2000,
indicând dominantă tehnologie de iluminare. N. B. LED – Diodă Emițătoare de Lumină.
Principala tehnologie de iluminare
1500
1700
1800
1600
1900
2000

Lumanari (seu)

Lumanari (îmbunătățit)

Gaslight

Kerosen

Electric (cu filament)

Electric (fluorescente)
Electric (LED)

§16.3 CONSUMULUI FINAL DE ENERGIE PE SECTOARE
Cele mai cumpărate de energie este folosit în (a) clădiri (atât rezidențiale, cât și
comerciale), (b) de transport, și (c) industrie (Fig. 16.3). Potențialul pentru
îmbunătățirea end-eficiența utilizării și/sau reducerea cererii în aceste sectoare
este acoperit în §16.4, §16.5 și §16.6 respectiv. §16.7 se concentrează pe
sectorul rezidential (de exemplu, energie utilizate în case), cu un accent special pe
aparate electrice, ca aceasta este una dintre zonele cele mai deschise la acțiune de către
persoane fizice.
TWIDELL PAGINARE.indb 574
01/12/2014 11:38
§16.3 consumului final de Energie pe sectoare
575
Fig. 16.3
o
Lumea consumului de energie primară de sector de utilizare finală. Aceste date sunt pentru anul 2008 (total = 531 EJ)
dar procentele se schimba doar încet de-a lungul timpului. Notă în mare parte pierdut în
generarea și distribuția de energie electrică.
b
Cererea de energie de serviciu. Aceste date sunt pentru marea BRITANIE în 2010, dar procentele sunt
similare pentru cele mai multe țări în zonele cu climă temperată și de a schimba doar încet de-a lungul timpului.
Sursa: Date pentru (a) de la NOI-EIA International Energy Outlook 2011; datele pentru (b) din marea BRITANIE Departamentul de
Energie și schimbări Climatice, Consumul de Energie din marea BRITANIE (actualizare 2012).
Comerciale
6%
Industriale
38%
Rezidential
10%
Transport
19%
De energie electrică
pierderile
27%
(a)
Spațiu de căldură
23%
Alte caldura
14%
Iluminat
6%
Transport
29%
Alte
28%
(b)
Energiei din marea BRITANIE, cererea de servicii (2010)
CUTIE 16.3 REZUMAT AL RE APLICAȚII ÎN ANUMITE SECTOARE DE UTILIZARE FINALĂ
Clădiri (inclusiv rezidențiale)
Micro-generarea energiei electrice cu celule fotovoltaice (§5.3.2).
Solare de încălzire a apei (Capitolul 3).
Solare pasive de proiectare (pentru încălzire și răcire) (§16.4).
Active solare de încălzire a spațiului (§16.4).
Solare Active de răcire (§4.4).

Biomasa pentru gătit și încălzire a spațiului (§10.3).

Transport
Biocombustibili lichizi (pentru vehicule) (§10.6.2, §10.9).
Vehicule electrice (§15.11) și de baterii (§15.7).
Hidrogen-alimentat de vehicule (dacă H
2

produs de RE) (§15.11).

Industria
Hidroelectricitate pentru topire aluminiu (Capitolul 6).
Un amestec de combustibili
Industrii arderea cărbunelui sunt capabili să includă biomasă pentru 'co-ardere în brute de combustibil se amestecă
fără mare dificultate, dar cu ajustări importante pentru fluxul de aer și de control al arderii (§10.3).
În mod similar, biogaz (§10.7) poate fi amestecat în conducte livrările de "gaze naturale" și biocombustibili lichizi amestecat cu
combustibilii petrolieri, mai ales ca motorina (§10.9).
Potențialul de implementare de furnizare de energie regenerabilă în fiecare dintre
aceste sectoare este rezumată în Caseta de 16.3, inclusiv referințe pentru mai multe
detalii în tehnologia capitole din această carte.
TWIDELL PAGINARE.indb 575
01/12/2014 11:38
576
Utilizarea eficientă a energiei
§16.4 CONSUM REDUS DE ENERGIE (SOLAR) CLĂDIRI
§16.4.1 concepte Generale
Păstrarea clădirilor cald în timpul iernii și se răcească în vara, conturi pentru
aproximativ un sfert din necesarul de energie din mai multe țări (de exemplu,
marea BRITANIE: Fig.16.3(b)). Prin urmare, proiectarea și adaptarea clădirilor pentru a utiliza
energia solară reduce costurile recurente și dispare cantități semnificative
de combustibili fosili, și, de asemenea, de obicei, îmbunătățește confortul și bunăstarea. Această
secțiune consideră că proiectarea și construcția de eficiente energetic,
solarfriendly, confortabil și cost-eficiente clădiri care ar trebui să fie un
aspect esențial al arhitecturii moderne. Cele mai bune rezultate nevoie de un sistem integrat de
abordare, optimizarea (a) energia solara date de intrare pentru caldura solara si umbrire
pentru răcire; (b) masa termică și izolare; (c) căldura internă
câștiguri în construirea de aparate și metabolice termic al
ocupanților; (d) alte surse regenerabile, dacă este necesar și disponibil (de exemplu, biomasa
încălzire); și (e) aspecte de control, atât activă și pasivă. Mai mult decât atât,
clădirea ar trebui să fie vizual atractiv, confortabil și de stimulare a
trăi în. Atent site-specific de energie design necesită creative și
inovatoare de arhitectură, care, de obicei, duce la stimularea design.
Confort termic în condiții climatice diferite necesită aceste
principii să fie aplicate în diferite moduri, în funcție dominantă
este nevoie de încălzire sau de răcire, și, de asemenea, pe predominante umiditate. Ne
uităm mai întâi la cerințele pentru încălzirea spațiului, deoarece acesta este cel mai mare
consum de energie pentru clădiri, nu în ultimul rând pentru că cele mai multe dintre țările mai bogate
din lume, bucurați-vă de un "moderat" climatul în care ocupantul
supraîncălzire este rareori o problemă. Succesive mai târziu subsecțiunile uita-te la exemple de
clădiri eficiente energetic pentru zonele cu climă rece, climă temperată, caldă, uscată
climate și cald, climat umed.
Deși acest capitol se concentrează pe direct consumul de energie, sustenabilitatea
parametri pentru clădirile nu sunt doar consumul de energie al ocupanților.
De exemplu, energia înmagazinată sechestrat în fabricarea
de componente de construcție și în construcții este important (a se vedea
discuțiile de dezvoltare durabilă în Capitolul 1 și al costului ciclului de viață'
în Capitolul 17). De asemenea, de energie electrică și termică micro-generație la
site-ul ar trebui să fie luate în considerare (de exemplu, prin fotovoltaice tablouri pe pereți sau
acoperiș: vedea, de exemplu, Fig. 5.8, Fig. 16.6(d) și de turbine eoliene mici: de ex.
§8.8.6), ca un pas spre a face construirea de auto-suficiente (de exemplu, nu
epuizează resursele din afară) și, într-adevăr, un furnizor de energie prin intermediul
rețelelor.
§16.4.2 Spațiu de încălzire: principii
O utilizare importantă de energie pentru încălzirea clădirilor în perioadele reci, care cu siguranță
includ, de obicei iarna, dar poate include, de asemenea, serile reci în caz contrar
perioadele calde. Confortul depinde de temperatura aerului, umiditate, a primit
TWIDELL PAGINARE.indb 576
01/12/2014 11:38
§16.4 consum redus de Energie (solar) clădiri
577
flux de radiații, viteza de mișcare a aerului, îmbrăcăminte, și fiecare persoană de activitate,
metabolismul si stilul de viata. În consecință, în interior (camera) temperatura T
r

poate fi considerat confortabil în intervalul de aproximativ 15°C la 25°C.


intern mediul construit ar trebui să fie la o temperatură de confort'
în timp ce folosind minimul artificiale de încălzire sau de răcire (P
boost
), chiar și atunci când
extern (ambiant) temperatura T
o

este bine în afara intervalul de confort.


Echilibrul termic din interiorul unei clădiri cu solar de intrare este descris
prin ecuații similare cu (3.1). Cea mai simplă formulare consideră solare
câștig și concentrate parametrii de masă m, capacitate de căldură specifică c și
întreaga structură rezistență termică R (unitate: W/K), ca și în Revizuirea 3 (Cutie R3.1).
Rețineți că aici R nu este 'R-value' (unitate: m
2
K/W) utilizate în clădire
comerț (a se vedea Caseta R3.1).
Din conservarea energiei:
mc
T
t
GA P
T
T
R
d
d
(
)
r
o
r
boost
ta
=
+


(16.5)
Detaliate modelarea matematică a unei clădiri este mai complex și este
realizată cu specialist pachete software. Cu toate acestea (16.5),
conține baza de toate aceste modelare, și anume energia fluxurilor de căldură și
a capacităților.
Cele mai bune rezultate sunt obținute de către permițându-pentru energie considerații
de proiectare și construcție. Acestea includ următoarele:

Potrivit orientarea clădirii (cu ferestre, sere si
alte spatii vitrate care se confruntă Ecuator pentru a prinde soare în timpul iernii
, dar cu nuante pentru a atenua verticale nedorite solare de intrare în vară).
Încorporarea unor astfel de site-caracteristici specifice, de asemenea, face clădirilor
din punct de vedere arhitectural interesant (a se vedea, de exemplu, Fig. 16.6).

Optimă geam și construcția de ferestre (geamuri duble sau mai bine în
climate mai reci, cu ierni reci).

Având în vedere temperatura la sol (care rămâne aproape constantă
pe tot parcursul anului la o adâncime de aproximativ 2 m) și nevoia de amplu
prin pardoseală izolație, care este ieftin.

Izolarea la exterior și interior mare masă termică, care oferă
stocare a energiei și pentru a evita variație de temperatură de zi cu zi, dar poate limita
dimensiunea camerei la unele site-uri.

De mai sus-izolare tavan, care este ieftin, dar poate limita spațiul pod.

Acoperiș suprafața exterioară, care pot include iarba de pe acoperișuri, solar electric (PV)
și solare termice (de încălzire a apei) panouri, și suprafețe reflectorizante.

Ploaie de captare a apei (alimentare cu apă este, de altfel, mari consumatoare de energie
și costisitoare).

Interne zilei de windows și soarele-conducte.

Etanseitatea, încă de ventilație; forțat extract de ventilație din bucatarii,
bai, dusuri, toalete, etc.

Calcul, probabil, cu software de specialitate, termice și
TWIDELL PAGINARE.indb 577
01/12/2014 11:38
578
Utilizarea eficientă a energiei
lumina zilei caracteristici, inclusiv 'energie gratuită câștiguri de gătit,
dispozitive alimentate și metabolismul.

Oportunitățile de pe site-ul de micro-generarea de energie electrică și termică.
Guvernele oferă îndrumări privind energia caracteristici necesare, în special
la nivelurile minime de izolare, prin intermediul codurilor de construcție, și
reglementări pentru a asigura respectarea acestora (a se vedea Caseta 16.4 pentru un exemplu).
CUTIE DE 16,4 CODURI DE CONSTRUCȚIE

Marea BRITANIE are coduri de la 1 la 6 (cel mai bun) pentru clădiri. Mai jos este codul 6 rezumat care, în practică, poate fi

cunoscut cu clădiri noi.

Marea BRITANIE Codul de Nivelul 6 standard pentru clădiri

De acasă trebuie să fie complet de carbon zero (adica zero net emisiile de CO


2

din tot consumul de energie în


casă), ca atins de toate sau unele dintre următoarele măsuri:

Folosind scăzut și zero de dioxid de carbon, cum ar fi panouri solare termice, fotovoltaice, micro-generare,
cazane pe biomasă, eoliene, și combinate de căldură și sisteme de energie (CHP).

Îmbunătățirea eficienței termice a pereților, ferestre și acoperiș.

Reducerea permeabilitatea la aer la minim, în concordanță cu cerințele de sănătate.

Instalarea de înaltă eficiență cazan în condensație pentru încălzire, sau de a fi pe un sistem de termoficare.

Cu atenție de proiectare tesatura de domiciliu, pentru a reduce puntile termice (de exemplu, la acoperiș margini, colțuri de
ziduri).

Folosiți nu mai mult de aproximativ 80 de litri de apă pe persoană pe zi, inclusiv de aproximativ 30% pentru non-potabilă
apa de la recoltarea apei de ploaie și/sau sisteme de reciclare a apei gri.

Materiale – resurse reduse de impact.

Maxim, accesibil dispoziție pentru reciclare internă de alimente și materiale de deșeuri.

Aparate eficiente energetic și de iluminat.

Îmbunătățirea iluminatului natural, izolare fonică și de securitate.

Evaluarea și minimizarea impactului ecologic al construcției.
Passivhaus standard pentru clădiri cu ultra-redus de energie utilizarea
Passivhaus este un riguros standard voluntar pentru eficiența energetică în clădiri, reducerea sale ecologice
amprenta. Aceasta duce la ultra-redus de energie clădiri care necesită mai puțină energie pentru încălzire sau răcire.
Mai mult de 10.000 de astfel de clădiri au fost construite în Germania și Scandinavia. Passivhaus
standarde sublinia superinsulation, triplu vitraj, avansate de tehnologie windows de specialitate cu acoperire
și umplere, solar câștig de atenție orientată spre geam, de ventilație cu recuperare de căldură de la "liber" câștigurile de căldură
(de exemplu.g. de lumini electrice și dispozitive, și de gătit), etanseitatea, și mulți alți factori integrate
de proiectare și excelente abilități de construcție.
Vezi www.planningportal.gov.uk/uploads/code_for_sust_homes.pdf.
Vezi http://en.wikipedia.org/wiki/Passive_house.
§16.4.3 solare Pasive clădiri
Solare pasive de proiectare în toate climatele constă din aranjarea catalogheaza
construirea masei m, soare-cu care se confruntă zona O și pierderea de rezistență R de (16.5)
pentru a realiza optim solare beneficia de proiectare structurală. Primul pas este
TWIDELL PAGINARE.indb 578
01/12/2014 11:38
§16.4 consum redus de Energie (solar) clădiri
579
pentru a izola clădirea bine (mare R: izolare este ieftin),
inclusiv pescaj prevenirea și, dacă este necesar, ventilație controlată cu căldură
de recuperare. Orientarea, dimensiunea și poziția de ferestre și
sere ar trebui să permită o cantitate suficientă de produs de GA (perpendiculare pe geam)
pentru semnificativă pasive de încălzire solară în timpul iernii, cu activ și pasiv
umbrire preveni supraîncălzirea în timpul verii. Ferestrele ei înșiși
ar trebui să aibă avansate, multi-suprafață de construcție, astfel încât rezistența la
transferul de căldură, altele decât unde scurte de intrare radiația solară, este mare.
Pentru solară pasivă clădiri la latitudini mai mari, căldură solară în timpul iernii
este posibil deoarece expunere la soare pe verticală spre soare geamuri și pereți
este în mod semnificativ mai mult decât pe suprafețe orizontale: a se vedea Fig. 2.18. La
sunfacing interne masă suprafețe ar trebui să aibă o culoare închisă, cu
o > 0.8 (Fig.
16.4(a)) și clădirea ar trebui să fie proiectate pentru a avea o masă mare de
interior, pereți și pardoseli (mare m) pentru stocare de căldură în izolație,
limitând astfel de variații în T
r

. Supraîncălzirea poate fi prevenită prin


montarea externe nuanțe și obloane, care, de asemenea, oferă un plus termice
de izolare pe timp de noapte. Construirea unui geamuri conservator pe soare-cu care se confruntă
părțile dintr-o clădire permite căldurii solare pentru a fi capturat, adiacente masă
a clădirii, prin urmare, oferă beneficii pentru încălzire dacă nu este controlat,
flux de aer (de exemplu, prin usi). Cu toate acestea, astfel de spatii vitrate oscila în
temperatura rapidă cu și fără soare, atât de activ sau pasiv
de ventilare este esențial. Sere ar trebui să fie utilizat atunci când
condițiile sunt confortabile, și nu sunt utilizate în caz contrar; este sărac practică de a
instala încălzire, de răcire sau în sere.
A lucrat Exemplu 16.1 ilustrează faptul că cele mai multe dintre sarcina de încălzire a unui
bine-proiectat casa poate fi de la energia solară, dar designul de
practică sistemele solare pasive este mai dificilă decât cea exemplu sugerează.
Mai recent și mult mai sofisticate casa ('Meridian Prima
Casa de Lumină') pentru condiții similare este prezentată în Fig.16.6(b). De exemplu,
Fig. 16.4
Direct câștig de încălzire solară pasivă: notă de orientare și masiv de culoare închisă suprafețele
de a absorbi și stoca expunere la soare. Rețineți importanța de orientarea clădirii și de a folosi de
masiv, de culoare închisă, spate-izolat suprafețele de a absorbi și de a stoca radiații.
o
sistem de bază;
b
lucarna fereastră (pentru a da direct câștig pe peretele din spate al casei).
(a)
(b)
TWIDELL PAGINARE.indb 579
01/12/2014 11:38
580
Utilizarea eficientă a energiei
A LUCRAT EXEMPLU 16.1 CĂLDURĂ SOLARĂ DE O CASA
La Solar de Casa Negru' se arată în Fig. 16.4(a) a fost conceput în anii 1980, ca o demonstrație pentru
Washington, DC (latitudine 38°N). Acesta dispune de o fereastra mare pe partea de sud și un masiv înnegrite
de perete pe partea de nord. Presupunând că acoperișul și pereții sunt atât de bine izolate ca toate pierderile de căldură prin
fereastră, a calcula radiația solară necesară, astfel încât încălzire solară directă singur menține camera
la temperatura de 20°C peste temperatura ambiantă.
Soluție
Dacă temperatura camerei este constantă, (16.5) se reduce la:
G
TT
r
r
o
ta =

unde r este termic de rezistivitate din încăperea în afară de o fereastră verticală, cu un singur geam. Prin
metode de Revizuire 3 și Capitolul 3, (a se vedea, de asemenea, Problema 16.2)
r
= 0.07 m
2
KW
-1
Lua paharul de transmisie
τ = 0,9 și perete absorptance o = 0.8, atunci:
G
20 C
(0,07 m KW ) (0.9)(0.8)
400Wm
0
2
1
2
=
=


Această radiație poate fi de așteptat pe verticală spre soare fereastră într-o zi senină de iarnă.
A LUCRAT EXEMPLU 16.2 PIERDERILE DE CALDURA DINTR-O CASA
Solar Negru Casă descrise în exemplul anterior măsuri de 2,0 m de mare de 5,0 m lățime de 4,0 m
adâncime. Temperatura din interior este de 20°C la 4 p. m. Calculați temperatura din interior la 8 dimineata a doua zi
pentru următoarele cazuri:
(a) Absorbție de perete 10 cm grosime, singură fereastră ca și mai înainte.
(b) de Absorbție de perete de 50 cm grosime, perdea groasă care acoperă interiorul ferestrei.
calculul arată doar că casa prezentate în Fig. 16.4 va fi
încălzite adecvat în mijlocul zilei, dar de căldură trebuie să fie, de asemenea,
reținute la noapte și trebuie să existe un schimb de aer pentru ventilație.
A lucrat Exemplu 16.2 arată importanța m și R în căldură
echilibru, și, de asemenea, importanța de a face părți ale casei
reglabilă să recunosc căldură de zi în timp ce închiderea în noaptea (de exemplu, perdele,
jaluzele).
Un dezavantaj serios de simplu direct de câștig sisteme este că
clădirea poate fi prea cald în timpul zilei, mai ales în timpul verii; astfel
supraîncălzirea este prevenită sau redusă prin umbrire de la lat acoperiș consolele,
obloane și nuanțe ('orbeste'). Confort îmbunătățit și o mai bună utilizare a
căldurii solare se realizează prin creșterea stocare de căldură al clădirii
în izolare prin creșterea internă termică a masei (strict,
TWIDELL PAGINARE.indb 580
01/12/2014 11:38
§16.4 consum redus de Energie (solar) clădiri
581
termice de capacitate C = mc), cu ziduri groase și parter de
piatră cu sub-podea izolare, dens beton sau dens de cărămidă. Dacă solare
și alte fluxurile de căldură sunt controlate în mod corespunzător, mare termice interioare
de masă este întotdeauna benefică pentru confort în ambele rece și zonele cu climat cald.
Cu toate acestea, având ziduri groase și foarte groase de izolare crește inițial de
cost și poate reduce spațiul util dacă site-ul este constrâns.
§16.4.4 solare Active construirea de sisteme de
Un spațiu alternativ-metoda de incalzire pentru confort de construcție este de a folosi
extern (separat) colectoare, încălzire cu aer (§4.2) sau apă (Capitolul 3) într -
un activ sistem solar, unde căldura este trecut la construirea în țevi
sau conducte. Pe baza de apa, pentru a avea sisteme schimbătoare de căldură (de exemplu, radiatoare')
pentru a incalzi camere, și aer-sisteme bazate pe nevoie substanțială de conducte. În
orice caz, o mare de stocare a căldurii este necesar (de exemplu, materialul de construcție, sau o piatră
de pat pat în subsol, sau un rezervor mare de apa; a se vedea §15.10). Un sistem de
Soluție
Cu G
= P
boost
= 0, (16.5), se reduce la:
T
t
T
T
RC
d
d
r
r
o

(
)
=−

(16.6)
în cazul în care termică capacitate este: C
= mc, iar c este capacitatea termică specifică
Soluția este:
T
T
T
T
t RC
exp[ / (
)]
r
r
t
o
o
0

(
)
−=


=
(16.7)
presupunând că T
o
este constantă. Aici produsul RC este constanta de timp, fiind timp pentru temperatura
diferența să scadă la 1/e (
= 1/2.72 = 37%) din valoarea inițială.
Ca și înainte, își asumă toate pierderile de căldură prin fereastră, de zona de 10 m
2
. Presupunem absorbant de perete este fabricat
de beton, cu datele din Tabelul B. 3.
(a)
R rA
C mc
RC
0.007 KW
(2.4 10 kgm )(2m) (5m) (0.1 m)] (0.84 10 kg K)
2.0 10 JK
14 10 s 3.8 sec
1
1
3
3
3
1
1
6
1
3
=
=
=
=
×
×
=
×
=
×
=





După 16 ore, temperatura excesul de mai sus ambiant este de


(20°C) exp (– 16/3.8)
= 0.4°C.
(b) Presupunând cortina este echivalent cu geamuri duble, presupun
r
≈ 0,2 m
2
KW
-1
(din Tabelul 3.1). Prin urmare:
R
= 0.02 K W
-1
C
= l0 x 10
6
JK
-1
RC
= 2.0 x 10
5
s
= 55 sec
T
r
– T
o
= (20°C) exp (– 16/55) = 15°C
TWIDELL PAGINARE.indb 581
01/12/2014 11:38
582
Utilizarea eficientă a energiei
pompele sau ventilatoarele sunt necesare pentru a circula fluidul de lucru, care este mai ușor de
controlat decât pur sisteme pasive, și poate, în principiu, să fie montate la
case existente. Cu toate acestea, colectorii trebuie să fie mari și retehnologizare
este de obicei mult mai puțin satisfăcătoare decât corecte de proiectare la construcție inițială.
Astfel de sisteme pasive, sisteme solare active va funcționa bine doar dacă căldură
pierderile au fost reduse la minimum. În practică așa-numitele case pasive sunt
mult îmbunătățite cu ventilatoare electrice controlate pentru a trece de aer între camere
și căldură magazine. Astfel, termenul "pasiv" tinde să fie folosit atunci cand
caldura Soarelui este prima prins în camere sau sere spatele windows,
chiar dacă ventilație controlată este utilizată în clădire. "Activ" tinde să fie
folosit în cazul în care căldura este prima prins într-un scop-construit exterior colector.
Analiza pentru case reale este complicată din cauza complexului
absorbant de geometrie, de transfer de căldură prin pereți, prezența unor
oameni în casă, și considerabile gratuit câștiguri de iluminat,
gătit, etc. Oamenii reglaje independente, cum ar fi deschiderea
windows sau desen perdele, care nu poate fi ușor de prezis. În
plus, metabolismul lor contribuie în mod semnificativ la echilibrul termic de o
energie conștientă clădire cu 100 la 150 W per persoană în
termenul
P
boost

de (16.5). Un număr rezonabil de schimburi de aer (între una și


trei pe oră) sunt necesare pentru ventilație, iar acest lucru va produce, de obicei,
semnificative
pierderea de căldură cu excepția cazului în schimbătoare de căldură sunt montate. Calculator
programe, cum ar fi Energy plus (SUA) și de performanță metode de evaluare
, cum ar fi BREEAM (marea BRITANIE) sunt concepute pentru a evalua interacțiunile dintre
toți factorii care afectează performanța energetică a unei clădiri și sunt
utilizate pe scară largă, dar este în continuare esențială pentru analiști să aprecieze
importanța efectelor individuale prin simplificată, ordin de mărime,
calcule, cum ar fi cele din Exemple 16.1 și 16.2.
§16.4.5 zonele cu climă Rece
În mod distinct zonele cu climă rece, în cazul în care dominante problemă este lipsa de încălzire,
și chiar și cele mai bune de energie conștientă clădiri va avea nevoie de unele active
încălzire, principala preocupare este de a minimiza pierderile de căldură. Un compact clădire de
formă, care minimizează suprafață / volum, este de dorit, deoarece sunt
mulți factori menționați mai sus.
Inuiții igloo, din zăpadă, exemplifică minim
de suprafață-tovolume raport și acumularea de căldură din metabolice și de viață activitatea de
locuitori (Fig. 16.5(a)).
Mai multe clădiri moderne au o gamă mai largă de construcții
materiale disponibile, și pot folosi solare pasive obține de la Ecuator-cu care se confruntă
windows, cu condiția ca fereastra să fie bine protejate împotriva pierderilor de căldură
pe timp de noapte (de exemplu, prin dublu sau triplu vitraj și/sau perdele), grosime perete
de izolare (de exemplu, r
= 4m
2

K/W sau mai mare; se referă la Cutie R3.1) și masă termică pentru a
stoca căldura de intrare (a se vedea Exemplul 16.1). Etanseitatea a
anvelopei clădirii este important în minimizarea pierderilor de căldură, dar inadecvate
TWIDELL PAGINARE.indb 582
01/12/2014 11:38
§16.4 consum redus de Energie (solar) clădiri
583
Fig. 16.5
Clădiri potrivite pentru un climat rece.
o
Un sat Inuit în 1865: iglu făcut din zăpadă.
Sursa: reprodus din C. F. Hall, Arctic Cercetări și Viața Printre
cele Esquimaux, Fratii Harper, New York (1865).
b
La Minto Roehampton clădire cu apartamente în centrul orașului
Toronto. Finalizat în 2007, acesta a fost primul multi-rezidențiale
clădiri pentru a realiza LEED-Canada Aur de certificare.
Sursa: Fotografie prin amabilitatea de UrbanDB.com.
(a)
(b)
ventilația poate duce la acumularea de nedorite de gaze de la unele
materiale de constructii.
La Minto Roehampton multi-clădire rezidențială din centrul orașului Toronto
(Fig. 16.5(b)) utilizeaza aspectele legate de energie-design conștient am
considerat mai sus, cu energie solară pasivă, să se incalzeste proaspete coridor
aerian. Acesta a fost proiectat pentru a fi cu 40% mai eficiente energetic decât Canadian
Modelul Național de Energie Cod pentru Clădiri la acel moment. O recuperare de căldură
sistem de ventilație oferă proaspăt, filtrat de aer la fiecare suită și circulă
aerul proaspăt pe tot parcursul suites. O economisire a energiei de toate-off switch' instalat
în fiecare din cele 148 de apartamente permite rezidenților să dezactivați toate plafonul
lumini și ventilatoare de evacuare de la un comutator ca aceștia să plece. Opac pereții
au termică mare rezistivitate ('R-valoare'
= r ) cu r = 15m
2
K/W, și
windows r
= 0,3 m
2
K/W fără perdele. (A se compara cele mai mici valori
ale r pentru "standard" pereți și ferestre în Tabelul B. 7 din Apendicele B.) La
complexul include, de asemenea alte caracteristici pentru a face mai "durabilă"
decât practica comună: ușor de mers pe jos distanta de magazine și facilități,
acoperite de biciclete de stocare, și o auto-piscină program pentru rezidenți (a se compara
§16.5), aparate eficiente energetic (§16.7), o riguroasă a deșeurilor
sistemul de gestionare și management atent al aprovizionării cu apă.
TWIDELL PAGINARE.indb 583
01/12/2014 11:38
584
Utilizarea eficientă a energiei
§16.4.6 climă Temperată
În "moderat" climate (de exemplu, cele mai multe din Europa), în condiții de iarnă
se apropie de cele pentru zonele cu climă rece, dar pentru perioade mai scurte de un an.
Soluții de construcție trebuie să permită atât iarna cât și vara condiții
(de exemplu, orice mare Ecuator-cu care se confruntă windows pentru iarna încălzire solară poate
cauza supraîncălzirea în timpul verii, astfel, necesită umbrire corespunzătoare streașină,
la fel de vizibil în Fig.16.6(b) și (d)). Umbrire unghiuri pot fi calculate din
formulele prezentate în Capitolul 2, sau obținute în diagrame pentru arhitecți, ca
în Szokolay (2008). În multe climă temperată, temperaturile de noapte
chiar și în timpul verii sunt de multe ori mai jos de "confort", nivele, deci un termica mare
capacitate de construcție poate fi de preferat. De exemplu, termică de timp
constantă de încălzire solară în masă-perete, așa cum se arată în Exemplul 16.2,
pot fi proiectate pentru a se potrivi diferența de timp între maxim
solar de intrare și de încălzire atunci când este nevoie. Izolatia este mandatat în UE
țări și în cele mai multe state din statele UNITE ale americii prin construirea codurile corespunzătoare
pentru fiecare regiune (a se vedea Caseta 16.4). Din păcate, astfel de standardele de construcție națională
de obicei, rămâne mult în urma standardelor de cunoscute cele mai bune practici, deoarece
clădirea industrii sunt conservatoare și teama creșterea costurilor de construcție.
Costurile pe termen lung ale viitorului ocupanților nevoile energetice sunt de multe ori
luate în considerare în serios.
Fig. 16.6 prezintă fotografii și contururi ale unor solar-conștient
clădiri pentru a da o idee de arhitectură varietate și oportunități
pentru stimularea design. Fig. 16.6(a) prezinta un mare complex de student
cazare în Scoția, oferind izolatie transparent și
shadeblinds peste ziduri și ferestre cu blind-uri individuale. Transparent
de izolare permite amplificarea energiei solare în timp ce încă păstrând căldura produsă în interiorul
clădirii. Sud fețe ale clădirii au lunar un câștig net
de energie în clădire pe tot parcursul anului, chiar și în mijlocul iernii, în
Glasgow.
La 'Prima Casa Lumina' în Fig. 16.6(b) a fost conceput pentru a satisface
cerințele de la Solar Decathlon (Cutie 16.5) pentru un preț accesibil,
energyefficient locuință de familie. Acesta oferă o piscină interioară top-geamuri "punte" (mijloc
de fotografie), care acționează ca un gătit și spațiu de luat masa, și o punte
de dormit și domenii de studiu. Anvelopa clădirii este extrem de izolate, dar este
, de asemenea, flexibil la condițiile climatice, cu obloane glisante pentru a maximiza
sau reducerea supraîncălzirii după cum este necesar. Extern lemn baldachin locuințe
panouri fotovoltaice și solare de încălzire a apei oferă o soluție elegantă
de multe ori greoaie integrarea de panouri solare. Bolta forme
partea de active și pasive solare strategie, umbrirea geamurilor în
lunile de vară și complicitate la răcire pasivă de panouri FOTOVOLTAICE. La
casa integrează aparate eficiente energetic, inclusiv de iluminat cu LED și
un revers-ciclul pompei de căldură cu un coeficient ridicat de performanță. La
utilizatorii de acasă pot monitoriza și optimiza consumul de energie
cu o interfață intuitivă acasă-sistem de monitorizare.
TWIDELL PAGINARE.indb 584
01/12/2014 11:38
§16.4 consum redus de Energie (solar) clădiri
585
Fig. 16.6
Patru clădiri potrivite pentru un climat temperat.
o
Student Solare Residences, Universitatea din Strathclyde, Glasgow, Scoția (latitudine
56°N). Fațada sudică arată transparent de izolare.
Sursa: Twidell et al. (1994).
b
'Meridian Prima Casa Lumina' construit pentru condițiile de la Washington, DC, statele UNITE ale americii (latitudine
38°N) de către studenții de la Victoria University of Wellington pentru Solar Decathlon 2011.
(Fotografie de Jim Tetro pentru Departamentul de Energie al SUA Solar Decathlon.)
Sursa: www.solardecathlon.gov.
c
Renovat bloc de birouri în South Melbourne, Australia (latitudine 38°S): fatada
(spre est),
d
detaliu din spate de aceeași clădire, arată panouri FOTOVOLTAICE utilizate pentru umbrire de vest-cu care se confruntă
windows).
Sursa pentru (c) și (d): reprodus din Baird (2010).
(a)
(b)
(c)
(d)
TWIDELL PAGINARE.indb 585
01/12/2014 11:38
586
Utilizarea eficientă a energiei
Clima din Melbourne (Australia) este cel mai cald limita de
'climat temperat, cu temperaturi de vară mare de 34°C la ~10% din
zilele de vară, dar o temperatură minimă iarna de
+8°C.
clădire de birouri prezentate în Fig.16.6(c) (d) a fost pe larg renovate
(modernizate) în 2005 să ajungă la un hotel de cinci stele rating energetic (cel mai bun la acea
dată). Fostul beton fațada a fost înlocuit cu înălțime completă, clară,
de joasă emisivitate geamuri duble, concepute pentru a maximiza de transmisie a luminii
și de a reduce căldură solară, completate de umbrire de la
steelperforated plasă vizibile în Fig.16.6(c). Spate (vest) windows au
același pahar, dar sunt umbrite de clădiri învecinate pentru o mare parte din
an. Deși clădirea este de ~5 ori cât de adânc lățimea fațadei sale
lumina naturală este maximizată prin montarea unei centrale de scara cu un luminator,
având în plan deschis birouri la ambele capete (pe fiecare din cele cinci etaje), și
geamuri câteva pereții interiori. Ventilarea naturală se realizează prin posibilitati de deschidere
de la ferestre pe ambele fațade și folosind stiva efect în open-benzii de rulare
scării pentru a trage aer pe birou și ieși prin lamele la
nivelul acoperișului. În timpul verii, acest sistem permite, de asemenea, clădirea să se răcească în jos
pe timp de noapte înainte de următoarea zi lucrătoare. Există, de asemenea, un sistem de Încălzire, Ventilare
și Aer Condiționat (HVAC) pentru perioadele când suplimentare de încălzire
sau de răcire este necesar. Toate aspectele sunt, de obicei, 'automat' controlat
de un sistem de management al clădirii. Încălzitoare de apă solare și FOTOVOLTAICE tablouri
sunt montate pe acoperiș (în plus față de matrice este prezentat în Fig. 16.6(d)).
Casele de Hockerton Locuințe de Asociere în Nottinghamshire,
Anglia sunt construite clasic solar-design conștient. Gros externe
izolare încapsulează vest, nord și est pereti; interiorul este gros,
dens bloc și convențional zid de cărămidă. Sere acoperi
CUTIE DE 16,5 SOLAR DECATHLON
Departamentul de Energie al SUA Solar Decathlon provocări echipe de colegiu pentru a proiecta, construi și
opera solar-alimentat case care sunt rentabile, eficiente energetic și atractiv. Câștigătorul
competiției este echipa care cele mai bune amestecuri de accesibilitate, apel de consum și excelență în design cu
optim producția de energie și eficiență maximă.
O echipă de obicei nevoie de doi ani pentru a proiecta și documentul de casa la "acasă" înainte de a re-montaj
pe concurență showground alături de cei de la ceilalți concurenți. Design-ul este necesar pentru a
lucra bine ca un locuința familiei pe site-ul. În mod ideal, ar trebui să fie de energie auto-suficiente, și costul <250.000 dolari
SUA,
inclusiv fitinguri și aparate.
Competiția arată consumatori cum de a economisi bani și energie cu prețuri accesibile energie curată
produse care sunt disponibile astăzi. Solar Decathlon, de asemenea, oferă studenților participanți cu mâinile-pe
experiență și unic de formare pe care le pregătește pentru a intra în energie curată forței de muncă. Solar
Decathlon a fost organizat 'o dată la doi ani' din 2005. Deschis pentru public gratuit, Solar Decathlon
oferă vizitatorilor posibilitatea de a tur case, aduna idei pentru a folosi în propriile lor case, și să învețe cum de
caracteristici de economisire a energiei poate le ajuta să economisi bani astăzi.
Pentru mai multe informații, a se vedea www.solardecathlon.org. O competiție similară este, de asemenea, a avut loc în Europa.
TWIDELL PAGINARE.indb 586
01/12/2014 11:38
§16.4 consum redus de Energie (solar) clădiri
587
sud fațade, cu ferestre interne și ușile care duc la camere.
În plus, nord zidurile sunt îngropate într-un val de pământ cu iarbă
și arbuști acoperi. (A se vedea www.hockertonhousingproject.org.uk/ pentru
informații despre activitățile prezente.)
§16.4.7 climat Cald, uscat
În cald, climat uscat temperaturile din timpul zilei poate fi foarte mare (>40°C)
, dar diurn gama este adesea mari (>20°C), astfel încât temperaturile pe timp de noapte
poate fi incomod rece (de exemplu, ~0°C în timpul iernii, în Australia centrală).
În consecință masă termică mare este cea mai importantă caracteristică pentru
o clădire confortabilă, cu masive ziduri umbrite și structuri de tavan
sub un reflectorizante acoperiș. Windows ar trebui să fie umbrită și sunt cel mai bine
păstrate închise ziua și deschise noaptea de răcire (gratii
pot fi necesare!). Suprafețele exterioare ar trebui să fie alb mat, astfel încât
să reflecte radiația solară, dar care să permită radiații infraroșii să fie emis
(de obicei clar) cerul de noapte. Temperatură la sol de aproximativ 2 m adâncime
este constantă pe an în toate climatele; în uscat și cald, climat această
temperatură este probabil să fie în jurul valorii de 20°C la 25°C. prin Urmare etaje
de clădiri ar trebui să fie într-un contact termic bun cu solul și nu
izolat. Într-adevăr, camere subterane și pivnițe poate fi termic foarte
confortabil.
Locuri cu clima asta de multe ori au un cald, plin de praf și, în general, 'ostile'
mediul exterior, deci clădiri cu un interior cu care se confruntă curte sunt
plăcute, de exemplu, așa cum este tradițional în Egipt și India de nord. La
aer în curte poate fi evaporatively răcit de un iaz sau fantana,
astfel oferind racit cu aer pentru camere adiacente, cu interior cu care se confruntă uși și
ferestre. Umbra de copaci și alte plante în curte spori
acest efect și de a crea o placuta de semi-spațiu de locuit în aer liber. În aceste uscat
clima, electric alimentat de masă și ventilatoare de tavan permite forțat
de răcire prin evaporare la nivelul pielii, astfel aducând atât în stare proaspătă de aer și de relief de bun venit
de la căldură. Instalatii de racire prin evaporare sunt la fel de bun venit, prin care externă
aer uscat este cu sufletul la gură de fani prin tampoane de libertate udate de paie și apoi în
camere, de la care aerul poate ieși. De răcire prin evaporare cu fanii necesită
semnificativ mai puțină energie electrică decât de aer condiționat cu agenți frigorifici
(~150 W, comparativ cu ~1500 W).
Fig. 16.7 prezinta un modern complex de cercetare în Gujarat (India), care
utilizează aceste principii. De energie electrică pentru aer condiționat și lumină artificială
este costisitoare și de multe ori nesigure, deci design-ul își propune să fie
energyefficient. Pasiv solar caracteristici oferă lumină naturală și ventilație,
în timp ce controlul pătrunderii prafului. Masă termică este asigurată de
beton armat construcții din materiale locale, cu cărămidă de umplutură
în pereți și gol blocuri de beton în acoperiș cuferele pentru
suplimentare de masă termică. Vermiculita este principalul material izolant unde
este cazul. Exteriorul este alb, inclusiv acoperișul. În fierbinte, uscat
TWIDELL PAGINARE.indb 587
01/12/2014 11:38
588
Utilizarea eficientă a energiei
Fig. 16.7
Climat cald, uscat zona.
Torrent Centrul De Cercetare, Ahmedabad, Gujarat, India.
o
Vedere exterioară a un bloc arată una dintre instalatii de racire prin evaporare (structura mare pe
acoperiș) și mai multe turnuri de evacuare.
b
Secțiunea transversală prezintă sistemul de ventilație.
Sursa: Foto și schița de la Baird (2010).
(a)
Laborator
Evacuare
(b)
Evacuare
Evacuare
Evacuare
De admisie
Micronisers
Birouri
TWIDELL PAGINARE.indb 588
01/12/2014 11:38
§16.4 consum redus de Energie (solar) clădiri
589
sezon, evaporatively răcit fluxurile de aer centrale de coridoare și
camere alăturate. De răcire este de apă pulverizată în turnuri prin
care aerul proaspăt intră; aceste acoperiș turnuri sunt vizibile în Fig.16.7. (a).
Mișcarea aerului este prezentată în Fig.16.7(b). Sondajele arată că
ocupanții, care sunt obișnuiți să climat cald, sunt confortabile la
construirea intern 'sezonul cald temperatura de 28 spre 29°C, cu
temperaturi de la aproximativ 40°C. mișcarea de proaspete, răcit cu aer din
de răcire prin evaporare oferă condiții plăcute la aceste interne
temperaturi, în ciuda lor fiind ușor mai mare decât temperaturile de obicei,
setat pentru acționate electric (frigider), aer condiționat.
O nouă provocare de design este cald, umed sezonul musonic în
Gujarat, când centrale de răcire prin evaporare sistemul este ineficient. Parțială
confort este obținut prin tavan și birou de fani pentru persoane fizice, în ciuda pielii
evaporare fiind redus în condiții de umiditate ridicată. În sezonului rece (cu
temperaturi exterioare ~15°C), ocupanții regla individual de windows
și de ventilație pentru a controla temperatura.
Pentru cei suficient de nefericit pentru a îndura nedorite căldură solară
într-un standard modern' cutia de sticlă clădire de birouri într-un climat cald, uscat, la
cererea de energie pentru răcire poate fi diminuat prin modernizarea
electrochromic windows (Cutie 16.6). Bine-unghi proeminențe umbrire (mai ales
dacă aceasta include PV, ca în Fig. 16.6(d)) ar fi chiar mai bine, dar poate
fi mai dificil să renoveze o clădire care a fost necorespunzător
proiectate inițial.
§16.4.8 Cald, climat tropical umed
În cald, climat umed tropical oceanic regiunile de coastă, unde
mare parte din populația lumii trăiește, temperatura maxima nu sunt la fel de
extreme ca și în cald, climat uscat. Ocean acționează ca o căldură tampon, astfel încât
diurne variație de temperatură este mic (~5°C) și masă termică de o
clădire poate avea pic de efect de răcire.
Cheia pentru confort este o mișcare a aerului, astfel încât aerul din jurul o persoană
se îndepărtează înainte de a deveni saturate, astfel permițând evaporarea la
nivelul pielii (transpirație) pentru a oferi fiziologice de răcire. Prin urmare, clădiri
CUTIE DE 16,6 ELECTROCHROMIC WINDOWS
Electrochromic windows poate fi un util noua tehnologie în vedere tehnologic sofisticat, dar fierbinte, locuri uscate
, cum ar fi California. Un mic de energie electrică de curent care trece printr-o electrochromic strat pe sticlă provoacă
fereastră a trece de la clar la nuanțat și înapoi. În clar de stat, până la 63% din lumina trece printr – ideal
pentru un cer de iarnă, ziua când căldură solară ajută încălzirea clădirii și lumină naturală reduce
nevoia de iluminat artificial. În fumurii de stat, cât de puțin de 2% din lumină și căldură solară vine prin
fereastra de sticlă, păstrând aproape toate nedorite de căldură în după-amiaza de vara oferind în același timp suficientă lumină
pentru a menține interne luminile.
TWIDELL PAGINARE.indb 589
01/12/2014 11:38
590
Utilizarea eficientă a energiei
sunt construite în mod tradițional, cu numeroase deschideri cu care se confruntă
predominantă a vântului, câteva interne impedimente la temperatura (ca în Fig. 16.8), și
, de asemenea, de multe ori ridicat de la sol pentru a prinde un puternic temperatura (amintiți -
variația vitezei vântului cu înălțimea: §7.3.2). Dacă o persoană fizică temperatura este absent,
low-viteza ventilatoare asigura bine ati venit mișcarea aerului.
Un alt aspect este că, în locații tropicale, traiectoria Soarelui este
aproape de zenit, astfel încât acoperișul primește foarte puternică insolație, care poate
crește plafonul temperaturi cu căldura care radiază puternic în
interior. Prin urmare, acoperiș suprafețele trebuie să fie extrem de reflectorizant sau altfel
de culoare albă, cu o ventilație adecvată sub și izolare termică mai sus
plafoane. Ferestre care se confruntă est sau vest ar trebui să fie umbrită, pentru a evita căldura
de la low-unghi de expunere la soare.
Electric alimentat cu aer condiționat, de asemenea, poate oferi confort, dar
consumul de energie (și costul inițial) este de cel puțin 10 ori de un
ventilator electric – mai mult, dacă temperatura este prea scăzută. Dacă utilizarea sa este considerată
absolut necesară (de exemplu, în unele laboratoare), cladiri sau in
camere ar trebui să fie bine izolate, ca în zonele cu climă rece; din păcate
acest lucru este rar făcut.
Fig. 16.8
Cald, umed zona.
o
Un "tradițional" fale în Upolu, Samoa (o insulă tropicală din Oceanul Pacific (latitude
13°S)), cu pereti de a bloca temperatura. Aceasta include de fier galvanizat, în
acoperiș, pentru că este mai ușor să se mențină decât paie (autor foto).
b
Tradițional 'Queenslander casa în Brisbane, Australia (latitudine 27°S) arată
altitudine de larg, verande pentru umbrire, și de construcții ușoare pentru
răcire rapidă pe timp de noapte.
Sursa: Foto de Wade Johanson, recoltate și utilizate sub Creative Commons Attribution
2.0 Generic de Licență.
(a)
(b)
TWIDELL PAGINARE.indb 590
01/12/2014 11:38

§16.5 Transport
591
§16.4.9 Compozit climate
Deși simplu de clasificare a climatelor prezentate de Szokolay (2008)
și folosite de mai sus descrie cele mai multe locuri în mod adecvat pentru arhitecturale
scopuri, unele locuri au semnificativă variație sezonieră. Ahmedabad în
Gujarat este un exemplu: după cum sa menționat în §16.4.7 are o fierbinți, sezonul uscat
, urmat de un cald, umed sezon, care face design de construcție mai
complicată. Washington, DC, este un alt exemplu, cu cald, umed
veri și ierni reci – o provocare îndeplinită prin Prima Casa Lumina
prezentat în Fig.16.6(b).
§16.5 TRANSPORT
§16.5.1 Fundal
De Transport este în general considerată ca circulația mărfurilor și a
oamenilor de vehicule cu motor. Astfel, în ciuda importanței lor, de mers pe jos
și ciclism tind să fie neglijate de către autoritățile de planificare a transportului și statisticieni.
Vehiculele alimentate reprezintă între 20% și 30% din energie primară
utilizați în cele mai multe economii. Cu toate acestea, în statisticile oficiale (vezi Fig
16.3), astfel de date de multe ori nu includ călătoriilor internaționale și
a comerțului internațional, prin aer și pe mare. Mișcările de persoane și de bunuri
și mărfuri (inclusiv a combustibililor fosili), sunt componente majore ale
economiei globale.
§16.5.2 Vehicule
Motivul-putere mecanisme comune moduri de transport terestru sunt
după cum urmează:
1
Metabolismul (mers pe jos, alergare, ciclism, animale de putere) (§17.2.3).
2
Vânt (navigatie-barci).
3
Motor Electric, conectate la rețea (electrică trenuri și tramvaie) (§15.4).
4
Motor Electric de la bord baterie reîncărcabilă sau celule de combustibil
(mașini electrice, ușoare utilitare/camioane, biciclete, motociclete și vehicule hibride) (a se vedea
§15.7).
5
Motoarele cu ardere internă cu lichid sau lichefiat combustibil (cu aprindere prin scânteie,
cu aprindere prin compresie/diesel, motoare cu reacție). (a se vedea Ch. 10).
6
Cu ardere internă cu combustibil gazos (motor cu aprindere prin scânteie rutiere pentru
vehiculele cu rezervoare de metan comprimat sau hidrogen) (pentru biogaz,
§10.8).
Metoda (5) este extrem de dominant astăzi, cu fosile de petrol în
principal de combustibil (Fig. 16.9). Energia din surse regenerabile este în măsură să alimenteze toate
mecanismele (1) - (6), și, prin urmare, prezintă considerabilă de alegere, așa cum este indicat
de către secțiunea de referințe din lista de mai sus; a se vedea, de asemenea, Caseta de 16.3.
TWIDELL PAGINARE.indb 591
01/12/2014 11:38
592
Utilizarea eficientă a energiei
Eficiența diferitelor motoare este în două clase – fie electric sau
termice de ardere:

Grid-connected motoare electrice (ca în trenuri și tramvaie) converti aproximativ
90% din energia electrică în energie cinetică; - eficiența globală
depinde de modul în care energia este generată și transmisă.

Cele mai alimentat cu baterie, motoare electrice sunt, de asemenea, aproximativ 90% eficient
(energie electrică la putere arbore), dar bateria de încărcare și
descărcare proces este, în practică, între 50% și 80% eficient în funcție
mai ales de vârsta și de utilizare a bateriei.

Generală a eficienței sistemelor de mașini electrice depinde de
eficiența de generare de energie electrică de la sursă și de pe grila
eficiența transmisiei.

Practic motoarele cu aprindere prin scânteie sunt, de obicei, aproximativ 35% eficient de
combustibil la putere arbore, restul de energie fiind emise de căldură de
nici o valoare într-un vehicul, altul decât confortul de încălzire, cu motoare diesel
ceva mai bine (a se vedea Caseta 16.1).

Generală a eficienței sistemelor de motoare termice depinde de
energia utilizată de sistemul de alimentare în furnizarea de combustibil (de exemplu, în
rafinării de petrol și transportul acestuia la bowser); acest lucru este în mod normal
necunoscut pentru consumator. La cutia de viteze (transmisie) și pierderile de
aer și drum de frecare duce la alte pierderi de energie în timp real a vehiculelor, așa
că bine să-jante de eficiență a vehiculelor cu astfel de motoare este
de obicei <10%.
Fig. 16.9
La nivel mondial combustibil lichid utilizat de către sectorul 2008 până în 2035 (milioane de barili de petrol/zi). Transportul este
la nivel mondial, utilizarea dominantă. (Date pentru anul 2015 este EIA de proiecție.)
Sursa: NOI de Energie de Administrare (2011), International Energy Outlook 2011, republicat. DOE/EIA-0484
(2011), Fig. 33.
112
108
103
93
98
120
80
86
40
0
2008
2015
2020
2025
2030
2035
Industriale
Transport
Clădiri
Energie electrică
TWIDELL PAGINARE.indb 592
01/12/2014 11:38
§16.5 Transport
593
§16.5.3 O nesustenabil sistem de transport?
Mai multe componente de prezent sistemele de transport a le face
problematică de mediu, economic și social:
1
Diminuarea fosili rezerve de petrol (sistemul actual este aproape în întregime
alimentat cu produse petroliere).
2
Global atmosferice de impact (CO
2
emisiile generate de combustibilii fosili sunt
de conducere schimbările climatice: a se vedea §2.9).
3
Locale, impactul asupra calității aerului (smogul urban cauzată în mare parte de vehicul
de eșapament este un pericol pentru sănătate, ca de plumb a fost de la benzina cu plumb).
4
Zgomot, în special de autostrăzi și autostrăzi, și în orașe.
5
Decese din accidente rutiere (o cauză importantă de deces, de exemplu, statele UNITE ale americii
~40,000/y la 14 la 100.000 de persoane pe an, Namibia 53/100,000,
Japonia 3.8/100,000).
6
Mobilitatea insuficientă în țările în curs de dezvoltare (slabă
infrastructură împiedică mulți oameni de a aduce produsele lor pe piață
și la accesarea ce facilitati sunt pentru sănătate și
educație).
RE (deosebit de biocombustibili, dar, de asemenea, cu vehicule electrice) contribuie la
atenuarea (1) și (2) (a se vedea Caseta 10.3), și, eventual, (3) și (4). Alte
probleme sunt reduse de resurse îmbunătățite, o mai bună tehnologie de control și
responsabilitate publică.
Atunci când călătoresc fost sever restricționate în marea Britanie de către cea de-a Doua Lume
de Război, era un slogan: 'Este călătoria într-adevăr necesar?' Aceeași
întrebare poate și ar trebui să fie întrebat astăzi de mult în creștere – dar
aproape sigur nesustenabile – cererea pentru turism.
Tehnologii de telecomunicații sunt adesea promovate ca o modalitate de a
reduce nevoia de a aduce fizic co-lucrătorilor împreună, și, astfel,
reduce cererea atât pentru naveta de zi cu zi de călătorie și pentru mai mult timp
călătoriilor (de multe ori prin aer) la conferințe de afaceri și altele. Cu toate acestea,
creșterea mare în distanță de schimb de informații, în special prin intermediul
internetului, este în sine semnificativ consumul de energie, cu un consum de energie de date
centre se ridică la aproximativ 2% din totalul consumului de energie electrică în SUA și
aproximativ 1,3% din totalul consumului de energie electrică, cu aceste proporții continuă
să crească (Koomey 2011). În plus, a crescut
de telecomunicații activitate poate chiar stimula o creștere în călătorie, și creșterea
bogăției în țări indică faptul că, chiar dacă persoanele fizice pot călători mai puțin,
suma totală a tuturor călători va crește. Prin urmare, necesitatea durabilă
a călători și a mecanismelor de transport este vitală.
§16.5.4 de Transport și forma urbană
Design Urban pentru durabilitate promovează eficiente energetic comunale
moduri de transport pentru a înlocui călătoriile personale a autovehiculelor.
TWIDELL PAGINARE.indb 593
01/12/2014 11:38
594
Utilizarea eficientă a energiei
În consecință, există o reducere semnificativă în numărul total de vehicule în kilometri și în
mediul urban poluarea aerului. Reducerea în totalul energiei folosite pentru transport
este de obicei mai mare decât prin mai eficient vehicul individual călătorii.
Fig. 16.10 parcele de transport consumul de energie per persoană împotriva urbane
cu densitate pentru o gamă largă de orașe din întreaga lume, se arată că compacte orașe
(care includ mai vechi orașe din Europa și unele orașe noi în
țările în curs de dezvoltare) folosesc mai puțin pe cap de locuitor de energie pentru transport decât extins
aglomerări urbane. Deși pare evident, mai puțină energie este
folosită parțial pentru călătoria distanțele sunt mai scurte, dar, de asemenea, pentru
80
Transport legate de consumul de energie
Gigajouli pe cap de locuitor pe an
Houston
70
60
50
40
30
20
10
0
0
25
50
75
100
125
150
200
250
300
Phoenix
Detroit
Denver
Los Angeles
San Francisco
Boston
Washington
Chicago
New York
Toronto
Perth
Brisbane
Melbourne
Sydney
Hamburg
Stockholm
Paris
Londra
Copenhaga
Amsterdam
Singapore
Moscova
Hong Kong
Urban densitate de locuitori pe hectar
Frankfurt
Zurich
Bruxelles
Munchen
Berlinul De Vest
Viena
Tokyo
Orașe din america de nord
Australian orașe
Europene, orașe,
orașe din Asia
Fig. 16.10
Transport consumul de energie pe cap de locuitor într-o serie de orașe. Notă-consum redus de energie pozițiile de
la Copenhaga și Amsterdam, din cauza utilizării pe scară largă a bicicletelor și în condiții de siguranță-ciclu de benzi
în aceste orașe. Alte low-energie poziții se referă la extinse de transport public.
Surse de date: Newman și Kenworthy (1999), Atlas Environnment du Monde
Diplomatique (2007).
Graficul luate de la a Lovi cu piciorul Obiceiul: ONU Ghid pentru Climat de Neutralitate, UNEP/GRID-Arendal.
TWIDELL PAGINARE.indb 594
01/12/2014 11:38
§16.5 Transport
595
oamenii de mers pe jos, ciclu și de a folosi transportul public fără a fi nevoie pentru privată
vehicule. Mersul pe jos și cu bicicleta sunt întotdeauna din surse regenerabile de energie (alimente
metabolismul) și din ce în ce transportul public utilizează biocombustibili și alte
surse regenerabile de energie orientate spre tehnologii. Extinderea urbană din secolul 20
orașe, cum ar fi Los Angeles, unde 'planificare' (dacă există) a fost bazat pe
privat autovehicule, este un factor major care contribuie la extravagante combustibili
utilizați în transporturi; cu toate acestea, astfel de orașe se pot prezenta cele mai bune
oportude șanse pentru vehicule electrice, probabil, acuzat de RE-pe bază de energie electrică
de la rețea sau de la interne micro-generare.
Până în 2010, aproximativ 50% din populația lumii trăia în orașe și
zone urbane; acest lucru este proiectat pentru a fi de 60% până în 2030, în principal din cauza
creșterii rapide de metropole în țările în curs de dezvoltare, cum ar fi
Shanghai, Mumbai și Cairo. De la o energie la nivel mondial și sustenabilitatea
perspectivă, prin urmare, este vital pentru a obține o bună planificare urbană în locul în
care metropole înainte de extinderea ei de sub control cât mai multe orașe din
SUA și din alte părți au făcut. De asemenea, în creștere rapidă
mijlocii orașe precum Curitiba (Caseta 16.7), dintre care multe încă au
terenului din jur disponibile pe care noi de dezvoltare pot fi controlate.
Planificarea adecvată prezintă oportunități pentru acceptarea de surse regenerabile de
energie, tehnologii, nu numai pentru transport, dar, de asemenea, pentru clădiri (a se vedea
Caseta 16.7).
CUTIE 16.7 CURITIBA: UN STUDIU DE CAZ DE DESIGN URBAN PENTRU DURABILITATE ȘI REDUCEREA
CERERII DE ENERGIE
Orașul Curitiba din Brazilia (populație ~2,3 milioane de euro) a stabilit noi standarde de planificare urbană,
sub sa pe termen lung primar, Jaime Lerner. Orașul a acționat pro-activ în 1970, pentru a evita extinderea urbană
și mahala dezvoltarea în mediul său natural.
Curitiba are un sistem de transport integrat, care include benzi dedicate pe străzile principale
pentru autobuze. Deși orașul s-a dublat în populație, deoarece sistemul a fost dezvoltat, rezidențiale
dezvoltare (mai mult de o densitate mare cu mai multe etaje) și dezvoltarea comercială au fost atent
clasificate ca fiind pentru a permite un acces ușor la sistemul de autobuz (care a fost mai ieftin și mai flexibil pentru a dezvolta
decât
alternative, cum ar fi o cale ferată subterană). Sistemul este folosit de 85% din Curitiba populației
(2,3 milioane de pasageri pe zi), autobuzul se oprește (Fig. 16.11) sunt aproape de pistele de biciclete din lungime totală de 160
km în
oraș. Autobuzele, fabricate de Volvo în Brazilia, sunt 28 m lungime, împărțită în trei secțiuni (bi-articulate)
și alimentat cu biodiesel din soia.
Durabilitatea și livability ale orașului sunt îmbunătățite printr-o retea de aproape 30 de parcuri urbane și
zonele împădurite, făcându-l unul dintre cele mai verzi orase din lume. În 1970, fiecare din orașul
de locuitori au avut mai puțin de 1 m2 de zonă verde. Un obiectiv-regizat de efort a sporit acest domeniu la 52 m2
pe cap de locuitor, iar orașul este încă în mod activ îmbunătățirea mediul său natural. Orașul a reușit să
introducă un Verde Schimb programul de ocupare a forței de muncă în beneficiul mediului și din punct de vedere social
- vulnerabile. 70% din Curitiba deseuri reciclate. Orașul de reciclare de hârtie conturi numai pentru
echivalentul a 1200 de copaci pe zi.
Sursa: daneză Centrul de Arhitectură, http://www.dac.dk.
TWIDELL PAGINARE.indb 595
01/12/2014 11:38
596
Utilizarea eficientă a energiei
Fig. 16.11
Una dintre stațiile de autobuz din sistem integrat de transport utilizate de către 85% din populația Braziliană
orașul Curitiba.
Sursa: Foto de Mario Roberto Duran Ortiz Mariordo, utilizată sub Creative Commons Attribution 3.0 Unported license.
§16.5.5 Îmbunătățit vehicule
Opțiunile tehnice prezentate aici sunt discutate în detaliu mult mai mult de
Harvey (2010) și Negru (2010). Ele completează substanțiale
reduceri de transport de energie utilizat de design urban și alegerea modului
(rutier, feroviar, maritim sau aerian) a discutat în alte subcapitole.
(a) Îmbunătățirea "convențională" de vehicule
Eficiență energetică de autoturisme crește cu îmbunătățiri în
motoare, transmisii, anvelope, organisme simplificate și mai ușoare
materiale. Tabelul 16.1 arată că economiile de energie până la ~50% sunt
fezabile pentru producția de modele cu motoarele convenționale. Pentru sursele regenerabile de
energie, cum ar îmbunătățiri reduce semnificativ cantitatea de biocombustibil
necesare pentru programele naționale. Special construite experimental masini au
călătorit extreme de 500 de km pe litru de combustibil (0.2 L/100km, 1200mile/
galon SUA, 1400mile/Brit galon), dar în condiții controlate cu grijă
și, de obicei, se transportă numai șoferul. Acest lucru indică faptul că mult mai mare de
îmbunătățiri decât cele enumerate în Tabelul 16.1 sunt posibile.
(b) vehicule Electrice
Motoarele electrice sunt de aproximativ trei ori mai eficiente energetic decât
de ardere (de căldură) motoare; aproximativ 90%, comparativ cu aproximativ 30%. În plus,
(a) individuale motoarele electrice pot fi cuplate direct la fiecare roata, astfel
de mai sus mecanice, cutii de viteze si transmisii, și care să permită
rate diferite de cotitură atunci când viraje; (b) motoare electrice poate deveni
TWIDELL PAGINARE.indb 596
01/12/2014 11:38
§16.5 Transport
597
Tabelul 16.1
Performanță posibilă a unui viitor "avansat" de masini cu motor cu ardere internă
motoarele
An
2001
2020
2020
2020
2020
2020
Starea
Baza
"Avansat"
Avansat
Avansat
Avansat
Avansat
Tip de motor(s)
SI GHEATA
SI GHEATA
SI hibrid
FC Hibrid
CI DE GHEAȚĂ
FC Hibrid (H
2
de combustibil)
Motor de capacitate (L)
2.5
1.65
1.11

1.75

Transmisie
Auto
ACT
CVT
Direct
ACT
Direct
în Masă (kg) (inc. 140 kg
sarcina utila)
1460
1130
1150
1370
1180
1260
Coeficientul aerodinamic
0.33
0.22
0.22
0.22
0.22
0.22
Baterie de energie specifică
(Wh/kg)



50

50
Urban consumul de combustibil:
3.7
(L benzina eq/100 km)
8.7
5.5
3.7
3.6
4.7
0.51
km/marea BRITANIE
galon
28
45
66
68
52
480
km/NE
galon
23
38
55
57
43
400
(MJ/km)
2.82
1.78
1.20
1.16
1.53
0.66
economisire a Energiei
în comparație cu baza (%)

36%
53%
53%
47%
66%
Note
SI
= cu aprindere prin scânteie, ICE = motor cu ardere internă, FC = celule de combustibil, CI = aprindere prin compresie ("diesel"),
ACT
= auto-ambreiaj transmisie CVT = transmisie continuu variabilă.
Sursa: Adaptat din Tabelul 5.22 de Harvey (2009). NE "compact" vehicul și NE ciclu de testare pentru consumul de combustibil.
generatoarele atunci când vehiculul încetinește; acest frânare regenerativă' permite
electric-tren puterea de a fi returnate la rețea și electric-masina de putere
pentru a reîncărca bateriile. Baterie-numai vehiculele electrice sunt din ce în ce mai
frecvente, mai ales pentru deplasările locale și asociate cu grid - connected
micro-generare pentru încărcare.
Vehiculele electrice hibride au atât un relativ mici-capacitatea termică
a motorului și, de asemenea, transmisii electrice; motorul termic este folosit pentru lungi
călătorii pe distanțe și pentru a încărca bateriile de la bord. Motoarele electrice
sunt folosite pentru scurte călătorii și impuls de energie (de exemplu, alpinism dealuri abrupte
și depășire). Motorul termic se oprește atunci când autovehiculul se află în staționare
(la semafoare) sau se deplasează încet, cu unitate de comutare de la baterie
putere numai. Beneficiile sunt îmbunătățite consumul de combustibil (a se vedea Tabelul 16.1)
și mai puțin zgomot și emisii în orașe. Plug-in hydrid masini au mai mare
capacitate de baterii care pot fi percepute de la rețeaua de energie electrică, care pot
fi generate de RE. Motorul termic combustibili pot fi biocombustibilii.
Impactul global asupra mediului al încărcate electric autovehicule și
trenuri electrice depinde de sursa de energie electrică. Dacă dintr-un thermal
power station, efectul net este de a transfera poluarea termică și
ineficiența din mediul de vehicul (de exemplu, străzi) de la
statia de alimentare. Numai o transformare majoră de energie electrică pentru a
TWIDELL PAGINARE.indb 597
01/12/2014 11:38
598
Utilizarea eficientă a energiei
surse regenerabile de energie pot face astfel de vehicule parte de o situare cu adevărat
durabilă a sistemului energetic. Plug-in vehicule electrice va contribui la această
transformare, ca intermitentă sarcina pe grila, în principiu, pot ajuta la
a se potrivi cererii de energie electrică pentru variabila de intrare de energie eoliană,
etc. (a se vedea §15.4.2).
(c) vehiculele pe bază de Hidrogen
Hidrogenul poate fi folosit ca combustibil pentru (a) motoare cu aprindere prin scânteie, și
(b) celule de combustibil producerea de energie electrică pentru vehicule electrice. (Deși H
2

este
ușor, mașină indicate în Tabelul 16.1 este grea din cauza rezervorului de combustibil
necesar: a se vedea §15.11.) Atunci când și în cazul în care este utilizat, emisie este doar apă
de vapori, deoarece în general o reacție chimică în care vehiculul este:
2H + O
2H O +142 MJ / (kg de hidrogen)
2
2
2
(16.8)
Prin urmare, vehiculele pe bază de hidrogen sunt potrivite în orașe și orașe în cazul în
care este necesar realimentare infrastructură, care necesită mari
investiții. Impactul global depinde de modul în care hidrogenul a fost
produs pentru care există două categorii de fabricație: (1) prin
reacție chimică, sau (2) prin electroliza apei. Reacția chimică
traseul este în mod normal de la combustibilii fosili de "natural" de gaz (metan), petrol sau
cărbune, dar ar putea fi, în principiu, din biogaz (metan) sau biomasă.
2

Alte
detalii de o economie a hidrogenului sunt prezentate în §15.9.1.
§16.5.6 de transport de Marfă
Transportul de mărfuri reprezintă 30% din transportul de utilizare a energiei în OECD
țări, și, probabil, mai mult în țările în curs de dezvoltare. Transport de mare
domină mișcarea internațională de mărfuri (peste 95%, măsurată de
tone-km; aproximativ 50% din valoare), cu motoare de nave, folosind slab rafinat,
dar relativ ieftine, de combustibili fosili. Pentru toate modurile de transport, energie
folosi pe tonă-km este mare pentru distanțe mai mici de 200 m, din cauza
energiei utilizate pentru încărcare și descărcare, dar aproximativ proporțională cu
distanța pentru distanțe mai mari. Intensitățile energetice (MJ pe tonă-km) pentru
distanțe lungi containere de marfă sunt ~0.7 pentru drum, ~0.3 pentru transportul feroviar și ~0.2
pentru transportul pe mare.
3

Intensitățile energetice pentru vrac (de exemplu, grâu sau ulei)


de mare sunt chiar mai puțin. Transportul maritim este energie eficient, de încredere și în condiții de siguranță,
dar de multe ori poluante; cu toate acestea, este lent, care este de ce evaluate (în $/kg)
sau perisabile, mărfuri pot fi transportate rutier, feroviar sau aerian. Deși
crește în dimensiunea navei și îmbunătățiri în design și motoarele au
o eficiență energetică îmbunătățită semnificativ în ultimii 30 de ani, nu este încă de
domeniul de aplicare pentru îmbunătățirea în continuare. Un factor important este că, alte lucruri
fiind egale – energie pentru a propulsa o navă crește ca fluid drag
(am.e. cu pătratul vitezei sale), atât de lent barci sunt cele mai energetice
eficiente.
TWIDELL PAGINARE.indb 598
01/12/2014 11:38
§16.6 industria Prelucrătoare
599
Au existat unele evoluții moderne folosind naviga structuri
auxiliare de putere pentru nave, dar pare puțin probabil că în previzibil
viitor vântul va deveni din nou dominant sursă de energie pentru transport,
așa cum a fost până pe la 1880. Prin urmare, de așteptat opțiune pentru surse regenerabile de
energie pentru marin putere este biocombustibili lichizi; cu toate acestea, biocarburanții sunt susceptibile de a
avea prioritate pentru transportul rutier.
§16.5.7 Aviației
Dominantă cerință pentru aviație combustibil este că ar trebui să fie de încredere,
la nivel internațional, disponibil și rămân lichide la temperatura rece de zbor
înălțimi. Cele mai multe avioane folosi motoare cu reacție, pentru care cea mai mare de combustibil trebuie să
fie
adecvat. Aeronave de mici dimensiuni pot avea motoare cu aprindere prin scânteie. Presiunea publică
și preocupări generale de sustenabilitate au încurajat mai multe mari
companii aeriene la proces utilizarea biocarburanților pentru motoarele de avion.
§16.6 INDUSTRIA PRELUCRĂTOARE
Consumul de energie pentru industrie au reprezentat peste 35% din consumul total de energie
la nivel mondial în 2008, cu aproape jumătate din această atribuite câteva deosebit de
mari consumatoare de energie heavy industries, și anume de fier și oțel, produse chimice și
petrochimice, metale neferoase și de celuloză și hârtie (Fig. 16.3(a)).
Există o mare sfera de aplicare pentru îmbunătățirea eficienței energetice în industrie și
comerț.
Reciclarea este o cheie pentru a reduce consumul de energie pentru industrie, și, astfel, pentru a
face mult mai disponibil pentru alte utilizări energetice. Ia din aluminiu
industrie ca un exemplu. Aluminiu primar (de exemplu, metal produs din
minereu) necesită o mare consumatoare de energie procesul de electroliza care conturile
pentru ~30% din costul de producție. Acest lucru a dat industriei de un puternic
stimulent pentru a îmbunătăți eficiența procesului; incrementală
schimbare tehnologică a redus intensitatea medie de 25 MWh/tAl în
1950 la 16 MWh/tAl (50 GJ/tAl) în 2010, cu primar corespunzător
consumul de energie ~100 GJ/tAl (utilizând energia hidroelectrică, permițându-pentru celelalte
componente ale procesului de producție). Cu toate acestea, mult mai mari economii
în intensitatea energetică din aluminiu poate fi făcută prin reciclare, deoarece
reformarea aluminiu necesită doar ~15 GJ/tAl de energie primară (în principal
de căldură), adica ~15% din care pentru producția primară. Prin urmare, în mișcare pentru a
(sa zicem) 90% producție secundară din materiale reciclate ar reduce
energie primară nevoie de un factor de ~5. Industria de aluminiu
a recunoscut de mult timp hidroelectricitate ca cea mai ieftină și cea mai adecvată
sursă de electricitate. Astfel și politica de încurajare
pentru reciclare aluminiu pentru a ridica proporția de secundar la ~25%
la ~90%, ar îmbunătăți sistemul general de eficiență energetică de
producție cu un factor de ~4, permițându-surplus de energie hidroelectrică pentru a se deplasa
pe bază de cărbune energie electrică.
TWIDELL PAGINARE.indb 599
01/12/2014 11:38
600
Utilizarea eficientă a energiei
Nu este corespunzator mare potențial de a reduce energie primară
utilizați în industria fierului și oțelului, deși mai puțin evident aplicare directă de
utilizare a RE în care industria appart de specialitate rafinare folosind cărbune.
De reciclare a produselor din hârtie reduce presiunea pe lume pădurilor pentru
fibre, și, de asemenea, energia necesară pentru a face hârtie. În plus, în
moderne fabrici de celuloză și hârtie nu este suficientă biomasă "deșeuri" de a furniza
toată energia necesară de către o instalație de cogenerare, care nu numai că furnizează
toate de energie electrică și termică are nevoie de moara, dar, de asemenea, exporturile
de energie electrică. Cu toate acestea, cele mai multe mori din întreaga lume încă nu sunt chiar de auto-
suficiente în
energie, în principal din cauza lor "deșeuri" de căldură nu este reutilizat în măsura în
care este tehnic posibil.
În multe industrii, o mare parte din consumul de energie merge pe
electric motors , care a pompei de lichid (inclusiv de ventilație, de aer condiționat sau
sisteme de aer comprimat) sau de a conduce transportoare, compresoare sau alte
utilaje, estimată la 40% din totalul de energie electrică (AIE 2011). Mari
economii de energie poate veni de la dimensionarea acestor motoare și/sau ajustarea lor
de încărcare, astfel că ei conduc la o eficiență optimă. (De exemplu, un motor tipic
poate avea o eficiență de 80% la sarcină maximă, dar numai 30% la 50% load.) Caseta
16.8 indică faptul că utilizarea mai largă conductelor cu pompele de dimensiuni pentru a se potrivi, plus
unități de viteză variabilă care permit motoare electrice să funcționeze la parametrii optimi
de eficiență chiar și cu sarcină variabilă, ar putea salva ~90% din energia utilizată
în unele astfel de aplicații; poate că până la ~15% din totalul consumului de energie electrică de
comerț și industrie la nivel național.
CUTIE DE 16,8 DIMENSIONAREA CORESPUNZĂTOARE A CONDUCTELOR ȘI POMPELOR ECONOMISEȘTE
ENERGIE
Pentru un motor care este de pompare de lichid, de exemplu, într-un solar sau un sistem de încălzire convențional, necesarul de
energie electrică este:
ηη
=
P
P
/(
)
mp
electric
lichid
(16.9)
în cazul în care P
lichid
este puterea care trebuie să fie aplicate lichid (de exemplu, sarcina) și
η
m
și
η
p
sunt
eficiența motorului și a pompei respectiv.
Puterea P
lichid

este necesar de a pompa lichidul împotriva frecare depinde de sistemul de conducte. Mai drept,
cu diametru mai mare și mai lin conducte au mai puțin pierderile prin frecare (vezi §R2.6). Problema 6.7 arată că, în
pomparea unui volum Q de-a lungul unei drepte de țeavă de diametru D, puterea P
lichid
este necesar ca scade D
-5
și
crește ca Q
3
. Prin urmare, la creșterea conducta cu diametrul de 20% și reducerea debitului de 50% reduce
pompare putere de
(1.2)
2
20
5
3
×
=
, care este o economie substanțială.
Arhitecți și constructori permite de multe ori lipsa de spatiu pentru conducte cu diametru mare (de exemplu, pentru încălzire și
sisteme de ventilație). Precaut ingineri apoi specificați pompe care sunt supradimensionate. Pompele convenționale
funcționează la viteză fixă, indiferent de cât de mult lichid este pompat, care necesită fluxul să fie
parțial obturat (sugrumat) dacă debitul necesar scade, reducând astfel
η
p
.
În consecință
η
m
scade, de asemenea, și de energie electrică necesară P
electric

crește și mai mult. Folosind unități de viteză variabilă


poate compensa mult de acest efect. Deoarece sistemele pot funcționa timp de cel puțin 20 de ani și, probabil, 100 de ani,
durata de viata de economii pot fi considerabile.
TWIDELL PAGINARE.indb 600
01/12/2014 11:38
§16.7 Interne de utilizare a energiei
601
§16.7 INTERNE DE UTILIZARE A ENERGIEI
Consumul de energie în locuințe, în general, conturile pentru >20% de energie naționale
de utilizare și costul de uz casnic este considerabilă. În principiu,
gospodari au posibilități considerabile de a gestiona propria lor energie, alege
potrivit aprovizionarea cu energie și furnizori, și de a obține și de a genera
propriul putere. Cu toate acestea, în practică, tradiționale și conservatoare comportamentul
și lipsa de înțelegere înseamnă că inovarea este lent. De obicei, cea
mai mare contribuție la interne consumul de energie pentru încălzire și răcire
internă constructii. Clădirea principiile de proiectare prezentate
în §16.4 poate duce la clădiri noi au nevoie de mult mai puțin achiziționat
energie decât clădirile mai vechi, poate doar 20% sau mai puțin, dacă există pe site-ul de
micro-generare de căldură și electricitate. Cu case vechi, suplimentare
izolație internă și externă, noi și secundare de geamuri, pescaj
prevenirea și mai eficiente de încălzire și/sau răcire pot fi
modernizate pentru creștere semnificativă în confort și de reducere energetică
costuri. Fig.16.12(a) sugerează că astfel de măsuri de bază în marea BRITANIE activat
intern consumul de energie pentru încălzire să fie reduse cu aproximativ 50% în 45 de ani
după prima criză a petrolului în 1973; în plus, media de confort sporit
(Boardman 2010).
Fig. 16.12
Economii de energie, în marea BRITANIE residences.
o
Salvarea din cauza la o mai bună izolație și eficiența de încălzire, 1970-2007. "Economii" prezentate
sunt relative la consumul de energie dacă tipic casa de izolare și încălzire a continuat cum
a fost în 1970.
b
Declin în consumul mediu de energie de un nou frigider sau congelator vândute în marea BRITANIE
din 1990 până în 2010.
Sursa: marea BRITANIE Departamentul de Energie și schimbări Climatice (2011) Consumul de Energie
din marea BRITANIE.
(a)
0
10
20
30
40
50
60
70
80
90
100
1970
1980
1990
2000
Milioane de
pentru a
nnes
ulei equi
va
len
t
Real de energie
consumul
Izolare de economii
Incalzitor de economii
(b)
0
20
40
60
80
100
120
1990
1995
2000
2005
2010
Index (1990 = 100)
Frigidere
Lăzi frigorifice
TWIDELL PAGINARE.indb 601
01/12/2014 11:38
602
Utilizarea eficientă a energiei
Multe sisteme de încălzire și răcire din întreaga lume sunt controlate de
termostate stabilit de către utilizator. Este important să realizăm că substanțială a costurilor
de economii pot fi realizate în clădiri prin setarea termostatului în mod corespunzător.
În general, fără pierderi de confort sau de comoditate, puteți pune cu ușurință pe
un strat suplimentar de haine de iarna sau de a lua off în timpul verii.
Consumul de energie electrică pot fi reduse în mod semnificativ cu compacte
fluorescente (CFL) și light-emitting-diode (LED) iluminare, cu bună
'alb mărfurilor, în special frigidere și mașini de spălat, cu solide
televiziunea de stat și ecrane de computer, și prin evitarea energie în modul standby.
În mod clar vizibile instrumente și monitorizarea periodică a promova comportamentale
modificări care în mod frecvent reduce consumul de cel puțin 10%.
Ca cu iluminare (Caseta 16.2), eficiența energetică a aparatelor a fost
mult îmbunătățit, condus de măsuri de politică, cum ar fi Energia Minimă a
Standardelor de Performanță. Fig. 16.12(b) indică faptul că energia medie
a consumului de noi frigidere și congelatoare-a înjumătățit în ultimii
20 de ani. Majoritatea țărilor OECD necesita putere de produse să fie
indicat în mod clar, astfel încât consumatorii să poată face alegeri în cunoștință de cauză.
Micro-generarea de pe site-ul de energie electrică, în special de pe acoperiș, panouri FOTOVOLTAICE,
și de căldură, mai ales de încălzitoare de apă solare, sobe de lemn și căldură,
pompe, a crescut rapid din 2000 cu succes
politica instituțională mecanisme (de exemplu, tarifele feed-in). Până în 2011, Germania a avut
3400 MW solar FOTOVOLTAIC de capacitate de aproape un milion rezidențiale
clădiri. În general, intervalul și numărul de micro-generație de
instalații sunt în creștere în mod semnificativ la nivel mondial, cu rezultatul că case
cu o astfel de tehnologie poate reduce achiziționate de energie considerabil.
§16.8 ASPECTELE SOCIALE ȘI DE MEDIU
§16.8.1 Negawatts sunt mai ieftine decât megawați!
Amory Lovins, în 1970 analiză a cererii de acțiuni, a inventat
termenul de 'negawatt' (negativ watt) pentru putere nu se consumă și așa "salvat".
Astfel de analize de acțiuni de reducere a consumului de energie și, prin urmare, costurile
(în termeni de amortizare: a se vedea Capitolul 17) și emisiile de GES (în termeni
de $/tCO
2

reducere) a stabilit utilizarea eficientă a energiei, ca o


disciplină recunoscută. Costul net peste câțiva ani de succes negawatt
măsuri este negativ, adică nu există atât energia, cât și financiare de economisire. De
exemplu, manual de comutare de pe neutilizate lumini electrice costă nimic, cu
imediate economii la facturile de energie. Instalarea automată de control al iluminatului în
birouri (de exemplu, lumina-intensitatea de comutare, un comutator de timp sau un senzor de mișcare)
poate rambursa investiția în termen de un an. Există multe astfel de exemple.
O întrebare politică-cheie este de ce astfel de oportunități nu sunt întotdeauna
puse în aplicare. Ne ignore ca demonstrabil absurd afirmația unor
economiști că astfel de oportunități nu poate exista pentru că, în
idealizat modele economice, toată lumea are perfectă de informații și, prin urmare,
TWIDELL PAGINARE.indb 602
01/12/2014 11:38
§16.8 aspectele Sociale și de mediu
603
orice oportunități de a face bani în acest mod sunt luate automat în sus.
Cu toate acestea, este evident că mulți oameni de afaceri nu sunt conștienți de
posibilitățile tehnologice pentru eficiență energetică, și nu monitorizează
cantitățile și costurile de utilizare a energiei. Acest lucru poate fi, deoarece ei cred că
energia este o mică parte din costurile totale. Pentru cererea de succes-partea
de acțiune, informații clare este esențială; dar mai puternice instrumente de politică, cum ar fi
minime standarde de performanță energetică, etc., de asemenea, sunt necesare (a se vedea
§17.5). Caseta 17.5 indică faptul că, fără astfel de măsuri de politică energetică globală
a cererii este de natură să crească cu ~40% până în 2030, dar asta politică puternică
măsuri pentru accelerarea tehnologică de îmbunătățire a eficienței energetice
ar putea limita creșterea cererii pentru
De maximum 5%, în timp ce nu limitează prosperitate.
§16.8.2 Impact privind energia din surse regenerabile
Reducerea utilizatorului final cererii de energie reduce costurile lor și, astfel,
crește practic posibilitatea de a folosi o sursă de energie regenerabilă
pentru a satisface această cerere. De exemplu, costul de panouri fotovoltaice și
echipamentele auxiliare pentru alimentarea cu energie electrică a unei gospodării depinde atât
de servicii necesare și eficiența de utilizare. Continuarea
tehnologice de îmbunătățire a eficienței energetice a aparatelor reduce
consumul de energie electrică, și, astfel, dimensiunea și costul de panouri FOTOVOLTAICE (și
, probabil, baterii) necesare. Această reducere a costurilor este complementară care
decurg din îmbunătățirea eficienței panourilor ei înșiși (Capitolul
5) și economiile de scară la crescut masiv de producție (Fig.
17.2). Toate aceste rezultate din feedback-ul pozitiv și o creștere suplimentară în
utilizarea energiei solare pentru micro-generație în ambele dezvoltate și
țările în curs de dezvoltare. Acest lucru deschide ușa pentru beneficii sociale pentru sănătate, educație
și de telecomunicații moderne de alimentare cu energie pentru milioane de oameni
din zonele rurale din țările în curs de dezvoltare.
§16.8.3 Căi de dezvoltare economică
Un aspect-cheie în viitor global , cererea de energie este natura de
dezvoltare economică și dacă și în ce măsură în curs de dezvoltare
țări trebuie să urmați istoric căi de dezvoltare de
țările industrializate. De 'blocare' efecte de infrastructură, tehnologie și
design de produs alegerile făcute de țările bogate în mijlocul secolului 20
și mai devreme (de exemplu, angajamentul față de centralele pe cărbuni, urban machete
dependentă de autoturisme, etc.) set cadru pentru consumul de energie per persoană
variind de la 125 de kWh/zi de persoană, în Europa, la 250 kWh/zi
per persoană în statele UNITE ale americii. Astfel de stabilit și de multe ori de energie ineficient
procedurile sunt responsabile pentru o mare parte din creșterea recentă în lumea
consumului de energie.
În țările în curs de dezvoltare, în cazul în care infrastructura este încă să fie
construit, spectrul de opțiuni viitoare este considerabil mai mare. În special,
TWIDELL PAGINARE.indb 603
01/12/2014 11:38
604
Utilizarea eficientă a energiei
țările în curs de dezvoltare pot ocoli energie-practici ineficiente și trece
direct la tehnologii mai curate și mai durabilă a mediului construit
(a se vedea Casetele 16.7 și 16.9).
§16.8.4 Clădiri
Costul marginal al solare pasive caracteristici pentru clădiri, cum ar fi
orientare, plasare fereastră și umbrire este relativ mic la proiectare și
construcție. De stabilit o clădire, o schimbare în orientarea este
imposibil, dar beneficii semnificative poate veni de la retehnologizarea cu izolație,
umbrire, perdele, luminatoare, îmbunătățit aparate, etc. Sunt anumite
îmbunătățiri în confort, cu financiare de recuperare de multe ori în termen de unu la cinci
ani. Micro-generație este o acțiune responsabilă, atât pentru noi, și
stabibazele de clădiri, cu recuperare de peste cinci la 20 de ani probabil, în funcție de
stimulente guvernamentale.
De amortizare pentru clădiri închiriate sunt adesea la fel de scurt, dar
'proprietar-chiriaș problemă se aplică: proprietarii sunt adesea reticente în a plăti
costul de capital de măsurile de eficiență energetică atunci când economiile financiare
suportate de chiriaș. Prin urmare, regulamentul guvernului este necesar,
mandatarea corespunzătoare standardelor minime de performanță energetică în închiriat
de proprietate, a se vedea Capitolul 17 cu privire la factorii instituționali.
§16.8.5 implicațiile asupra Mediului de eficiență energetică
Majore cu impact pozitiv asupra mediului de îmbunătățirea eficienței energetice
este de a reduce emisiile de gaze de seră asociate cu combustibili fosili
CUTIE 16.9 CONSUMUL DE ENERGIE ÎN CHINA
Legat de modelul numeric de §1.2 este următoarea identitate:
Total la nivel național a cererii de energie
cererea de energie
PIB
PIB
persoana
populație
=
×
×
(16.10)
În China, PIB-ul pe persoană a crescut la rata rapidă de ~8%/an pentru mai mult de un deceniu. Populația
a rămas aproximativ constant la ~1 miliard datorită puternică a politicii guvernamentale. Cererea de energie
pe PIB-ul (în mod eficient o măsură națională de eficiență energetică) nu a scăzut în mod semnificativ pentru a compensa
creșterea de PIB/persoană, atât naționale, cererea de energie se accelerează puternic.
Mult de prezent, furnizarea energiei din combustibili fosili, care va deveni nesustenabile, în viitorul apropiat,
din cauza poluării și limitarea aprovizionării. De exemplu, dacă numai 20% din populația Chinei propria obișnuit
masini, lume de petrol ar deveni grav restricționate.
Până în 2012, China a avut mai multe surse regenerabile de energie electrică capacitate (și mai încălzitoare solare de apă)
decât
orice altă națiune (280 GW, din care 25% au fost non-hidro). China a condus lumea în rata de creștere de
sursă de energie regenerabilă la~30 GW/y (REN21 2012). Cu toate acestea, această creștere de 30 GW/an în 2012
a fost copleșit de creșterea cererii de energie, și a fost mai mică decât creșterea electrice pe bază de cărbune
de votare. China este tot mai mare accent pe energia regenerabilă se referă la enigma de echilibrare a
cererii în creștere pentru serviciile energetice împotriva nevoie pentru un mediu curat și dezvoltare durabilă.
TWIDELL PAGINARE.indb 604
01/12/2014 11:38
§16.8 aspectele Sociale și de mediu
605
utilizare. Numeroși analiști, în urma Lovins, au subliniat
oportunitățile de scăzut sau chiar negativ costurile nete pe tona de CO
2

s-au redus. De
numere în acest capitol sugerează că economiile de cel puțin 20% din globală
CO
2
emisiile sunt potențial disponibile; ER scenariul descris în
Cutie de 17,5 sugerează că măsuri puternice ar putea salva de 40%.
În general, efecte negative asupra mediului de orice structură sau de acțiune
(inclusiv cele de furnizare și utilizare a energiei) este mai puțin dacă structurile devin
mai mici și acțiuni mai puțin consumatoare de resurse, și anume dacă structurile și
acțiunile sunt eficiente. Impactul asupra mediului este complex și variat; acolo
sunt mulți alți parametri decât eficiența; cu toate acestea, nevoia de
eficiență, combinată cu creșterea RE este, probabil, universal.
REZUMAT CAPITOL
De energie sisteme includ ambele utilizări finale (cerere) și-a generație (de aprovizionare). Oamenii nu au nevoie de energie ca
atare, dar energia, serviciile oferite, cum ar fi de iluminat, de încălzire, de comunicare și de transport. Există
, de obicei, mai multe etape de energie primară de intrare (de exemplu, produse chimice de energie din biomasă) la utilizarea
finală (de exemplu,
transport într-un vehicul alimentat cu biodiesel). Fiecare pas are o eficiență energetică
= (producția de
energie)/(energia
de intrare), care este, de obicei, din punct de vedere istoric săraci, dar care poate fi îmbunătățită prin tehnologia și înțelegere
utilizator.
Economiile de energie pot avea loc prin metode alternative (de exemplu, de călătorie în condiții de siguranță pe piste de bicicleta
in loc de
masina). În general, eficiența energetică scade costul total de energie electrica achizitionata pentru utilizatori și reduce
emisiile globale de gaze cu efect de seră din arderea combustibililor fosili. Măsuri de economisire a energiei, de obicei, sunt mai
rentabile pe termen mediu și lung interval decât schimbări în aprovizionare cu energie. Utilizarea eficientă a energiei favorizează
introducerea de sisteme de energie regenerabilă.
Utilizarea eficientă a energiei nu este simplu sau evident, necesită educație, informare, etichetarea
bunuri și monitorizare. Legislația guvernamentală și obligațiile sunt întotdeauna importante.
Păstrarea clădirilor cald în timpul iernii și se răcească în vara, reprezintă aproximativ un sfert din energia
cerințele din multe țări, dar ai grijă design și layout poate aduce foarte economii substanțiale de energie.
Factorii cheie sunt energia solară câștig, masă termică, izolație și micro-generare; astfel de beneficii ar trebui să fie
obligatorie în codurile de construcție. Este mult mai ușor să includă energie solară și alte tipuri de energie-eficiență beneficii
în clădiri noi decât în a stabilit clădiri. Cu toate acestea, modernizate îmbunătățiri semnificative
beneficii pentru clădiri mai vechi. Climate diferite necesită diferite stiluri (de exemplu, într-un climat fierbinte și umed,
mișcarea aerului și minime de energie solară oferi confort, în timp ce în zonele cu climă rece la halbă-proofing, castig solar
și izolare termică sunt necesare.
Cererea de energie pentru transport se referă la formă de aglomerări urbane (urban densitatea, localizarea și petreceri,
transport public, etc.). O planificare atentă acum pot reduce substanțial cererea viitoare în multe rapidă
creștere orașe. Complementare economii de energie poate veni de la incremental îmbunătățiri tehnologice
pentru vehiculele care îmbunătățesc eficiența energetică a acestora și a permite utilizarea pe scară mai largă a energiei
regenerabile prin lichid
biocombustibili sau de rețeaua de electricitate.
Câteva industrii mari consumatoare de energie reprezintă cea mai mare parte din consumul de energie de către industrie, care
totalizează ~30%
din cererea globală de energie. Economii substanțiale de energie poate proveni din procesul de îmbunătățiri, mai mare
de reciclare de produse (de exemplu, oțel, aluminiu și hârtie) și de mai atentă dimensionare de motoare și pompe.
În piața internă a sectorului, persoane fizice câștig de economisire a energiei și a costurilor prin renovarea casei, alegerea atentă a
aparatelor și micro-generare cu surse regenerabile de energie.
TWIDELL PAGINARE.indb 605
01/12/2014 11:38
606
Utilizarea eficientă a energiei
ÎNTREBĂRI RAPIDE
Notă: Răspunsurile la aceste întrebări sunt în textul de la secțiunea relevantă
din prezentul capitol, sau poate fi ușor dedusă din aceasta.
1
Numele serviciile energetice furnizate în imediata
mediu acum, și să clarifice care sunt din surse regenerabile (inclusiv
biomasă).
2
Ce de măsurare a energiei și metode de monitorizare ai folosit; cum
ai putea să le îmbunătățească?
3
Ceea ce este teoretic randamentul ciclului Carnot de un simplu motor cu abur
care lucrează într-un mediu temperatură de 20°C?
4
Denumirea a cel puțin cinci tipuri de iluminare și lista de acestea, în scopul de eficientă
utilizare a energiei.
5
Pentru clădiri, ceea ce este un non-solare de energie liberă se obține? Dați cinci
exemple.
6
În ce direcție(s) ar trebui să se confruntă cu ferestrele, pentru a maximiza căldură solară
câștig? Ce caracteristici beneficia de geamuri?
7
De ce sunt instalatii de racire prin evaporare eficiente în Alice Springs (central
Australia), dar nu în Singapore?
8
Cinci metode pentru utilizarea energiei regenerabile în vehicule.
9
De ce sunt biciclete eficiente energetic?
10
Deși regenerabile de energie ajunge la sursă, fără costuri, explica
de ce gestionarea utilizării energiei din surse regenerabile este necesar.
PROBLEME
Notă: *indică o "problemă", care este deosebit de potrivit pentru clasa de
discuție sau grupul de tutoriale.
16.1
Pentru fiecare dintre următoarele cazuri, pentru a identifica pașii în trecerea de la
o sursă de energie primară la final de servicii. Pentru fiecare pas,
indica aproximativ eficiența de conversie a energiei
implicate, și comentariu la cum ar putea fi îmbunătățit.
(a) Un gospodar foloseste panouri FOTOVOLTAICE pe acoperiș pentru a genera energie electrică pentru
puterea ei frigider.
(b) Un motociclist într-un motor diesel de automobile setează automat
clapetei de accelerație' pentru a menține viteza la 80 km/h în sus și în jos deal.
(c) un Alt automobilist are o mașină electrică, pentru care se reîncarcă
bateriile din reteaua de alimentare cu energie electrică furnizată de o bază de cărbune
stație de putere.
16.2
Pierderea de căldură prin ferestre
O camera are doua ferestre de sticlă fiecare 1,5 m de mare, de 0,80 m lățime
și 5.0 mm grosime (Fig. 16.13). Temperatura aerului și de perete
suprafață în interiorul camerei este de 20°C. temperatura aerului exterior
TWIDELL PAGINARE.indb 606
01/12/2014 11:38
Probleme
607
este de 0°C. nu Există nici un vânt. Utilizând metodele prezentate în Review 3,
se calculează pierderile de căldură prin sticlă:
(a) presupunând că (în mod fals) că numai rezistența la fluxul de căldură este de
conducție prin materialul de sticla;
(b) de a permite (corect) pentru rezistența termică a aerului obligat-
ary straturi de sticlă, așa cum se arată în Fig. 16.13.
Sugestie: presupune ca o primă aproximare, care T
2
≈ T
3
≈ ½ (T
1
+ T
4
).
Justifica această ipoteză după aceea.
(c) sunt corespunzătoare termică resistivities (r )? Compara
ei cu resistivities de un gol zid de cărămidă sau o foarte
wellinsulated perete.
(d) se Calculează corespunzător pierderilor de căldură și r pentru un termopan
fereastra. Presupune o 3 mm, distanța dintre foile de sticla, si
nu convecție în decalajul.
16.3
Doi elevi să împărtășească un vechi "convențională" casa într-un loc răcoros nord
climatice. Pe o după-amiază de iarnă, temperatura de afara este peste 0°C.
camera De zi este încălzită de un incalzitor electric controlat de un
termostat. Elevul a îi place să țină de cald și de a seta
termostatul la 20°C. Elevul B crede că acest lucru este prea cald și deschide două
ferestre să 'într-un pic de aer rece'. Acest lucru creează un curent de aer, care rezultă
în opt complete de schimburi de aer în camera pe oră.
Sticla
În interiorul
aer
T
1
T
2
R
12
R
34
R
g
T
3
T
4
(a)
(b)
În afara
aer
Termică limita
straturi
Fig. 16.13
Pierderea de căldură printr-o fereastră; a se vedea Problema 16.2.
TWIDELL PAGINARE.indb 607
01/12/2014 11:38
608
Utilizarea eficientă a energiei
(a) în Cazul în care camera măsoară 6 m
× 4 m × 3 m, se calculează rata de
pierderea de căldură din acest proiect.
(b) să Calculeze cât de multă energie ar fi nevoie pentru a menține tem-
temperatura de la 20°C împotriva acestei pierderi. Dacă costurile de energie electrică cu 15 cenți
pe kWh, cât de mult bani ar costa pe oră?
*16.4
(a) În țara dumneavoastră, ceea ce este tipic putere nominală pentru fiecare dintre
următoarele aparate de uz casnic: (i) televizor; (ii) frigider;
(iii) 'desk-top' fan; (iv) aer conditionat (per cameră de service). (Indiciu: la
putere nominală este de obicei listat pe un producătorului autocolant
pe undeva pe aparat.) (b) Estimarea (sau măsura!) pentru cât de multe
ore pe zi în fiecare dintre aceste aparate se execută la putere nominală.
(c) Comentariu asupra lor relativă a consumului de energie.
NOTE
1
Lumenul este o unitate de măsurare putere radiantă, așa cum este percepută de ochiul uman. (Energia de la o sursă de lumină
în fiecare lungime de undă de bandă în funcție de sensibilitatea medie a unui ochi uman în acea bandă.)
2
Mai multe detalii la http://en.wikipedia.org/wiki/Hydrogen_production.
3
Date în Harvey (2009) bazat pe un IMO raport de 2000.
BIBLIOGRAFIE
Cărți în domeniul managementului energiei, în general,
Beggs, C. (2009, 2nd edn) Energie: Management, aprovizionare și conservare, Butterworth-Heinemann, Londra.
Nu prea tehnice.
GEA (2012) Global de Energie de Evaluare: Spre un viitor mai durabil, ed. T. Johansson, N. Nakicenovic, A.
Patwardhan și L. Gomez-Echeverri, Cambridge University Press, Cambridge. Pe scară largă internaționale de revizuire,
inclusiv capitolele privind utilizarea energiei în industrie, transport, clădiri și sisteme urbane. Unele capitole sunt
disponibile online la www.globalenergyassessment.org.
Harvey D. (2010) Energetică și Noua Realitate 1: eficiența Energetică și cererea pentru servicii energetice, Earthscan,
Londra. Imagine de ansamblu excelentă, care acoperă energia de bază și consumul de energie în clădiri, industrie, transport,
agricultură,
etc. Cele mai multe dintre numerele utilizate în §s16.5 și 16,6 se bazează pe datele adunate în această carte.
IEA (2011) Wade, P. și Brunner, C. V., Eficiența Energetică Opțiuni de Politică pentru Motor Electric-Sisteme de acționare,
IEA
de Presă, la Paris.
Kreith, F. și Goswami, D. Y. (eds) (2007) Manual de Eficiență energetică și Energie Regenerabilă, CRC, Londra.
Multi-autor tome; ușor NE concentrat.
MacKay, D. (2009) Energia Durabilă – Fără Aer Cald, UIT, Cambridge. Se referă în mod clar de uz casnic familiare
servicii/utilizări energetice la nivel național și la scară globală, folosind exemplar diagrame și ordin de mărime
calcule. Textul complet este disponibil online la www. withouthotair.com.
REN21 (2012) surse Regenerabile de energie Global Status Report 2012, Energia din surse Regenerabile în Rețea
(REN21). Disponibil online la
www.ren21.org. Include o caracteristică specială pe sinergiile dintre domeniul energiei regenerabile și eficienței energetice.
TWIDELL PAGINARE.indb 608
01/12/2014 11:38
Bibliografie
609
Clădiri
ASRAE (2006, 2nd edn) Verde Ghid: proiectarea construirea și exploatarea durabilă a clădirilor,
ButterworthHeinemann, Londra. Concis carte de referință, menite să ajute un inginer decide 'această tehnologie ar putea fi utile
la acest proiect?'.
Baird, G. (2010) Clădirile Durabile în Practică: Ce cred utilizatorii, Routledge, Abingdon. Studii de caz cu
accent pe arhitectură și servicii energetice pentru fiecare clădire; multe fotografii și planuri.
Balcomb, J. D. (ed.) (1991) Pasive Solare Cladiri, MIT Press, Cambridge, MA. Unul dintr-o serie pe caldura solara
tehnologiilor rezumând NOI de cercetare în anii 1970 și 1980.
Eicker, U. (2003) Tehnologii Solare pentru Cladiri, Wiley, New York. Tradus din germană original din 2001.
Include capitole despre solar de încălzire și răcire, și pe absorbția de răcire.
Givoni, B. (1998) Considerațiilor legate de Climă în Construcție și Design Urban, Van Nostrand Reinhold, New York. Un
review clasic de una dintre cele mai moderne pionieri în domeniul său.
Griffiths, N. (2007) Eco-casa Manual, Haynes Publishing, Londra. Plin de stimulare bun simț și practice
sfaturi pentru gospodari care doresc să trăiască în mod durabil. Aplică în special pentru marea BRITANIE.
Harvey, L. D. (2006) - Un Manual de Clădirile cu consum energetic Redus și Districtul Sisteme de Energie: noțiuni
Fundamentale, tehnici
și exemple, Earthscan, Londra. O tehnologie compendiu, care cu succes se explică modul de economisire a energiei
tehnologii și o abordare integrată pot atinge reduceri mari în utilizarea energiei, fără a compromite pe
clădire confort sau servicii.
Mumovic, D. și Santamouris, M. (eds) (2009) - Un Manual de construcții Durabile de Design și Energie, Earthscan,
Londra. Multi-autor, ușor de citit, dar încă cu o bună tehnică de detaliu.
Nicholls, R. și Hall, K. (eds) (2008) Clădire Verde Biblie, Vol. 2, Clădire Verde de Presă, Llandysul
(www.greenbuilding apăsați.co.marea britanie). Se concentrează pe construirea și renovarea clădirilor pentru eficiență energetică
și
conservare a apei; tehnice excelente explicații și ilustrații.
Somon, C. (1999) Design Arhitectural pentru Regiunile Tropicale, Wiley, New York. Include selectate climatice profile,
climatice și de considerente de proiectare și liniile directoare de proiectare.
Santamouris, M. (ed.) (2003) Tehnologii Solare Termice pentru Clădiri: starea de arta, James & James,
Londra. Parte dintr-o serie pe clădiri, energie și tehnologie solară.
Snell, C. și Callahan, T. (2005) Clădire Verde, Ciocârlia Cărți de Sterling Publishing, Ontario, Canada. Superb
ilustrat pentru arhitecți și constructori, inclusiv auto-construi; a cererii internaționale.
Szokolay, S. (2008, 2nd edn) Introducere în Știința Arhitectural, Arhitectura de Presă/Elsevier, New York.
Include concis tratamentul eficient energetic modele (de multe ori tradiționale) pentru o serie de climă, împreună cu multe
utile grafice și tabele.
Vale, B. și Vale, R. (2000) Noua Casa Autonoma, Thames & Hudson, Londra. Proiectare și construcție
a scăzut de energie, solară-conștient și durabilă-materiale de locuințe, cu specific marea BRITANIE exemple. Un studiu serios de
un subiect comun.
Weiss, W. (ed.) (2003) Sisteme Solare de Încălzire pentru Case, James & James, Londra. Unul dintr-o serie de
publicații în curs de dezvoltare din Solar de Încălzire și Răcire Program de Agenția Internațională pentru Energie. Această carte
se axează pe combi-sisteme (de exemplu, utilizarea de încălzitoare de apă solare integrate cu alte încălzire pentru clădiri).
TWIDELL PAGINARE.indb 609
01/12/2014 11:38
610
Utilizarea eficientă a energiei
Transport
Negru, W. R. (2010) Transport Durabile: Probleme și soluții, Guilford Press, New York. Foarte ușor de citit,
student la nivel de text, deși în mare parte bazat pe date din america și subțire cu privire la planificarea urbană.
http://en.wikipedia.org/wiki/Alternative_fuel_vehicle. Sursă utilă pe vehiculele alimentate cu carburanți alternativi, actualizate
din
timp în timp.
Istoric
Fouquet, R. (2008) de Alimentare de Căldură și Lumină: Revoluții în servicii energetice, Edward Elgar, Cheltenham. Un tur de
forță de istorie economică, urmărirea costurilor și utilizare de servicii energetice (căldură, lumină, putere mecanică, transport)
în Anglia de la 1300 la 2000!
Personal
Există numeroase cărți și chiar mai multe site-uri cu sfaturi cu privire la cum pot indivizii să-și reducă energie
cererii și cu un stil de viață mai durabil (care include în mod necesar redus de energie). Acestea sunt
în mare parte destinate cei din 'Vest' economii și includ multe tipice (și, uneori, surprinzătoare) numerice
exemple.
Goodall, C. (2007) , Cum să Trăiască o Viață emisii Scăzute de carbon, Earthscan, Londra.
Vale, R. și Vale, B. (2009) Timp pentru a Mânca Câinele? Real ghid pentru o viață durabilă, Thames & Hudson, Londra.
Referințe specifice
Boardman, B. (2010) De Fixare A Sărăciei Față De Combustibil, Earthscan, Londra. O autoritate de text, cele mai multe detaliu
care vizează marea BRITANIE
, dar principiile se aplică la nivel internațional.
Chen, L. et al. (1999) 'Efect de transfer de căldură legea cu privire la performanța unui generalizate ireversibile Carnot motor',
Jurnalul de Fizica D: Fizică Aplicată, 32, 99-105.
Fouquet, R. (2008) de Alimentare de Căldură și Lumină: Revoluții în servicii energetice, Edward Elgar, Cheltenham.
Koomey, J. (2011) Creșterea în Centrul de Date de consumul de energie Electrică 2005 până în 2010, Analiză de Presă,
Oakland, CA. Disponibil
online la http://www. analyticspress.com/datacenters.html.
Sinapsa (2008) Costurile și Beneficiile Electric Utilitar de Eficiență Energetică în Massachusetts, Synapse Energie
Economie pentru (Massachusetts) Nord-Est pentru Eficiență Energetică Consiliului. Disponibil la www.synapse-energy.
com/Downloads/SynapseReport.2008-08.0.MA-Electric-Utilități-Energie-Eficiență.08-075.pdf.
Twidell, W., Johnstone, C., Zuhdy, B. și Scott, A. (1994) 'Strathclyde University pasive solare, energie scăzută,
reședințe cu izolatie transparent', a Energiei Solare, 52, 85-109.
Reviste și site-uri web
Unele relevante reviste tehnice includ: Energie și Clădiri, de Conversie a Energiei și de Management, International
Journal of Energiei Durabile și Energiei Solare.
Există nenumărate site-uri care se ocupă cu subiecte de acest capitol, unele excelente și multe dintre dubioase aca-
dacat valoarea. Utilizați un motor de căutare pentru a localiza acestea și să dea cele mai crezare la site-urile oficiale ale
organizațiilor, cum
TWIDELL PAGINARE.indb 610
01/12/2014 11:38
Bibliografie
611
cu exemplele citate mai jos. Utile termeni de căutare ar putea include 'utilizarea energiei în clădiri", " durabilă a orașelor,
etc.
Consiliul American pentru Energie Eficientă a Economiei. www.aceee.org/. Include multe indicii concrete, plus
discutarea politicilor, și un set de link-uri suplimentare.
Asociația de Mediu Conștient Clădiri (marea BRITANIE) www.aecb.net; literatură și informații practice.
Australian Dezvoltarea Verde Forum. www.agdf.com.au/ vitrina.asp. Anterior Australian Energetică a Clădirii
Consiliului (http://www.netspeed.com.au/abeccs/). Studii de caz din Australia.
Bază de Date interactivă pentru eficiență Energetică Arhitectura (IDEA). nesa1.uni-siegen.de/projekte/idea/idea_1_e.htm.
Numeroase studii de caz din Europa, atât rezidențiale și clădiri comerciale.
International Solar Energy Society (ISES). www.ises.org. Cel mai mare, mai veche, și cea mai mare autoritate profesională,
organizație care se ocupă cu tehnologia și punerea în aplicare a energiei solare.
Marea BRITANIE energie consumul de statistică.
www.decc.gov.uk/en/content/cms/statistics/publications/ecuk/ecuk.aspx.
TWIDELL PAGINARE.indb 611
01/12/2014 11:38

CAPITOLUL

17
Instituționale și economice factori
CONȚINUTUL
Obiective de studiu
612
§17.1 Introducere
614
§17.2 Socio-politic factori
614
§17.2.1 politică energetică Națională
614
§17.2.2 țările în curs de Dezvoltare
618
§17.2.3 Rolul individului
619
§17.3 Economie
620
§17.3.1 Bază
620
§17.4 analiza ciclului de Viață
622
§17.5 instrumente de Politică
623
§17.5.1 politicile Guvernamentale
623
§17.5.2 Guvernamentale proceduri
625
§17.6 Cuantificarea alegere
626
§17.6.1 analiza de Bază
626
§17.6.2 fluxului de numerar Actualizat
(DCF) tehnici:
valoarea actuală netă
630
§17.7 stadiul actual al surselor regenerabile de energie 635
§17.8 calea De urmat
635
Rezumat capitol
641
Întrebări rapide
642
Probleme
642
Notă
643
Bibliografie
643
Caseta 17.1 proiecțiile privind schimbările Climatice
și impactul
615
Cutie 17.2 costurile Externe ale energiei
621
Cutie de 17.3 impactului asupra Mediului
assessment (EIA) matricea
625
Cutie 17.4 Unele definiții
627
Cutie de 17,5 Contrastante energie scenarii:
"Business as Usual" vs
Energy Revolution'
640
OBIECTIVE DE STUDIU

Apreciez factori socio-economici pentru noile
evoluții din domeniul energetic.

Înțeles soiuri de economică și
analiză a ciclului de viață.

Știu instrumente de politică pentru a încuraja regenerabile
de energie.

Înțeles fluxurilor de numerar actualizate.

Știu cum de sursele regenerabile de energie moderne început și
sunt susceptibile de a continua.
TWIDELL PAGINARE.indb 612
01/12/2014 11:38

www.shahrsazionline.com
Lista de tabele
613
LISTA DE FIGURI
17.1 Proiectat de temperatura se ridică pentru o gamă largă de scenarii de emisie.
616
17.2 Câteva exemple de curbe de învățare', arată scăderea costurilor de energie din surse regenerabile și utilizarea
crește.
638
17.3 Schema curbele de cost pentru energia din surse regenerabile, convenționale (maro) de energie (a costat convențional),
și maro energie, inclusiv externe (sociale) costurile.
639
17.4 Două scenarii ilustrative pentru 2050 pentru dezvoltarea (a) cererii de energie, (b) CO
2
emisiile
din surse de energie, (c) proporția de energie din surse regenerabile (RE) în oferta globală de energie.
640
LISTA DE TABELE
17.1 Unele estimări ale costurilor externe de energie electrică din cărbune sau nucleare (în USc/kWh).
622
17.2 Totalizare a impactului de mediu al energiei eoliene.
626
17.3 Prezent valoarea de incalzitor solar de apa factor în anul n.
632
17.4 Cost și de a beneficia de fluxuri de la o fermă eoliană.
634
17.5 Evoluția tehnologică, mediul economic și politic pentru " noi " energie din surse regenerabile
sisteme. 637
TWIDELL PAGINARE.indb 613
01/12/2014 11:38
614
Instituționale și economice factori
§17.1 INTRODUCERE
Capitolele anterioare au fost dat impresia că inovarea
și aplicare depinde doar de știință și inginerie. Cu toate acestea, o astfel de
opinie este extrem de naivă; evoluțiile practice depind de aproximativ 75%
pe 'factori instituționali și doar aproximativ 25% în știință și inginerie.
De 'factori instituționali' sunt conduse de politicieni, planificatorii, finantatori,
avocații, oamenii de stiinta sociale, mass-media, publicul și, din cauza etice,
religioase și valori culturale, artiști, autori, teologi și
filosofi. Oamenii de știință și inginerii să devină mai influentă atunci când vor
participa la aceste alte zone.
Aici, vom revizui unele dintre cele socio-politice și economice factorii
care influențează sistemele energetice. De obicei plin externe și societale costurile
de energie convenționale nu sunt incluse în preț (de exemplu, poluare, a se vedea
Caseta 17.2), care distorsiunile ales împotriva mai multor sisteme durabile de energie,
inclusiv surse regenerabile de energie. Instrumente de politică care acest atac sunt explicate. §17.6
prezintă economice și contabile, metodele de cuantificare alegere, inclusiv a
fluxurilor de numerar actualizate. În sfârșit ne-am examina cum tehnologice,
sociale, politice, economice și de mediu pentru sursele regenerabile de energie a evoluat și
este încă în evoluție. Sursele regenerabile de energie sunt sectoare de creștere a economiilor, cu
potențialul de a oferi în mod durabil cea mai mare din lume de energie de la mai multe
milioane de site-uri; acest lucru necesită cunoștințe, viziune, experiență, finanțe,
piețe și alegerea, în funcție bună știință și tehnologie.
§17.2 SOCIO-POLITIC FACTORI
Acțiune în cadrul societății depinde de mulți factori, inclusiv cultura,
tradițiile, cadre politice și de finanțare. Astfel de influențe variază foarte mult
și de a schimba cu timpul; de asemenea, ele se referă la disponibilitatea și gradul de conștientizare
de tehnologie.
§17.2.1 politică energetică Națională
Socio-politice factorii care influențează politica energetică, în special pentru sursele regenerabile de energie,
inclusiv, în ordinea aproximativă a importanței:
1
Securitatea energetică. Economiile nu poate funcționa fără fiabile și
disponibile în mod continuu livrările de energie și carburanți, energie electrică și termică.
Importate de alimentare cu energie este vulnerabil la o perturbare de război, comerț
sancțiuni și prețul crește, ca instanced istoric de mai multe ori. De exemplu,
multe țări de import de combustibil de ulei costă 30% sau mai mult din PIB, făcându -
le vulnerabile economic (§17.2.2). Cu toate acestea, fiecare țară are
propriul său set distinct de resursele de energie regenerabile în cadrul
territorial limite; astfel, folosind aceste pentru a reduce importurile de combustibili fosili
crește securitatea aprovizionării împotriva ostil și perturbări ale pieței,
și diversifica opțiunile.
TWIDELL PAGINARE.indb 614
01/12/2014 11:38
§17.2 Socio-politic factori
615
2
Costul de optimizare înseamnă, de obicei, preț scăzut consumatorilor într -
o piață competitivă fără includerea costurilor externe (de exemplu,
poluarea). În plus față de evidentă a costurilor de aprovizionare, prețurile de consum este
puternic influențată de impozite, subvenții, monopol influențe și
furnizor profit. Cutii 17.2 și §17.6 descrie metode pentru costul economic
comparații de surse regenerabile de energie (capital inițial mare cost, dar la un nivel scăzut
- cost), cu combustibili fosili sisteme (inversă).
3
Durabilitatea și schimbările climatice. Cum s-a discutat în §1.2,
problemele de mediu trebuie să fie luate în considerare, inclusiv preocupare la nivel mondial pentru
dezvoltarea durabilă și schimbările climatice. Baza pentru aceasta din urmă a fost
Convenția-Cadru a ONU privind schimbările Climatice (UNCED din 1992)
și sale asociate Protocolului de la Kyoto (1997). Următoarele acestea, aproape toate
țările au luat unele măsuri pentru a reduce, sau cel puțin reduce creșterea
de', lor de gaze cu efect de seră și să prezinte un raport privind progresele înregistrate pentru acest lucru.
Deoarece principala sursă de gaze cu efect de seră emisiilor de CO
2

din
arderea combustibililor fosili (a se vedea Caseta 2.3), aceasta încurajează utilizarea eficientă a
energiei și o creștere a energiei din surse regenerabile pentru a atenua negative
de impact (a se vedea Caseta 17.1 și IPCC Synthesis Report 2007 și 2014).
CASETA 17.1 SCHIMBĂRILE CLIMATICE: PROIECȚII ȘI EFECTE
Fondul științific la efectul de seră și semnificația de gaze cu efect de seră (Ges) sunt
prezentate în §2.9.
Agențiile internaționale înregistra cantități de combustibili fosili arși la nivel global și, prin urmare, masa de
emise gaze cu efect de Seră, în special CO
2
. În plus, creșterea concentrației atmosferice de CO
2

este măsurată
direct la locații de la distanță pentru media globală. Diferența permite calculul constantei de timp
pentru a elimina CO
2

din atmosferă prin procese naturale, în principal mare de absorbție. Predictii viitoare


ale emisiilor de GES permite modele climatice pentru a calcula consecință, "forțând" de medii globale la suprafață
temperatura (GMST), după cum se explică în §2.9. Subliniem că fizica de absorbție în infraroșu de
gazoși CO
2

este exact și a stabilit știință. Asociat probleme științifice, inclusiv magnitudinea de


efecte de feedback, variația climatelor regionale și temperatura mării, sunt mai dificil de analizat; de
exemplu, GMST poate crește ca o medie la nivel mondial, dar schimbările climatice pot provoca unele regiuni
pentru a deveni mai rece. Extreme de vreme sunt, de asemenea, a prezis să se schimbe.
Viitorul anuală a emisiilor de GES sunt dependente de diverse viitorul factori, inclusiv de condițiile economice,
a populației și a cererii de energie, furnizarea și utilizarea finală a tehnologiilor. Raportul Sinteză al IPCC
(2007) are o serie de scenarii care acoperă astfel de factori, cu estimarea emisiilor și schimbările climatice.
Fiecare scenariu este o descriere plauzibilă a viitorului corespunde un anumit set de ipoteze
("linie de poveste"). Folosind aceste scenarii în modele climatice globale, GMST se preconizează o creștere între
1.1°C și 6.4°C mai mult decât 1980/1999 medie (Fig. 17.1).
Deși o schimbare în GMST de aproximativ 5°C poate părea neimportantă, care este egal cu diferența în GMST
între vârf de la ultima Eră Glaciară și acum. Apoi, nivelul mării a fost de 120 de metri mai mic decât acum și
locație din New York a fost sub 1000 de metri de gheata! Acest lucru implică faptul că un nivel global schimbările de
temperatură din
intervalul prezis de către IPCC are implicații semnificative.
O creștere în GMST de 4°C la 5°C, cu modificările asociate în precipitații, temperatura mării și a altor
factori climatici, ar avea consecințe asupra ecosistemelor, aprovizionarea cu apă, produse alimentare, zonele de coastă și de
sănătate, care
ar fi inacceptabil într – adevăr periculos – pentru o mare parte din populația lumii. Aceste
TWIDELL PAGINARE.indb 615
01/12/2014 11:38
616
Instituționale și economice factori
include sute de milioane de persoane expuse la apă a crescut de stres (de exemplu, seceta din Africa) și
mai multe milioane expuse la inundațiile din zonele de coastă în fiecare an, în joase regiuni, cum ar fi delta și atoli. La
consecințele frecvente apă sărată inundațiile amenință supraviețuirea atoll națiunilor. Naturale și a culturilor
ecosistemelor ar avea de suferit semnificativ extincția atât terestre și marine, plante și animale, cu
aproximativ 30% din globală umede de coastă a pierdut, astfel, de exemplu, în scădere de cereale productivitatea în low-latitudine
regiuni (de exemplu.g. orez). O crescut substanțial povara asupra serviciilor de sănătate (de exemplu, extinderea răspândirii
malariei) este, de asemenea, proiectat (IPCC GL2 2007). Migrații umane ar fi mare.
Pământul, considerat ca un sistem, are condiții controlate de mediu și ecologice
parametri. De exemplu, cu acoperire nor, o creștere în GMST crește evaporarea, care crește
cloud, care reflectă mai insolație, care scade GMST; acest lucru ar fi un feedback negativ
de control. Cu toate acestea, a crescut de vapori de apă din Atmosferă crește de absorbție în infraroșu, care
crește temperatura, care este feedback-ul pozitiv. Alte răspunsuri au feedback pozitiv, cum ar fi
gheața Arctică se topește, care crește întuneric zona mării și scade reflectorizante suprafață de gheață, care crește apa
temperatura, care se topește mai gheata (ca în ultimii observații: Fig. 2.19(c)). În astfel de sisteme, unele
efecte devin mai intense și dincolo de controlul normal dacă anumite 'puncte maxime' sunt trecut; exemple
pot fi ireversibile de topire din Groenlanda de gheață și pe scară largă eliberarea de metan din
permafrost regiuni. Deși creșterea precisă în GMST necesare pentru aceste puncte de basculare este incert,
schimbări catastrofale ar putea să apară ca urmare a creșterii în GMST de 4°C sau mai mult (Schellnhuber et al. 2006;
Smith et al. 2009).
Unele creșteri mari în GMST sunt în intervalul de previziunile prezentate sintetic în Fig. 17.1, și ar putea
apărea dacă globale utilizarea combustibililor fosili continuă să crească fără constrângere. IPCC (2007) a concluzionat că
restricționarea GMST crește la 2°C la 2,4°C necesită atmosferice GES concentrații să fie în intervalul de
140
120
100
Emisii istorice
Prezent și viitor scenarii
80
60
40
20
-20
1940
1960
1980
2000
2020
2040
2060
2080
2100
0
An
Final
temperatura
crește (°C)
4.0

4.9 °C
2.4

2.8 °C
Lumea CO
2
emisiile (GtCO
2
/y)
Eu : 445-490 ppm CO
2
–eq
II : 490-535 ppm CO
2
–eq
III : 535-590 ppm CO
2
–eq
IV : 590-710 ppm CO
2
–eq
V : 710-855 ppm CO
2
–eq
VI : 855-1130 ppm CO
2
–eq
post-MO gama
Fig. 17.1
Proiectat de temperatura se ridică pentru o gamă largă de scenarii de emisie. Axa verticală este anual
emisiile globale de CO
2

, care este majoră de GES. Fiecare bandă corespunde o (îngust)


gama de proiectat stoc de gaze cu efect de Seră în atmosferă până în anul 2100, reflectând
rezultatele la o serie corespunzătoare de scenarii analizate de către IPCC. Creșterile de temperatură
indicate în partea dreaptă a diagramei pentru două benzi crește în GMST de mai sus
'pre-industriale' (temperaturi curente sunt deja ~0.7°C peste nivelul pre-industrial). Punctată
linii indica diferite scenarii.
Sursa: Adaptat de la IPCC Synthesis Report ( 2007), fig. SPM-11.
TWIDELL PAGINARE.indb 616
01/12/2014 11:38
§17.2 Socio-politic factori
617
4
Sănătate și siguranță. Ca și alte instalații de energie, precum energia nucleară,
termocentrale, rafinării de petrol și de transport de înaltă tensiune linii,
instalații de energie regenerabilă pot fi periculoase, cu ușor de recunoscut
dificultate în menținerea siguranței la multe și locații dispersate.
De lucru de lângă utilajele rotative electrice și sisteme de alimentare,
alpinism structuri și manipulare materiale combustibile prezente pericole.
În practică, multe din surse regenerabile instalații la scară relativ mică
a operațiunilor, astfel încât personalul implicat în mai multe variate sarcini. Deși
acestea oferă interesant și responsabil, o astfel de variație
prezintă pericole.
Poluarea poate fi definită ca efecte negative, de obicei, chimice
emisii, care nu sunt prezente în mediul natural. Fosile și nucleare
proceselor energetice (maro energie: Capitolul 1) concentrat și apoi
emit substanțe chimice și radiații ionizante fi poluarea. Precursorii de
această poluare au fost deja prezente în materii prime, care
au fost, cu toate acestea, "în siguranță" în subteran. În general, sursele regenerabile de energie (a căror
energie să provină de la persoane fizice a fluxurilor de energie) de a evita pe scară largă
a riscurilor de poluare pentru sănătate asociate cu maro aprovizionarea cu energie. O
excepție este arderea incompleta a biomasei, care este comună
de ardere lemne de foc sau slab reglementate mașini care utilizează biocombustibili
(a se vedea §10.10).
5
Interese. Un puternic factor politic în multe țări este
moștenire de combustibili fosili și a industriilor nucleare, care caută să-și protejeze
activele și să-și păstreze poziția dominantă în infrastructura energetică.
Banii din acest energetice industriale complexe de autovehicule
companii, industria cărbunelui, neiluminate utilități, petrol și gaze
companiilor a ascuns ecologice adevărul de situația în care ne aflăm,
și a subminat capacitatea noastră de a inginer inteligent politicile necesare
pentru a stabili aprovizionarea durabilă cu energie.
6
Condițiile economice. Relativ mari costurile de capital și împrumuturile inițiale
pentru sursele regenerabile de energie necesită o relativ lungă perioadă de amortizare (de multe ori de la 10 la
15
ani și mai mult). Incertitudine economică, cum ar fi 'Financiare Globale
Crizei din 2008, descurajează potențialii investitori; cu toate acestea, s-a stabilit
economii, cu rate mici ale inflației și ale dobânzii de împrumut, serviciu
de astfel de investiții de capital. Instrumente de politică disponibile pentru guvernele includ
legislația pentru a forma structura piețelor de energie și de planificare
a procedurilor (§17.5).
445 la 490 ppm CO
2
-eq. Acest lucru necesită emisiile globale de CO
2

: (i) să scadă cu 50 la 85% față de cele din


2000 până în 2050; și (ii) pentru a începe să scadă, nu mai târziu de 2015, așa cum este indicat în Fig. 17.1. Astfel scade
legate de utilizarea combustibililor fosili nevoie de o mare, dar este posibil, extinderea surselor regenerabile de energie.
TWIDELL PAGINARE.indb 617
01/12/2014 11:38
618
Instituționale și economice factori
§17.2.2 țările în curs de Dezvoltare
Factorii de mai sus sunt importante pentru politica peste tot; cu toate acestea,
unele extra societale și de factori instituționali se aplică în curs de dezvoltare
de țări. Țările în curs de dezvoltare nu au moștenirii istorice de mari dimensiuni
stabilite infrastructurilor energetice (de exemplu, rețelele naționale de electricitate), nici
economică și suport tehnic imediat să stabilească și să mențină
o astfel de infrastructură. Livrările de energie tind să fie instalat în primul rând în
marile orașe în comparație cu mediul rural, dar chiar și în orașe,
de aprovizionare pot fi neregulate și în imposibilitatea de a satisface cererea. De exemplu,
Africa subsahariană (cu excepția RSA) cu o populație de aproximativ 800 de milioane
generează în prezent aproximativ la fel de mult de energie electrică și Elveția, care are o
populație de 8 milioane de euro. Astfel, deși consumul național de
comerciale de energie poate fi relativ mici pe cap de locuitor, potențial și
o creștere preconizată a consumului de energie ridică probleme economice și
sustenabilitatea ecologică, după cum se consideră în Capitolul 1. Costul de import
de combustibil poate fi un macroeconomice semnificative constrângere. Guvernul
subsimoare reducerea prețului combustibililor fosili pentru consumatori, probabil, scăderea
cheltuielilor pentru sănătate, educație și dezvoltare rurală. Durabilă
calea spre viitor cu energie din surse regenerabile și eficiența energetică ar trebui să
atenua multe astfel de dificultăți pe termen lung.
De obicei, o mare parte din populație trăiește în comunitățile rurale -
legături nevoie de îmbunătățirea aprovizionării cu energie (de exemplu, din cauza ineficiente
lemn de foc și de iluminat). De multe ori femeile din sate de mers pe jos mulți kilometri
în fiecare zi să-i aducă apă și lemne de foc. Astfel de "sărăcie energetică"
handicapuri furnizarea de apă curată, de telecomunicații și de studiu acasă.
Acest lucru perpetuează inegalitatea socială și neagă sociale și economice
avansare.
Potențialul cererii mici de energie electrică din mediul rural și insula
gospodării de multe ori nu justifică guvernul plătește costul de rețea
de electrificare pentru servicii esențiale, cum ar fi lumini, televiziune, radio și aparate
(de exemplu, scule de mână, mașini de cusut și pompe de apă). Cu toate acestea,
corespunzătoare furnizare de energie regenerabilă pot oferi, de obicei, aceste servicii dacă
sunt combinate cu energia-dispozitive eficiente. Fotovoltaice de putere este aproape
întotdeauna aplicabile (§5.7), cu eoliene mici (§8.8.6) și hidro (§6.6)
turbine cu cele mai benefice dacă condițiile locale sunt favorabile. Generatoare Diesel
alimentate cu biocombustibili sunt, de asemenea, posibil, dar nu (încă) comun. De microfinanțare
aranjamente oferi împrumuturi cu dobândă mică pentru a răspândi costul de-a lungul timpului;
de rambursare este de obicei întâlnit de economii pe achiziționat combustibili (de exemplu, kerosen),
și câștigurile pe activitățile de afaceri alimentat de noi surse regenerabile
de alimentare (de exemplu, mașini-unelte și mașini de cusut). Este important
să apreciem că la nivel mondial crește de micro-generare cu grila de
conexiune încurajează scară similară cu tehnologie baterie de stocare
înlocuirea grilei.
TWIDELL PAGINARE.indb 618
01/12/2014 11:38
§17.2 Socio-politic factori
619
§17.2.3 Rolul individului
Creșterea energiei din surse regenerabile din criza petrolului din 1973 (când Organizația
Țărilor Exportatoare de Petrol (OPEC) drastic reduse de combustibil exporturile
din motive politice) datorează mult la acțiuni de către persoane fizice și mici
comunități care doresc independența și creativitatea pentru propria lor energie
de alimentare de la tinerei regenerabile. Mult de aspirația lor legate de
utilizarea corespunzătoare și în tehnologia intermediară, care a fost prevăzut
pentru a beneficia atât 'supradezvoltat" și "subdezvoltate " lume". Peste
următorii 10 ani, ideile lor și succesele pătrunsă în sus prin
societate, devenind puternice influențe în universități, comunitățile
și tehnologie în curs de dezvoltare. Acest lucru a fost ajutat de acțiune colectivă prin
consolidarea și inițierea de asociațiile profesionale și comerciale,
și prin publicații de specialitate și reviste. Astfel, de la sfârșitul anilor 1980,
mari companii și guverne au devenit implicat serios.
Această creștere a amestecat în 1990, cu necesitatea de a reduce utilizarea de combustibili fosili
combustibili din cauza lor a emisiilor de GES, astfel încât până în anul 2000 marea majoritate
de guverne (în special, inclusiv cele din Uniunea Europeană) au
vizat politicile de promovare a surselor regenerabile de energie; cu toate acestea, multe
guverne menține în continuare politicile de asistență producția de maro energie.
Până în 2010, a fost evident în cele mai multe țări care sursele regenerabile de energie moderne sunt
aici pentru a rămâne.
Totuși, activitatea și rolul individului și organizațiile mici sunt încă
vitale. Tehnologii sunt disponibile pentru persoane fizice, întreprinderi mici,
cooperativelor și a comunităților pentru a produce și/sau de cumpărare toate lor de energie
are nevoie de surse regenerabile de energie. Mulți sunt în măsură să exporte în exces, în special
energie electrică, și toate sunt susceptibile de a crește eficiența de utilizare a energiei. Astfel de
auto-suficiență și independență' oferă, de obicei, pe termen lung semnificative
economii de costuri, mai ales în cazul în care guvernele să ofere stimulente (de exemplu,
subvenții și tarife feed-in). Aceste informat cetățenilor poate avea zero
și negativă amprenta de carbon, poate ușura preocuparea lor despre fosilă și
nucleară combustibili, poate ridica moralul lor și se pot da o mai mare
securitate pentru viitor.
Experiența arată că astfel de sustenabilitate este ajutat de următoarele:

Măsurarea și monitorizarea utilizării resurselor, astfel încât enumerarea de carbon
footprint; simplul act de monitorizare cu feedback-ul de informații aproape
întotdeauna duce la mai puține resurse (de exemplu, energie electrică), fiind utilizat, cu
economie de ~10% justifice necesare instrumente și efort.

Folosind aparate eficiente energetic și, în special, îmbunătățirea energetică
eficiența de case și întreprinderi.

Călătoresc cu transportul public, mersul cu bicicleta și mersul pe jos.

Alegerea vehiculelor cu consum redus, inclusiv cei cu electrice si
motoare cu biocombustibil.

Limitarea călătoriilor aeriene și utilizarea de comunicații electronice (de exemplu prin intermediul
internetului).
TWIDELL PAGINARE.indb 619
01/12/2014 11:38
620
Instituționale și economice factori

În mod constant, folosind economii financiare pentru a crește investițiile în energia din surse regenerabile
(de exemplu, biomasa-sobelor si cazanelor, micro-generare, construirea
de izolare, iluminat cu LED, masina electrica, membru de auto-ajutor de energie
cooperative, etc.).

Aderarea adecvate locale și grupuri naționale, nu în ultimul rând pentru a continua
"jos în sus" lobby asupra guvernelor.

Exportul și vânzarea de excesul de micro-generat de energie electrică și combustibili, astfel încât
scăderea amprentei de carbon.
§17.3 ECONOMIE
§17.3.1 Bază
Economie urmărește să analizeze și să dezvolte instrumente pentru persoane fizice,
organizații și guverne pentru a lua decizii raționale cu privire la lor
de alocare a resurselor limitate. Dominantă parametru utilizat este de bani,
cu toate acțiunile având atât beneficii cât și costuri; evaluările caute pentru a
găsi care dintre mai multe opțiuni alternative, inclusiv status quo-ul, are
cele mai favorabile echilibru. Mai multe întrebări apar imediat în
contextul sistemelor de energie regenerabilă:
1
Ale căror costuri financiare și beneficiile vor fi evaluate: proprietarii,
utilizatorii finali sau cele ale națiunii sau lumea ca un întreg? De
exemplu, costurile efective a daunelor cauzate de poluare emis de un
sistem centralizat de ardere a cărbunelui energie electrică stație de putere (de coroziune de ploi acide,
schimbările climatice de gaze cu efect de seră, de curățare contaminate
efluenți, etc.) sunt cea mai mare parte nu sunt incluse în cadrul financiar intern conturile
de generatoare de electricitate a companiei sau clienților săi, dar sunt plătite
de către alții. Acestea sunt numite costuri "externe", așa cum este descris în Caseta 17.2
privind energia. Mai mult decât atât, există și alte efecte nedorite ale
emisiilor (de exemplu, pierderea biodiversității) care nu poate fi identificat în
orice financiar-contabil. În schimb, energie fotovoltaica nu produce
emisii și a redus costurile externe. Dacă PV putere înlocuitori
pentru utilizarea de cărbune, apoi economii reale sunt realizate în societate; totuși, aceste
economii nu poate reveni PV generator al cărui financiare provocare
este să plătească pentru costurile de capital sistemului PV. Această comparație
favorizează combustibili fosili stație de putere, deoarece costurile externe nu sunt
incluse.
Există multe controverse despre cum să pună o valoare monetară pe
mulți dintre factorii relevante pentru surse de energie regenerabile, cum ar fi
a avea un mediu mai curat decât în caz contrar. Pentru că astfel de factori
au fost greu de cuantificat, ele au fost adesea luate în considerare,
în detrimentul celor care promovează sisteme de energie regenerabilă.
2
Care parametri sau sisteme ar trebui să fie evaluate: primare,
surse de energie sau final-utilizare serviciu? De exemplu, gospodari
TWIDELL PAGINARE.indb 620
01/12/2014 11:38
§17.3 Economie
621
iluminat casele lor pe timp de noapte sunt interesat de costul și cantitatea de
iluminare, mai degrabă decât energia ca atare. Costul de o baterie de ceas este
considerat niciodată în termeni de Wh livrate, dar întotdeauna în termeni de
servicii oferite pentru a cunoaște timpul.
3
În cazul în care evaluarea se aplică? Costurile de instalat din surse regenerabile
sisteme de energie (SRE) sunt specifice site-ului. Deoarece acestea sunt concepute pentru a
robinet în naturale existente fluxurilor de energie (Capitolul 1), este evident că
un anumit RES va fi favorabil unde corespunzătoare fluxului deja
există, și nefavorabil în caz contrar. Astfel hidroelectrice sisteme sunt
doar practice și viabile din punct de vedere economic în cazul în care există un debit suficient
de apă. Costul de o bază de biomasă sistem depinde de
disponibilitatea și costul de biomasă; dacă acest lucru este deja pe site-ul și deșeuri, ca într -
o trestie de zahăr mill (Caseta 9.2), operațiunea este mult mai ieftin decât atunci când
biomasa trebuie să fie achiziționate și transportate la site-ul.
4
Atunci când sunt costurile și beneficiile să fie evaluate? Din surse regenerabile de energie
sisteme au, în general, mici, costurile operaționale și mari inițială,
a capitalului, costul. Combustibili fosili planta are inversa, mai ales dacă nu există
emisii de prevenire. Economiștii au dezvoltat instrumente pentru
combinarea viitor și continuă cunoscut costuri cu costurile inițiale, cum s-a discutat
în §17.6.
De la un termen mai lung de vedere economic, o critică întrebarea este: ce
costuri suplimentare ar trebui să fie atribuită folosirea acum de o resursă care poate
deveni extrem de limitate pentru generațiile viitoare? Nu există încă nici un
acord răspunsuri.
Diferite puncte de referință (1) - (4) dă răspunsuri foarte diferite
la întrebarea dacă un anumit sistem de energie regenerabila este
"economic". Ca un profesor de economie este raportat că au spus (atunci când este
provocat despre repetarea identică întrebările de examinare de la an
la an): În economie, este de răspunsuri, care se schimbă în fiecare an, nu la
întrebări!'
CUTIE 17.2 COSTURILE EXTERNE ALE ENERGIEI
Costurile externe sunt reale și a costurilor reale rezultate dintr-un proces, dar care nu sunt incluse în prețul
produsului și, prin urmare, trebuie să fie plătite de către public. Generarea de energie electrică din cărbune sau nucleare pot
avea un impact semnificativ costurile externe. De exemplu, arderea ridicat de sulf, cărbune produce ATÂT de
2

emisiilor, care
dau "ploii acide", care provoacă daune pădurilor, structuri metalice și patrimoniu clădiri de piatră. Particule
emisii pot provoca boli pulmonare. Costurile de eliminare și depozitare pe termen lung a deșeurilor nucleare sunt
, de obicei, un semnificativ costurile externe (deoarece în practică semnificativ costurile sunt plătite de către guvernele ulterioare
din impozitele generale și nu din vânzarea de energie electrică), precum și la costurile de cercetare și dezvoltare de
reactoare (care în multe țări a atras pe apărare și bugetele de cercetare). Utilizarea autovehiculelor în mod similar
majoră a costurilor externe, care rezultă din schimbările climatice, smog, pe teren productiv "pierdut" la drumuri, și la
sănătate și pierderile de productivitate cauzate de leziuni și decese în accidente rutiere.
La începutul anilor 1990, au existat mai multe studii mari care a încercat să evalueze numeric, cum ar
externalități, în special în producția de electricitate. Unele orientative rezultatele sunt prezentate în Tabelul 17.1. La
TWIDELL PAGINARE.indb 621
01/12/2014 11:38
622
Instituționale și economice factori
§17.4 ANALIZA CICLULUI DE VIAȚĂ
Analiza ciclului de viață (LCA) enumeră consecințele asupra mediului
asociate cu fabricarea, funcționarea și dezafectarea
unei anumite acțiuni sau de construcție. Atât aspectele interne și cele externe
sunt incluse pentru întreaga viață și consecințele acestui proces.
Astfel, pentru o turbină eoliană, LCA include fabricarea de componente
din nou și materiale reciclate, extracția de minereuri, obținerea de noi
materiale, impactul asupra mediului de astfel de minerit și de preparare,
fabrica de construcție și întreținere, aprovizionare cu energie, în construcție,
operare și dezafectare, etc. Reducerea analiză a
cuantificabile sumele pentru analiză matematică necesită unități comune, care
este de multe ori bani, dar poate fi, de asemenea, masa, energia înmagazinată sau cu efect de seră
gaze (GES) pe unitatea de producție (energie) de-a lungul ciclului de viață al
sistemului. Lca completa evaluările economice, care se concentrează pe
costurile curente.
rezultate acoperă o gamă largă, care reflectă nu numai dificultăților metodologice, dar, de asemenea, faptul că, în unele
țări centralele electrice sunt în populat zone (deci o anumită cantitate de poluare va provoca daune mai mult
la oameni și clădiri), iar în altele cărbune are mai puțin de conținutul de sulf sau este ars mai eficient (atât
provoacă mai puține emisii). Tabelul 17.1 include, de asemenea, ceva mai târziu, estimări de cost potențial de clima
se schimba, bazat pe IPCC estimeaza de carbon taxele necesare pentru a reduce emisiile să îndeplinească obiectivele de
la Kyoto. Este de asemenea posibil, în principiu, pentru a estima costurile de daune din cauza schimbărilor climatice
schimba (dar acestea sunt extrem de sensibile la rata de actualizare de peste 100 de ani sau mai mult) sau din
costurile de adaptare (construirea de ziduri mare și diguri, etc.).
SRREN (2011) rezumă altele, mai târziu, de studii, de care de mult aceleași concluzii.
Tabelul 17.1
Unele estimări ale costurilor externe de energie electrică din cărbune sau nucleare (în
USc/kWh). Pentru a compara aceste tipic de energie prețurile de vânzare cu amănuntul atunci de 5-10 USc/kWh. Bazat pe ONRL
(1994); Comisia Europeană (1995) și Hohmeyer (1988).
Efect
USc / kWh
Note
CĂRBUNE energie ELECTRICĂ
ploi Acide (de la ASA
2
)
0.02 la 20
Mai mare estimare este de mare de sulf, cărbune în
zonele urbane.
Schimbările climatice (de la CO
2
)
0,4-12
Mai mare estimare își asumă nici emisiilor
de tranzacționare.
Energia NUCLEARĂ
Subvenții pentru R&D
1.2
Impactul asupra sănătății de accidente
0.1 la 10
Costul de protejare a deșeurilor
Necunoscut
Peste mii de ani.
Locul de reactoare
Site-ul și societatea specifice. General și utilizare la nivel mondial.
TWIDELL PAGINARE.indb 622
01/12/2014 11:38
§17.5 instrumente de Politică
623
În măsura în plin de factori să ia în considerare este enorm, dar
'pe unitatea de impact devine mai puțin, eliminat în continuare factor. În practică,
numai influențe semnificative sunt incluse, dar, de obicei, există o dezbatere despre
ce sunt acestea și cum sunt evaluate și cuantificate; de exemplu,
(a) impact vizual, evaluate, probabil prin modificarea locală a prețurilor la locuințe, sau
(b) ocuparea forței de muncă, evaluată poate de schimbări în guvern plăți
de "șomer". O dificultate deosebită pentru resurse finite este de a evalua
valoarea nu folosindu-le, după plecarea din resurse subterane
dispare poluare, și încă resurse ar putea fi utilizate de viitoarele
generații. Pentru sursele regenerabile de energie, dificultatea este cum de a evalua implicațiile de
variabilitatea naturală a resurselor (de exemplu, pentru energia eoliană fiind absent
în zile fără vânt
1

). Ipoteze și de a schimba caracteristicile


de fond a sistemului energetic (de exemplu, intensitatea carbonului) afectează Lca de cele mai multe
RE tehnologii, deoarece ciclul lor de viață impactul stem aproape în întregime din
componente de fabricație. Printre provocări se numără potențialul de
dublă contabilizare atunci când se evaluează mari interconectate sistemele de energie,
și a sistemului de limita probleme.
Graficul D3 în Anexa D prezintă estimări ale ciclului de viață GES sime-
sion (g CO
2

eq /kWh) pentru categorii largi de generarea de energie electrică


tehnologii, inclusiv 'verde' și 'maro' tehnologii. Două elemente
ies în evidență: (a) emisiile de GES din combustibili fosili sisteme sunt mai mari cu un
ordin de mărime decât cele din surse regenerabile sisteme;
cu toate acestea, este important să se cuantifică diferența. (b) Pentru fiecare tehnologie,
estimările acoperă o gamă largă din cauza problemelor ridicate în
alineatul precedent și din cauza tehnologice diferite caracteristici
(de exemplu, proiectare, factor de capacitate, variabilitatea, serviciu de viață și de epocă,
locație geografică, fundalul sistemului energetic caracteristicile, sursa de date,
LCA tehnica, co-produs de alocare, emisii evitate, și a sistemului de
limite). Economia nu este o știință exactă!
§17.5 INSTRUMENTE DE POLITICĂ
Tabelul 17.1 indică faptul că costurile externe ale non-regenerabile de energie electrică
generație de combustibili nucleari și fosili sunt în jur de prețuri de
energie electrică facturat consumatorului, adică interiorizarea ei ar fi, probabil,
dublul prețurilor de consum. În consecință, nu de internalizare a externalităților
nu pe deplin încurajarea consumatorilor să utilizeze energia electrică în mod eficient și
nu încurajează utilitare pentru generarea din surse regenerabile de energie.
§17.5.1 politicile Guvernamentale
Guvernele folosesc diverse instrumente de politică. Avem în vedere trei metode de'
aici. Metoda 1: costurile externe sunt incluse de către non-regenerabile de energie
generatoare a prețurilor lor. Metoda 2: (a) subvențiile sunt acordate pentru energiile regenerabile
TWIDELL PAGINARE.indb 623
01/12/2014 11:38
624
Instituționale și economice factori
generatoare; de asemenea, (b) subvențiile pot fi eliminate de la non-regenerabile.
Metoda 3: se acordă subvenții. Exemple sunt după cum urmează:
1
Tehnologice de eliminare a poluantului: Metoda 1. Combustibili fosili
generatoare pot plăti pentru extragerea poluării la sursă, deci reducerea
costurilor externe. Acest internalizează costurile externe și ridică
prețurilor de consum. De exemplu, în multe țări, legislația convingătoare
'de desulfurare a gazelor de ardere' a redus în mod semnificativ ATÂT
2
emisiile
și, prin urmare a "ploii acide". Concurența pe piața de energie electrică beneficii
generator care elimină poluarea, la un cost minim.
2
Metoda 1 taxe de Mediu:. Impunerea unei taxe pe carbon pe combustibili fosili
energie electrică și, probabil, folosind veniturile pentru subvenționarea
energiei regenerabile. Subvenția este în vigoare o plată pentru surse regenerabile de energie pentru
reducerea poluării. Dificultăți includ: (a) stabilirea unei
adecvate numerice de încărcare, având valorile din Tabelul 17.1; (b) reticența
consumatorilor (alegătorii) să plătească mai mult; (c) afaceri internaționale
competitivitatea dacă unele țări au o taxa de carbon și altele nu; (d) dacă
taxa este prea mic, poluare continuă.
3
Negociabile de emisii permise (certificate): Metoda 1. Guvernul dă
autorizații și certificate pentru industria pentru sumele vizate de poluare lor,
cu totalul reducerea fiecare an. Aceste certificate sunt returnate la
o agenție centrală ca poluarea este emis. O industrie care poluează
dincolo de permise țintă, fie plătește o amendă de la guvern sau de
achiziții de rezervă permise de industrii care au reușit să reducă
poluarea lor și așa au liber permite să-și vândă.
4
Metoda 2b. Eliminarea subvențiilor pentru surse poluante: În multe
țări, generarea de energie electrică din combustibili fosili și/sau de energie nucleară
este subvenționată pentru motive sociale (de exemplu, prin reduceri de taxe sau subvenții).
Scăderea aceste subvenții reduce o barieră pentru sursele regenerabile de energie. La
obiectivele sociale pot fi îndeplinite printr-plăți în favoarea bunăstării.
5
Subvențiile pentru regenerabile: Metoda 2a. Limitată în timp subvenții scopul de a
crește piața de energie regenerabilă, astfel încât să permită schimburile comerciale să crească și să
se maturizeze, în timp ce în același timp, recunoaște că aceste surse regenerabile de energie
abate poluante generație. Mecanismele includ: (i) facilități fiscale pentru
generatoare de energie din surse regenerabile, ca de multe ori folosite în SUA; (ii) legislația
obligă utilitati că o anumită proporție de energie electrică
trebuie să fie furnizate din surse regenerabile (de exemplu, marea BRITANIE și Australia), de
exemplu, prin licitație competitivă atât de puțin-cost sursele regenerabile de energie sunt
în mod preferențial adoptat, și/sau un mecanism pentru a oferi un maxim de
"plafonul" prețului; (iii) tarifele feed-in permite sursele regenerabile de energie generatoare să fie
plătite au crescut sumele pentru energia electrică exportată către rețea și
generate pentru ei înșiși; utilitatile sunt obligate să coopereze și metoda
este simplu din punct de vedere administrativ.
6
Publice de cercetare și granturi de dezvoltare: Metoda 3. În comun cu
cele mai inovare, companii și organizații de cercetare sunt acordate
TWIDELL PAGINARE.indb 624
01/12/2014 11:38
§17.5 instrumente de Politică
625
subvențiile pentru regenerabile de cercetare, dezvoltare și demonstrații.
Ajutor pentru surse regenerabile de energie în 1970 a anilor 1980, această măsură s-au dovedit
insuficiente pentru a depăși instituționale și financiare prejudecăți împotriva
surse regenerabile de energie (a se vedea §17.8).
Astfel de mecanisme de a promova ocurbă de învățare' pentru stabilirea surselor regenerabile
de tehnologie (§17.6). Exemplele noastre se referă la generarea de energie electrică, dar
legate de mecanisme există pentru a biocombustibililor în transport, și pentru
energyefficient clădiri. Un mix de politici este, în general, necesare pentru a aborda
diferite bariere, pentru a RE. Nu există nici o-size-fits-all' politica de potrivite pentru
toate țările, la toate ori. La examinare aprofundată de către IPCC SRREN (2011)
arată că, politici diferite sau combinații de politici pot fi mai multe
eficace și eficient în funcție de factori cum ar fi nivelul de
maturitate tehnologică, disponibilitatea de capital la prețuri accesibile, și locale și naționale
RE resurse de bază.
§17.5.2 Guvernamentale proceduri
Legislație. Toate guvernele au legi cu privire la aprovizionarea cu energie. Aceste legi
ar trebui să reglementeze securitate, diversitate, costuri, siguranta, structura pieței și
anumite efecte asupra mediului.
Procedurile de planificare. Guvernele stabilesc planificarea legislație și
proceduri, care variază foarte mult între națiuni și state. Deși
se poate să se implice îndeaproape în mare și influentă evoluții (de exemplu,
pe scară largă hidroenergie și energie eoliană offshore), deciziile cu privire
mici și mijlocii evoluții au tendința de a locui cu administrația locală.
În aceste zile, o parte importantă a procedurilor de planificare este luarea în considerare
de o Evaluare a Impactului asupra Mediului (Cutie 17.3). Drepturile democratice
poate oferi cetățenilor o influență considerabilă în planificarea
procedurilor, dar, de obicei numai de a prezenta argumente pentru factorii de decizie.
CUTIE DE 17.3 EVALUARE A IMPACTULUI ASUPRA MEDIULUI (EIA) MATRICEA
În practică, o EIM se va referi la naționale și locale, legile și reglementările de mediu, care variază foarte mult
de la o jurisdicție la alta, dar, de obicei, nevoie de mai multe detalii. Aici pur și simplu clasifica efectele esențiale
într-o matrice, folosind puterea vântului ca un exemplu.
Impactul poate fi pozitiv sau negativ, sau pot fi neutre.
Tabelul 17.2 prezinta o 3 x 4 matrice de factori, cu o " axă " pentru scala de impact (Global
~100.000 km, Regionale ~200 m, Local ~500 m), iar cealaltă axă pentru tipul de impact (chimice,
fizice, ecologice și 'emoționale'). Aceste categorii sunt evidente, în afară de 'emoționale', care acoperă
personale și psihologice răspunsuri de la persoane fizice făcut din cauza propriilor sentimente personale și
opinii. În fiecare dintre cele 12 categorii este introdus efectele relevante pentru o anumită tehnologie.
Tabelul 17.2 prezinta acestea pentru energie eoliană.
Notă cum o astfel de analiză seturi de factori pozitivi (de exemplu, securitatea energetică), alături de factori negativi
(de exemplu, zgomot acustic). De prea multe ori, factorii pozitivi sunt subestimate ca furia preia despre cele negative.
TWIDELL PAGINARE.indb 625
01/12/2014 11:38
626
Instituționale și economice factori
§17.6 CUANTIFICAREA ALEGERE
Instalarea din surse regenerabile de energie, ca orice dezvoltare, necesită un
angajament de bani, timp și efort. Alegerile trebuie să fie făcute, unele
financiare și altele etice. Nu va fi beneficiile, dezavantajele și
multe alte efecte. Unele decizii vor fi luate personal, altele pe
afaceri și pe criterii politice. Această secțiune examinează diferitele metode
utilizate pentru a analiza și cuantifica astfel de decizii. Cu toate acestea, este esențial să se
aprecia că nu există absolut sau "perfect" metode, în sensul
folosit în știință și inginerie. Persoanele fizice și societățile lor de a face
alegeri bazate pe diferite criterii. Să sperăm că analiza începe prin
discutarea valorilor și apoi cu ajutorul matematicii și a criteriilor economice
pentru a cuantifica luare a deciziilor.
§17.6.1 analiza de Bază
Pentru luarea deciziilor financiare, metode (1) - (4) de mai jos sunt, în esență,
'plic' de luare a deciziilor, utilizate pentru evaluarea preliminară.
Beneficiul acestei matrice nu este faptul că este absolut "adevărul", dar permite discuția să se concentreze obiectul de
subiect, într-ușor de perceput limitele.
Tabelul 17.2
Prezentarea tabelară a impactului de mediu al energiei eoliene, indicând efecte pozitive (
+),
acceptabil impactul (o) și negative (-)
Global
Regionale
Locale
chimice
+ nu CO
2
+ nu AȘA
2
,, nu, nu
×

+ nu fumez etc
+ nu de apă de răcire
+ combustibil de transport
fizice
+ securității energetice
o limita strat de vânt
+ nu radioactivitate
+ nu deșeuri
− radar
− cuptor cu microunde comm
o energie electrică gen.
+ acces deschis
+ grila de armare
− putere variabilitatea
− zgomot acustic
− TV
− marin coliziune (offshore)
ecologice
+ schimbărilor climatice reducere
− specii rare?
+ durabilitatea
− pasăre populației?
+ piscicultura
+ eco-compensare
+ agricultură (de exemplu, piese grele,
venit suplimentar)
− bird & lovitură de bâtă
+ eco-compensare
emoțională
(om)
+ impact vizual (pentru sustinator)
− sunshine flicker
− impact vizual (semnificativă
a populației)
TWIDELL PAGINARE.indb 626
01/12/2014 11:38
§17.6 Cuantificarea alegere
627
Numai dacă un proiect promițător pe aceste baze este în valoare de cotitură la
matematic mai sofisticate metode care implică numerar actualizate
a fluxurilor de cum este utilizat de către contabili și bancheri. Aceste tehnici sunt
întotdeauna utilizate în scopuri comerciale-proiecte de anvergură care necesită împrumut de la
o bancă. Cutie 17.4 enumeră unele definiții utilizate în astfel de analize.
1
Gut-simt. Cele mai personale și de familie, și un surprinzător
număr de afaceri și opțiuni politice, sunt luate pentru un
individ sau un grup ajung la o concluzie, instinctiv sau după discuții.
De obicei, dar nu întotdeauna, consecințele eșecului sunt mici, astfel încât
alte metode sunt inutile. Avea o gradina de legume sau de a instala o
sobă cu lemne într-o cameră de zi poate fi un exemplu, dar un alt
CUTIE 17.4 UNELE DEFINIȚII
Producător: persoană sau organizație de planificare și de coordonare a unui proiect (în acest caz, furnizarea și utilizarea de
energie din surse regenerabile).
Valoarea actualizată: valoarea viitoare tranzacție financiară din punctul de vedere al prezentului.
Energia înmagazinată (a unui produs sau serviciu, adică de un "bun"): totalul comerciale de energie cheltuită în
toate procesele și provizii pentru un bun, calculată pe unitate de bun. Notă: Prin această definiție soare
pe culturi nu este o parte din energia înmagazinată, dar căldura de ardere a comerciale a biocombustibililor este inclus.
De recuperare a energiei (timp): energia înmagazinată de producere a energiei, echipamente împărțită său anual
de producție de energie.
Capital: fonduri în proprietate de la producător; de obicei obținute din acțiunile vândute acționarilor de către o
societate cu răspundere limitată. Creditele nu sunt de capitaluri proprii, astfel încât finanțarea proiectului este suma dintre
capitalurile proprii și împrumuturi.
Costurile externe: reale și a costurilor reale rezultate dintr-un proces, care nu sunt incluse în prețul
produsului și așa trebuie să fie plătite de către public în caz contrar (de exemplu, ploi acide de cărbune centrale electrice de ardere
deteriorarea structurilor metalice). (A Se Vedea Caseta 17.2.)
Amprenta: impactul unei acțiuni (de exemplu amprenta de carbon): emisiile de dioxid de carbon din combustibili fosili care
decurg de
acțiune, de obicei pe an.
Inflația: o scădere generală în valoare de bani (creșterea prețurilor), de obicei, măsurată de către un resortisant
rata medie anuală a inflației eu.
Costurile interne: costuri incluse în prețul unui produs sau serviciu.
Cost nivelat (de exemplu, pentru producția de energie electrică): costul mediu de producție pe unitate de-a lungul vieții
de sistem, pentru a permite actualizarea în timp.
Împrumut de bani puse la dispoziția unui producător și care necesită, de obicei, plata dobânzilor de împrumut de la
creditor în plus față de rambursare a sumei împrumutate. Contractele de obicei stipulează că, în caz de
faliment, i-a împrumutat banii trebuie să fie rambursate ca o prioritate față de interesele altora (de exemplu, băncile care oferă
credite sunt rambursate în preferință pentru acționarii și furnizorii de bunuri).
Costurile de operare și întreținere (O&M): acestea pot fi fixe (de exemplu, chiria, periodică a personalului) sau
variabilă (de exemplu, piese de schimb, contract de personal).
Preț = cost
+ profit + taxe.
Tarif: de la contabilitate, aceasta înseamnă o proporție de bani per perioadă de timp, de obicei pe an.
Contabili și economiști, de obicei, lasa pe an (de exemplu, rata dobânzii de 5% "înseamnă" 5%/y').
Preț de vânzare cu amănuntul index: o măsură a inflației (sau deflație) facute de periodice costa de un set fix de
comune cheltuieli.
TWIDELL PAGINARE.indb 627
01/12/2014 11:38
628
Instituționale și economice factori
a fost decizia de la Paris pentru a construi Turnul Eiffel. Satisfacția
și plăcerea sunt obținute, în plus față de beneficiul perceput. De multe ori,
o "declarație" este făcut pentru publicul larg de dezvoltare (de exemplu,
având module fotovoltaice pe intrarea acoperiș de un prestigios
birou ca un semn de autonomie și durabilitate). Astfel de non-analitice
de decizie poate fi influențată de aviz etic ca '
corect'.
2
Non-dimensional matrice de analiză. Decide pe n criterii sau valori (de exemplu,
prețul, zgomot, estetica, durata de viață, etc.), fiecare cu greutatea w
j
(să zicem, de la 1 la 10).
Apoi evalua fiecare posibilitate de acordare a unui semn m
j
în fiecare
criteriu. Scorul total pentru fiecare variantă posibilă este:

=
=
S
wm
j
j
j
n
1
(17.1)
Apoi accepta alegerea cu cel mai mare scor, sau reevalueze
ponderi și criteriile pentru o perioadă suplimentară de scor. Astfel de non-dimensional metode
sunt utile în cazul în care criteriile nu au toate aceeași unitate de cont (de exemplu
fericire sau bani).
3
Capital timpul de recuperare T
p
. Primul pas este de a decide reale (interne-
zată) bani beneficia pe an, B
am

, obținute sau salvate (redus) de un proiect


de costul de capital C (de exemplu, un incalzitor solar de apa substituie (atenuare)
a achiziționat energie electrică). Apoi:
T
CB
/
p
am
=
(17.2)
Acest lucru oferă un prim criteriu, de obicei, conduce la noi
discuție sau analiză. De obicei, timpul de amortizare se calculează prin compararea
cu o alternativă, inclusiv a continua cu practica actuală. La
"economii" și "costuri" necesare pentru a calcula amortizare în (17.2) reprezintă
diferența între cele două căi posibile (ca în Exemplul 17.1).
De afaceri pot aștepta T
p
de doi ani, întrucât o persoană privată
pot accepta 10 ani.
4
Simplu randamentul investițiilor (simplu rata de rentabilitate) R
s
. Exprimat ca
procent pe an, acest lucru este inversul timpul de recuperare:
=
R
T
1/
s
p
(17.3)
De exemplu: T
p
de 10y dă R
s

de 10% /an. Cu toate acestea, există mai


semnificative și "profesionist" metode de calcul a rentabilității financiare,
astfel încât utilizarea de R
s
poate fi înșelătoare.
5
Externe de amortizare și de a beneficia criterii. Pot exista și alte beneficii sau
non-beneficii (de exemplu, reducerea poluării) decât internalizate bani
câștigul sau pierderea. Este absolut rezonabil pentru dezvoltator pentru a include astfel de
non-monetare factori în alegerea, probabil, pentru valorile etice singur sau să
proclame un bun exemplu. Dacă factorii pot fi cuantificate monetar
TWIDELL PAGINARE.indb 628
01/12/2014 11:38
§17.6 Cuantificarea alegere
629
unități, chiar și aproximativ, atunci beneficiile externe poate fi
internalizat ca B
e
și a adăugat reale să câștige bani și de economii. Amortizare
timpul devine apoi T
p
= C/(B
am
+ B
e
). De exemplu, pentru un sistem de tratare a
pentru creșterea porcinelor a deșeurilor pentru generarea de biogaz pentru energie, B
e

ar fi beneficiile
de evitare a poluării cauzate de deșeurile netratate, poate măsurată prin
bine, care ar fi fost aplicate la poluator. (Într-un sens, o astfel de
amendă internalizează costul poluării.)
EXEMPLUL 17.1 RĂZBUNARE DE UN INCALZITOR SOLAR DE APA
În Perth (Australia), într-un oraș însorit, la latitudinea de 32°S, o gospodărie tipic foloseste aproximativ 160 de litri/zi fierbinte
(potabilă) cu apă. Un sistem integrat de acoperiș solar de încălzire a apei cu un colector zona de 4.0 m
2

și depozitare de 320
litri furnizează această sumă la 60°C pe tot parcursul anului cu aproximativ 30% suplimentar electric de încălzire.
Prin urmare 'fracție solară este de 70% (adică 70% din energia de intrare pentru apă caldă vine de la Soare).
(Reglementările în Australia nevoie de tot atât de fierbinte ca apa să fie încălzită la >55°C pentru a proteja împotriva legionarilor
boala.) Astfel de incalzitor solar (inclusiv electrice "impuls" de încălzire) costurile legate de O$5100 instalat, mai puțin un
guvern de carbon de reducere a emisiilor de grant de 1000 de dolari, adică O 4100 dolari net, întrucât în întregime electric de
încălzire și
rezervor pentru a produce aceeași cantitate de apă caldă costă 1300 de dolari instalat. Presupunând că nici o schimbare în orice
prețuri
cu timpul, dacă costurile de energie electrică gospodar 25c(A) pe kWh, iar apa trebuie să fie încălzit de la 10°C,
ceea ce este timpul de recuperare pentru a instala un încălzitor de apă solar? (Ratele de conversie la momentul de exemplu
au fost 1.0 O$ ~0.9 NE$~0.6 Euro.)
Soluție
La căldură 160 de litri de apă prin 50
°
C (de exemplu, 10
°
C la 60
°
C) necesită un aport de energie de:
160kg
× 4.2 kJ/kgK × 50K = 33600 kJ
= De 33,6 MJ
× (1 kWh/ 3.6 MJ) = 9.33 kWh.
Dacă toată această energie este furnizată de energie electrică, la un randament de 0,8 la prețul de$0.25/kWh,
asta
costă anual:
9.33 kWh/ zi
× 365 zile/ an × (1/0.8) × O$0.25/kWh = O$1064/y.
Cu o fracție solară de 0,7, costul energiei electrice utilizate în solar pe bază de sistem (1.0 – 0.7) =
30% din aceasta, respectiv O$319/y.
Prin urmare,
=
timpul de recuperare =
(costul de capital diferență, solar - convenționale)
(economii anuale, solar - convenționale)
=
(5100-1000-1300) O$
(1064-319) O$/y
3.8 y
TWIDELL PAGINARE.indb 629
01/12/2014 11:38
630
Instituționale și economice factori
§17.6.2 fluxului de numerar Actualizat (DCF) tehnici: net prezent
valoarea
Cuvântul "reducere" în contabilitate a fost inițial folosit în cea de-a 17
- lea a vrut să dea sau să primească cadou în valoare de o tranzacție
înainte de aceasta se datorează'. Astfel, prin plata mai devreme, mai puțin bani a fost plătită pentru
o "reducere" a fost permis. Cantitatea de reducerea a fost negociat
între părți, fiecare cu diferite motivații. Corolarul este
că păstrarea proprietății de bani permite să fie crescut (de exemplu, prin
interesul pe bani de la banca de economii). Astfel, banii de valoare acum V
0
este
tratate ca având valoarea viitoare:
V
1
= V
0
(1
+ d) după 1 an
(17.4)
unde d este rata de actualizare. Privit invers, primind
bani V
1
la un an după ziua de azi are aceeași valoare ca a primit astăzi
valoarea actualizată:
V
0
= V
1
/ (1
+ d) = V
1
(1
+ d)
-1
(17.5)
Acest concept referitoare la valoarea actualizată a viitoarelor tranzacții oferă
un puternic contabilitate instrument pentru analiza proiectului. Daca tranzacții diferite
la momente diferite pot fi aduse la prezenta valori monetare, acestea
pot fi adăugate ca o sumă pentru 'valoarea actualizată a proiectului.
Derivarea 17.1 și a Lucrat Exemple 17.2 și 17.3 arată modul în care această
idee poate fi extinsă din punct de vedere matematic pentru a calcula valoarea prezentă a
unei sume de bani de la n ani în viitor, și, astfel, pentru a calcula
valoarea actualizată netă a unui proiect propus și beneficiile sale financiare (sau
non-beneficii), comparativ cu o propunere alternativă (de exemplu, pentru a menține
status quo-ul).
DERIVAREA 17.1 UNELE FORMULE PENTRU FLUXUL DE NUMERAR ACTUALIZAT
Continuând metoda de (17.4) în viitor, după n ani valoarea viitoare a unei sume de V
0
ar fi:
V
n
= V
0
(1
+ d)
n
(17.6)
dacă rata de actualizare d este considerată constantă. Pozitiv d (cazul obișnuit) se referă la creșterea (umflate)
valoarea în viitor.
Astfel, pentru o tranzacție de n ani in viitor, sa prezinte valoare este:
V
0
= V
n
/ (1
+ d)
n
= V
n
(1
+ d)
−n
(17.7)
Factorul (1
+d)
−n
se numește factor de actualizare.
Fiecare tranzacție implicat într-un proiect (de exemplu, cumpărarea de echipamente de înlocuire în anul 3, primirea de plată
pentru x
unități de producție în anul 5) pot fi tratate în mod independent, în acest fel, și, astfel, total prezent
valoarea tuturor tranzacțiilor într-un proiect pot fi calculate până la un an s - n. Rețineți că, prezente și viitoare,
tranzacțiile pot fi pozitive sau negative, adică a veniturilor sau cheltuielilor; a se vedea Exemple.
Acordul trebuie să fie atins pentru o valoare a ratei de actualizare d (de exemplu, guvernele pot specifica fel de mult
ca 8% sau cât de puțin de 0,5% pentru propriile finanțe). Pentru 8% rata de actualizare, o tranzacție evaluată la $1000
TWIDELL PAGINARE.indb 630
01/12/2014 11:38
§17.6 Cuantificarea alegere
631
Rețineți că mai mult timp înainte de tranzacție, cu atât mai puțin este
valoarea actualizată dacă d > 0. Deci, analiza reflectă noastră practică preocuparea pentru
prezent și viitorul apropiat, mai degrabă decât de viitorul îndepărtat. De asemenea,
mai mică rata de actualizare, cel mai important este viitorul. Astfel de
implicații de contabilitate metode au o semnificație importantă pentru sustenabilitatea
și inginerie de calitate.
Ambele venituri (să zicem, pozitiv) și cheltuieli (de exemplu, negativ) poate avea
valoare prezentă, deci, dacă o întreagă complexitate de prezent și viitor,
cheltuielile și veniturile sunt introduse într-o foaie de calcul, total, din toate prezente
valori (valoarea actualizată netă, VAN) poate fi calculată, ca și în Mers
Exemple; specialist software-ul de calculator este, de asemenea, disponibile pentru acest scop.
Dacă valoarea curentă netă totală de beneficii minus costuri este pozitiv, atunci
acest lucru este luat ca un semn de succes.
Cu toate acestea, întregul calcul este sensibil la oarecum
arbitrar valoarea dată de rata de actualizare d. Prin urmare, aceste tehnici sunt
de mai mare valoare în a face comparații între proiecte alternative (unul
dintre care poate fi bine status quo-ul), în cazul în care acestea oferă avantajul de
a face ipoteze folosite în alternative explicite și comparabile.
în trei ani de zile (să zicem, o sarcină de întreținere) are V
0
= $1000/(1.08)
3

= $794. Rețineți că nu este nevoie de


nici o strictă relație între rata de actualizare și rata dobânzii bancare. De exemplu, plata
anticipată poate fi atractivă din cauza inflației ridicate rate sau pentru că banii au fost furați, nici factor
neapărat se referă la rata dobânzii de o anumită bancă.
Este posibil să se includă naționale ratele inflației în calcul a valorilor actuale, și deci include
efectivă (reală) sumele tranzacționate. Cu toate acestea, deoarece inflația viitoare nu este cunoscută, o alternativă este de a
enumera
toate tranzacțiile în echivalent pentru un anumit an (de exemplu, în dolari SUA (în anul 2000)).
Cu inflația, un viitor tranzacții în anul n de valoare monetară S
n
va cumpăra mai puțin de o cantitate mult
acum; sa actuală a puterii de cumpărare, S'
n
prin urmare , este redus. Dacă rata inflației, eu, a fost constantă,
apoi:
'=
+
S
S
am
/ (1 )
n
n
n
(17.8)
Actualizarea această sumă, valoarea actualizată a umflat tranzacției devine:
V
S
d
S
d
am
(1
)
[(1
) (1 ) ]
n
n
n
n
n
0

=
'
+
=
+
⋅+
(17.9)
Dacă atât rata de actualizare și rata inflației sunt <10%/ y, atunci o aproximare satisfăcătoare este de:

++
=
+
V
S
di
S
p
(1
)
(1
)
n
n
n
n
0
(17.10)
în cazul în care suma de actualizare și rata inflației, p = d
+eu, este 'rată de piață a dobânzii'. Rețineți că investitorii
din economiile vor aștepta economiile lor pentru a câștiga ratele dobânzilor de cel puțin p, și că aceste bănci la rândul lor, pot
împrumuta bani de mai sus, un astfel de ritm. Astfel de așteptări mixte explica în continuare cum reducere și ratele dobânzilor
pot fi diferite.
TWIDELL PAGINARE.indb 631
01/12/2014 11:38
632
Instituționale și economice factori
A LUCRAT EXEMPLU 17.2 INTERN, INCALZITOR SOLAR DE APA
Pentru încălzitorul de apă în Exemplul 17.1, utilizați analiza fluxului de numerar actualizat pentru a compara net
valoarea actuală a sistemului solar de incalzire a apei de către rețeaua de energie electrică din anul de instalare
la 15 ani, la o rată de scont de 5% și nu inflația. Care este timpul de recuperare? Echipamentul de viață
ar trebui să fie de cel puțin 20 de ani, deci este solare de încălzire a apei o investiție bună?
Soluție
Tabelul 17.3 stabilește calcule, folosind datele din Exemplul 17.1. Notă: 'PV' aici înseamnă
"Valoarea Actuală"
Tabelul 17.3
Valoarea actuală de incalzitor solar de apa si electrice convenționale de încălzire de la 1 an
începând. Pentru rata de actualizare d = 5%, prin urmare, factorul de actualizare în anul n este (1
+ 0.05)
−n
. (Toate costurile într-Un$ la fel ca în
Exemplul 17.1; SWH = incalzitor solar de apa, CEWH = convenționale boiler electric.)
An (n)
SWH
CEWH
Diferența
(D)
Discount
factor (F)
PV = (D)
x(F)
VAN a
(D)
=∑
n
(PV)
Costul de instalat
(cu grant)
0
4100
1300
2800
1.000
2800
2800
Costul anual al
energiei electrice
1
319
1065
-745
0.952
-710
2090
2
319
1065
-745
0.907
-676
1414
3
319
1065
-745
0.864
-644
771
4
319
1065
-745
0.823
-613
158
5
319
1065
-745
0.784
-584
-426
6
319
1065
-745
0.746
-556
-982
7
319
1065
-745
0.711
-530
-1512
8
319
1065
-745
0.677
-504
-2016
9
319
1065
-745
0.645
-480
-2497
10
319
1065
-745
0.614
-457
-2954
11
319
1065
-745
0.585
-436
-3390
12
319
1065
-745
0.557
-415
-3805
13
319
1065
-745
0.530
-395
-4200
14
319
1065
-745
0.505
-376
-4577
Note:
am
În calcul simplu de amortizare (ca în Exemplul 17.1), eficient presupune rata de actualizare d = 0, deci
factor de actualizare (1
+d)
–n

este de 1.00 pentru toți n.


iii
În acest caz factorul de discount este același pentru ambele alternative, astfel încât acesta a fost aplicat la diferența (D) pentru a
calcula valoarea actualizată netă. Care este, valoarea actualizată netă a diferenței dintre alternative este egală cu diferența dintre Ains.
iii
Pentru n <5, VAN solare de încălzire este mai mare decât a celui electric; pentru n >5, VAN solare de încălzire este mai mică
decât cea alternativă. Asta este, sistemul solar costă mai puțin decât cea alternativă, după cinci ani, deci amortizare
timp la o rată de scont de 5% este de cinci ani, în conformitate cu această analiză.
iv
În practică, pentru stimularea sistemului solar s-ar folosi, probabil, mai ieftin 'off-peak' energie decât convenționale
de sistem, care ar îmbunătăți timpul de recuperare împotriva unui preț întreg sistemul electric, deși o comparație echitabilă
ar putea fi cu un off-peak non-sistem solar.
v
Pentru o mai mare rată de actualizare, perioada de amortizare a capitalului
−intensive alternativă este mai lung (a se vedea Problema 17.1).
vi
Ați putea dori pentru a reface calculele fără subvenție guvernamentală, pentru a vedea cum acordarea în mod semnificativ reduce
amortizare ori.
TWIDELL PAGINARE.indb 632
01/12/2014 11:38
§17.6 Cuantificarea alegere
633
A LUCRAT EXEMPLU 17.3 COST NIVELAT DE O FERMĂ EOLIANĂ
Un parc eolian este situat pe o câmpie deschisă în New South Wales (Australia). Acesta cuprinde 10 turbine, fiecare
evaluat la 3.0 MW și cu o tăietură în viteză de 4 m/s. Viteza medie a vântului la 10 m înălțime este
u
= 6.0 m/s;
pe site-ul factor de capacitate de turbine este de 0.22. Pentru fiecare turbină, ex-fabrică costul este de$5,4 milioane de euro
, iar costul de instalare (inclusiv civilă și inginerie electrică) este NE$1.1 milioane de euro. Funcționare și
costurile de întreținere sunt constante la NOI 150.000 de dolari/an pe turbina. (Aceste costuri exclude costul terenului.
Utilizați analiza fluxului de numerar actualizat pentru următoarele calcule.
o
Se calculează media ('egalizate') costul de producție de energie electrică de la site-ul pentru o rată de actualizare de 5%
și un presupus viitor de viață a sistemului de 20 de ani.
b
Sub regimul de comercializare a emisiilor de acord la instalarea, generatorul primeste credit pentru carbon
dioxid salvat de la US$30/tCO
2.
Ceea ce este acum la cost efectiv de producție?
c
Fermierii pe al cărui teren de pășunat eolian este construit continua să pască vacile lor acolo.
Trei muncitori de întreținere sunt angajate la ferma de bovine, și încă 100 de vizitatori pe an la
raion pentru a putea vizualiza și instalare. Discuta despre "cost" (de fapt, beneficiul) pentru regiunea locală.
d
Un sistem similar este instalat la un alt site unde vântul este mai puternic (similar cu Orkney, Scoția,
vezi Fig. 7.7). Factorul de capacitate este de 0,40. Ceea ce este costul de energie electrică generată sub similare
ipoteze financiare?
Soluție
o
Un factor de capacitate de 0,22 înseamnă că fiecare turbină produce o fracțiune 0.22 a ceea ce s-ar produce dacă
fugi la evaluare completă pentru un an întreg. Astfel, energia electrică produsă pe an pe turbina
= 3.0 MW
× 8760 h/y x 0.22 = 5780 MWh/y
Dacă această energie electrică este vândută la q $/kWh, atunci flux de costuri și beneficii din sistem va fi la fel de
indicate în Tabelul 17.4 cu gata cost-beneficiu a
(q$/kWh
× 5780 MWh/y) × (10
3
kWh/MWh)
× (1 M$/10
6
$)
Prin urmare, nivelat prețul la care beneficiile totale pentru parcul eolian de proprietari se va potrivi lor total
costurile în termeni de valoare actualizată după 20 de ani, dacă
q = 8.313/69.87 = $0.119/kWh
Rețineți că prețul pentru consumatorii ar trebui să fie mai mare decât aceasta, pentru a permite unele de profit de la fermă eoliană
de
proprietari.
b
Pentru energia electrică produsă din cărbune, Problema 17.4 arată că fiecare kWh produs presupune emisii de
aproximativ 1,0 kg CO
2
. Prin urmare, anuale credite de carbon de la fiecare turbină în acest sistem este:
5800 MWh
× (1.0 tCO
2
/MWh)
× (30 de dolari/tCO
2
) = $173,000
Acest lucru poate fi luat în considerare în Tabelul 17.4 prin scăderea acestei sume de la costul anual de funcționare
(de exemplu, prin înlocuirea 0.15 prin (0.15
− 0.173 = -0.023). Făcând acest lucru, PV de costurile modificări la $8.113 m
a $5.932 m, iar costul per unitate devine:
q' = 6.22/69.87 = $0.089 $/kWh.
c
Din activitatea agricolă normală poate continua sub turbine, taxa de închiriere pentru facilitatea este
în întregime un câștig pentru arendaș. În consecință, costurile de închiriere terenuri sunt în mod substanțial mai puțin decât
funcționare
și costurile de întreținere, care sunt principalele costul anual de funcționare. Dacă personalul de întreținere a trăi la nivel local,
salariile reprezintă un beneficiu pentru comunitatea locală din (extra) fluxul de numerar.
TWIDELL PAGINARE.indb 633
01/12/2014 11:38
634
Instituționale și economice factori
Tabelul 17.4
Costuri și beneficii fluxuri de la o fermă eoliană (milioane de dolari SUA per 3.0 MW turbină).
Beneficiile sunt date în ceea ce privește prețul unitar de energie electrică vândute de către eolian q(dolari SUA/kWh) (care
definește q în beneficiile coloane). 'PV' este "Valoarea Actuală".
COSTURI/(M$)
BENEFICII/(M$)
An
Capital
Anual
PV
Discount
factor
Bani
PV
Utilaje fosta fabrica de
Site-ul de inginerie
în Curs de desfășurare
5.4
1.1
0
6.5
6.500
1.000
1
0.15
0.143
0.952
5.7816 q 5.506 q
2
0.15
0.136
0.907
5.7816 q 5.244 q
3
0.15
0.130
0.864
5.7816 q 4.994 q
4
0.15
0.123
0.823
5.7816 q 4.757 q
5
0.15
0.118
0.784
5.7816 q 4.530 q
6
0.15
0.112
0.746
5.7816 q 4.314 q
7
0.15
0.107
0.711
5.7816 q 4.109 q
8
0.15
0.102
0.677
5.7816 q 3.913 q
9
0.15
0.097
0.645
5.7816 q 3.727 q
10
0.15
0.092
0.614
5.7816 q 3.549 q
11
0.15
0.088
0.585
5.7816 q 3.380 q
12
0.15
0.084
0.557
5.7816 q 3.219 q
13
0.15
0.080
0.530
5.7816 q 3.066 q
14
0.15
0.076
0.505
5.7816 q 2.920 q
15
0.15
0.072
0.481
5.7816 q 2.781 q
16
0.15
0.069
0.458
5.7816 q 2.649 q
17
0.15
0.065
0.436
5.7816 q 2.522 q
18
0.15
0.062
0.416
5.7816 q 2.402 q
19
0.15
0.059
0.396
5.7816 q 2.288 q
Σ (valoare actuală)
8.313
10.899
69.87 q
d
Cu toți ceilalți factori financiari neschimbată, costul unitar este invers
proporțională cu totalul kWh generat, adică factorul de capacitate. Prin urmare,
costul unitar la vânt, site-ul este 0.065 c/kWh. (Rețineți că factorul de capacitate este
nu direct proporțională cu viteza medie a vântului; vezi §8.7.)
A lucrat Exemplu 17.3 ilustrează puncte în §17.3 despre costurile
evaluate în funcție de care se evaluează, ce costuri sunt incluse,
și în cazul în care și atunci când evaluarea se face. În special:
1
Pentru o utilizare intensivă a capitalului astfel de proiect, costul nivelat depinde
puternic pe asumate de viață a sistemului, deoarece cu o viață mai scurtă
, există mai puține unități de energie produsă pe care să o "medie" de
costul inițial (a se vedea Problema 17.2).
2
De internalizare beneficiile externe poate face o foarte semnificativă
diferența la cost de producție.
TWIDELL PAGINARE.indb 634
01/12/2014 11:38
§17.8 calea De urmat
635
3
Un tabel ca Tabelul 17.3 este ușor de adaptat la situația în care
costurile anuale variază semnificativ de la an la an (de exemplu, dacă majore
componentele sunt înlocuite la fiecare cinci ani).
§17.7 STADIUL ACTUAL AL SURSELOR REGENERABILE DE ENERGIE
Surse regenerabile de energie în anul 2012 a reprezentat pentru aproximativ 13% din global de
aprovizionare cu energie primară, dar, de departe, cea mai mare parte din acest lucru a fost tradițional
de utilizare a biomasei (de exemplu, în principal lemn de foc) în țările în curs de dezvoltare (10% din globală
TPE), urmată de hidrocentrale (2.3%). (A se vedea datele în Parte D2 din Anexa D.)
Consumul Global de energie continuă să crească, determinată de creșterea populației
și dezvoltarea economică, în special în mari țări în curs de dezvoltare din
China, India și Brazilia, asociate cu creșterea industrializării. Global,
consumul de energie a crescut cu o rată medie de 2,5% p.o. (compus) între 2000
și 2010. Cu toate acestea, creșterea utilizării energiei din surse regenerabile a fost mult
mai rapid, mai ales că din nou a energiei din surse regenerabile, deși de la un nivel de bază:
generarea de energie electrică din vânt, PV și geotermale de energie electrică (combinate)
a crescut de la 12% p.un., și biocombustibili lichizi a crescut de la 20% pa (a se vedea Anexa D).
O încurajarea indicator este total nou de investiții în surse regenerabile de energie
(inclusiv energia hidroelectrică) l-a depășit pe combustibili fosili generație în fiecare
an, începând din 2008, iar excesul este în creștere; de asemenea, surplusul
de energie nucleară de investiții. În aceeași perioadă, proporția de
investiții globale în energie regenerabilă sisteme în țările în curs de dezvoltare
(unde este nevoie, fără îndoială, mai mare) a crescut de la aproximativ 30% în 2007
la aproximativ 45% din 2007; proporția de mici (<1 MW) distribuite
de capacitate (în principal PV) are, de asemenea, a crescut de la 10% la aproximativ 30%
(UNEP 2013).
Starea actuală de maturitate tehnologică a diferitelor tehnologii
a fost examinat în capitolele anterioare ale acestei cărți. Câteva RE
tehnologii (de exemplu, hidroenergie, energie geotermală și hidrotermală) au fost
concurente favorabil cu combustibili fosili sisteme de zeci de ani. Până în 2013, vânt
de putere și concentrare a energiei solare a devenit viabil comercial
fără subvenții în locații favorabile, și cu atât mai mult în cazul în care
plățile preferențiale sunt realizate, în vigoare pentru internalizarea costurilor externe ale
redus de combustibili fosili. O gamă mult mai largă de RE tehnologii sunt disponibile
comercial, an de an. (A Se Vedea Fig. D. 5 din Anexa D, care compară
o serie de costuri pentru regenerabile și surse fosile.) Cele mai multe, totuși, nevoie de
politici de sprijin, care crește scara lor de implementare, astfel încât
să beneficieze de economii de scară și deci reducerea costurilor. Scopul
pretutindeni este durabilă a sistemelor de energie (a se vedea §17.5 și §17.8).
§17.8 CALEA DE URMAT
Istoria modernă a sistemelor de energie regenerabile, prezentate sintetic în Tabelul
17.5, arată evoluția lor în stare. În afară de hidroenergie, 40 de ani
TWIDELL PAGINARE.indb 635
01/12/2014 11:38
636
Instituționale și economice factori
în urmă cele mai multe dintre aceste tehnologii au fost considerate 'la scară mică
curiozități promovate de către idealiști, dar astăzi moderne sursele regenerabile de energie au devenit
mainstream tehnologii, produse și operate de companii
concurente într-o piață tot mai deschis.
În general, global energy scena de la începutul secolului 21 este una dintre
vești bune și vești proaste. Vestea proastă este: (a) aproximativ 80% din
consumul de energie din combustibili fosili, prin urmare, forțând schimbărilor climatice și
crearea unei dependențe că, pe termen mediu este nesustenabilă; (b) 10% este
bazat pe sperie de foc, de multe ori folosite ineficient și grăbindu -
defrișări; (c) energie nucleară de fisiune a fost de acord nici metoda de a dispune de
nivel înalt a deșeurilor radioactive și este scump atunci când sunt utilizate și este interzis
în mai multe țări; (d) fuziunea nucleară ca sursă de energie rămâne doar
o cercetare de aspirație. Vestea bună este că energia din surse regenerabile și
utilizarea eficientă a energiei sunt din ce în ce mai acceptat ca din punct de vedere tehnic capabil
de a oferi populația globală durabilă, cu căldură, energie electrică și
combustibil în echitabile și satisfăcătoare de viață.
Implicit în Tabelul 17.5 este modul în care costurile de energie din
cele mai promițătoare tehnologii de energie regenerabilă au în mod constant
scăzut. Fig. 17.2 ilustrează acest lucru pentru cazurile de energie eoliană și
fotovoltaică. Astfel scade costul pe unitatea de output sunt comune ca
noile tehnologii sunt dezvoltate de la stadiul de cercetare, prototipuri
pentru a largi utilizarea în scopuri comerciale într-un mediu competitiv. Curbele de genul celor
prezentate în Fig. 17.2 sunt curbe de învățare pentru că ele reflectă cât
de sarciniers învăța prin experiență modul de a face tehnologia mai fiabile, mai
eficient, și utilizatorii să învețe cum să integreze noua tehnologie în
practici; în acest caz, în rețelele de electricitate.
În cazul sistemelor de energie regenerabilă, de mult de acest tehnologice
de învățare provine de la R&D finanțate în anii 1970 și 1980, care a condus la
evoluțiile tehnice descrise în capitolele anterioare, multe dintre ele
implicând materiale noi și microelectronice de control (a se vedea Tabelul 17.5).
Acestea au contribuit la "push" pentru o astfel de tehnologie modernă. La
realizarea că substanțială a cererii de sisteme de energie regenerabilă
produce un mediu mai curat de reducere a emisiilor de combustibili fosili și nucleari
combustibili a oferit o potrivire "pull". Măsuri politice și economice
pentru a încuraja largă de o tehnologie poate avea un feedback pozitiv:
ca mai sunt utilizate, prețul coboară. Acest lucru se datorează economiilor de
scara'; de exemplu, costul de proiectare și scule pentru un nou wind
turbine este de mult la fel dacă 1 sau 100 de turbine sunt
produse, dar dacă mai sunt vândute, costul poate fi răspândit mai subțire, astfel încât
prețul pe unitate este mai mică, astfel încât mai mulți utilizatori găsi că este "economic". Prin urmare,
încă mai multe sisteme sunt aduse în producție, la rândul său de conducere continuare
tehnice și economice de îmbunătățire într-un ciclu virtuos.
În mod constant îmbunătățirea tehnologiilor descrise în această carte sunt necesare
urgent pentru a aduce surse regenerabile de energie în poziție dominantă necesare pentru a face
sistemele globale de energie durabilă. De multe ori le-a luat aproape 25 de ani
TWIDELL PAGINARE.indb 636
01/12/2014 11:38
§17.8 calea De urmat
637
după ce inițial de comercializare al unui primar formă de energie pentru a obține
1% din cota de piață globală; acest interval este, desigur, chiar mai mult de la
original dezvoltare științifică. Încă de succes tehnologii extinde
exponențial, astfel încât într-un nou deceniu tehnologii pot deveni ferm
stabilit, mai ales în anumite economii.
În practică, există o bandă de costuri pentru o tehnologie, cum este indicat
în Smochine D. 5 D. 6 din Apendicele D. Acest lucru reflectă în principal site-ul
Tabelul 17.5
Evoluția tehnologică, mediul economic și politic pentru " noi " din surse regenerabile
sisteme de energie (SRE), din 1970 la 2030
Perioada
Mediul tehnologic
Mediul Economic
Socială/politică a mediului
1960-1973

Tradiționale și elementare
tehnologii

RES promovat ca
'intermediar
tehnologie', mai ales
pentru țările în curs de dezvoltare

RES aproape niciodată
cost-eficiente

Sustinatorii vazut ca
'hippie', de multe ori trăiesc
în mici, idealist
comunități
1973–
c. 1987

Fondurile publice pentru cercetare

Multe 'ciudat' idei

RES începe să includă
materiale compozite și
microelectronică

Dezvoltarea de RES
văzută ca o "asigurare"
împotriva lipsa și/
sau creșterea costurilor de
energie convențională

Ratele ridicate ale dobânzilor
descuraja
capitalintensive proiecte

Spaima determinat de ulei
criză (creșterea de preț) din
1973 (OPEC)

O mare îngrijorare în sărăcie
țări despre costul de
importurile de energie

Prima Ministerele de Energie
stabilit
c. 1987–
c. 1999

Dezvoltarea consolidează
în jurul cel mai mult
(din punct de vedere economic)
tehnologiile promițătoare

La scară comercială
proiectele încep cu
asistență de subvenții și
alte stimulente

Externalitățile
considerat

Vorbim prea mult de durabile
dezvoltare următoarele
Bruntland (1987) și Rio
Earth Summit (1992)

Energia nucleară cade din
serviciu

Mulți dintre foștii hippies
acum, managerii
c. 2000 –
2030

RES parte din "mainstream"
tehnologia

Eficiența energetică
îmbunătăți

Cele mai R&D pe RES acum
de industria în sine

Multe RES încorporat
în rețelele

Deschide piețele pentru
energie

Mai ieftin de capital

"Poluatorul plătește"
(costurile de mediu ale
fosili sisteme din ce în ce
internalizate)

De Carbon de reducere a emisiilor de tranzacționare

Sustenabilitatea un ghidare
principiul în practică, nu
doar în teorie

Diversitatea aprovizionării cu energie
văzut ca important

Politicile privind schimbările climatice
de acord
c. 2030–

Eficient și distribuite
RES încorporat ca
mare parte din national de
sisteme de energie

Externalitățile pe deplin
internalizat

Creșterea PIB-ului nu mai
văzut ca un centru de
bunăstare

Schimbările climatice și legate
tratatele au semnificativă
efect
TWIDELL PAGINARE.indb 637
01/12/2014 11:38
638
Instituționale și economice factori
dependența de sisteme de energie regenerabilă; de exemplu, energia eoliană este
evident, mai ieftin într-o zonă cu puternic predominante viteze ale vântului. De
asemenea, gama rezultă din variații, în special, tehnologia presupune
(de exemplu turbina tip), și în rata de actualizare presupune. Astfel de costuri
curbe, prin urmare, pot fi folosite numai pentru orientare generală. După cum a fost
subliniat în această carte, evaluarea unui anumit proiect la un
anumit site-ul necesită o evaluare adecvată a resurselor de energie de la
care site-ul și caracteristicile specifice ale sistemului propus.
Fig. 17.3 compară un generic curba costurilor pentru energia din surse regenerabile de aprovizionare
(de exemplu un compozit de curbele din Fig. 17.2) a două generic curbele de cost
pentru energia furnizată de convenționale ('maro') surse. În timp ce costurile
de energie din surse regenerabile reduce de-a lungul timpului, cei de la maro energie poate
fi de așteptat să crească de-a lungul timpului. Pentru combustibilii fosili, acest lucru reflectă
producător preferința de a aduce pe piață primul rând acele resurse care sunt
mai ușor, și, astfel, mai ieftin, extras. În plus, inovarea în
$ 100
1976
1
10
100
Capacitate totală (MW)
Modul PV
prețurile (20
06$/W)
1,000
10,000
1980
1990
2000
2006
$ 10
(a)
$1
$ 10
(b)
$1
10
100
Capacitate totală (MW)
Energia eoliană
prețurile (20
06$/W)
1,000
10,000
100,000
1981
1990
2000
2006
Fig. 17.2
Câteva exemple de curbe de învățare', arată scăderea costurilor de energie din surse regenerabile ca
utilizare crește. Notă scale logaritmice.
o
Module FOTOVOLTAICE 1976-2006;
b
costul de capital de turbine eoliene 1981 până în 2006.
Sursa: G. Nemet (2009) Intermediare monitorizare de dinamica costurilor pentru public sprijinit de energie
technologies, Politica Energetică, 37, 825-835.
TWIDELL PAGINARE.indb 638
01/12/2014 11:38
§17.8 calea De urmat
639
combustibil tehnologii extractive și în combustibil-tehnologii de utilizare continuă, chiar
dacă acestea sunt relativ tehnologii mature, condus de către comerciale
presiune pentru a menține aceste tehnologii cost-competitiv posibil.
Mai mult decât atât, ca o mare literatură pe ulei atestă, prețul real creștere
nu este constantă, datorită concurenței, factori politici, producătorii încearcă să
subcotat pe alții, etc. Pentru energia nucleară, costurile au crescut de-a lungul
timpului, ca pe termen lung costurile asociate cu ciclu complet nuclear
devenit din ce în ce mai evident, inclusiv de securitate, de tratare a deșeurilor și
de eliminare, și de dezafectare.
Punctul în care scăderea energiei "verzi" costul curba
intersectează creșterea 'maro' energie curba prezentată în Fig. 17.3 reprezintă
cruce-peste punctul în care această formă de energie din surse regenerabile devine
din punct de vedere economic a favorizat. Deși nu există valori numerice sunt indicate în această
diagramă schematică, valorile reale prezentate în Fig. 17.2 demonstra
că astfel de cross-over-uri apar la scurt timp după nu este susținută de tranzacționare inițială.
Într-adevăr, pentru hidroenergie în locații adecvate, cruce-peste mult timp a
trecut, cum este acum pentru windpower, fotovoltaică și biomasă în
multe situații.
Există două curbe de maro energie este prezentat în Fig. 17.3: cele mai
scumpe include social (societal) și costurile de mediu, care
în prezent nu sunt incluse în prețurile practicate, adică include
"efectele externe" de Cutie 17.2. Tabelul 17.5 indică faptul că societatea este deja
de a face ceva bani de buzunar pentru aceste externalități, și poate fi de așteptat pentru a
face mai mulți bani în viitor. În acest caz (Fig. 17.3), sursele regenerabile de energie
a devenit atât din punct de vedere ecologic și economic a favorizat opțiune.
La rândul său, acest lucru încurajează lumea furnizorii de energie spre o mai mare
utilizare a RE în loc de combustibili fosili, cum este exemplificat în ER Scenariu
este prezentat în Fig. 17.4.
Pret
timpul (t)
maro + sociale
costurilor
maro
regenerabile
Fig. 17.3
Schematic curbele de cost pentru energia din surse regenerabile, convenționale (maro) de energie (a costat
convențional), maro și energie, inclusiv externe (sociale) costurile.
Sursa: După Hohmeyer (1988).
TWIDELL PAGINARE.indb 639
01/12/2014 11:38
640
Instituționale și economice factori
CUTIE DE 17,5 CONTRASTANTE ENERGIE SCENARII: "BUSINESS AS USUAL" VS. ENERGIE
REVOLUȚIA'
Fig. 17.4(b) arată diferența în impactul pe global CO
2
emisiile între două scenarii, care
reprezintă intervalul de intervenție politică. (Acestea diferă de la IPCC-EOA scenarii prezentate în Fig. 17.1,
acordând mai multă atenție la detalii cu privire la mixul energetic, și nu ia în considerare alte sectoare, cu excepția
utilizatorilor de energie.) AIE-WEO scenariu este eficient referință: arată ce este de așteptat să se întâmple
fără modificări substanțiale în politica guvernului și doar creșteri moderate ale prețurilor la combustibilii fosili.
Odată cu creșterea populației și a activității economice, cererea de energie este în continuă creștere (Fig. 17.4(a)). Deși
valoarea absolută a RE în uz, de asemenea, crește cu 80%, abia modificări ca un procent din energia
de alimentare (Fig. 17.4(c)). În consecință, emisiile globale crește substanțial și schimbărilor climatice devine
mai rău.
ER ('revoluție energetică') scenariu explorează modul de a realiza reducerea emisiilor globale de 3.7 GtCO
2

/
y
, până în 2050. Așa cum se arată în Fig. 17.1, astfel de o reducere dramatică (la mai puțin de 14% din CCG emisiilor în
2007) ar putea fi necesare pentru a menține creșterea viitoare în GMST la <2°C. ER scenariu exploateaza (i)
un mare potențial de eficiență energetică, folosind disponibile în prezent cele mai bune practici de tehnologie (a se vedea
Capitolul 16),
și (ii) tehnologia îmbunătățiri, reducere de preț și RE capacitatea de creștere dincolo de care apar prin
economii de scară. În consecință, în ER scenariu, până în 2050 RE va furniza 77% din energie la nivel mondial
a cererii. Se uită la Fig.17.1(c), în cazul în care RE consumabile și creșterea eficienței de utilizare a energiei împreună pentru a
atenua cele mai grave efecte ale schimbărilor climatice, apoi o creștere de 400% în RE capacitatea de faptul că, în 2007, este
necesar, împreună cu voința politică să facă acest lucru.
(a)
400
600
800
2000
2025
2050
cerere (EJ/y)
(b)
0
25
50
2000
2025
2050
GtCO
2
/y
(c)
0
50
100
2000
2025
2050
AIE-WEO (de afaceri ca de obicei)
ER (cu surse regenerabile de energie și
eficiență energetică)
% RE
Fig. 17.4
Două scenarii ilustrative pentru 2050 pentru dezvoltarea (a) cererii de energie, (b) CO
2

emisiile provenite de la surse de energie, (c) proporția de energie din surse regenerabile (RE) în energie la nivel global
de aprovizionare. AIE-WEO (linie continuă) este în esență o continuare a trecutului modele și politici, în timp ce
'revoluție energetică (ER)' (linie punctată) include eforturi semnificative pentru a exploata atât energie
eficiența și RE.
Sursa: Bazat pe IPCC SRREN (2011), tabelul 10-3.
TWIDELL PAGINARE.indb 640
01/12/2014 11:38
§17.8 calea De urmat
641
Pentru a realiza o profundă schimbare de politică, este necesar de a schimba nu numai
răspunsuri (ca și în economie: a se vedea §17.3), dar, de asemenea, să convingă pe cei de la
taxa pentru a schimba întrebarea. Acest lucru este dificil, deoarece fundamentale
întrebare (de exemplu durabilitatea ecologică) de multe ori contravine formare,
experiență și filosofice poziție de cei care au crescut prin
instituțiile existente de putere.
Putem concluziona că sursele regenerabile de energie sunt în creștere în zonele de dezvoltare, cu
potențialul de a furniza cea mai mare parte din energia mondială de milioane de locale
și site-uri adecvate. Astfel de succes necesită cunoștințe, viziune,
experiență, finanțe, piețe, și opțiunile individuale și colective. Cu toate acestea,
ne avertizează că pentru sistemul energetic național să fie cu adevărat durabilă, nu
numai trebuie să sursele de energie să fie durabilă, dar așa trebuie să energiei sale
de consum. În consecință, eficiența și scopul consumului final de energie
sunt vitale (a se vedea Capitolul 16). Acesta este un punct cheie în revoluția energetică
scenariul descris în Caseta 17.5 și alte similare, cu emisii reduse de scenarii.
Sursele regenerabile de energie sunt suficiente pentru a satisface în creștere la nivel mondial
a cererii de servicii energetice, dar numai dacă este combinată cu utilizarea eficientă
a energiei. Este astfel posibil succesul? Noi credem că este.
REZUMAT CAPITOL
Acest capitol trece în revistă social-politică și economică a factorilor care influențează dezvoltarea de surse regenerabile de
sisteme de energie (RES). Acestea includ politica energetică națională, condițiile economice, prețurile de consum,
costurile externe, schimbările climatice, securitatea energetică, și corporative și mecanismele pieței de energie
aprovizionare. RES dezvoltare depinde de aproximativ 75% pe astfel de factori instituționali și aproximativ 25% în știință
și inginerie.
Metode pentru cuantificarea alegeri între sistemele includ matricea de analiză, 'timpul de recuperare și
fluxurilor de numerar actualizate. Evaluarea economică a "costuri" depinde foarte mult pe ale căror costuri sunt
evaluate (consumatori sau producători) și pe perioade de timp și locație. Furnizare de energie din surse regenerabile
este, în general, site-specific cu dominantă în avans costurile de capital, spre deosebire de livrările de combustibili
fosili. Instrumente de politică
face aprovizionarea durabilă cu energie mai cost-eficient și pe scară largă în ultimii 40 de ani
includ limitele de reglementare a poluării, subvențiile pentru RD&D, scăderea subvențiilor pentru non-durabile sisteme de
subvenții pentru energia din surse regenerabile, de impozitare ajustări, permise negociabile, sursele regenerabile de energie
obligații, feed-in
tarife. Chiar și așa, plin externe și costuri sociale convenționale de combustibili fosili și energia nucleară
furnizează de obicei, sunt încă nu sunt pe deplin incluse în prețul lor, care distorsiunile ales împotriva durabilă
a energiei regenerabile.
Îmbunătățiri tehnologice și economii de scară prin creșterea producției și utilizarea continua
pentru a reduce costurile de RES. Sursele regenerabile de energie sunt acum puternic în creștere în majoritatea țărilor, cu
potențial
de a furniza în mod durabil cea mai mare din lume de energie de multe milioane de site-uri. Pentru a realiza acest potențial
necesită cunoștințe, viziune, experiență, finanțe, piețe și alegere, toate bazate pe bună știință și
tehnologie.
TWIDELL PAGINARE.indb 641
01/12/2014 11:38
642
Instituționale și economice factori
ÎNTREBĂRI RAPIDE
Notă: Răspunsurile la aceste întrebări sunt în textul de la secțiunea relevantă
din prezentul capitol, sau poate fi ușor dedusă din aceasta.
1
Lista și să clarifice cel puțin cinci instituțională a factorilor care afectează creșterea de
energie din surse regenerabile.
2
Ce factori influențează politicile naționale pentru furnizarea de energie?
3
Pentru propria ta țară, numele a cinci factori favorizând absorbția de
energie din surse regenerabile, și de cinci factori care descurajează o astfel de asimilare.
4
În opinia dumneavoastră, cine beneficiază cel mai mult din surse regenerabile de energie, bogați
sau săraci? Discuta.
5
Da exemple de "costurilor externe" în țara dumneavoastră; aveți oricare dintre
aceste s 'interiorizat', și dacă da, cum?
6
Defini, analiza ciclului de viață'.
7
Descrie cinci metode folosite de guverne pentru a încuraja
absorbția de energie din surse regenerabile.
8
Rezuma cele trei metode utilizate pentru a cuantifica alegere pentru luarea deciziilor.
9
Știința economică nu este o știință exactă, și dacă nu, de ce nu?
10
Va crește utilizarea surselor regenerabile de energie, și dacă da, cum?
PROBLEME
17.1
Pentru încălzire solară a apei sistemul prezentat în Exemplul
17.2, se calculează timpul de recuperare împotriva electrice convenționale
sistem:
(a) folosind o rată de actualizare de 10%.
(b) utilizând o rată de actualizare de 5%, dar cu nici o subvenție guvernamentală.
(Indiciu: construi un tabel similar cu Tabelul 17.3.)
17.2
Pentru eolian, se arată în Exemplul 17.3, se calculează
costul nivelat al energiei electrice în următoarele ipoteze:
(a) o rată de actualizare de 10% și un sistem de viață de 20 de ani;
(b) rata de actualizare de 5% și un sistem de viață de șase ani (ca s-ar putea
întâmpla dacă o neprotejate corespunzător sistemul a fost distrus
de o furtună severă);
(c) o rată de actualizare de 10% și un sistem de viață de șase ani.
Comentariu cu privire la efectele relative la costul nivelat de reducere
rata și durata de viață a sistemului.
(Indiciu: construi un tabel similar cu Tabelul 17.4).
17.3
Discuta modul în țara ta stă în ceea ce privește socio-politice
factorii prezentați în §17.2. Identifica orice forțe care acționează pentru a
schimba această poziție.
TWIDELL PAGINARE.indb 642
01/12/2014 11:38
Bibliografie
643
17.4
Estimarea CO
2

emisiile pe unitate de energie electrică produsă de


o conventionale centrale pe bază de cărbune. (Indiciu: Cărbune este de aproximativ 80%
de carbon. Face ipoteze rezonabile cu privire la eficiența de
conversie de la căldură la putere.)
NOTĂ
1
A se vedea Caseta 15.3 și suplimentare web informații cu privire la rețea integrată de energie eoliană.
BIBLIOGRAFIE
Politica si aspecte instituționale
Chamnanphony, R. și Kaul, S. (2000) Resurse Regenerabile pentru Energie Electrica, Cvorumul Cărți, Connecticut, statele
UNITE ale americii.
Non-tehnic cont de tehnologii, subliniind importanța factorilor instituționali și utilizarea finală a eficienței.
Flavin, C. și Lenssen, N. (1995) Supratensiune: Un ghid pentru venirea revoluție energetică, Earthscan/James
& James, Londra. Susține că, spre deosebire de cele mai elementare analize, o revoluție a energiei durabile
sistem bazat pe o mai mare eficiență și din surse regenerabile este posibil și de dorit.
Friedman, T. L. (2008) Cald, Plat și Aglomerat: de Ce lumea are nevoie de o revoluție verde și cum putem reînnoi
viitorul nostru global, Allen Lane, Londra. Plin de viață polemică, plin de citate bune.
GEA (2011) Global de Energie de Evaluare: Către un viitor energetic durabil, Cambridge University Press,
Cambridge. Autoritate și internațional cuprinzător de revizuire a cadrului și resurse pentru astăzi
sistemul energetic și cum au nevoie pentru a schimba pentru a asigura durabilitatea. Măsura
> 2000 pagini (mai multe
capitole,
disponibile online la www.globalenergyassessment.org).
Geller, H. (2003) Revoluție Energetică: Politici pentru un viitor durabil, Island Press, Washington, DC. Pune accentul pe
trei teme: surse de curent și modele de utilizare a energiei sunt nesustenabile; o 'revoluție energetică este posibil
și de dorit prin mult mai mare eficiență energetică și RE; barierele pot fi depășite printr-o
luminat a politicilor publice.
Glasson, J., Therival, R. și Chadwick, A., (2011, 4 edn) Introducere în Evaluarea Impactului asupra Mediului,
Routledge/Taylor & Francis, Abingdon. Abordare structurată pentru standardele profesionale și metode; exemple
includ parcuri eoliene.
Goldemberg, J. și Lucon, J. (2009, 2nd edn) Energie, Mediu și Dezvoltare, Routledge, Londra.
Ample și ușor de citit expunerea de link-uri între energie și dezvoltarea economică și socială și
sustenabilitate, cu luarea în considerare a capitalului propriu în cadrul și între țări, de către un expert Brazilian.
Hunt, S. și Shuttleworth, G. (1996) Concurența și posibilitatea de Alegere în energie Electrică, Wiley, Chichester. Explică cum
naționale de energie electrică de alimentare industrii s-au schimbat de liberalizare și privatizare.
International Solar Energy Society (2004) Trecerea la un Viitor pentru Energia Regenerabilă – O Hârtie Albă. Disponibil
online la www.ises.org. Se concentrează pe tehnologia de comercializare și schimbările de politică necesare.
IPCC GL2 (2007) schimbările Climatice 2007: Contribuția Grupului de Lucru II la cel de-al Patrulea Raport de Evaluare al
IPCC: Rezumat pentru factorii de decizie, grupul Interguvernamental privind schimbările Climatice. Disponibil online la www.
ipcc.ch. IPCC Grup de Lucru 2 se referă la impact, adaptare și vulnerabilitate aspectele legate de schimbările climatice.
IPCC WG1 (2013) privind schimbările Climatice 2013: Contribuția Grupului de Lucru II la cel de-al Cincilea Raport de
Evaluare al
IPCC: Fizică de Bază: Rezumat pentru factorii de decizie, grupul Interguvernamental privind schimbările Climatice,
disponibil online la www.ipcc.ch.
TWIDELL PAGINARE.indb 643
01/12/2014 11:38
644
Instituționale și economice factori
Laird, F. N. (2001) a Energiei Solare,Tehnologie, Politică și Instituțională Valori, Cambridge University Press, Cambridge.
Studiu academic de NOI politici energetice 1946 1979, arată cum factorii instituționali blocat RE de a deveni
o parte importantă a NE sistemului energetic în perioada respectivă.
Mallon, K. (2006) Regenerabilă, Politica Energetică și Politica – Un manual pentru decizia de a facers, Earthscan, Londra.
Structurată în capitole de experți, analiza strategiilor guvernamentale și implicațiile pentru energie regenerabilă,
dezvoltarea și punerea în aplicare la nivel național cântare. Include studii de caz. Urmărește general cuantificate concluzii, dar
fără a analiză matematică.
Mitchell, C. (2008) Economia Politică a Energiei Durabile, Macmillan, Londra. Plasarea marea BRITANIE în
perspectivă comparativă, Mitchell susține pentru un nou mod de abordare a politicii față de energie și sustenabilitate.
Mitchell, C., Sawin, J., Pokharel, G. R., Kammen, D., Wang, Z., Fifita, S., Jaccard, M., Langniss, O., Lucas, H.,
Nadai, A., Trujillo Blanco, R., Usher, E., Verbruggen, A., Wüstenhagen, R. și Yamaguchi, K. (2011) Politică,
finanțarea și punerea în aplicare', în O. Edenhofer, R. Pichs Madruga, Y. Sokona, K. Seyboth, P. Matschoss, S.
Kadner, T. Zwickel, P. Eickemeier, G. Hansen, S. Schlömer și C. von Stechow (eds), IPCC Raport Special privind
Sursele de Energie Regenerabile și Atenuarea efectelor schimbărilor Climatice, ch.11, Cambridge University Press, Cambridge.
Autoritate de revizuire de experiență internațională cu RE politici; adesea menționată ca SRREN.
Scheer, H. (2001) Solar Economie: energie Regenerabilă durabilă globală viitor, Earthscan/James &
James, Londra. Un bine-formulat polemică argumentând că un sistem energetic bazat aproape în totalitate pe sursele regenerabile
de energie este
posibil dar va fi nevoie de "distrugerea creatoare" vechi fosile bazate pe sistemul.
SRREN (2011) Vezi Mitchell et al. (2011).
UNCED (1992) La Summit-ul Pământului. Mai multe rezultate Națiunilor Unite publicații, inclusiv: Agenda 21,
Declarația de la Rio privind Mediul și Dezvoltarea, Declarația de Pădure Principii, organizația Națiunilor Unite a
Convenției-Cadru privind schimbările Climatice și Convenția Națiunilor Unite privind Diversitatea Biologică.
Wilkins, G. (2002) Transferul de Tehnologie pentru Energie Regenerabilă, Earthscan/James & James, Londra. Examinează
practice de a aduce energie din surse regenerabile în utilizarea pe scară largă în țările în curs de dezvoltare, cu referire la Kyoto
Protocolul de mecanisme și studii de caz de biomasă de co-generare și de uz casnic sisteme fotovoltaice.
Yergin, D. (1992) Premiul: epic quest pentru petrol, bani și putere, Simon & Schuster, New York. Divertisment
cont de politica și personalități din lumea petrolului.
Mediu manualele de economie
Comune, M. și Stagl, S. (2005) Ecologice Economie: O introducere, Cambridge University Press, Cambridge.
Expunerea clară la nivel introductiv de relevante principii economice și de instrumente.
Tietenberg, T. și Lewis, L. (2009, 8 edn) Mediu și a Resurselor Naturale Economie,
Pearson/AddisonWesley, Citesc, MA. Un text standard la puțin nivel mai avansat decât Comune și Stagl.
Externalitățile
Comisia europeană (1995) ExternE: Externalități de energie, 7 volume, E. C., la Bruxelles.
Hohmeyer, O. (1988) Costurile Sociale ale Consumului de Energie: efecte Externe de energie electrică în
Republica Federală Germania, Springer Verlag, Berlin.
Oak Ridge National Laboratory și Resurse pentru Viitor (1994) Ciclul de Combustibil Externalități, statele UNITE ale americii.
A se vedea, de asemenea, Fischedick et al. (2011).
TWIDELL PAGINARE.indb 644
01/12/2014 11:38
Bibliografie
645
Instrumente pentru analiza valorii prezente
Awerbuch, S. (1996) problema La valorificarea noilor tehnologii energetice', Politica Energetică, 24, 127-128. Introducerea
de noi tehnici de evaluare a merge dincolo de "tradițional" de utilitate și valoarea actualizată tehnici.
Boyle, G. (ed.) (2004, 2nd edn) surse Regenerabile de Energie, Oxford University Press, Oxford. Anexa privind investiția
în surse regenerabile de energie' este puternic recomandat.
Agenția internațională a Energiei (1991) Orientări pentru Analiza Economică a tehnologiilor de Energie Regenerabilă
Aplicații, IEA, Paris. Foarte de cont detaliate cu exemple, deși ignorarea externalități.
Scenarii pentru viitor
Aitken, D. W,, Billman, L. L. și Taur, S. R. (2004) 'climatul de stabilizare provocare: poate regenerabile de energie
surse îndeplini obiectivul?', Energie Din Surse Regenerabile Lume, Decembrie, 56-69. O revizuire a publicat diverse
scenarii, care concluzionează că RE-ar putea face 50% din aprovizionarea cu energie până în 2050.
Fischedick, M., Schaeffer, R., Adedoyin, A., Akai, M., Bruckner, T., Clarke, L., Krey, V., Savolainen, I., Teske, S.,
Ürge-Vorsatz, D. și Wright, R. (2011) 'a potențialului de Atenuare și costurile în O. Edenhofer et al. (eds), IPCC Speciale
Raport privind Sursele de Energie Regenerabile și Atenuarea efectelor schimbărilor Climatice, ch. 10, Cambridge University
Press,
Cambridge. Clienți o gamă largă de energie scenarii.
IPCC (2007) schimbările Climatice 2007: raport de Sinteză, grupul Interguvernamental privind schimbările
Climatice/Cambridge
University Press, Cambridge. Sintetizează și integrează trei IPCC volumele fizice de știință,
impactul și adaptarea la acestea, și de atenuare. Disponibil online la www.ipcc.ch.
Krey, V. și Clarke, L. (2011) Rolul energiei din surse regenerabile în atenuarea schimbărilor climatice: o sinteza recenta scenarii',
Politica privind schimbările Climatice, 11,1131–1158. O sursă majoră pentru Fischedick et al. (2011).
Schellnhuber, H. J., Cramer, W., Nakicenovic, N., Wigley, T. și Yohe, G. (coord) (2006) Evitarea Periculos
Cimate Schimba, Cambridge University Press, Cambridge.
Smith, J. B., Schneider, S. H. et al. (2009) 'Evaluarea schimbărilor climatice periculoase printr-un update al
grupului Interguvernamental privind schimbările Climatice (IPCC) "motive de îngrijorare"', Proceedings al Academiei Nationale
de Stiinte, 106, 4133-4137.
Teske, S., Pregger, T., Simon, S., Naegler, T., termometru vreme, W. și Lins, C. (2011) Energy [R]evolution 2010 – o
lume durabilă energy outlook', Eficiența Energetică, 4, 409-433. Prezintă un radical low-carbon energy scenariu.
Weir, T. (2012) 'schimbările Climatice și energia din surse regenerabile: implicații pentru Insulele din Pacific a global
perspective', Jurnalul din Pacific Studii, 32(2), 4-30.
Unele studii de caz
Ling, S., Twidell, J. Boardman, B. (2002) 'de uz Casnic piața fotovoltaică din Xining, Provincia Qinghai, China:
rolul autorităților locale PV de afaceri', a Energiei Solare, 73, 227-240.
Lipp, J. (2001) 'Micro-finanțare a energiei solare: Sri Lanka SEMINȚE model', Reorienta, octombrie, p.18-21.
Mandela, M. (2000) de Sprijin pentru energia din surse regenerabile: o perspectivă de Banca de Dezvoltare a Africii de Sud',
Se Reorienteze, August, 15-17.
TWIDELL PAGINARE.indb 645
01/12/2014 11:38
646
Instituționale și economice factori
Reviste și site-uri web
Politica energetică (publicat de Elsevier) este un jurnal academic axat pe teme economice, politice și instituționale
aspecte (de exemplu, impactul politicilor privind schimbările climatice) din toate formele de energie, inclusiv energie din surse
regenerabile.
Se reorienteze revista (publicat de Elsevier pe numele International Solar Energy Society) a numeroase
articole privind electrificarea rurală.
Regenerabile de Energie (Rețea de REN21) (www.ren21.org), o rețea internațională de experți care publica anual
sondaje de RE-utilizarea și politici din întreaga lume (Regenerabile Global Status Report).
UNEP (2013) Tendințele Globale în domeniul Energiei Durabile Investiții 2013, Programul Națiunilor Unite pentru Mediu
(http://www.unep.org/sefi-ren21/). Actualizat anual.
TWIDELL PAGINARE.indb 646
01/12/2014 11:38

Comentariu 1 energie Electrică pentru energia din


surse regenerabile
CONȚINUTUL
Obiective de studiu
647
§R1.1 Introducere
648
§R1.2 transport de energie Electrică: principii 648
§R1.3 rețelele de energie Electrică (rețele)
650
§R1.4 DC grile
651
§R1.5 AC putere activă și reactivă:
transformers
651
§R1.6 mașini Electrice (generatoare
și motoare)
652
§R1.7 provocări și oportunități
pentru sursele regenerabile de energie electrică
656
Întrebări rapide
659
Bibliografie
659
OBIECTIVE DE STUDIU
Cele mai tehnologiile regenerabile sunt utilizate pentru a
genera energie electrică. Din această Revizuire ar trebui să
fie capabil să explice (a) elementele de bază ale diferitelor
producătorii de energie electrică incluse în această carte, (b)
funcționarea rețelelor de distribuire a puterii,
și (c) unele aplicații.
LISTA DE FIGURI
R1.1 transmisiei Electrice.
649
R1.2 generator Sincron: principiu.
653
R1.3 de două ori de inductie generator.
656
R1.4 Energie diagrama flux pentru statele UNITE ale americii în 2011.
658
TWIDELL PAGINARE.indb 647
01/12/2014 11:38

www.shahrsazionline.com
648
De energie electrică pentru energia din surse regenerabile
648
Comentariu 1 energie Electrică pentru energia din surse regenerabile
§R1.1 INTRODUCERE
Această Revizuire prezintă principiile fizice de producere,
distribuție și utilizare a energiei electrice, în special în ceea ce privește energia din surse regenerabile.
Electrice de stocare (baterii, etc.) este acoperit în Capitolul 15.
Energia electrică este termodinamic o calitate înaltă formă de energie, deoarece
aceasta poate fi convertită în lucru mecanic cu pierderi mici. Acesta este considerat
"esențial" (în practică) pentru motoare electrice, de iluminat, de comunicare,
de calcul, de refrigerare și de gătit, dar nu este esențială pentru electrice
rezistive de încălzire a spațiului, pentru care este adesea folosit. Energia generată
din surse diferite, inclusiv cele mai multe surse regenerabile de energie, poate fi integrat
și distribuit de ușor ca și de energie electrică, astfel încât furnizarea de vastă și vital
servicii. Ineficiența nu se referă la energie electrică în sine, dar a (a) generarea
de căldură (de exemplu.g. cărbune și biomasă ardere, reacții nucleare), fără
utilizarea căldurii reziduale, (b) utilizarea ineficientă (de exemplu, lumini incandescente: a se vedea
Caseta 16.2), și (c) pierderile în transport și distribuție.
Generarea de energie electrică și de distribuție a beneficiilor multe surse regenerabile de energie
sisteme. Acest lucru este evident atât pentru energie fotovoltaică, care este electric
în origine. De asemenea, se aplică pentru energia regenerabilă, care este: (a) mecanice în
origine (de exemplu, hidro, val și vânt); (b) un imobil sursă de căldură (de exemplu,
geotermală, mare solare concentrate); (c) un exces de aprovizionare (de exemplu, în caz contrar
pierderile de căldură). Generarea de energie electrică din arderea biomasei este
discutabilă pentru că low-temperatura de ardere duce la o slabă eficiență
(<30%) dacă nu este combinată de căldură și energie, în cazul în care
energia electrică este un produs de alimentare cu căldură (de exemplu, în fabricile de zahăr: Caseta 9.2).
Un beneficiu major pentru regenerabile în aprovizionarea cu timpul variabile de ieșire (de exemplu,
eoliană, a valurilor, a mareelor, energia solară) este că integrarea lor într-o electrică
de distribuție rețea permite tuturor producția lor să fie împărtășite, comercializate și utilizate. În
plus, producția lor în grila de obicei înlocuiește energie electrică în caz contrar
generate din combustibili fosili.
Alimentarea cu energie a utilizatorilor, care este esențial energie electrică este, de obicei, doar
~15% din totalul național de aprovizionare cu energie, cu restul de ~85%
dominat de nevoia de căldură și de transport. Cu toate acestea, din cauza
dominația centralele termice cu eficiență slabă (nucleare
~30%, cărbune de ~35%, gaz ~40%) și, în consecință, pierderea de căldură,
centralizate de energie electrică necesită ~35% din cele mai primare naționale
aprovizionarea cu energie. Real proporții varia în funcție de țară. Cerințe astfel de
informații din statisticile naționale nu este ușor, și este, probabil, cel mai bine
perceput prin intermediul energetic național diagrame de flux (de exemplu.g. Fig 1.3 pentru Austria și
Fig. R1.4 pentru statele UNITE ale americii); a se vedea, de asemenea, Fig. 16.3(b)).
§R1.2 TRANSPORT DE ENERGIE ELECTRICĂ: PRINCIPII
Ia în considerare două alternative de sisteme de transmitere la fel de utile putere
P (= I
1
V
1
= Am
2
V
2
) la o sarcină R
Am
la diferite tensiuni V
1
, V
2
din sârmă de același
TWIDELL PAGINARE.indb 648
01/12/2014 11:38
§R1.2 transport de energie Electrică: principii
649
rezistența R
w
(Fig. R1.1(a)). Corespunzătoare curenților sunt eu
1
= P/ V
1
,
Am
2
= P/ V
2
și firul pierderile sunt eu
1
R
2
w
și eu
2
R
2
w
. Prin urmare, raportul de
puterea a pierdut (P') din sârmă de rezistență R
w
în cele două sisteme este:
P
P
IR
IR
P
V
V
P
V
V
w
w
1
2
1
2
2
2
1
2
2
2
2
2
1
2
'
'
=
=








=
(R1.1)
Astfel, în mod semnificativ mai puțină energie este disipată în firele de conectare
(cabluri) în sistemul funcționează la înaltă tensiune. O distribuție de joasă tensiune
de sistem are aceeași pierdere ca o înaltă tensiune sistemul numai dacă gros, și
, prin urmare, scumpe, cabluri de conectare sunt folosite. Pentru energia electrică
distribuită la interne tensiune de alimentare (~110 V sau ~220 V), costurile de cablare
devine prohibitiv pentru distanțe mai mari de aproximativ 200 m. Cablul
costurile deveni chiar mai mare la foarte joasă tensiune, ~12 V.
Acești factori guvernează proiectarea de energie electrică rețele.
Generatoarele de curent ALTERNATIV sunt fabricate într-o gamă largă de putere
și tensiuni, potrivite pentru o anumită utilizare și amplasare (de exemplu, (a) pentru
centrale, stații electrice ~500 MW la 10 kV 25 kV cu cele mai recente modele de
la 400 kV; (b) generatoare de turbine eoliene ~3 MW la ~600 V). Ușurința cu
care de curent alternativ (AC) pot fi transformate pentru a mari sau mai mici
de tensiune explică de ce AC sistemele de transport au fost standard pentru
toate, dar cele mai mici rețele. După cum este indicat în Fig. R1.1(b), puterea poate
fi generat la o tensiune mai mică, a urcat pentru transmisie la o mai mare
tensiune, și apoi din nou în jos la o altă tensiune pentru consum. Rețineți
că, solid-state componente electronice de putere din ce în ce permite AC/DC,
DC/AC și DC/DC transformare la putere mare și un cost rezonabil.
Transmisie tensiune este constrânsă de străpungere dielectrică a
aerului în jurul valorii de cabluri aeriene, de izolare a cablurilor de
metal, turnuri, care sunt la pământ (zero) potențial și de rezistență
Generator
V
P
Am
2
Am
R
Am
=
R
Am
R
w
R
w
= P'/I
2
V
gen
V
trans
V
contra
(a)
(b)
Sârmă
Sarcina
Fig. R1.1
Transmisie electrică. (un) Puterea de transmisie de la o sarcină de rezistență R
Am

, printr-un fir
de rezistență finită R
w

. (b) Mai multe sanse de realizare, folosind transformatoare generate de tensiune


transformat " sus "în tensiune pentru transmisia la o pierdere mai mică și apoi" jos " pentru consum.
TWIDELL PAGINARE.indb 649
01/12/2014 11:38
650
De energie electrică pentru energia din surse regenerabile
650
Comentariu 1 energie Electrică pentru energia din surse regenerabile
de cabluri. Cele mai transmisia este AC din cauza folosirii convenționale
transformatoare, dar DC sisteme au avantaje pentru eficientizarea
transmisiei pentru că nu există inducție pierderile și constanței de vârf
de tensiune permite o putere maximă de curgere. Aeriene de transport de tensiuni
pentru linii lungi sunt, de obicei, aproximativ 200 kV 500 kV, cu
'ultra-highvoltage' de aproximativ 1000 kV (AC) și 800 kV (DC). Supraconductoare fire
de rezistență zero de operare de la foarte scăzut 'criogenice' temperatura sunt utilizate
în unele echipamente (de exemplu, intens electromagneți), dar nu încă pentru
linii de transmisie din cauza costurilor și a complexității.
§R1.3 REȚELELE DE ENERGIE ELECTRICĂ (REȚELE)
Comunitare și naționale producerea energiei electrice link-uri la sarcina
cererii printr-o rețea comună, adesea numit "grila". Generarea
poate fi de la alimentare centralizată stații sau de capacitate mai mică
încorporat generație, cum ar fi fotovoltaice, micro-generare sau de la turbinele eoliene.
Grila permite schimbul de producere și consum, și astfel
oferă o încredere și mai cost-eficient de aprovizionare.
În ciuda numelui lor și intenția inițială, începând cu 1930
de energie electrică 'grile s-a caracterizat prin centralizat de expediere de energie electrică
la tensiune înaltă (>~100 kV) la un număr mic de interconectate
foarte mare putere stații de ~1000 MW de capacitate. Aproape de punctul de
utilizare finală, de energie electrică este transformată în jos la ~10 kV (de exemplu, pentru o suburbie),
și apoi la ~230 V sau ~110 V (de exemplu, pentru o stradă de case). Mediul rural în vârstă
de grile de la dispeceratul central poate pierde 10 la 20% din energie în
transport, distribuție și transformare, cu cele mai bune urban grile având
aproximativ 5% pierderi. În ciuda dominanței central generație, cele mai multe
grile poate încorpora o răspândire semnificativă din surse descentralizate (de exemplu,
micro-generare de gospodării individuale cu grid-connected' PV
sisteme (§5.3) sau dispersate ferme eoliene (Capitolul 8)). Cu moderne
solidstate electronice de putere de control și de supraveghere de la distanță, grilele sunt
din ce în ce mai capabil să accepte și să distribui dispersat de putere, inclusiv de la
o gamă largă de surse regenerabile de energie generație. Unele dintre caracteristici tehnice
de astfel de rețele sunt prezentate în §15.4.
Energie electrică într-o rețea este predominant instantanee de transport sau vector
de energie, deoarece la data de depozitarea acestora in baterii si celule de combustibil este neglijabil
pe național. Prin urmare, echilibrul pe o rețea de alimentare cu
cererea (și a cererii cu oferta) trebuie să fie controlate. Regenerabile
de energie generatoare (de exemplu, turbine eoliene) au ieșire care se potrivește lor
de mediu schimbătoare de intrare, astfel încât producția lor variază continuu și
poate înceta. De asemenea, cererea ("load") de la consumatori pe o rețea
variază. De comandă a transmisiei operatorii au reușit întotdeauna
acest echilibru minut cu minut, prin mici modificări în linia de tensiune
și frecvență, prin integrarea variabilă a cererii cu variabile de intrare, și
prin utilizarea controlabile generație (de exemplu, hidro si electrice cu turbine pe gaz) și
TWIDELL PAGINARE.indb 650
01/12/2014 11:38
§R1.5 Ac putere activă și reactivă: transformers
651
controlabil sarcini (de exemplu, încălzitoare de apă, metal rafinării). Convențional,
toate marile sisteme de rețea pentru a satisface rapid cererea de fluctuațiile de ~20%,
în principal având în plus capacitatea de producție disponibile; prin urmare,
aceleași metode pentru a satisface rapid fluctuațiile ofertei, de asemenea, de ~20% (de exemplu,
deconectarea bruscă de centrale generatoare de plante). Aceste tehnici
găzdui variabilă, și cea mai mare parte previzibil, sursele regenerabile de aprovizionare.
Atunci când variabila generație component depășește ~20% din totalul ofertei,
apoi strategii speciale pot fi folosite, cum ar fi creșterea de așteptare
generație, control de la distanță de generatoare și de control de sarcinile specificate
(a se vedea §15.4).
Rețineți că o rețea electrică poate fi mult mai mic decât la scară națională
(de exemplu, o micro-rețea pentru un singur sat izolat sau o mică insulă).
§R1.4 DC GRILE
Tensiunea de grilă este limitată de defalcare de la scântei și
evacuările care apar la vârf de tensiune, care este trecătoare în cadrul fiecărei AC
ciclu. Cu toate acestea, de vârf de tensiune pe o rețea de curent continuu este constant, astfel încât să permită
un cablu pentru a transporta mai multă putere ca DC decât AC. În plus: (a) AC curenții
au tendința de a trece de-a lungul exterior o regiune dintr-un cablu (efectul de piele), deci
crește rezistența eficientă, întrucât DC curenți trec de-a lungul
cablu secțiune transversală; (b) există inducție și capacitate de pierderi cu
AC, care sunt absente pentru DC. De transmisie a puterii ca DC este folosit pentru mai multe
mari-linii electrice de transport care sunt lungi sau sub apă; de multe ori aceste
integreze inter-statale și internaționale de producție și a cererii. În
special, atât de lungi și de mare putere de autostrăzi de energie electrică sunt folosite pentru a transmite
hidroenergetice (de exemplu, (a) 2000 MW pentru ~1000 km la 450 kV (DC) între
Quebec (Canada) și Boston (SUA), și (b) 1900 MW de 1420 m, la
500 kV (DC) de la Cambora Bassa (Mozambic) spre Johannesburg (South
Africa)). Sisteme similare au fost propuse pentru transmiterea offshore
de energie eoliană din marea BRITANIE apele în Europa Continentală, și de energie solară din
Africa de Nord spre Europa Centrală și de Vest.
§R1.5 AC PUTERE ACTIVĂ ȘI REACTIVĂ:
TRANSFORMERS
Dacă există doar o rezistență în circuitul exterior, atunci tensiunea și
curentul sunt în fază, caz în care puterea disipată sub formă de căldură în
circuit extern este P = V
p
Am
p
păcat
2
ωt și puterea medie este P
av
= V
p
Am
p

/2.
Aceasta este puterea activă, care, în acest caz, vârfuri la o frecvență de două ori
care dintre tensiune și curent (jumătate de perioadă). Este o practică comună
pentru a apela V
p
/
√2 = V tensiunea (strict, rădăcină-mean-square tensiune) și
Am
p
/
√2 = am actuale, astfel încât puterea P = VI ca pentru curent continuu.
Dacă circuitul exterior este pur inductiv cu rezistență zero,
apoi, tensiune AC (push) și curentului (curgere) sunt
φ = 90° din faza
TWIDELL PAGINARE.indb 651
01/12/2014 11:38
652
De energie electrică pentru energia din surse regenerabile
652
Comentariu 1 energie Electrică pentru energia din surse regenerabile
(împinge maxim cu flux zero; debit maxim cu zero push). În
general, dacă diferența de fază este
φ, atunci produsul de V
p
păcat
ωt și eu
p
sin(
ωt + φ) are frecvența 2ωt și amplitudine Q, unde:
Q
V I păcat
φ
=
(R1.2)
Această funcție Q are unități de putere, dar nu este disipată sub formă de căldură; aceasta
este puterea oscilând înainte și înapoi în câmpul magnetic al
inductanței. Timp-o medie de putere în circuitul exterior este zero,
și se numește putere reactivă; ea nu poate apărea sub formă de căldură sau de lucru.
Dacă circuitul exterior este inductiv și rezistiv, atunci
φ nu este de 90°.
Produsul de V
p
păcat
ωt și eu
p
sin(
ωt + φ) poate fi acum separate într -
o parte reactivă (oscilează cu amplitudine egală între pozitiv și
negativ) și o parte activă (întotdeauna pozitivă). Valoarea medie a
puterii active
P
av
= V I cos
φ (R1.3)
și valoarea medie a puterii reactive este zero, la fel ca înainte.
Efecte similare apar cu capacitanță în circuitul exterior, dar
cu actualul lider de tensiune. Este o practică comună să se adapteze
compensator capacitate de a nega puterea reactivă efecte de
inductanță, și vice-versa. Ingineri electrice vorbesc de "activ" și
"reactive" puterea ca parametrii separate; fiecare separat instrumentate
și cuantificate. În general, puterea reactivă nu este dorit, și așa
gridconnected utilizatori ar putea fi taxat pentru consumul de putere reactivă, care
vor provoca sau de a utiliza, în ciuda faptului că nu a fi utilizabile putere.
Motivul pentru care AC este atât de utilizat pe scară largă este că tensiunea unui
curent alternativ pot fi modificate cu ușurință printr-un transformator (vezi Fig. R1.1(b)).
În esență, un transformator este format din două bobine de sârmă (cu diferite
numărul de spire N ) pe același miez feromagnetic. Din moment magnetic
flux
Φ este efectiv limitată la bază, pentru fiecare buclă Φ este același,
și așa mai de (R1.4) tensiunea V în fiecare înfășurare este proporțional cu N.
Din aceasta cauza ușurința de transformare, și, de asemenea, ușurința de generație și de
capacitatea sa de motoare electrice, de curent ALTERNATIV este de obicei pentru sisteme de rețea.
§R1.6 MAȘINI ELECTRICE (GENERATOARE ȘI
MOTOARE)
(a) elementele de Bază
Funcționarea de bază a tuturor generatoarelor este simplu, dar mulți complexitatea
și variații sunt folosite pentru a da proprietăți particulare și îmbunătățirile
în materie de eficiență. În esență, un câmp magnetic este dispus să taie un fir cu
o viteză relativă, deci induce un curent electric prin Efectul Faraday.
Fiecare generator are o statorice (bobine de sârmă sau de magneți permanenți, care
rămâne static) și un rotor (magneți sau bobine de sârmă, care se rotesc în
TWIDELL PAGINARE.indb 652
01/12/2014 11:38
§R1.6 mașini Electrice (generatoare și motoare)
653
stator); unul dintre acestea are o bobina (de lichidare) în care a generat
curentul este produs, iar celălalt are alte înfășurări sau permanentă
magneți pentru a produce câmpuri magnetice. Vom da o scurtă cont aici;
pentru mai multe detalii, a se vedea manualele și site-uri pe mașini electrice în
Bibliografia de la sfârșitul acestui Review.
Un magnet mutat peste un conductor electric va induce o
diferență de potențial electric în conductor (un electro-motive de forță (EMF);
sau o 'tensiune'). Dacă conductorul formează un circuit închis, apoi, un electrice de
curent este indusă. EMF/tensiune este:
V
Nd
dt
/
Φ
=−
(R1.4)
în cazul în care există N conductoare în serie, fiecare tăiat de un flux magnetic
Φ de rata de schimbare dΦ/ dt. O bobină de sârmă (electromagnetice) care transportă un circuit electric de
curent produce un câmp magnetic, la fel ca un magnet. Dacă bobina are un
miez feromagnetic (de exemplu fier), atunci câmpul magnetic este foarte considerabil
îmbunătățită (cu un factor de ~1000). Prin urmare, bobine cu feromagnetice
nuclee (electromagneți) sunt utilizate în cele mai multe mașini electrice, și anume,
generatoare și motoare. Echivalentul nord și polul sud de astfel de bobine
se numesc poli importante'. Limitările de spațiu și simplitate a încuraja
utilizarea de magneți permanenți în loc de electromagneți, ca și în unele
multipole generatoare pentru turbine mari de vânt.
(b) generatoare Sincrone (alternatoare)
Fig. R1.2 ilustrează bază AC generator sincron; de aici cu doi
magneți permanenți în stator produce un câmp magnetic staționar.
O singură bobină de cotitură în acest domeniu are un induse EMF, care produce o
Sinusoidale
a tensiunii de ieșire
Energia mecanică de intrare la
un generator transformă bobina în
câmp magnetic.
Polii magnetici
Inele de alunecare
O tensiune proporțională cu
rata de schimbare din
zonă se confruntă magnetic
câmp este generat în
bobina. Acesta este un exemplu
al legii lui Faraday.
Fig. R1.2
Generator sincron: principiu.
TWIDELL PAGINARE.indb 653
01/12/2014 11:38
654
De energie electrică pentru energia din surse regenerabile
654
Comentariu 1 energie Electrică pentru energia din surse regenerabile
curentul într-un circuit extern conectat prin perii de pe inele de alunecare circular
la colector. Deoarece firele de la bobina alternativ taie
câmp magnetic în sus și în jos, curent alternativ (AC) este generat la aceeași
frecvență ca ax de rotație.
Curentul indus în rotirea bobinei (lichidare) se produce o
câmp magnetic, B
rotor
, care se rotește. Axul este condus de externe
cuplu mecanic
Γ
m
(de exemplu, ca într-un hidro-arborele turbinei). Cu toate acestea,
indus rotor câmp magnetic B
rotor
stabilește un opuse cuplu
Γ
le

din
electromecanice efecte. Echilibrul este atins atunci când arborele
condus mecanic de cuplu,
Γ
m
, este egal cu indusă electromagnetic
cuplu
Γ
le

. Puterea mecanică de intrare este egală cu puterea electrică


generată, mai puțin de frecare și pierderile electrice. AC energie electrică generată
a tensiunii și curentului variază sinusoidal cu timpul în sincronism
cu viteza de rotație a arborelui. Comerciale generatoare de curent ALTERNATIV funcționează pe
acest principiu, dar cu mai multe câmpuri magnetice, formând mai multe pole-perechi'
cu câmpurile magnetice de trecere de mai multe bobine. Eficiență de generare
de energie mecanică în energie electrică este de obicei ~97%, adică doar 3% pierdută
sub formă de căldură.
Deoarece câmpurile magnetice sunt create de magneți permanenți sau
DC curenți, generate sinusoidale de curent are un curent ALTERNATIV de frecvență (f
1
)
în sincronizare cu ax de rotație frecvență (f
s

) a generatorului.
Cu n pol-perechi, fiecare acționând ca un singur magnetic nord/sud
polepair', nf
s
= f
1

exact. O astfel de generator este un sincron de curent ALTERNATIV generator


cu frecvența de ieșire blocată la arborele de frecvență. Acest lucru necesită
arborele de frecvență să fie îndeaproape controlate.
De obicei, puterea va fi luată de la grid-connected staționare bobine pe
stator, și bobine pe rotor produce câmp magnetic sunt
conectate, prin intermediul inele de alunecare, de la un generator de curent continuu. Rețineți că puterea este
extras
din stator, care este conectat la rețea. Un beneficiu al sincrone
generatoare este că puterea reactivă poate fi controlat și redus la minimum,
astfel încât maximizarea puterii reale. În cel mai comun acord, stator
bobine sunt conectate direct la rețea, în cazul în care puterea este doar
exportate atunci când rotorul se transformă exact așa nf
s
= f
1
. În consecință, f
1
a
pentru a fi controlat exact
egal cu retea de frecvență (de exemplu, 50 Hz
, în cele mai multe țări, sau 60 Hz în America de Nord). Obținerea sincronismului
la pornire și în funcționarea nu este greu pentru scara de utilitate termice și
hidro a plantelor, în cazul în care sincrone generatoare sunt norma. Cu toate acestea, acest
control cerință inițial descurajat sincrone generatoare pentru
turbine eoliene cu viteză variabilă. Cu toate acestea, dacă un generator sincron este
decuplat de la rețea prin intermediul unui convertor (redresor de curent continuu conectat la invertor la
rețeaua de curent ALTERNATIV), apoi exact viteza de sincronism nu este necesar, la orice etapă.
O importantă modificare este o bază generator de curent continuu. Aici stator
magnetii produc un câmp magnetic staționar, dar bobine de rotor sunt
conectate prin semicirculare inele de alunecare la colector, astfel încât curentul
inversează direcția ca fiecare bobina rotorului trece perpendicular pe stator
TWIDELL PAGINARE.indb 654
01/12/2014 11:38
§R1.6 mașini Electrice (generatoare și motoare)
655
domeniu. Astfel, curentul de ieșire este unidirecțională; variază în amplitudine ca
modulul de păcat
ωt. Aceasta variind de curent continuu pot fi atenuate în format electronic
la echilibru/constanta DC. Astfel generatoarele produc curent continuu pentru
electromagneți de mare AC generatoare sincrone, fiind montate pe
același ax.
(c) generatoarele de Inducție (asincrone, generatoare)
De inducție de curent ALTERNATIV generator este strict o masina de inductie, de la
același dispozitiv poate fi un motor sau un generator, și este ușor de conectat la
rețea, fără nici o preocupare pentru sincronism. Această generalitate de design
vă permite generatoarele de inducție a fi mai ieftin decât generatoare sincrone.
Aranjamentul obișnuit este că înfășurările statorice (bobine) sunt conectate
la AC grilă, deci produce un câmp magnetic de rotație în jurul axului
mașinii. Rotorul este un rotor în colivie cu bare de cupru set
paralel cu axa, și legate între ele prin inele la fiecare capăt. Curenții
sunt astfel indusă termen scurt-circuitat bobine de pe ax. Acesti
curenti indusi se produce câmpuri magnetice, care, la rândul său
genera energie în bobinele statorului, dar numai în cazul în care ratele de rotație a
arborelui de câmp magnetic și bobine stator diferă. Faza relațiile
sunt astfel că puterea poate fi transferat între mecanice cu rotor
arbore și putere electrică în circuitul statorului.
Pentru masina de inductie: (a) există seturi de înfășurări pe stator
simularea n pol-perechi; (b) în cazul în care grila de frecvență este f
g
și rotorul frecvență
este f
r
atunci sincronism apare atunci când nf
r
= f
g
; (c) alunecare s este definit ca:
s = (f
g
– nf
r
) /f
g
; (R1.5)
(d) atunci când s = 0, nu există sincronism și nici o putere nu este generat sau utilizate;
(e) atunci când s este negativ (nf
r
mai repede decât f
g
) aparatul este un generator; (f)
când s este pozitiv (nf
r
mai lent decât f
g

) masina este un motor. Generator de


alunecare mărime este de obicei mai puțin de 10%, și între 0,5% și 5% pentru
un motor.
Un generator de inducție poate genera doar atunci când induse închis
buclă rotor curenți au fost inițiate la conexiune; ulterior se
continua în mod automat. În general există două metode pentru acest lucru: (a) pentru
grid-connected masini, putere reactivă este atras de live grid la
care generatorul de ieșire este conectat; sau (b) pentru autonomă
exploatarea, auto-excitat generație este făcut posibil de condensatoare conectate
între ieșire și pământ. Beneficiile metodei (a) (grilă de legătură)
sunt: (i) simplitatea și ieftinătate de sistem; (ii) de siguranță, deoarece
generatorul nu ar trebui să genereze dacă rețeaua de alimentare este oprit, și (iii) grila
poate fi utilizat pentru a exporta energie atunci când există surplus și de import de energie de la
alte ori. În metoda 2 nu trebuie să fie un magnetism rezidual în
cadrul sau mediul de generator pentru a furniza curent initial,
cu condensatori menținerea corectă faza de relații.
TWIDELL PAGINARE.indb 655
01/12/2014 11:38
656
De energie electrică pentru energia din surse regenerabile
656
Comentariu 1 energie Electrică pentru energia din surse regenerabile
Din cauza ieftinătate și ușurința de operare, de bază de inducție
generatoare sunt comune pentru întreprinderile mici (~10 kW) și mediu (~100 kW) scară
de generare de energie mecanică (de exemplu, hidro și generatoare diesel).
Sincronism la grila de frecvență este dificil cu turbine eoliene, și într-adevăr
nu a vrut să permită îmbunătățirea de captare a energiei eoliene, cu diferite grade de turbine
viteza rotorului (vezi Fig. 8.19(b)). Pentru turbine eoliene, este posibil să crească
alunecare de generator de inducție și, astfel, permite o creștere a variației
de viteză a rotorului pentru a menține constant și lama-sfat pentru viteza vântului raport
(sfat-viteza-raport). Cea mai veche metodă implică impedanta schimbare de
altfel fără legătură înfășurările rotorului, dar în detrimentul crescut
generator de încălzire. Cea mai modernă metodă (de două ori de inductie
generator) are rotorul înfășurările conectate prin inele de alunecare de la o sursă externă de
putere-control electronic de tensiune și de fază (Fig. R1.3). În acest mod:
(a) viteza de rotație a rotorului poate varia considerabil de la
sincronism cu rețeaua de curent ALTERNATIV de frecvență, și (b) puterea poate fi apoi luate de la
rotorul circuite precum și stator. Astfel de două ori de inductie
generatoare, cu asociat electronică de putere, permite turbine eoliene pentru a
avea variabila viteza rotorului și, prin urmare meci viteza vântului pentru cea mai
eficientă putere de extracție.
§R1.7 PROVOCĂRI ȘI OPORTUNITĂȚI
PENTRU SURSELE REGENERABILE DE ENERGIE ELECTRICĂ
'Regenerabile de energie electrică poate fi definită ca energie electrică inițial generat
de o sursă regenerabilă și apoi să fie utilizat imediat, sau
distribuite pe o grilă sau stocate pentru mai târziu de aprovizionare (ca într-o baterie). Această definiție
acoperă o gamă foarte largă de aplicații, diferite:
Rotor
Stator
Trei faze
AC
DC
AC/DC/AC converter
AC
Fig. R1.3
De două ori fed generator de inducție cu putere generat de stator (ca într-un simplu
generator de inducție) și, de asemenea, de la rotor care are slip-ring conexiune la o AC/DC/AC
convertor care controlează amplitudinea și faza de AC conexiune de alimentare a rotorului.
Prin urmare, rotorul frecvența (viteza) poate fi independent de grila de frecvență (de exemplu,
într-o turbină de vânt).
TWIDELL PAGINARE.indb 656
01/12/2014 11:38
§R1.7 provocări și oportunități pentru surse regenerabile de energie electrică
657
(i)
De putere: de la watts (de exemplu, PV de așteptare de securitate) și kW (de exemplu,
de uz casnic-micro generație) la GW (scară largă hidro).
(ii)
În funcție de locație: de izolate și autonome (de exemplu, sat de la distanță,
Pământ prin satelit) pentru metropolis (de exemplu, Oslo).
(iii)
De variabilitate: de la constanta (de exemplu, centrale hidro cu o substanțială rezervor,
stocate biocombustibil) variabile (de exemplu, energia eoliană).
(iv)
De oportunitate: din exces (de exemplu, uragane) la minim (de exemplu, hidro
într-un deșert).
(vi)
De impact: de la rău (de exemplu, inundat valea de hidro rezervor) pentru
acceptabile (de exemplu, PV de acoperiș hambar).
(vii)
De atenuare: de la reducerea emisiilor de carbon (de exemplu, atunci când
substituirea de energie electrică de la centralele pe bază de cărbune) la moștenire (de exemplu, instalarea de
surse regenerabile de energie susține viitor).
Astfel de caracteristici sunt considerate, în general, în §1.4 și Tabelul 1.1 pentru toate
formele de energie, dar acestea sunt cele mai profunde pentru energie electrică din cauza
universalității de aplicare. Vezi Fig. R1.4 proporția din
surse regenerabile de energie electrică în totalul energiei produse de o națiune – statele UNITE ale americii.
Mare variație în formă și localizare a surselor regenerabile de energie generație
contrastează cu "tradițional" central generație în rețele mari, mai ales la
mare, centrala termica de GW de capacitate. În consecință tradiționale ierarhie
a consumului de energie electrică de alimentare tinde să se opună distribuite generație, mai ales
de la relativ mici-capacitate privat generatoare și micro-generatoare.
Cu toate acestea, în general, ce poate merge într-un fel într-o putere electrică
ditribution rețea 'poate merge în altă parte'. Deci grila e la "cota"
de energie electrică, mai degrabă decât un sistem cu un singur sens de centrul generație exterior.
Evident, linii de grilă nu trebuie să fie supraîncărcat și toate echipamentele conectate
trebuie să fie de tip aprobat'. Comerciale moderne regenerabile echipamente
utilizează solid-state controlere electronice și interconexiuni în condiții de siguranță și
de încredere conectarea la rețele, astfel încât, ca și condițiile se schimbă, puterea poate fi
fie importate (achiziționate) sau exportate (vândut) de către clienți (în mod tradițional
numit "consumatori", chiar și atunci când acestea sunt micro-generare exportatorii!).
Există variații considerabile în fluxul de putere cu timpul în toate părțile de
o rețea de încredere, cum s-a discutat în §15.4.2 etc. Acest lucru este în principal din cauza
cererii se schimbă cu timpul de zi și de noapte, și cu sezonul. Grila
operatorii trebuie să mențină un continuu echilibru al cererii și ofertei.
Pentru întreprinderile mici, temporare variante, este o practică obișnuită pentru a permite rețelei
de tensiune și frecvență pentru a schimba ușor pentru a menține acest echilibru
automat. Pentru perioade mai lungi, există un surplus de generare de o serie de
tipuri, astfel încât controlere de rețea poate răspunde pentru defectele, neașteptate
generator de deconectări și de întreținere de rutină întreruperi. În plus,
și ca o regulă de degetul mare, de 20%, surplusul de capacitate de producție și low-tarifare
disconnectable de încărcare sunt disponibile pentru a acoperi o astfel de eventualitate. Pompat
hidro furnizarea, dacă este disponibil, este valoros în acest sens pentru rapid
furnizarea de energie sau de sarcină în plus după cum este necesar.
TWIDELL PAGINARE.indb 657
01/12/2014 11:39
658
De energie electrică pentru energia din surse regenerabile
658
Comentariu 1 energie Electrică pentru energia din surse regenerabile
Solar
0.158
0.0175
8.26
7.74
18.0
12.6
4.83
3.23
0.0396
0.430
0.0197
0.0512
0.110
0.140
0.0179
3.15
1.17
0.163
Respins
energie
55.6
Energie
servicii
41.7
Comerciale
8.59
Rezidential
11.4
Industriale
23.6
Nucleare
8.26
Hidro
3.17
Vântul
1.17
Geotermale
0.226
Naturale
gaz
24.9
Cărbune
19.7
0.444
0.288
0.735
1.61
1.15
2.27
0.0260
25.1
6.76
20.3
18.9
8.06
8.32
3.33
4.50
4.86
1.14
4.72
1.72
9.15
2.29
26.6
0.683
Biomasa
4.41
Petroliere
35.3
De energie electrică
generație
39.2
Net de energie electrică
importurile
0.127
Estimat sua consumul de Energie în 2011: ~97.3 Careu
Lawrence Livemore
Laboratorul Național
Trans-
transport
27.0
6.87
R1.4
Energie diagrama flux pentru statele UNITE ale americii în 2011.
Sursa: Lawrence Livemore Național De Laborator.
Atenție: NOI unități: 1 quad = 10
15
BTU
≈ 10
18
J = 1 EJ.
Caracteristica majoră de cele mai multe surse regenerabile de energie consumului de energie electrică
generație
este variabilitatea, de care de mult este previzibil. În unele privințe variație
în generație apare pe rețea ca variație în Íload (creșterea
generație având același efect ca și o scădere în sarcină, și vice-versa), astfel încât
stabilite metode de control al rețelei poate fi aplicat
în mod continuu echilibrul cererii și ofertei, așa cum am discutat în Capitolul 15. Ca o regulă
de degetul mare, dacă capacitatea de generare de energie din surse regenerabile variabile este mai mică decât
20% din
totalul ofertei capacitatea (respectiv >80% este controlat termic si hidro genuri-
tion), atunci nu nerezolvat dificultate apare pentru grila de control. Ca variabile
surse regenerabile de energie generație crește peste aproximativ 20%, apoi speciale de
prevedere trebuie să fie făcut, cum ar fi în plus de așteptare generație (de exemplu,
biocombustibili sau de combustibili fosili-a generație) și având temporară disconnectedable
de încărcare (de exemplu, de la distanță schimbat încălzitoare de apă, de rafinare a metalelor cuptoare). Dacă
sursele regenerabile de energie generație este prea mult, atunci centrale, controlere nevoie pentru a
fi capabil de a opri generatoare de la distanță (de exemplu, parcuri eoliene). Tot un astfel de control
este, în esență, tehnice, dar atent structurat tarifelor atât pentru importatori
(consumatori) și exportatorilor (generatoare) susține astfel de opțiuni de control
(de exemplu, energie electrică mai ieftină pentru sarcini care pot fi deconectate de centrale,
controlere, etc.).
TWIDELL PAGINARE.indb 658
01/12/2014 11:39
Bibliografie
659
ÎNTREBĂRI RAPIDE
Notă: Răspunsurile la aceste întrebări sunt în textul de la secțiunea relevantă
din prezentul capitol, sau poate fi ușor dedusă din aceasta.
1
Ce este de energie electrică considerată o calitate înaltă formă de energie?
2
Care sunt beneficiile tehnice de o rețea electrică rețea?
3
Ceea ce este esențial de energie electrică'?
4
Ce characterisitcs beneficia de o distanță lungă linie de transmisie?
5
Cum este de curent alternativ de energie electrică, de obicei schimbat în tensiune?
6
Explica de putere reactivă.
7
Care sunt caracteristicile distinctive ale unui generator sincron?
8
Care sunt caracteristicile distinctive ale unui generator asincron?
9
Care sunt caracteristicile distinctive ale o de două ori de inductie
generator?
10
Lista de tipuri de surse regenerabile de energie, generatoarele că, atunci când generarea de
energie electrică sunt: (a) constantă sau a fi controlate, (b) periodice,
(c) variabilă.
BIBLIOGRAFIE
El-Sharkawi, M. A. (2012, 3rd edn) Energie Electrică – O Introducere, CRC Press / Taylor & Francis, Abingdon.
Daneză a Industriei Eoliene site-ul Asociatiei (www.windpower wiki.dk/ ).
TWIDELL PAGINARE.indb 659
01/12/2014 11:39
660
Esențiale ale dinamicii fluidelor

Comentariu 2 Esențiale ale dinamicii fluidelor


CONȚINUTUL
Obiective de studiu
660
§R2.1 Introducere
661
§R2.2 Conservare a energiei:
Ecuația lui Bernoulli
661
§R2.3 Conservarea impulsului
663
§R2.4 Vâscozitate
664
§R2.5 Turbulențe
665
§R2.6 Frecare în conducta de tur
666
§R2.7 Ridicați și trageți forțele
668
Întrebări rapide
671
Bibliografie
671
OBIECTIVE DE STUDIU
După ce a citit această Revizuire, ar trebui să fie familiarizat
cu ecuațiile de bază și terminologie de lichid
mecanica, care sunt utilizate în alte capitole ale acestei
carte, și cu principiile fizice care stau
în spatele acestor ecuații.
LISTA DE FIGURI
R2.1 Ilustrează conservare a energiei în fluxul de lichid.
662
R2.2 O turbină într-o conductă.
663
R2.3 Fluxul între două plăci paralele.
664
R2.4 Calea linii de curgere într-o conductă: (a) laminar, (b) turbulent.
665
R2.5 Graficul de factorul de frecare f pentru conducta de tur.
667
R2.6 Schițe pentru a ilustra forțele pe un obiect scufundat într-un fluid de curgere.
669
R2.7 Variația de ridicare și coeficienți de frecare cu unghiul de atac
α pentru un tipic de dirijare în intervalul de lucru. 670
LISTA DE TABELE
R2.1 Aproximativă conducta de rugozitate
ξ
667
TWIDELL PAGINARE.indb 660
01/12/2014 11:39

www.shahrsazionline.com
§R2.2 Conservarea energiei: ecuația lui bernoulli
661
§R2.1 INTRODUCERE
Transferul de energie către și de la un fluid în mișcare este baza de hidro,
eoliană, a valurilor și unele sisteme de energie solară, și de meteorologie. Ne -
comentariu aici, dinamica fluidelor vom folosi în analiza noastră de aceste
aplicații. Cititorii caută explicații suplimentare ar trebui să consultați referințele
enumerate în Bibliografie.
Vom începe cu legile de bază de conservare de masă, energie și
impuls. Termenul de lichid include ambele lichide și gaze, care, spre deosebire de
solide, nu rămân în echilibru atunci când este supus la apariția forțelor de forfecare.
Hidrodinamice distincție între lichide și gaze este că gazele
sunt mai usor de comprimat, întrucât lichide au volume diferite
doar puțin cu temperatura și presiunea. Cu toate acestea, pentru aer, care curge
la viteze
< 100 m/s și nu sunt supuse la variații mari de presiune sau de
temperatura, densitatea schimbare este neglijabil. Prin urmare, pentru energie eoliană
(și, după caz, în alte sisteme de energie regenerabilă) se deplasează
aerul este tratat ca un incompresibil lichid, adică ca și cum acesta este un lichid.
Thisconsiderably simplifică analiza, fără a introduce semnificativ
de eroare.
Multe importante fluxuri de fluide sunt de echilibru, în sensul că în special
fluxul de model de la o locație nu variază cu timpul. (Desigur lichidul
în sine este în mișcare!) Fluxul în sine poate fi reprezentat prin simplifică,
paralel cu viteza instantanee vectori la fiecare punct, care poate
reprezenta fie laminar sau turbulent de curgere (§R2.5). Cu toate acestea, chiar și în
turbulențe, care simplifică rămână în limitele bine definite (deși imaginar)
flux de tuburi.
§R2.2 CONSERVAREA ENERGIEI: LUI BERNOULLI
ECUAȚIA
Consideră constantă, fluxul de incompresibil. În primul rând, vom presupune că nici un lucru
se face prin deplasarea de lichid (de exemplu, pe o turbina).
(a) Nr. de intrare de căldură
Fig. R2.1 se prezintă o secțiune a unui tub de flux între înălțimile z
1
și z
2

.
- Și asume nici o energie de schimb de căldură sau de lucru pe fluxul de tuburi,
așa cum este adesea cazul. Tubul este îngust în comparație cu alte
dimensiuni, deci z este considerată constantă pe fiecare secțiune transversală a tubului.
Un fluid de masă m =
rO
1
u
1
Dt intră în volumul de control la 1, și un egal
de masă m =
rO
2
u
2
Dt frunze la 2 (în cazul în care
r este densitatea fluidului, tratate
la fel de constant). Deci:
schimbare în energie potențială + schimbare în forțele de presiune
= schimbare în energie cinetică + frecare
TWIDELL PAGINARE.indb 661
01/12/2014 11:39
662
Esențiale ale dinamicii fluidelor
662
Comentariu 2 Esențiale ale dinamicii fluidelor
mg z
z
pOut
pOut
mu
u
E
(
) [(
)(
)(
)(
)]
(
)
1
2
11
1
22
2
1
2
2
2
1
2
f
-
+
D-
D=
-
+
(R2.1)
în cazul în care (i) forța de presiune p
1
O
1
acționează printr-o distanta de u
1
Dt, și în mod similar
pentru p
2
O
2
, și (ii) E
f
este căldura generată intern de frecare.
Neglijarea frecare fluid E
f
și rearanjând termenii, randamente:
p
gz
u
p
gz
u
(/)
(/)
1
1
1
21
2
2
2
1
22
2
r
r
+
+
=
+
+
(R2.2)
sau, echivalent,
p
g
z
u
g
++
2
= constantă de-a lungul unui simplifica, fără pierdere de energie.
2
r
(R2.3)
Oricare dintre aceste forme de ecuație se numește Bernoulli ecuația lui.
Suma termenilor de pe partea stângă a (R 2.3) ca dimensiunile de lungime
și se numește total cap de lichid (H ), cu o relevanță deosebită pentru
hidroenergie.
Rețineți că R (2.2) și R (2.3) se aplică la lichide tratate ca ideal, adică cu zero
vâscozitate, zero compresibilitate și zero conductivitate termică și fără
surse interne de căldură. Aceste aproximări lucra bine pentru aproape toate
calculele în această carte despre eoliene și hidro turbine. (În
ipoteza de zero vâscozitate, sau echivalent zero frecare internă, este de obicei valabil
, cu excepția foarte aproape de suprafetele solide: a se vedea §R2.4.) Energia ecuație
poate fi modificat pentru a include non-ideal caracteristicile de ardere
a motoarelor și a altor dispozitive termice (de exemplu, high-temperatura de concentrare
colectoare solare (vezi Bibliografia)).
(b) Cu intrare de căldură
În sisteme de încălzire solară și schimbătoare de căldură, căldura E = P
th

Dt este adăugat ca
un aport de energie în Fig. R2.1. Masa m care vin în control
Fig. R2.1
Ilustrează conservare a energiei în curgere a fluidului: un tub de flux se ridică la înălțimea z
1

la z
2
. În
unele cazuri, putere termica P
th
poate fi adăugat la fluxul de căldură.
De masă m =
ρ
Ou
1

t
u
1
u
2
z
1
z
2
Căldură
E = P
th
D
t
Temperatura
T
1
Temperatura
T
2
1
2
TWIDELL PAGINARE.indb 662
01/12/2014 11:39
§R2.3 Conservarea impulsului
663
volum, la temperatura T
am
pot fi considerate a avea conținut de căldură mcT
1

(unde c este capacitatea termică specifică a fluidului), și că ieșirea


are conținut de căldură mcT
2
. Acest lucru vă oferă o ecuație corespunzătoare (R2.2),
și anume:
p
gz
u
cT
P
Q
p
gz
u
cT
(/)
(/
)(/)
1
1
1
21
2
1
th
2
2
1
22
2
2
r
r
r
+
+
+
+
=
+
+
+
(R2.4)
în cazul în care debitul volumetric
Q Ua
=
(R2.5)
În cele mai multe sisteme de incalzire, termice contribuții domina energia de
echilibru, cu mișcări fluide nesemnificative. Deci, practic
scopuri, (R2.4) se reduce la:
P
cQ T
T
(
)
th
2
1
r
=
-
(R2.6)
§R2.3 CONSERVAREA IMPULSULUI
Doua lege a lui Newton de mișcare pot fi generalizate de particule
lichide: 'În orice clipă în flux constant, de forță rezultantă care acționează asupra unui
lichid se deplasează într-un volum fix este egal cu fluxul net de impuls
din acest volum.' Acest lucru este cunoscut sub numele de teorema impulsului. Newton
a treia lege (acțiune și reacțiune) poate fi aplicată pentru fluide într-un mod similar.
De exemplu, ia în considerare fluid care trece peste o turbină într-o conductă. În
Fig. R2.2, fluidul care curge cu viteza u
1
în stânga suprafață de control
poartă impuls
ru
1
pe unitatea de volum în direcția de curgere, și ieșirile
de la dreapta la viteza u
2
. Impulsul teorema ne spune că rata de
schimbare de impuls este egal cu forța, F, pe de lichid și de reacție,
–F este forța exercitată pe turbina si teava de lichid. Deci:
F =
r (O
2
u
2
2
- O
1
u
1
2
)=
mu
2
-
mu
1
(R2.7)
Zona O
2
u
2
y
x
u
1
Zona O
1
Control
de suprafață
1
2
Fig. R2.2
O turbină într-o conductă. Linia punctată arată suprafața de control în care impulsul
teorema este aplicat.
TWIDELL PAGINARE.indb 663
01/12/2014 11:39
664
Esențiale ale dinamicii fluidelor
664
Comentariu 2 Esențiale ale dinamicii fluidelor
în cazul în care
m
=|
rO
1
u
1
|=|
rO
2
u
2

| este fluxul masic (întotdeauna luate ca pozitive)


și semne în (R2.7) indică direcții, care sunt evidente în acest caz.
În cazuri mai complexe, cum ar fi în interiorul unei turbine, impulsul și
forțele trebuie să fie tratate ca vectori (de exemplu, direcția contează!).
§R2.4 VÂSCOZITATE
Ia în considerare două plăci paralele, cu lichid între ele și placa superioară
se deplasează la o viteză u
1

relativ la cel de jos (Fig. R2.3). Axele


au x în direcția de mișcare, și y peste decalajul între
plăci. Se constată experimental că lichidul nu aluneca la o suprafață solidă,
adică lichidul imediat adiacente pentru fiecare placa are aceeași viteza și
direcția de mișcare ca placa.
La scară microscopică, mișcarea aleatorie a moleculelor din lichid
transferuri mai mari de impuls (achiziționat de la placa de sus) în jos
și mai mici impuls (achiziționat de la placa de jos) în sus. Asta
difuzie de impuls de limitele de viteza de gradient că lichidul poate
susține, produce o frecare internă care se opune orizontală alunecare în
flux. Se constată că tensiunea de forfecare (de exemplu, forța pe unitatea de suprafață, în
direcția indicată în Fig. R2.3) este
u
y
/
tm∂

(
)
=
(R2.8)
în cazul în care
m este vâscozitatea dinamică (unitate N s m
-2
). Această vâscozitate este inde-
pendent de
t și ∂u / ∂ y, și depinde numai de compoziția și tem-
temperatura fluidului.
O strâns legată de lichid parametru este viscozitate cinematică:
/
nmr
=
(R2.9)
În fluide incompresibile, modelul de curgere depinde de multe ori mult mai direct
pe
n decât pe m. Prin combinarea (R2.8) și (R2.9), constatăm că unitățile de
cinetică vâscozitate
n sunt:
-
-
-
-
-
(kg ms )m
kg m
m
ms
=ms
2
2
3
1
2
1
Fig. R2.3
Curge între două plăci paralele.
τ
τ
u
1
x
z
y
TWIDELL PAGINARE.indb 664
01/12/2014 11:39
§R2.5 Turbulențe
665
Astfel
n are caracter de difuzibilitate; de exemplu, schimbări în impuls
difuze pe o distanță x , în timp ~ x
2
/
n (compara difuzibilitate termică
k definit
în §R3.3). Valori tipice de
n sunt date în Anexa B, Tabelele B. 1 și B. 2.
§R2.5 TURBULENȚE
Curgere turbulentă se produce deoarece majoritatea mișcare fluidă este instabilă. Să presupunem că
fluidul este inițial curge printr-o conductă într-o ordine, mod stabil, ca și în
calea de linii prezentate în Fig. R2.4(a). Considerăm că o mică mișcare de volum
de lichid, la care ne vom referi aici ca o pată de cerneală " sau "pachet'. Ceva va
deranja mișcare (de exemplu, o oscilație sau un bat pe conducta), provocând
mici forțe să acționeze pe pete. Dacă acestea sunt în mișcare suficient de rapid, fluid
de frecare nu va fi suficientă pentru a le menține în original căi, astfel
cauzând instabilitate în flux. Deranjat elemente atunci deranja pe ceilalti
din apropiere pete de lichid din căi originale, și, în curând întregul flux
este în semi-haotic de stat numit de turbulență, ilustrate în Fig. R2.4(b).
Apa care curge de la robinet sau fum în creștere de la un taper de multe ori arată această
schimbare de buna (laminar) fluxul de turbulențe. Vântului în câmp deschis
de mediu este mereu turbulentă și devine doar laminar, deoarece îndeplinește
marginea de aerodinamică lame sau aripi.
Non-dimensional numărul lui Reynolds
R = uX / n (R2.10)
este cheia pentru a stabili dacă o curgere este laminar sau turbulent; reprezintă
raportul dintre lichid de impuls (care rezultă din forțele de inerție) să vâscos
frecare. Aici u este viteza medie de curgere, X este un nominalizat
lungime caracteristică a sistemului (pentru conducte, diametrul lor), și
n este cinematic de
vâscozitate a fluidului. Numai curge cu valori relativ mici ale lui R va fi
laminar; cel mai practic fluxurile turbulente cu valori mai mari de R. De
exemplu, în conducte, fluxul este probabil să fie turbulentă dacă R > ~2300.
În curgere turbulentă, efectul lateral propuneri de lichid
fluid de transport de mică viteză de aproape de o suprafață solidă (de exemplu, peretele conductei)
Fig. R2.4
Calea linii de curgere într-o conductă:
o
laminar,
b
turbulent.
(a)
(b)
TWIDELL PAGINARE.indb 665
01/12/2014 11:39
666
Esențiale ale dinamicii fluidelor
666
Comentariu 2 Esențiale ale dinamicii fluidelor
spre partea principală a curge lichid și lichidul de mare viteză în
direcția opusă. Impulsul atât de transferat de pete de lichid
este mai mare decât cea transferată de mișcare moleculară pentru că o picătură de
fluid poate muta un drum lung (de exemplu, jumătate de drum într-o țeavă) într-un singur salt.
Acest transfer de impuls de la lichid la un statice suprafață solidă creează
o importantă forță de frecare care se opune mișcării fluidului. Astfel,
prezența unor turbulente în conducte crește frecare în comparație cu
flux laminar.
Dacă pereții conductei sunt mai tare decât fluidul de intrare, aceste rapidă
spre interior și spre exterior propuneri de transfer de căldură rapid la cea mai mare parte din
lichid. Un element de lichid rece poate sari de la centrul conductei, pick
up de căldură prin conducție de la cald perete, și apoi se duce mult mai
rapid in centrul conductei decât ar putea moleculară conducție.
Astfel turbulențe , de asemenea, crește transferul de căldură (a se vedea §R3.4). Criterii pentru
laminar sau turbulent de curgere în transferul de căldură sunt discutate în Revizuire 3.
§R2.6 FRECARE ÎN CONDUCTA DE TUR
Din cauza frecării, în caz contrar energie utilă și presiunea sunt spus să fie "pierdut"
sau 'disipat' când un fluid curge prin conducte; de exemplu, în
sistem de conducte care să conducă la o turbină hidroelectrică. Să
Dp fi presiunea
depășirea frecare la fel de fluid se mișcă la viteza medie u, prin conducta de lungime
L și diametru D. Observație indică faptul că
Dp crește ca L crește
și D scade. Lui Bernoulli ecuația arată că cantitatea ½
ru
2
a
aceleași dimensiuni ca și p, respectiv kg/(ms
2
). Toate acestea pot fi exprimate în
ecuația:
p
fLDu
2 ( / )(
)
2
r
D=
(R2.11)
Aici f este adimensional de frecare conductă factor care modifică valoarea cu
condiții experimentale. (Atenție: (1) În alte cărți
f
f
4
'=
este
numit factor de frecare, și un echivalent ecuație este folosit în loc de
(R2.11); în această carte, vom folosi doar f. (2) Nici unul din factorii de frecare' f
sau f
'este legat 'coeficientul de frecare' descrie frecare între
două solide suprafețe.) Ca și în multe non-dimensional factori în
inginerie, amploarea f caracterizează condițiile fizice
în mod independent de scara, in functie de modelul de curgere, adică forma
din simplifică.
Factorul de frecare
f este proporția de energie cinetică ½
ru
2
introduceți-
ing unitatea de suprafață a conductei care trebuie să fie aplicate ca locul de muncă externă (
Dp) pentru a
depăși forțele de frecare. Depinde de (a) adimensional Reynolds
numărul R de (R2.10) și (b) raportul dintre înălțime, ξ, de la suprafață
nereguli (rugozitatea) la diametrul conductei, D. Fig. R2.5 parcele
- o serie de curbe de factorul de frecare față de numărul Reynolds, cu o
curbă pentru fiecare rugozitate raport
ξ /D.
TWIDELL PAGINARE.indb 666
01/12/2014 11:39
§R2.6 Frecare în conducta de tur
667
Tabelul R2.1
Aproximativ conducta de rugozitate
ξ
Material
ξ / mm
De sticlă, PVC și cele mai multe alte materiale plastice
0.0015
Fonta
0.25
Noua oțel
0.1
Netezite beton
0.4
0.02
0.04
0.02
0.01
0.004
0.001
0.0004
0.0001
"Buna" (
x
/D = 0)
Rugozitatea relativă
x
/D
Factorul de frecare
f
0.0175
0.015
0.0125
0.0075
0.005
0.0025
10
3
10
4
10
5
10
6
10
7
Laminar
Numărul Reynolds uD/
n
0.01
Fig. R2.5
Graficul de factorul de frecare f pentru țeavă de curgere (a se vedea (R2.11)).
Cu condiția ca valoarea corespunzătoare de
ξ este folosit, aceste curbe da
o estimare rezonabilă a conductei de frecare. Valori tipice de
ξ sunt prezentate în
Tabelul R2.1, dar ar trebui să fi dat seama că rugozitatea conductei tinde
să crească odată cu vârsta și, foarte vizibil, cu acumulare de sedimente
și încrustări. Acest lucru se aplică în multe situații, inclusiv de încălzire
sisteme în fabrici și clădiri și arterele din corpul uman. Notă
excepțională finețe de plastic curat materiale și acoperiri
(de exemplu, pe palele turbinei eoliene).
A LUCRAT EXEMPLU R2.1
Ce este capul pierdere datorită frecării atunci când apa curge printr-o conducta de beton de lungime de 200 m și
cu diametrul de 0,30 m, la un debit volumic de 0,10 m
3
/s?
Soluție
Medie de viteza apei este:
uQO
/
0,1 m s
(0,15 m)
1.4 ms
3
1
2
1
=
=
=
-
-
π
TWIDELL PAGINARE.indb 667
01/12/2014 11:39
668
Esențiale ale dinamicii fluidelor
668
Comentariu 2 Esențiale ale dinamicii fluidelor
Din (R2.10), numărul lui Reynolds
n
=
=
×
=
×
-
-
-
uD
(1.4 ms )(0,3 m)
1.0 10 m s
0.4 10
1
6
2
1
6
R
în cazul în care valoarea de
n este preluat din Anexa B, Tabel B. 2. Deoarece
R
>> 2000, curgere este turbulent.
Pentru beton (din Tabelul R2.1),
ξ = 0,4 mm. Astfel, raportul
D
/
0,4 mm
300 mm
0.0013
ξ
=
=
Pentru aceste valori ale
R
și
ξ / D, Fig. R2.5 dă
f = 0.0050,
Exprimarea (R2.11) în ceea ce privește pierderea cap din cauza frecării,
H
p
g
gripa
Dg
/
2
/
f
2
r
=D
=
(R2.12)
Prin urmare:
H
(2)(5.0 10 )(200 m)(1.4 ms )
(0,3 m)(9.8 ms )
1.3 m
f
3
12
2
=
×
=
-
-
-

Fig. R2.5 prezintă doar o singură curbă pentru R < 2000 indice de curgere este laminar;
la "model" de apă în mișcare este independent de conducte interne
de suprafață în acest interval de numărul Reynolds. În flux laminar este posibil să se
calculeze căderea de presiune
Dp în mod explicit de la (R2.8), și, prin urmare, acesta poate fi
arătat că factorul de frecare este:
n
=
f
uD
16 / ( ) (laminar)
(R2.13)
§R2.7 RIDICAȚI ȘI TRAGEȚI FORȚELE
Ridicați și trageți-forțele se aplică la orice obiect solid cufundat într-un fluid de curgere
(de exemplu, aripi de pe o aeronavă sau lame pe o turbină eoliană cu rotor).
În Fig. R2.6(a) un obiect solid este cufundat într-un fluid care curge de la stânga
la dreapta (în raport cu obiectul). Cu toate acestea, din cauza complexitatea fluxului
model de trecere la obiect, care rezultă forța pe obiect este puțin probabil să
fie paralele amonte de curgere. Dacă totale (vector) forța exercitată asupra
corpului este F, care trage forța F
D
este componenta forței în direcția
din amonte curge și lift forță F
Am
este componenta normală la
fluxul. Este lift forță care răsturnări de situație și se transformă în obiect.
Un important caz special este o paletei. Aceasta este o structura buna de
latime (coardă) mult mai mică decât lungimea (durata), iar grosimea mult mai puțin
TWIDELL PAGINARE.indb 668
01/12/2014 11:39
§R2.7 Ridicați și trageți forțele
669
decat sa coardă, având o relativ ascuțit posterior și mai curbat
pe partea de sus decât în partea de jos. Exemple sunt o aripă de avion sau eoliene
turbine blade (Fig. R2.6(b)). Airfoil formă și suprafețe netede
încuraja cu flux de aer laminar, astfel încât cu palete stabilit la un unghi mic
la fluxul de aer, ridicare la putere este mult mai mare decât forța de frecare. În
gama de operare de numărul Reynolds (de obicei >10
5

), curgerea în jurul
paletei este aproape de ideal (adică vâscozitate zero, zero compresibilitate), cu excepția cazului în
un subtire strat limită' aproape de suprafață. Acest lucru simplifică foarte mult
de modelare de ridicare și trageți, așa cum este descris în manualele de aerodinamica (a se vedea
Bibliografia).
Cu avioane, lift forță depășește forțele gravitaționale și
avionul nu scade. Pentru a înțelege acțiunea de lame de turbine eoliene, de
ridicare și trageți forțe trebuie să fie rezolvată și în afară de planul de rotație;
făcând astfel, arată că rezultatul net este o forță de cotitură lama peste
amonte direcția vântului; vezi §8.6.1 pentru o discu.
Ridicați și trageți de un profil aerodinamic sunt caracterizate prin două non-dimensional
parametri:
la coeficientul de ridicare
r
='
C
F
uc
/(
)
Am
Am
1
2
2
R2.14)
și
la coeficientul aerodinamic
C
F
uc
/(
)
D
D
1
2
2
r
='
(R2.15)
în cazul în care
'
F
Am
și
F
D
'
sunt, respectiv, de ridicare și de forțele de frecare pe unitatea de lungime
de span, iar c este lungimea de coardă linie (vezi Fig. R2.6).
Ambele C
Am
și C
D
sunt funcții de numărul lui Reynolds R, și a
unghiul de atac
α, care este unghiul dintre incident fluxul de aer și
pe linie acord între conducere și la sfârșit margini. (În avion
F
Am
F
F
Am
F
D
F
D
u
c
α
u
(a)
(b)
Fig. R2.6
Schițe pentru a ilustra forțele pe un obiect scufundat într-un fluid de curgere.
o
Orice obiect: lift forță F
D

(paralel cu flux viteza u ), lift forță F


Am
(normal să F
D

), total
(vectorială) forței F.
b
Caz Special de o pernă de aer (de exemplu, turbine eoliene lama) la unghiul de atac
α.
TWIDELL PAGINARE.indb 669
01/12/2014 11:39
670
Esențiale ale dinamicii fluidelor
670
Comentariu 2 Esențiale ale dinamicii fluidelor
Paletei
de mișcare
Lift
Lift
Drag
Lift
(b)
Drag
Drag
0
0.2
0.4
0.6
0.8
1.0
1.2
1.4
1.6
(a)
0
5
10
15
20
o
(deg)
C
Am
C
D
Stand punct
Fig. R2.7
Variația de ridicare și coeficienți de frecare cu unghiul de atac
α pentru un tipic de dirijare în
gama de lucru:
o
graficul de C
Am

și C
D
împotriva unghiul de atac
α. Dacă α ∼5°, condițiile sunt departe de a bloca și de
acceptabil drag.
b
simplifică de curgere.
context,
α este adesea numit 'unghi de incidență'.) Fig. R2.7 prezinta o
tipice de variație a C
Am
și C
D
cu
α pentru o anumită dirijare în lucru
gama de R. Pentru valori mici ale lui α, C
Am
este direct proporțională cu
α; notă
schimbarea raportului între lift și forțele de frecare în primele două diagrame de
TWIDELL PAGINARE.indb 670
01/12/2014 11:39
Bibliografie
671
Fig. R2.7(b). Pentru o anumită valoare a
α între 10° și 20°, liftul scade,
și de dirijare devine stagnat, cu fluxul de separare din partea de sus
a suprafeței și trageți creșterea în mod substanțial, la fel ca în partea de jos diagrama
din Fig. R2.7(b).
ÎNTREBĂRI RAPIDE
Notă: Răspunsurile la aceste întrebări sunt în textul de la secțiunea relevantă
din prezentul capitol, sau poate fi ușor dedusă din aceasta.
1
Ce poate fi tratat ca incompresibil în cele mai multe din surse regenerabile de energie
aplicații?
2
Scrie ecuația lui Bernouilli. Nu se referă în primul rând la
viteza, ritmul, sau energia unui fluid?
3
Distinge vâscozitate dinamică din kinenatic vâscozitate.
4
Definește numărul Reynolds. De ce este atât de important în calculele de
curgere a fluidului?
5
Care este diferența dintre flux turbulent și flux laminar?
6
De ce este conducta de frecare mai mare decât în flux laminar?
7
Defini ridicați și trageți.
8
De ce sunt aripi de aeronave, de obicei, subțire în comparație cu lungimea sau grosimea?
9
Compara densitatea aerului și a apei, și a discuta despre efectul pe
turbina de design.
10
Ce este viteza de deplasare a lichidului la o buna limita de suprafață?
BIBLIOGRAFIE
Următoarea selecție din multe cărți și site-uri de mecanica fluidelor se pot dovedi utile. Există multe
alte cărți bune în afară de cele enumerate. Pentru munca pe turbo-mașini, cărți scrise de ingineri, de obicei, sunt
mai utile decât cele scrise de matematicieni, care de prea multe ori ignora frecare și forțele. Deoarece elementele de bază ale
dinamicii fluidelor nu s-au schimbat, manuale vechi poate fi încă utilă, mai ales dacă se folosesc unități SI (care de multe
cărți mai vechi, nu). Cu toate acestea, moderne de inginerie practica face mult uz de pachetele de programe de calculator,
cu pericol de abuz dacă principiile de bază nu sunt înțelese de către utilizator.
Cărți
Batchelor, G. K. (1967) O Introducere în Dinamica Fluidelor, Cambridge University Press, Cambridge. Text clasic,
republicată neschimbată în anul 2000. O mai precisă declarație de fundații (a se vedea în special ch. 3), cu multe
exemple. Răsplătește lectură atentă, dar, probabil, nepotrivit pentru incepatori.
Çengel, Y. A. și Cimbala, J. (2009, 2nd edn) Mecanica Fluidelor: Fundamente și aplicații, McGraw-Hill,
New York. Explicații clare și detaliate cu accent pe principii fizice. Foarte student-friendly, cu un
exemplar care însoțește învățarea sida.
Francis, J. R. (1974, 4 edn) Un Manual de Mecanica Fluidelor, Edward Arnold, London. Clar scris face acest lucru
ușor de citit pentru incepatori. Mai multe detalii inginerie decât Kay și am ceva sa iti arat.
TWIDELL PAGINARE.indb 671
01/12/2014 11:39
672
Esențiale ale dinamicii fluidelor
672
Comentariu 2 Esențiale ale dinamicii fluidelor
Kay, J. M. și am ceva sa iti arat, R. M. (1985) Mecanica Fluidelor și Transfer de Procese, Cambridge University Press,
Cambridge. Un concis și amplă introducere.
Mott, R. L. (2005, 6 edn) Aplicată Mecanica Fluidelor, Prentice Hall, Englewood Cliffs, NJ. Utilizate pe scară largă student
text la nivel de incepator, cu extrem de explicații clare.
Potter, M. și Wiggert, D. (2007) Mecanica Fluidelor, Schaum Conturul Serie. Multitudinea de exemple.
Una din numeroasele cărți similare în acest student-friendly serie.
Tritton, D. J. (1988) Fizică Dinamica Fluidelor, Oxford Știință Publicații, Oxford University Press, Oxford.
Atent formularea matematic, legate strâns de realitate fizică.
Webber, N. (1971) Mecanica Fluidelor pentru Ingineri Civile, Chapman and Hall, Londra. Fermecător simplu, dar util
introducere pentru elevii cu un pic de cunoștințe de fizică sau inginerie.
Site-uri
en.wikipedia.org/wiki/Fluid_dynamics. Ca de obicei pentru științe de bază, Wikipedia, articolul este excelent, dar repede
intră evoluții nu sunt necesare pentru această carte.
www.youtube.com/watch?v=dY3daNK1Tek&feature=related. Un vechi mili militare de formare film cu extrem de
clare explicații de bază aerodinamica.
TWIDELL PAGINARE.indb 672
01/12/2014 11:39

S-ar putea să vă placă și